Complete Physics
Complete Physics
QUIZRR 3
WAVE OPTICS
In our discussion of lenses, mirrors and optical instruments we use the model of geometric optics
where we represent light as rays which are straight lines that are bent at a reflecting or refracting
surface. Many aspects of the behavior of light canÊt be understood on the basis of propagation of
rays. In some situations we have to consider its wave properties explicity. If two or more light
waves of the same frequency overlap at a point, the total effect depends on the phases of the
waves as well as their amplitude. The resulting patterns arise from of the wave nature of light
and canÊt be understood on the basis of rays.
Two monochromatic sources with the same frequency and with a definite and constant phase
relationship with each other are said to be coherent.
Huygens in 1678 assumed that a body emits light in the form of waves. According to him :
(1) Each point source of light is a centre of disturbance from which waves spread in all directions.
The locus of all the particles of the medium vibrating in the same phase at a given instant
is called a wavefront. The shape of wavefront depends on the source producing the waves
and is usually spherical, cylindrical or plane as shown.
(2) Each point on a wavefront is a source of new disturbance, called secondary wavelets. These
wavelets are spherical and travel with speed of light in that medium.
(3) The forward envelope of the secondary wavelets at any instant gives the new wavefront.
A´ A A A´
A A´
O O
B B´
B´ B B B´
(A) (B) (C)
(4) In a homogeneous medium, the wavfront is always perpendicular to the direction of wave
propagation. Hence, a line drawn normal to the wavefront gives the directions of propagation
of a wave and is called a ray.
Interference
The term interference refers to any situation in which two or more waves overlap in space. When
this occurs, the displacement at any point at any instant of time is governed by the principle of
superpositiion which states :
WAVE OPTICS
4 QUIZRR
ÂÂWhen two or more waves overlap, the resultant displacement at any point and at any instant
may be found by adding the instantaneous displacements that would be produced at any point
by the indivisual wavesif each were present aloneÊÊ
let the two waves be :
y1 = A1 sint
y2 = A2 sin(t + )
Where is the phase difference
y = y1 + y2
= A sin(t + )
and
I = I1 I 2 2 I1I 2 cos
{as I A2}
Please note that the derivation is not given in an exhaustive manner as it has already been
covered in the chapter ÂÂwaves and soundsÊÊ
For Iresultant to be maximum
cos = 1 = 2n
= n Where n = 0, 1, 2 ______
2
and A 2max = (A + A)
2
I max = I1 + I2
2
also,
1
= n
2
and Amin = |A1 A2|
2
Imin = I1 ~ I2
WAVE OPTICS
QUIZRR 5
In order to obtain a sustained (permanent or stable) and observable interference pattern, the
following conditions must be fulfilled.
(i) sources must be coherent : In order to produce a stable interference patter the individual
waves must maintain a constant phase relationship with one another, i.e., the two interfering
sources must emit waves having a constant phase difference between them. If the phase
difference between two sources does not remain constant then the places of maxima and
minima shift. In case of mechanical waves it is possible to kep a constant phase relationship
between two different sources. But in case of light two different light sources canÊt be
coherent. This is because of the way light is emitted. In ordinary light sources, atoms gain
excess energy by thermal agitation or by impact with accelerated electrons. An atom that is
ÂexcitedÊ in such a way begins to radiate energy and continues until it has lost all the enrgy
it can, typically in a time of the order of 10 8 s. The many atoms in a source ordinarily
radiate in an unsynchronized and random phase relationship, and the light that is emitted
from two such sources has no definite phase relationship. Hence, to obtain a stable interference
in light a single source is split into two coherent sources. Any random phase change in the
source affects these secondary sources equally and does not change their relative phase.
Laser light is much more coherent than ordinary light.
(ii) Same frequency or wavelength : Phase relationship betweep two waves can be kept
constant only when their frequecies are same. Thus we can say, that two coherent sources
must have the same frequency.
(iii) Equality of amplitudes : The amplitudes of two interfering waves should be equal or
appromixately equal. Otherwise the difference between the intensities of maximas and
minimas will be too small and the contrast will be poor. Maximum constrast is, however
obtained when A1 = A2, because then minimum intensity will be zero.
(iv) Both should be travelling in same direction
We consider two identical sources of monochromatic waves S1 and S2 along the y-axis, equiduistant
from the origin.
Consider a point ÂaÊ on X-axis. From symmetry the two distances from S1to a & S2 to a are equal;
waves from the two sources thus require equal times to travel to a. Hence waves that leaves S1
and S2 in phase arrive at a in phase. The two waves add and the total amplitude at ÂaÊ is twice
the amplitude of each individual waves. This is true for any point on the x-axis.
b
S1
a
S2 c
WAVE OPTICS
6 QUIZRR
S1
b
r1=7
r2=9 r1=9.75
S1 S2
r2=7.25
S2
c
Similarly, the distance from S2 to point b is exactly two wavelengths greater than the distance
from S1 to b. A wave rest from S1 arrives at b exactly two cycles earlier than a emitted at the
same time from S2 and and again the two waves arrive in phase. As at point O, the total
amplitude is the sum of the amplitudes of the waves from S1 and S2.
In general, when waves from two or more sources arrive at a point in phase, the amplitude of
resultant wave is the sum of the amplitudes of the individual wave; the individual waves reinforce
each other. This is called constructive interference. Let the distance from S1 to any point P be r1
and let the distance from S2 to P be r2.
For constructive interference to occur at P, the path difference r2 r1 between two waves must
be an integral multiple of the wavelength ;
1
r2 r1 = m (m = 0, μ1, μ2, μ 3...) ...(ii)
2
When a light travels from rarer to denser then there is a phase difference of and path difference
of .
2
WAVE OPTICS
QUIZRR 7
Now, when a glass slab is placed in its path, then path difference can be :
(d+( 1)t)
A´ B´ (optical path)
A B (origional path)
Thomas Young in the year 1801 first demonstrated the phenomenon of interference in light. He
allowed sunlight to pass through a pinhole S and then through two pinholes S1 and S2, placed
at a good distance away from S as shown in Fig. The interference pattern obtained on the screen
AB consists of a few coloured bands of different widths with a white central band.
B y
S1
S S2
z
A
y
x
D
d
Now a days pinholes S1 and S2 are replaced by two narrow identical slits* (which increase the
brightness) and sunlight by monochromatic light (which increases the number of bands). The
pattern consists of a large number of euqally spaced alternate bright and dark bands (called
fringes) running parallel to the length of the-slit.
Through this experiment measuring the wavelength of light (characteristic of wave), Young
conclusively established the wave-nature of light.
WAVE OPTICS
8 QUIZRR
P rinciple
Monochromatic light (single wavelength) from a narrow vertical slit S falls on two other narrow
slits S1 and S2 which are very close together and parallel to S. S1 and S2 act as two coherent
sources (both. being derived from S). If S, S1 and S2 all are very narrow, diffraction (bending of
light at openings whose width is of the order of wavelength of light) causes the emerging beams
to spread into the region beyond the slits. Superposition occurs in the shaded area, where the
diffracted beams overlap. Alternate bright and dark equally spaced vertical bands (interference
frignes) can be observd on a screen placed at some distance from the slits. If either of S1 or S2
is covered, the fringes disappear.
Diffracted beam
from S1
S1 Interference
Monochromatic effects in
S region where
light source
S2 beams overlap
Single Double
slit slit
Diffracted beam
from S2
Theory
An expression for the separation of two bright (or dark) fringes (called fringe width W) can be
obtained as under.
Figure shows the light waves from S1 and S2 meeting at an arbitrary point P on the screen. Since
D>>d, the two light rays are approximately parallel with a path diffference.
to P
Screen
P
r1
y to P
S1 r2
d
d O
S2
D
r2 r1 = d sin
WAVE OPTICS
QUIZRR 9
x = S2P S1P
d sin ...(i)
For maximum intensity at P,
x = n (n = 0, μ 1, μ2,...)
or d sin = n (n = 0, μ 1, μ2,...) ...(ii)
The bright fringe corresponding to the integer n is called the nth order (or just nth) bright fringe.
The bright fringe for n = 0, is know as the central fringe and its centre (point O) is called the
central (or zero order) mamimum. Higher order bright fringes are sutuated symmetrically about
the central fringe.
For minimum intensity at P,
x 2n 1 (n = μ 1, μ2,...)
2
or d sin 2n 1 (n = μ 1, μ2,...) ...(iii)
2
The first minima (n = μ1) are adjacent to the central maximum on either side. It is useful to obtain
expressions for the positions of the bright and dark fringes measured vertically from O. In
addition to our assumption that D >> d, we assume that d>>. These can be valid assumptions
because in practice D is often of the order of 1 m, d a fraction of a milimetre and a fraction of
a micrometre for visible light. Under these conditions is small, thus, we can use the appromixation,
y
sin tan
D
nD
ybright = (n = 0, μ1, μ 2,...) ...(iv)
d
Fringe Width () : Distance between two adjacent bright (or dark) fringes is called the fringe
width. It is denoted by . Thus.
n D (n 1) D D
d d d
D
d
Alternatively we can show that distance between two successive dark fringes is also D .
d
WAVE OPTICS
10 QUIZRR
Angular fringe width
y
As is very small, the equation tan can be reduced to
D
y
Therefore ... (2)
D
d
D
But , therefore from (2) we obtain.
d
d
If the light waves of wavelength interfere, then the fringe width is given by
D
d
´ ... (1)
D´
´ ... (2)
d
D
´
d
WAVE OPTICS
QUIZRR 11
This gives the new fringe width. From (1), differentiating we get,
D
...(3)
d
D
( )´ ...(4)
d
If the wavelength is constant, that is the medium is not changed, but slits separation is varied
then,
D
d´ taking 2d = d´.
d´
Note :
I I1 I2 2 I1 I2 cos
Putting I1 = I2 = I0
We have I = I0 + I 0 + 2 I 0 I0 cos
I = 4I0 cos2
2
cos 1 or n (n = 0, μ1, μ2,...)
2 2
2
or = 2n or x 2n
or x = n or d sin = n
yd nyD
or n or y
D d
and this condition, we have already discused earlier. Further, the maximum intensity is
Imax = 4I0
or I =Imax cos2
2
WAVE OPTICS
12 QUIZRR
Minimum intensity on the screen is found at points where,
1
cos 0 or n (n = μ1, μ2, μ3...)
2 2 2
2
or = (2n – 1) or x (2n 1)
or x (2n 1) or d sin (2n 1)
2 2
yd (2n 1)D
or (2n 1) or y
D 2 2
2 3 O 3 2 d sin
2 2 2 2
3. Fringe width (w) is the distance between two successive maximas or minimas. It is given by,
D
w
d
w
Two conclusions can be drawn from this relation
(ii) If white light is used in place of a monochromatic light then coloured fringes
are obtained on the screen with red fringes of larger size than that of violet, because
red > violet.
WAVE OPTICS
QUIZRR 13
But note that centre is still white because path difference there is zero for all colours. Hence,
all the wavelengths interfere constructively. At other places light will interfere destructively
for those wavelengths for whom path difference is /2, 3/2,... etc. and they will interfere
constructively for the wavelengths for whom path difference is , 2,... etc.
4. Shifting of fringes : Suppose a glass slab of thickness t and refractive index ø is inserted
onto the path of the ray emanating from source S1 then the whole fringe pattern shifs
D tD
upwards by a distance . This can be shown as under,
d P
Geometric path difference between S2P and S1P is,
y
yd S1
x1 S2 P S1 P
D
yd
x ( 1)t
D
yd
or ( 1)t n
D
nyD ( 1)tD
y
d d
nyD
Earlier it was
d
1 tD
Shift
d
WAVE OPTICS
14 QUIZRR
(b) Shift is independent of , i.e., if white light is used then,
shift of red colour fringe = shift of violet colour fringe.
shift
(c) Number of fringes shiftd
fringewidth
1 tD / d
1 t
D / d
These numbers are inversely proportional to . This is because shift is same for all colours but
fringe width of the colour having smaller value of is small, so more number of fringes will shift
of this colour.
Example 1
In youngÊs experiment, the interfering waves have amplitudes in the ratio 3:2. Find the
ratio of (a) amplitudes and (b) intensities, between the bright and dark fringes.
Solution :
In case of interference
Resultant amplitude, for given A1 and A2, is maximum for cos = 1 and minimum for cos = 1.
2
Amax A12 A22 2 A1 A2 ( A1 A2 )2
= (A1 A2)2
or Amin = A1 A2, assuming A1 > A2; otherwise A2 A1, (dark fringe)
A max A1 + A 2
Here we have to obtain the ratio = and also the corresponding ratio of
A min A1 - A 2
I max A 1 + A 2
2
intensities I =
min A1 - A 2 2
A1 3
(given)
A2 2
WAVE OPTICS
QUIZRR 15
A1 + A 2 32
By componendo and dividendo 5
A1 - A 2 32
A max
Hence =5
A min
I max A 1 + A 2
2
and = 25 Ans.
I min A1 - A 2 2
Example 2
Find the maximum intensity in case of interference of a identical waves each of intensity
I0 if the interference is (a) coherent (b) incoherent.
Solution :
In case of interference of two waves.
I = I1 + I2 + 2
I1I2 cos
(a) In case of coherent interference at a given point, does not vary with time and so I will
be maximum when cos = max = 1, i.e.,
I max Co = n I0
2 2
I0 + I0 ...
= n2I0
(b) In case of incoherent interference at a given point, varies randomly with time,
So, (cos )av = 0
and hence, (IR)Inco = I1 + I2
So, in case of n identical waves,
(IR)Inco = I0 + I20 + ... = nI0 Ans.
Example 3
YoungÊs double slit experiment is performed inside water (ø = 4/3) with light of frequency
6 ï 1014 Hz. If the slits are separated by 0.2 mm and the screen kept 1m from the slits, find
the fringe width. Using the same set-up, what will the fringe width be if the expriment is
performed in air ?
WAVE OPTICS
16 QUIZRR
Solution :
Wavelength of given light in air can be obtained from the relation
c = fair
c 3 108
air 5 10 7 m
f 6 1014
air 5 107
water
water 4/3
water 3.75 10 7 m
As the experiment is performed in water,
water D
fringe width
d
3
d = 0.2 mm = 0.2 10 m
Here D = 1m
3.5 107
m
0.2 103
or = 1.87 mm
If the experiment is performed in air,
air D 5 10 7
fringe width, m
d 0.2 103
or = 2.5 mm Ans.
Example 4
Fig. shows two radio antennas kept at a distance 400m. The antennas broadcast the ame
signals at the same wavelength. A person, having a radio, is standing at the position of thid
order maximum. Find the wavelength of signals.
A1
800m
400m
A2 600m
P(Person)
WAVE OPTICS
QUIZRR 17
Solution :
So as to obtain path difference between the signals from A1 and A2 at P, the path lengths A1P
and A2P need to be determined.
A1
N
200m 800m
M
200m
L
A2 600m
P (Person)
2t (2n 1) n = 1, 2, 3, ...
2
1
or 2t n as.
2
2t n n = 0, 1, 2...
Example 5
Calculate the minimum thickness of a soap bubble film (ø = 1.33) that results in contructive
interference i the reflected light if the film is illuminated with light whose wavelength in
free space is = 600 nm.
Solution :
For constructive interference in case of soap film,
1
2t n n = 1, 2, 3, ...
2
or 2t
2
600
or t
4 4 1.33
= 112.78 mm Ans.
WAVE OPTICS
QUIZRR 19
Example 6
Solution :
Ray 1 has a longer path than that of ray 2 by a distance d sin 45 , before reaching the slits.
Afterwards ray 2 has a path longer than ray 1 by a distance d sin . The net path path difference
is therefore, d sin d sin 45 .
(a) Central maximum is obtained where, net path difference is zero,
or d sin d sin 45 = 0
or = 45 Ans.
(b) Third order maxima is obtained where net path d9fference is 3, or
d sin d sin 45 = 3
3
sin = sin 45 +
d
Putting d = 20
3
we have sin = sin 45 +
20
or 59 Ans.
DIFFRACTION
If an opaque obstacle (or aperture) is placed between a source of light and a screen, in accordance
with rectilinear propagation of light, usually a sharp shadow or an illuminated region is obtained
on the screen as shown if the size of an obstacle or aperture is comparable with the wavelength
of light, light deviates from rectilinear propagation near the edges of the obstacle (or aperture)
and enters the geometrical shadow. This flaring out or encroachment of light in the shadow zone
as it passes around obstacles or through small apertures is called diffraction.
A´ Light
No light
A A
A´
Light Shadow
S S B´
B B
No light B´
Light
(A) Aperture (B) Obstacle
WAVE OPTICS
20 QUIZRR
As a result of diffraction, the edges of the shadow (or illuminated region) do not remain well
defined and sharp but become blurred and fringed.
Consider a narrow slit AB of width d kept perpendicular to the plane of the paper. Lat a plane
wavefront of monochromatic light be incident normally on this slit. As the width d of the slit is
comparable to the wavelength of the light, the light gets diffracted on passing through the slit.
The converging lens L2 helps in focusing the diffraction pattern on the screen. The diffraction
pattern consists of a bright band at its centre (O) with alternate dark and bright bands on both
sides. The intensity of the bright bands decreases very sharply as one move away from O on both
sides.
W2 L2
P
A t
d
S O
M
B N screen
slit
L1 W2 D
Plane wave front
Explanation : As the plane wavefront W1 W2 is incident normally on the slit, the entire slit AB
gets, illuminated. According to HygenÊs principle, each point on the portion AB of the wavefront
becomes the source of secondary disturbance. The secondary wavelets start spreading from AB in
all directions.
The secondary wavelets travelling parallel to MO are focussed at O by the converging lens I2.
Actually, the secondary wavelets orginating from any two points equidistant from the centre of
the slit M (one each in MA and MB) cover equal distances before converging at O. Hence, thepath
difference between them is zero. As a result, they produce the brightest band at O on the screen.
This is called the central maximum.
Consider the secondary wave travelling in a direction makig an angle with MO. All these waves
are focussed by lens L1 at point P. Such waves start from all the points of slit AB in the same
phase but travel different distances in reaching the point P. The intensity at point P will, therefore,
depend on the path difference between the secondary waves emitted from the corresponding
points (equidistant from M) of the wavefront AB.
WAVE OPTICS
QUIZRR 21
Let us calculate the path difference between two secondary waves emitting from points A and B
of the wavefront. Draw AN perpendicular to the wave coming from B.
Path difference between the secondary wavelets which originate from A and B and reach
point P on the screen, BN = AB sin = d sin .
sin n
d
For small values , sin n = n n
d
Secondary maxima : Let the path difference between the wavelets originating from A and B
and reaching an off point P1 (not shown in the figure) be 3/2.
d sin = 3/2
We assume that the slit AB is divided into three equal parts so that the difference between the
wavelets originating from the corresponding points in the first two parts is /2. Such wavelets
cause destructive interference at P1. However, the wavelets originating from different point of the
third part reinforce each other (not completely as for them, 0 > d sin < /2) and give rise to the
first secondary maxima at P1. The intensity at the first secondary maxima is much smaller than
that of the central maxima.
Similarly, d sin = 5/2 give the second secondary maxima with much lower intensity than the
first maxima.
In general, for nth secondary maxima the have d sin = (2n + 1)/2
WAVE OPTICS
22 QUIZRR
Where n = 1, 2, 3 ....
The direction of nth secondary maxima is
(2n 1)
sin = n =
2d
Thus, the diffraction pattern due to a single slit consists of central maxima flanked by alternate
minima and secondary maxima. The intensity distribution in the pattern on the screen in shown
in figure.
Intensity
st
I minima
st
I maxima
nd
II minima
nd
II maxima
3 2 0 2 3
The graph shows that as we go away from the central maxima O on both sides, the intensity of
secondary maxima decreases very rapidly.
It is the distance between the first secondary manimum an either side of the central maxima
Let y be the distance of a first minimum from the centre (O) of the central maxima.
D
d sin = and sin
d
Let f be the focal length of the converging lens L2 (placed very close to slit AB) and D is the
distance of the screen from AB.
D = f
y y
Hence,
f D
y D
Comparing equation, we get , y
D d d
WAVE OPTICS
QUIZRR 23
2D 2f
width of central maxima = 2y = , y
d d
Figure shows the widths of the central maxima (CD) and the first secondary maximum
(DE or CF)
Let y1 and y2 respectively be the distance of the first minima and the second minima from O. Then
2D D D
width of the first secondary maximum = y2 y1 = .
d d d
The width of the central maxima is twice the width of the first secondary maximum.
Diffraction Pattern At Single Slit Due To Monochromatic Light and White Light
When a parallel beam of monochromatic light is incident normally on the slit, the diffraction
pattern obtained on the screen consists of alternate bright and dark bands of equal width. The
intensity is maximum at the central bright band and it decreases very rapidly for successive
secondary bright bands i.e. maximum.
When the beam of white light is incident on the slit, the diffraction pattern is coloured.The central
maxima is white but the other bands are coloured. As the band width the red band will be
wider than the violet band (as R > v).
We can use phasors to determine the light intensity distribution for a single slit diffraction
pattern. The proof is beyond our syllabus, we are just writing here the intensity at angle .
2
sin / 2 a sin
I I0 Here
/ 2 2
2
sin a sin /
or I I0 (at angle )
a sin /
a sin
m
or sin m m 1, 2,...
a
WAVE OPTICS
24 QUIZRR
a sin
Note : The sin = 0, corresponds to central maxima while , corresponds to first minima.
I
I0
2 2 2
Diffraction Grating : It consists of large number of equally spaced parallel slits. If light is
incident normally on a transmission grating, the direction of principal maxima is given by.
d sin = n
Here d is the distance between two consecutive slits and is called grating element.
n = 1, 2, 3, ... is the order of principal maximas.
The diffraction grating is most useful for measuring wavelengths accurately. Like the prism, the
diffraction grating can be used to disperse a spectrum into its wavelength components. Of the two
devices, the grating is the more precise if one wants to distinguish two closely spaced wavelengths.
For two nearly equal wavelengths 1 and 2 between which a diffraction grating can just barely
distinguish, the resolving power R of the grating is defined as
R
2 1
Example 7
Light of wavelength 6328 AĈ from a He-Ne LASER falls on a single slit of width 0.4 mm and
the resulting diffraction pattern is observed on a screen 1.8 m away. Find the (a) distance
between the firt dark fringes on either side of central bright firnge and (b) distance of
second order brihgt fringe from the (centre of) central bright fringe, assuming a small angle
of diffraction.
Solution :
10
Give = 6328 A = 6328 10 m
WAVE OPTICS
QUIZRR 25
d = 0.4 mm = 0.4 10 3m
D = 1.8m
(a) Distance between the first dark fringes on either side of central bright fringe is also the
width of central maximum and, for small angle of diffraction, it given by
2D
x
d
= 5.69 10 3m
= 5.69mm Ans.
(b) Second order maximum is formed between second and third minima.
Condition for minima can be expressed as
d sin = n
n
sin
d
y
For small , sin tan [y is the distance of any point]
D
yn n
D d
nD
yn 3rd minimum
d 2nd maximum
2nd minimum
Hence second order minimum is fromed at
O
2D 3D
distance from O and formed at distance from O.
d d
Therefore, second order maximum, formed between second
and third minima, will be at distance, D
2D 3D
d d 5D from O.
2 2d
= 7.1 10 3m
or, y = 7.1 mm Ans.
WAVE OPTICS
26 QUIZRR
Example 8
A screen is placed 50 cm from a single slit, which is illuminated with 6000 A light. If distance
between the first and third minima in the diffraction pattern is 3.00 mm, what is the width
of the slit ?
Solution :
In case of diffraction at single slit, the position of minima is given by
d sin = n
and for small : sin = = (y/D) ...(2)
So from Eqs. (1) and (2), we have :
y D
d n, i.e., y n
D d
D D
So that, y3 y1 3 (2)
d d
0.50 (2 6 107 )
and hence d 3
2 104 m
3 10
= 0.2 mm Ans.
Example 9
A single slit is illuminated with a parallel beam of light of wavelength 5890 A and a
diffraction pattern is obtained on a screen 1.2 m from the slit. The distance between positions
of zero intensity on both sides of central maximum is found to be 3.5 mm. Find the (a) width
of slit (b) angle of diffraction for third order bright fringe (c) angular width of the thrid
order bright fringe, assuming small angles of diffraction.
Solution :
10
Given = 5890 A = 5890 10 m
D = 1.2 m
(a) Distance between positions of zero intensity on both sides of central maximum is also the
width of central maximum.
3
Here it is 3.5 mm, i.e., 3.5 10 m.
2D
Width of central maximum =
d
WAVE OPTICS
QUIZRR 27
d = 0.0004 m
Width of slit d = 0.4 mm
(b) Condition for nth order minimum can be expressed as path difference = d sin = n.
For 3rd dark fringe
d sin = 3
3rd bright fringe is formed between third and fourth minima.
For 3rd bright fringe.
3 4 7 4th dark
d sin
2 2 3rd bright
3rd dark
7
sin
2d O
7 5890 10 10
2 0.4 103
D
[d = 0.4 10 3m]
= 0.0051
= 2.29
Unpolarised Light
The ordinary light also called as unpolarised light, consists of a very large number of vibrations
in all planes with equal probability at right angle to the planes with equal probability at right
angle to the direction of propagation. So unpolarised light is symmetrical about its direction of
propagation.
Polarised Light
The light which has acquired the property of one sideness is called polarised light or the lack of
symmetry of vibration around the direction of wave propagation is called polarisation. Polarisation
of light waves exhibits that they are transverse waves. When the vibrations are confined only to
a single direction in a plane perpendicular to the direction of propagation, it is called a plane
polarised light. A plane passing through the direction of propagation and perpendicular to the
plane of vibration is known as plane of polarisation.
WAVE OPTICS
28 QUIZRR
Production of Plane Polarised Light
Nicol’s Prism
NicolÊs prism is a device for producing and analysing a plane polarised light. It is based on the
phenomenon of double refraction. Bartholinus discovered that when a beam of ordinary unpolarised
light is passed through a calcite crystal, the refracted light is split up into two refracted rays. The
one which always obeys the ordinary laws of refraction and having vibrations perpendicular to
the principal section is known as ordinary ray. The other, in general does not obey the laws of
refraction and having vibration in principal section is called as extra-ordinary ray. Bot the rays
are plane polarised. The phenomenon is known as double refraction. The crystals showing this
phenomenon are know as doubly refracting crystals.
WAVE OPTICS
RAY OPTICS
QUIZRR 3
1 . I N T RODU CT I ON
The branch of physics called optics deals with the behaviour of light and other electromagnetic
waves. Light is the principal means by which we gain knowledge of the world. Consequently the nature
of light has been the source of one of the longest debates in the history of science.
Electromagnetic radiation with wavelengths in the range of about 4000 Å to 7000 Å, to which eye
is sensitive is called light.
Our investigation of light will revolve around two questions of fundamental importance (1) What
is the nature of light and (2) How does it behave under various circumstances ? The answers to these
two questions can be found in MaxwellÊs field equations (which is out of JEE syllabus). These equations
predict the existence of electromagnetic waves that travel at the speed of light. They also describe how
these waves behave. Interestingly, not all light phenomena can be explained by MaxwellÊs theory.
Experiments performed at the beginning of this century showed that light also has corpuscular, or
particle like properties.
We investigate the behaviour of a beam of light when it encounters simple optical devices like
mirrors, lenses and apertures. Under many circumstances, the wavelength of light is negligible compared
with the dimensions of the device as in the case of ordinary mirrors and lenses. A light beam can then
be treated as a ray whose propagation is governed by simple geometric rules. The part of optics that
deals with such phenomena is known as geometric optics. However, if the wavelength is not negligible
compared with the dimensions of the device (for example a very narrow slit), the ray approximation
becomes invalid and we have to examine the behaviour of light in terms of its wave properties. This
study is known as physical optics.
2 . RA Y
A ray can be defined as an imaginary line drawn in the direction in which light is travelling.
Light behaves as a stream of energy propagated along the direction of rays. The rays are directed
outward from the source of light in straight lines.
2 .1 B e a m o f L i g h t
A beam of light is a collection of these rays. There are mainly three types of beams.
(i) Parallel beam of light : A search light and the headlight of a vehicle emit a parallel beam
of light. The source of light at a very large distance like sun effectively gives a parallel beam.
(ii) Divergent beam of light : The rays going out from a point source generally form a
divergent beam.
OPTICS
4 QUIZRR
(iii) Convergent beam of light : A beam light that is going to meet (or convery) at a point is
known as a convergent beam. A parallel beam of light of passing through a convex lens
become convergent beam.
3 . REFLECT I ON
When a ray of light is incident at a point on the surface, the surface throws partly or wholly the
incident energy back into the medium of incidence. This phenomenon is called reflection.
Surface that causes reflection are known as mirrors or reflectors.
3 .1 I m p o r t a n t t e rm s r e l a t e d t o r e f l e c t i o n :
A N B i
N O
r
i r
B
Plane Mirror Concave Mirror
Angle of incidence : Angle which the incident ray makes with the normal at the point of
incidence. (denoted by i)
Angle of reflection : The angle which the reflected ray makes with the plane normal of the
point of incidence. (denoted by r).
Glancing Angle : The angle which the incident ray makes with the plane reflecting surface is
called glancing angle. It is generally denoted by g. g = 90 ă i.
i
N O
r
B
Convex Mirror
OPTICS
QUIZRR 5
3 .2 L a w o f Re f l e c t i o n
(i) The incident ray, the reflected ray and the normal to the reflected surface at the point of
incidence lie in same plane
(ii) The angle of incidence is equal to the angle reflection, i.e. i = r. These laws hold for all
reflecting surfaces either plane or curved.
Question: Do incident ray and reflected ray differ in terms of frequency, wavelength and speed ?
Solution : None of frequency, wavelength and speed changes due to reflection. However, intensity
and have amplitude (I A2) usually decreases. However in ray optics we do not consider this change.
Question : What will happen if the ray falls normal to the surface of the mirror ?
Solution : In this case, when i = 0, the ray retraces its path. i.e. r = 0.
C C
3 .3 Re a l / V i rt u a l Ob j e c t (I m a g e ) :
When a group of incident rays strike a surface, the point from which they diverge or the point
a which they try to converge is known as object. After reflection we get a group of reflected rays. The
point where reflected rays converge or the point from which the reflected rays appear to diverge is
known as the image of the object.
Objects and images may be real or virtual. If the light, after reflection, actually passes through
the point where the image is located, the image is called as a real image; if the light only appears to
come from the point where the image is located, the image is called a virtual image. A real image will
appear on a screen placed at its position while a virtual image will not.
Similarly, if the object is a point where the light energy is actually present, it is a real object.
If the light rays only tend to go towards the point to here the image is located, it is a virtual object.
(Real
object)
O I (Virtual
image)
I (Real image
virtual object)
Convex Mirror
OPTICS
6 QUIZRR
3 .4 Re f l e c t i o n f r o m a p l a n e s u r f a c e (o r p l a n e m i rr o r)
Almost everybody is familiar with the image formed by a plane mirror. If the object is real, the
image formed by a plane mirror is virtual, erect, of same size and at the same distance from the mirror.
The ray diagram of the image of a point object and of an extended object is as shown below.
B B´
O I
A A´
Important points :
1. To find the location of image of an object from an inclined plane mirror, you have to see the
perpendicular distance of the object from the mirror.
I
M
O I
M
O
Correct Wrong
OM = MI OM = MI
2. Suppose an object O is placed in front of two plane mirrors M1 and M2. Size of M2 is more
than that of M1. In this case the intensity of the images formed by M2 (i.e., I2) will be more
than that formed by M1 (i.e. I1). This is because I2 is formed from more number of reflected
O I1 O I1
M1 M2
rays. Or we can say it is formed from more light. The same is the case with an image formed
by a lens of large aperture.
OPTICS
QUIZRR 7
O I1 I2
O
In the figure shown, I2 will be more intense. This is the reason why we choose a lens of large
aperture in telescopes to see the distant objects. You might have heard that the strength of
an observatory is measured from the aperture of its lens. Now, you can think if this is the
case then why donÊt we take a lens of an aperture as large as we wish. Actually some
technical problems arise in casting a lens of large aperture. So we have some limitations in
doing so.
3. Field of view of an object for a given mirror : Suppose a point object O is placed in
front of a small mirror as shown in Fig. (a), then a question arises in mind whether this
mirror will form the image of this object or not or suppose an elephant is standing in front
of a small mirror, will the mirror form the image of the elephant or not. The answer is yes,
it will form. A mirror whatever may be the size of it forms the images of all objects lying
in front of it. But every object has its own field of view for the given mirror. The field of
view is the region between the extreme reflected rays and depends on the location of the
object in front of the mirror. If our eye lies in the field of view then only we can see the
image of the object otherwise not. The field of view of an object placed at different locations
in front of a plane mirror are shown in Fig. (b) and (c).
O I
O
O I
(a) (b) (c)
4. Suppose a mirror is rotated by an angle (say anticlockwise), keeping the incident ray fixed
then the reflected ray rotates by 2 along the same sense, i.e., anticlockwise.
OPTICS
8 QUIZRR
R´
N´ iă2
N iă
I R I
iă
i i X
(a) (b)
In figure (a), I is the incident ray, N the normal and R the reflected ray.
In figure (b), I remains as it is. N and R shift to N´ and R´.
From the two figures we can see that earlier the reflected ray makes an angle i with y-axis
while after rotating the mirror it makes the angle i ă 2. Thus, we may conclude that the
reflected ray has been rotated by angle 2.
H
5. The minimum length of a plane mirror to see oneÊs full height in it is , where H is the
2
height of man. But the mirror (F) reaches the eye at C. Similarly the ray starting from the
foot (E) after reflecting from the lower end (G) also reaches the eye at C. In two similar
triangles ABF and BFC, AB = BC = x (say), Similarly in triangles CDG and DGE,
CD = DE = y (say)
A
X F
B
X
C
(X+Y)
Y
D
G
Y
E
Man
Now, we can see that height of the man is 2 (x + y) and that of mirror is (x + y), i.e., height
of the mirror is half the height of the man.
Note : The mirror can be placed anywhere between the centre line BF (of AC) and DG
(of CE).
6. A man is standing exactly at midway between a wall and a mirror and he want to see the
full height of the wall (behind him) in a plane mirror (in front of him). The minimum length
H
of mirror in this case should be , where H is the height of wall. The ray diagram in this
3
case is drawn in Fig. below.
OPTICS
QUIZRR 9
In triangles HBI and IBC, HI = IC = x (say), Now, in triangles HBI and ABF,
AF FB A H
>
HI BI 2x x B
F
I x
AF 2d
or (x+y) C (x+y)
x d
G K y
or AF = 2x y E
2y >
D J
Wall Man Mirror
d d
7. Number of images formed between two mirrors inclined at an angle with each other is
given by :
360
m
Ex a m p l e 1
A point source of light S, placed at a distance L in front of the centre of a mirror of width
d, hangs vertically on a wall. A man walks in front of the mirror along a line parallel to
the mirror at a distance 2L from it as shown. The greatest distance over which he can see
the image of the light source in the mirror is
(a) d/2
(b) d S
(c) 2d
d
L
(d) 3d
2L
OPTICS
10 QUIZRR
Solution :
(d) The ray diagram will be as shown in Fig.
G
HI = AB = d
C
d
DS = CD =
2 A
H
D
Since, AH = 2AD
S
d E
GH = 2CD = 2 d I
2
B
Similarly IJ = d
F
GJ = GH + HI + IJ
J
= d + d + d = 3d
Ex a m p l e 2
At ray of light is travelling at an angle of 20Ĉ below horizontal. At what angle with horizontal
must a plane mirror be placed in its path so that it becomes vertically upwards after
reflection ?
Solution :
Let us first place the mirror horizontally. The reflected ray now goes at an angle of 20Ĉ above
horizontal as shown.
20Ĉ 20Ĉ
Now, for the reflected rays to be vertical, let us assume that the mirror makes an angle with
the horizontal.
70
Ĉă
70
Ĉ
ă
20Ĉ
Now, from the figure, if reflected ray is vertical, then 20Ĉ + (70Ĉ ă ) + (70Ĉ ă ) = 90Ĉ
70Ĉ = 2 = 35Ĉ
OPTICS
QUIZRR 11
Ex a m p l e 3
A boy is 1.5 m tall and just sees his image in a vertical plane mirror 3 m away. His eyes are
1.4 m from the floor level. Determine the vertical dimension of the mirror.
Solution : A M A´
0.1 m
D
E E´
C
H
1.4m
3m 3m
B R B´
Since a plane mirror forms an image at a distance same as that of object, hence image will be
formed at 3m behind the mirror. Now let AB be the height of boy, the image formed is A B . Now,
1.4
triangles AA E and AEE are congruent, also MD = DC = 0.05 m. Similarly, CH = HR = = 0.70 m.
2
Hence, height vertical dimension is DC + CH = 75 cm.
3 .5 Re f l e c t i o n a t Sp h e r ic a l M i rr o rs
(a) Some important definitions :
(i) Spherical mirrors : A spherical mirror is a part of follow sphere or a spherical surface.
They are classified as concave or convex according to the reflecting surface being concave
or convex respectively.
A A
Incident
Incident Light Light
P
+ve C
C
B B
(a) Concave Mirror (b) Convex mirror
(ii) Centre of currature : The centre C of the sphere of which mirror is a part is called the
centre of curvature of the mirror.
(iii) Pole or Vertex : P, the centre of the mirror surface is the pole.
(iv) Radius of curvature (R) : The distance CP is called the radius of curvature.
Note : All distances are measured from point P. We can see using the rules of sign convention
given earlier that R is positive for convex mirror and negative for concave mirror.
OPTICS
12 QUIZRR
(v) Principal axis : The line CP joining the pole and the centre of curvature of the spherical
mirror is called the principal axis.
(vi) Focus (F) : If a parallel beam of rays, parallel to the principal axis and close to it, is incident
on a spherical mirror; the reflected rays converge to a point F (in case of a concave mirror)
or appear to diverge from a point F (in case of convex mirror) on the principal axis. The point
F is called the flows of the spherical mirror.
F
P F
P
SI GN CON V EN T I ON M´
Although a number of books follow different sign conventions, but we will be using the following sign
conventions :
(1) Whenever and wherever possible, the ray of light is taken to travel from left to right.
(2) The distances above principal axis are taken to be positive while below it is negative.
(3) Along the principal axis, distances are measured from the pole and in the direction of light are
taken to be positive while opposite if is negative. This is why in our convention, u (or v) is taken
to be ăve if in front of the mirror and +ve if behind it. +
Light +ve
ă +
P +ve
C F
ă ve
ă ve
ă
3 .5 .1 Pa r a x i a l Ra y s :
Rays which are close to the principal axis and make small angles with it, i.e., they are nearly
parallel to the axis, are called paraxial rays.
Our treatment of spherical mirrors will be restricted to such rays which means we shall consider
only mirrors of small aperture. In diagrams, however, they will be made larger for clarity.
And rays far away from principal axis are called Marginal Rays.
OPTICS
QUIZRR 13
3 .5 .2 Re l a t io n b e t w e e n f a n d R
A ray AM parallel to the principal axis of a concave mirror of
small aperture is reflected through the principal focus F. If C is the A M
centre of curvature, CM is the normal to the mirror at M because the
P
radius of a spherical surface is perpendicular to the surface. From first C F
law of reflection,
i = r
or AMC = CMF \ (say) f
R
But AMC = MCF (alternate angles)
CMF = MCF
Therefore, D FCM is thus isosceles with FC = FM. The rays are paraxial and so M is very close
to P. Therefore,
FM FP
FC = FP
1
or FP = CP
2
R
f
2
3 .5 .3 Ra y T ra c i n g :
(i) Ray parallel to principal axis after reflection passes through focus.
(ii) Rays parallel to each other but not parallel to principal axis form image on the focal plane.
(iii) Incident Rays passing through C retrace their path.
(iv) The incident and reflected ray make equal angle with principal axis if the ray is incident
at 0. {this is used to locate the position of mirror}.
3 .5 .4 Ru l e s f o r I m a g e Fo r m a t i o n :
(i) A ray parallel to principal axis passes (or appear to pass) through focus after reflection.
P
F P F
Concave mirror
Convex mirror
OPTICS
14 QUIZRR
(ii) A ray passing through or directed towards focus after reflection from the spherical mirror
becomes parallel to the principal axis.
P
F P F
(iii) A ray passing through or directed towards the centre of curvature, after reflection from the
spherical mirror, retraces its path.
P
C F C
Ć Above rule holds, if incident ray is travelling from left to right else directions just
reverses.
O
F O F
Ć Distances measured above principal axis are taken to be positive while distances measured
below principal axis are taken to be negative.
notations used :
u: Distance of the object from pole of spherical mirror.
v: distance of the image from the pole of the spherical mirror.
f: focal length of the spherical mirror.
R: Radius of curvature of the spherical mirror.
Ć All the symbols used are assigned values with proper sign convention. i.e. for concave mirror
f is always negative and so on.
OPTICS
QUIZRR 15
3 .5 .6 . I m a g e Fo r m a t i o n b y Sp h e r ic a l M ir ro r s
(a) For concave mirror
S.No. Position of Ray-diagram Details of image
object
1. At infinity M Real, inverted,
3 .5 .7 . T h e M i r ro r Fo r m u l a
In figures (a) and (b) a ray OM from a point object O on the principal axis is reflected at M so
that the angle , made by the incident and reflected rays with the normal CM are equal. A ray OP
strikes the mirror normally and is reflected back along PO. The intersection I of the reflected rays MI
and PO in figure (a) gives a real point image of O and in figure (b) gives a virtual point image of
O. Let , and be the angles as shown. As the rays are paraxial, these angles are small, we can
MP MP MP
take tan , and .
OP CP IP
M M
O C I P
O P C
u u
R v
v R
(a) (b)
Now, let us take the two figures simultaneously
Concave Convex
In triangle CMO, = + (the exterior angle) In triangle CMO,
or = ă ... (i) = + ... (iv)
In CMI, = + (the exterior angle)
= ă ... (ii) In CMI = +
From Eqs. (i) and (ii) or = ă ... (v)
2 = + ... (iii) From Eqs. (iv) and (v)
Substituting the values of , and , we get 2 = ă ... (vi)
Substituting the values of , and , we get
2 1 1
= + ... (A)
CP IP OP 2 1 1
= + ... (B)
CP IP OP
If we now substitute the values with sign, i.e.,
If we now substitute the values with sign, i.e.,
CP = ă R, IP = ă v and OP = ă u
CP = ă R, IP = ă v and OP = ă u, we get
2 1 1
we get, = + 2 1 1
R v u = +
R v u
1 1 1 R
or + as f 2 1 1 1 R
v u f or + as f 2
v u f
OPTICS
QUIZRR 17
Note : Out of u, v and f values of two will be known to us and we will be asked to find the third.
In such type of problems two cases are possible. Case 1 is when signs of all three will be known to us
from the given information. Under this condition substitute all three with sign, then answer (i.e., the
third quantity) will come without sign. Only numerical value of the unknown comes. Case 2 is when
the sign of third unknown quantity is not known to us. Under such situation substitute only the known
quantities with sign. Sign with numerical value will automatically come in the answer.
Power of Mirror
It is defined as,
1
P The unit of power is dioptre.
f (in m)
3 .5 .8 . L i n e a r M a g n i f ic a t i o n
For linear objects, the ratio of the image size (I) to the object size (O) is called linear magnification
or transverse magnification or lateral magnification. If is generally denoted by m.
v f ( f v)
m = ă =
u f u f
Ex a m p l e 4
An object 5 mm high is placed 30 cm from a convex mirror whose focal length is 20 cm. Find
the position (in cm), size (in mm) and nature of the image.
Solution :
Given
u = ă 30 cm, f = + 20 cm
1 1 1
we know +
v u f
uf ( 30) ( 20)
or v 12 cm
u f 30 20
I v 12
Now, m O u 30
OPTICS
18 QUIZRR
2 2
or, I= O 5 2 mm
5 5
He the height of the image = 2 mm. The positive sign indicates that the image is erect.
Ex a m p l e 5
A plane mirror is placed 22.5 cm in front a concave mirror of focal length 10 cm. Find where
an object can be placed between the two mirrors, so that the first image in both the mirrors
coincide.
M
22.5
IP
X
P
C O F
IC
Solution : M´
As shown in figure, if the object is placed at a distance x from the concave mirror, its distance from
the place mirror will be (22.5 ă x). So, plane mirror will form equal and erect image of object at a
distance (22.5 ă x) behind the mirror.
Now as according to given problem the image formed by concave mirror coincides with the image
formed by concave mirror, therefore for concave mirror.
v = ă [22.5 + (22.5 ă x)] = ă (45 ă x) and u = ă x
1 1 1 45 1
So, or
(45 x) x 10 45 x x2 10
Ex a m p l e 6
Find the distance of object from a concave mirror of focal length 10 cm so that image size
is four times the size of the object.
Solution :
Concave mirror can form real as well as virtual image. Here nature of image is not given in the
question. So we will consider two possible cases.
When image is real : Real image is formed on the same side of object, i.e., u, v and f all are
negative. So let,
OPTICS
QUIZRR 19
v
then v 4 x as m 4 and f = ă 10 cm
u
1 1 1
Substituting in, +
v u f
1 1 1
We have
4 x x 10
5 1
or
4 x 10
x = 12.5 cm Ans.
Note : |x|> |y| and we know that in case of a concave mirror, image is real when object lies
beyond F.
When image is virtual : In case of a mirror image is virtual when it is formed behind the mirror,
i.e., u and f are negative while v is positive. So let
u = ă y, also m = + 4 for image to be virtual
then v = + 4y and f = ă 10 cm
1 1 1
Substituting in, +
v u f
1 1 1
We have
4 y y 10
3 1
or
4 y 10
y = 7.5 cm Ans.
Note : Here |y|< |f| as we know that image is virtual when the object lies between F and P.
3 .5 .9 L o n g i t u d i n a l M a g n i f i c a t io n
When an object is kept parallel to the principle axis, then let u1, and u2 be the distance of the
end points of the object and let v1, v2 be the ends points of the image formed.
Here, AB is the object and A B is the image formed.
OPTICS
20 QUIZRR
u2
u1
A´ B´
B A
v2
v1
The point can be seen, as image of the objects closer to the mirror are created farther as compared
to the images of the objects farther from the mirror.
(v2 v1 )
Here, m
(u2 u1 )
dv
also, m {for short objects}
du
So m e I m p o r t a n t Po i n t s :
1. For spherical mirrors
v
m
u
Positive value of m means v and u are of opposite sign. So is u is negative then v is positive and
vice-versa. Thus, if m = + 2 for a real object, it means image is virtual, erect and two times greater
1 1
in size. Similarly m = ă means image is real, inverted and times in size (that of object).
2 2
2. Method of finding coordinates of image of a point object if the coordinates of object
are known : Suppose coordinates of a point object (x0, y0) with respect to the coordinate axes
shown in figure are known to us. The coordinates of image (xi, yi) can be obtained using
1 1 1 1 1 1
x0 xi f v u f Y
fx0
or xi ... (i)
x0 f (X0, Y0)
yi v x X
Similarly m i P
y0 u x0
fy0
or yi ... (iii)
f x0
OPTICS
QUIZRR 21
4. If an object is placed with its length along the principal axis, then so called longitudianal
magnification becomes,
I v v1 dv
mL 2 (for small objects)
O u2 u1 du
1 1 1
From, we have
v u f
ă vă2 dv ă uă2 du = 0
2
dv v
or
dy u
2
dv v
mL m2
du u
or
1 1 1
+
v u f
with respect to time, we get
dv du
v2 . u2 0 (as f = constant)
dt dt
dv v2 du
or
dt u2 dt
OPTICS
22 QUIZRR
du
Here is the rate by which u is changing. Or it
dt
is the object speed if mirror is stationary. Similarly,
I
dv P
is the rate by which v (distance between image O C F
dt
and mirror) is changing. Or it is image speed if mirror
is stationary. So if at a known values of v and u, the
v
object speed is given, we can find the image speed
u
from the above formula.
Let us take an example for a concave mirror.
Suppose the object is moved from infinity towards focus.
As u is decreasing therefore,
du
dt = rate of decrease of u (speed of object)
dv
Therefore, = rate of increase of v (speed of image)
dt
dv du
dt dt
Hence, as the object is moved towards mirror the image (which is real) will recede from the mirror
with speed less than the speed of object.
dv du
When the object is at C, image is also at C, i.e., v = u or . Hence, speed of image
dt dt
is equal to the speed of object. When the object lies between C and F. v > u, i.e., image speed is
more than the object speed when object comes inside F image becomes virtual i.e., u and f are
negative while v is positive.
1 1 1
Hence, ă ă
v u f
1 1 1
or
u v f P
C F
2 du 2 dv
or u dt v dt 0
u v
dv v du
2
dt 2 dt
u
OPTICS
QUIZRR 23
1 du
Now as u is further decreased, v also decreases to keep f constant. So, is rate with which
dt
dv
object is approaching towards mirror and is rate by which image is approaching towards
dt
mirror. Further in this case we know that image is always enlarged or v > u. Therefore, image
speed is more than the object speed. Thus, we may conclude the above discussion as under :
When an object is moved from ă to F, the image (real) moves from F to ă and the when
the object is further moved from F to P image (now virtual moves from + to P.
Note : when the object is either at centre of curvature C or at pole P the two speeds are equal.
When the object is at pole it can be assumed as if the image is forming by a plane mirror due
to the small aperture of the mirror.
1 1
6. Graph between versus : Let us first take the case of a concave mirror. Here, two cases
v u
are possible.
Case 1. When the image is real, i.e., object lies between F and inifinity. In such a situation u,
v and f are negative. Hence, the mirror formula
1 1 1
becomes
v u f
1 1 1
1
v u f v
1
f
1 1 1
or again
v u f
45Ĉ 1
1 1 1 1 u
or f
v u f
Comparing with y = ă x + c, the desired graph will be a straight line with slope ă1 and slope equal
1
to f .
1 1 1
v u f
OPTICS
24 QUIZRR
1
Comparing it with y = x ă c the desired graph is a straight line with slope + 1 and intercept .
f
The graph is thus shown in Fig. (a) The two graphs can be drawn in one single graph as in Fig. (b)
1 1
v v
1
f
45Ĉ 1 45Ĉ 45Ĉ 1
1 u 1 u
f f
ă1 ă1
f f
(a) (b)
1 1 1 1
Note : Here and are really the magnitudes of and (i.e., without sign)
u v u v
For a convex mirror image is always virtual, i.e., u is negative while v and f are positive. Hence,
the mirror formula becomes. 1
v
1 1 1
v u f
1
Comparing with y = x + c, the desired graph is a straight line of f
1 45Ĉ 1
slope + 1 and intercept f . u
Ex a m p l e 7
A thin rod of length (f/3) is placed along the principal axis of a concave mirror of focal
length f such that its image which is real and elongated, just touches the rod. What is the
magnification?
Solution :
As the image is real and enlarged, the object must be between C and F. Also as the image of one
end coincides with the end itself, i.e., vA = uA,
M
1 1 1 5 f
2
vA vA f
A B
i.e., vA = uA = ă 2f P
C F 5
B´ A´ f
i.e., the end A is at C. f
3 3
2f
M´
OPTICS
QUIZRR 25
Now as the length of the rod is (f/3), its other end B will be at a distance [2f ă (f/3)] = (5/3) f from
P. So if t he dist ance of image of end B fr om P is V B,
1 1 1 5
i.e. vB f
vB (5 / 3) f f 2
5 1
vB vA f 2f f
2 2
(vB v A ) (1 / 2) f 3
and m Ans.
(uB u A ) (1 / 3) f 2
Negative sign implies that image is inverted with respect to object and so is real.
Ex a m p l e 8
If a luminous point is moving at a speed V0 towards a spherical mirror, along its axis, show
that the speed at which the image of this object is moving will be given by
2
f
Vi V0
u f
Solution :
In case of spherical mirrors
1 1 1
... (1)
v u f
1 dv 1 du
or 0 [as f is constant]
v2 dt u2 dt
2
v dv du
or Vi V0 as dt Vi and dt V0
u
1 1 1 uf
But from Eq. (1) i.e. v
v f u (u f )
2
f
So Vi V0 Ans.
u f
OPTICS
26 QUIZRR
Note : From this result it is clear that in case of concave mirror, i.e., with f – f if :
u = ă , v= ăf and Vi = 0
u = ă 2f, v = ă 2f and V i = V0
And u = ă f, v= ă and Vi = ă
i.e., if the object moves with constant speed V0 towards the mirror from to F, the image moves
away from the mirror from F to with speed Vi increasing from 0 to . The two speeds are equal
when both cross each other at C.
Ex a m p l e 9
An object is placed at right angles to the principal axis of a concave mirror at such a
position that the image formed is half the size of object. The object is now shifted to
another position, displaced relative to the earlier position by x = 10 cm, and the size of
image now becomes one fourth of the object. Find initial distance of object from the pole
of mirror and also the focal length of mirror.
Solution :
Initially, let the distance of object from the pole be ÂxÊ.
u = ă x
Since the given mirror is concave and the image is diminished, it is also real and inverted so that
I 1
magnification has a negative sign and m . Further, the description of image implies that the
O 2
object is between C and so that the image is between F and C.
v 1
m
u 2 P
x
u x
v [ u x]
2 2
1 1 1
Applying
v u f
2 1 1
x x f
3 1
... (1)
x f
On shifting the position of object by 10 cm, size of image becomes one fourth of the object while
in the earlier position, it was half. It implies that the object has been shifted by x = 10 cm away from
OPTICS
QUIZRR 27
the pole (As mentioned above, object is between C and so that the image is between F and C. If the
object is shifted away from C towards , the image is shifted between F and C to towards F. Thus
v
u increases and v decreases resulting in a reduced value of m as in the present case). Now,
u
distance of object from pole = x + 10
u = ă (x + 10)
v 1
and m (diminished image in case of concave mirror
u 4
also implies real and inverted)
u ( x 10)
v
4 4
1 1 1
Using
v u f
4 1 1
x 10 x 10 f
5 1
...... (2)
x 10 f
3 5
x x 10
3x + 30 = 5x
or x = 15 cm Ans.
Thus the object, initially, is at a distance 15 cm from the pole.
From Eq. (2),
5 1
x 10 f
5 1
15 10 f
f = ă 5 cm Ans.
OPTICS
28 QUIZRR
Ex a m p l e 1 0
Show that for a virtual object, a convex mirror gives (a) real and erect image for u < f and
(b) virtual and inverted image for u > f.
Solution :
(a) Fig. show rays converging towards the point O and intercepted by a convex mirror. O acts as a
virtual object and, as shown, its distance from pole u < f. Rays are reflected from the mirror and
from real image at I (since they actually meet at I).
Using
1 1 1
v u f
I P O F C
1 1 1 u f
v f u uf
uf
v ... (1)
u f
v
u being positive and v negative, magnification, m will have a positive value.
u
1 1 1
Using
v u f
uf P F I C O
v
u f
v
m = , both v and u being positive, m will be negative. Hence the image is inverted.
u
OPTICS
QUIZRR 29
Ex a m p l e 1 1
A concave mirror of focal length 10 cm and a convex mirror of focal length 15 cm are placed
facing each other 40 cm apart. A point object is placed between the mirrors, on their
common axis and 15 cm from the concave mirror. Find the position and nature of the image
produced by the successive reflections, first at concave mirror and then at convex mirror.
Solution :
According to given problem, for concave mirror,
u = ă 15 cm and f = ă 10 cm
1 1 1 f = +15 f = ă10
So , i.e., v = ă 30 cm M
v 15 10 M 40 cm
P2 I1 O
i.e., concave mirror will form real, inverted and enlarged image P1
I2 15cm 15cm
I1 of object O at a distance 30 cm from it, i.e., at a distance 40 ă 30
M´ M´
= 10 cm from convex mirror.
For convex mirror the image I1 will act as an object and so for it u = ă 10 cm and f = + 15 cm.
1 1 1
... v 10 15 , i.e., v = + 6 cm Ans.
So final image I2 is formed at a distance 6 cm behind the convex mirror and is virtual as shown
in Fig.
30 6
Note : As here m1 = = ă 2 and m2 = = +0.6 so m = m1 ï m2 = (ă2)
15 10
(+0.6) = ă 1.2, i.e., final image is inverted with respect to object, virtual, 1.2 times of the
object size and is at a distance 6 cm behind the convex mirror.
Ex a m p l e 1 2
Two plane mirrors intersect at an angle . An object is placed on the bisector of the angle
between them. Show the location of the closest four images if :-
(a) = 30Ĉ and (b) = 120Ĉ
Solution : 1 S1
S2
(a) a
a
a
30Ĉ
a
S2
1
S1
OPTICS
30 QUIZRR
Here, S1 is the image of source S in the top mirror and is distance a behind the mirror. Now, this
(b)
S
a
a a
60Ĉ
60Ĉ 60Ĉ
S1 60Ĉ In the case, there is only one image located
a
behind each mirror. No other images appear
because the initial images lie in the planes of
S2
the respective mirrors.
Ex a m p l e 1 3
E
An object ABED is placed in front of a concave mirror beyond B
centre of curvature C as shown in figure. State the shape of
P
the image. A D C F
Solution :
Object is placed beyond C. Hence, the image will be real and it will lie between C and F. Further
u, v and f all are negative, hence the mirror formula will become
1 1 1
v u f
1 1 1 u f
or
v f u uf D´ A´
P
C F
f B´
v E´
or f
1 VAB
u
VED
Now uAB > uED
vAB < vED
v
and |mAB| < |mED| as m u
OPTICS
QUIZRR 31
Ex a m p l e 1 4
It is desired to cost the image of a lamp, magnified 5 times, upon a wall 1.2 m distant from
the lamp. What kind of spherical mirror is required and what is its position ?
Solution :
Hence, for a real image v is positive, and u is also positive. Thus the image will be inverted.
v
Now, 5 and we also know that |v ă u| = ă 1.2 {distance between lamp and wall}
u
or 5u ă u = ă 1.2
u = ă 0.3 m, v = ă 1.5 m
The mirror is thus 0.3 m from the lamp and 1.5 m from the wall.
1 1 1
Also, u v f f = ă 0.25 m
Ex a m p l e 1 5
An object is placed 42 cm in front of a concave mirror of focal length 21 cm. Light from the concave
mirror is reflected onto a small plane mirror 21 cm in front of the concave mirror. where is the final
image ?
Solution :
Given : u = ă 42 m, f = ă 21 cm.
O
1 1 1
Now,
u v f 21cm
F
1 1 1 plane
I
v 21 42 mirror
q
v = ă 42 cm
{or we could have directly seen the final image would be at 42 cm from the fact that the object
is at centre of curvature, hence the image will also be a centre of curvature}.
Now, use the mirror formula for the plane mirror, with f = and u = ă 42 ă (ă 21) = ă 21 cm,
since the image produced by the concave mirror alone is the object for the plane mirror.
1 1 1 1 1 1
p q f 21 q
q = 21 cm.
Hence, the position of the final image I is 21 cm in front of the plane mirror.
OPTICS
32 QUIZRR
4. REFRA CT ION OF L IGH T
Whenever a ray of light comes from one transparent medium to another, it gets deviated from its
original path while crossing the interface of the two media (except in case of normal incidence). This
phenomenon of deviation or bending of light rays from their original path while passing from one
medium to another is called refraction.
N
A Medium - I
i B
K L
Medium - II
r
N´ C
AB incident ray BC refracted ray NBN´ Normal
i angle of incidence r angle of refraction KL interface
l If the refracted ray bends towards the normal with respect to the incident ray, then the
second medium is said to be optically denser as compared to the first medium.
Medium - I
i (Rarer)
Medium - II
r (denser)
l If the refracted ray bends away from the normal, then the second medium is said to be
(optically) rarer as compared to the first medium.
Medium - I
i
(denser)
r
Medium - II
(Rarer)
4 .1 L a w s o f Re f r a c t i o n
(i) The incident ray, there fracted ray and the normal to the refracting surface at the point of
incidence, all lie in the same plane.
(ii) The ratio of sine of the angle of incidence to the sine of the angle of refraction is constant
for any two given media.
sin i
i.e. costant 12 Refractive index
sin r
This is called SnellÊs law.
OPTICS
QUIZRR 33
4 .2 Re f r a c t i ve I n d e x (μ o r n )
(A) Absolute Refractive Index
It is defined as the ratio of speed of light in free space c to that in a given medium v,
c
i.e., ø or n ...(1)
v
1 1
c and v
0 0
c
n = R R ... (2)
v 0 0
(3) As in vacuum or free space, speed of light of all wavelengths is maximum and equal to c
so for all wavelengths the refractive index of free space is minium and is
c c
1
v c
(4) For a given light, denser is the medium, less will be the speed of light and so greater will
be the refractive index, e.g.,
as vglass < vwater, øG > øW
i.e., for a given light refractive index depends on nature of medium [i.e., ø (1/v)].
(5) For a given medium (other than free space), the speed of light of different wavelengths is
different, i.e. v and ø = (c/v), ø (1/), i.e., greater the wavelength of light lesser will
be the refractive index, e.g. R > B, so in water or glass øR < øB, i.e., for a given medium
(other than free space) refractive index depends on wavelength of light.
(6) As for light in free space c = f 0 and in a medium v = f
c 0
...(3)
v
i.e., for a given light and medium refractive index is equal to the ratio of wavelength of light
in free space to that in the medium.
OPTICS
34 QUIZRR
(B) Relative Refractive Index
When light passes from one medium to the other, the refractive index of medium 2 relative to 1
is written as 1ø2 and is defined as,
2 c / v2 v1
1 2 ...(4)
1 c / v1 v2
1 v2
(1) 2 1
2 v1
v1 v2
So that 1 2 2 1 1
v2 v1
1
i.e. 1 2 ...(5)
2 1
(2) Usually ÂøÊ is used for relative refractive index and it implies the refractive index of denser
medium relative to rarer one, i.e.
D vR
1
R vD
(3) In lens theory ø is used for the refractive index of material of lens relative to the medium, i.e.,
lens
medium
i
1 ø1
and can be greater than, less than or equal to unity.
2 ø2
(4) As according to SnellÊs law
ø1 sin i = ø2 sin r r
the refractive index of medium 2 relative to 1 will be given by
2 sin i
1 2 ...(6)
1 sin r
4 .3 Pri n c i p l e o f Re v e r s ib i l it y o f L i g h t Ra y s
A ray travelling along the path of the refracted ray is reflected along the path of the incident ray.
In the same way, a refracted ray reversed to travel back along its path will get refracted along
the path of the incident ray. Thus, the incident ray and the refracted ray are mutually reversible. This
is called the reversibility of light.
OPTICS
QUIZRR 35
sin i
1 2 ... (1)
sin r
sin r
2 1 ... (2)
sin i
1
or 1ø2 =
2 1
De v i a t i o n o f r a y d u e t o Re f r a c t i o n
Hence, from the figure we can see that the deviation = |i ă r|
i i
Rarer Denser
Denser Rarer
r
r
= i = i
ă r ă r
4 .4 Cr it ic a l A n g l e
When a ray of light goes from denser medium to a racer medium, the angle of refraction is greater
than the angle of incidence.
From this statement, we can also observe that, in some cases the angle of refraction can increase
to become 90Ĉ or more than 90Ĉ.
The angle of incidence for which the angle refraction becomes 90Ĉ is called the article angle for
that interface.
It is generally denoted by c.
Let øR be the refractive index of the rarer medium and øD be the refractive index of the denser
medium. Obviously, ør < øD.
sin c
Hence, from SnellÊs law r
sin 90 D
1
or sin c =
D
where ø = is the refractive index of the denser medium with respect to the rarer medium.
R
OPTICS
36 QUIZRR
4 .5 T o t a l I n t e r n a l Re f l e c t i o n
While a light ray goes from a denser medium to a rarer medium, the angle of refraction is greater
than the angle of incidence. If the angle of incidence is increased, the angle of refraction may eventually
become 90Ĉ.
r
Rarer
Denser
i
i<c
refraction
If a ray of light travelling in a denser medium strikes a rarer medium at an angle of incidence
i which is greater than the critical angle C, it gets totally reflected back into the same medium. This
phenomenon is called as total internal reflection.
c
rarer rarer
denser denser
i i i
i=c i>c
total internal
reflection
So m e I l lu s t ra t i o n s o f T o t a l I n t e r n a l Re f l e c t i o n
(1) Shining of air bubble
An air bubble in water shines due to total internal reflection at its outer surface. Here light is
propagating from denser medium (water) to rarer medium (air) and if i > C(49Ĉ) TIR will take
place at the surface of the bubble. This is shown in figure (A)
(2) Sparkling of diamond
The sparkling of diamond is due to total internal reflection inside it. As refractive index for
diamond is 2.5 so C = 24Ĉ. Now the cuttings of diamond are such that i > iC, so TIR will take
place again and again inside it. The light which beams out from a few places in some specific
directions makes it sparkle. Recently developed synthetic rutile (titanium dioxide) displays even
greater brilliance as for it ø ~ 2.9 and so C ~ 20Ĉ.
OPTICS
QUIZRR 37
-fibre
ptical
Air
O ø1 ø2
ø1 > ø2
Light pipe
(A) (B)
4 .6 Sin g l e Re f r a c t i o n f r o m a Pla n e Su r f a c e
Following four results can be drawn after refraction from a plane surface.
D 1 2
C
i
Air Air A
Medium A B Medium
i r x
I øx
r
O I
(a) (b)
O O
øx
x I x
x
Air ø Air
A Medium A Medium
1 2
(c) (d)
In all the four figures, single refraction is taking place through a plane surface. Refractive index
of medium (may be glass, water etc.) is ø. In figures (a) and (d) the ray of light is travelling from denser
OPTICS
38 QUIZRR
to rare medium and hence, it bends away from the normal. In figures (b) and (c) the ray of light is
travelling from a rare to a denser medium and hence, it bends towards the normal. Now, let us take
the four figures individually.
(a) Object O is placed at a distance x from A. Ray OA, which falls normally on the plane surface,
passes undeviated as AB. Ray OB, which falls at angle r (with the normal) on the plane
surface, bends away from the normal and passes as BC in air. Rays AD and BC meet at I
after extending these two rays backwards. Let BC makes an angle i (> r) with normal.
AB
sin i tan i ... (i)
AI
AB
and sin r tan r ... (ii)
AO
sin i AO
Dividing Eq. (i) by (ii), we have
sin r AI
AO sin i
or as
AI sin r
AO x
AI
If point O is at a depth of d from a water surface, then the above result is also sometimes
written as,
dactual
dapparent
AO x
Where AI .
Note : In all the four cases the change in the value of x is ø times whether it is increasing or
decreasing. All the relations can be derived for small angles of incidence as done in part (a).
OPTICS
QUIZRR 39
4 .6 Re f r a c t i o n T h r o u g h A Gla s s Sla b i
AIR
When a light ray passes through a glass slab having parallel
faces, it gets refracted twice before finally emerging out of it. Glass
r
First refraction takes place from air to glass. So,
ø
sin i
...(i)
sin r r
sin i sin e
i = e
sin r sin r
Illust rat io n
In the adjacent system what is the angle between incident ray and emergent ray ?
Solution :
Applying SnellÊs law at first interface i
air 1
sin i = sin r1 ..................... (i)
r1
Applying SnellÊs law at 2, 3 & 4 r1 ø1
2
We get r2
r2 ø2
ø1 sin r1 = ø2 sin r2 ........... (ii)
3
ø2 sin r2 = ø3 sin r3 ........... (iii) r3 r3 ø3
ø3 sin r3 = sin e ................... (iv)
4
Combining all the form equation we get
e
sin i = ø1 sin r1 = ø2 sin r2 = ø3 sin r3 = sin e
or sin i = sin e
i = e
emergent ray is parallel to incident ray. This result is independent of no. of slabs used and
holds as long as incident ray and emergent ray are in same medium.
OPTICS
40 QUIZRR
4 .7 Sh i f t d u e t o Gla s s Sla b
4 .7 .1 L a t e r a l s h i f t
The perpendicular distance between the incident ray and the emergent ray and the light is
incident obliquely on a parallel sided refracting glass slab is called lateral shift ?
In right-angled triangle OBK, we have
A i
BOK = i ă r AIR
d O Glass
sin (i ă r) = r
OB
or d = OB sin (i ă r) ... (i) t
In right angled triangle ON´B, we have K
r
ON' t
cos r or OB ... (ii)
OB cos r AIR B
From (i) and (ii) we get
C
t
d sin i r
cos r
4 .7 .2 N o r m a l Sh i f t
Here again two cases are possible.
C E M C E
N
M N P
I1 O I A B A B O I
ø ø
D F D F
t t
(a) (b)
(a) An object is placed at O. Plane surface CD forms its image (virtual) at I1. This image acts
as object for EF which finally forms the image (virtual) at I. Distance OI is called the normal
shift and its value is,
1
OI I t
OPTICS
QUIZRR 41
BI1 t
BI x (Refraction from EF)
OI = (AB + OA) ă BI
t
= (t + x) ă x
1
= 1 t Hence Proved.
(b) The ray of light which would had met line AB at O will now meet this line at I after two
1
times refraction from the slab. Here OI 1 t
So m e I l l u s t r a t io n s o f Re f r a c t i o n
(1) Bending of an object
When a point object in a denser medium is seen from a rarer medium it appears to be at a depth
(d/ø).So if a linear object is dipped inclined to the surface of a liquid, (say water) actual depth
will be different for its different points and so apparent depth. Due to this the object appears to
be inclined from its actual position BE as shown in Fig. (A).
(2) Visibility of two images of an object
When an object is in a glass container and is seen from a level higher than that of liquid in the
container as shown in Fig. (B), two images I1 and I2 of object O can be seen simultaneously·
one due to refraction at the upper surface while the other at the side surface.
A
C´
B I2
E´
C
I1
E O
(A) (B)
(3) Twinkling of stars
Stars are self-luminous distant objects, so only a few rays of light reach the eye through the
atmosphere. However, due to fluctuations in refractive index of atmosphere the refraction becomes
irregular and the light sometimes reaches the eye and sometimes it does not. This gives rise to
twinkling of stars. If from moon or free space we look at a star this effect will not take place and
star light will reach the eye continuously.
OPTICS
42 QUIZRR
Ex a m p l e 1 6
Monochromatic light falls at an angle of incidence ÂiÊ on a slab of a transparent material,
refractive index of this material being ÂøÊ for the given light. What should the relation between
i and ø be so that the reflected and the refracted rays are mutually perpendicular ?
Solution :
In Fig., r is the angle of reflection and r´ the angle of refraction.
According to the given condition, considering the reflected and the refracted rays to be perpendicular
to each other,
r + 90Ĉ + r´ = 180Ĉ
r´ = 90Ĉ
or r´ = 90Ĉ ă i [i = r, law of reflection]
According to SnellÊs law,
1 sin i = ø sin r´ i r
sin i = ø sin (90Ĉ ă i)
90Ĉ
sin i = ø cos i r´
ø = tan i
i = tană1 (ø) Ans.
Ex a m p l e 1 7
In Fig. a ray of light falls on a glass plate of thickness 6 cm and
refractive index 1.5 such that it deviates through 30Ĉ due to refraction P Q
at the boundary PQ. Find the lateral displacement of the ray as it
emerges out of the plate. ø = 1.5
S R
Solution :
In Fig. (B)
i = r + 30Ĉ (opposite angles)
r = i ă 30Ĉ
Applying SnellÊs law at the boundary PQ.
1 sin i = ø sin r
sin i = 1.5 sin (i ă 30Ĉ) i
sin i = 1.5 (sin i cos 30Ĉ) P Q
OPTICS
QUIZRR 43
1.5 3 1.5
sin i sin i cos i
2 2
cos i
Lateral displacement y = t sin i 1 2 2
sin i
cos68.2
y = 6 sin 68.2Ĉ 1
(1.5) sin 68.2
2 2
y = 3.83 cm Ans.
Ex a m p l e 1 8
4
In a tank, a 4 cm thick layer of water floats on a 6 cm thick layer of an organic
3
liquid (ø = 1.48). Viewing at a normal incidence, how far below the water surface does the
bottom of tank appear to be ?
Solution :
d1 d2
dAP
1 2
6 4
=
1.48 4 / 3
DAP = 7.05 cm Ans.
Ex a m p l e 1 9
B C
An air bubble is trapped inside a glass cube of edge 30 cm. Looking
through the face ABEH, the bubble appears to be at normal distance A
D
12 cm from this face and when seen from the opposite face CDGE,
F
it appears to be at normal distance 8 cm from CDGF. Find refractive E
index of glass and also the actual position of the bubble. H G
OPTICS
44 QUIZRR
Solution : In Fig. below :
AH represents face ABEH
DG represents face CDGF
AD represents face ABCD
and HG represents face EFGH
All observations are made from outside the cube. Let O be the actual position of bubble at distance
x from AH. Obviously, its real distance from DG is 30 ă x. I1 is the apparent position of bubble as seen
from the side of AH and I2 its apparent position seen from the side of DG.
A D
I1 I2
L O M
x 30ăx
H G
here ø is refractive index of denser medium (glass) relative to the surrounding rarer maximum (air).
For refraction at AH
LO MO
ø = ø = MI
LI1 2
LO x MO 30 x
LI1 = MI1 =
x 30 x
= 12 ... (1) = 8 ... (2)
x 30 x
= 20
ø = 1.5 Ans.
OPTICS
QUIZRR 45
x
= 12
x
or = 12
1.5
x = 18 cm Ans.
Thus actual position of bubble is at normal distance 18 cm from AH (face ABEH) or at a distance
12 cm from DG (face CDGF).
Ex a m p l e 2 0
A container completely filled with water (ø = 4/3) has a scratch at its bottom. An observer
looks at the scratch from a height 1 m above the surface of water. A small hole is now made
in a wall of the container and very close to its bottom so that water flows out. When the
level of water reduces to half, the hole is closed. It appears to the observer that the scratch
has moved away by a distance 25 cm relative to its initial position. Find the initial level
(height) of water in the container.
Solution :
Initially, let water be filled to a height ÂhÊ m. Actual depth of scratch from the surface is ÂhÊ.
h
However, due to refraction at water-air boundary, it will appear to be so that the observer finds the
h
scratch to be at a distance 1 . When the container is half filled, as shown in Fig. actual depth of
h
scratch below the surface of water is h/2 but, due to refraction, it appears to be to the observer
2
h h
located in air. In this case, the scratch appears to the observer to be at distance 1 .
2 2
Observer Observer
1m 1m
h/ø h/2
h
I h/2ø
h/2
I´
S S
(A) (B)
OPTICS
46 QUIZRR
According to the given condition,
h h h
1 = 1 0.25 [25 cm = 0.25 m]
2 2
h h
= 0.25
2 2
since ø = 4/3
h 3h
= 0.25
2 8
h =2m Ans.
Ex a m p l e 2 1 Observer 1
Observer 2
A point source of light is placed at the bottom of a tank filled
4
with water to a depth 4 m. Find apparent depth of the 4m
3
30Ĉ
source below the surface as viewed by (a) observer 1 and (b)
S
observer 2.
Solution :
(a) Observer 1 receives light rays from the source which fall on the water air boundary at
normal incidence (i 0) and we can use the relation
Actual depth
ø =
Apparent depth
Actual depth 4
Apparent depth = 3m Ans.
4/3
(b) The relation used in (a) is not valid for observer 2 since incidence (i = 30Ĉ) is much away
from the normal.
In this situation, apparent depth can be expressed as,
dAC cos
dAP
2 sin2
Here is the angle of vision at the surface, i.e. the angle of refraction as light passes from
water to air.
Applying SnellÊs law,
OPTICS
QUIZRR 47
4 1 2
sin =
3 2 3
cos = 1 4/ 9 5/9
3
4 4/3 5/9
d AP = 4/3
4 / 3 2 2 / 32
Ex a m p l e 2 2
A fish rising vertically to the surface of water in a lake uniformly at the rate of 3 m/s observes
a king-fisher (bird) diving vertically towards the water at a rate of 9 m/s vertically above it.
If the refractive index of water is (4/3), find the actual velocity of the dive of the bird.
Solution :
If at any instant, the fish is at a depth x below the water surface while the bird is at a height
y above the surface, then the apparent height of the bird above the surface as seen by fish will be
dAC 1 y 1
, i.e., or dAP = øy
dAP 2 dAP
B´
So the total total apparent distance of the bird as seen by fish in water at depth
B
x will be øy
y h
h = x + øy
dh dx dy dy x x
or , i.e., 9 3 F
dt dt dt dt
dy 6
4.5 m/s
dt 4 / 3 Ans.
Ex a m p l e 2 3
A vessel having perfectly reflecting plane bottom is filled with water (ø = 4/3) to a depth d.
A point source of light is placed at a height h above the surface of water. Find the distance
of final image from water surface.
Solution :
As shown in Fig., water will form the image of bottom, i.e., mirror MM´ at a depth (d/ø) from its
surface [as d AC/d AP) = (ø1/ø2)]. So the distance of object O from virtual mirror mm´ will be h + (d/ø).
OPTICS
48 QUIZRR
Now as a plane mirror forms image behind the mirror at the same distance O
h h
as the object is in front of it, the distance of image I from mm´ will be A C
h + (d/ø) and as the distance of virtual mirror from the surface of water d
d ø
is (d/ø), the distance of image I from the surface of water will be
m m´
M M´
d d 2d 3 4 (h + d )
ø
h h h 2 d as 3 Ans.
Ex a m p l e 2 4
A concave mirror of radius of curvature one metre is placed at the bottom of a tank of
water. The mirror forms an image of the sun when it is directly overhead. Calculate the
distance of the images from the mirror for 80 cm of water in the tank.
Solution :
The focal length of the mirror is f = (R/2) = (100/2) = 50 cm and the sun is at infinity, i.e.
ø = ; so from mirror formula we have
1 1 1
i.e., v = ă 50 cm
v 50
So when the length of water column in the tank is 80 cm, the image of F
sun will be 50 cm from the mirror as shown in Fig.
Ex a m p l e 2 5
A rectangular block of glass is placed on a printed page lying on a horizontal surface. Find
the minimum value of the refractive index of glass for which the letters on the page are not
visible from any of the vertical faces of the block.
Solution :
The situation is depicted in Fig. Light will not emerge out from the vertical face BC if at it
i > C
or sin i > sin C
1 D C
or sin i ... (1)
R P
1
i
as sin C r
O
A Paper B
OPTICS
QUIZRR 49
sin 2 1
1 , i.e., 2 1 sin 2
2
ø2 > 2, i.e., 2
so ømin = 2
Alternative Solution : According to theory of prism, light does not emerge from a prism if angle
of prism A > 2C, whatever be the angle of incidence. Now as the angle of prism is the angle between
the faces on which light is incident and from which it emerges, so here A = 90Ĉ. Hence light will not
emerge from vertical faces for all values of , if 90Ĉ > 2C, i.e., 45Ĉ > C
1 1
or sin 45Ĉ > sin C, i.e.
2
2, i.e., min 2
Ex a m p l e 2 6
A bulb is placed at a depth of 2 7 m in water and a floating opaque disc is placed over the
bulb so that the bulb is not visible from the surface. What is the minimum diameter of the
disc ?
Solution : R
D A
As shown in Fig., light from bulb will not emerge out of water C
if at the edge of disc, i > C Water
h i > c
or sin i > sin ..... (1) ø
C
Now if R is the radius of disc and h is the depth of bulb from it.
B
OPTICS
50 QUIZRR
R 1
sin i
2 2 and sin C
R h
R 1 h
R
R 2 h2 or 2 1
... (2)
2 7
So R min 6m
16 / 9 1
So diameter of disc = 2R = 2 ï 6 = 12 m Ans.
Note : Here it is worthy to note that :
(i) If there had been no opaque disc, only that light will emerge out of the water which was
initially confined in the cone of angle of vertex = 2C.
(ii) Further in accordance with principle of reversibility, a fish or diver inside the water will see
the whole outer would confined in a circular patch whose angular size at the eyeÊs of
2
observer is 2C = 2 sină1 (1/ø) while linear size 2 R = 2h / 1 .
5. PRI SM -T H EORY
5 .1 Pr i s m
Prism is a transparent medium bounded by any number of
A A
A
A
(A) (B) (C) (D)
surfaces in such a way that the surface on which light is incident and the surface from which
light emerges are plane and non-parallel.
Principal
section of A A
A
a prism 45Ĉ
C 90Ĉ 45Ĉ 90Ĉ
B C B C B
Equilateral Right-angled Right-angle
prism isosceles prism prism
(A) (B) (C)
OPTICS
QUIZRR 51
A A= 0 A
i1 = r2
t i1 =r i 2 i2
den Em
c i i i2 erg i1 1
In ray 1 ray ent i1 r1 r2
r1 r2
ø2 = ø1
B C = 0 B C
(A) (B) (C)
(3) If the faces of a prism on which light is incident and from which it emerges are parallel, then
the angle of prism will be zero and as incident ray will emerge parallel to itself, deviation
will also be zero, i.e. the prism will act as a transparent plate.
(4) If ø of the material of the prism is equal to that of surroundings, no refraction at its faces
will take place and light will pass through it undeviated, i.e. = 0 [Fig. 3 (C)].
5 .2 Co n d i t i o n o f N o e m e r g e n c e
The light will not emerge out of a prism for all values of angle of incidence if at face AB for i1
= max = 90Ĉ, at face AC
r 2 > C ... (1)
Now from SnellÊs law at face AB, we have
1 ï sin 90Ĉ = ø sin r1
i.e. r1 = sină1 (1/ø) A
or r 1 = C ...(2)
So from Eqs. (1) and (2) i1 P r1 r2 R
r1 + r2 > 2C ... (3)
However, in a prism Q
r1 + r2 = A* ... (4) B C
So from Eqs. (3) and (4)
A > 2C ... (5)
or sin(A/2) > sinC
i.e. ø > [cosec (A/2)] ... (6)
i.e., a ray of light will not emerge out of a prism (whatever be the angle of incidence) if A > 2C,
i.e. if ø > cosec (A/2).
OPTICS
52 QUIZRR
5 .3 Co n d i t i o n o f Gra zi n g Em e r g e n c e
If a ray can emerge out of a prism, the value of angle of incidence i1 for which angle of emergence
i2 = 90Ĉ is called condition of grazing emergence. In this situations as the ray emerges out of face AC,
i.e. TIR does not take place at it,
r < ...(1) A
2 C
But as in a prism
i1 P
r2 R
r1 + r2 = A r1 i2
So r1 = A ă (< C) Q
i.e., r 1 > A ă C ...(2) B C
Now from snellÊs law at face AB, we have
1 sin i1 = ø sin r1
which in the light of Eq. (2) gives
sin i1 > ø sin (A ă C)
i.e., sin i1 > ø [sin A cos C ă cos A sin C]
or i1 sin 1
2
1 sin A cos A
or i1 min sin 1
2
1 sin A cos A
...(3)
i.e., light will emerge out of a prism only if angle of incidence is greater than (i1)min given by Eq. (3).
In this situation deviation will be given by = (i1 + 90Ĉ ă A) with i1 given by Eq. (3).
5 .4 De v i a t i o n Pro d u c e d b y Pri s m
A
If light is incident on face AB of a prism of prism angle A, at an angle
of incidence i1 and emerges from face AC at an angle of emergence i2 as R
P i2
shown in Fig., the angle of deviation will be i1
r
r1 2
= (i1 ă r1) + (i2 ă r2) Q
i.e. = (i1 ă r2) ă (i1 ă r2) ...(1) B C
Now as in PQR
r1 + r2 + = 180Ĉ
OPTICS
QUIZRR 53
A A
Q
i2 P R
P i1 i2
i1 = 90Ĉ r
r1 2 r1 r2
60Ĉ Q
60Ĉ
B C B C
(A) (B)
Maximum-deviation Minimum - deviation
and as in a prism
(r1 + r2) = A, r2 = (A ă C)
OPTICS
54 QUIZRR
So at surface AC,
ø sin r2 = 1 sin i2
i.e., sin i2 = ø sin (A ă C)
or i 2 = sină1 [ø sin (A ă C) ... (7)
So Eq. (6) gives maximum deviation while (7) angle of emergence in condition of maximum
deviation i.e., i1 = 90Ĉ. This situation is reverse of grazing emergence, i.e., case (C), and may also
be viewed as deviation at grazing incidence.
(c) Minimum deviation
= (i1 + i2) ă A
Theory and experiment show that will be minimum when
i1 + i2 = i (say) ... (8)
But when i1 + i2 = i, SnellÊs law at faces AB and AC respectively gives
1 sin i = sin r1 and ø sin r2 = 1 sin i
i.e., sin r1 = sin r2 or r1 = r2 (say) ...(9)
So in the light of Eqs. (8) and (9), (3) reduces to
min = (2i ă A) with r = (A/2) ...(10)
In this situation angle of emergence (or incidence) will be obtained by applying SnellÊs law, i.e.,
ø sin r = sin i
or i = sină1 [ø sin (A/2)] [as r = (A/2)] ...(11)
Here it is worth noting that in the situation of minimum deviation :
(1) Angle of incidence is equal to angle of emergence and is given by
i = sină1 [ø sin (A/2)]
(2) Angle of refraction inside the prism is equal to half the angle of prism, i.e.,
r = (A/2) = (A/2)
(3) As r1 = r2, (90Ĉ ă r1)= (90Ĉ ă r2), so that AP = AR, and if the prism is equilateral [or
isosceles with AB = AC], the ray inside the prism will be parallel to its base.
(4) As from Eq. (10), i= (m + A/2) with r = (A/2)
I m p o r t a n t Po i n t s
The deviation produced by a prism depends on angle of incidence, i1, angle of prism A, ø of the
material of the prism and wavelength of light . So
(1) For a given A, material of prism and light, as angle of incidence increases, angle of deviation
first decreases, reaches a minimum and then increases, as shown in Fig. (A).
OPTICS
QUIZRR 55
ø, and
A = constt. A
i - curve I
v
ht
m lig
ite
Wh R
i1 = i2 i B V
C
(A) (B)
(2) For a given material of prism, light and angle of incidence, as angle of prism increases, angle
of deviation also increases,
i.e. A
e.g., deviation in 60Ĉ prism will be more than in 30Ĉ prism.
(3) For a given angle of incidence, angle of prism and light, as material of prism is changed,
deviation will also change. Greater the ø or the prism, greater will be the deviation as,
(ø ă 1)
e.g., ø of flint glass is more than that of crown glass, so F > C, for same i1, A and .
(4) For a given angle of incidence, angle of prism and material of prism, if wavelength of light
is changed, deviation will also change as ø of a prism is different for different wavelengths.
With increases in wavelength deviation decreases, i.e., deviation for red is least while maximum
for violet [Fig. (B)]. (This is discussed in detail in the next section).
5 .5 Di s p e r s i o n o f L i g h t
When white light passes through a prism it splits up into constituent colours. This phenomenon
is called dispersion and arises due to the fact that refractive index of a prism is different for different
wavelengths. So different wavelengths in passing through a prism are deviated through different
angles and as (ø ă 1), violet is deviated most while red least giving rise to display of colours known
as spectrum [Fig. (B)]. Rainbow, the most colourful phenomenon in nature, is primarily due to the
dispersion of sunlight by rain-drops suspended in air.
In case of dispersion of light the angle between the extreme rays is a measure of dispersion and
is called angular dispersion. So
Angular dispersion = (Vă R) ...(13)
and the ratio of dispersion to mean deviation is called dispersive power i.e.,
V R
= ...(14)
V R d V R
= Y ...(15)
1 1 2
OPTICS
56 QUIZRR
From expressions (14) and (15) it is clear that :
(1) Dispersive power like refractive index has no units and dimensions and depends on the
material of the prism.
(2) As for a given prism dispersive power is constant,
i.e., dispersion of different wavelengths will be different and will be maximum for violet and
minimum for red
V B G Y OR VB G Y O R
V R
= 0Ĉ, i.e., A ' ' ' A ...(16)
V R
i.e., the two prisms must be placed with their prism angles in opposite directions so that condition
given by Eq. (16) may be satisfied. In this situation
V R
A
= (ø ă 1) A ă (ø´ ă 1)
'V 'R
V R ' 1
i.e. = (ø ă 1) A 1
1 'V 'R
OPTICS
QUIZRR 57
V R A
But as 1 = (ø ă 1) A, =
1
' V 'R ,
and 1 = = 1[1 ă (/´)] ...(17)
' 1
usually ´>, so is in the same direction as produced by the first prism [Fig. (A)].
A
A Flint V
ht
Flint
te lig
Whi R R
ht 1 ´
lig
hit
e ´ CrownV
W Crown A´
A´ Dispersion without deviation
Deviation without dispersion (B)
(A)
The prism which produces deviation without dispersion is called Âachromatic prismÊ.
(b) For dispersion without deviation
1 A
= 0, i.e. A´ = ...(18)
' 1
In this situation,
1 A
= (øV ă øR) A ă (ø´V ă ø´R) ï
' 1
V R ' V 'R
i.e. = (ø ă 1)A 1
' 1
or = [ ă ´] ...(19)
If ´ > , the resultant dispersion is negative, i.e., opposite to that produced by the first prism.
The prism which produces dispersion without deviation is called Âdirect vision prismÊ.
Ex a m p l e 2 7
Find the minimum and maximum angle of deviation for a prism with angle A = 60Ĉ and
ø = 1.5.
Solution : Minimum deviation :
The angle of minimum deviation occurs when i = e and r1 = r2 and is given by
A m
sin
ø = 2 = 2 sină1 (ø sin A/2) ă A
A m
sin / 2
OPTICS
58 QUIZRR
Substituting ø = 1.5 and A = 60Ĉ, we get
m = 2 sină1 (0.75) ă 60Ĉ = 37Ĉ
Maximum Deviation : The deviation is maximum when i = 90Ĉ or e = 90Ĉ that is at grazing
incidence or grazing emergence.
Let i = 90Ĉ
1 1
r1 = C sin
r sin 1 / 42
2
3
sin r2 1
Using sin e , we have
Ex a m p l e 2 8
Calculate the dispersive power of crown and flint glass-prism from the following data :
For crown glass øV = 1.522; øR = 1.514 an d f or f l i n t gl ass μV = 1.522; øR = 1.514
Solution :
For crown glass øV = 1.522; øR = 1.514
V R 1.522 1.514
øy =
2 2
øy = 1.518
Hence, the dispersive power of crown glass
V R 1.522 1.514
y 1.518 1
W = 0.01544
For flint glass øV = 1.662; øR = 1.644
V R 1.662 1.644
ø= = 1.6
2 2
OPTICS
QUIZRR 59
Ex a m p l e 2 9
Find the angle of a prism of dispersive power 0.021 and refractive index 1.53 to form on
achromatic combination with the prism of angle 4.2Ĉ and dispersive power 0.045 having
refractive index 1.65. Also calculate the resultant deviation.
Solution :
W = 0.021; ø = 1.53; W´ = 0.045; ø´ = 1.65
A´ = 4.2Ĉ
For no dispersion
W + W´´ = 0
or W (ø ă 1)A + W (ø´ ă 1)A´ = 0
6 . Re f r a c t io n f r o m Cu r ve d Su r f a c e s
If the boundary between two transparent media is not plane and an object O in medium of
refractive index ø1 forms an image I in medium of refractive index ø2 as shown in Fig., then from
SnellÊs law at the boundary AB
DA
ø1 sin i = ø2 sin r i r
ø1 ø2
y
and as for small angles,
O P R C I
sin = tan = u v
the above equation reduces to B
ø1 ï i = ø2 ï r ...(1)
But in OCD i = ( + )
while in ICD, = (r + )
So substituting the values of i and r from these in Eq. (1), we get
ø1 ( + ) = ø2( ă )
or ø1 + ø2 = (ø2ă ø1)
y y y
1 2 2 1
or
v =
R
2 1
2 1
or = ...(2)
v u R
OPTICS
60 QUIZRR
This is the desired result. If we compare it with mirror formula,
1 1 1
v u f
m
I
v / 2 1 . v
...(3)
O u / 1 2 u
While using Eqs. (2) and (3) keep in mind that :
(1) These are valid for all single refracting surfaces-convex, concave or plane. In case of plane
refracting surface
R , so Eq. (2) reduces to
2 1 u 1
0 i.e.,
v u v 2
dAC 1
or
dAP 2
(2) The rules for signs for a single reflecting surface are same as for spherical mirrors.
(3) If object or image itself is present at a refracting surface, refraction at that surface is not
considered.
Ex a m p l e 3 0
An object O is stuck on the surface of a transparent solid sphere of
radius 20 cm. Find refraction index of the sphere such that rays from
the object after refraction from the opposite side emerge as a parallel O
beam, as shown. Also prove that refractive index has the same value,
in the given situation, for any value of radius of sphere.
Solution :
For refraction at a spherical surface
2 1 2 1
v u R
u is the distance of object from the pole of refracting surface; it may be inferred as the normal
distance of object from the refracting surface.
OPTICS
QUIZRR 61
v is the distance of image from the pole of refracting surface; it may be inferred as the normal
distance of image from the refracting surface.
ø1 is the refractive index of the medium in which the object is located so that light is refracted
while passing from medium of refractive index ø1 to medium of refractive index ø2.
R is the radius of curvature of the refracting surface.
In this problem,
ø 2 = 1 (air)
u = ă 40 cm (object is at distance 2R from
the refracting surface)
and v =
1 1 1 1
40 =
20
ø1 = 2
Thus refractive index of the sphere is 2.
In general, for any radius of the sphere, say R.
1 1 1
2R = R
1 1 1
=
2R R
or ø1 = 2 Ans.
Therefore, in the given situation, refraction index of sphere will be 2 for any value of radius.
Ex a m p l e 3 1
A mark on the surface of a glass sphere (ø = 1.5) is viewed from a diametrically opposite
position. It appears to be at a distance 10 cm from its actual position. Find the radius of
sphere.
Solution :
As the mark is viewed from the diametrically opposite position, refraction takes place at side II
of the surface (the mark being on side I as shown).
10 cm I II
I C P
Mark
OPTICS
62 QUIZRR
Here ø 1 = 1.5, ø2 = 1
u = ă 2R
2 1 2 1
Using =
v u R
1
1.5 1 1.5
v 2R = R
or v = ă 4R
negative sign indicates that the image is formed to the left of refracting surface as shown in Fig.
Further, it is given that the image of mark is at a distance 10 cm from the object.
Hence 4R = 2R + 10
R = 5 cm Ans.
Ex a m p l e 3 2
The slab of a material of refractive index 2 has a curved surface A C
APB of radius of curvature 10 cm and a plane surface CD. On ø = 2 ø = 4/3
ø=1
the left of APB is air and on the right of CD is water with 15cm O
P I
refractive indices as given in the figure. An object O is placed Left Right
B D
at a distance of 15 cm from the pole P as shown. What is the 20 cm
30 cm
distance of final image of O from P as viewed from the left ?
Solution :
In case of refraction from a curved surface, we have
2 1
2 1
=
v u R
Here ø 1 = 2; ø2 = 1
R = ă 10 cm and u = ă 15 cm
1 2 12
v = ă 30 cm
So v 15 10 , i.e.,
i.e., the curved surface will form virtual image I at a distance of 30 cm from P. Since, the image
is virtual there will be no refraction at the plane surface CD (as the rays are not actually passing
through the boundary), the distance of final image I from P will remain 30 cm.
OPTICS
QUIZRR 63
Ex a m p l e 3 3
A
An air bubble in glass (ø = 1.5) is situated at a distance 3 cm from a
ø = 1.5 ø=1
convex surface of diameter 10 cm as shown in Fig. 3.6. At what
distance from the surface will the bubble appear ? CO I P
Solution :
3cm
In case of refraction from curved surface
5cm
2 1 2 1
v u R
1 1.5 1 1.5
,
So
v 3 5 i.e., v = ă 2.5 cm Ans.
i.e., the bubble will appear at a distance 2.5 cm from the curved surface inside the glass.
1 1.5 1 1.5
, i.e. v = ă 2 cm.
v 3
Ex a m p l e 3 4
A glass sphere (ø = 1.5) of radius 20 cm has a small air bubble 4 cm below its centre. The
sphere is viewed from outside and along a vertical line through the bubble. Find the
apparent depth of the bubble below the surface of sphere.
Solution : Here u = ă 24 cm
ø 1 = 1.5 P
ø2 = 1
20 cm
2 1 2 1
Using =
v u R C
4 cm
O
1 1.5 1 1.5
=
v 24 20
1 1 15
=
v 40 24
v = ă 26.67cm I
negative sign have indicates that the direction from P to I is opposite to the direction of incident
light as shown. Hence, the bubble appears to be at a depth 26.67 cm from the surface of sphere.
OPTICS
64 QUIZRR
Ex a m p l e 3 5
A quarter cylinder of radius R and refractive index 1.5 is placed on
a table. A point object P is kept at a distance of mR from it. Find
the value of m for which a ray from P will emerge parallel to the P
table as shown in the figure. mR R
Solution :
For refraction at plane surface,
2 1 2 1
=
v u R
Here u = ă mR,
P1
ø 2 = 1.5, P
mR R
ø 1 = 1, R = 1.5mR
v = 1.5 mR
For curved surface, P1 acts as an object. For this surface,
given ø1 = 1.5, ø2 = 1.0, u = ă (1.5 mR + R), v =
2 1 2 1
... =
v u R
1 1.5 1 1.5
1.5m 1 R = R
1.5 1
1.5m 1
= or 3 = 1.5 m + 1
2
4
m = Ans.
3
Ex a m p l e 3 6
A glass sphere (ø = 1.5) of diameter 50 cm has a small air bubble. Looking from outside
along a diameter, the bubble appears to be at distance 10 cm from the surface. Find the
apparent position of the bubble when it is viewed from the diametrically opposite position.
Also find the actual position of the bubble.
Solution :
Looking from outside, say from the left, and along the diameter P1P2, the bubble which is actually
at O, appears to be at I1 such that P1I1 = 10 cm.
2 1 2 1
Using
v u R
OPTICS
QUIZRR 65
I
II
I2 P1 I1 O C P2
1 1.5 1 1.5
=
10 u 25
1.5 1 1
or =
u 10 50
u = ă 12.5 cm
Hence the bubble is actually at a distance 12.5 cm from P1.
For refraction at side II of the surface
Looking from outside, this time from the right, let us apply the relation for refraction at side II.
Distance of bubble (O) from P2 = 50 ă 12.5 = 37.5 cm
ø 1 = ă 37.5 cm
R = ă 25 cm
ø 1 = 1.5
ø2 = 1
v = ă 50 cm Ans.
Thus the image will be formed 50 cm left of P2 which implies that it forms at P1 (diameter =
50 cm, given). Hence I2 in the figure will coincide with P1.
7 . T H I N L EN SES
A lens is one of the most familiar optical devices for a human being. A lens is an optical system
with two refracting surfaces. The simplest lens has two spherical surfaces close enough together that
we can neglect the distance between them (the thickness of the lens). We call this a thin lens.
OPTICS
66 QUIZRR
As there are two spherical surfaces, there are two centres of curvature C1 and C2 and
correspondingly two radii of curvature R1 and R2.
The line joining C1 and C2 is called the principal axis of the lens. The centre P of the thin lens
which lies on the principal axis, is called the optical centre.
Incident light
R1 > 0
R2 < 0
C2 R2 P C1
R1
R1
Principal Focus : A lens has two surfaces and hence two focal points. First focal point is an
object point on the principal axis for which image is formed at infinity
OPTICS
QUIZRR 67
F1 F1
(A) (B)
while second focal point is an image point on the principal axis for which object lies at infinity.
F2 F2
O O
f f
(A) (B)
Focal length (f) : f is defined as the distance between optical centre of a lens and the point where
the parallel beam of light converges or appears to converge.
Aperture : In reference to a lens, aperture means the effective diameter of its transmitting area.
So the brightness, i.e., intensity of image formed by a lens which depends on the light passing through
the lens will depend on the square of aperture, i.e.
I (Aperture)2
7 .1 Sig n Co n v e n t i o n
It is same as explained in case of mirror, i.e.,
(1) Whenever and where ever possible, rays of light are taken to travel from left to right.
R1 O R2 R1 R2
C2 F C1 C1F O C2
OPTICS
68 QUIZRR
(i) To calculate an unknown quantity the known quantities are substituted with sign in a given
formula.
(ii) In the result sign must be interpreted as there are number of sign conventions and same
sign has different meaning in different conventions.
7 .2 Ru l e s f o r I m a g e Fo r m a t i o n
In order to locate the image formed by a lens graphically following rules are adopted :
(1) A ray passing through optical centre proceeds undeviated through the lens.
(2) A ray passing through first focus or directed towards it, after refraction from the lens,
becomes parallel to the principal axis.
(3) A ray passing parallel to the principal axis after refraction through the lens passes or
appears to pass through F2.
Only two rays from the same point of an object are needed for image formation and the point
where the rays after refraction through the lens intersect or appear to intersect is the image of the
object. If they actually intersect each other the images is real and if they are near to intersect the image
is said to be virtual.
7 .3 I m a g e Fo r m a t i o n b y L e n s
The image formed by a lens depends on the position of object and nature of lens. This all in a
tabular form is given below :
(a) For Convergent or Convex Lens
S.No. Position of Object Ray-Diagram Details of Image
1. At infinity Real, inverted
F2 Diminished (m << 1)
F1 At F
3. At 2 F Real, inverted
I
Equal (m = ă 1)
O
At 2F
OPTICS
QUIZRR 69
5. At F Read. inverted
O F 2F Enlarged, (m >> ă 1)
2F F At infinity
F Enlarged (m > + 1)
I 2F FO Between and object
on same side
7 .4 Fo r m u l a e
In case of image formation by a lens, the incident ray is refracted twice, at first and second surface
respectively. The image formed by the first surface acts as object for the second. So from the formula
for refraction at curved surface. ø2
ø1 ø1
2 1
2 1
=
v u R
O R1 R2
u v
For first surface
L M
L M
= ...(1)
v1 u R1
M L M L
...(2)
v v1 R2
OPTICS
70 QUIZRR
So adding Eqs. (1) and (2), we have
1 1 1 1
M = L M
v u R1 R 2
1 1 1 1
or = 1
v u R1 R 2
L
with ø = ...(3)
M
Now if object is at infinity, image will be formed at the focus, i.e., for u = ă , v = f so that above
equation becomes
1 1 1
f
= 1 ...(4)
R1 R 2
which is known as Lens-makerÊs formula and in its light Eq. (3) for a lens becomes
1 1 1
= f ...(5)
v u
I v
m = ...(6)
O u
1
D
while power* P =
f in m
100
dioptre
= in cm
f ...(7)
7 .5 I m p o r t a n t Po i n t s
(1) For real extended objects if the image formed by a single lens is erect (i..e., m is positive)
it is always virtual. In this situation if the image is enlarged the lens is converging (i.e.,
convex) with object between focus and optical centre [Fig. (A)] and if diminished the lens
is diverging (i.e., concave) with image between focus and optical centre [Fig.(B)].
OPTICS
QUIZRR 71
I FO O F I
(A) (B)
(2) For real extended object, if the image formed by a single lens is inverted (i.e., m is negative)
it is always real and the lens is convergent, i.e. convex. In this situation if the size of image
is :
smaller than object object is equal to object object is at larger than object object
between and 2F image is 2F image is at 2F is between C and F and
between F and 2F image between C and
I O 2F
O 2F F F 2F 2F F F I 2F F F 2F
(3) In case of inverted image formed by a lens the inversion is true, i.e., left is turned right and
up, down as shown in Fig. (A).
A B
B A
Object Lens Image
(A) (B)
(4) As every part of a lens forms complete image, if a portion (say lower half) is obstructed (say
covered with black paper) full image will be formed but brightness, i.e. intensity will be
reduced (to half) [Fig. (B)].
(5) If an object is moved at constant speed towards a convex lens from infinity to focus, the
image will move slower in the beginning and faster later on, away from the lens. This is
because in the time the object moves from infinity to 2 F, the image will move from F to 2F
and when the object moves from 2F to F, the image will move from 2F to infinity. At 2F the
speed of object and image will be equal. In case of lens, speed of image
2
f
Vi V0
u f
OPTICS
72 QUIZRR
where V0 is the speed of object (u and f are to be substituted with proper sign).
(6) In case of sun-goggles, the radii of curvature of two surfaces are equal with centre on same
side, i.e.,
R1 = R2 = R
1 1 1
So = 1 = 0
f R R
R1 R2 R2 R1
f f
(A) (B)
1 1 1
= 1
f R1 R 2
(8) If an equi-convex lens of focal length f is cut into two equal parts by a horizontal plane AB
Fig. B], then as none of ø, R1 and R2 will change, the focal length of each part will be equal
to that of initial lens, i.e.,
1 1 1 2 1
= 1
f R R R
However in this situation as light transmitting area of each part becomes half of initial, so
intensity will be reduced to half and aperture to 1 / 2 times of its initial value [as I
2
(Aperture) ]. C
f
f´ f´
R ø R A B
f f
D
(A) (B) (C)
OPTICS
QUIZRR 73
However, if the same lens is cut into two equal parts by a vertical plane CD [ Fig. (C)], the
focal length of each part will become
1 1 1 1 1
f'
= 1
R R 2f
i.e., f´ = 2f
i.e., focal length of each part will be double of initial value. In this situation as the light
transmitting area of each part of lens remains equal to initial, intensity and aperture will
not change.
(9) If a lens is made of a number of layers of different refractive indices as shown, ø1
for a given wavelength of light will have as many focal lengths or will form as ø2
many images as there are øÊs as,
ø1
1 ø2
1
f ø1
(10) As focal length of a lens depends on ø, i.e., (1/f) (ø ă 1), the focal length of a given lens
is different for different wavelengths and maximum for red and minimum for violet whatever
be the nature of lens.
V
White light White light R
R
V F FR Fv
V v FR
R
fv R
fv
fR fR V
(A) (B)
(11) If a lens of glass (ø = 3/2) is shifted from air (ø = 1) to water (ø = 4/3) then as :
1 3 / 2 1 3 / 2
1 K and 1 K
fA 1 fW 4 / 3
1 1
with K
R1 R 2
fW 8 K
i.e., f = 4f
fA K 2 W A
i.e., focal length of a lens in water becomes 4 times of its value in air and so power one-
fourth [as P = 1 (1/f)].
OPTICS
74 QUIZRR
(12) If a lens is shifted from one medium to the other, depending on the refractive index of the
lens and medium, the following three situations are possible :
(a) øM < øL but øM increases : In this situation ø = (øL/ øM) will remain greater than unity
but will decrease and as (1/f) (ø ă 1), (1/f) will decrease, i.e., f will increase (without
change in nature of lens) as explained in point (11).
(b) øM = øL. In this situation ø = (øL/øM) = 1, so that (1/f) (ø ă 1) = 0, i.e., f = , lens
will neither converge nor diverge but will behave as a plane glass plane.
øL øL
øM øM øM øM
øL = øM
(A) (B)
(c) øM > øL : In this situation ø = (øL/øM) < 1, so in lens-makerÊs formula sign of f and
hence nature of lens will change, i.e., a convergent lens will behave as divergent and
vice-versa [Fig.].
øL øM øL
øM
øM
øM
(A) (B)
Qu e s t io n I :
In case of a thin lens of focal length F if an object is placed at a distance x 1 from first focus
and its image is formed at a distance x 2 from the second focus, show that x 1x 2 = f 2
Answer :
As in case of thin lens the distance of either foci from the optical centre is f,
u = (f + x1) u v
x1 f f x2
and v = (f + x2) O F1 F2 I
Substituting these values of u and v in lens-formula with proper sign
1
1 1 x1 x2 2 f 1
f x2 f x1 = f
or
f x1 f x2 f
i.e. fx1 + fx2 + 2f2 = f2 + fx1 + fx2 + x1x2
or x1x2 = f2
Note : Formula x1x2 = f2 is called NewtonÊs formula.
OPTICS
QUIZRR 75
Qu e s t io n I I :
A thin converging lens of focal length f is placed between an object and a screen fixed at
a distance D apart. Show that if D > 4f, there are two positions of the lens at which a sharp
image of the object is formed on the screen.
Answer :
If the object is at a distance μ from the lens, the distance of image from the lens v = (D ă u). So
from lens-formula,
1 1 1
=
v u f
1 1 1
we have
D u u =
f
i.e. u2 ă Du + Df = 0
1
so that u = D DD 4 f ...(1)
2
D
x
D
O I2
u u1
O Object Lens Screen v1
u2
v2 I1
(A) (B)
Now there are three possibilities :
(a) If D < 4f; u will be imaginary, so physically no position of lens is possible.
(b) If D = 4f : In this situation u = D/2 = 2f. So only one position is possible and in this situation,
v = D ă u = 4f ă 2f = 2f (= u)
(c) If D > 4f : In this situation both the roots of Eq. (1) will be real, i.e.,
1
u1 = D D D 4 f
2
1
and u2 = D D D 4 f ...(2)
2
So if D > 4f : there are two positions of lens at distances u1 and u2 from the object for which
real images is formed on the screen.
OPTICS
76 QUIZRR
Note : This method is called ÂDisplacement methodÊ and is used to determine the focal length
of convergent lens. In case of displacement method :
(i) If the distance between two positions of the lens is x
x = u 2 ă u1 = D D 4 f [From Eq. 2]
1
v1 = D ă u1 = D ă D DD 4 f
2
1
= D DD 4 f = u
2 2
1
and v2 = D ă u2 = D D DD 4 f
2
1
= D D D 4 f u1
2
i.e., for two positions of the lens object and image distances are interchangeable.
(iii) As x = u2 ă u1 and D = v1 + u1 = u2 + u1 [as v1 = u2]
i.e., u1 (= v2) = (D ă x)/2 and u2 (= v1) = (D + x)/2
So the magnification for two positions of the lens will be respectively,
I1 v1 D x
m1 = = u = D x
O 1
I2 v2 D x
and m2 = = u = ...(4)
O 2 D x
And hence :
2
m1 I1 D x D x D x
(b) = ...(6)
m2 I2 D x D x D x
D x D x 4Dx
(c) m1 ă m2 = D x D x 2 2
D x
x x
m1 m2 , i.e., f ...(7)
f m1 m2
OPTICS
QUIZRR 77
Ex a m p l e 3 7
Diameter of a plano-convex lens is 6 cm and its thickness at the centre is 3 mm. What is
the focal length of the lens if the speed of light in the material of lens is 2 ï 108 m/s ?
Solution :
According to lens-makerÊs formula,
1 1 1 L
1 with
f R
1 R 2 M
1 1 R
or f 1
f R ...(1)
c 3 108
1.5 ...(2)
v 2 108
y
And if r is the radius and y is the thickness of lens (at the centre), the radius A
of curvature R of its curved surface in accordance with Fig. will be given by R
r
O
2 2
R = r + (R ă y) 2 B
Răy
C
r2 6 / 2
2
i.e. R 15 cm ...(3)
2 y 2 0.3
So substituting the values of ø and R from Eqs. (2) and (3) in (1),
15
f 30 cm
1.5 1 Ans.
Ex a m p l e 3 8
A glass convex lens of refractive index (3/2) has got a focal length equal to 0.3 m. Find the
focal length of the lens if it is immersed in water of refractive index (4/3).
Solution : As according to lens-makerÊs formula,
1 1 1 L
f
= 1 with
M
R1 R 2
1 3 / 2
So = 1 K
fA 1
OPTICS
78 QUIZRR
1 3 / 2
and = 1 K
fW 4 / 3
1 1
with K =
R1 R 2
fW 8 K
i.e., = 4
fA K 2
Ex a m p l e 3 9
A plano-convex lens has a thickness of 4 cm. When placed on a horizontal table with the
curved surface in contact with it, the apparent depth of the bottom-most point of the lens
is found to be 3 cm. If the lens is inverted such that the plane face is in contact with the
table, the apparent depth of the centre of the plane face of the lens is found to be (25/8) cm.
Find the focal length of the lens.
Solution :
In case of refraction from a curved surface,
2 1 2 1
v u R
So when curved surface is in contact with the table, refraction will take place at plane surface.
1 1 4
So = , i.e., ...(1)
3 3
3 cm I 4 cm 25/8 cm 4 cm
O O
(A) (B)
And when plane surface is on the table, refraction will take place at curved surface.
1
4 / 3 1 4 / 3
So =
25 / 8 4 R
8 1 1
i.e., = or R = 25 cm ...(2)
25 3 3R
Now the lens is thick; as it has thickness 4 cm, there are two possibilities :
(a) If the plane surface faces the incident parallel beam of light [Fig. (A)] : In this situation
refraction will take place only on the curved surface with
OPTICS
QUIZRR 79
u = ă , v = f; ø1 = (4/3); ø2 = 1 and R = ă 25 cm
2 1 2 1
So from
v u R
1 4 / 3 1 4 / 3
we have
f 25
F I
75cm 72cm
100cm
(A) (B)
i.e., f = + 75 cm. So the focal length, i.e., distance of focus from plane surface = 4 + 75 =
79 cm
(b) If the curved surface faces the incident parallel beam of light [Fig. 3.35 (B)] : In this
situation from refraction at curved surface, we have
4 / 3 1 4 / 3 1
, i.e., v1 = 100 cm
v1 25
So the distance of I from plane surface will be 100 ă 4 = 96 cm. The image I will act as
virtual object for plane surface so that
1 4 / 3 1 4 / 3
, i.e., f = 72 cm
f 96
i.e., in this situation focal length is 72 cm, i.e., focus will be at a distance of 72 cm from the
plane surface.
Note : Calculating focal length of the lens by using lens-makerÊs formula.
1 1 1
1 which gives f = 75 cm
f R
1 R 2
is not proper as the lens is thick and so its focal length will depend on the fact that whether
the plane or curved surface faces the incident parallel beam. Further more, if curved surface
faces the incident light, u for plane surface will not be v of curved surface [but v ă 4) as
is taken here in case (b)]. So lens-makerÊs formula is not valid here.
Ex a m p l e 4 0
A converging beam of light forms a sharp image of a screen. A lens is placed in the path
of the beam, the lens being 10 cm from the screen. It is found that the screen has to be
moved 8 cm further away from the lens to obtain a sharp image. Find the focal length and
nature of lens.
OPTICS
80 QUIZRR
Solution :
As shown in Fig., in this problem object O is virtual while image I real, so that
u = 10 cm and v = (10 + 8) = 18 cm
and hence from lens formula,
1 1 1
10cm 8cm
v u f
I
O
1 1 1
we have u
18 10 f v
S S´
which on simplification gives
0
f 22.5 cm
4
i.e., the lens is diverging (i.e., concave) of focal length 22.5 cm. Ans.
Ex a m p l e 4 1
An object 25 cm high is placed in front of a convex lens of focal length 30 cm. If the height
of image formed is 50 cm, find the distance between the object and the image.
Solution :
As object is in front of the lens, it is real and as
I v 50
m 2
O u 25
v
m 2, i.e. v = ă 2u
u
1 1 1
we have =
2u u 30 45cm 90cm
3 1
i.e., =
2u 30
OPTICS
QUIZRR 81
i.e., u = ă 45 cm
So v = ă 2u
= (ă 2)(ă 45)
= 90 cm
As in this situation object and image are on opposite sides of lens, the distance between object and
image d 1 = u + v = 45 + 90 = 135 cm. Ans.
(b) If the image is erect (i.e., virtual)
v
m = 2
u
15cm
i.e., v = 2u
So from lens-formula, we have I O
30cm
1 1 1
=
2u u 30
i.e., u = ă 15 cm
So v = 2u = 2 (ă 15) = ă 30 cm
As in this situation both image and object are in front of the lens, the distance between object and
image d 2 = v ă u = 30 ă 15 = 15 cm. Ans.
Ex a m p l e 4 2
A point object O is placed at a distance of 30 cm from a convex lens (focal length 20 cm)
cut into two halves each of which is displaced by 0.05 cm as shown. Find the position of
the image. If more than one image is is formed, find their number and distance between
them.
L1
O 2 ï 0.05cm
L2
30cm
f = 20 cm
Solution :
Considering each part as separate lens with u = ă 30 cm and f = 20 cm, from lens-formula.
1 1 1
=
v u f
1 1 1
we have = i.e., v = 60 cm
v 30 20
OPTICS
82 QUIZRR
So each part will form a real image of the point object O at 60 cm from the lens as shown in Fig. (B).
As there are two pieces, two images are formed. Now in similar triangles OI1I2 and OL1L2
I1
L1
O
Q P
L2
I2
u = 30cm v = 60cm
I1I2 OP u v
=
L1L 2 OQ u
90
i.e., I1I 2 = 2 0.05 0.3 cm
30
Ex a m p l e 4 3
A convex lens is kept 50 cm above the bottom of an empty tank. A coin is placed at the
bottom of tank and its image is formed 25 cm above the lens. A liquid is now filled in the
200
tank to a depth 40 cm. In this situation, image of the coin is formed cm above the lens.
7
Find the refractive index of liquid.
Solution :
In Fig. , the tank is empty.
u = ă 50 cm
v = 25 cm
using lens-formula,
1 1 1
= f
v u
1 1 1
= f 50 cm
25 50
50
f= cm
3
When a liquid is filled in the tank to a depth 40 cm, apparent position of the coin shifts towards
the lens by
OPTICS
QUIZRR 83
1
40 1 cm
1 40
50 ă 40 1 = 10
In this case,
40
u = 10 cm
200
v= cm (given)
7
50
f= cm (calculated earlier)
3
1 1 1
Using =
v u f
50ă40 1 ă 1
7 1 ø
Apparent 50cm
200 40 = 3 position 40cm
10 50
of coin
40 1 ă 1
ø
1
40 = 3 7 1
or 10 50 200 40
40
10 = 40
ø = 1.33 Ans.
Ex a m p l e 4 4
(a) A screen is kept at a distance 1 m from an object. A converging lens between the object
and the screen, when placed at any of the two positions which are 60 cm apart, forms
a sharp image of the object on the screen. Find focal length of the lens.
(b) In the two positions of the lens, lateral size of the image is 4 cm and 9 cm. Find the
size of the object.
Solution :
This problem is based on ÂDisplacement MethodÊ which is commonly used to determine the focal
length of a converging lens.
OPTICS
84 QUIZRR
D2 x2
(a) f
4D
100 2 60 2
f =
4 100
or f = 16 cm Ans.
(b) Size of the object can be obtained from the relation
O I1I 2
O 4 9 6 cm Ans.
I I1 I2
m = ...
O O I1
i.e. m = m1 ï m2 ï ...
In case of two thin lenses in contact if the first lens of focal length f1 forms the image I1 (of an
object O) at a distance v1 from it,
1 1 1
...(1)
v1 u f1
Now the image I1 will act as an object for second lens and f1 f2
if the second lens forms image I at a distance v from it, then
O
I I1
u v
v1
OPTICS
QUIZRR 85
1 1 1
...(2)
v v1 f2
1 1 1 1
=
v u f1 f2
1 1 1 1 1 1
or = with F f f
v u F 1 2
i.e., the combination behaves as a single lens of equivalent focal length F given by
1 1 1
= f f or P = P1 + P2 ...(3)
F 1 2
1 1 1 d
so P = P1 + P2 ă P1P2d
F f1 f2 f1 f2
1 1 1
= 0
F f f
i.e., F = and P = 0
i.e., the system will behave as a plane glass plate.
(2) If two thin lenses of same nature are put in contact, then as
1 1 1
F f1 f2
1 1 1 1
and
F f1 F f2
OPTICS
86 QUIZRR
(4) If a lens of focal length f is divided into two equal parts as shown in Fig. (A) and each part
has a focal length f´ then as
1 1 1
i.e., f´ = 2f
f f' f'
L2 L2 L2
L1 L1 L1
(A) (B) (C)
1 1 1
, i.e., F = f (= initial value)
F 2f 2f
(5) If a lens of focal length f is cut in two equal parts as shown in Fig. (A), each part will have
focal length f. Now if these parts are put in contact as shown in Fig. (B), the resultant focal
length will be
L2 L2 L2
L1 L1 L1
(A) (B) (C)
1 1 1
, i.e., F = (f/2)
F f f
However if the two parts are put in contact as shown in Fig. (C), first part will behave as
convergent lens of focal length f while the other divergent of same focal length (being
thinner near the axis); so in this situation
1 1 1
, i.e., F = or P = 0
F f f
Qu e s t io n I I I :
A plane glass plate is constructed by combining a plano-convex and a plano-
concave lens of different materials as shown in Fig. Will it act as a lens ? If so,
what will be its focal length and nature ?
OPTICS
QUIZRR 87
Answer :
As øC and øD are refractive indices of convergent and divergent lens respectively and R the
radius of curvature of common interface, then by lens-makerÊs formula,
1 1 1 C 1
= C 1 ...(1)
fC R R
1 1 1
and = D 1
fD R
D 1
= ...(2)
R
1 1 1 D
= C
F fC fD R
R
i.e.,
C D
F =
As øC øD, the system will act as a lens. The system will behave as convergent lens if øC > øD(as
its focal length will be positive) and as divergent lens if øC < øD (as F will be negative).
Ex a m p l e 4 5
Two plano-concave lenses of glass of refractive index 1.5 have radii of curvature of 20 and
30 cm. They are placed in contact with curved surfaces towards each other and the space
between them is filled with a liquid of refractive index (4/3). Find the focal length of the
system.
Solution :
As shown in Fig, the system is equivalent to combination of three thin lenses in contact,
1 1 1 1
i.e., =
F f1 f2 f3
1 3 1 1
= 1
f1 2 20
1
=
40
OPTICS
88 QUIZRR
1 4 1 1
= 1
f2 3 20 30
5
=
180
1 3 1 1 1
= 1
f3 2 30 60
1 1 5 1 1 9 10 6
So , i.e. or F = ă 72 cm.
F 40 180 60 F 360
i.e., the system will behave as a divergent lens of focal length 72 cm. Ans.
Ex a m p l e 4 6
In the given figure, L is a lens such that a parallel beam of light incident on it, after
refraction, converges to a point at a distance ÂxÊ from it and M is a mirror such that a
parallel beam of light incident on it, after reflection, converges to a point at distance y from
it. An object of size 2 cm is kept as shown. Find the nature, position and size of the image
that will be seen by an observer looking towards the mirror through the lens.
M
L
2x 2(x + y)
Solution :
It is given that a parallel beam incident on the lens, after refraction, converges to a point at
distance x from the lens. This implies that focal length of the lens is ÂxÊ.
It is also given that a parallel beam incident on the mirror, after reflection, converges to a point
at distance ÂyÊ from it. This implies that the mirror is a concave of focal length ÂyÊ.
Obviously, the object O is kept at 2F of lens L. Hence its image I1 forms on the other side at 2F,
i.e., at distance 2x right of the lens. The image will be real, inverted and magnification of this image
v 2x
m1 1.
u 2x
Image I1 of O formed by lens L acts as object for the mirror. Its distance from the pole of mirror
will be 2(x + y) ă 2x = 2y. This implies that it will be at the centre of curvature of the concave mirror
(y being the focal length of mirror). Therefore, the concave mirror forms a real, inverted (relative to I1)
OPTICS
QUIZRR 89
L M
O I2
2x
I3 I1
2x 2y
image I2 at the same position. Magnification of I2 (relative to I1), m2 = ă 1. Finally, I2 acts as object
for the lens. Its distance from the lens is 2x. Hence, it is at 2F of the lens (x being the focal length of
lens). Therefore, the lens will form a real and inverted image of I2 at 2F on the other side, i.e., to its
left. Obviously, I3 will be at the same position as of the object.
L M
I2
O
I3
I1
2x 2x 2y
Ex a m p l e 4 7
A convex lens ÂAÊ of focal length Âf 1Ê and a concave lens ÂBÊ of focal length Âf 2Ê are kept along
the same axis with a distance ÂdÊ between them. For what value of ÂdÊ does a parallel beam
of light incident on A leave B as a parallel beam ?
Solution :
A parallel beam incident on the convex lens A, after refraction, will converge towards its focus.
If the concave lens B is so placed that the point of convergence of rays, which is the focus of lens A,
A B
FA FB
d f2
f1
OPTICS
90 QUIZRR
also becomes the first principal focus of lens B, then the rays leaving B will form a parallel beam. In
the other words, the lens B intercepts the rays which are converging towards its focus so that the rays
after refraction become parallel. This is shown in the given figure.
1 1 1
Applying t he r elat ion v u f for relation at B.
1 1 1
f1 d =
f2
f1 ă d = f2
or d = f1 ă f2
Therefore, in the given situation, the two lenses have to be kept at a separation that is equal to
the difference of their focal lengths.
Ex a m p l e 4 8
A concave lens of focal length 20 cm is placed 15 cm in front of a concave mirror of radius
of curvature 26 cm and further 10 cm away from the lens is placed an object. The principal
axis of the lens and the mirror are coincident and the object is on this axis. Find the
position and nature of the image.
Solution :
The lens will from the image I1 of the object O at distance v from it such that
1 1 1 20
i.e., v cm
v 10 20 3
i.e., at a distance (20/3) cm in front of the lens. So the distance of this image I1 from the mirror
will be 15 + (20/3) = (65/3) cm.
The image I1 will act as an object for the mirror and hence, the mirror will form image I2
(of object I1) such that
1 3 1
i.e., v = ă 32.5 cm
v 65 13
M
I3 I2O I1 L
10cm 15cm
140cm
OPTICS
QUIZRR 91
i.e., the mirror will form image I2 at a distance of 32.5 cm in front of it. However, as the lens at
a distance of 15 cm from the mirror, the image I2 will act as an object for the lens again u = 32.5 ă
15 = 17.5 cm, so that
1 2 1
i.e., v = 140 cm
v 35 20
i.e., final image I3 is at a distance of 140 cm in front of the lens and as here
m = m1 ï m2 ï m3
20 / 3 65 / 2 140
i.e., m = 1 8
10 65 / 3 35 / 2
Ex a m p l e 4 9
Consider a co-axial system of two thin convex lenses of focal length f each separated by a
distance d. Draw ray diagrams for image formation corresponding to an object (a) d < f (b)
(c) f < d < 2f (d) d = 2f and (e) d > 2f.
Indicate the nature of the combination (concave, convex or plane) in each case.
Solution :
In case of two thin lenses separated by a distance d, we have
1 1 1 d
= f f f f
F 1 2 1 2
For f 1 = f, f2 = f, f
1 1 1 d F
= f f 2 F1
F f
d<f
f2
i.e., F =
2f d
So (a) If d < f : F will be positive and < f, so the system will behave as convex lens of focal
length < f
(b) If d = f : F =f, i.e., the system will behave as convex lens of focal length f
F
d=f
f < d < 2f
(B) (C)
OPTICS
92 QUIZRR
(c) If f < d < 2f : F will be positive and > f, so the system will behave as convex lens of focal
length > f as shown in Fig.(C).
(d) If d = 2f : F will be infinite, i.e., the system will behave as a plane glass plate of infinite
focal length [Fig.(D)].
F
d = 2f d > 2f
(D) (E)
(e) If d > 2f : F will become negative, i.e., the system will behave as concave lens as shown in
Fig. (E).
Ex a m p l e 5 0
Two thin convex lenses of focal lengths f 1 and f 2 are separated by a y
f2
horizontal distance d (d < f 1 and d < f 2) and their centres are displaced f1
by a vertical separation as shown in Fig. 3.67 (A). Taking the origin
O
of co-ordinates O at the centre of first lens, what would be the x and
y co-ordinates of the focal point of this lens system for a parallel beam d
of rays coming from left ?
Solution :
As the incident beam is parallel, first lens L1, in absence of second lens L2, will form the image
I1 at its focus, i.e., at a distance f1 from O. This image I1 will act as an object for second lens and so
for second lens u = (f1 ă d). If the second lens forms image I2 of I1 at a distance v from it,
y
L1 L2
´
O
I2 y
I1
v
d u
f1
1 1 1 f2 f1 d
v
v f1 d f2 , i.e., f1 f2 d ...(1)
f2 f1 d
x =d + v= d +
f1 f2 d
d f1 d f1 f2
= ...(2)
f1 f2 d
OPTICS
QUIZRR 93
I v
m
O u
' f2 f1 d
So =
f1 f2 d f1 d
f2
´ = f f d
i.e.,
1 2
f2
y = ' 1
f1 f2 d
f1 d
= ...(3)
f1 f2 d
9. L EN S WI T H ON E SI L V ERED SU RFA CE
If the back surface of a lens is silvered and an object is placed in front of it then :
(1) First, light will pass through the lens and it will form the image I1.
R1 A A
R2 A
= + +
O O I1 I2 I1 I2 I3
(2) The image I1 will act as an object for silvered surface which acts as curved mirror and forms
an image I2 of object I1.
(3) The light after reflection from silvered surface will again pass through the lens and lens will
form final image I3 of object I2.
In such situation power of the silvered lens will be
P = PL + PM + PL
1 1 1 1
with PL = where f 1 R R
fL L 1 2
OPTICS
94 QUIZRR
1 R2
and PM = where fM
fM 2
1
F=
P
To make this all clear we now consider the case of a silvered plano-convex lens under
following circumstances.
(A) When the plane surface is silvered and the object is in front of curved surface :
In this situation
1 1 1 1
= 1
fL R R
and FM = =
2
1 1
So PL = f =
L R
1 1
and PM = f = 0
M
1 0 2 1
i.e., P = 2 ...(1)
R R
1 R
F = P 2 1
So
...(2)
i.e., the lens will behave as a concave mirror of focal length [R/2(ø ă 1)].
O O
(A) (B)
OPTICS
QUIZRR 95
(B) When the curved surface is silvered and the object is in front of plane surface :
In this situation
1 1 1 1
fL
= 1
R R
R
and FM =
2
1 1 1 2
So PL = f = and PM =
L R fM R
2 1 2 2
i.e., P = ...(3)
R R R
1 R
So F = ...(4)
P 2
i.e., the lens will be equivalent to a converging mirror of focal length (R/2ø).
Ex a m p l e 5 1
A pin is placed 10 cm in front of a convex lens of focal length 20 cm, made of material
having refractive index 1.5. The surface of the lens farther away from the pin is silvered
and has a radius of curvature 22 cm. Determine the position of the final image. Is the image
real or virtual?
Solution :
As radius of curvature of silvered surface is 22 cm, so
R 22
fM 11 cm = ă 0.11 m
2 2
1 1 1
and hence PM = f 0.11 0.11 D
M
Further as the focal length of lens is 20 cm, i.e., 0.20 m, its power will be
given by : I O
10cm
1 1
PL = f 0.20 D 11cm
L
OPTICS
96 QUIZRR
Now as in image formation, light after passing through the lens will be reflected back by the
curved mirror through the lens again
P = PL + PM + PL = 2PL + PM
2 1 210
i.e., P = D
0.20 0.11 11
1 11 110
F m= cm
P 210 21
i.e., the silvered lens behaves as a concave mirror of focal length (110/21) cm. So for object at a
distance 10 cm in front of it,
1 1 21
i.e., v = ă 11 cm
v 10 110
i.e., image will be 11 cm in front of the silvered lens and will be real.
1 0 . M ICROSCOPE
It is an optical instrument used to increase the visual angle of near objects which are too small
to be seen by naked eye.
1 0 .1 Co n s t r u c t i o n
It consists of two convergent lenses of short focal lengths and apertures arranged co-axially. Lens
(of focal length f0) facing the object is called objective or field lens while the lens (of focal length fe)
facing the eye, eye-piece or ocular. The objective has a smaller aperture and smaller focal length than
eye-piece. The separation between objective and eye-piece can be varied.
1 0 .2 I m a g e Fo r m a t i o n
The object is placed between F and 2F of objective, so the image IM formed by objective (called
intermediate image) is inverted, real, enlarged and a distance greater than 2f0 on the other side of the
lens. This image IM acts as object for eye-piece and is within its focus. So eye-piece forms final image
I which is erect, virtual and enlarged with respect to
intermediate image IM. So the final image I with fo L fe
u v ue
respect to object is inverted, virtual enlarged and at a
Fo Fe
distance D to from eye on the same side of eye-piece h
Fo IM h´
as IM.
OPTICS
QUIZRR 97
1 0 .3 M a g n i f y in g Po w e r (M P)
Magnifying power of an optical instrument is defined as :
h h'
0 and
D ue
h ' D h' D
So MP =
ue h h ue
I v
m =
O u
h' v
i.e., = [as u is ă ive]
h u
vD
So, MP =
u ue
1 1 1
i.e., ue = fe = maximum
ue fe
vD
MP with L = v + f ...(2)
u fe e
A microscope is usually considered to operate in this mode unless stated otherwise. In this mode
as ue is maximum, MP is minimum for a given microscope.
OPTICS
98 QUIZRR
(b2) If the final image is at D (Near point)
In this situation as for eye-piece v = D
1 1 1 1 1 D
, i.e., 1
D ue fe ue D fe
v D
MP = 1
u fe
feD
with L = v f D ...(3)
e
In this situation as ue is minimum MP is maximum and eye is most strained.
In case of microscope as f0 is small and object is close to objective u ~ f0. Also as intermediate
image is in front of eye lens which has very short focal length, length of tube.
L = v + ue v [as ue << v]
So for normal adjustment, i.e., from Eq. (2)
L D
MP ...(4)
f0 fe
1 0 .4 I m p o r t a n t Po i n t s :
(1) As magnifying power is negative, the image seen in a microscope is always truly inverted,
i.e., left is turned right with upside down simultaneously.
(2) As intermediate image is between the two lenses, crosswire (a measuring device) can be
used. So with microscope measurements can be made along with observations.
v D
(3) As m and m , Eq. (1) gives MP = m ï m
u ue
1 1 1 v D D
i.e., me e 1 m
D ue fe ue ue fe
So MP = m ï m = m ï me = Linear magnification
OPTICS
QUIZRR 99
(5) For a microscope MP is minimum when final image is at and maximum when final image
vD v D
is at least distance of distinct vision D, i.e., MP min u f and MP max 1
e u fe
(6) For a given microscope MP for normal setting remains practically unchanged if field and eye
lens are interchanged as MP ~ [LD/f0fe].
(7) As the aperture of both the lenses in a microscope is small, the defects of images particularly
spherical aberration is minimum.
(8) As MP for normal setting is (LD/f0fe), so to have large MP, f0 and fe must be as small as
practically possible. This is why in a microscope both the lenses have small focal lengths.
(9) Out of f0 and fe, f0 is taken to be smaller, so that field of view may be increased and objective
being closer to object may collect more light to increase brightness of the image. This also
ensures that intermediate image is in front of eye lens within its focus.
(10) While working with a microscope the eye must be close to eye-piece, i.e, persons wearing
specks should take off their specks (if possible) otherwise MP will be adversly affected. This
is because if eye is at a distance d from the eye-piece, the distance of final image from eye-
piece will be D´ = (D ă d) and hence MP will become
LD' L D d d
MP1 MP 1 MP
f0 fe f0 fe D
(11) In reference to a microscope, the minimum distance between two lines at which they are just
distinct is called limit of resolution and reciprocal of limit of resolution is called resolving
power which for a microscope varies inversely with wavelength, i.e.,
1 1
RP *
x
Smaller the limit of resolution of higher the resolving power, more details of object will be
visible in the image. In case of microscope RP is increased by decreasing the wavelength of
light used and we have ultra-microscope and electron-microscope.
Note : In case of electron-microscope it has been shown that wavelength of electron
5
proportional to V and can be upto about 10 times that of optical microscope.
Ex a m p l e 5 2
A compound microscope has a magnifying power 30. The focal length of its eye-piece is 5
cm. Assuming the final image to be at the least distance of distinct vision (25 cm), calculate
the magnification produced by objective.
OPTICS
100 QUIZRR
Solution :
In case of compound microscope,
MP = m ï m ...(1)
And in case of final image at least distance of distinct vision,
D
m = 1 ...(2)
fe
D
MP = m 1 +
fe
25 30
So 30 m 1 , i.e., m 5 Ans.
5 6
Ex a m p l e 5 3
A compound microscope has an objective of focal length 2 cm and an eye-piece of focal
length 5 cm. If an object is placed at a distance of 2.4 cm in front of the field lens, find the
magnifying power of the instrument and length of the tube if (a) final image is at infinity
(b) final image is at least distance of distinct vision (= 25 cm).
Solution :
As object is at a distance of 2.4 cm in front of field lens of focal length 2 cm, field lens will form
its image at distance v such that
1 1 1
= i.e., v = 13 cm
v 2.4 2
v 12
so that m = 5
u 2.4
1 1 1
For eye-piece, i.e., u = ă 5 cm
ue 5 e
OPTICS
QUIZRR 101
and L = v + ue = 12 + 5 = 17 cm
In this situation eye is said to be relaxed and for a given microscope MP is minimum while length
of tube maximum.
(b) If final image is at D (Near point)
1 1 1 25
For eye-piece, 25 u 5 , i.e., ue 4.17 cm
e 6
D 25
Note : In case (b) me u 25 / 6 6 m
e
So here MP = m ï m = m ï me = linear magnification
Ex a m p l e 5 4
A compound microscope is used to enlarge an object kept at a distance 0.03 m from its
objective which consists of several convex lenses in contact and has focal length 0.02 m.
If a lens of focal length 0.1 m is removed from the objective, find out the distance by which
the eye-piece of the microscope must be moved to refocus the image.
Solution :
If initially the objective forms the image at distance v1,
1 1 1
= i.e., v1 = 6 cm
v1 3 2
1 1 1 1 1
= ... or
F f1 f2 F f1 F'
1 1 1
with = ...
F f2 f3
So if one of the lenses is removed, the focal length of the remaining lens system
1 1 1 1 1
= F f 2 10 i.e., F´ = 2.5 cm
F' 1
OPTICS
102 QUIZRR
This lens will form the image of same object at a distance v2 such that
1 1 1
i.e., v2 = 15 cm
v2 3 25
So to refocus the image, eye-piece must be moved by the same distance through which the image
formed by the objective has shifted, i.e., 15 ă 6 = 9 cm away from the objective.
Ex a m p l e 5 5
The focal lengths of the objective and the eye-piece of a compound microscope are 2.0 cm
and 3.0 cm respectively. The distance between the objective and the eye-piece is 15.0 cm.
The final image formed by the eye-piece is at infinity. Find the distance of object and image
produced by the objective, from the objective lens.
Solution :
As final image is at infinity, the distance of intermediate image from eye lens ue will be given by
1 1 1
i.e., ue = ă fe = ă 3 cm
ue fe
and as the distance between the lenses is 15.0 cm, the distance of intermediate image (formed by
objective) from the objective will be
v = L ă ue = L ă fe = 15 ă 3
= 12 cm
and if u is the distance of object from objective,
1 1 1
i.e., u = ă 2.4 cm
12 u 2
So object is at a distance of 2.4 cm in front of objective.
1 1 . T EL ESCOPE
It is an optical instrument used to increase the visual angle of distant large objects such as a star,
a planet or a cliff, etc.
Astronomical telescope consists of two converging lenses. The one facing the object is called
objective or field lens and has large focal length and aperture while the other facing the eye called eye-
piece or ocular has small focal length and aperture. The distance between the two lenses is adjustable.
fo L fe
u= v = fo ue
0 0 Fe Fo
Fo y IM
Field
lens Eye
lens
OPTICS
QUIZRR 103
As a telescope is used to see distant objects, in it object is between and 2F of objective and hence
image formed by objective is real, inverted and diminished and is between F and 2F on the other side
of it. This image (called intermediate image) acts as an object for eye-piece and by shifting the position
of eye-piece, it is brought within its focus. So final image I, with respect to intermediate image, is erect,
virtual, enlarged and at a distance D to from the eye. This is turn implies that final image with
respect to object is inverted, enlarged and a distance D to from the eye.
1 1 .1 M a g n i f y in g Po w e r (M P)
Magnifying power of a telescope is defined as,
y y
0 and
f0 ue
f
So, MP = 0
0 ue
1 1 1
i.e., ue = fe
ue fe
1 1 1 1 1 f
= i.e., 1 e
D ue fe ue fe D
OPTICS
104 QUIZRR
f0 fe
MP = 1 D
fe
feD
with L = f0 f D ...(3)
e
In this situation ue is minimum, so for a given telescope MP is maximum while length of tube
minimum and eye is most strained.
In case of a telescope, if object and final image are at infinity and total light entering the telescope
leaves it parallel to its axis as shown in Fig.,
D d
fo fe
f0 Aperture of objective
=
fe Aperture of eye-piece
f0 D
i.e., MP = ...(4)
fe d
1 1 .2 I m p o r t a n t Po i n t s
(1) As magnifying power is negative, the image seen in astronomical telescope is truly inverted,
i.e., left is turned right with upside down simultaneously. However, as most of the astronomical
objects are symmetrical this inversion does not affect the observations.
(2) For given telescope, magnifying power is minimum when final image is at infinity (Far
point) and maximum when it is at distance of distinct vision (Near point), i.e.,
f f fe
MP min 0 and MP max 0 1 D
fe fe
(3) In a telescope, if field and eye lenses are interchanged MP will change from (f0/fe) to (fe/f0),
i.e., it will change from m to (1/m), i.e., will become (1/m2) times of its initial value.
(4) As MP for normal setting as (f0/fe), so to have large MP, f0 must be as large as practically
possible and fe small.
(5) If an astronomical telescope, the convergent eye-piece is replaced by a divergent lens which
is placed in such a way that rays from objective are directed towards its focus final image
OPTICS
QUIZRR 105
IM
fe
fo
Galilean telescope
will be erect, enlarged and virtual. This telescope is also used to see distant terrestial objects
and is called Galilean telescope and for it
f0
MP with L = f0 = fe
fe
In this telescope as intermediate image is outside the tube, the telescope cannot be used for
making measurements.
Qu e s t io n V :
How does magnifying power vary with change in length of tube for a given telescope ?
Answer : For a telescope,
|MP| = (f0/ue) with L = (f0 + ue)
and as for a given telescope f0 is constant, with increase in length of tube L, ue (distance of
intermediate image from eye-piece) will increase [from a minimum value Dfe/(D + fe) to the maximum
fe] and hence magnifying power will decrease.
v D
MP with L = v + ue
u ue
and as for a given microscope, with increase and hence MP will decrease.
Ex a m p l e 5 6
An astronomical telescope has an angular magnification of magnitude 5 for distant objects.
The separation between the objective and eye-piece is 36 cm and the final image is formed
at infinity. Determine the focal length of objective and eye-piece.
Solution :
In case of astronomical telescope if object and final image both are infinity,
MP = ă (f0/fe) and L = f0 + fe
So here ă (f0/fe) = ă 5 and f0 + fe = 36
Solving these for f0 and fe, we get
So here f 0 = 30 cm and fe = 6 cm Ans.
OPTICS
106 QUIZRR
Ex a m p l e 5 7
A telescope has an objective of focal length 50 cm and an eye-piece of focal length 5 cm.
The least distance of distinct vision is 25 cm. The telescope is focused for distinct vision on
a scale 2 m away from the objective. Calculate (a) magnification produced and (b) separation
between objective and eye-piece.
Solution :
As objective has focal length 50 cm and object is 2 m from it, it will form the image of object at
a distance v such that
1 1 1 200
i.e., v cm
v 200 50 3
v 200 / 3 1
with m0
u 200 3
and as focal length of eye-piece is 5 cm and it forms as image 25 cm in front of it, the distance
of object (image formed by objective) from it will be
1 1 1 25
= i.e., ue cm
25 ue 5 6
25
me = 25 / 6 6
with
and hence
(a) Magnification m = m0 ï me = (ă 1/3) ï 6 = ă 2, i.e., final image is inverted, virtual, double
of object and is at a distance of 25 cm in front of eye lens.
(b) As distance of intermediate image (which is between the two lenses), from objective is (200/
3) cm while from the eye-lens is (25/6) cm, so separation between the objective and eye-piece,
200 25 425
L 70.83 cm Ans.
3 6 6
Ex a m p l e 5 8
A Galilean telescope consists of an objective of focal length 12 cm and eye-piece of focal
length 4 cm. What should be the separation of the two lenses when the virtual image of a
distant object is formed at a distance of 24 cm from the eye-piece ? What is the magnifying
power of telescope under this condition ?
OPTICS
QUIZRR 107
Solution :
As object is distant, i.e., u = ă , so
1 1 1
i.e., v = f0 = 12 cm
v f0
i.e., objective will form the image IM at its focus which is at a distance of 12 cm from O.
Now as eye-piece of focal length ă 4 cm forms image I at a distance of 24 cm from it,
1 1 1 24
i.e., ue 4.8 cm
24 ue 4 5
i.e., the distance of IM from eye lens EA is 4.8 cm. So the length of tube
L = OA ă EA = 12 ă 4.8 = 7.2 cm
Now by definition :
tan
AB / EA OA
MP =
0 tan 0 AB / OA EA
f0 12 10
So, MP = 2.5 Ans.
ue 4.8 4
fo =12cm
I
fe
fo
o ue
O A
o IM
L=7.2cm
24cm B
OPTICS
ALTERNATING
CURRENT
QUIZRR 3
1. Alternating Current
As the name suggests, alternating current (ac) is current that flows in one direction in a conduc-
tor, then changes direction and flows in the other direction.
The reason for this alternation is the way current comes from the electric generators. The emf and
current produced by the generator does not alternate instantly between maximum values in each
direction, but they build up to maximum values and then decrease, change dierction and build
to maximum values in the other direction.
I Conductor I (d)
(a)
+
(a)
(b)
I=0
t
(b)
I
(c)
–
I (c)
(d)
emf E
Current I
t
If the current or voltage varies periodically as ÂsinÊ or ÂcosÊ function of time, the current or voltage
is said to be sinusoidal.
+
+ +
I = I0 sin t
I or V t
V = V0 sin t
·
·
ALTERNATING CURRENT
4 QUIZRR
where, I = instantaneous current
I 0 = maximum instantaneous current
= frequency of oscillation
V = instantaneous voltage
V0 = maximum instantaneous voltage
If the ÂaverageÊ or Âmean valueÊ of alternating current or voltage is defined for full cycle, it will
T T
be be zero as 0 sin t dt or 0 cos t dt 0 , so it is defined for positive (or negative) half cycle
as,
T/ /
2
Iav or Imean =
0 I dt
0
I0 sin t dt
2I0
T/ /
0 0
2
dt dt
RMS Value
Effective, virtual or rms value of alternating current is defined as the square root of the average
of I2 during a complete cycle, i.e.,
1/
1/ 2
2 2 2
dt
T 2
I I0 sin 2 t dt I0
I rms 0
T 0 2 /
0 dt 2
0 dt
2
Similarly, Vav V0
V0
Vrms
2
or
I0 > Irms > Iav
ALTERNATING CURRENT
QUIZRR 5
Example 1
Calculate (a) peak and rms value of the voltage, (b) average voltage, (c) frequency of ac.
Solution :
(a) As in case of ac, V = V0 sin (t ă )
The peak value
V0
Vrms ; V = 200 V Ans.
2 rms
(b) In case of ac
2 2
Vav V0 311 198.17 V Ans.
314
i.e., f 50 Hz Ans.
2
Example 2
dc
I a ac
b
t
+ I = ?
t
ALTERNATING CURRENT
6 QUIZRR
Solution :
1/
2
T I2 dt 1/
= 0 1 T 2
0
2
So, I eff
T (a b sin t) dt
0 dt T
1/
2
1 T
0
2 2 2
i.e. Ieff = ( a 2 ab sin t b sin t) dt
T
1 T 1 T 1
0 sin t 0 0 sin
2
but as and t dt
T T 2
1/
2
1
So I eff a2 b2
2
V = IR i.e. V
I=
R
V0
or I= sin t
R
V0 I
or I = I0 sin t where I0
R VR
Phasor Diagram
Thus, it is clear that :
(1) The frequency of current in the circuit is and is same as that of applied voltage.
(2) In a resistance, applied voltage is in phase with the resulting current
ALTERNATING CURRENT
QUIZRR 7
dI
EL 0
dt L
dI
L E0 sin t
dt
E0 V = Vo sint
or I= cos t
L
E0
i.e., I I0 sin t with I0
2 t
/2
IL
Phasor Diagram
I
E0
2
O 3
E
t
Wave diagram
ALTERNATING CURRENT
8 QUIZRR
E0
(2) As here I0 , the quality L is called „inductive-reactance‰ and is represented by XL and
L
represents the opposition of a coil to ac, i.e.,
XL = L = 2fL
q = C V0 sin t
dq
or I C V0 sin t
dt
or I = I0 sin t with I0 = V0 C
2
V = V0 sin t
Thus, it is clear that :
(1) Current in the circuit has same frequency as the applied voltage but leads it by
2
[or voltage across a capacitor lags the current by ]
2
Ic
/2
Vc Phasor Diagram
I
E0
3
2
E
t
Wave diagram
ALTERNATING CURRENT
QUIZRR 9
E0 1
(2) As here I0 = CE0 = with X C
X
C C
In addition to inductance, most ac circuits have resistance in the form of lights or other resistors.
Impedance is a measure of the total opposition to current flow in an ac circuit resulting from the
effect of both the resistance and the inductive reactance on the circuit. OhmÊs law in an ac circuit
can be written as
E R
I=
Z
E L
where I = current
E = voltage
Z = impedance
The impedance of a series circuit containing a resistance and an inductance is
Z = R 2 X 2L
where Z = impedance
R = resistance
XL = inductive reactance
f = frequency
L = inductance
The impedance can be represented as a vector as the
hypotenuse of the right triangle shown in fig. The resistance y
is always drawn as a vector pointing in the positive
x-direction. The inductive reactance is drawn as a vector
pointing in the positive y-direction. The angle shown
between the resistance and impedance vectors is the phase
Z)
angle and equals the amount by which the current lags e(
anc Inductive
d
behind the voltage. The phase angle is given by pe
Im reactance (XL)
XL
tan Resistance (R)
R
ALTERNATING CURRENT
10 QUIZRR
Capacitance and Resistance in Series
The combined effect of capacitance and resistance in series is measured by the impedance, Z of
the circuit.
Z = R2 XC
2
2
1
Z = R2
2fC
where Z = impedance
R = resistance
XC = capacitive reactance
f = frequency
C = capacitance
E
The current is given by OhmÊs law : I=
Z
where I = current
E = voltage y
Z = impedance R
The phase angle can be found by drawing the resistance as a
vector in the positive x-direction and the capacitive impedance as
a vector in the negative y-direction as shown in fig. 18.25. The Xc
Z
phase angle gives the amount by which the voltage lags behind
the current.
XC
tan
R
Many circuits that are important in the design of electronic equipment contain all three types of
circuit elements discussed in this chapter. The impedance of a circuit containing resistance,
capacitance, and inductance in series can be found from the equation.
Z = R2 X L X C
2
ALTERNATING CURRENT
QUIZRR 11
where Z = impedance
R = resistance
XL = inductive reactance
XC = capacitive reactance
The vector diagrm for this type is shown in fig. The phase angle is given by
XL XC
tan
R
In a circuit containing R, L and C components, the circuit is inductive if XL > XC and the current
lags behind the voltage. A circuit is capacitive if XC > XL, in which case the voltage lags behind
the current. If XC = XL, the current is resistive; the voltage and current are in phase. If the circuit
is to be maximized, the voltage and current must be kept in phase. The current in this type of
circuit is given by
E E
I=
Z 2
1
R 2 2fL
2fC
y y
XC
Z
XL
R
x x
R
Z XC
XL
Inductive Capacitive
Resonance
The current in a circuit containing resistance, capacitance and inductance is given by the equation
E
I=
R2 XL XC
2
ALTERNATING CURRENT
12 QUIZRR
When the inductive reactance equals the capacitive reactance, they nullify each other, and the
current is given by
E
I=
R
which is its maximum possible value. When this condition exists, the circuit is in resonance with
the applied voltage. To have resonance, it is essential for the circuit to have both capacitance and
inductance.
Resonance circuits are used in radios and televisions. The frequency of a certain station is tuned
in when a resonant circuit (antenna circuit) is adjusted to that frequency. This is accomplished
by changing the capaciance until the capacitive reactance equals the inductive reactance. The
applied voltage is the radio signal picked up by the antenna.
XL = X C
1
2fL = 2fC (By substitution)
1
f2 = 2 (solve for f2)
4 LC
1
f = (Take the square root of both sides.)
2 LC
The circuit can be adjusted to any frequency by varying the capacitance or the inductance.
Power in AC Circuit
V0 1 X
with, I0 = and tan R
Z
T
P dt 1 V I T sin t sin t dt
Pav 0 T 0 0 0
T
0 dt
ALTERNATING CURRENT
QUIZRR 13
V0 I0 T
or Pav 0 cos cos 2 t dt
2T
2
which on simplification with T = gives,
1
Pav V0 I0 cos Vrms I rms cos
2
R R
PF = cos = 1
Z R
1
So, Pav = Vrms Irms ï 1 = V0 I0 max
2
R 0
PF = cos = 0
Z X
an ac circuit if the phase difference between V and I is .
2
i.e., if V = V0 sin t
I = I0 cos t
the power consumed by the circuit is zero. Such current is called ÂwattlessÊ.
Example 3
A box P and a coil Q are conneced in a series with an ac source of variable frequency. The
emf of source is constant at 10 V. Box P contains a capacitance of 1 øF in series with a
resistace of 32 . Coil Q has a self-inductance 4.9 mH and a resistance of 68 in series. The
frequency is adjusted so that the maximum current flows in P and Q. Find the impedance
of P and Q at this frequency. Also find the voltage across P and Q respectively.
ALTERNATING CURRENT
14 QUIZRR
Solution :
As this circuit is a series LCR circuit, current will be maximum at resonance.
1 1 105 rad
i.e.,
LC (4.9 103 )(106 ) 7 s
E0 10 V 1
with I m ax A
R (32 + 68) 10
So the impedance
1/ 2 1/ 2
ZP = R12 (1/ C)2 (32)2 (7 /105 106 )2
= 5924 77 Ans.
1/ 2 1/ 2
And Z Q = R 22 (L)2 (68)2 (4.9 103 105 / 7)2
And hence
1
VP = IZP = (77) 7.7 V Ans.
10
1
And VQ = IZQ = (97.6) 9.76 V Ans.
10
ALTERNATING CURRENT
QUIZRR 15
Example 4 L
V
and so, IL = sin t = ă 0.8 cos t
XL 2
V
IC = sin t = + 0.6 cos t
XC 2
Example 5
For a resistance R and capacitance C in series, the impedance is twice that of a parallel
combination of the same. What is the frequency of applied emf ?
Solution :
As shown in Fig. (A), in case of series combination,
1/ 2
Zs R 2 XC
2
R2 (1 / C)2
R
R
IR
I
C
IC
C
(A) (B)
ALTERNATING CURRENT
16 QUIZRR
In case of parallel combination,
V V
IR = sin t and IC sin t
R XC 2
V V
So, I = IR + IC = R sin t X cos t
C
V V
with I0 cos = and I0 sin
R XC
1/ 2
V 2 V 2
V
So, I0 = R X
C ZP
1/2
1 1 1
2 R
2 ZP
i.e., , i.e.,
ZP R XC 1 + 2 C2 R2
(R22C2 1) R2
i.e., = 4
2C2 (1 R2 2C2 )
1 1
or, i.e., f Ans.
RC 2RC
Example 6
A box contains L, C and R. When 250 V dc is applied to the terminals of the box, a current
of 1.0 A flows in the circuit. When an ac source of 250 V rms at 2250 rad/s is connected, a
current of 1.25 A rms flows. It is observed that the current rises with frequency and becomes
maximum at 4500 rad/s. Find the values of L, C and R. Draw the circuit diagram.
ALTERNATING CURRENT
QUIZRR 17
Solution : R
V 250
i.e. R =
I 1
1 1 1
0 , i.e., LC = ... (2)
LC 02 (4500)2
V 250
IR = sin t sin t 1sin t
R 250
V
IX = sin t
X
V
So, I = IR + IX = 1 sin t + sin t
X
or I = I0 sin (t + )
V
with, I0 cos = 1 and I0 sin =
X
2 2
V 250
i.e., I02 = 1+ or (1.25)2 1
X X
250 1000
i.e., X = =
0.75 3
ALTERNATING CURRENT
18 QUIZRR
1 1000 1
i.e. L ~ = as X = L ~
C 3 C
1 1000
or, 2250 ~ ...(3)
2250C 3
rad
as = 2250 s
Substituting the value of L in terms of C from Eqs. (2) and (3), we have,
1 1 1000
2250 2
~ =
C(4500) 2250C 3
1 2250 1 1000
or, ~ =
C 4500 4500 2250 3
1 1 1 1000
~
C 9000 2250
or,
3
1 3 1000
or i.e., C = 10ă6 F = 1 øF Ans.
C 9000 3
1 1 4
L= 6
0.049 H Ans.
C02 10 (4500) 2 81
ALTERNATING CURRENT
MAGNETISM
QUIZRR 3
1. Co n c e p t o f M a g n e t i c Fi e l d :
What is magnetic field and how is it Generated :
Consider the situation when a charge placed near an electrical
circuit. When the switch is closed and the charge is at rest, then s
there is no force between the current carrying wire and charged i +q
S N
A magnet
Also, in case of permanent magnet. We have permanent magnetic field because the electrons
present inside the atoms rotate in the same sense.
2. M o t i o n o f a Ch a r g e I n s i d e M a g n e t i c Fi e l d :
When a charge is moving a magnetic field, then the force acting on that charge is given by
F m q ( v B) ... (i)
This is called Lorentz force. Here. Fm is the magnetic force on a charge q moving with velocity
v in a magnetic field B .
Fm = Bqv sin
where is the angle between v and B .
MAGNETISM
4 QUIZRR
(ii) Fm is zero when,
(iii) Fm is maximum at = 90 and this maximum value is Bqv.
(iv) The units of B must be the same as the units of F/ qv. Therefore, the SI unit of B is
N s
equivalent to . This unit is called the tesla (abbreviated as T), in honour of Nikola
C m
Tesla, the prominent Serbian-American scientist and inventor. Thus,
1N-s 1N
1 tesla = 1T =
C-m A m
4
The CGS unit of B , the gauss (1G = 10 T) is also in common use.
(v) The formula is true for both uniform and non-uniform magnetic field.
(vi) In equation number (i) q is to be substituted with sign. If q is positive magnetic force is along
v B and if q is negative magnetic force is in a direction opposite to v B .
(vii) Direction of Fm : From the property of cross product we can infer that Fm is perpendicular
to both v and B or it is perpendicular to the plane formed by v and B . The exact direction
of Fm can be given by any of the following methods:
(a) Direction of Fm = (sign of q) (direction of v B ) or, as we stated earlier also,
Fm v B if q is positive and
Fm v B if q is negative.
MAGNETISM
QUIZRR 5
(b) FlemingÊs left hand rule : According to this rule, the Thumb F
m
forefinger, the central finger and the thumb of the left
hand are stretched in such a way that they are mutually
perpendicular to each other. If the central finger shows B
Forefinger
the direction of velocity of positive charge ( v q ) and
v+q
Central finger
forefinger shows the direction of magnetic field ( B ),
then the thumb will give the direction of magnetic force Fm . If instead of positive
charge we have the negative charge, then is in opposite direction.
(c) Right hand rule : Wrap the fingers of your right hand around the line perpendicular
to the plane of v and B as shown in figure, so that they curl around with the sense
of rotation from v and B through the smaller angle between them. Your thumb then
points in the direction of the force Fm on a positive charge. (Alternatively, the direction
of the force Fm on a positive charge is the direction in which a right hand thread screw
would advance if turned the same way).
Fm Fm = V B
Right-hand rule
V
B B
v+q
ds
(viii) Fm v or . Therefore, Fm ds or the work done by the magnetic force in a static
dt
magnetic field is zero.
W 0
Fm
So, from work energy theorem KE and hence the speed of the charged particle remains
constant in magnetic field. The magnetic force can change the direction only. It can not
increase or decrease the speed or kinetic energy of the particle.
MAGNETISM
6 QUIZRR
Note : By convention the direction of magnetic field B perpendicular to the paper going inwards
is shown by and the direction perpendicular to the paper coming out is shown by .
Direction of B perpendicular Direction of B perpendicular
to paper inwards to paper outwards
q
S N
V
Note : Actually in this situation the field is not steady, and a time varying magnetic field
produces an electric field due to which charge experiences a force and sets in motion.
E F
A B
V
d I
H F
G
D
C
B
MAGNETISM
QUIZRR 7
and as the magnetic field is along the y-axis, i.e. 1 B Bj→ ,
So, F q ( v B)
F e (vi→ B→j )
i.e. F ev Bk→
As the force on electrons is towards face ABCD, the electrons will accumulate on it and hence it
will acquire lower potential.
3. Pa t h o f A Ch a r g e d Pa r t i c l e i n U n i f o r m M a g n e t i c Fi e l d
The path of a charged particle in uniform magnetic field depends on the angle (the angle
between v and B ). Depending on the different values of , following three cases are possible.
B B
or
q + + q
v v
Fm = 0
Case 2. When = 90
When = 90 , the magnetic force is Fm = Bqv sin 90 = Bqv. This magnetic force is perpendicular
to the velocity at every instant. Hence, path is a circle. The necessary centripetal force is
provided by the magnetic force. Hence, if r be the radius of the circle, then
mv2
Bqv
r
mv
or r
Bq
mv P 2 Km 2 qVm
r
Bq Bq Bq Bq
MAGNETISM
8 QUIZRR
Here, P = momentum of particle
P2
K = K.E. of particle = or P 2 Km
2m
We also know that if the charged particle is accelerated by a potential difference of V volts, it
acquires a KE given by,
K = qV
Further, time period of the circular path will be
mv
2
2r Bq 2m
T
v v Bq
2m
or T
Bq
2 Bq
or the angular speed () of the particle is,
T m
Bq
m
1
Frequency of rotation is, f
T
Bq
or f
2m
Following points are worthnoting regarding a circular path :
(i) T, f and are independent of v while the radius is directly proportional to v.
v1 v2
+ +
q,m q,m
(ii) Hence, if two, charged particles of equal mass and charge enter in a magnetic field B with
different speeds v1 and v2 (> v1) at right angles, then
T1 = T2
but r2 > r1
as shown in figure.
MAGNETISM
QUIZRR 9
Ex a m p l e 1
A charged particle q enters a magnetic field region which spreads over a length d with a
velocity v. Find the angular and sideways deflection and discuss various cases.
Solution :
mv
r qB B
O
r V
d qBd
Now (sin )
r mv R Q
q
P
1 qBd v
sin
mv d
Side ways deflection, PR =r r cos
qBd
= r (1 cos ) where sin =
mv
Cases :
(i) When (d > r), then deviation is .
Ex a m p l e 2
A charged particle is directed towards the centre of a circular field
region having magnetic field B and radius R. The charge and
the velocity of the particle are q and v respectively. Find the angular
deflection of the particle. v
Solution : q1
MAGNETISM
10 QUIZRR
Here OA = R, radius of given circle.
O´A = r , radius of the curvature of the path made by charged particle.
mv
Where r = qB
Now, if we consider OAB and OAB, then length AB in both the triangles will be,
In OAB
180
AB = 2 sin = 2R cos . ... (1)
2 2
In O´AB
AB = 2r sin . ... (2)
2
R Rq B
2 r mv
qRB
2 tan 1
mv
H EL I CA L M OT I ON
Case 3 : When v makes certain angle with the magnetic field
In this case the motion executed by the charged particle is helical motion.
In this case, resolve the velocity vector into two components, one parallel to B field and second
perpendicular to the magnetic field.
V = v sin B
y
V|| = v cos
radius of the helix V
z = ( r)
mv y=0
mv sin
= = x
qB qB
z
2m
Time period of 1 complete rotation =
qB
MAGNETISM
QUIZRR 11
Pitch of the helix = axial distance covered during one complete rotation
P = V||T
2m
P (V cos )
qB
Fe w i m p o r t a n t p o i n t s t o n o t e :
(i) The axis of the helix is always parallel to B field.
(ii) The plane of circular motion is perpendicular to the magnetic field lines.
(iii) Pitch of the helix = (v cos ) T
(iv) A line drawn parallel to axis from the initial point of projection of the particle is tangent to
the helix and after making every full rotation, the particle will touch this tangent again and
again.
If the tangent is taken along the x-axis, then the particle will hit the axis after every pitch.
i.e. it will touch the x-axis at a distance.
x = np
Ex a m p l e 3
An electron gun emits electrons each with K.E. = 2 KeV and directed along positive
x-axis. All electrons are required to hit the spot S, where l = GS = 0.1 m. A uniform magnetic
field exists along GS.
Find the minimum valve of B required.
S
1m
0.
l=
= 60 x-axis
G
V
Solution :
Let the initial velocity v be directed along x-axis,
V|| = v cos = v cos 60
{This is the velocity parallel to GS} S
V = v sin = v sin 60
{This velocity is perpendicular to GS}
MAGNETISM
12 QUIZRR
The GS is the tangent line, along with the particle will be touching again and again after every
complete rotation
n (pitch of helix) = GS
n (v cos 60 T) = l
2(K.E.) 4000 e
and v = J,
m m
2m
T
eB
v 2m
n l
2 eB
nm 4000 e
B
el m
m 4000 e
B
el m
Ex a m p l e 4
A charged particle having charge 10 C and mass m = 3 mg has velocity 10 (i + 2j) cm/s at
origin at t = 0.
Another uncharged particle initially located at (50 cm, 0, 0) is moving with constant velocity
along ve x-axis. The particle collides and stick together and start moving in a circle. Then
find the possible values mass of uncharged particle, if a uniform magnetic field, B 0.6 T i→
exists in the region.
Also find the radius of combined massÊs circular motion.
Solution :
B = (0.6)T
y
V2 = 0.2 m/s
(0.5, 0, 0)
Note that particle 1 follow a helical
m0 path with axis of helix parallel to B
V1 = 10 cm/s field.
= 0.1 m/s V0
z
MAGNETISM
QUIZRR 13
2m 2 3 106
T 1s
qB 10 106 6 101
(3 0.1)
v0
= m .... (2)
n (3 0.1)
(0.5 0.1 n) = V0 n =
m
0.3 n
m (mg)
(0.5 0.1n)
possible values of n = 1, 2, 3, 4
3 9
possible values of m = mg, 2 mg, mg, 12 mg
4 2
m1 v2 mv
Conserve momentum along y-axis, v r
m m qB
1 0
4.
Mot ion of t he c harged part ic le in c om bined field E and B
E
qE
Case I. When E and B and v are mutually perpendicular.
qE
vx v0 cos t
m (v0 cos) E
x
)
But the (v0 sin) component remains same.
n
si
B
Now, after any time t v0
0
(v
B
vy (v0 sin ) sin = (v0sin) sin (t) z
qBt
= (v0 sin ) sin
m
qBt
= (v0 sin ) cos t = (v0 sin ) cos
m
v (vx i→ vy →j vz k→ )
v3
Radius of curvature of the trajectory =
|a v|
Case III. Electric and Magnetic field are mutually perpendicular but v is not :
F q E q ( v B )
Assume the velocity at any instant as V vxi→ vy →j vz k→ and then write the indivisual force
equation.
MAGNETISM
QUIZRR 15
Ex a m p l e 5
An electron is emitted with negligible speed from the negative plate of a parallel plate
capacitor, Charged to potential drop V. Show that the electron will fail to strike the opposite
plate if,
1
2m V 2
d [Assume gravity free space]
eB2
e)(E →j e vxi→ vy →j Bk→
= eE →j e Bvy i→ Bvx →j
Now, use
dv dvy
m x = F ; m Fy
dt x dt
1
2
m v2x v2y eE y ... (3)
MAGNETISM
16 QUIZRR
Now, using eqn. (1),
vx
eB
dvx m . vy dt
0
eB
vx ( y)
m
e2 B2 2 2 2 eE
y vy y
m2 m
2 2eE e2 B2 2
vy y y
m m2
Put vy = 0, to obtain the y - co-ordinate when the particle has only vx component of its velocity < d
2eE e2B2
y y 0
m m2
e2 B2 2eE 2mE
y y
m2 m eB2
2 mE V
d E
eB 2 d
1
2mV 2 mV 2
d d
eB2 d eB2
MAGNETISM
QUIZRR 17
d 2v
y eB dvx
dt m dt
2
d 2v e2B2
y eB eBvy
v
dt2 m m m2 y
Solution of the above Differential equation is,
eB
vy = – A sin (t + ) where =
m
To obtain the values of ,
Put t = 0,
vy = A sin () = 0 (v = 0 at t = 0)
( = 0, )
dvy
A cos (t )
dt
eE
At t = 0, a = [ v 0, FB 0]
m
A
eE eB
cos
m m
E
A = ( = 0)
B
E
Hence vy = sin (t)
B
dy E E
or = sin t y cos t
dt B B
vm cos t vm
y , Now y < d d2
dB(eB) eB2
Ex a m p l e 6
A particle of mass m and charge q is moving in a region where uniform, constant electric
and magnetic fields E and B are present. E and B are parallel to each other. At time
t = 0, the velocity v0 of the particle is perpendicular to E (Assume that its speed is always
<< c, the speed of light in vacuum). Find the velocity v of the particle at time t. You must
express your answer in terms of t, q, m, the vectors v0 , E and B and their magnitudes v0,
E and B.
MAGNETISM
18 QUIZRR
Solution :
→ E B → vo v B
Let us assume, j or ;i and k→ o
E B vo vo B
Y
Force due to electric field will be along y-axis. Magnetic force will not
affect the motion of charged particle in the direction of electric field (or E and B
y-axis). So,
F qE
ay e = constant X
m m vo
qE
Therefore, vy a y t .t ... (i) Z
m
The charged particle under the action of magnetic field describes a circle in x z plane (perpendicular
to
B
) with
2m 2 qB
T or
Bq T m
Initially (t = 0) velocity was along x-axis. Therefore, magnetic force (Fm ) will be along positive
z-axis [F m q (vo B)] . Let it makes an angle with x-axis at time t, then = t.
qB
vx v0 cos t v0 cos t ... (ii) vz
m Z vo
qB
and vz v0 sin t v0 sin t ... (iii) vx
m
From Eqs. (i), (ii) and (iii),
v vxi→ vy →j vz k→ X
vo
qB vo qE E qB vo B
v v0 cos t t E vo sin m t v B
m vo m o
qB q qB vo B
or v cos t (vo ) t (E) sin t
m m m B
The path of particle will be a helix of increasing pitch. The axis of the helix is parallel to y-axis.
MAGNETISM
QUIZRR 19
Ex a m p l e 7
A positively charged particle is moving in the x – y plane along the x axis in the +x
direction. After the point P uniform E and B are present. The path of the particle becomes
curved and non-circular. Path remains in the x y plane. Which one of the following
combinations of E and B is possible.
(a) E 0; B bj→ c k→ (b) E a i→ ; B ck→ a i→
(c) E 0; B cj→ bk→ (d) E a i→ ; B ck→ a →j
Solution :
(a) and (c) are not possible as E 0 will create a circular path.
(d) option is also incorrect because in this case force has a z-component.
The positive and negative charges will move on circles of different radius as shown in the
diagram. The neutral particles neutrons will have zero deviation.
5. M a g n e t i c Fo r c e o f A Cu r r e n t Ca r ry i n g Co n d u c t o r
A charged particle in motion experiences a magnetic force in a
Fm
magnetic field. Similarly, a current carrying wire also experiences B
a force when placed in a magnetic field. This follows from the A
Vd i
fact that the current is a collection of many charged particles in
motion. Hence, the resultant force exerted by the field on the
I
wire is the vector sum of the individual forces exerted on all the
charged particles making up the current. The force exerted on
the particles is transmitted to the wire when the particles collide
with the atoms making up the wire.
MAGNETISM
20 QUIZRR
Suppose a conducting wire carrying a current i, is placed in a magnetic field B . The length of
the wire is l and area of cross-section is A. The free electrons drift with a speed vd opposite to the
direction of current. The magnetic force exerted on the electron is,
d F m e(vd B)
If n be the number of free electrons per unit volume of the wire, then total number of electrons
in volume Al of the wire are, nAl. Therefore, total force on the wire is,
F m e (nAl) (vd B)
If we denote the length l along the direction of the current by l , then the above equation
becomes
F m i ( l B)
where neAvd = i
B B
D
i
dl C i
(a) (b)
d F m = i ( d l B) ... (ii)
MAGNETISM
QUIZRR 21
To calculate the total force F m acting on there wire shown in figure, we integrate over the length
of the wire.
D
Fm i ( d l B) ... (iii)
A
Now let us consider two special cases involving Eq. (iii). In both cases, the magnetic field is taken
to be constant in magnitude and direction.
Case 1. A curved wire ACD as shown in Fig. (a) carries a current i and is located in a uniform
magnetic field B . Because the field is uniform, we can take B outside the integral in Eq. (iii)
and we obtain.
D
F m i dl B ... (iv)
A
D
But the quantity dl represents the vector sum of all length elements from A to D. From the
A
polygon law of vector addition the sum equals the vector directed from A to D. Thus,
F m i ( l B)
or we can write, F ACD F AD i (AD B) in uniform field.
Case 2. An arbitrarily shaped closed loop carrying a current i is placed in a uniform magnetic
field as shown in Fig. (b). We can again express the force acting on the loop in the form of Eq.
(iv), but this time we must take the vector sum of the length elements dl over the entire loop,
F m i ( dl ) B
Because the set of length elements forms a closed polygon, the vector sum must be zero.
Fm 0
Thus, the net magnetic force acting on any closed current loop in a uniform magnetic field is zero.
Fm
(v) The direction of F m can be given by FlemingÊs left hand rule
Thumb
According to this rule, the forefinger, the central finger and the
thumb of the left hand are stretched in such a way that they are
B
mutually perpendicular to each other. If the central finger shows Forefinger
i or l
Central finger
MAGNETISM
22 QUIZRR
the direction of current (or l ) and forefinger shows the direction of magnetic field (B) , then the
thumb will give the direction of magnetic force ( F m ).
(vi) The point of application of magnetic force is taken to be centre of Mass of the body.
Ex a m p l e 8
B
Find the magnetic force on the ring when B is directed as shown. i
Solution :
a
The force on an element will be given by
d F i (dl B)
since, B is always perpendicular
= idlB sin
2
to any element
F = iB (2a)
B i B
Ex a m p l e 9
The magnetic field is directed as shown. Then find the
magnetic force on the ring (Radius = a , current = i)
Solution : O
Resolve each of field lines, one perpendicular to the plane and the other along the plane of ring.
The force due to all of the Bcos component will cancel out and the net force will be downward
due to B sin component
Bcos Bcos
B
Bsin Bsin
F = i (B sin ) 2a
MAGNETISM
QUIZRR 23
6. T o r q u e Ex p e r ie n c e d b y a c u r re n t l o o p i n u n i f o r m f i e l d
Y
. B
l
D F1 C
b X
F2 O F2
F1
A B
(i) When the plane of the loop is placed perpendicular to the field lines :
Hence, net force = 0 and net couple = 0 (line of action of the forces coincide).
(ii) Now, the plane of the coil is rotated by an angle with the vertical
Net force still remain zero
But, now, a couple is formed due to length of wire AB and CD. F1
F (perpendicular distance)
= [iBl] b cos b
2
2
= iBlb sin
= i BA sin
F1
Note : In this case, the coil rotates about on axis passing through its CM, there is no translational
motion of the CM of the body.
torque experienced by the loop =
i (A B)
where A area of the loop
B magnetic field.
If N turns are present in the loop,
Ni (A B) or (M B)
where M magnetic moment of the loop = (i A N) .
MAGNETISM
24 QUIZRR
Importance of Number of Turns
If the coil has uniform binding and the size and the shape also remain uniform, then n turns
means n identical loops placed one after another very close to each other. {(ni) means effective
current}.
Area Vector
The area vector of a loop is perpendicular to the plane of loop. The magnitude of area vector is
equal to the area of loop.
Sign Convention
A M
A M
Anti clock wise clock wise current
7. Po t e n t i a l En e r g y
When the magnetic moment is perpendicular to the loop is said to be in the zero energy state.
M
B
M
When the loop rotates due to the torque given by magnetic
field. Then the rotation K.E. of the loop increases and the P.E.
decreases. B
Calculating the work done by Magnetic Field :
WB = . d
= MB sin d
2
WB = MB cos
MAGNETISM
QUIZRR 25
or U = M. B
Graph of U v/s :
U
B M
+MB
/
2 O
/ 3 /2
2
MB M
B
When M and B are in the same direction then the loop is in stable equilibrium state. And when
they are in opposite direction then the loop is in unstable equilibrium state.
Ca l c u l a t i n g M a g n e t i c Di p o l e M o m e n t v e c t o r i n d i f f e r e n t c a s e s :
Case 1 :
A1 A2
M = (A2 A1)
In this case if A1 = A2, then inside a uniform magnetic field the net torque and the net force acting
on the loop will be zero. The loop will be in the state of neutral equilibrium.
Case 2 :
Solonoid : Consider a solonoid of length l and radius R. Let n be the number of turns per unit
length
B
R
M
L
[M = (nL) i R2]
MAGNETISM
26 QUIZRR
Case 3 :
3 Dimensional Loop
y
R i L2 S R S
i i i
=
L1 L1 L1
M X
Q Q L2 M
L
i +
i M
L Q
P L i N
2
L
Z
P N
The 3 Dimensional loop can be divided into two 2 dimensional loop. When we join the segment
QM of both such loop their currents cancel out and we get the original loop.
M1 iL1 L2 k→
M2 iLL 2 →j
The net dipole moment vector of the 3 D loop will be M1 M2 .
Ca l c u l a t i n g T h e M a g n e t i c Di p o l e M o m e n t o f Ro t a t i n g Ch a r g e s
In the diagram shown the ring carrys uniformly distrubed charge Q and it is rotating with
constant angular velocity .
2
Time taken by the ring to complete one revolution will be .
w
+ + + +
With respect to an observer standing in the state of rest, the charge Q will
R
2
cross him in . + + + +
w
i
2 / w
Magnetic moment M R 2 i R 2
2 / w
MAGNETISM
QUIZRR 27
If the ring has uniformly distributed mass m, then the angular momentum of the ring will be
MR2W.
L = Iw
L = mR2
M Q
L 2m
*If mass and charge are both distributed in the same manner then the value of is always .
L 2M
M a g n e t i c Di p o l e m o m e n t o f A u n i f o r m ly c h a r g e d d i s c
Consider a disc of radius R carrying uniformly distributed charge Q on it. We have to find out
magnetic moment.
M Q
L 2m
M Q
mR2 2m
w
2
QwR
M
4
Ex a m p l e 1 0
A square current loop is placed in a uniform magnetic
A B
field as shown which makes an angle of 45 with the
x-axis. Mass of each wire side = m, and length l.
45
Find :
(i) Net torque acting on the loop and find its direction.
(ii) Angular displacement in the small time t. Neglect the D C
variation in torque during the entire time instant.
Solution :
y
B
B
2
i→ →j , M il2 ( k→) S R
x
= (M B) O
iBl2 → → P
= (i j ) k→ Q
2
MAGNETISM
28 QUIZRR
iBl2 → →
(i j )
= 2
2
| | = iBl
torque is constant,
2
iB l 3iB
=
I IQS 2m
1 ml 2 ml 2 2
IQS 4 ml 2
2 12 4 3
1 1 3i B 2 3i B 2
t2 t t
2 2 2m 4m
Note : In the question if do not neglect the variation in the torque, then,
(MBcos )
dw MB cos
w
d I
w
MB
(wdw) I cos d
0 0
w2 MB 3i B
sin w sin
2 I M
Ex a m p l e 1 1
A current carrying loop is placed on a horizontal plane as shown, and each side wire has
a length l and total mass of the frame = m. Find the minimum value of current, so that it
starts toppling. A horizontal magnetic field B exists the x-axis.
MAGNETISM
QUIZRR 29
x
B Bi→ O
l
mg = mg sin will go on decreasing.
2
In critical conditions
B mg
l
Bil 2 mg
2
mg
to just start topping, i
2 lB
Ex a m p l e 1 2
A light thin horizontal rod is suspended by 2 identical vertical strings. A non-conducting
uniform disc of radius R is mounted on the rod as its axle and has a uniform charge
distribution of charge Q along its edge circumference. The disc is rotated with constant
when a vertical magnetic field is switched on in the region. Find Bmax, so that the string
3To
does not break, if the breaking tension is . The initial tension in each is To.
2
Solution :
y
3T0 T0 x
2 2 O
+
+ +Q z
+ +
A B
+ + B
+ +
+
MAGNETISM
30 QUIZRR
During initial position
weight of the disc = 2 (tension in each string)
mg = 2T0 ... (1)
Current due to rotation of the disc
Qw
i M i (R 2 ) i→
2
(M B) iR 2B i→ →j iR 2Bk→
Thus, due to above torque developed, it (rod) tries to rotate about the point A. Thus, the tension
in the string A increases but that in string B decreases but the total sum 2T0 is constant, since
the rod is is equilibrium.
During critical conditions, (balancing the net torque) (Take axis about CM)
3T0 l 2
T0 l
2 2 iR B 2 2
l 2
T0 iR B
2
T0 l T0 l (2)
Bmax
2(iR 2 ) 2Q w R 2
T0 l
Bmax
QwR 2
Ex a m p l e 1 3
A circular current loop of radius R carrying current I is suspended by 2 vertical strings
with its plane making an angle 60 with the horizontal. Find the tension developed in each
string if the loop is in equilibrium.
Solution :
T2
Concept : Resolve the M vector into two components. T1
B
One along the B field [for which 0 ] and other
P
30
perpendicular to the B field. Q
MAGNETISM
QUIZRR 31
i (R2 )B
M B (Msin 30) B =
2
i R 2 B
T1(2 R cos 60)
2 = mg (R cos ) ... (2)
T1 = mg iRB
and T2 = iRB
8. B I OT -SA V A RT L A W
dl
r
i dl r P
dB o
4
| r |3
i
r position vector directed to the point from the current element.
0 T m
where 107
4 A
Note : Magnitude of d B is given by,
idl sin
| dB| 0
4 r2
| d B|= 0 at = 0 or 180 and maximum at = 90
When the charged particle is moving and the field created (magnetic) due to its motion is given
by
0 q ( v r )
dB
4 r3
(q is very small)
MAGNETISM
32 QUIZRR
dQ
i dl dl
dt
dl
i dl d Q dQ v q v
dt
7 Tm
= 4 10
A
A p p l ic a t i o n o f B i o t -Sa v a r t L a w
(A) Straight conductor.
According to Biot-Savart Law,
dl r
B 0
4 i r3
... (i)
A
Here, every element of the wire contributes to B in the same dy r
direction (which is into the plane of paper)
y
P
idl sin i dy sin
thus, B 0
4 r2
0
4 r2
I
0 i 0 i
B
4 d cos d B sin sin
4d
MAGNETISM
QUIZRR 33
(2) Point lying on the perpendicular of one end when the other end is infinitity long
1 0 , 2 =
2
0i / 2 0i
B sin
4 d 0 4 d P
A
(3) Magnetic field at a distance d from a infinitely long wire
1 2
2
P
d
0 i i
B sin 2 0
4 d 2d
2 A
0i i B
B sin 2 0 sin 1 sin 2 2
4 d 1 2d 1
P
Note : Students please take care while taking the angle 1 and 2. It is the angle made by PB
with axis per pendicular to AB
(B ) M a g n e t i c I n d u c t io n a t t h e Ce n t re d u e t o Ci rc u l a r l o o p
Consider an element dl of the ring. dl
0 i dl r
dB
4 r 3
R
i dl. R
dB 0
4 R3
0 i 0 i i
B
4 R 2 dl
4 R2
(2R) 0
2R
Note : The direction of the B field is perpendicular to the plane of the circular loop and given
by the right hand rule.
MAGNETISM
34 QUIZRR
o i
Magnetic field at the centre of a circular ring =
2R
o i
Magnetic field at the centre of a circular arc subtending angle at the centre =
2R 2
dl d Bcos
r
R
P(x,0,0) 2dBsin
Z
dl
d Bcos
We can see from the figure that the vertical component of magnetic fields (due to diameterically
opposite lengths dl) will cancel out.
And the net magnetic field will be along x-axis. i.e.
B = 2 d Bsin
R
20 idl r
=
4 r3
sin
0
R
20 R
= 4 3
r
dl
0
0 R 2i
B =
2 ( x2 R 2 )3 / 2
Note : The direction of the B field at the axial position is same throughout and given by the
right hand rule.
MAGNETISM
QUIZRR 35
ø0i
2R
x
A
0 iR 2
B =
2x3
o i R 2 i iR
2
= 2 o 2
4x3 4 x3
0 2M
B =
4 x3
0 2M
B = at an axial point
4 x3
o M
for any point P (r, ) is space B 3cos2 1
4
MAGNETISM
36 QUIZRR
PA T T ERN OF T H E FI EL D I N V A RI OU S CA SES
(1) Magnetic field line pattern due to a circular Due to two parallel current wires
loop. (i) When carrying current in same
direction
(2) Magnetic field lines are concentric circles (ii) When carrying current in opposite
centred at the wire and lying in the plane direction
perpendicular to the wire
Ex a m p l e 1 4
Find the force between the two circular loops in given arrangement, if :
(i) d >> R2 (ii) d >> R1, R2 for both
R1
R2
O1 O2
d
MAGNETISM
QUIZRR 37
Solution :
(1) Concept : For whichever circular loop, d >> R, replace that loop with a dipole.
So, in this case replace loop 2 with a magnetic dipole.
dB
F=M ,
dr
where
M i R 22
0 i R12
Now, B
2 (R12 + x2 )3 / 2
3
R12 x2
1/ 2
2 2x 2
iR 0i R1 x
Now, dB 0 1 2
3
=
R12 x2 R12 x2
dx 2 3 2 5/2
2
3 0 i R1 d
Fi R 22
3/2
2
R12 d 2
0 2M1
B1 =
4 x3
dB1 6M1
= 0 4
dx 4 x
dB 6M1M2
F M2 1 0
dx 4 d 4
MAGNETISM
38 QUIZRR
Force between two parallel current carrying wires.
i
d F i2 dl 0 1
2d
i1 d
dF 0i1i2 i2
dl 2d
i i
F 01 2
2d
The force is attraction between the wires carrying current in same direction.
The force is repulsive between the wire having the current in the opposite direction.
Ex a m p l e 1 5
(II)
A current carrying wire is bent in the form as shown. Field the
magnetic field at the centre O of the ring. (III)
Solution :
(I)
Let the wires be (I), (II) and ((III) as shown in figure.
Now, Magnetic field due to each of them will be
0 I
B1 = 4d 1 sin 4
0 I 1
=
4d 1
2
0I
B2 = 2R
0 I
B3 = 4d 1 sin 4 .
Net B B1 B 2 B 3
0 I 1 0 I 0 I 1
=
4 d 1 2R 4 d 1
2 2
0 I I
2 0
4 d 2R
=
MAGNETISM
QUIZRR 39
Ex a m p l e 1 6 O
An infinitely long arrangement of wire is laid out in such a way r
ar
that the radius of each of the circular arc increases Âa Ê times the a2r
previous. Find the B at 0.
Solution :
Note that the magnetic field due to straight segment is zero.
i 0 i 0 i
B1 0 . . , B2 , B3 . .
2r 2 2( ar) 2 2(a2 r) 2
Net B
0 i 0 i i i
= 0 0 ...
2r 2(ar) 2a r 2 3
2a r 2
0 i 1 1 1
= 1 ...
2r a a 2
a 3
2
a
= 0 .
2r ( a 1) 2
Ex a m p l e 1 7
r2
A circuit consists of 8 alternating arc of radius r 1 and r 2 as
shown. Find the B field at the centre of the circuit. Also find
the force experienced by the arc AC and straight wire CD of
the circuit due to current in the straight conductor. Each arc r1 C
subtends the same angle at the centre of the circuit. D
A
Solution :
Since each arc subtends the same angle at the centre, hence
2
8 4
i 4 i
B1 0 0
2r1 4 2 4 r1
MAGNETISM
40 QUIZRR
and magnetic field due to arcÊs of radius r2
i 4 i
B2 0 0
2r2 4 2 4 r2
Bnet B1 B2
i 1 1
0
4 r1 r2
(b) The force experienced by the arc AC = 0 {Reason : the magnetic field is in same direction as that
of current element}.
The force experienced by the wire CD
r2
0 i 2 0 i2 r
F 2x
dx
2
ln 2
r1
r1
Ex a m p l e 1 8
A current (circular) loop is bent along a diameter and placed mutually perpendicular. A
ch ar ged par t i cl e i s pr oj ect ed w i t h i n i t i al v el oci t y v0 i→ and has a charge q and mass m.
(a) Find the instantaneous force experienced by the charge placed at the origin.
(b) If an external field Bj→ is switched on in the region then find the net force experienced
by current loop.
Solution :
Field due to the circular arc ABC.
i 1
B1 0 →j
2R 2
Y
field due to the circular arc CDA
D
C
i
B2 0 i→
4R
X
B
0 i → →
Net B
4R
i j A
MAGNETISM
QUIZRR 41
F
= q (
v B)
→ 0 i i→ →j
= q v0 ( i )
4R
0 i
= q v0 [ k→ ]
4R
l eff 2R k→ B B0 →j
F 1 = I ( l B)
= I 2R ( k→) B( →j )
= 2 BRI i→
force on the arc CDA
l eff = 2R ( k→ )
F 2 = I 2R k Bo j
→ →
= I B0 2R ( i→ )
9. A m p e r e ’s Ci rc u i t a l L a w
Statement. The circulation of magnetic field along a closed loop is 0 times the total current
passing through normally, of the area of the closed loop.
B . dl 0Ienclosed
This law is analogous to Gauss Law already covered in electrostatics.
For Example
i5
The figure shows the current carrying i1
i4 i6
conductors enclosed by a closed loop.
i2 i3
i8
Then, B . dl 0 i1 i2 i3 i4 i5 i7
MAGNETISM
42 QUIZRR
Im port ant point s t o not e :
(a) The direction of circulation is assumed to be positive and accordingly any current in the
direction of circulation will be positive and the other negative.
(b) If the current is at an angle with the plane of the loop, then take its two components, one
parallel to the loop, and the other perpendicular to the loop. Consider only the normal
component in the formula.
(c) The above form of AmpereÊs Law is not valid when the electric field through the area of loop
is not constant.
In that case we need to apply the modified form of AmpereÊs Law, called Maxwell AmpereÊs
Law.
d E
B . dl 0 i dt
0
dE
The term 0 is called as displacement current.
dt
(d) Note that in the AmpereÊs formula, B is due to all the elements in the space, but I contains
the summation of the enclosed currents.
A p p l i c a t i o n o f A m p e r e ’s L a w i n c a l c u l a t i n g t h e f i e l d
(1) Field due to a infinitely long current carrying wire :
I
B . dl 0I
B (2r) 0 I
r
I
B 0
2r
Note that for a symmetrical field the direction of the E field is tangent to the path, whereas it
is radial in the case of E field,
(2) Field due to along cylindrical conductor of radius R.
(a) Field inside the conductor, r < R : Consider a closed circular loop at a distance r from
central axis.
I 2
Now, current enclosed by the loop is r
R 2
MAGNETISM
QUIZRR 43
B . dl 0 i
R
Ir 2 r
B. 2r 0
R2
Ir
B 0
2 R 2
It can also be written as B 0 ( j r)
2
B . dl 0I
R
I
B 0 r
2 r
B
ø0I
2R
r=R
R2 ( j r)
B 0
2
r
I
Where j is the charge density i.e. j
R 2
MAGNETISM
44 QUIZRR
Ex a m p l e 1 9
Find the magnetic field due to a cylindrical shell of inner and outer radii R and 2R with
j0r
current density j for
2R
R
(i) r <R
(ii) R < r < 2R
(iii) 2R < r 2r
(i) for r < R, B = 0
r
B(2r) = 0 j 2r dr dr
R
r
2R R
r
j0 r
B(2r) = 0 2R 2r dr
R
0 r3 R3 r R
3 3
B = 2Rr j0 0 j0
3 6R r
(iii) r > 2R
2R
B(2r) = 0 j 2r dr
R
3
B(2r) = 0 7R j0
R 3
7 0 R2 j0
B
6 r
MAGNETISM
QUIZRR 45
R
B
P
0 dIR 2
dB =
2 (x2 R 2 )3 / 2
0 i(ndx)R 2
dB = x = R tan
2 (x2 R 2 )3 / 2
dx = R sec2 d
i (nR 2 ) R sec 2 d
d B = 0 2R3 sec3
1
in
B = 0
2 cos d
2
B
ø0ni
0 in
B = sin 1
2 2
in
B 0 sin 1 sin 2
2
MAGNETISM
46 QUIZRR
For a long solenoid
1 = 2 = /2
B 0 in [2] 0 in
2
B = 0 in
This is the magnitude of B field not only at the axial position, but for all positions inside the
solenoid.
Note : Outside the solenoid, the magnitude of the B field is zero, for a closely wound solenoid.
Reason : The B field from the adjacent coilings cancel each other. Hence the net B field outside
the solenoid is zero.
T o f i n d m a g n e t ic m o m e n t o f c h a r g e d b o d i e s r o t a t i n g a b o u t a n a x i s .
M = d iA
Q
di = (rdr ) w A
R 2
R
Qw
M= R 2 (rdr) (r 2 ) r dr
0
Qw R 4 QwR 2
= A´
R 2 4
4
A l t e r n a t i v e Sh o r t k u t M e t h o d
q
Magnetic Moment = . {Mechanical moment (angular momentum)}
2m
Note : This formula is true always, provided the charge and the mass distribution are identical
about the axis.
Ex a m p l e 2 0
A uniformly charged non-conducting rod is placed in a horizontal plane at a distance l
from the origin and rotated about a vertical axis on the plane passing thorough the origin.
Find the magnetic induction at the origin and the magnetic moment of the rod.
MAGNETISM
QUIZRR 47
Solution :
Consider an element dx at the distance x. (dx) l
l
q
= charge per unit length (q,m)
l x
dq = (dx)
Note: The small element (dx) carrying the charge dq behaves a circular loop current.
0 di 0 dq
dB = (2 x)
2x 2
2l
0 w dx 0
B = 4 ln 2
x 4
l
M =
w
(di A) dq 2 x2
2l
= 2 x dx 6 (7l )
2 3
7Q wl 2
M = 6
Ex a m p l e 2 1
A non conducting spherical shell of radius R with uniform surface charge density , is
rotated with constant angular velocity about its diameterical axis. Then find the magnetic
induction at the centre.
Solution :
Consider an elemental ring of thickness Rd making an angle with the horizontal. Now, charge
on it.
dq w
di =
2
MAGNETISM
48 QUIZRR
0 di R 2 sin2
= d
2(R)3
0 2 3
B = 2 R (2) (2R ) sin d
0
0 R 4
=
2 3
20 w R
B =
3
M q q 2 2
M mR
L 2m 2m 3
qR 2
M
3
Ex a m p l e 2 2
In the previous example, find the magnetic induction at the centre if the sphere is solid.
Solution :
In the previous questions we have already found the magnetic field due to the thin spherical shell
2
B 0 R .
3
Use dr
MAGNETISM
QUIZRR 49
2 2
dB 0 w (dr) r 0 w rdr
3 3
R
2
B dB = 0 w rdr
3
0
2 0R 2
3 2
wR 2
B 0
3
Pro p e r t i e s o f M a g n e t i c L i n e s o f Fo r c e s :
(1) Magnetic lines of forces always form a closed loop.
(2) Magnetic lines of force may or may not be terminating at right angle to the surface of the
magnetic material.
(3) Magnetic lines of forces originate from the north pole and end at the south pole.
(4) The magnetic flux through a closed surface is always zero.
Reason : Since the magnetic lines form a closed loop, the number of
field lines coming out and join into the surface are same. Hence net flux
= 0.
B . dr 0
Ex a m p l e 2 3
A square current loop of side a is located in the same plane as that of a long current wire,
carrying a current I0. Find the net ampere force acting on the loop.
y
I0 I
S R
a P Q
y´
MAGNETISM
50 QUIZRR
Solution :
To find the force acting on the loop
(Note that the force in the arms PQ and RS are equal and opp. and hence they cancel each other).
Force on arm PS
I 0 II0
= (IaB) Ia 0 0 2 (towards the longwire)
2a
I II
Force on the arm QR = IaB Ia 0 0 0 0 (away from long wire)
22 a 4
Ex a m p l e 2 4
Two long conductors are placed in the same plane at a separation Âa Ê as shown. Find the
force of interaction per unit length between the two conductors.
Solution :
Consider a strip at a distance x from the long wire of the thick wire, which again behaves as if
a long wire.
I
di dx
b
dx
0 i1 i2 x
Force per unit length =
2x
( a b) ( a b)
0 I0 dr 0 I0 I dx I0
F 2x
2 b x
I
a a
I I ( a b) a
F 0 0 ln a b
2b
Ex a m p l e 2 5
A circular current loop is placed at a distance 2R from a long straight conducting wire as
shown in the same plane. Find the force of interaction between them.
Solution :
Consider an arc element d at an angular position of .
MAGNETISM
QUIZRR 51
d Fsin = 0
dF
Fnet d F cos I0 I
r
R d
(IRd) 0 I0 cos R
2 (2R R cos )
I
dF
2
I IR cos d
Fnet = 0 0
2R 2 cos
0
Ex a m p l e 2 6
Three long straight wires each carrying current (i) are located in X Y plane. The middle
wire is placed at x = 0, while other two conductors are placed at x = + d.
(i) Find the locus of points where net B is zero.
(ii) If the central wire is slightly displaced along the Z-axis and then released. Then find
the time period of small oscillations.
Solution :
i
Note that B field will be zero only in xy plane. i i
(i) B field will be zero at x = 0 x
x= d O x1 x=d
At any other point along x axis
0 i 0i 0 i
2x1 2( x1 d ) 2( d x1 )
1 1 1
x ( d x) ( d x)
1 1 1 2x
x ( d x) (d x) d x2
2
d2 x2 = 2x2
d 2 = 3x2
d
x required locus with (x = 0)
3
MAGNETISM
52 QUIZRR
(ii) Net force = 2F sin (towards origin) r
F F
d2 x
2F sin
dt2
O
d d
d2x 20 i2 2
0 i x
sin
dt2 (2 r ) 2
(d 2 x2 )1 / 2
d 2 x 0 i 2 x
if ( d x)
dt2 d 2
0 i2 d 2
T 2
d 2 0 i 2
1 0 . T OR OI D
Current in the toroid = i
Total no. of turns = N
a
B . dl 0 I net B=0 r
b
B (2r) = 0 (iN)
iN
B 0
2r
iN
B 0 for ( a r b)
2r
B
1
B
r
ø0iN
2a However, if r is very large compared with the cross sectional radius
of the toroid, the field is approximately uniform.
ø0iN
2b N
n = number of turns / length
2r
O a b r B = 0 ni
MAGNETISM
QUIZRR 53
0 i x
dvy ( qv0 cos ) q, m
v0
2 x0 x
or (origin)
(x0 x)
dt m
x0
dvy dx 0iq 1
. (v0 cos )
dx dt 2m x0 x
dx
Here Vx v0 cos
dt
dvy qi 1
0
dt 2m x0 x
v0 xmax
0 qi dx xmin
dvy 2m x0 x
0 0
v0 xmax
0 qi dx y
dvy 2m x0 x
0 0
l
origin x
qi x xmax v0
v0 0 ln 0
2m x0 xmax
x0
2mv0
0 qi
x0 xmax x0 e
Here, x0 xmax is the minimum distance between the particle and the wire.
2mv0
0 qi Ans.
xmin x0 e
MAGNETISM
54 QUIZRR
Ex a m p l e 2
Two long straight conductors are placed at y = + a along the z-axis and carry current I. A
current carrying conductor AC is placed along x-axis.
(i) Find the net force experienced by the current segment AC.
(ii) Also, find the force when the current in one wire is reversed.
Solution :
Consider an element of length dx at a distance x from origin.
Now, net B acting on the element (dx)
= 2d B cos y
0 I 0 x 0 I 0 ( x)
= 2 2r r
(a 2 x2 )
I a
dF = ( dB)net dx
l C dx A
x
x
0 I0 x I
dx a
( a2 x2 ) l dB dB
l
I I x dx
F 0 0 (d 2 x2 )
l
0
I I 1 1
0 0
l d d 2 l2
Ex a m p l e 3
A wire of mass 3 m and length 3l is bent in the shape of 3 sides of a square. It can freely
rotate about the axis XX´ about a vertical plane.
B
i
x x´
m, l m, l
m, l
MAGNETISM
QUIZRR 55
A uniform magnetic field is switched on in the region and a current i is passed through the
wire. Then, find :
(i) the deflection of the frame from the vertical plane in equilibrium.
Solution :
x x´
To complete the wire frame loop, replace the incomplete side with current wires having current
as shown. The torque of the single wire about the axis XX´ is zero. {Here we are taking about
the torque of the wire formed due to super position method}.
Torque due to the magnetic field B
(90 )
M B
M
= MB sin (90 ) = MB cos
= Bi (l2) cos .
{This is torque when the loop has rotated by an angle }
This torque is balanced by the torque due to the weight.
l l
wt = mg sin mg l sin mg sin
2 2
= 2 mg l sin
Equate these at the equilibrium position,
Bil 2 cos = 2mgl sin
Bil
tan =
2 mg
Bil
tan 1
2 mg
Ex a m p l e 4
A charge Q is uniformly distributed over the surface of a non-conducting circular disc. It
is now rotated with constant angular velocity w, about an axis perpendicular to the plane
of the disc passing through its centre.
MAGNETISM
56 QUIZRR
Then find the magnetic induction at a distance x from the centre on the axis.
Solution :
Consider a circular element at distance r from the centre of the disc.
z
Surface charge density =
R 2
y
dq = (2r) dr
w Qw (2r ) dr
di = ( dq)
2 2 2 R 2 r x
(x,0,0)
Qw (rdr )
=
R 2
0 di r 2
Hence, dB = {This is due to an elemental disc}
2 ( x2 r 2 )3 / 2
R
0 Qw r3 dr
=
2R 2
2
x2 )3 / 2
0 (r
0 Qw R 2 2 x2
Bnet = 2 x
2
2R 2
R x
2
Ex a m p l e 5
A charged particle +q of mass m is placed at a distance d from another charged particle
2q of mass 2 m in a uniform magnetic field B as shown in figure. If the particles are
projected towards each other with equal speeds v.
(a) Find the maximum value of projection speed vm so that the two particles do not
collide.
(b) Find the time after which collision occurs between the q,m 2q, 2m
particles if projection speed equals 2vm .
v v
MAGNETISM
QUIZRR 57
Solution :
(a) The particles will move in circular paths, as velocity vector is perpendicular to magnetic field. Time
2m
period of both the particles is same T . So, for collision not to take place,
Bq
r1 + r2 < d
mv 2mv Bqd
or d or v
Bq 2Bq 2m
Bqd Bqd
or vm
2m 2m
d
(b) From symmetry they collide at .
2
y
Bq
t t 2vm
m
q,m 2q, 2m x
x-component of velocity at time t is, 1 2
2vm 2vm
Bq
vx 2vm cos 2vm cos t d d
m 2 2
t t
d Bq
vx dt 2vm cos t dt
2 m
0 0
t
d 2mvm Bq
or sin m t
2 Bq 0
d 2mvm Bq
sin t
2 Bq m
Bqd
Substituting vm , we have
2m
Bq 1
sin t
m 2
MAGNETISM
58 QUIZRR
Bqt
m 6
m
or t Ans.
6Bq
Bq m
(c) At the time of collision, t 30
m 6Bq 6
vy
2vm 2vm
vx
q,m 2q, 2m q, 3m
2 2 2
Speed of combined mass v vx vy vm
3
2
(3m) vm
R
(3m) (v)
3 2 3 mvm
Bq Bq Bq
Bqd
Substituting, vm , we get R 3d Ans.
2m
MAGNETISM
ELECTRO
MAGNETIC
INDUCTION
QUIZRR 3
1. Introduction
Almost every modern device has electric circuits at its heart. We learned in the chapter of current
electricity that an electromagnetic force (emf) is required for a current to flow in a circuit. But for
most of the electric devices used in industry the source of emf is not a battery but an electrical
generating station. In these stations other forms of energy are converted into electric energy. For
example in a hydroelectric plant gravitational potential energy is converted into electric energy.
Similarly, in a nuclear plant nuclear energy is converted into electric energy.
But how this conversion is done ? Or what is the physics behind this? The branch of physics,
known as electromagnetic induction gives the answer to all these queries.
2 Magnetic Flux
Let us first discuss the concept of magnetic flux. We can represent any magnetic field by magnetic
lines. Unlike the electric lines of force it is wrong to call them magnetic lines of force, because
they do not point in the direction of the force on a charge. The force on a moving charged particle
is always perpendicular to the magnetic field (or magnetic field lines) at the particleÊs position.
The magnetic field at any point is tangential to the field line at that point. Where the field lines
are close, the magnitude of field is large, where the field lines are far apart, field magnitude is
small. Also, because the direction of B at each point is unique, field lines never intersect. Unlike
the electric field lines, magnetic lines form a closed loop.
Magnetic flux is the product of the average magnetic field times the perpendicular area that it
penetrates.
Magnetic field
Magnetic flux = B A
Area perpendicular
to magnetic field B
Consider an element of area dS on an arbitary shaped surface as shown in figure. If the magnetic
field at this element is B , the magnetic flux through the element is,
d B B d S Bd S cos
Here dS is a vector that is perpendicular to the surface and has a magnitude equal to the area
4. Faraday’s Law
This law states that „when the magnitude flux through a conducting loop changes with
time an emf is induced in the loop‰ and is given by
d
induced emf
dt
[-ve sign shows that the induced emf has the opposing tendency].
Some Points :
induced emf 1 d
(i) induced current =
R R dt
(ii) Total charge flown through the loop = idt R
(iii) As we have seen, induced emf is produced only when there is a change in magnetic flux
passing through a loop. The flux passing through the loop is given by,
= BS cos
This, flux can be changed in several ways :
(a) The magnitude of B can change with time. In the problems if magnetic field is given
a function of time, it implies that the magnitude field is changing. Thus,
B = B(t)
(b) The current producing the magnetic field can change with time. For this the current
can be given as a function of time. Hence,
i = i(t)
(c) The area enclosed by the loop can change with time.
This can be done by pulling a loop inside (or outside) a magnetic field. By doing so,
the area enclosed by loop (hatched area) can be changed.
(d) The angle between B and the normal to the loop can change with time.
This can be done by rotating a loop in a magnetic field.
d
loop, an emf is induced in every loop, thus the total induced emf e N
dt
Example 1
A 10 ohm coil of mean area 500 cm2 and having 1000 turns is held perpendicular to a
1
uniform field of 0.4 gauss. The coil is turned through 180Ĉ in s . Calculate
10
(a) the charge in flux
(b) the average induced emf
(c) average induced current
(d) the total induced charge
Solution :
(a) When the plane of a coil is perpendicular to the field, the angle between area S and field
B is 0 . So the flux linked with the coil,
B
S
B
S
2 = NSB cos 180
= NSB
= 2 1 = NSB (NSB)
= 2 NSB
i.e. || = 2 103 (500 10 4) (0.4 10 4)
= 4 mWb
(b) As in turning through 180 , i.e. in change of flux , the coil take (1/10)s.
eav 0.04
(c) I av 4 mA
R 10
5. Lenz’s Law
According to the Lenz law, a loop will always oppose the change in the flux passing through it.
This law can be used as an replacement for finding the direction of induced emf by faradayÊs law.
Example 2
A bar magnet is freely falling along the axis of a circular loop as shown in figure. State
whether its acceleration a is equal to, greater than or less than the acceleration due to
gravity g.
S
N
a
Solution :
a < g. Because to LenzÊs law, whatever may be the direction of induced current, it will oppose the
cause. Here the cause is, the free fall of magnet and so the induced current will oppose it and
the acceleration of magnet will be less than the acceleration due to gravity g. This can be
understood in a different manner. When the magnet falls downwards with its north pole downwards.
The magnetic field lines passing through the coil in the downward direction increase. Since the
induced current opposes this, the upper side of the coil will become north pole, so that field lines
of coilÊs magnetic field are upwards. Now like poles repel each other. Hence, a < g.
Thus, the from LenzÊs law, we can say that the direction of induced emf will be such as to oppose
the change that causes it i.e., the direction of induced increase in flux while it will be in same
direction if there is decrease in flux linked with the closed circuit. Analytically,
d
e
dt
Here, negative sign shows that if flux increases e is negative, i.e. is in the direction opposite to
that of applied emf and vice-versa.
Example 3
As shown in the figure, the rod AB is sliding on the frame with a velocity v in the direction
shown. Magnetic field exists with the direction into the plane of loop. Find the direction
of the induced emf and its magnitude.
A
V l
Solution : B
d
Find the magnitude of induced emf using E
dt
d dx B
Bl Blv
dt dt
l
d
B lv
dt
2nd Method :
Example 4
A circular conducting loop of radius a , is bent along its diameter and placed perpendicular
to uniform B field. Now, the upper half is rotated with content about the common
diameter. Then, find the current induced in the loop and plot it. (resistance = R)
Solution : X
=B
a 2
B
a 2
cos
2 2
Z
B a cos t
B
B a 2 2
=
2 2
Y
d Ba2
sin t
dt 2
B a2
induced current = sin t
2R
Ba2
2R
2
Example 5
A semi-circular conducting loop of radius a is rotated with constant . Find the emf induced
in the loop as function of time.
Solution :
1
BA B r 2
2
1 d Ba
2
d
B a2
dt 2 dt 2
induced emf
Ba
2
t
2 3
Example 6
Find the direction of the induced current in loop when a magnetic dipole NS is moved
towards it as shown (a << d).
d
N S
Solution :
2M
B 0 3 Since flux is increasing therefore, the current induced produces a field is such a
4 d
Example 7
dB
1 T / s . Resistance /unit length = 1 /m. Then 1m 1m 1m
dt
A 1m E m B
1 BA B (1)
D 1 F C
d1 d B
1 1V
dt dt
1 1 1
Now, flux in loop BCFE E = 1v E2 = 1
2
1
2 BA B
2 A E B
d 2 d B 1 1
2 V
dt dt 2 2
D 1 F C
1 1
E2 = i i I i i
2 2
1 1 2 1
= 3i I ...(2) 1
2
6. Motional Emf
e v B eE = 0
v B E
Now, the potential drop that exists along the ends MN of the conductor is given by
VM VN E. dl
v B . dl
VMN
VMN B. dl v
The direction of induced emf can be obtained using the FlemingÊs Right Hand Rule.
(ii) Resolve the velocity vector v in the plane of motion of the rod into two components, one
along the length of the rod and one perpendicular to the length of the rod.
(No contribution of the component of v parallel to the rod).
(iiii) Apply V B .v dl
Note : If B, v are mutually perpendicular.
Emf induced = B. dl v
= B. dl v
= B. leff v
Induced emf = B. leff v only when B is perpendicular to v
For closed loop, dl 0, emf induced = 0
Note that, when B and v are perpendicular and we have a closed loop then net emf induced in
the entire loop is zero, (but not zero for individual segments).
Example 8
Two rods making an angle 2 between them and of equal length fall vertically with velocity
v when uniform magnetic field B exists as shown. Find the emf induced across MN.
2 B
M V
N
Solution :
Emf induced O
= Bv (leff)
= Bv (2l sin)
l l
= 2 Bvl sin
VN VM = 2 Bvlsin 2l sin
M( ) N(+)
M Vsin N
M otional emf
Motional emf is equal to ratio of change of flux through the generated area.
= Blx
d
(Blx) Blv l
dt
Motional Emf induced along the length of a rod which is rotated in a magnet field.
Note that an emf is induced even in the absence of a magnetic field when a conductor is rotated
about an axis because of the centrifugal force experienced by each particle.
But the net situation after considering the effect of centrifugal force can be described as :
eE = m2x + eB (x)
m
E 2 x Bx
e
(m2x) + e(x)B
the actual potential drop across the ends is given as
r0 0 eE
V0 VM E.dx Edx
rm l O
l
m m2 l2
e
2
= x B x dx B
0 e 2
m m2 l 2
But, since is much smaller, the quantity is neglected and hence we derive the
e 2e
particular result.
Example 9
A disc is rotated with uniform angular velocity, . Find the potential drop between the
centre and a circumferencial point.
Solution :
In this case,
eE = m2x (m2x)
A
m O e
E 2 x
e
R
m 2 m2 R 2
Potential drop = e
x dx
2e
0
Example 10
A conductor falls freely under gravity in the region of magnetic field created by a straight
current wire. Find the emf induced.
x
dx
M N
i0 a
Solution :
l a
0 i0 iv l a
VN VM v dx 0 0 l n
a
2x 2 a
0 i0 l a
emf induced will be a function of time =
2
l n gt
a
General result for emf induced when any shape of conductor is rotating in magnetic field.
rF
Note that the dl vector will be directed to the tangent
B
direction of the shape of conductor at that point
dl
The velocity vector (r) is directed perpendicular to the r
(r)
vector
Let the angle between them be . ri
rf
B 2
= B rdr rf ri2
2
ri
Thus to find the emf induced between any points of any shape of conductor rotating about an
axis.
B 2
V rf ri2
i f 2
Direction from (ri rf) when B is directed (and is clockwise)
ri rf
1.
a i
e = Bvl
l R V R
r
In the figure shown, we can replace the moving rod ab by a battery of emf Bvl with the
positive terminal at a and the negative terminal at b. The resistance r of the rod ab may
be treated as the internal resistance of the battery. Hence, the current in the circuit is,
e
i
Rr
Bvl
or I
Rr
2. Induction and energy transfers : In the figure shown, if you move the conductor ab with
a constant velocity v, the current in the circuit is,
Bvl
i (r = 0)
R
a
B2 l 2 v
A magnetic force Fm ilB acts on the
R F, v
conductor in opposite direction of velocity. So, l R Fm
to move the conductor with a constant velocity
v an equal and opposite force F has to be applied b
in the conductor. Thus,
B2 l 2 v
F Fm
R
B2 l 2 v2
The rate at which work is done by the applied force is, Papplied Fv
R
2
2 Bvl B2 I2 v2
Pdissipated i R R
2 R
This is just equal to the rate at which work is done by the applied force.
3. Motional emf is not a different kind of induced emf, it is exactly the induced emf described
by FaradayÊs law, in the case in which there is a conductor moving in a magnetic field.
d B
e is best applied to problems in which there is a changing flux through a closed
dt
loop while e = Bvl is applied to problem in which a conductor moves through a magnetic
field. Note that, if a conductor is moving in a magnetic field but circuit is not closed, then
only P. D will be asked between two points of the conductor. If the circuit is closed, then
current will be asked in the circuit. Now let us see how these two are similar.
d a i=0 d a
d a
l v v v
c c b c
b b
x
x
(a) (b) (c)
Refer figure (a) : A loop abcd enters a uniform magnetic field B at constant speed v.
Using FaradayÊs equation
dB d BS d Blx dx
e Bl Blv
dt dt dt dt
For the direction of current, we can use LenzÊs law. As the loop enters the field magnetic
field passing through the loop increases, hence, current in the loop is anticlockwise. ( IN).
From the theory of motional emf, e = Bvl and using right hand rule, current in the circuit
d B
is anticlockwise. Thus, we see that e and e = Bvl give the same result. In the similar
dt
manner we can show that current in the loop in figure (b) is zero and in figure (c) it is
clockwise.
Example 11
Two parallel rails with negligible resistance are 10.0 cm apart. They are connected by a 5.0
resistor. The circuit also contains two metal rods having resistances of 10.0 and 15.0
along the rails. The rods are pulled away from the resistor at constant speeds 4.0 m/s and
2.0 m/s respectively. A uniform magnetic field of magnitude 0.01 T is applied perpendicular
to the plane of the rails. Determine the current in the 5.0 resistor.
b f
10.0 d 5.0
How to proceed : Here two conductors are moving in uniform magnetic field. So, we will use
the motional approach. The rod ab will act as a source of emf.
3
e1 = Bvl = (0.01) (4.0) (0.1) = 4 10 V
and internal resistance r1 = 10.0
Similarly, rod ef will also act as a source of emf.
3
e2 = (0.01) (2.0) (0.1) = 2.0 10 V
and internal resistance r2 = 15.0
From right hand rule we can see that
Vb > Va and Ve > Vf
Now either by applying KirchhoffÊs laws or applying principle of superposition (discussed in the
chapter of current electricity) we can find current through 5.0 resistor. We will here use the
superposition principle. You solve it by using KirchhoffÊs laws.
Solution :
3 3
In the figures R = 5.0 , r1 = 10 , r2 = 15 , e1 = 4 10 V and e2 = 2 10 V
i i´2
e2 e2
r1 r1 R r1 R r2
R +
r2 i2 i1 r2 i´2
e1 e1
Rr1 10 5 55
Refer figure (b) : Net resistance of the circuit = r2 15
R r1 10 5 3
e2 2 103 6
current i 10 3 A
net resistance 55 / 3 55
r1 10 6
current through R, i1 = i 103 A
R r1 10 5 55
4
= 103 A
55
4
= mA
55
Rr2 5 15 55
Refer figure (c) : Net resistance of the circuit = r1 10
R r2 5 15 4
e1 4 10 3 16
current i´ 103 A
net resistance 55 / 4 55
r 15 16 3
current through R, i1´ 2 i´ 10 A
R r2 15 5 55
12
= mA
55
8
i1´ i1 mA from d to c. Ans.
55
Example 12
A conducting rod shown in figure of mass m and length l moves on two frictionless parallel
rails in the presence of a uniform magnetic field directed into the page. The rod is given
an initial velocity v0 to the right and is released at t = 0. Find as a function of time,
(a) the velocity of the rod.
(b) the induced current and
a
(c) the magnitude of the induced emf
How to proceed : The initial velocity will produce
an induced emf and hence, an induced current in l R v0
the circuit. The current carrying wire will now
experience a magnetic force Fm in opposite b
direction of its velocity. The force will retard the motion of the conductor. Thus,
Solution :
(a) Let v be the velocity of the rod at time t.
Bvl
Current in the circuit at this moment is, i ...(i) i a B
R
Here negative sign denotes that the force is to the left and retards the motion. This is the
only horizontal force acting on the bar, and hence, NewtonÊs second law applied to motion
in horizontal direction gives,
dv B2 l 2
m Fm v
dt R
dv B2 l2
dt
v mR
Integrating this equation using the initial condition that, v = v0 at t = 0, we find that
v dv B2 l 2 t
v0 v
mR 0 dt
t/
Solving this equation, we find that v = v0e ...(ii) Ans.
mR
where
B2 l 2
This expression indicates that the velocity of the rod decreases exponentially with time
under the action of the magnetic retarding force.
Bvl
(b) i
R
Blv0 t /
Substituting the value of v from Eq. (ii), we get i e Ans.
R
t/
(c) e = iR = Blv0e Ans.
i and e both decrease exponentially with time. v-t, i-t and e-t graphs are as shown in Fig.
v i e
v0 Blv0 Blv0
R
t t t
Example 13
A circular loop is being rotated about the point O. Find the induced emf about (1) AC (2)
CD (3) AD (4) OA.
C B
A D
O
Solution :
(i) OA
B2 2 B rA 2 R
VA V0
rA r02
[2R 2 ] BR 2
r 0
2 2
(ii) AC
B2 2 B 2
VC VA
rC rA2
4R 2R 2 BR 2
2 2
(iii) CD
B 2 B2
VD VC rD rC2 2R 2 4R 2 BR 2
2 2
VC VD = + BR2
(iv) OA
VD VO = + BR2
Note that net emf induced in loop = 0
B = f(t)
(i) For r R
consider a circular region at an axial distance r.
d
From FaradayÊs Law, potential energy in the loop =
dt
r
d
dl dt B. A
E.
dB 2
E (2r) = r
dt
dB r
E r R
dt 2
(ii) for r R
d
E. dl dt
B R 2
dB 2
r
E (2r) =
dt R
R
dB R
2
E
dt 2r
Some important results for linear conductor placed in the time-varying field
(i) r R
Emf induced in the conductor
B
= E. dl
O
= E dx cos r
d E
1 dB
= 2 dt r dx cos l2 l 1
l1
1 dB
{d = r cos } d
2 dt dx
l2
(ii) r>R
= E dx cos
1 dB R2 d
= dx
2 dt r r
al
1 dB 2 dx
= 2 dt
R d
a d 2
x2
al
1 dB 2 1 x
= R d tan 1
2 dt d d a
1 dB 2 1 a l 1 a
= R
2 dt tan d tan d
1 dB 2 1 x
Emf induced in the conductor = R tan
2 dt d
dq E a [about centre O]
= Q E a
Due to this sudden torque, an angular impulse is imparted which is equal to change in
angular momentum.
dt I
Q Ea dt ma
2
1 dB 2
Q 2 dt
a dt ma 2
Qa 2
2
2
B ma
QB
2ma
2 Q 2 Ba
Magnetic moment = iA = Q a2 Q QB a
2 2 2ma 4m
2
only the expression for E will change E = 1 dB R
2 dt r
dt I
1 dB R 2
2 dt a a dt = Iw
Q
1
2
dB
QR 2 dt ma 2
dt
1
2
QR 2 B ma2
QR 2 B
2ma 2
Q 2 Q 2 R 2 B Q 2 R 2 B
Magnetic moment = iA a
2 2. 2m 4 m
8. Self Inductance
When the current in a loop varies with time, the magnetic flux through the conducting loop
change with time and an emf is induced in the loop. This is called self induction.
B i
and B i
i
d di
e L
dt dt
Find the self inductance of a circular coil of radius R and number of turns N.
Solution :
We will find the self inductance for very small circular coil for which the B field can be almost
assumed to be constant inside the ring.
i N
B= 0
2R
i N
0
2R
2
R N
N2 R
0 i
2
L 0 RN2
2
N2 R 2 N
0 i here n is the number of turns per unit length n
l l
0 N2 R 2
L
l 2
2
0 n R l
9. Mutual Inductance
When the current in the primary loop varies with time the magnetic flux through the secondary
loop changes with time hence an emf is induced across the secondary coil. This is called mutual
induction.
i(t)
O1 O2
s ip
d s dip
M
dt dt
Example 16
(a) Two concentric coplanar circular coils of radii a and b are having turns N1 and N2 (a
<< b) are given. Find the mutual inductance of the pair.
(b) If the outer loop is connected with a source of emf V = (4 + 2.5t), then find the induced
current in smaller loop. (a = 10ă2m), b = 0.1 m, = 10ă4 /m
Solution :
(a) s = (BA) N1
0 i p N2
= 2b a N1
2
ip, N2
N1
0 a2 N1 N 2
ip O
= 2b
b
a 2 N1 N 2
M= 0
2b
4 2.5t
(b) current in outer loop =
R0
d d d M
induced emf in inner loop = M iP 2.5t 4
dt dt dt R 0
M 2.5
=
R0
M 2.5
current in secondary =
R0 R i
Example 17
Two coplanar concentric square loops are placed as shown with (a << b). Find the mutual
inductance of the system.
Solution :
Magnetic field at the centre due to each wire
0 i 4
= sin b
4 (b) / 2
4 a
0 i 2 0 i 2
= 4 (2)
b 2 2b
0 i 2 2 2 0 i
= 2b 4 b
2 2 0 i p 2
s a
b
a2
M= 2 2 0
b
Example 18
Two circular coils are placed co-axially at a distance d >> R2. Hence, find the mutual
inductance of system.
Solution :
0 ip R12
B
3/2
2 R12 d 2
ip R1
0 i p R12
R
2 R2
2
2 3/2
2 R12 d
0 R 22 R12
M
3/2
2 R12 d 2
Example 19
Find the mutual inductance of l length of two co-axial solenoid of radii r 1 and r 2 and turns
n 1 and n 2.
Solution : ip
B = ø0 n2 ip n2
n1
0 n2 i p r12 n1l
r1 r2
Example 20
Two concentric coplanar circular loops are placed as shown. A constant current i 0 is
flowing in the outer loop. Now the smaller loop is rotated with constant about the
common diameter. Then, find the current induced in the smaller loop.
Solution :
i
s 0 a2 cos t
2b
d 0 i0 a 2
s sin t r1
dt 2b
b
1 0 i a 2
i sin t
R 2b
di
(Va Vb) = L
dt
Leq
L1 L2
i i
V = (V1 + V2)
di di di
L eq L1 L 2
dt
dt
dt
Leq = L1 + L2
di di1 di2 i1 L1
dt dt dt
i
V V V 1 1 1
L eq L1 L 2 L eq L1 L 2
It is given by M= L1 L2
Leq = L1 + L2 2M
(a)
L1 di di
M dt L2 di di
M dt
dt dt
Leq = L1 + L2 + 2M
(b)
Leq = L1 + L2 2M
Note : When the current is increasing in inductor, the induced emf opposes the flow of current
and hence is induced in the opposite direction of current flow.
di L R
L iR
dt
di 1
dt
iR L i
is t
di 1
dt
iR L 0
i0 E
ln
isR R t
i0 R L
Rt
isR i0R e L
i t i0 is i0 1 et /
di is i0 t /
dt
e
It is the time in which the current through an induction would acquire its steady state value, had
it been increasing with initial constant rate.
is i0
di
dt t 0
i(t) di
dt
(is i0)
is
(is i0) di
tan = = dt
t=0
t
i0 O t=
t
O t=
i (t) t
di R
i
dt
L0
i0
Rt
i t i0 e L
Rt
di i L
e
dt
Time constant is also defined as the time is which the current through an inductor would completely
vanish, had it been decreasing with initial constant rate.
di Decay Rate of Decay Circuit
dt
i
i0 t=
t
O
t
O t=
Tip : The time constant for LR circuit can also be found by the equivalent resistance across the
terminals of inductor, just like in RC circuits.
di
Now, to find the dt , find the potential drop across the terminal of the inductor and use
t0
di
L V
dt
di V
dt L
di
(iii) Among four unknown i0, is and dt t 0 , find any three and use the relation
di i i
s 0
dt t 0
Example 21
Solution :
(i)
R2 = 2 L = 400 mH
=2
R1 = 2 R 1
+
Em = 12 V
L = 400 mH
R2 = 2
is = 6A, i0 = 0,
L 400
200 ms = 0.2s
R 2
t
i t 6 i e / 0.2
5t
i(t) = 6(i e )
5t
current through the battery = 6 + 6 (1 e )
di
= L 0.4 30 e
dt
5t
12 e5t
L 0.4
0.1s
R eq 4
is = 0, i0 = 6A
i t i0 is i0 1 et /
t
1 e0.1
= 6 + ( 6)
10t
i(t) = 6e current through resistor R1
1 1
=
2
L imax 0.4 36 7.2 J
2 2
R1 = R2
heat dissipated through each resistor is equal
= 3.6 J
Example 22
Find the current through the inductor and the capacitor at any time t.
C
R1 R2
R3 L
E S
Solution :
The above circuit can be split up in two individual circuits of LR and RC only
C
R1 R3 L
R2
E E
RC Circuit :
q0 0, qs R2 C
R1 R2
R R
i0 1 2 C
R1 R1 R 2
t /
i t e
R1
RL Circuit :
L
i0 0, is ,
R3 R3
3
R t
i t 1 e L
R3
Example 23
In the circuit shown, the switch is swapped from 1 to 2. Then find the current through the
battery at time t.
L1
1 2
R L2
Solution :
During steady state at t = 0, the current in the circuit is, i . Just as the switch is swapped
R
from 1 to 2, the inductor L2 gets disconnected from the circuit.
Now, we need to find the initial current through L1, so for this we apply
Flux Conservation
(L1 + L2)i = L1 i0
L L2 L2
i0 1 i 1
L1 L1 R
ELECTRO MAGNETIC INDUCTION
40 QUIZRR
Now is
R
L1
R
i t i0 is i0 1 e t /
Rt
L2 L 2 L1
= 1 L R R L 1 e
1 1
Rt
1 L 2 e L1
= R L1
Example 24
Find the current through each inductor at any time t, after S is closed at t = 0
i1 L1
i2 L2
Solution :
L1 Ler
Leq.
i1
L2
i2
E (R)
di di di
L1 1 L2 2 L eq
dt dt dt
L eq L eq
i1 i, i2 i
L1 L2
i = (i1 + i2)
L L2
1
Rt
L1 L2
i 1 e
R
L L2 Rt
1
L2 L1L2
i1 R 1 e
L1 L 2
di
L iR [Multiply with (idt)]
dt
i dt Li di i
2
Rdt
E
1
Wb Li2 Heat dissipated in resistor
2
1 2
Energy stored in an inductor = Li
2
u 1 Li2
Energy density =
Al 2 Al
2 2
1 0 r1 n l i
2
=
2 r12 l
=
1
2
0 n 2 i2
1 1 B2
= 2 0 ni 2
2
0 0
1 B2
Energy stored per unit volume in the magnetic field =
2 0
15. LC Oscillations
q0 q0 i q q
L L
dq
(At t = 0) i
dt
At any time t,
q di d2q
L L 2
C dt dt
d2q q
dt2
LC
1
, T = 2 LC
LC
3
cos = 0, , q > 0
2 2 2
q t A sin t q0 cos t
2
i(t) = q0 sin t
= q0 cos t
2
q0
q(t)
2
t
q0
i(t)
Another important concept to be kept in mind is that the sum of energy stored across
capacitor and the inductor is constant in ideal LC oscillation.
q2 1 2 1 2 1 q2
Li const. = Limax 0
2C 2 2 2 C
di
If the initial charge on the capacitor is 200 øC then find the current and at a time when
dt
q = 100 øC.
Solution :
To solve question on LC oscillation, it is always easier to use energy conservation.
1 2 1 q2 1 q02
Li
2 2 C 2 C
At q = 100 øC
1 2 1 100 1 200
2 2
Li
2 2 C 2 C
di
To find , just differentiate the above equation.
dt
di q
iL i 0
dt C
di q
L
dt C
di q
dt LC
Example 26
At t = 0, S1 is closed, but S2 remains open. At t LC , S
1 E
2
L
is opened and simultaneously S2 is closed.
(a) Find the charge on the capacitor and the current in
S1 S2
the inductor at t LC just before opening S .
1
2
(b) Then, find the maximum current in the inductor after closing S2.
Solution :
When the switch S1 is closed at t = 0
At any time t, let the current through the circuit be i, apply KVL
i
q di
L
c dt C
d2i E
i
Differentiate wrt time, C L 2 0 L
dt
d 2i i 1 S1
dt2
LC
LC
(At any time t)
di
0 A cos
dt L L
A = 0
L
i t sin t
L
1 t
q(t) cos t 0 2 1 cos t
L L
LC 1
q t 1 cos t 1 cos t
2 L L LC
CE
i t sin t
LC
Now, at t LC , S1 is opened and S2 is closed.
2
CE
q LC CE , i LC
2 2 LC
which the charge and current through the capacitor and the inductor just before opening
S1 .
CE CE
CE 2 2
CE CE CE
2 2
i i
1 2 1 E2
Limax 2 C Initial energy before closing S2
2 2 4
1 C2 E2 1 C2 E2 2
= L CE
2 C 2 LC
1 2 3CE2
i
2 max 4
L
3CE2 3C
imax E
2L 2L
MISCELLANEOUS EXAMPLES
Example 1
dv dv
m Kx B CBl dt l
dt
dv
dt m CB l Kx
2 2
dv
dt
m B 2 l 2 C Kx
d2x Kx
K
dt2
m CB2 l2
m CB2 l2
2 m CB2 l2
= 2
K
ELECTRO MAGNETIC INDUCTION
48 QUIZRR
(b) Solution of the above DE is given by
x(t) = x0 sin (t)
v(t) = (x0) cos (t)
Maximum velocity acquired by the connector
K
= x0 x0
m CB2 l2
K
= CBl x0
m CB2 l2
Example 2 B
In the arrangement shown, the connector starts
b
sliding along two parallel smooth inclined rails g
mass= m
connecting with an uncharged capacitor. Find
(a) the velocity of the connector when it reaches l
the bottom of the parallel rails.
(b) Time required to reach the bottom of the
incline.
Solution :
(Bsin)
First of all, find the comp. of B field in plane of the
incline which is B sin v and B cos (r to plane) Bcos
F = (Bil)
Only perpendicular comp. of B field induces emf
Current flows in the circuit due to motional emf. Hence,
consider any general time t, when the current in the mgsin
circuit is i and charge on the capacitor is q.
q = CE = C (Bvl)
+q q
dq dv dv
i CBl C B cos l
dt dt dt
dv
Now, write the force eqn. for the conductor, m mg sin i Bl
dt
dv dv
m mg sin B x cos B cos l C cos l
dt dt
dv
m
dt
dv
mg sin B2 l 2 C cos2
dt
a m B2 l2 C cos2 = mg sin
mg sin
a
m B2 l 2 C cos2
To find the velocity, when the conductor reaches the bottom, use
(a) v2 = 2al
2 mg sin l
v
m B2 l 2 C cos2
1 2
(b) To find the required time, at l
2
2l
t
a
Example 3
In the previous question, find the velocity and the time when the rails are not smooth.
(take friction coeff. = ø)
Solution :
In this case, we need to just consider the friction force also in the force eqn.
N = mg cos + (B sin ) i l
fk = øN = ø[mg cos + (B sin ) i l]
dv
m mg sin mg cos Bsin il B cos il
dt
mg sin cos
a
m sin cos B2 l 2 Ccos
2l
v 2al and time t
a
Example 4
B
In the arrangement shown, the connector rod of mass m, length r
(m,r,R)
and and resistance R, is rotating in a horizontal plane about 0
semi circular rail of radius r. An initial velocity of w 0 was imparted
at t = 0 Find
(a) the angular velocity of the rod as the function of time t and
angular displacement () R0
(b) the current through the resistance as a function of time t.
Solution :
Due to induced emf, a current flows in the circuit. +
i
induced emf B r 2
resistance 2 R R0 R0
Magnetic torque experienced by the rod
r Bi r 2
B Bi dx x
0 2
Write the torque eqn. on the rod
x dx
mr 2 Bi r 2 B2 r 4
B
4 R R0
3 2
To find w ()
mr 2 d B2 r 4
3 d 4 R R 0
3B2 r 2
d
4 m R R0 0
d
0
3B2 r 2
0
4m R R0
O 4m0(R+R0)
=
3B2r2
mr 2 d B2 r 4
To find (t),
3 dt 4 R R 0
t
d 3B2
4 m R R dt
00
0 0
3B2 t
l n
0 4 m R R 0
f
O
3B2t
4 m R R0
0 e
3B2t
Br2 4 m R R0
i 0 e
2 R R0
Example 5
Bl2
Emf induced across its ends = 2
t
R
i t 1 e L
R
Bl2
R
t
i t 1 e L
2R
(i)
(ii) after the steady state is reached, the current through the inductor becomes constant. Hence,
due to this a constant magnetic torque is clockwise direction is created.
To keeping rotating rod with constants, an anti-clockwise torque must be applied to balance
t and B.
l Bil2
req.= mg cos wt
2 2
l B 2 l 4
= mg cos t
2 4R
Example 6
In the circuit shown a straight conductor of mass, length and resistance m, l and R can
smoothly move along two parallel conducting rails. Now, the switch is closed at time t = 0,
Then find the charge on capacitor and the velocity of the conductor at any time t.
S R
B
m, l
Solution :
At any instant, let the current through the circuit be i.
Apply KVL,
i
q +
C iR + Bvl ...(1)
V
Write the force eqn. for the conductor
dv ml
m Bil ...(2)
dt
t t E
m dv B il dt
0 0
mv = Blq
Blq
v ...(3)
m
q dq B2 l 2 q
R
C dt m
dq 1 B l 1
2 2
q
dt R R m C
dq
Let a b q
dt
q t t
dq
dt
a bq 0
0
a
q t 1 e bt
b
1 B2 l2 1
t
R m C
q t . 1 e
B2 l2 1
m C
mCE
maximum charge on the capacitor = B l C m
2 2
1 B2l2 1
t
Bl Bl mCE R m C
v q 1 e
m
m CB2 l2 m
Example 7
The system is released from rest at t = 0. Find the velocity of the connector as a function
of displacement x, from the initial position.
m, l
L B
Solution :
Now, when the system is released from rest, the mass m fall freely and the rod, acquires a velocity
v which induces an emf in the circuit
di
L Bvl ...(1)
dt
dv mg Bil g Bil
a ...(2)
dt 2m 2 2m
L di Bl vdt Li = Blx
Bl
i = x
L
dv g B2 l2
v
2mL
x
dx 2
v x x
g B2 l2
vdv 2 0
dx
2 mL x dx
0 0
v2 gx B2 l2 x2
2 2 2 mL 2
B2 l 2 2
v gx x
2mL
dx B2 l 2
gx
2mL
x
dt
dx
B2 l2
dt , integrate to get answer
gx x
2mL
di dv g B il
L Bvl,
dt
dt 2 2m
Differentiate eqn. (2) again with time
d 2v Bl di
dt2
2m dt
di 2m d v
2
L L
dt Bl dt2
d 2 v B2 l2
2 v
dt 2mL
B2 l 2
v(t) = A sin (t + )
2 mL
To find A and ,
At t = 0
v(0) = 0, = 0,
At t = 0
g g
a 0 A cos
2 2
g
A , 0
2
g 2mL
v t sin t
2 B2 l2
Example 8 R1
R2
Solution :
dv
m mg Bil ...(1)
dt
induced emf 1 1
i Bvl ...(2)
resistor R
1 R 2
dv B2 l 2
g v
dt
m Re q
B2l2
v t
mg R e q
1 e
t
m Re q
B2 l 2
Bvl 2 Bvl 2
P1 and P2
R1 R2
1 P1 P2
Re q
Bvl 2
mg B2 l2 vt2 P1 P2
Vt vt
B2 l2 P1 P2 mg
mg
Bvl P1 P2
Bl
P1 P2
v
mg
x
Example 9
E F
A frame of wire (EFGH) falls freely under gravity in a region
B a
where B field varies as B = B0 y k→
a a H
a G
y
ELECTRO MAGNETIC INDUCTION
58 QUIZRR
Find
(i) t h e i n du ced cu r r en t i n t h e l oop at t i m e t.
(ii) the total ampere force acting on the loop
(iii) the velocity of the loop as a function of time t. v (t) = ?
the resistance of loop is R.
Solution :
x
Note that no emf is induced along the lengths EH and FG. F1
Net induced emf in the loop E y F
+ B1
= (B2va B1va)
= (B2 B1) va F2
G
B2
H
B0 B y
= y a 0 va B0 va
a a y
B0 va
(b) Current in the loop =
R
B0 va B02 a 2
= B0 a
R
v
R
dv B2 a 2
(c) m mg v
dt R
v t
dv
B2 a 2
dt
0 g v 0
mR
B2 a 2
t
gmR mR
v t 2 2 1 e
B0 a
du d K 2
i R
dt dt
dv B2 v2 l2
mgv mv
dt R
dv B2 va 2
g
dt mR
v t
dv
B2 a 2
dt
0 g v 0
mR
B2 a2
t
gmR mR
v t 2 2 1 e
B0 a
Example 10 P
Solution :
i1 + i
Induced emf = (Bvl) Q 2
dv
i2 CB l ...(3)
dt
i1 + i2
dv
F Bil m
dt
dv Bvl dv
m F Bl i1 i2 F Bl CB l
dt R dt
dv v dv
m F B2 l2 C
dt R dt
dv B2 l 2 v
m B l C F
2 2
dt R
v t
dv dt
22
0F B l v
0 m B l C2 2
R
B2 l 2 v t
(i) v t
FR
1e
R m B2l2C
2 2
B l v
Bl
(ii) Hence, current through resistor = v t
R
dK d q2
F. v 2
i R +
dt dt 2C
2
dv Blv q dv
Fv mv R R C lBl dt
dt
dv B l 2
2 2
2 2 dv
Fv mv v CB l v
dt R dt
B2 l 2
F
R dv
v m CB2 l 2
dt
dv dt
B2 l 2
F
R
v m CB l
2 2
Integrate to get same result.
Example 11
A simple pendulum consists of a small conducting ball of mass m and a light conducting
rod of length l. The pendulum oscillates with angular amplitude 0 in a vertical plane about
a horizontal axis passing through O such that the
O
ball remains always just in contact with a metallic
S
strip AD bent into a circular arc of radius l as
shown in figure. In the space, a uniform magnetic B
field of induction B normal to plane of oscillation
exists in the space. At time t = 0 when the ball is
at its lowest position and moving towards right, A D
the switch S is closed. Neglecting self inductance
of the circuit calculate external torque required
to keep the pendulum oscillating as before. Assume C
that 0 is small.
Solution :
2
At time t = angular position of the rod is, 0 sin t ...(i)
T
O i
l
Here T 2
g
Fm
Differentiating Eq. (i) w.r.t. time, the angular speed
of the rod at this instant is, i
A D
d 20 2t Q
= cos P +
dt T T t =t
t=0
q
1
Vo VQ = V B l 2
2
0 l 2 B 2t
= T cos T
From right hand rule we can see that V0 VQ. This is also the P.D across the capacitor. Hence,
charge stored in the capacitor at this instant is,
0 l2 CB 2t
q = CV = cos
T T
As the rod move towards D, its decreases. Hence, q will decrease. Thus, current in the circuit
is anticlockwise. This current is given by
dq 2B2 0 l 2C 2t
i = sin
dt T 2 T
In the rod it points from O to Q. Magnetic force on the rod due to this current is,
22 0 B2 l3 C 2t
Fm = il B sin
T 2 T
This force acts on the rod in the direction shown in figure at the centre of the rod and perpendicular
to it. Hence anticlockwise torque due to this force about point O is,
l 2 0 B2 l4 C 2t
= Fm 2
sin
2 T T
So we will have to apply, this much torque in clockwise directions, to keep the rod rotating as
before.
2 0 B2 l4 C 2t
Thus, clockwise = 2
sin Ans.
T T
Example 12
Li = constant L
i
Solution :
(a) At time t = 0, steady state current in the circuit is i0 = E/R. Suddenly L reduces to nL
i0 E
(n < 1), so current in the circuit at time t = 0 will increase to . Let i be the current
n nR
at time t.
Applying KirchhoffÊs loop rule we have,
nL R
di
E nL iR = 0
dt i
di 1
= dt
E iR nL
E
1 t
di 1
E iR = nL dt
i0
/n 0
i0 t / L
Solving this equation, we get i = i0 i0 e Ans.
n
E nL
Here i0 = and L i
R R
i0
n
i0
From the i-t equation, we get at t = 0 and i = i0 at t =
n
t
i0
Note : At t = 0, current in the circuit is . Current in the circuit in steady state will be again
n
i0
i0. So it will decrease exponentially from to i0. From the i-t graph the equation will
n
be formed without doing any calculation.
i i i
i0
n i0
i0
n
+
i0 i0
t t t
i t / L
i i0 0 i0 e
n
1 1
W = U f Ui L f i2f Li ii2
2 2
2 2
1 E 1 E
= nL 2 L R
2 n R
2
1 E 1
= L 1
2 R n
2
1 E 1 n
L
2 R n
=
Example 13
A loop is formed by two parallel conductors connected by a solenoid with inductance L and
a conducting rod of mass m which can freely (without friction) slide over the conductor.
The conductors are located in a horizontal
plane in a uniform vertical magnetic field B.
The distance between the conductors is l. At
the moment t = 0, the rod is imparted an V0 x
initial velocity v0 directed to the right. Find
m,l
the law of its motion x (t) if the electric
resistance of the loop is negligible.
Solution :
Let at any instant of time, velocity of the rod is v towards right. The current in the circuit is i.
In the figure, a i d
Va Vb = Vd Vc
di dx dx
or L = Bvl Bl as v dt
dt dt Fm V
B2 l 2
Fm = ilB x ...(ii)
L
Since, this force is in opposite direction of v , so from NewtonÊs second law we can write,
d2 x B2 l 2
m 2 = x
dt
L
d2x B2 l 2
or 2 = x
dt mL
d2 x
= 2x
dt2
Bl
we have, =
mL
Therefore, the rod will oscillate simple harmonically with angular frequency Bil mL . At time
t = 0, rod was at x = 0 and it was moving towards positive x-axis. Hence, x-t equation of the rod
is
x = A sin t ...(iii)
dx
To find A, we use the fact that t = 0, v or have a value v0. Hence,
dt
dx
= v = Acost
dt
or A = v0 (at t = 0)
v0
or A =
Substituting in Eq. (iii), we have
v0 Bl
x= sin t, where = Ans.
mL
Alternate method of finding A : At x = A, v = 0, i.e., whole of its kinetic energy is converted
into magnetic energy. Thus
1 2 1
Li = mv02
2 2
Substituting value of i from Eq. (i), with x = A, we have
2
Bl
L A = mv02
L
mL v
or A = v0 0
Bl
Bl
as =
mL
Example 14
l
A long insulating cylinder of radius R and length l carries a
uniformly distributed surface charge q. A string is wound
around the cylinder from which a block of mass m hangs. The
mass is free to move downwards and can rotate the cylinder.
Neglecting the moment of inertia of the cylinder, calculate the m
acceleration of the block.
How to Proceed : If there were no charge on the cylinder, tension would had been zero (as
moment of inertia of the cylinder is zero, so no torque is required for its rotation). In this case
acceleration of the block would had been g downwards. But due to rotating charge on the cylinder
a magnetic field will appear. Further due to angular acceleration of the cylinder this magnetic
field will be changing. The change in magnetic field (and hence, the magnetic flux) producdes an
electric field. So the cylinder will experience a torque due to electric force on the charge over it.
Hence, tension in the string will not be zero, or acceleration of block will be less than g.
Solution :
Let a be the acceleration of the block downwards and T the tension in the string. T
The equation of motion of the block is,
mg T = ma ...(i) a
If the system is released from rest, the velocity of block at time t would be,
v = at
So, angular velocity of cylinder at this instant is,
mg
v at
=
R R
at
or frequency is, f =
2 2R
The surface charge q of the cylinder spins with it. The effective current over the surface of the
qat
cylinder is, i = qf =
2R
The cylinder can now be treated as a solenoid with one turn, carrying a current i. The number
1
of turns per unit length is, n =
l
0 qat
The magnetic field on its axis is, B = ø0ni =
2Rl
dB 0 qa
=
dt 2Rl
R d B 0 qa
of this electric field is, E =
2 dt 4 l
This electric field interacts with the charge on the surface of the cylinder and causes a torque.
The magnitude of this torque is,
0 q2 Ra
= qER =
4 l
From LenzÊs law direction of this torque is to oppose the motion of the cylinder. Further, moment
of inertia of the cylinder is zero. Hence, net torque on the cylinder should be zero.
So, torque of tension should balance this torque, or
0 q2 Ra
TR =
4 l
0 q2 a
or T =
4 l
0 q2 a
mg = ma
4l
g
or a = Ans.
q2
1 0
4 ml
ELECTRICITY
1
1 .1 I N T RODU CT I ON
An electrical circuit consists of some active and passive elements. The active elements such as a
battery or a cell supply electric energy to the circuit. On the contrary, passive elements consume or store
the electric energy. The basic passive elements are resistor, capacitor and inductor.
A resistor opposes the flow of current through it and if some current is passed by maintaining a
potential difference across it, some energy is dissipated in the form of heat. A capacitor is a device which
stores energy in the form of electric potential energy. It opposes the variations in voltage. An inductor
opposes the variations in current. It does not oppose the steady current through it. Fundamentally,
electric circuits are a means for conveying energy from one place to another. As charged particles move
within a circuit, electric potential energy is transferred from a source (such as a battery or a cell) to
a device in which that energy is either stored or converted to another form, into sound in a stereo
system or into heat and light in a toaster or light bulb. Electric circuits are useful because they allow
energy to be transported without any moving parts (other than the moving charged particles themselves).
In this chapter we will study the basic properties of electric currents. WeÊll study the properties of
batteries and how they cause current and energy transfer in a circuit. In this analysis we will use the
concepts of current, potential difference, resistance and electromotive force.
1 .2 EL ECT RI C CU RREN T
Electric current is the rate of charge flow past a given point in an electric circuit, measured in
Coulombs/second which is named Amperes. In most DC electric circuits, it can be assumed that the
resistance to current flow is a constant so that the current in the circuit is related to voltage and
resistance by Ohm’s law. The standard abbreviations for the units are 1 A = 1C/s.
At room temperature, the free electrons in a conductor move
randomly with speeds on the order 105 m/s. Since the motion of
the electrons is random, there is no net charge flow in any
direction. For any imaginary plane passing through the conductor,
the number of electrons crossing the plane in one direction is
equal to the number crossing it in the other direction.
Random motion of free electrons
in a conductor
vd
E
V
+ ă
ELECTRICITY
4 QUIZRR
When a constant potential difference V is applied between the ends of the conductor an electric field
E is produced inside the conductor. The conduction electrons within the conductor are then subjected
to a force ă e E and move in the direction of increasing potential. However, this force does not cause
the electrons to move faster and faster. Instead, a conduction electron accelerates through a very small
distance (about 5 10ă8m) and then collides with one of the atoms of the conductor. Each collision
transfers some of the electronÊs kinetic energy to the atoms, resulting in an increase in the vibrational
energy (and therefore in the temperature) of the atoms. Because of the electric and collision forces a
conduction electron moves slowly along the conductor or we can say it acquires a drift velocity vd in
the direction opposite to E , in addition to its random motion.
This chaotic motion of the electrons can be understood by a simple
model. Let us imagine that the conduction electrons form an Âelectron gasÊ
vd
whose particles are moving randomly, while the gas as a whole is moving
slowly down the conductor with a drift velocity vd in the direction opposite
to E . ItÊs interesting to note that the magnitude of the drift velocity is of
the order of 10ă4 m/s, or about 109 times smaller than the average speed
of the electrons between collisions. When averaged over time, the random
motion of an electron due to its collisions does not result in any net displacement. Hence, we can ignore
this random motion and treat the electrons as though they were all moving down the conductor at the
drift velocity vd.
The net movement of charge through a conductor is represented by electric current (or just current)
I. Current is defined quantitatively in terms of the rate at which net charge passes through a cross-
section area of the conductor.
dq dq
Thus, I or i
dt dt
We can have following two concepts of current, as in the case of velocity, instantaneous current and
average current.
dq
instantaneous current = = current at any point of time
dt
q
and average current =
t
Hence-forth unless otherwise referred to, current would signify instantaneous current. By convention,
the direction of the current is assumed to be that in which positive charge moves. In the SI system,
the unit of current is ampere (A).
1A = 1 C/s
ELECTRICITY
QUIZRR 5
I m p o r t a n t Po i n t s t o Re m e m b e r
1. The current is the same for all cross-sections of a conductor of non-uniform cross-section. Similar
to the water flow, charge flows faster where the conductor is smaller in cross-section and slower
where the conductor is larger in cross-section, so that charge rate remains unchanged.
2. Electric current is very similar to water current, consider a water tank kept at some height and
a pipe is connected to the water tank. The rate of flow of water through the pipe depends on
the height of the tank. As the level of water in the tank falls, the rate of flow of water through
the pipe also gets reduced. Just as the flow of water depends on the height of the tank or the
level of water in the tank, the flow for current through a wire depends on the potential
difference between the end points of the wire. As the potential difference is changed the current
will change. For example, during the discharging of a capacitor potential difference and hence,
the current in the circuit decreases with time. To maintain a constant current in a circuit a
constant potential difference will have to be maintained and for this a battery is used which
maintains a constant potential difference in a circuit.
3. Though conventionally a direction is associated with current
i1
(opposite to the motion of electrons), it is not a vector as the
direction merely represents the sense of charge flow and not
a true direction. Further, current does not obey the law of i = i1 + i2
i2
parallelogram of vectors, i.e. if two current i1 and i2 reaches
a point we always have i = i1 + i2 whatever be the angle
between i2 and i2.
4. According to its magnitude and direction current is usually divided into two types :
(i) Direct current (DC) : If the magnitude and direction of current does not vary with time,
it is said to be direct current (DC). Cell, battery or DC dynamo are its sources.
(ii) Alternating current (AC) : If a current is periodic (with constant amplitude) and has
half cycle positive and half negative, it is said to be alternating current. (AC). AC dynamo
is the source of it.
5. If a charge q revolves in a circle with frequency f, the equivalent current,
i = qf
ELECTRICITY
6 QUIZRR
Ex a m p l e 1
A wire carries a current of 2.0 A. What is the charge that has flowed through its cross-
section in 1.0s. How many electrons does this correspond to ?
q
Solution. i =
t
q 2.0
n =
e 1.6 10 –19
Ex a m p l e 2
The current in a wire varies with time according to the relation
A
i = (3.0A) + 2.0 t
s
(a) How many coulombs of charge pas a cross-section of the wire in the time interval
between t = 0 and t = 4.0s ?
(b) What constant current would transport the same charge in the same time interval?
dq
Solution. (a) i =
dt
q 4
0
dq =
0
idt
4
q= (3 2 t ) dt
0
= [3 t t 2 ]40 [12 16 ]
= 28 C Ans.
q 28
(b) i =
t 4
=7 Ans.
1 .4 . CU RREN T DEN SI T Y (J ) :
The current flowing per unit normal area at a point is called current density at that point.
It is a vector quantity whose direction is taken in the direction of current at the point.
ELECTRICITY
QUIZRR 7
ds
di
i j = ds cos ...(1)
ds cos
Ex a m p le 3
Find the current through a cylindrical wire whose current density varies as :
r
j = j0 1 +
R
Solution. Consider this circular ring cross-section of wire, at distance r from axis.
R
r
i = j .d s j0 1 R 2 r dr
0
i
R
R 2 R 2 10 j0 .R 2
= 2 j0
2 3 6
1 .5 . DRI FT V EL OCI T Y (V d )
Let n = number of particles per unit volume
e = charge of each particle x
Then, dq = (Adx) ne
dq dx
= nA e
dt dt
dx
dx
i = n A (Vd ) e vd
dt
ELECTRICITY
8 QUIZRR
So m e Po i n t s :
.. .. . . .
1. In the absence of the external fields, inside the conductor all free
.
elect r ons move under r andom mot ion wit h V rms =
3RT
M
and .. . . . ..
follow a zig-zag path.
2. In the absence of electric field, the total number of free electrons passing through a cross-section
from left to right is equal to total no. of free electrons passing from right to left. Hence, there
is no current through the conductor.
3. But when an external field is applied inside the conductor, all the free electrons experience an
electric force and acquires a velocity called drift velocity, and hence a current is established in
the conductor.
dV eE
= ă
dt m
V – eE
t
Vrms dv = m dt
0
eE
V ă Vrms = – t
m
To find the drift velocity take the average value for a large number of electrons
eE
V ă Vrms = – t
m
[ Vrms is a randomly oriented vector, hence Vrms = 0]
eE
Vd =
m
eE
Vd =
m
ELECTRICITY
QUIZRR 9
ne2 E
j=
m
ne2
j= E = E
m
ne2
where = electric conductivity =
m
1 m
Resitivity =
n e2
1 .7 . M OB I LI T Y
Drift velocity acquired per unit electric field is called mobility.
Vd e
= E m = ne
m
e2
n = n0eăEg/kT
where Eg = is the difference valence of energy level between valence and conduction band.
ELECTRICITY
10 QUIZRR
Eg
= eă kT = e
0
2 . OH M ’S L A WS
Since, j = E
i V
=
A l
l
V = i
A
STATEMENT : The current flowing through a conductor is directly proportional to the potential
difference applied across the ends provided the
i.e V I V (V i)
or V = IR
Where R is the resistance of the conductor.
tan R
O
i
The materials following the ohmÊs law are called ohmic or linear conductors.
Ex a m p l e 4
v
T2
T1
0
i
Which of the temperature T1, T2 are greater for the V v/s i graph for a given ohmic
resistor.
ELECTRICITY
QUIZRR 11
Solution. (R slope)
R 2 > R1
as, (T2 > T1)
3 .1 . RESI ST A N CE
The electrical resistance of an object is a measure of its opposition to the passage of a steady electric
current. An object of uniform cross section will have a resistance proportional to its length and inversely
proportional to its cross-sectional area, and proportional to the resistivity of the material.
Electrical resistance shares some conceptual parallels with the mechanical notion of friction. The SI
unit of electrical resistance is the ohm, symbol . ResistanceÊs reciprocal quantity is electrical conductance
measured in siemens, symbol S.
The resistance of a resistive object determines the amount of current through the object for a given
potential difference across the object, in accordance with Ohm’s law :
V
I =
R
where,
R is the resistance of the object, measured in ohm, equivalent to J.s/C2
V is the potential difference across the object, measured in volts.
I is the current through the object, measured in amperes.
For a wide variety of materials and conditions, the electrical resistance does not depend on the
amount of current through or the amount of voltage across the object, meaning that the resistance R
is constant for the given temperature. Therefore, the resistance of an object can be defined as the ratio
of voltage to current :
V
R=
I
3 .2 . DC RESI ST A N CE
The resistance R of a conductor of uniform cross section can be computed as
l
R=
A
where,
l is the length of the conductor, measured in meters.
A is the cross-sectional area, measured in square meters.
is the electrical resistivity (also called specific electrical resistance) of the material, measured in
Ohm meter. Resistivity is a measure of the materialÊs ability to oppose electric current.
ELECTRICITY
12 QUIZRR
3 .3 . RESI ST IV I T Y
Electrical resistivity (also known as specific electrical resistance or volume resistivity) is a measure
of how strongly a material opposes the flow of electric current. A low resistivity indicates a material that
readily allows the movement of electrical charge. The SI unit of electrical resistivity is the ohm meter
(m).
De f i n i t i o n s
E
=
J
where,
is the static resistivity (measured in volts metres per ampere, V m/A);
E is the magnitude of the electric field (measured in volts per metre, V/m);
J is the magnitude of the current density (measured in amperes per square metre, A/m2).
The electrical resistivity can also be given by,
A
= R
where
is the static resistivity (measured on ohm-metres, m);
R is the electrical resistance of a uniform specimen of the material (measured in ohm,) ;
is the length of the piece of material (measured in metres, m) :
A is the cross-sectional area of the specimen (measured in square metres, m2).
Finally, electrical resistivity is also defined as the inverse of the conductivity (sigma), of the
material, or
1
ELECTRICITY
QUIZRR 13
)
3 .4 . T EM PERA T U RE COEFFICI EN T OF RESI ST IV I T Y (
The fractional change in resistivity for unit 1ĈC or 1 K change of temperature is called temperature
coefficient of resistivity ().
=
(Average value)
= 0 (1 + ) R = R0 (1 + )
d
= d (Absolute value)
d
= 0 d
0
ln =
0
d
0
is + ve for a conductor.
In general,
Variation of :
Conductor Semi-Conductor
O T T
O
ELECTRICITY
14 QUIZRR
O
TC
Ex a m p l e 5
A steady current flows through a conductor of a non-uniform cross-section. Then, which
of the following quantities are constant ?
(i) current, (ii) current density, (iii) electric field, (iii) drift velocity.
Solution. (i) current remains constant.
(ii) current density is not constant
S2
1 S1
j
S
i
(iii) Electric field is again not constant
j = E
(iv) drift velocity is also not constant
eE
Vd =
m
Ex a m p l e 6
A potential drop V is applied across the ends of a conductor of length . Then, find the
change in the drift velocity if :
(i) V is doubled
(ii) is doubted
(iii) diameter is doubled.
Solution. (i) When V is doubled : V = E (V E)
E is also doubled, and hence Vd is also doubled.
(ii) is doubled : E is halfed
Vd is also halved
(iii) Diameter is doubled : Vd remain unchanged.
ELECTRICITY
QUIZRR 15
Ex a m p le 7
Find the resistance of a cylindrical hollow tube which is connected between,
(a) inner and outer surfaces of the cylindrical shell
(b) when connected between the ends a and b.
.b
a .
Solution. (a) When connected between inner and outer surfaces :
In this case, consider co-axial cylinders of thickness dr and all such cylinders are in series
b dr b
R= ln dr
a 2 rl 2 a
rr
l d x
d R 0
b2 – a 2 dx
R= l a
. x .
b
b2 – a 2
Alternative methods : (Using the concept of current density)
j = E
E = j
– dV
= j Apply this
dx
dV I I I
– a s j
(a) dr = 2 r A 2 rl
Vb
b I
– dV = a 2 r l d r
Va
ELECTRICITY
16 QUIZRR
I b
(Va ă Vb) = ln
2 l a
n (b / a )
R=
2 l
dV I
(b) – = P
dx (b2 – a 2 )
Vb I
. .
d V = (b 2 – a 2 )
dx a
– 0
b
Va
I
(Va ă Vb) =
(b 2 – a 2 )
W
Thus, E=
q H L
ELECTRICITY
QUIZRR 17
The name electromotive force is misleading in the sense that emf is not a force it is work done per
unit charge. The SI unit of emf is J/C or V (1 V = 1 J/C).
I m p o r t a n t Po i n t s t o Re m e m b e r
This force drives positive ions towards positive terminal and negative + ă
ions towards negative terminal. As positive charge accumulates on +q
.
Ć
anode and negative charge on cathode a potential difference and
Fn Fe
hence an electric field E is developed from anode to cathode. This
electric field exerts an electrostatic force F e = q E on the ions. This
force is opposite to that of Fn . In equilibrium (steady state)
Fn = Fe
and no further accumulation of charge takes place.
2. The emf is the work done per unit charge by the battery force Fn E
which is non electrostatic in nature.
3. When the terminals of the battery are connected by a conducting
wire, an electric field is developed in the wire in the direction shown
+ ă
in Fig. The free electrons in the wire move in the opposite direction
and enter the battery at positive terminal. Some electrons are
withdrawn from the negative terminal. Thus, potential difference
and hence, Fe decreases in magnitude while Fn remains the same. Thus, there is a net force on
the positive charge towards the positive terminal. With this the positive charge rush towards
positive terminal and negative charge rush towards negative terminal. Thus, the potential
difference between positive and negative terminal is maintained.
4. During the charging of a battery current is driven into a battery in the reverse direction. In
such a case positive charge enters the battery at the positive terminal and leaves the battery
from the negative terminal.
5. In an open circuit the electrostatic force F e and non-electrostatic force Fn on a charge q are
equal.
6. The work done by Fn on a positive charge q moving from negative terminal to positive terminal
is qE, where E is the emf of the battery.
ELECTRICITY
18 QUIZRR
Ex a m p l e 8
Why canÊt we use a capacitor as a battery ? What is the difference between a battery and
a capacitor ?
Solution. A capacitor cannot maintain constant potential difference across a circuit. After the capacitor
discharges completely no current is obtained. A capacitor discharges in a very short interval of time.
On the other hand, a battery can maintain a constant potential difference for a long period of time.
A battery also gets discharged, but discharging may take years also.
8V 14V 10V E
<
i i
<
i
< <
R
2 (c)
4
(a)
(b)
net em f E
i n et
net resist a nce Rn et
For example :
In Fig. (a ) : Net emf is 8V and net resistance is 2. Therefore,
8
i = 4A
2
4
Therefore, i = 1A
4
In Fig. (c) : We have n cells each of emf E. Of these polarity of m cells (where n > 2m) is reversed.
Then net emf in the circuit is (n ă 2m) E and resistance of the circuit is R. Therefore,
(n – 2 m ) E
i
R
ELECTRICITY
QUIZRR 19
5 . Re s i st o rs i n Se r ie s a n d i n Pa r a l le l
A
A
R1 V1
V
In Series :
V R
R2 V2
B B
Figure represents a circuit consisting of a source of emf and two resistors connected in series. We
are interested in finding the resistance R is of the network lying between A and B. That is, what single
equivalent resistor R would have the same resistance as the two resistors linked together.
Because there is only one path for electric current to follow, i must have the same value everywhere
in the circuit. The potential difference between A and B is V. This potential difference must somehow
be divided into two parts V1 and V2 as shown, subject to the condition
V = V1 + V2
= iR 1 + iR 2
or V = i (R1 + R2) ...(i)
Let R be the equivalent resistance between A and B, then
V = iR ... (ii)
From Eqs. (i) and (ii),
R = R1 + R2 for resistors in series
This result can be readily extended to a network consisting of n resistors in series.
R = R1 + R2 +.....+Rn
i
i
V i2
In parallel : V
i1 R R2 R
1
ELECTRICITY
20 QUIZRR
In Fig. the two resistors are connected in parallel. The voltage drop across each resistor is equal to
the source voltage V. The current i, however, divides into two branches, which carry currents i1 and
i 2.
i = i1 + i2 (iii)
If R be the equivalent resistance, then
V V
i= , i1 R
R 1
V
and i2 =
R2
1 1 1
(for resistors in parallel)
R R1 R2
This result can also be extended to a network consisting of n resistors in parallel. The result is,
1 1 1 1
.....
R R1 R2 Rn
Ex a m p l e 9
Compute the equivalent resistance of the network shown in Fig. and find the current i
drawn from the battery.
1/3V
i i
10
6
2
Solution. The 10 and 2 resistances are in parallel. Their equivalent resistance is,
1 1 1
= 1
V i
R 10 2 3
5
5
or R= 6 3
3
23 5 23
Now this 6 and resistances are in series and their equivalent resistance is 6
3 3 3
ELECTRICITY
QUIZRR 21
1
V
i 3
23
3
net emf 1
i =
net resistance 23
1
= A Ans.
23
4 .1 .1 . K I RCH H OFF’S L A WS
Many electric circuits cannot be reduced to simple series-parallel combinations. For example, two
circuits that cannot be so broken down are shown
A
C D
R1 E1 B
A B R1 R2 R3 R4
R2 E2
D C
E2 E3
E1
R3
F E F
E G
i H
R5
(a) (b)
However, it is always possible to analyze such circuits by applying two rules, devised by KirchhoffÊs
in 1845 and 1846 when he was still a student.
First here are two terms that we will use often.
ELECTRICITY
22 QUIZRR
J un c t ion
A junction in a circuit is a point where three or more conductors meet. Junctions are also called node
or branch points.
For example, in figure (a) points D and C are junctions. Similarly in figure (b) points B and F
junction.
Lo op
A loop is any closed conducting path. For example, in figure (a) ABCDA, DCEFD and ABEFA loops.
Similarly, in figure (b), CBFEC, BDGFB are loops.
I = 0
Note the convention we have chosen here : current flowing into the node are taken to be negative,
and currents flowing out of the node are positive. It should not really matter which you choose to be
the positive or negative current, as long as you stay consistent.
4 .1 .3 . K I RCH H OFF’S V OL T A GE L A W
KirchhoffÊs Voltage Law (or KirchhoffÊs Loop Rule) is a result of the electrostatic field being
conservative. It states that the total voltage around a closed loop must be zero. If this were not the case,
then when we travel around a closed loop, the voltages would be indefinite. So
ELECTRICITY
QUIZRR 23
V = 0
In Figure the total voltage around loop 1 should sum to zero, as does the total voltage in loop 2.
Furthermore, the loop which consists of the outer part of the circuit (the path ABCD) should also sum
to zero.
A . .
B
loop 1
D
. loop 2
.
C
Around a closed loop, the total voltage should be zero
We can adopt the convention that potential gains (i.e. going from lower to higher potential, such
as with an emf source) is taken to be positive. Potential losses (such as across a resistor) will then be
negative.
In applying the loop rule, we need sign conventions. First assume a direction for the current in each
branch of the circuit. Then starting at any point in the circuit, we imagine, travelling around a loop
adding emfÊs and iR terms as we come to them.
When we travel through a source in the direction from ăto +, the emf is considered to be positive,
when we travel from + to ă, the emf is considered to be negative.
.
A E
.
B
.
A E
.
B
Path Path
V = VB ă VA = + E V = VB ă VA = ă E
When we travel through a resistor in the same direction as the assumed current, the iR term is
negative because the current goes in the direction of decreasing potential. When we travel through a
resistor in the direction opposite to the assumed current, the iR term is positive because this represents
a rise of potential.
R i R i
Path Path
V = VB – VA = – iR V = VB – VA = + iR
Note : It is advised to write H (for higher potential) and L (for lower potential) across all the
batteries and resistances of the loop under consideration while using the loop law. Then writeămoving
from H to L and + for L to H. Across a battery write H on positive terminal and L on negative terminal.
Across a resistance keep in mind the fact that current always flows from higher potential (H) to lower
ELECTRICITY
24 QUIZRR
L H
A D
R2
Ex a m p l e 1 0
Find currents in different branches of the electric circuit shown in figure.
A 4 B 2
C
2V 4V 6V
F 2 E 4 D
How to Proceed : In this problem there are three wires EFAB, BE and BCDE. Therefore we have
three unknowns i1, i2 and i3. So, we require three equations. One equation will be obtained by
applying KirchhoffÊs junction law (either at B or at E) and the remaining two equations, we get from
the second law (loop law). We can make three loops ABEFA, ACDFA and BCDEB. But we have to
choose any two of them. Further, we can choose any arbitrary direction of i1, i2 and i3.
Solution. Applying KirchhoffÊs first law (junction law) at junction B,
i1 = i2 + i3 ...(i)
Applying KirchhoffÊs second law in loop 1 (ABEFA),
4 4
A H L H L C
i1 i3
i2
H L H
1 4V 2 6V
2V L H L
i1 L H i3 L H
F D
2 4
ELECTRICITY
QUIZRR 25
8
i2 = A
3
5
i3 = – A
3
Here, negative sign of i3 implies that current i3 is in opposite direction of what we have assumed.
Ex a m p l e 1 1
In previous example find the potential difference between points F and C.
How to Proceed : To find the potential difference between any two points of a circuit
you have to reach from one point to the other via any path of the circuit. It is advisable
to choose a path in which we come across the least number of resistors preferably a path
which has no resistance.
Solution. Let us reach from F to C via A and B,
VF + 2 ă 4i1 ă 2i3 = VC
VF ă VC = 4i1 + 2i3 ă 2
5
Substituting, i1 = 1A and i3 = – A, we get
3
4
VF ă VC = – volts Ans.
3
ELECTRICITY
26 QUIZRR
V = E ă ir
This can also be shown as below :
E r
A B
i
VA ă E + ir = VB
or VA ă VB = E ă ir
Following three special cases are possible :
(i) If the current flows in opposite direction (as in case of charging of a battery), then V = E + ir.
(ii) V = E, if the current through the cell is zero.
(iii) V = 0, if the cell is short circuited.
This is because current in the circuit
E
i =
r
Short
circuited
E r
or E = ir
E ă ir = 0
or V= 0
Thus, we can summaries it as follows :
E r
V = E ă ir or V< E
i
E r
V = E + ir or V> E
i
V= E if i = 0
E r
V = 0 is short circuited
E
i=
r
E r
ELECTRICITY
QUIZRR 27
I m p o r t a n t Po i n t s t o Re m e m b e r
1. In figure (a ) : There are eight wires and hence, will
A B
have eight currents or eight unknowns. The eight wires
are AB, BC, CE, EA, AD, BD, CD and ED. Number of 1
loops are four. Therefore, from the second law we can 4 2
D
make only four equations. Total number of junctions are 3
five (A, B, C, D and E). But by using the first law, we can E C
make only four equations (one less). So, the total number
of equations are eight.
A B
In figure (b) : Number of wires are six (AB, BC, CDA, BE, 1
AE and CE). Number of loops are three so, three equations
will be obtained from the second law. Number of junctions are 3 E 2
four (A, B, C and E) so, we can make only three (one less) D C
equations from the first law. But total number of equations
are again six.
2. Short circuiting : Two points in an electric circuit directly R1
connected by a conducting wire are called short circuited. Under
such condition both points are as same potential.
For example, resistance R1 in the adjoining circuit is short circuited,
i.e., potential difference across it is zero. Hence, no current will R2
flow through R1 and the current through R2 is therefore, E/R2.
E
3. Earthing : If some point of a circuit is earthed then its potential
is taken to be zero. 6V
D E
For example, in the adjoining figure,
VA = VB = 0
2
VF = VC = VD = ă 3V 3V
VE = ă 9V C B
VB ă VE = 9V
or current through 2 resistance is F A
4
VB – VE 9
or A (from B to E)
2 2
Similarly, VA ă VF = 3V
VA – VF 3
and the current through 4 resistance is or A (from A to F)
4 4
ELECTRICITY
28 QUIZRR
4. For a current flow through a resistance there must be a potential difference across it but
between any two points of a circuit the potential difference may be zero.
For example, in the circuit,
net emf = 6V
A
and net resistance = 12
2V
6 1 4
current in the circuit, i = A
12 2
1 4
VA ă VB : VA 2 – 4 = VB 2V
2
i
or VA – VB = 0
B
or by symmetry, we can say that C
4 2V
VA = VB = VC
So, the potential difference across any two vertices of the triangle is zero, while the current
in the circuit is non-zero.
5. The principle of superposition : Complex 10.8 4
network problems can sometimes be solved
easily by using the principle of superposition.
12
8
This principle essentially states that when a
number of emfÊs act in a network, the solution
is the same as the superposition of the solutions
for one emf acting at a time, the others being 2 14.4V
shorted.
Figure shows a network with two loops. The currents in various branches can be calculated
using KirchhoffÊs laws. We can get the same solution by considering only one battery at a time
and then superposing to two solutions. If a battery has no internal resistance, it must be left
in place when the emf of the battery is removed. Figure shows how the superposition principle
can be applied to the present problem.
10.8V 4 4 1.2A
0.4A
12 12
8 +
8
1A
0.4A 0.6A
2 2
14.4V
(a) (b)
ELECTRICITY
QUIZRR 29
10.8V 4 1.8A
8
1.8A
2 14.4V
(c)
The current values in figure (a) and (b) are easily verified. For example when the 10.8 V
battery alone is acting, the total resistance in the circuit is,
12 8
4 2 10.8
12 8
10.8 V
This makes the total current = 1 A. This current splits between 8 and 12 in the ratio
10.8
3 : 2.
Similarly, the total resistance when only the 14.4 V battery is acting is,
12 6
8 12
12 6
14.4 V
Therefore, the total current is 1.2A
12
The superposition principle shows that there is no current in the 12 resistance. Only a current
of 1.8A flows through the outer loop. All these conclusions can be verified by analyzing the
circuit using KirchhoffÊs laws.
6. Distribution of current in parallel connections : When more than one resistances are
connected in parallel, the potential difference across them is equal and the current is distributed
among them in inverse ratio of their resistance, as
V
i =
R
R
i1
i i2 2R
i3
3R
ELECTRICITY
30 QUIZRR
1
or i for same value of V
R
1 1 1
i3 : i2 : i3 = : : 6 :3 :2
R 2R 3R
6 6
i1 = 6 3 2 i 11 i
3 3 2 2
i2 i = i and i3 i i
6 3 2 11 6 32 11
7. Distribution of potential in series connections : When more than one resistances are
connected in series, the current through them is same and the potential is distributed in the
ratio of their resistance, as
V = iR
or V R for some value of i.
R 2R 3R
V1 V2 V3
i
1 V
V1 = 1 2 3 V 6
2 2
V2 = 1 2 3 V 3
3 V
and V3 = 1 2 3 V 2
ELECTRICITY
QUIZRR 31
Ex a m pl e 12
n et em f 6
Current in the circuit i = = 15. A i
t ot a l r esist a n ce 4
V1 V2
Now, V1 = E1 ă ir1 = 10 ă (1.5) (1)
= 8.5 volt
and V2 = E2 + ir2 = 4 (1.5) (1) R
= 5.5 volt
6 . H EA T I N G EFFECT S OF CU RREN T
An electric current through a resistor increases its thermal energy. Also, there are other situations
in which an electric current can produce or absorb thermal energy.
dW dq
E
dt dt
dq dW
By definition, = i, the current through the battery and = P, the power output of (or input
dt dt
ELECTRICITY
32 QUIZRR
E i
Energy is transferred from the source at a rate Ei
E i
Energy is transferred to the source at a rate Ei.
U q
P= = V. Vi
t t
P = Vi
which with the help of OhmÊs law can also be written in the forms,
V2
P = i 2R or P=
R
Power is always dissipated in a resistance. With this rate the heat produced in the resistor in time
t is,
H = Pt
V
V2
H L i
H = Vit i Rt
2
t
R
Joule heating occurs whenever a current passes through an element that has resistance. To prevent
the overheating of delicate electronic components, many electric devices like video cassette recorders,
televisions and computer monitors have fans in their chassis to allow some of the heat produced to
escape.
Ex a m p l e 1 3
In the circuit shown in figure, find
10V 4V
(a ) the power supplied by 10V battery
(b) the power consumed by 4V battery and
i
(c ) the power dissipated in 3 resistance.
Solution. Net emf of the circuit = (10 ă 4) V = 6V
3
Total resistance of the circuit = 3
ELECTRICITY
QUIZRR 33
n et em f 6
Current in the circuit i = 2A
t ot a l r esist a n ce 3
Ex a m p le 1 4
In the circuit shown in figure, find the heat developed across each resistance in 2 seconds.
6
3
3 5
20V
Solution. The 6 and 3 resistances are in parallel. So their combined resistance is,
1 1 1 1
or R = 2
R 6 3 2
V
5
i
20V
n et em f 20
i = = 2A
t ot a l r esist a n ce 325
ELECTRICITY
34 QUIZRR
V2 (4)2 16
H6 = t (2) J oule
R 6 3
V2 (4)2 (2) 32
H3 = t J oule
R 3 3
Ex a m p le 1 5
Find the current in each branches of the circuit.
5 A 4
21V
6 5V
i2
i1
8
E B
1 C
i3
D
2V 16
Solution. It is possible to use KirchhoffÊs laws in a slightly different form, which may simplify the
solution of certain problems. This method of applying KirchhoffÊs laws is called the loop current
method.
In this method we assign a current to every closed loop in a network.
5 4
A
21V 0.5A
6 5V
2A 1.5A
8
E B
C
1 0.25A
0.25A
D
2V 16
ELECTRICITY
QUIZRR 35
Suppose current i1, i2 and i3 are flowing in the three loops. The clockwise or anticlockwise sense
given to these currents is arbitrary. Applying KirchhooffÊs second law to the three loops, we get
21 ă 5i1 ă 6 (i1 + i2) ă i1 = 0 ....(i)
5 ă 4i2 ă 6 (i1 + i2) ă 8 (i2 + i3) = 0 ...(ii)
and 2 ă 8 (i2 + i3) ă 16i3 = 0 ....(iii)
Solving these three equations, we get
1 1
i1 = 2A, i2 = – A and i3 = A
2 4
Ex a m p l e 1 6
In which branch of the circuit shown in figure a 11 V battery be inserted so that it dissipates
minimum power. What will be the current through the 2 resistance for this position of the
battery ?
2 4 6
Solution. Suppose, we insert the battery with 2 resistance. Then we can take 2 as the internal
resistance (r) of the battery and combined resistance of the other two as the external resistance (R).
The circuit in that case shown in Fig.
R
E
E2
Now power, P=
Rr
ELECTRICITY
36 QUIZRR
This power will be minimum where R + r is maximum and we can see that (R + r) will be maximum
when the battery is inserted with 6 resistance as shown in Fig. below :
i1 i
i2
2 4 6
11V
24 22
6 =
24 3
11
i = 1.5 A
22 / 3
This current will be distributed in 2 and 4 in the inverse ratio of their resistances.
i1 4
i2
= 2
2
2
i1 = (1 .5) 1 A Ans.
2 1
Ex a m p l e 1 7
5A D 2 3V 2A
E
C 1 4 B
12V
3 6
6A
Figure shows part of a circuit. Calculate the power dissipated in 3 resistance. What is the
potential difference VC ă VB ?
Solution. Applying KirchhoffÊs junction law at E current in wire DE is 8A from D to E. Now further
applying junction law at D. The current in 3 resistance will be 3 A towards D.
ELECTRICITY
QUIZRR 37
5A D 8A E 2A
C 1 2 4 B
12V 3V 6A
3A
3 6
Illust rat io n
In the given cube (12345678) find the equivalent 5 8
resistance between the function :
6
(i) 1 and 8 7
(ii) 1 and 3
(iii) 1 and 2
4
3
1
2
4
3
1 2
ELECTRICITY
38 QUIZRR
Tip : In any circuit element, if there is no current through it, then it can be removed from the
circuit.
After finding the equipotential points, remove the circuit elements if any connected between these
points and else, you can also join these equipotential points.
Re-draw the circuit : Placing the equipotential points as shown.
1.
1 (2, 4, 6) (3, 5, 2) 8
2.
(2, 4, 6)
1 (3, 5, 7) 8
R R R 5R
Net equivalent resistance =
3 6 3 6
5 8
6 R/2 (5, 7) R/2 8
6
7
R R/2 R
4 R/2
3 R/2 3
1 (2, 4)
1 2
3R
Req. =
4
ELECTRICITY
QUIZRR 39
5 8
6
7 (4, 6) and (3, 7) are equipotential junction pairs.
4
3
1 2
Hence,
R
R 5
R
1 R (3, 7) R
R (4, 6) R
2
R
8 R
R/2
1 R/2 R R/2
(4, 6) R/2 2
R/2
8
R
R
2R R/2
1
R/2
R/2
R
1 2
7R
12
ELECTRICITY
40 QUIZRR
7R
Req. =
12
R/2
(4, 6) (3, 7)
R/2 R/2
1 R 2
7R
Req. =
12
II. If the line joining the initial and final point (i.e AB) is the axis of symmetry, then all points lying
on the perpendicular of the line AB are equipotential.
eg.
2 In this network points (2,4, 6), (1,5) and (3, 7)
are equipotential.
R/2 R/2
(4, 6)
III. If the line AB is not the axis of symmetry, but its perpendicular bisector is the axis of symmetry,
then all points lying on the perpendicular bisector are equipotential.
ELECTRICITY
QUIZRR 41
Illustrat ion
M R R
R R
O R R Req. = 5R
4
R R
A B R R
A B
N
Here, (M,O,N) are equipotential points and hence, OM, ON resistors can be removed from the
circuit.
IV. Connection Removal Method (Image Method) :
Conditions to apply this method :
(1) Circuit should be symmetric relative to the initial and final points.
(2) If we place a mirror perpendicular to the line AB in such a way that image of A becomes the
image of B, then we can apply this method.
You cannot apply the image method is the cases as one shown be.
These two must be same to apply image method.
4R
R R R
R
R1
R1
R R
2R
Concept : The current in the resistor and its image are equal.
NOTE : If the resistor and its image coincide at a junction, then the junction can be separated
from other.
Illust rat io n
Find the net resistance between A and B
R R R R
Ć Ć Ć Ć
A B A B
R R
R R
R
4R
Req. =
5
ELECTRICITY
42 QUIZRR
Illust rat io n
Find net resistance between 5 and 7.
R 7
R
Ć7 5 R
R
5Ć R
R
R
R
R
R
3R
Req. =
4
V. Star-Delta Method
A A
RA RA
r3 r2 r3 r2
RB
Rc RB Rc
B r1 C B r1 C
(Delta) (Star)
(i) Delta to star conversion
r2 r3 r3 r1 r1 r2
RA = RB = RC =
r1 r2 r3 r1 r2 r3 r1 r2 r3
Illust rat io n
B 4r/3 E
Ć Ć
r r
Ć r r Ć
A F
r
r
Ć Ć
C r/3 D
ELECTRICITY
QUIZRR 43
Solution :
Symmetry methods do not apply in above such problems.
Convert delta to star for ABC and DEF
ABC and DEF are similar
B
ĆB
r
r2 r2
r + r + r r + r + r
AĆ r
A
r2
r + r + r
r
Ć C
C
B
r/3
r/3
A
r/3
4r/3
r/3 C
r/3 r/3
B
A r/3
r/3 r/3
r/3
2r
r/3 r/3
A B
r
A
4r B
3
4r
Req. =
3
ELECTRICITY
44 QUIZRR
RA
r3 r2
RC
Ć Ć RB
B C Ć Ć
r1 B C
(Delta) (Star)
R A R B R B RC RC R A
r1 = RA
R A R B R B RC RC R A
r2 = RB
R A R B R B RC RC R A
r3 = RC
1R
2R 2R
AĆ 2R 3R
ĆB
2R
ELECTRICITY
QUIZRR 45
11 11
R R
3 2
2R
2R
A B
2R
11R
22 22
R R
17 15
22
R
13
A B
22 32
17 15
A B
22
13
Req. = 1.049
Ex a m p l e 1 8
A B
ELECTRICITY
46 QUIZRR
B
A
R
2R D
Req. = (after removing OD and DB) C
3
2R R
R
B
A R
D C
(ii) About AB : Apply image method
A B
D R C
R R
8R R Ć R
Req. = Ć R
15 R
Ć Ć
A R B
D C
A B
ELECTRICITY
QUIZRR 47
Hence
R
(D, R) Ć R
Ć
R
R
R R
R
A
(D, B) R/2
Ć C
R/2
R
R/2
O
R
Ć
A
A R O
Ć Ć
(D, B)
R/2 R/2
3R
2
A R O
R/2 3R
8
7
Req. = R
15
ELECTRICITY
48 QUIZRR
Ex a m p l e 1 9
Find the equivalent resistance between A and B. F
B
D
A
C
Solution. Apply rule 1.
(D, F) and (C, E) are equipotential junctions.
Hence, remove the resistors connected between them
(i.e.) DF and CE
Simplified circuit :
R
(D, F) B
R
R R R
E
(E, C) R A
3R
Req. =
5
Ex a m p l e 2 0
Find the equivalent resistance between AB.
r r
A1 r
B1
r r
r r
A r r
Q B
ELECTRICITY
QUIZRR 49
r r
r
r r
r r
A 2r B
10 r
Req. =
9
Ex a m p l e 2 1
In previous question, the question is extended and there are infinite triangular networks
embedded is each other as shown, then find the equivalent resistance between AB.
r r
A1 Ć r/2 Ć r/2 ĆB
1
r r/2 r/2 r
Ć Ć
r/2 r/2
AĆ Ć ĆB
r r
Solution. Let the resistance between A and B be x.
Then the smaller triangle with A1B1 has resistances hall in each arm compared to the triangle with
AB.
x
Hence the equivalent resistance between A, B, =
2
r r
A1 B1
x/2
r r
A B
r r
ELECTRICITY
50 QUIZRR
2 xr
4 r x 2 r 2 r x
(Req.)AB =
2 xr 4 r
4r x
4 ( 7 – 1) r
x=
6
8 . GROU PI N G OF CEL LS
8 .1 . Gro u p i n g o f I d e n t i c a l Ce l ls
is the emf of each cell and r is the internal resistance
1. Series grouping
n
I = r r r
(R + n r )
n
I =
R
R
The series grouping is ideal when R > > r.
2. Parallel grouping
n I/n E, r
I = <
r (n R + r )
R +
n < r
r
r
eq = , Req =
n
ELECTRICITY
QUIZRR 51
n
I =
nr
R +
m
m n
I = R
(m R n r )
2 2
(mR + nr) = mR nr
2
= m R – nr 4 m nr R
2
= m R – nr 2 mn r R
mn mn n m
I = = = =
m R +n r 2m R 2R 2r
Ex a m p l e 2 2
40 identical cells each of emf = 6V and internal resistance are to be connected across
a load of R = 10. Find the most effective grouping ?
Solution. In the most effective grouping the current through the load is maximum.
Let n cells be series of each row, with network having m rows.
We have mn = N = 40
For maximum current
mR = (nr)
m (10) = (1) n
n = 10 m
10m2 = 40 (m = 2)
ELECTRICITY
52 QUIZRR
n = 20
mn m n 20 6
I = = 6A
2 m nR 2 m R 2 10
Most effective grouping must have 20 cells in each row with 2 rows, in all.
n n
i eq = i R
i 1
i 1
I = n n
R ri Req. = ri
i 1
i
(2) Parallel grouping : First we will derive for two cells, then we will generalize the result.
I = (i1 + i2)
i 1 1 , r
( – IR) – IR 1
i1 = r1
; i 2
= r
2
r
i 2 2 2
– IR – IR
I = r r
1 2
I R
R R
= I 1 1 2
r1 r2 r1 r2
1 2 1 1
–1
1 2
r1 n r1 r2
r1 r2
I = =
1 1 1 1
–1
R
1 R +
r1 r2 r1 r2
1 1
–1
1 2
r1 r2 r1 r2
I =
1 1
–1
R +
r1 r2
ELECTRICITY
QUIZRR 53
In general,
–1
i 1
ri ri
I =
–1
1
R +
ri
i
req 1
ri 1 1 1 1
or, I = where r r r r .... r
R + (r e q ) eq 1 2 3 n
Ex a m p l e 2 3
12 identical cells are given, each of emf , and internal resistance r, and are connected in
series and kept in a box. Same cells are wrongly connected by mistake. Now, when the box
is connected in series with two other identical cells, the current is 3A, when they support
each other and is 2A, when they oppose each other.
Solution. Let the no of wrongly connected cells be x
Net emf is series combination
= + (N ă x) ă x = (N ă 2x) = (12 ă x) { N = 12}
Tip : 1. When in series combination, a cell is wrongly connected, then its emf is added as ă.
2. If out of N cells is series, x are wrongly connected, net emf = (N ă 2x) .
(12 – 2 x) 2
Now, = 3 ...(1)
14 r
(14 – 2 x) 3
=
(10 – 2 x) 2
x = 1
9 . M a x i m u m Po w e r T ra n s f e r T h e o r e m
„For maximum power (i.e. maximum current) being dissipated through an external load, connected
in a circuit with battery of internal resistance r, the external resistance must be equal to internal
resistance of the battery.ÊÊ
ELECTRICITY
54 QUIZRR
Derivation : r
i =
(R r )
2 R R
P = i2R =
(R r )2
2 R r – 2 R[2(R + r )]
2
dP
dR = 2
(R r )4 4R
2 (r – R) O R=r
= Load
(R r )3
dP
Hence, d R = 0, when (r = R)
1 0 .1 Ch a r g i n g o f Ca p a c i t o r
So, far, we have been concerned with the capacitors in the steady state, i.e. the capacitors which
have already been charged to their steady state voltages.
Now consider a circuit where an uncharged capacitor C is connected to a cell of emf E through a
resistance R.
C R
Applying loop rule
q
iR =
c
q
iR = –
c
Now, due to the flow of corrent, the upper plate begins to acquire positive charge and lower becomes
negetively charged. The voltage across capacitor begins to grow.
ELECTRICITY
QUIZRR 55
dq CE – q
R =
d t C
dq
RC = (CE ă q)
dt
q (t ) t
dq dt
(CE – q )
= RC
0
q0
CE – q –t
ln =
CE – q0 RC
t/
q (t) = q0 + (qs ă q0) [1ă e– )
where,
q0 inital charge ; qs final steady state charge
time-constant = RC
Differentiating the above result, we get the result for current,
q – q0 – t /
i (t) = s e i0 e– t /
q0
t t
ELECTRICITY
56 QUIZRR
Time constant : The time in which a capacitor will get completely charged if it would have been
charging with initial constant rate, is called time constant.
qs
q0
Ex a m p l e 2 4
Which capacitors 1 or 2, has the time constant is greater, for which the charging curves
are shown ?
q
1
2
t
q
Solution. tan = s {as q0 = 0}
1
Slope a t t 0
1 0 .2 . Di s c h a r g i n g o f t h e Ca p a c i t o rs
q
– i R = 0 C
+ ă R
C
dq = q
– R C
dt
dq
q(t ) t
dt
q0
= –
q RC
0
ELECTRICITY
QUIZRR 57
q (t ) –t
n =
0
q RC
q (t) = q0 e– t / RC
– q0 – t / RC
i (t ) = e
RC
i (t)
q0
t
ă q0
t RC
A N A L Y SIS OF RC CI RCU I T
I. At time t = 0.
Get the initial charge through the capacitor. And then find the C R
initial current through the capacitor.
· The initial current can be found by short-circuiting the
capacitor, only if the capacitor is initially uncharged.
· Otherwise, apply toop rule to get the current through the
capacitor.
i0 =
R
qs – q
i0 0
ELECTRICITY
58 QUIZRR
Among four unknown, three can be found using above concept and the fourth can be obtained,
hence.
Note : You can also find the time-constant by finding the equivalent resistance between the
terminals of the capacitor.
Remember : To find the time constant directly, short-circuit the battery and then find the equivalent
resistance between the capacitor terminals.
Ex a m p l e 2 5
At time = t = 0, the switch S is closed. Find the function,
(i) q (t) = ?
(ii) i (t) = ?
S R
C
E R
Solution. At t = 0, q0 = 0
RÊ
E C
R
R
Let i0 be the current through capacitor, and I be the current through battery.
Then,
R
E R C
1 2
ELECTRICITY
QUIZRR 59
In loop (1), E = IR + I – i0 R
E = 2 i0 R + i0 R i0 E /3R
At t =
A R
Ć
E 1 R
Ć
R B
In loop (1)
E = 2iR iR = E/2
CE
qs =
2
qs – q0 CE 3R 3RC
Hence, = =
i 2 E 2
CE
q (t) = (1 – e –2 t /3 RC ) ...(i)
2
–2 t / 3 RC
We get, i (t) = (e ) ...(ii)
3R
ELECTRICITY
60 QUIZRR
Ex a m p l e 2 6
Find the function of q (t) and i (t) when the initial charge through the capacitor is zero.
R R
R
E C
R
R
Solution : At t= 0
Short-circuit the capacitor R R
8R
Req = R
3
R
E
3E
I =
8R
R
3E 2 E
i0 =
8R 3 4R
At t =
R R
a
R b
R
E
R
CE
qs =
2R
( q s – q0 ) C E 4R
= (2R C)
i0 2R E
Ex a m p le 2 7
Find the potential drop across the 20 resistor connected through the capacitor after 1 ms.
(Initially the capacitor is uncharged)
ELECTRICITY
QUIZRR 61
Solution. Find the time constant directly using the equivalent resistance concept.
40F 20
10
10
= 20V
Equivalent resistance is calculated across the capacitor and shorting the battery.
A B 20
10
10
10
Req = 20 25
2
400 – t
i (t) = (e ) 10 –6 A
10 –3
= 0.4 (e–t) A
0.4
i (t) = t A
e
0.4 8
= 20 V
e e
1
ELECTRICITY
62 QUIZRR
New Concept : If no circuit element is connected in series with the cell, then we can split the
branches of the circuit into different individual circuits.
10 +
= 20V
i = i (t) = 20V
i = 2A
In the above diagram,
i (t) = current through the capacitor = 1eăt/ where = 40 20 10ă6 = (8 10ă4) s
But current through the battery = 2 + eăt/
Ex a m p l e 2 8
In the arrangement shown, the switch S is closed at the time t = 0.
The capacitor has charges as shown
Find :
(i) current through the cell as a function of time t.
(ii) q (t) on capacitor
(iii) charge flown from the battery
(iv) total heat dissipated in the circuit.
ă CE + CE
2 2 R
E S
Solution. At, t = 0 –
CE CE
2 2 R
Apply KVL as capacitor is charged
E
E + – iR = 0
2
3E CE
i0 = , q0 = – E
2R 2
ELECTRICITY
QUIZRR 63
At t =
qs = (CE)
= (RC)
q (t) = q0 + (qs ă q0) (1 ă eăt/)
–CE 3CE
1 – e
– t /RC
=
2 2
3CE – t / RC CE
(i) q (t) = CE – e = [2 – 3 e – t / RC ]
2 2
3E – t / RC
(ii) i (t) = e
2R
= (Qf) ă (Qi) = CE – –
CE 3CE
2 2
3CE 1 2 E2
0 E = C E – H
2 2 4
Ex a m p l e 2 9
q0
t i l l t h e ch ar ge on t h e capaci t or becam e 2 in time t1. Now
S1
S1 is opened and S2 is closed till the charge on the capacitor
3q0 2R
becomes 4 in time t. Find the ratio (t1/t2).
S2
ELECTRICITY
64 QUIZRR
q0
= q0e(t1/RC)
2
t1 = (RC ln2
(ii) When S1 opened and S2 is Closed :
q0 q
q(t) = q0 – 0 (1 – e
– t / 3RC
)
2 2
q0 q0 q0
q(t) = 2 2 – 2 e
– t /3 RC
3 q0 q0 – t 2 /3 RC
4 = ( q0 ) – 2 e
q0 q0 – t2 /3 RC
= e
4 2
t2
ln2 = 3RC t2 = 3RC ln2
t1 1
=
2
t 3
Ex a m p l e 3 0
Find the current through the battery just after closing the C R
switch and after longtime. Also, find the charge on each
capacitor and the total heat dissipated after the steady state R C
is reached. R R C
Solution. At t = 0 E
The short circuit capacitor circuit.
ELECTRICITY
QUIZRR 65
E R
i = R
R R R
At t =
Short circuit the resistor in which there is no current
E
qc = 0, qA = qB = (CE)
i = 0 (through the battery)
A
Net charge flown from the battery = (2CE ă 0) = 2 CE
Heat dissipated, B C
Wex + Wb = V + H
1
O + 2CE2 = CE 2 2 H
2
H = CE2
Ex a m p le 3 1
In the circuit shown initially S1 and S2 are both closed. 2F 6
When the steady state is reached the key S1 is opened
(at t = 0). S1
Then find the charge flown through the battery and 3 4F
heat dissipated till the steady state is again reached after
opening S1.
Solution. At t = 0, charge distribution was,
S2
18
2A = 18V
Current (i) in the circuit =
9
3 B
ELECTRICITY
66 QUIZRR
After S1 is opened at t = 0 and the steady state is reached once again with S2 closed.
q
C
qA = 36 E 18V= C
qB = 72C
2F 6
3 4F
= 18V
Charge flow from the battery
= [(36 + 72) ă (12 + 48)] C = 48 C.
Heat dissipated
Wex + Wb = H + U
H = 216 10ă6J
Derivation :
R`1 R2 R3
H1 = R 1 i d t R 1 i d t
2 2
0 0
H2 = R
0
2 i 2 d t and so on
ELECTRICITY
QUIZRR 67
But, Htotal = i 2 R eq d t = Req i 2 d t
0 0
i
2
The factor d t is con st a n t
0
H H H H
i
1 2 3 Tot a l
2
dt = R H R R
0 1 2 3 eq
H
Hi = R i T ot a l
R eq
C
V2 1
H1 = R d t R V d t
2
0 1 1 0
R1
1
H2 = R V d t and so on.
2
R2
2 0
R3
= V 2 dt K =(H1R1 ) (HT Re q )
0
= .....= (H2Req)
HT Req
H1 =
R1
Ex a m p l e 3 2
A capacitor of 2F is allowed to be discharged through resistors of R1 = 3 and R2 = 2 after
being charged to a potential of 10V. Calculate the heat dissipated through each if connected.
ELECTRICITY
68 QUIZRR
C = 2F
R1
R2
R1 R2
HT =
1
HT = CV2
2 1
= CV 2
2
1
= 2 (100) 100 J
2 = 100 J
H1R1 = H2R2 = HT Req
H1 H 2 H T 100
R R = Req 5 H T R eq
1 2 H1 = R 1
100 3 100
H1 = R 1 60 J 100 6
5 5 = 40 J
35
H2 = (40J)
H2 = 60J
1 2. EL ECT RI CA L M EA SU RI N G I N ST RU M EN T S
So far we have studied about current, resistance, potential difference and emf. Now we will study
how these are measured. The basic measuring instrument is galvanometer, whose pointer shows a
deflection when current passes through it. A galvanometer can easily be converted into an ammeter
for measuring current, into a voltmeter for measuring potential difference. For accurate measurement
of potential difference or emf a potentiometer is more preferred. Resistances are accurately measured
by using post office box or meter bridge which are based on the principle of ÂWheatstone bridge.‰
1 2 .1 Ga l v a n o m e t e r
Many common devices including car instrument panels, battery chargers measure potential difference,
current or resistance using dÊ Arsonval Galvanometer. It consists of a provided coil placed in the
magnetic field of a permanent magnet. Attached to the coil is a spring. In the equilibrium position, with
no current in the coil, the pointer is at zero and spring is relaxed. When there is a current in the coil,
the magnetic field exerts a torque on the coil that is proportional to current. As the coil turns, the spring
exerts a restoring torque that is proportional to the angular displacement. Thus, the angular deflection
of the coil and pointer is directly proportional to the coil current and the device can be calibrated to
measure current.
ELECTRICITY
QUIZRR 69
The maximum deflection, typically 90Ĉ to 120Ĉ is called full scale deflection. The essential electrical
characteristics of the galvanometer are the current ig required for full scale deflection (of the order of
A to 10mA) and the resistance G of the coil (of the order of 10 to 1000).
10
The galvanometer deflection is proportional to the current in the coil. If the coil obeys OhmÊs law,
the current is proportional to potential difference. The corresponding potential difference for full scale
deflection is,
V = i gG
1 2 .2 A m m e t e r
It is a device used to measure the current through a circuit element and connected in series with
that element.
E
E
Actural Current i1
R r
= E
R {Measured
R
A current}
R r
R
Now, I = , IÊ =
R r
1
R
I
I’ = 1 r / R
I If
G
(I ă If)
R
(4) To convert a galvanometer into an ammeter, a small value resistance called shunt is connected
in parallel across the galvanometer.
ELECTRICITY
70 QUIZRR
I Ig
G
(I, ăIg) R
I gG
R= I–I
g
1 2 .3 V o l t m e t e r
It is a device which can measure the potential difference across a circuit element and connected in
parallel with that element.
I R
I I’ R
Vactual = (IR)
(I ă I’) V’ =I’R
v r
Vobserved = (I‘R)
(I ă IÊ)r = (IÊR)
Ir = I‘ (r + R)
Ir I
I‘ = (V‘ < V)
(r R) (1 R / r )
As r (I’ I)
Few important points to note :
(1) The reading of the voltmeter is always less than the actual value.
(2) Greater the resistance of the voltmeter, more accurate is the reading obtained.
(3) For an ideal voltmeter, resistance should be infinite, which has been realized in practice through
a potentiometer.
To convert a galvanometer into voltmer of given range V :
We connect a high value resistance R is series with the galvanometer.
ELECTRICITY
QUIZRR 71
Rg
ig R
+ G ă (Vab = V)
i gR + Rgi g = V
V
R= – Rg
ig
Illust ra t io n
A galvanometer of Rg = 30 and i g = 2mA is to be converted into an ammeter of 0.3A range
and a voltmeter of 0.2V range. Find the resistance required in each case.
Solution. To convert into an ammeter,
igR g 2 10 –3 30 2 10 –3 30 10
S = 10 0.2
(I – i g ) (0.3) – (2 10 –3 ) 3
V 0.2
R= – Rg – 30 (70 )
Ig 2 10 –3
Ex a m p l e 3 3
A cell of emf 3.4V and internal resistance r = 3 is connected in series with an ammeter of
resistance 2 and an external resistance of 100 when a voltmeter is connected is parallel,
ammetter reading is 0.04 A.
=V r = 3
0.04A R =100
A
RA = 2
VR 3.2
Req. = I 0.04 80
ELECTRICITY
72 QUIZRR
100R V
Now, (100 R V )
= 80 RV = 400
VR = 68 100
21
Ex a m p le 3 4
To convert into voltmeter
1g = 1mA
Rg = 50 R1 R2 R3
G
X Y Z
1V 10V 100V
V 1
Solution. Through X : R 1 1 – R g –3 – 50 (950)
ig 10
V2 10
Through Y : (R1 + R2) = i – R g = – 50 9950 R2
g 1 10 –3
V
Through Z : (R1 + R2 + R3) = 3 – R g (10 5 – 50)
i
g
R3 = 90000
1 2.4 POT EN T I OM ET ER
The potentiometer is an instrument that can be used to measure the emf or the internal resistance
of an unknown source. It also has a number of other useful applications.
ELECTRICITY
QUIZRR 73
E1
i i
a b
c
i2 = 0
G
E 2, r
Pr i n c i p l e o f Po t e n t i o m e t e r
The principle of potentiometer is schematically shown in figure. A resistance wire ab of total
resistance Rab is permanently connected to the terminals of a source of known emf E1. A sliding contact
c is connected through the galvanometer G to a second source whose emf E2 is to be measured. As
contact c is moved along the potentiometer wire, the resistance Rcb between points c and b varies. If
the resistance wire is uniform Rcb is proportional to the length of the wire between c and b. To
determine the value of E2, contact c is moved until a position is found at which the galvanometer shows
no deflection. This corresponds to zero current passing through E2. With i2 = 0, KirchhoffÊs second law
gives,
E2 = iRcb
With i2 = 0, the current i produced by the emf E1 has the same value no matter what the value
of emf E2. A potentiometer has following applications.
E1 E1
i
l1 l2
i i
a b a b
c1 C2
i2 = 0 i2 = 0
G G
Ek EU
We calibrate the device by replacing E2 by a source of known emf EK and then by unknown emf
EU. Let the null points are obtained at lengths l1 and l2. Then,
EK = i (l1)
and EU = i (l2)
ELECTRICITY
74 QUIZRR
EK l2
EU
= l2
l2
or EU = l E K
1
So, by measuring the lengths l1 and l2, we can find the emf of an unknown battery.
T o f i n d t h e i n t e rn a l r e s is t a n c e o f a n u n k n o w n b a t t e r y
To find the internal resistance of an unknown battery let us derive a formula.
E r
i
R
E
i = ...(i)
R r
E
r = R – 1
V
Thus, if a battery of emf E and internal resistance r is connected across a resistance R and the
potential difference across its terminals comes out to be V then the internal resistance of the battery
is given by the above formula. Now let us apply it in a potentiometer for finding the internal resistance
of the unknown battery.
E1
i i
l1
a b
c1
i2 = 0
G
E r
ELECTRICITY
QUIZRR 75
i E1 i
i
l2
a b
C2
i2 = 0
E r
G
i0
R
where V = potential difference across the terminals of the unknown battery. Hence,
V = il 2 ...(ii)
From Eqs. (i) (ii),
E l1
= l
V 2
E
Substituting in r = R – 1 , we get
V
l1
r = R l – 1
2
So, by putting R, l1 and l2 we can determine the internal resistance r of unknown battery.
Ex a m p l e 3 5
A potentiometer wire of length 100 cm has a resistance of 10 ohm. It is connected in series
with a resistance and a cell of emf 2 volts and of negligible internal resistance. A source
of emf 10mV is balanced against a length of 40 cm of the potentiometer wire. What is the
value of external resistance ?
Solution. From the theory of potentiometer,
Vab = E, if no current is drawn from the battery
ELECTRICITY
76 QUIZRR
E1
E1
or R ab = E
R + R ab
R
Here, E1 = 2V, Rab = 10
a b
c
40
Rcb = 10 4
100 E
G
and E = 10 10ă3 V
Substituting in above, we get
R = 790 Ans.
ig = 0
The figure shows the circuit designed by him. The bridge P
A Q C
is said to be balanced when deflection in galvanometer is zero, G
Similarly, VB ă VC = VD ă V C
or i1Q = i 2S
i1 S
or i2
=
Q
R S
From Eqs. (i) and (ii), =
P Q
P R
=
Q S
ELECTRICITY
QUIZRR 77
B B
P Q P Q
A G C A C
R S R S
D D
P R
=
Q S
Q
G
R
(a)
Q P R
P
G G
R Q S
S
(c)
(b)
ELECTRICITY
78 QUIZRR
P l1
= l
Q 2
P Q
l2 l2
E Meter Bridge
Ex a m p l e 3 6
A potentiometer wire of length l = 100 cm, R = 10 is connected in series with a cell of emf
= 2V, and an external resistance R. A source of 10 mV is balanced against 40 cm length
of potentiometer wire. Find the external resistance R and the maximum potential that can
be measured with the potentiometer.
2
Solution. i =
(10 R)
40
Resistance of 40 cm of wire = 10 4
100
Now, i (4) = 10 mV
24
= 10 10ă3
(10 R)
800 = 10 + R
R = 790
2
i = 0.25 10 –2 2.5 10 –3 A
800
ELECTRICITY
QUIZRR 79
Ex a m p l e 3 7
A potentiometer wire AB is 100 cm long. When AC = 40 cm, no deflection occurs in
galvanometer. Find R.
10 R
A C B
10 AC
Solution : =
R CB
CB 100 – 40
R = 10 10
AC 40
60
= 10 15
40
Ex a m p l e 3 8
In the arrangement of the potentiometer shown, the first balanced point is achieved at 75
cm from one end A when the key K is opened. After closing the key K, the second balanced
point is achieved at 60 cm from the end A. Find the internal resistance of the cell and the
length of the potentiometer wire AB.
Solution. (a) When the key is opened
Drop V1 = = 1.5 V
(b) When the key is closed
Drop V2 = ( ă ir) 75m
A B
2 1
= – 60cm
R r
r
r (a)
5r
4 = 5 –
Rr
ELECTRICITY
80 QUIZRR
5r
R r = 1 4r = R
10
r = 2.5
4
0
l0
=
l
2 1.5
0
=
(75)
75
l0 = 2 cm = 100 cm
1.5
Ex a m p l e 3 9
A uniform conducting wire of length AB = 100 cm is connected with an unknown resistance
x and a known resistance R by a thick metallic strip.
X R
A B
Now, a battery and a galvanometer with a sliding jockey is given. Make appropriate
connection to find the resistance X if the balance point is achieved at a point 60 cm from
the end A.
Solution. To convert the given arrangement, think the balanced wheastone bridge, with connections
made as shown.
D
R X
A
G
C
J X R
A B
J
G
ELECTRICITY
QUIZRR 81
R AJ 60
X = J C 40
4 4
X= R = 12 8
6 6
L EA K A GE CU RREN T
Leakage current is the current flowing through the di-electric placed between the walls of the
capacitor.
+q
j = E ăq
q
j = K S
0
q ds q
i = j .d s = K s K
0 0 (dictector)
– dq q
i = = K
dt 0
q (t ) t
dq
–1
q0
q =
K 0 0
dt
q (t) = q0eăt/ 0
d K 0 S
= (K0) (RC) K 0
s d
q
Potential drop at any time =
c
Ex a m p l e 4 0
The gap between the plates of a parallel plate capacitor is filled with a dielectric of restivity
and dielectric constant K. It is connected with a battery of emf V. Then find the current
(leakage) through the capacitor.
ELECTRICITY
82 QUIZRR
q = CV
q CV
i = =
P k0 ( rk 0 e 0 )
Ex a m p l e 4 1
The gap between the electrodes of a spherical capacitor is filled with a dielectric of dielectric
constant K and charged to a potential drop of V0. Now it is found that the potential drop
across the capacitor decreases to n times is the time to. Then, find the resistivity of the
dielectric.
q
Solution. V = V0eăt/ V=
C
V –t / t0
Ve ln =
t0
and = ln n
t0
K = ln n
0
t0
= K ln
0
Ex a m p l e 4 2
Two resistors with temperature coefficients of resistance 1 and 2 have resistances R01
and R02 at 0ĈC. Find the temperature coefficient of the compound resistor consisting of the
two resistors connected.
(a ) in series, (b) in parallel.
Solution. In series :
ELECTRICITY
QUIZRR 83
R 01 1 R 02 2
= R 01 R 02
In parallel :
R 01 R 01 R 02
R=
R 01 + R 02
R 02
1 1 1
At tĈC, =
R 0 (1 t ) R 01 (1 1 t ) R 02 (1 2 t )
R 01 R 02 1 1
or = R (1 t ) R (1 t )
R 01 R 02 (1 t ) 01 1 02 2
1 1
(1 – t ) = R (1 – 1 t ) R (1 – 2 t )
1 1
(1 – t )
R 02 R 01 01 02
1 1 1
i.e. at = t 2 t
R 01 R 02 R 01 R 02
1 R 02 2 R 01
or = R 01 R 02
Ex a m p l e 4 3
An ammeter and votmeter are connected in series to a battery with an emf E = 6.0 volt.
When acertain resistance is connected in parllel with the voltmeter, the reading of the
latter decreases two times, whereas the reading of the ammeter increase the same number
of times. Find the voltmeter readings after the connection of the resistance.
Soution. Let R1 = resistance of ammeter
and R2 = combined resistance of ammeter and voltmeter
In the first case current in the circuit,
6
i = R ...(i)
2
ELECTRICITY
84 QUIZRR
1 6R
or V = 6– R ..(ii) i
2
6 12
i’ = = R ...(iii)
R2 /2 2
and V’ = 6 ă (i’) R1
12R
1
or V’ = 6 – R ...(iv)
2
V
Further, it is given that V’ =
2
12R 1 3R 1
or 6– = 3– R
R2 2
R1 1
or R2
=
3
1
V’ = 6 – (12)
3
or V’ = 2 volt Ans.
Ex a m p le 4 4
The gap between two plane plates of a capacitor equal to d is filled with a gas. One of
the plates emits n 0 electrons per second, which while moving in an electric field, ionize gas
molecules. This way each electron produces new electrons (and ions) along a unit length
of its path. Find the electronic current at the opposite plate, neglecting the ionization of
gas molecules by formed ions. Take charge on an elecron as e.
Solution. Let n be the number of electrons at x = x.
dn
Now, = increase in n per unit length
dx
=n
dn
= .d x
n
ELECTRICITY
QUIZRR 85
n dn d
or n0 n
d x
0
dx
n
ln = d
n0
n
or ed
n0
x=0 x=x x=d
or n = n0ed
Here, n is number of electrons reaching at other plate per unit time.
Electronic current at opposite plate = charge reaching per unit time
or i = en
or i = en0ed Ans.
Ex a m p le 4 5
A voltmeter of resistance R1 and an ammeter of resistance R2 are connected in series
across a battery of negligible internal resistance. When a resistance R is connected in
parallel to voltometer, reding of ammeter increases three times while that of voltmeter
reduces to one third. Find R1 and R2 in terms of R.
Solution. Let E be the emf of the battery.
E E
A G
i 3i
i
C 3 D
A V A V F
B R1
R2 R1 R2
8i
3
R
i 8i
V C ă VD = R 1 R
3 3
or R1 = 8R Ans.
ELECTRICITY
86 QUIZRR
R
E = 3 i (R 2 ) (i / 3)(R 1 ) i 3R 2 1 ...(ii)
3
R1
R1 + R2 = 3R 2
3
2R 1
or 2R2 =
3
R1
or R2 =
3
8R
or R2 = Ans.
3
Ex a m p l e 4 6
A rod of length L and cross-section area A lies along the x-axis between x = 0 and x = L.
The material obeys OhmÊs law and its resistivity varies along the rod according to,
(x) = 0 eăx/L
The end of the rod at x = 0 is at a potential V0 and it is zero at x = L
(a ) Find the total resistance of the rod and the current in the wire
(b) Find the electric potential V (x) in the rod as a function of x.
Solution. (a) Resistance of elementary section dx at x = x is,
x
x=0 x=L
dx
( x) d x 1
dR = R = A
A
x
–
0 e L . d x
=
A
L
0 L – Lx L
Therefore, R = dR = e dx = 0 (1 – e–1 )
0
A 0 A
ELECTRICITY
QUIZRR 87
0 L 1
or R=
A 1–
e
Ans.
V0 V0 A e
Current in the wire, i = R L e–1 Ans.
0
V0 A e 0 e– x / L
= L dx
0 e–1 A
V0 e – x / L
(dV) = e dx
L e–1
e – x/L
x
V0
e
V
dV = dx
0 L e– 10
e – x/L
or V ă V0 = V0 (1 – e )
1 – e
V0 (e – x / L – e–1 )
V= Ans.
1 – e–1
Ex a m p le 4 7
What amount of heat will be generated in a coil of resistance R due to a charge q passing
through it if the current in the coil
(a ) decreases down to zero uniformly during a time interval t0 ?
(b) decreases down to zero halving its value every t0 seconds ?
How to Proceed : Heat generated in a resistance is given by,
H = i 2 Rt
We can directly use this formula provided i is constant. Here, i is varying. So, first we
calculate i at any time t, then find a small heat dH in a short interval of time dt. Then by
integrating it with proper limits we can obtain the total heat produced.
Solution. (a) The corresponding i-t graph will be a straight line with i decreasing from a peak value
(say i0) to zero in time t0.
i
i0
t
t0
ELECTRICITY
88 QUIZRR
i0
i = i0 ă t (y = ămx + c) ...(i)
t0
Here, i0 is unknown, which can be obtained by using the fact that area under i-t graph gives the
flow of charge. Hence,
1
q= (t0 )(i0 )
2
2q
i0 = t
0
2q t
i = 1 –
t0 t0
2 q 2 qt
or i = – 2
t0 t0
2
= 2 q – 2 qt R dt
t 2
t0
0
t0
2
2 q 2 qt
t0
or H = – 2 R dt
0 0
t t0
4 q2R
= Ans.
3 t0
(b) Here, current decreases from some peak value (say i0) to zero exponentially with half life t0.
i-t equation in this case will be
i = i0eăt
ln (2)
Here =
t0
ELECTRICITY
QUIZRR 89
i
i
q = i d t i0 e
– t
Now, dt 0
0 0
i0
i0 = q
i = (q) eăt
dH = i2R dt = 2q2eă2t Rdt
q R
H = d H = q R e dt
–2 t
2 2
or
0
0
2 t0 t
ln (2) q 2 R ln (2)
Substituting = , we have H = Ans.
t0 2 t0
Ex a m p l e 4 8
A conductor has a temperature independent resistance R and a total heat capacity C. At
the moment t = 0 it is connected to a dc voltage V. Find the time dependence of the
conductorÊs temperature T assuming the thermal power dissipated into surrounding space
to vary as q = k (T ă T0) where k is a constant, T0 is the environmental temperature (equal
to conductorÊs temperature at the initial moment).
V2
How to Proceed : Here energy is being generated in the resistance at a rate of . On
R
which part of energy is being lost in the environment and the rest is utilized in raising the
temperature of conductor. So, this is basically a problem of conservation of energy.
Solution. Energy supplied by the dc source per unit time = energy lost in environment per unit time
+ energy used in raising the temperature of conductor per unit time.
V2
= k (T – T0 ) C
dT
Hence,
R dt
dT V2
C = – k (T – T0 )
dt R
dT dt
= 2
V C
– k (T – T0 )
R
T t
dT dt
= V 2
= C
T0
– k (T – T0 ) 0
ELECTRICITY
90 QUIZRR
This kind of question will become clearer if thermodynamics has been done by the student).
Ex a m p l e 4 9
An electrical circuit is shown in figure. Calculate the V
potential difference across the resistor of 400 ohm as will 400
be measured by the voltmeter V of resistance 400 ohm either
by applying KirchhoffÊs rules or otherwise. (J ee 1996) 100 100 200
i2 i1 100
i
10V
Solution. The given circuit actually forms-a balanced Wheatstone bridge (including the voltmeter) as
shown in figure.
A
Rv = 400
0
Q
=
V Q = 200 10
20
400 =
0
P
P = 100 A100 B i1
100 100 200 i2
S = 200 R
0
=
R = 100 10
20
100 0
=
S
B
10V
10V 10V
(b)
(a) (c)
P R
Here, we see that Q
= (Bridge is balanced)
S
Therefore, resistance between A and B can be ignored and equivalent simple circuit can be drawn
as shown in figure (c).
The voltmeter will read the potential difference across resistance Q.
10 1
Currents i1 = i2 A
100 200 30
1 20
Potential difference across voltmeter = Qi1 = (200) V= V
30 3
20
Therefore, reading of voltmeter will be V.
3
ELECTRICITY
QUIZRR 91
Ex a m p l e 5 0
Find the emf (V) and internal resistance (r) of a
single battery which is equivalent to a parallel r2 V2
+ ă
combination of two batteries of emfs V1 and V2 A B
and internal resistances r1 and r2 respectively,
with polarities as shown in figure. r1 V1
Solution. (i) Equivalent emf (V) of the battery : Potential difference across the terminals of the
battery is equal to its emf when current drawn from the battery is zero. In the given circuit :
Net em f V V2
Current in the internal circuit i = 1
Tot a l resist an ce r1 r2
i=0 i i=0
r2 V2
A B
r1 V1
V V2 V1 r2 – V2 r1
VA ă VB = V1 – 1 r1
r1 r2 r1 r2
V1 r2 – V2 r1
V= r1 r2
1 1 1
= r r
r 1 2
r1 r2
r= r1 r2
ELECTRICITY
92 QUIZRR
Ex a m p le 5 1
The thin uniform wire AB of length 1 m, an
unknown resistance X and a resistance of 12
are connected by thick conducting strips, as
shown in the figure. A battery and a
galvanometer (with a sliding jockey connected
to it) are also available. Connections are to
x 12
be made to measure the unknown resistance
X using the principle of Wheatstone bridge. A B C D
Answer the following questions.
(a ) Are there positive and negative terminals on the galvanometer ?
(b) Copy the figure in your answer book and show the battery and the galvanometer
(with jockey) connected at appropriate points.
(c ) After appropriate connections are made, it is found that no deflection takes place in
the galvanometer when the sliding jockey touches the wwire at a distance of 60cm from A.
Obtain the value of the resistance X.
Solution. (a) There are no positive and negative terminals on the galvanometer because only zero
deflection is needed.
(b)
x 12
j B
A D
(c) AJ = 60 cm
BJ = 40 cm
If no deflection is taking place. Then the Wheasone bridge is said to be balanced. Hence,
X R BJ
= R
12 AJ
X 40 2
or =
12 60 3
or x = 8
ELECTRICITY
ELECTROSTATICS
QUIZRR 3
ELECTROSTATICS
1 . I N T RODU CT I ON
When we comb our hair on a dry day and bring the comb close to tiny pieces of paper, we note
that they are swiftly attracted by the comb. Similar phenomena occur if we rub a glass rod or an amber
rod with a cloth or with a piece of fur. Why does this happens? What really happens in an electric
circuit? How do electric motors and generators work?
The answers to all these questions come from a branch of physics known as electromagnetism,
the study of electric and magnetic interactions. These interactions involve particles that have a property
called electric charge, an inherent property of matter that is as fundamental as mass.
We begin our study of electromagnetism in this chapter by the electric charge. We will see that
it is quantized and obey a conservation principle. Then we will study the interactions of electric charges
that are at rest, called electrostatic interactions. These interactions are governed by a simple relationship
known as CoulombÊs law. This law is more conveniently described by using the concept of electric
field.
2 . EL ECT RI C CH A RGE
+ +
Two positively charged ions repel each other
+
A positively charged ion and electron repel each other
Atoms are composed of three particles with an electric charge : electrons, protons and uncharged
neutrons. Electrons have a negative charge, and protons have a positive charge. The negative charge
of the electron has the same magnitude, or amount of charge, as the positive charge of the proton.
Electrical charge is quantized, with the smallest unit of electrical charge found in a single electron
or proton. If we double the number of electrons or protons, we double the amount of negative or positive
electric charge. If we triple the electrons or protons, we triple the electric charge. By increasing the
amount of single unit electric charges, we have a net charge. A net charge is the summed value of
single electrical charges.
We donÊt typically refer to single units of charge. We typically refer to net charges. Atoms are
composed of protons and electrons and generally have a net neutral electrical charge. The magnitude
of total positive and total negative charges inside the neutral atom is equal. If we were to remove an
electron (called a Valence electron) from the atom, the net charge of the atom would be positive. There
is now more positive charge in the atom. This atom is called a positive ion, and is quantized with a net
positive charge.
What happens when we keep removing Valence electrons from a group of neutral atoms? We end
up with a group of positive ions and a group of free Valence electrons. Each group has a net positive
or net negative electrical charge. We can call each charged group a body of charge.
ELECTROSTATICS
4 QUIZRR
We canÊt really say what an electrical charge is, but only describe its behavour. Simply started two
single protons or two single electrons repel each other. A single proton and a single electron attract each
other. The same can be said for net charges - a net negative charge will attract a net positive charge.
Points to Remember
there are two kinds of charge, positive and negative.
like charges repel, unlike charges attract
positive charge comes from having more protons than electrons; negative charge comes from
having more electrons than protons
charge is quantized, meaning that charge comes in integer multiples of the elementary charge e
charge is conserved
Probably everyone is familiar with the first three concepts, but what does it mean for charge to
be quantized? Charge comes in multiples of an indivisible unit of charge, represented by the letter e.
In other words, charge comes in multiples of the charge on the electron or the proton. These things
have the same size charge, but the sign is different. A proton has a charge of + e, while an electron
has a charge of e.
Electrons and protons are not the only things that carry charge. Other particles (positrons, for
example) also carry charge in multiples of the electronic charge. Those are not going to be discussed,
for the most part, in this course, however.
Putting „charge is quantized‰ in terms of an equation, we say :
q = n e
q is the symbol used to represent charge, while n is a positive or negative integer, and e is the
electronic charge, 1.60 10 19 Coulombs.
2 .1 T h e L a w o f Co n s e r va t i o n o f Ch a r g e
The Law of conservation of charges states that the net charge of an isolated system remains
constant.
If a system starts out with an equal number of positive and negative charges, thereÊs nothing we
can do to create an excess of one kind of charge in that system unless we bring in charge from outside
the system (or remove some charge from the system). Likewise, if something starts out with a certain
net charge, say + 100 e, it will always have + 100 e unless it is allowed to interact with something
external to it.
Charge can be created and destroyed, but only in positive-negative pairs.
Table of elementary particle masses and charges :
Particle mass charge
31 10
electron 9.11 10 kg 1.60 10 C ( e)
27 10
proton 1.672 10 kg + 1.60 10 C (+ e)
27
neutron 1.674 10 kg 0
ELECTROSTATICS
QUIZRR 5
4 . CH A RGI N G OF A B ODY
Mainly there are following there methods of charging a body :
(i) Charging by rubbing
The simplest way to experience electric charges is to rub certain bodies against each other. When
a glass rod is rubbed with a silk cloth the glass rod acquires some positive charge and the silk cloth
acquires negative charge by the same amount. The explanation of appearance of electric charge on
rubbing is simple. All material bodies contain large number of electrons and equal number of protons
in their normal state. When rubbed against each other, some electrons from one body pass onto the
other body. The body that donates the electron becomes positively charged while that which receives
the electrons becomes negatively charged. For example when glass rod is rubbed with silk cloth, glass
rod becomes positvely charged because it donate the electrons while the silk cloth becomes negatively
charged because it receives electrons. Electricity so obtained by rubbing two objects is also known as
frictional electricity. The other places where the frictional electricity can be observed are when
amber is rubbed with wool or a comb is passed through a dry hair. Clouds also become charged by
friction.
Ebonite rod
++
+ ++ ++ + ++
+
+ + +
+
Grounding +
++ + +
+ Metal +
++ +
+ + wire
+ sphere +
+ +
+
+
++
++
++
+
Insulated Earth
stand
ELECTROSTATICS
6 QUIZRR
In figure (a) a negatively charged rod brought close to (but does not
+
touch) a metal sphere. In the sphere, the free electrons close to the rod move Ebonite rod
+
to the other side (by repulsion). As a result, the part of the sphere nearer + Plastic
to the rod becomes positively charged and the part farthest from the rod +
+
Ex a m p l e 1
How many electrons are there in one coulomb of negative charge?
Solution :
The negative charge is due to the presence of excess electrons, since they carry negative charge.
Because an electron has a charge whose magnitude is e = 1.6 10 19 C, the number of electrons is
equal to the charge q divided by the charge e on each electron. Therefore, the number n of electrons
is
ELECTROSTATICS
QUIZRR 7
q 1.0
n
e 1.6 1019
Ex m p l e 2
If we comb our hair on a dry day and bring the comb near small pieces of paper, the comb
attracts the pieces, why?
Solution :
This is an example of frictional electricity and induction. When we comb our hair it gets positively
charged by rubbing. When the comb is brought near the pieces of paper some of the electrons accumulate
at the edge of the paper piece which is closer to the comb. At the farther end of the piece there is
deficiency of electrons and hence, positive charge appears there. Such a redistribution of charge in a
material, due to presence of a nearby charged body is called induction. The comb exerts larger attraction
on the negative charges on the paper piece as compared to the repulsion on the positive charge. This
is because the negative charges are closer to the comb. Hence, there is a net attraction between the
comb and the paper piece.
Ex a m p l e 3
Does the attraction between the comb and the piece of papers last for longer period of time?
Solution :
No, because the comb loses its net charge after some time. The excess charge of the comb transfers to
earth through our body after some time.
Ex a m p l e 4
Can two similarly charged bodies attract each other?
Solution :
Yes, when the charge on one body (q1) is much greater than that on the other (q2) and they are
close enough to each other so that force of attraction between q1 and induced charge on the other
exceeds the force of repulsion between q1 and q2. However two similar point charges can never attract
each other because no induction will take place here.
Ex a m p l e 5
Does the mass of a body change during charging ?
Solution :
Yes, as charging a body means addition or removal of electrons and electron has a mass.
ELECTROSTATICS
8 QUIZRR
Ex a m p l e 6
Why a third hole in a socket provided for grounding?
Solution :
All electric appliances may end with some charge due to faulty connections. In such a situation charge
will be accumulated on the appliance. When the user touches the appliance he may get a shock. By
providing the third hole for grounding all accumulated charge is discharged to the ground and the
appliance is safe.
5. COU L OM B ’S L A W
The interaction between charged objects is a non-contact force which acts over some distance of
separation. Every electrical interaction involves a force which highlights the importance of these three
variables. Whether it is a plastic golf tube attracting paper bits, two like-charged balloons repelling or
a charged Styrofoam plate interacting with electrons in a piece of aluminium, there is always two
charges and a distance between them as the three critical variables which influence the strength of the
interaction.
5 .1 Fo r c e a s a V e c t o r Qu a n t it y
The electrical force, like all forces, is typically expressed in units of Newtons. Being a force, the
strength of the electrical interaction is a vector quantity which has both magnitude and direction. The
direction of the electrical force is dependent upon whether the charged objects are charged with like
charge or positive charge and upon their spatial orientation. By knowing the type of charge on the two
objects, the direction of the force on either one of them can be determined with a little reasoning. In
the diagram below, objects A and B have like charge causing them to repel each other. Thus, the force
on object A is directed leftward (away from B) and the force on object B is directed rightward (away
from A). On the other hand, objects C and D have opposite charge causing them to attract each other.
Thus, the force on object C is directed rightward (toward object D) and the force on object D is directed
leftward (toward object C). When it comes to the electrical force vector, perhaps the best way to
determine the direction of it is to apply the fundamental rules of charge interaction (opposites attract
and likes repel) using a little reasoning.
Electrical force also has a magnitude or strength. Like most types of forces, there are a variety of
factors which influence the magnitude of the electrical force. Two like-charged balloons will repel each
other and the strength of their repulsive force can be altered by changing three variables. First, the
ELECTROSTATICS
QUIZRR 9
quantity of charge on one of the balloons will effect the strength of the repulsive force. The more
charged a balloon is, the greater the repulsive force. Second, the quantity of charge on the second
balloon will affect the strength of the repulsive force. Gently rub two balloons with animal fur and they
repel a little. Rub the two balloons vigorously to impart more charge to both of them, and they repel
a lot. Finally, the distance between the two balloons will have a significant and noticeable affect upon
the repulsive force. The electrical force is strongest when the balloons are closest together. Decreasing
the separation distance increases the force. The magnitude of the force and the distance between the
two balloons is said to be inversely related.
5 .2 Co u l o m b ’s L a w Eq u a t io n
The quantitative expression for the affect of these three variables on electric force is known as
CoulombÊs law. CoulombÊs laws states that the electrical force between two charged objects is directly
prportional to the product of the quantity of charge on the objects and inversely proportional to the
square of the separation distance between the two objects. In equation form, CoulombÊs law can be
stated as
K Q1 Q2
F
d2
where Q1 represents the quantity of charge on object 1 (in Coulombs), Q2 represents the quantity
of charge on object 2 (in Coulombs), and d represents the distance of separation between the two objects
(in meters). The symbol k is a proportionality constant known as the CoulombÊs law constant. The value
of this constant is dependent upon the medium that the charged objects are immersed in. In the case
of air, the value is approximately 9 109 Nm2/C2.
1
This constant k is often written as , where („epsilon-nought‰) is another constant. This
4 0 0
appears to complicate matters, but it actually simplifies many formulae that we will encounter in later
chapters. Thus, eq. can be written as,
1 q1 q2
Fe
4 0 r 2
Here,
1
9 109 N m2 / C2
4 0
Since CoulombÊs law applies to point charges, the distance d in the equation is the
distance between the centers of charge for both objects (not the distance between their nearest
surfaces).
The symbols Q1 and Q2 in the CoulombÊs law equation represent the quantities of charge on the
two interacting objects. Since an object can be charged positively or negatively, these quantities are
ELECTROSTATICS
10 QUIZRR
often expressed as „+‰ or „ ‰ values. The sign on the charge is simply representative of whether the
object has an excess of electrons (a negatively charged object) or a shortage of electrons (a positively
charged object). When using the „+‰ and „ ‰ signs in the calculation of force, the result will be that a
„ ‰ value for force is a sign of an attractive force and a „+‰ value for force signifies a repulsive force.
Mathematically, the force value would be found to be positive when Q1 and Q2 are like charge-either
both „+‰ or both „ ‰. And the force value would be found to be negative when Q1 and Q2 are of opposite
charge - one is „+‰ and the other is „ ‰. This is consistent with the concept that oppositely charged
objects have an attractive interactions and like charged objects have a repulsive interaction. In the end,
if youÊre thinking conceputally (and not merely mathematically), you would be very able to determine
the nature of the force-attractive or repulsive-without the use of „+‰ and „ ‰ signs in the equation.
Regarding CoulombÊs law following points are worth noting :
(1) CoulombÊs law stated above describes the interaction of two point charges. When two charges
exert forces simultaneously on a third charge, the total force acting on that charge is the
vector sum of the forces that the two charges would exert individually. This important
property, called the principle of superposition of forces, holds for any number of charges.
Thus,
Fnet F1 F 2 ... F n
(2) The electric force is an action reaction pair, i.e., the two charges exert equal and opposite
forces on each other.
(3) The electric force is conservative in nature.
(4) CoulombÊs law as we have stated above can be used for point charges in vacuum. If some
dielectric (insulator) is present in the space between the charges, the net force acting on each
charge is altered because charges are induced in the
q1 q2 Fe
molecules of the intervening medium. We will describe this Fe r
effect later. Here at this moment it is enough to say that In vacuum
the force decreases K times if the medium extends till
infinity. Here K is a dimensionless constant which depends on the medium and called dielectric
constant of the medium. Thus,
1 qq
Fe . 1 2 (in vacuum)
4 0 r2
Fe 1 qq 1 q1 q2
Fe' . 1 22 . (in medium)
K 4 0 K r 4 r 2
Points to Remember
1. In few problems of electrostatics LamiÊs theorem is very useful.
ELECTROSTATICS
QUIZRR 11
F2
According to this theorem, „if three concurrent forces F1 F and F3
2
F1
shown in figure are in equilibrium or if F1 F F3 0 , then
2
F1 F F
2 3
sin sin sin
F3
2. Suppose two charges q1 and q2 are placed in vacuum at a distance
r0 and the electric force between them is
1 qq
F0 . 1 22
4 0 r0
Now the same charges are placed in a dielectric constant K at a distance r (< r0) such that
the electric force between them
1 qq
F . 1 2
4 0 K r 2
remains the same of F = F0
1 qq 1 qq
Then . 1 22 . 1 22
4 0 r 4 0 r0
or r0 K r
Thus, we see that, for the electric force between two charges a distance r in a dielectric medium
Ex a m p l e 7
Suppose that two point charges, each with a charge of + 1.00 Coulomb are separated by a
distance of 1.00 meter. Determine the magnitude of the electrical force of repulsion between
them.
Solution : Given : Find :
Q1 = 1.00 C Felect = ???
Q2 = 1.00 C
d = 1.00 m
k Q1 Q2
Felect
d2
Felect = (9.0 109 N m2/C2) (1.00 C) (1.00 C) / (1.00 m)2
Felect = 9.0 109 N
The force of repulsion of two + 1.00 Coulomb charges held 1.00 meter apart is 9 billion Newtons.
ELECTROSTATICS
12 QUIZRR
This problem was chosen primarily for its conceptual message. Objects simply do not acquire
charges of the order of 1.00 Coulomb. In fact, more likely Q values are on the order of 109 or possibly
10 6 Coulombs. For this reason, a Greek prefix is often used in front of the Coulomb as a unit of charge.
Charge is often expressed in units of micro Coulomb (øC) and nano Coulomb (nC). If a problem states
the charge in these units, it is advisable to first convert to Coulombs prior to substitution into the
CoulombÊs law equation. The following unit equialencies will assist in such conversions.
1 Coulomb = 106 micro Coulomb
1 Coulomb = 109 nano Coulomb
Ex a m p l e 8
Two balloons with charges of + 3.37 øC and 8.21 øC attract each other with a force of 0.626
Newtons. Determine the separation distance between the two balloons.
Given Find :
6
Q1 = + 3.37 øC = + 3.37 10 C d = ???
6
Q2 = 8.21 øC = 8.21 10 C
Felect = 0.0626 N (use a - force value since it is repulsive)
k Q1 Q2
Felect
d2
k Q1 Q2
d2
Felect
kQ1 Q2
d
Felect
d
9.0 109 Nm2 / C2 8.21 10 6 C 3.37 106 C
0.062 N
d = + 1.99 m
Ex a m p l e 9
What is the smallest electric force between two charges placed at a distance of 1.0 m.
Solution :
1 qq
Fe . 12 2
4 0 r
9.0 10 9
we have Fe min
1.0 2 1.6 10 19
28
= 2.304 10 N Ans.
ELECTROSTATICS
QUIZRR 13
Ex a m p l e 1 0
Three charges q 1 = 1øC, q 2 = 2øC and q 3 = 3øC are placed on the vertices of an equilateral
triangle of side 1.0 m. Find the net electric force acting on charge q 1.
How to Proceed : Charge q will attract charge q (along the line joining q3
2 1
them) and charge q3 will repel charge q1. Therefore, two forces will act on q1, one
due to q2 and another due to q3. Since, the force is a vector quantity both of these
q1 q2
forces say F1 and F 2 will be added by vector method. Following are two methods
of their addition.
Solution :
Method 1 : In the figure,
1 qq
F1 F1 . 1 22
4 0 r
= magnitude of force between q1 and q2
=
1.02
2
= 1.8 10 N
1 qq
Similarly, F 2 F2 . 1 3
4 0 r 2
= magnitude of force between q1 and q3 q3
=
1.0 2 q1 F1
q2
2
= 2.7 10 N 120
F2
Now, Fnet F12 F22 2F1F2 cos120 Fnet
1
= 1.8 2 2.72 2 1.8 2.7 102 N
2
2
= 2.38 10 N
F2 sin 120
and tan
F1 F2 cos120
2.7 10 0.87 2
=
1.8 10 2.7 10 12
2 2
or = 79.2 .
ELECTROSTATICS
14 QUIZRR
2
Thus, the net force on charge q1 is 2.38 10 N at an angle = 79.2 with a line joining q1 and
q2 as shown in the figure. Ans.
Method 2 : In this method let us assume a co-ordinate axes with q1 at origin as shown in
figure.The co-ordinates of q1, q2 and q3 in this co-ordinate system are (0, 0, 0), (1 m, 0, 0) and (0.5
m, 0.87 m, 0) respectively. Now
F1 force on q1 due to charge q2 y q3
1 q1 q2
= . r1 r 2
4 0 3
r1 r 2
q1 q2
=
9.0 10 1.0 10 2.0 10 0 1 i→ 0 0 →j 0 0 k→
9 6 6
1.03
2
= 1.8 10 i→ N
and F 2 = force on q1 due to charge q3
1 q1 q3
= . r1 r3
4 0 3
r1 r3
=
9.0 10 1.0 10 3.0 10 0 0.5 i→ 0 0.87 →j 0 0 k→
9 6 6
1.0 3
2
= 1.35 i→ 2.349 →j 10 N
Therefore, net force on q1 is F F1 F 2
Note : Once you write a vector in terms of i→, →j and k→ , there is no need of writing the magnitude
Ex a m p l e 1 1
Two identical balls each having a density are suspended from a common point by two
insulating strings of equal length. Both the balls have equal mass and charge. In equilibrium
each string makes an angle with vertical. Now, both the balls are immersed in a liquid.
As a result the angle does not change. The density of the liquid is . Find the dielectric
constant of the liquid.
ELECTROSTATICS
QUIZRR 15
Solution :
Each ball is in equilibrium under the following three forces :
(i) tension, (ii) electric force and (iii) weight
So, LamiÊs theorem can be applied
T'
T
Fe'
Fe
W'
W In liquid
In vacuum
' Fe
In the liquid, Fe
K
W Fe
Applying LamiÊs theorem in vacuum sin 90 sin 180
W F
or e ...(i)
cos sin
W´ F'
Similarly in liquid, e ... (ii)
cos sin
Dividing Eq. (i) by (ii), we get
W Fe
W ' Fe'
W F
or K as e' K
W upthrust Fe
Vg
= (V = volume of ball)
Vg Vg
or K Ans.
Note : In the liquid Fe and W have been changed. Therefore, T will also change.
ELECTROSTATICS
16 QUIZRR
k Q1 Q 2 G m1 m2
Felect Fgrav
d2 d2
( k 9 10 9 Nm2 / C2 ) G 6.67 10 11
Nm2 / kg 2
The two equations have a very similar form. Both equations show an inverse square relationship
between force and separation distance. And both equations show that the force is proportional to the
product of the quantity that causes the force-charge in the case of electrical force and mass in the case
of gravitational force. Yet there are some striking differences between these two forces. First, a comparison
of the proportionality constants - k versus G - reveals that the CoulombÊs law constant (k) is significantly
greater than NewtonÊs universal gravitation constant (G). Subsequently a unit of charge will attract
a unit of charge with significantly more force than a unit of mass will attract a unit of mass. Second,
gravitational forces are only attractive; electrical forces can be either attractive or repulsive.
The inverse square relationship between force and distance which is woven into the equation is
common to both non-contact forces. This relationship highlights the importance of separation distance
when it comes to the electrical force between charged objects.
7 . EL ECT RI C FIEL D
A charged particle cannot directly interact with another particle kept at a distance. A charge
produces something called an electric field in the space found it and this electric field exerts a force on
any other charge (except the source charge itself) placed in it.
Thus, the region surrounding a charge or distribution of charge in which its electrical effects can
be observed is called the electric field of the charge or distribution of charge. Electric field at a point
can be defined that all these are functions of positions r x, y, z . The field propagates through space
with the speed of light, c. Thus, if a charge is suddenly moved, the force it exerts on another charge
a distance r away does not change until a time r/c later.
7 .1 Ele c t ri c Fi e l d St r e n g t h E
Like its gravitational counterpart, the electric field strength (often called electric field) at a point
in an electric field is defined as the electrostatic force Fe per unit positive charge. Thus, if the
electrostatic force experienced by a small test charge q0 is Fe , then field strength at that point is
defined as,
Fe
E lim
q0 0 q0
The electric field is a vector quantity and its direction is the same as the direction of the force Fe on
a positive test charge. The SI unit of electric field is N/C. Here it should be noted that the test charge
ELECTROSTATICS
QUIZRR 17
q0 does not disturb other charges which produces E . With the concept of electric field, our description
of electric interactions has two parts. First, a given charge distribution acts as a source of electric field.
Second, the electric field exerts a force on any charge that is present in this field.
7 .2 A n Ele c t ri c Fi e l d l e a d s t o a Fo r c e
Suppose there is an electric field strength E at some point in an electric field, then the electrostatic
force acting on a charge + q is qE in the direction of E , while on the charge q it is qE in the opposite
direction of E .
7 .3 Ele c t ri c Fi e l d i n t e n s i t y d u e t o a Po i n t Ch a r g e
The force experienced by a unit positive charge at a distance r due to point charge ÂqÊ is
called electric field intensity at that point due to the charge q.
+1
q r P
Kq q
F 12 2 where K = 9 109 Nm2/C2
r
1
and K where 0 = 8.85 10 12
C2/Nm2
4 0
We define electric field at a point as
KQ.1 KQ
E 2 2
r r
KQ Q
E or E
r 2 4 0 r 2
In vector form
P (x2, y2, z2)
E E. n→ r
r
KQ KQ
= 2 . n→ r 3 r
r r Q (x1, y1, z1)
KQ
E 3 r r x2 x1 i y2 y1 j z2 z1 k
r
Q
E r
4 r3
0
l Net field intensity in the space due to a number of point charges at a point is given by principle
of superposition.
ELECTROSTATICS
18 QUIZRR
E E1 E 2 E3 ..... E n
[Principle of superposition is applicable only when the effect remains unchanged, due to presence
of others (charges in this case).]
l Net force experienced by a point charge in the electric field
F qE
Ex a m p l e 1 2 +q a +q
Four identical charges are placed at the corners of a square of side a . Find
the net field at a point lying on the perpendicular drawn to the plane of a a
a
square and passing through its centre at a height .
2 +q a +q
Solution :
Observe that the net field lie in the vertical direction as the opposite charges cancel out each others
horizontal component of the electric field.
Esin
E
Enet 4E sin
Ecos
Kq 1 +q 45 a/ 2
+q
= 4 2
a 2
a/ 90
2
2 2 Kq
E 2 (away from centre)
a
8 . EL ECT RI C POT EN T I A L
„Potential is the potential energy per unit charge.‰ Electric potential at any point in an electric
field is defined as the potential energy per unit charge, same as field strength is defined as the
force per unit charge.
U
Thus V or U = q0V
q0
The SI unit of potential is Volt (V) which is Joule per Coloumb. So 1 Volt = 1 J/C
ELECTROSTATICS
QUIZRR 19
If we want a picture of a field, we usually think of many lines lines of „flux‰ emanating radially
away from the charge :
For negative charges, the lines point inward. These flux lines indicate the direction of a positively
charged test particle would move due to the electric force between it and the configuration charge. Note
that the „density per unit area‰ of the flux decreases as the distance from the center increases. That
is, the lines are much closer together when nearer to their origin. Since we live in three spatial
dimensions, and the surface area of a sphere centered on the charge increases with the square of the
distance, we have a model for the inverse square force law : the magnitude of the force is proportional
to the density of the field lines („flux density‰), which decreases with the square of the distance. We
can also see why spherical source is equivalent to a point source at its center : the flux lines emanating
from the surface can be extrapolated inwards to the center with no change in the physical forces felt
outside the surface.
The electric field can be thought of as the gradient of the electrical potential field. But since the
electric field is a force per unit charge, the electric potential must be an energy per unit charge. It is
a scalar field, having a magnitude (but no direction) at every point in space.
q
V r
4 0 r
also U = q0 V
As with the cases of the electric field and force, the electrical potential field superposes lineraly,
so that the field at a point due to a collection of configuration charges is equal to the sum of the fields
due to the individual charges, at that point. For example, if in a vacuum two protons (q c1 ) lie in the
xy plane at (x1, y1) = ( 2, 1) Angstroms and an electron (q c2 ) lies at the origin (x 2, y 2) = (0, 0), the
energy of an electron (qt) at (xt, yt) = (1, 1) Angstroms would be
U =U tc1
+ U tc2
= qt (V c1
+ V ))
c2
( e) (2e) ( e) (e)
=
10
4 0 3 10 4 0 2 10 10
20
= 9.33 10 J,
ELECTROSTATICS
20 QUIZRR
where the distances between charges are
r xt x i 2 y t y i 2
and „i‰ denotes either of the configuration charges.
8 .1 Re l a t io n b e t w e e n Ele c t ri c Po t e n t i a l a n d Ele c t ri c Fi e l d
In order to compute the direction of the force felt by a test charge in a potential field, one can take
V
Ex
x
If we were to plot the electrical potential field due to a pair of configuration charges, we would
see that there are lines, called „equipotentials‰, on which the electrical potential has the same value
everywhere :
+2
The direction of the gradient (and hence the electric field) is perpendicular to the equipotential
lines since its component along (tangent to) the equipotentials is zero (V is a constant). We can
therefore immediately trace the path of any test particle in the field, since charges move in the
direction of least energy.
8 .1 A b s o l u t e Po t e n t i a l a t So m e Po i n t
Suppose we take the point b at infinity and as a reference point assign the value Vb = 0, the
equations can be written as
Thus, the absolute electric potential at point a in an electric field can be defined as the work done
ELECTROSTATICS
QUIZRR 21
in displacing a unit positive charge from infinity to a by the external force or the work done per unit
positive charge in displacing it from a to infinity.
Note : Following three formulae are very useful in the problems related to work done in electric
field.
(Wa )
b electric force
= q0 (Va Vb)
(Wa )
b external force
= q0 (Vb Va ) = (Wa )
b electric force
(W )
a external force
= q0Va
Here, q0, and Vb are to be substituted with sign.
Ex a m p l e 1 3
The electric potential at point A is 20 V and at B is 40 V. Find the work done by an
external force and electrostatic force in moving an electron slowly from B to A.
Solution :
Here, the test charge is an electron, i.e.
19
q0 = 1.6 10 C
VA = 20 volt
and VB = 40 volt
Work done by external force
(WB )
A external force
= q0 (VA VB)
19
= ( 1.6 10 ) [(20) ( 40)]
18
= 9.6 10 J Ans.
Work done by electric force
(WB )
A electric force
= (WB )
A external force
18
= (9.6 10 J)
18
= 9.6 10 J Ans.
Note : Here we can see that the electron (a negative charge) moves from B (lower potential) to
A (higher potential) and the work done by electric force is positive. Therefore, we may conclude that
whenever a negative charge moves from a lower potential to higher potential work done by the electric
force is positive or when a positive charge moves from lower potential to higher potential the work done
by the electric force is negative.
Ex a m p l e 1 4
Find the work done by some external force in moving a charge q = 2 øC from infinity to a
point where electric potential is 104 V.
Solution :
Using the relation,
(W )
a external force
= q0Va
We have, (W )
a external force
= (2 10 6) (104)
= 2 10 2 J Ans.
ELECTROSTATICS
22 QUIZRR
8 .4 Ele c t ri c Po t e n t i a l Du e t o a Po i n t Ch a r g e q
Work done against the field in slow moving a unit positive charge from infinity to the point in the
field is called electric potential at that point.
1 qq
. 0
4 0 r
From the definition of potential, V U
q0 q0
1 q
V .
4 0 r
Here, r is the distance from the point charge q to the point at which the potential is evaluated.
If q is positive, the potential that it produces is positive at all points; if q is negative, it produces a
potential that is negative everywhere. In either case, V is equal to zero at r = .
8 .5 Ele c t ri c Po t e n t i a l Du e t o a Sy s t e m o f Ch a r g e s
Just as the electric field due to a collection of point charges is the vector sum of the fields produced
by each charge, the electric potential due to a collection of point charge is the sclar sum of the potentials
due to each charge.
1 qi
V
4 0 i ri
In this expression, ri is the distance from the ith charge, qi, to the point at which V is evaluated.
For a continuous distribution of charge along a line, over a surface, or through a volume, we divide
the charge into elements dq and the sum in the above equation becomes an integral,
1 dq
4 0 r
V
1 q 1 dq
Note : In the equation V 4 ri or V 4 r if the whole charge is at equal distance
0 i i 0
1 q
V . net
4 0 r0
Where qnet is the algebraic sum of all the charges of which the system is made.
ELECTROSTATICS
QUIZRR 23
10V 40V
20V 30
30V 20 V
V E
40V 10V
(ii) For a point charge, equipotential surface are concentric spherical surfaces centred at q.
(V1 > V2 > V3 when q < 0) V1
V2
V3
2
ELECTROSTATICS
24 QUIZRR
T o f i n d t h e f i e l d i n t e n s i t y w h e n Eq u i p o t e n t ia l Su r f a c e a r e g i v e n
Ex a m p l e 1 5
The figure shows parallel equipotential surfaces which are parallel planes. Find the
magnitude and direction of field intensity.
Y
40V 30V 20V 10V
30
30
c X (cm)
5 10 15 20
Solution :
Since the equipotential surface are equidistant and parallel, hence electric field intensity is
magnitude an perpendicular to the equipotential surfaces
30
dV V d V
Ex dx is uniform
dx x
40 30
E cos60 = V/m
5 10 102
E 10
102 V / m 2 102 V/m
2 5
E = 4 102 V/m
ELECTROSTATICS
QUIZRR 25
Ex a m p l e 1 6
The given figure represents part of concentric spheres which are equipotential surfaces.
Find the magnitude of the field.
15V
20V
30V
60V
r=0
r = 10 cm
0 cm
r=2 30 c
m m
r3 = 40c
r4 =
dV
Er =
C / r2
dr
E
6 E
E= 2
r
Ex a m p l e 1 7
Find the relation between the field intensity at A, B and C.
50V 40V 30V 20V
Solution : (VC > VB > VA)
dV
Reason : E
dr
A B C
Since V is same between two equipotential surface.
1
and E where r is small E will be large.
r
In general rule : To find the field due any type of charge distribution.
ELECTROSTATICS
26 QUIZRR
use dE dEx
9 . Ele c t ri c Po t e n t i a l A n d f i e l d i n t e n s i t y d u e t o V a r io u s Ch a r g e Di s t r ib u t io n
I. U n i f o r m l y c ha r g ed c i r c u l a r r i n g
(i) Elec tric a l potential
dq
O x P
Consider an infinitesmal element of charge dq on the circumference of the ring, then the electric
potential due to this charge is
dq
dV
4 0 x2 R 2
Integrating,
1
V dq
4 0 x2 R 2
Q
V
4 0 x2 R2
This result for potential remains same even if the charge distribution is non-uniform.
(ii) Electric field intensity at an axial point
dq
Esin
E
R Ecos
+
x p Ecos Consider small elements of charge
E dq on the circumference
Esin
dq
ELECTROSTATICS
QUIZRR 27
Note that the net field will be in the axial direction, since the vertical components will cancel out.
d E x dE cos
dq x
d Ex x
4 0 R R cos
1
2 2
x 2
/2
x2 2 2
R x
x Qx
E dq
4 0 R 2
2 3/ 2
3/2
x 4 0 R 2 x2
Qx
E
3/2
4 0 R 2 x2
1 Q
V
4 0 R x2
2
dV Q 2x Qx
and Er 3/2
dx 4 0
3/ 2
2 R2 x2 4 0 x2 R 2
(1)
V
KQ
R
R 2 O R 2 r
(shape change from
Convex to Concave)
ELECTROSTATICS
28 QUIZRR
E
R O
2
R 2 R 3 r
2
(shape change from
Convex to Concave)
Ex a m p l e 1 8
A uniformly charged ring of charge density and radius R is located in y-z plane with its
centre at origin. Now a point charge q is projected with velocity v0 from a point P as
shown. Find the minimum value of v0 so that the particle will not return to point P.
Solution :
Let us consider that the ring is placed in the Y-Z plane centred at origin. The point P is then
taken to be at P R 3, 0, 0 on the X-axis. The point charge is projected towards the centre of the ring
along the X-axis.
Y
O P (R 3, 0 , 0)
X
v0 q,m
Concept : v0 must be sufficient so that the particle just reaches the centre for minimum required
velocity.
So, we take VP = V0 {Given}
and vfinal = 0
Now, applying energy conservation
ELECTROSTATICS
QUIZRR 29
K + U = 0
1 1 2
m v P m v final q V0 VP
2
2 2
Q Q
also, V0
4 0 R 2 4 0 R
Q = (2R)
(2R)
V0
4 0 R
2R 2R
VP
4 0 2R
2
4 0 R2 R 3
1 2 2R 1 1
m v0 q R 2R
2 4 0
1 2 q
m v0
2 40
q
v0
2m0
Ex a m p l e 1 9
Two uniformly charged ring are separated by a distance R / 3 . A point charge q of mass
m is projected with initial velocity v0 from A to B. Find the velocity when it reaches the
centre of negetively charged ring.
+ ,R ,R
q
O1 R O2
3
Solution :
Let the two rings be centred at O1 and O2 having charge densities + and rspectively.
A charge q is projected with a velocity v0 from O1 toward O2.
Let v be the velocity when it reaches O2. Now, potential at centre O1
R R
VO
20 R 2
1
2 0 R 2 R 3
ELECTROSTATICS
30 QUIZRR
R R
= 2 R 4 R
0 0
VO
1 40
also, potential at O2
R R
VO
20 R
2 2
2 0 R 2 R 3
= 2 4
0 0
VO
2 40
1
2 1 2
2 mv 2 mv0 q VO1 VO 2
1
2
2
m v2 v0
q
20
2 q
v v0
m0
Ex a m p l e 2 0
If a negative charge is slightly displaced from the centre of a uniformly charged ring. Show
that it will execute SHM, and find its time period.
Solution :
When the charge is slightly displaced, it experiences a force towards the centre of the ring as the
electric field due to the ring is directed away from it, hence a negatively charged particle will experience
a force towards its centre.
Hence, Net force = ma
d2 x Q
or qE m
dt2
P
Qx
E x –q
3/2
and 40 R 2 x2
ELECTROSTATICS
QUIZRR 31
d2 x qQx
m
x
2 3/2
dt 4 0 2
R2
d2 x qQx
m
dt 4 R 3 {equation for S.H.M.}
2
0
4 0 m R 3
T 2
Qq
Ex a m p l e 2 1
A small charged particle of mass m and charge q is suspended vertically at one end of ideal
spring, and initially lying at the centre of ring in of charge Q in equilibrium. When it is
slightly displaced, find its
(a) time period of oscillation
(b) required condition to execute SHM
Solution :
Similar to the example above, the particle experiences a force qE after being displaced. Hence the
force balance in this condition is
d2x qQx
m 2 kx
dt
3/2
4 0 x2 R 2
K
qQ
= k x
40 R3 q Q1R
kx
4 0 R 3
T 2
4
(a) qE
0 R 3 k qQ
qQ
K
4 0 R 3
Ex a m p l e 2 2
Find the field intensity at the centre due to a uniformly charged circular arc having an
angle 0 subtended at the centre.
ELECTROSTATICS
32 QUIZRR
Solution : Y
Consider an element of thickness Rd making an angle with Rd
the Y-axis. The electric field d E due to this element at the centre
d
R
is directed at angle from the negative Y-axis. The component of this
field along the X-axis will cancel out with the compound of electric
field from the corresponding element making an angle from the X
Y-axis. Thus we will have a net electric field in the Y-axis direction.
0
dEcos
2
R d cos dE
Enet d E cos
0 4 0 R 2
2
0
0
= sin 2 0 2sin 2
4 0 R 4 0 R
2
Enet sin 0
2 0 R 2
Point to Remember
sin 0
0 Q 2
Electric field intensity due to a uniformly charged arc = sin
2 0 R 2 4 0 0
2
subtending an ange 0 at centre
9 .1 Fi e l d i n t e n s i t y a t t h e c i r c u m f e r e n c e o f t h e r in g
= 2 (d = 2d) { is the exterior angle}
Also, r = 2R cos (triangle in semi-circle) dq=Rd
Rd =(2Rd)
dq = (2R d)
Net field at point P will be along x-axis because of r
d
symmetric cancellation of y components.
dEcos P
O
E d Ecos
dE
K dq
= 2 cos
r
ELECTROSTATICS
QUIZRR 33
/2
k 2R d
= cos
/2 2R cos 2
s
co
k 2R d
2R
/2
cos
r=
=
/ 2 2R cos
2
/2 2R
k
= 2R
sec d
/2
E = indeterminate
But note when will be some function of E will may become determinate.
Potential energy at any point due to a point charge associated with a charge
F qE
U = qV
Kq1 q2
U
r0
Ele c t ri c p o t e n t ia l a t t h e c i r c u m f e r e n c e o f t h e r in g
kdq k 2 R d k d
dV
r 2R cos cos
/2
V k sec d = (indeterminate)
/2
(Note again that though this limit is indeterminate you may calculate it when varies with )
Ex a m p l e 2 3
A point charge q is brought to the centre from the circumference of a non-uniformly
charged ring of charge density = 0 cos . Find the work required by the external agent
in doing so.
Solution : Now in this question potential is not indeterminate
/2
V K0 sec cos d K0
/2
Now, calculate the potential at the centre
ELECTROSTATICS
34 QUIZRR
2
K 0 cos R d
dV R
0
2
V K0 sin 0 0
Work required by external agent
Wex = U
= q (V0 Vp) = q (0 k0)
= kq0
Ex a m p l e 2 4
A point charge q is placed at the centre of a uniformly charged () circular ring. Find the
increase in the radius of ring if Y = YoungÊs modulus and area of cross-section = S
Solution :
Consider the force on a small element of the ring of charge dQ due to point charge q.
K q dQ
F
R2
As can be seen from the figure, the net force of tension on the
d d
element AB comes towards the centre which is Tcos Tcos
2 A B 2
d T
2T sin T d T
2
Hence, d d
2 2
F = Td
K q dQ
Td
R2
Now, dQ = (Rd)
K R q K q
T 2
R R
also, using YoungÊs modulus
T
Y=
S
l
l
T R
Y { l = 2R}
S R
Kq R K q
R
R YS YS
ELECTROSTATICS
QUIZRR 35
dx
P(r,0,0)
R
For analyzing a charged disc, consider a ring of thickness dx at a distance x from the centre;
dq = (2x) dx {charge on the ring of thickness dx}
Now, Potential at P due to the ring
K dq
dV
x2 r 2
R
K 2x dx
V
0 x2 r 2
R
x dx
= K 2
0 x2 r 2
V = K 2 R 2
r2 r
(ii) Electric Field Intensity
Now, we know that
dV 2r
E 1
dr 2 0 2 R 2 r 2
r
E 1
2 0 R 2 r 2
ELECTROSTATICS
36 QUIZRR
V
E
R
2 0
2 O
r
r
Plot of potential v/s distance from the disc Plot of field intensity v/s distance from the disc
If r 0, E
2 0
Thus, near a uniformly charged surface, E can be taken =
2 0
Q
The shell has a charged density and radius R,
4R2
So, charge on a ring of thickness Rd
dq = (2R sin ) R d
R l
dq d
dV
4 0 l p
r
dV
2R 2 sin d
4 0 l
KQ
2R sin l d KQ
2 sin d
=
4 R 2 2 l
r2 R2 l2
cos
2rR
2rR cos = r2 + R2 l2
2r R sin d = 2l dl
ELECTROSTATICS
QUIZRR 37
sin d
dl = rR
l
rR
dl
r R r R
KQ KQ KQ
V dV
2 rR 2rR r
r R
KQ
V when r > R
r
for r < R
R r
KQ dl KQ
V dV , r R
2rR R
Rr
Su m m a r i zi n g (U n i f o r m l y c h a r g e d s h e l l)
(i) For any external part (r > R)
(a) Electric potential
Q
=
4 0 r
dV Q
(b) Electric field =
dr 4 0 r 2
Note : For an external point a shell has similar formulae as a point charge hence this point
can be very useful for some objective questions.
dV
(b) Electric field = 0
dr
V
E
O R r R r
Plot of potential v/s distance Plot of electric field intensity V/s
from the centre of shell. distance from the centre of shell.
ELECTROSTATICS
38 QUIZRR
Ex a m p l e 2 5
A uniformly charged circular ring is placed at R / 3 from a uniformly charged shell. Find
the force of interaction between the sphere and the ring.
Solution :
Qx
E R
3
2 2
4 0 x R 3
2R x
E
3/2
4 0 x2 R 2
B Q, R
Rx 3
E
2
2 x R 2 3/2
0 ER
3
16 R 0
Since a shell can be treated as a point charge outside its surface.
Q 3
F QE
16 O R
I V . U n i f o r m ly Ch a r g e d Sp h e r e
(A) Electric Potential
To calculate electric potential at a point inside the sphere, we divide the entire spherical volume into
two spherical parts I and II i.e. the region containing charge outside the point and the region inside
the point.
I
II
r
Q
4 3
R
3
ELECTROSTATICS
QUIZRR 39
dq
d V1 K
r x
{Using the formula for potential of external point for concentric shell} r
r r
dq 4 3
V1 K , dq 3 r
0
r 0
K 4
V1 r 3
r 3
Q
4 3
R
3
K Q r2
V1
R3
Consider Region I
dq
dV2 = K , dq = (4 x2) dx
x
R
4x dx
2
V2 K
x
r
dx
R x r
V2 = K (4) xdx
r
R
R2 r2
= 4 K
2
4 Q K R 2 r2
= 4 R3 2
3
V2
3 KQ
2 R3
R 2
r2
Hence, net potential (Vnet)
K Qr 2
Vnet V1 V2
R 3
3K Q
2R 3 R 2
r2
KQ
2R 3 3R 2
r2
ELECTROSTATICS
40 QUIZRR
Su m m a r i zi n g f o r a u n i f o r m ly c h a r g e d s p h e r e :
(i) For an internal point (r < R)
(ii) r > R
Net potential outside the sphere
K KQ Q
=
r dq
r 4 O r
dV Q
Net field = dr
4 O r 2
Note : A uniformly charged sphere also behave as a point charge for any external point.
E
V
3KQ
KQ
2R
R2
KQ
R
O r=R r
O r=R
1 0 . EL ECT RI C FIEL D I N T EN SI TY I N A CA V I T Y
Ex a m p l e 2 6
Find the field intensity in the region of intersection of two uniformly charged sphere with
volume charge density + and . The distance between centres is l.
ELECTROSTATICS
QUIZRR 41
Solution :
+ –
P
r1 r2
O l O'
At the point P, electric field due to sphere O and O´ can be calculated and added vectorially
r1 ( ) r2
EO , E O'
3 O 3 O
Enet E E O E O '
r1 r2
l
E 3 3 r1 r2 3 OO' = 3
3
O O O O O
Remark :
(i) Thus, it can be easily seen that the field intensity comes out to be consistent inside the
common area and is directed from positive to negative centres of the sphere.
(ii) Here, and must be same.
Ex a m p l e 2 7
CAVITY (it is a special case of previous question).
A sphere with centre O is filled with a charge density + . There exists a cavity inside this sphere
which is also a spherical cavity with centre O´ at a distance l from the centre O. Find the electric field
strength at a point P inside the cavity ?
+
P
r1 r2
O'
O l
ELECTROSTATICS
42 QUIZRR
Solution :
Concept : Fill the cavity with charge densities + and so that the cavity is filled.
This makes it a special case of previous example where the common volume between the 2 spheres
is same as one of the sphere.
Hence, E0
r1
, E0 '
r 2
3 0 3 0
r1 r2
E E0 E0 '
3 0
l
= r1 r2 l
3 0
Ex a m p l e 2 8
Find the field intensity at the centre of a charged spherical shell whose surface charge
density varies as a . r where a is a constant vector and r is the radius vector.
Solution :
For convenience, take the direction of the a vector along the diametric line, and the angle is
measured from the a vector direction.
dq R cos
(d
dqx
dE 3
3 a
4 O R 4 O R O
aR
E
and directed opposite to constant vector a
3
O
ELECTROSTATICS
QUIZRR 43
Ex a m p l e 2 9
Three point charges q 1 = 1øC, q 2 = 2øC and q 3 = 3 øC are placed at (1 m, 0, 0), (0, 2m, 0)
and (0, 0, 3m) respectively. Find the electric potential at origin.
Solution :
The net electric potential at origin is,
1 q1 q2 q3
V
4 0 r1 r2 r3
Substituting the values, we have
1
V 9.0 109 2
3
10
1.0 2.0 3.0
6
Ex a m p l e 3 0
A charge q = 10 øC is distributed uniformly over the circumference of a ring of radius 3 m
placed on x-y plane with its centre at origin. Find the electric potential at a point P
(0, 0, 4 m).
z
Solution : p
= 32 4 2 5m
5
and q = 10 øC = 10 C
Substituting the values, we have
V
9.0 10 10 1.8 10
9 5
4
volt Ans.
5.0
Ex a m p l e 3 1
Find out the points on the line joining two charges + q and 3q (kept at a distance of 1.0
m) where electric potential is zero.
Solution :
Let P be the point on the axis either to the left or to the right of charge + q at a distance r where
potential is zero. Hence,
ELECTROSTATICS
44 QUIZRR
p +q p 3q
+q 1.0m 3q
or
r 1.0 r
r
(a) (b)
q 3q
VP 0
For case (a) 4 O r 4 O 1 r
q 3q
Further, for case (b) VP 4 r 4 1 r 0
O O
which gives r = 0.25 m
Thus, the potential will be zero at point P on the axis which is either 0.5 m to the left or 0.25 m
to the right of charge + q.
Ex a m p l e 3 2
Four point charges + 8 øC, 1 øC, 1 øC and + 8 øC are fixed at the points
27
BP = DP = x2
2
Electric potential at point P will be
2KQ 2Kq
V
BP AP
1
where K 9 109 Nm2 / C2
4 0
ELECTROSTATICS
QUIZRR 45
8 10 6 106
V 2 9 109
27 2 3 2
2 x x
2
8 1
V 1.8 104
27 2 3 2 ... (i)
2 x x
2
Electric field at P is
1 27 3 / 2 3 / 2
dV 1 3
E 1.8 104 8 x2 1 x2 2x
dx 2 2 2 2
E = 0 on X-axis where
8 1
3/ 2
3/2
27 2 3 2
2 x 2 x
4 3 / 2
1
3/ 2 3/2
27 3 2
x2 x
2 2
27 2 3 2
2 x 4 2 x
5
This equation gives x m .... (i)
2
The least value of kinetic energy of the particle at infinity should be enough to take the particle
5
upto x m because
2
5
at x m, E = 0 Electrostatic force on charge q0 is zero or Fe = 0
2
5
for x m, E is repulsive (towards positive X-axis)
2
5
and for x m, E is attractive (towards negative X-axis)
2
5
Now, from Eq. (i), potential at x
2
ELECTROSTATICS
46 QUIZRR
8 1
V = 1.8 104 27 5 3 5
2 2 2 3
5
Applying energy conservation at x = and x =
2
1
mv02 q0 V ... (ii)
2
2 q0 V
v0
m
2 10 7 2.7 104
Substituting the values v0
6 10 4
v0 = 3 m/s Ans.
Minimum value of v0 is 3 m/s
potential at origin (x = 0) is
8 1
V0 1.8 104 27 3 2.4 10 V
4
2 2
Let T be the kinetic energy of the particle at origin.
Applying energy conservation at x = 0 and at x =
1
T + q0V0 = mv02
2
1
But mv02 q0 V
2
T = q0 (V V0)
7
T = (10 ) (2.7 104 2.4 104)
4
T = 3 10 J Ans.
5
Note : E = 0 or Fe on q0 is zero at x = 0 and x m. Of these x = 0 is stable equilibrium
2
5
position and x m is unstable equilibrium position.
2
ELECTROSTATICS
QUIZRR 47
Ex a m p l e 3 3
A non-conducting disc of radius a and uniform positive surface charge density is placed
on the ground with its axis vertical. A particle of mass m and positive charge q is dropped,
along the axis of the disc from a height H with zero initial velocity. The particle
has q/m = 40 g/.
(a) Find the value of H if the particle just reaches the disc.
(b) Sketch the potential energy of the partical as a function of its height and find its
equilibrium position. (JEE 1999)
Solution : As we have derived in the theory
p q,m
2
VP a H2 H
20 H
a
V0 (H = 0)
20
(a) Particle is released from P and it just reaches point O. Therefore, from conservation of
mechanical energy.
Decrease in gravitational potential energy = increase in electrostatic potential energy
(KE = 0 because Ki = Kf = 0)
mgH = q [V0 VP]
q
or gH = a a2 H 2 H ... (i)
m 20
q 40 g q
2g
m 20 m
Substituting in Eq. (i), we get
gH 2 g a H a2 H2
H
or a H a 2 H2
2
H H2
or a 2 H2 a or a 2 H2 a2 aH
2 4
3 2 4
or H aH or H a and H = 0
4 3
H = (4/3) a Ans.
ELECTROSTATICS
48 QUIZRR
(b) Potential energy of the particle at height H = Electrostatic potential energy + gravitational
potential energy
U = qV + mgH
Here V = Potential at height H
q 2
U a H2 H mgH ... (ii)
20
dU
At equilibrium position, F 0
dH
Differentiating Eq. (ii) w.r.t. H
q 1 1
q
mg 2H 1 0 2mg
or 20 2 a 2 H2
20
H
mg 2mg 1 0
a2 H2
2H
1 2 0
or
a 2 H2
2H
1
a H2
2
H2 1
2 2
or 2
a H 2 4 or 3H = a
a
or H Ans.
3
From Eq. (ii), we can see that,
U = 2mga at H = 0 and
a
H
U = Umin = 3 mga at 3 U
Therefore, U-H graph will be as shown.
a 2mga
Note that at H , U is minimum.
3
3mga
a
Therefore, H is stable equilibrium position.
3
O a
H
3
ELECTROSTATICS
QUIZRR 49
1 1 . EL ECT RI C DI POL E
A pair of equal and opposite point charges q, that are separated by a fixed distance is known
as electric dipole. Electric dipole occurs in nature in a variety of situations The hydrogen fluoride
molecule (HF) is typical. When a hydrogen atom combines with a fluorine atom, the single electron of
the former is strongly attracted to the latter and spends most of its time near the fluorine atom. As
a result, the molecule consists of a strongly negative fluorine ion some (small) distance away from a
strongly positive ion, though the molecule is electrically neutral overall.
Every electric dipole is characterized by its electric dipole moment which is a vector P directed
from the negative to the positive charge.
The magnitude of dipole moment is,
p
P = (2a) q –q – +q
+
2a
Here, 2a is the distance between the two charges.
1 1 .1 Ele c t r i c Di p o l e Po t e n t ia l
The potential of an electric dipole can be found by superposing the point charge potentials of the
two charges.
1 1 r r1
V Kq Kq 2
r1 r2 r1 r2
kpcos
V
r2
where p q d is defined as the dipole moment.
ELECTROSTATICS
50 QUIZRR
El e c t r ic Po t e n t i a l a t
d
(a) End on position (a point lying on the axis of the dipole)
p +q –q p
= 0, or
Kp p
V 2
r
1 1 .2 Ele c t ri c f i e l d i n t e n s i t y d u e t o a n e l e c t ri c d i p o l e
(Note that the electric field intensity must be lying in the same plane containing the point, the
axis of the dipole
Find the field along the Er and E
Er
V Kp cos Kp cos p
Er ^
r r r 2 r 3
E
V Kp cos Kp sin
E
r r 2 r3
–q +q
Kp
= 3
4 cos2 sin2
r
Kp 3cos2 1
E
r3
To find the direction
E tan
tan
Er 2
1
tan 1 tan where is the angle with E .
2 r
ELECTROSTATICS
QUIZRR 51
Kp Kp
Er = 0, E = 3 Enet E
r r3
E p
1 1 .3 T o r q u e e x p e r ie n c e d b y a n e l e c t ri c d i p o l e p l a c e d i s a n u n i f o r m e l e c t ri c f i e l d
PE –q
Re m a r k s a b o u t c o u p l e a n d c h a r a c t e ri s t i c s
(i) It remains same about whichever point you calculate the torque
(ii) Couple is said to be formed when equal and opposite forces are acting on the body separated
by same distance
(iii) Couple is a free vector
Ex a m p l e 3 4
An electric dipole having dipole moment P is placed parallel to electric field. Now it is
slightly rotated and released. Find the time period of oscillation.
Solution :
When it is kept parallel to electric fied, = 0 as sin = 0
Now, when is is rotated by an angle , it experiences a torque
= pE sin
E
d 2 +q
= I = I 2 d
dt
{sin as it is slightly displaced 0} –q
d 2
I 2 pE
dt
ELECTROSTATICS
52 QUIZRR
d 2 pE
2
dt I
pE
w
I
I
time period of oscillation = 2
pE
md 2 md 2
where I = 2
4 2
In this question if the two charges are connected at the ends of a rod, then take the moment of
inertia of rod also into consideration.
1 1 .4 Po t e n t i a l En e r g y o f a n e l e c t ri c d i p o l e i n u n i f o r m f i e l d
Work done against the field in rotating slowly the dipole from = 90 to .
Note : U = 0 (assumed) at an angle /2 from E
dU = |d|
U = + pE sin d
/ 2
E
= pE cos / 2 p
U = pE cos
U U/2 = pE cos
U/2 = 0
U = pE cos = p.E
Ex a m p l e 3 5
Find the word required is rotating a dipole from = 30 to = 60
Solution : Wex = U
= U60 U30
1 3
= pE (cos 60 cos 30) = pE 2
3 1
Wex = pE 2
ELECTROSTATICS
QUIZRR 53
Ex a m p l e 3 6
A point electric dipole is placed at a distance r from the centre of a uniformly charged ring
and has dipole moment equal to p . Find the force and torque experienced by the dipole
and the work required to rotate it by /3.
Solution :
–q +q
(E) (E+dE)
p
(point dipole)
r
F q E q E d E
F qd E
Now since the dipole is placed along r, we can change the above vector equation into scalar
equation.
F = q dE
dE
F q dr
dr
dE
F = P where p = q dr (length of dipole is very small)
dr
Important Note :
This above equation to be used directly to calculate the field when :
(i) dipole is directed along r
But F qd E can be used in general
ELECTROSTATICS
54 QUIZRR
KQr
Now, E {Field due to Charged ring}
R
3/2
2
r2
2
3/ 2 1/ 2
2
dE R r 3r R 2 r 2
KQ
dr
R r
3
2 2
2
3/2 1/ 2
2
R r 3r 2 R 2 r 2
KQ
=
R
3
2
r2
R 2 r 2 3r 2
1/ 2
2 2
KQ R r 3
=
R2 r2
KQ R 2 2r 2
=
R
3/2
2
r2
F
pKQ R 2 2r 2
R
3/2
2
r2
R
Net force will be towards ring if r {attractive}
2
(b) = P E 0 P E
1 pE
= pE 1
2 2
ELECTROSTATICS
QUIZRR 55
Ex a m p l e 3 7
A point electric dipole is placed at a distance r from a uniformly charged long thread (+ ).
Find the force and the torque experienced by dipole if
(a) (i) dipole is placed along r
(ii) dipole is placed perpendicular to r and parallel to the thread
(iii) dipole is placed perpendicular to the plane containing the thread and
r .
(b) Also find the work done required to rotate the dipole by /3, /2, in each case.
Solution : (a) (i) Just use
dE +
F P
dr
(ii case)
p
dE
E , r
2 0 r dr 2 0 r 2 (i case)
p
p
F
2 r 2
0
Here force will be attractive between the dipole and the thread.
(ii) F = 0
d E 0 [ field remain same at a same perpendicular distance]
ELECTROSTATICS
56 QUIZRR
d E Ed u→ (where u→ is a unit vector parallel to the length of dipole).
F q (d E)
= q Ed u→
d
E rd
= q . u→
r
pE →
F .u
r
pE
Magnitude of force = and directed along p
r
Now, torque in each case
p
(i) p E pE [when dipole is placed perpendicular to length of thread]
2 0 r
(ii) = 0
(iv) p E pE
Tip : Use d method only when the potential energy function is not available.
Ex a m p l e 3 8
An electric dipole of length 2a is placed at a distance 4a from an infinitely long thread with
l i n ear ch ar ge den si t y . Find the work required to rotate the dipole by , /2, /3.
Solution :
In the question, you can calculate the torque and then apply d to find the work. But that
would be lengthy, so use the potential energy method.
(i) Wex = U [work done in rotation by ]
= (Uf Ui )
Uf = qVA qVB
Ui = qVA + qVB
Wex = U = Uf Ui = 2q(VA V B) rA
a a
= 2q (VA V B) VA VB
4a
r
= 2q ln B rB
2 0 rA
q 5a q
n n 5 3
0 3a 0
ELECTROSTATICS
QUIZRR 57
q
= 2 ln (5 / 3)
0
(iii) In case of rotation by an general angle (Here is the made by dipole with positive
direction of r )
p
Wex = U a
–q a +q
= (Uf Ui)
Q
Now, 4a
4 a a cos
Uj = q V p V q
2 0 4 a a cos
q 4 cos
u ln
2 0 4 cos
q 4 12
Uf n
Now, 3 2 0 4 1 2
1 1 .5 N a t u re o f Eq u i p o t e n t ia l s u r f a c e s i s c a s e o f d i p o l e
–q +q
1 3 . L I N E OF FORCES
Line of forces are imaginary curves tangent to which at any point gives the direction of field
intensity at that point.
(The tangent direction must be
along the direction of field intensity)
ELECTROSTATICS
58 QUIZRR
1 3 .1 Pro p e r t i e s o f l in e o f f o r c e s :
(1) The number of field lines passing per unit normal area at a point gives the field intensity
at that point (in magnitude). Hence crowder the field lines, more strong in the field intensity.
(2) Line of force are always emitted by a positive charge and terminates at a negative or
extends to infinity.
+2q –q
(3) Two lines of force never intersect each other, (otherwise there will be two directions of field
intensity at a single point which is absurd).
(4) Electric field lines never form a closed loop (except the non-conservative induced electric field
due to time varying magnetic fields).
E. d l 0 (Property of a conservative field : work done in moving a positive charge
A B A B
This is not possible (electric field line) This line of force is possible in electric field
(5) The number of lines of force emitted (or terminating) at a charge is proportional to the
magnitude of the charge.
n q
q
n
0
(6) The line of forces have a tendency to contract longitudinally (just like a stretched elastic
string) to produce force of attraction between unlike charges and have a tendency to repel
laterally to produce the force of repulsion between like charges, and to produce EDGE
EFFECT.
ELECTROSTATICS
QUIZRR 59
Bending of the line of forces near the edge of the conductor is termed as the edge effect.
+
+
+ + +
+
+
(7) Line of forces are always terminating or emitting normally to the surface of a conductor.
(+q)
(8) Line of forces are absent when the field intensity is zero. ThatÊs why no line of forces exist
inside the conductor as (E = 0).
Fe w i m p o r t a n t r e m a r k s :
(i) If a point charge free to move, is placed on a line of force curve, then it will follow the curve
if the curve is a straight line, otherwise it will not.
Reason : No normal acceleration component to change its direction.
(ii) Equidistant field lines show electric field is constant in both magnitude and direction.
(Uniform)
(Non uniform hypothetical)
(iii) EA
EB
ELECTROSTATICS
60 QUIZRR
Ex a m p l e 3 9
Find the equation of the line of force in the region where
(i) E varies as E a xi→ yj→ , a 0
dy y
Solution :
dx x
dy dx
y
x
[direction of tangent line P(x,y)
E a x2 y2 magnitude of the
E
Note that how you decide the direction of the arrow, just use the logic, Ex = – ax, Ey = ay
1 3 .2 Fl u x o f A n Ele c t ri c Fi e l d (E ) :
Let us consider a plane surface of area S placed in an electric field E . Electric flux through an
elementary area d S is defined as the scalar product of ds and E i.e., dE = E . ds , where ds is the
area vector, whose magnitude is the area ds of the element and whose direction is along the outward
normal to the elementary area. Hence, the electric flux through the entire surface is given by
E E. ds ....(i)
or E E ds cos
ELECTROSTATICS
QUIZRR 61
To show the integration over a closed surface, a circle is shouwn on the integration symbol :
E E.ds .......(ii)
1 3 .4 I m p o r t a n t Po i n t s Re g a r d i n g Ele c t ri c Fl u x :
(i) It is a scalar physical quantity with units (volt m) or N/m2/c
(ii) It will be maximum when cos = max = 1 i.e.
= 0 , i.e., electric field is normal to the surface with (dE)max = EdS
E
ds
1 4 . GA U SS’S L A W :
This law gives the relation between the electric flux through any closed hypothetical surface (called
E) through
a Gaussian surface) and the charge enclosed by the surface. It states „the electric flux (
1
any closed surface is equal to times the ÂnetÊ charged enclosed‰.
0
That is,
q
E E. ds 0
ELECTROSTATICS
62 QUIZRR
where q denotes the algebraic sum of all the charges enclosed by the surface. If there are several
charges + q1, + q2, + q3, q4, q5 inside the Gaussian surface then
q = q1 + q2 + q3 q4 q5
Remarks :
(1) Gaussian surface is an imaginary closed surface.
(2) Gaussian surface cannot be passed through a discrete charges but can be passed through
continuous charge distributions.
Note : The electric field is directed in radial direction for spherically symmetry charge distribution
hence the use of GaussÊs law simplifies the position.
1 4 .1 Ele c t ri c f i e l d i n t e n s i t y o b t a i n e d u s i n g t h e Ga u s s ’s l a w
I. Uniformly charged non conducting sphere
(a) For an internal point
Consider a gaussion surface at a distance from the centre inside the surface
Note that the direction of the electric field will be directed radially outward
E. d s E ds E 4r 2 E
4
r 3
E 4 r 2
3
0
r
E when r > R
3 0
Q
E when r > R
40 r 2
ELECTROSTATICS
QUIZRR 63
Illust rat io n
Find the field intensity inside a thick conducting shell of inner and outer radii, R and 2 R,
0 r
whose volume charge density varies as,
R
Solution : Using gaussÊs law by making a gaussian surface at a distance r from the centre
2 0 4 r 3 dr
qinside = (4 r ) dr
R
2R
Hence according to gaussÊs Law
R
r
r
0 r 4 r 2 dr
E 4 r 2
R
R
0
E
0 r 4 R4
Rr 2
40
E
E +
2 0 r + d
C
Let us consider an infinite non-conducting charged plane sheet having a surface charge density
m2
ELECTROSTATICS
64 QUIZRR
By symmetry, the electric field intensity at all points on either side near the sheet will be perpendicular
to the sheet, directed outward (if the sheet is positively charged). Thus E is perpendicular to the plane
ends of the cylinder and parallel to the curved surface.
Therefore, flux through the two plane ends is
++
+
+ + + +
+
E
+ +
+ + + +
+ + + +
+ + +
E E. r 2 E. r 2 2 E.r 2
qinside r
2
0
0
2 E. r 2
E
20
r
l
R
Since charge is uniformly distributed through out the volume of cylinder of radius R. For finding
the electric field at a distance (r < R) from the centre, we choose a cylindrical Gaussian surface of radius
r, co-axial with the charge distribution.
The magnitude E of the electric field has the same value at every point on the Gaussian surface
ELECTROSTATICS
QUIZRR 65
r 2 l
E.ds E 2rl 0
r
E r
2 0
in vector E
2 0
Thus,
E. 2rl
R 2 l
0
Y
R 2
E R
2 0 r
2 0
in vector form
R 2
E r X
2 r2 (r = R) r
0
Illust rat io n
Find the field intensity inside long cylinder having inner and outer radii, R and 2R and
r
having charge density given by 0 1 for
R
qinside = 0 0 E = 0
ELECTROSTATICS
66 QUIZRR
Consider a cylindrical gaussian surface
at a distance r
q
inside
0
r
2rl dr
E. d s 0
R
r
r 1 2rl dr r
E 2rl 0
R
R 2R
R 0
r
r2 r3
Er 0
0 2 3R
R
5r 2 5R 2
E 0
r 0 6 6
r2 R2
= E 0
6 0 r
(iii) For r > 2R :
Again consider a cylindrical gaussian surfare.
From gaussÊs law
2R r 2 r 3
2R r
r
qinside 0
1
R
2 rl dr 0 2l
R 2 3R R
2R
0 l(23R 2 )
E. 2r l
3 0
230 R 2
E =
6 0 r
ELECTROSTATICS
QUIZRR 67
1 4 .2 Ca v i t y i n s i d e a c y l i n d r ic a l i n f i n i t e l y l o n g v o l u m e :
Ex a m p l e 4 0
Find the field intensity inside a long cylindrical cavity which is uniformly charged and
distance between the axis of cylinder and that of cavity is a .
Solution :
P
r
r
O a O´
Where E1 is the field intensity due to uniformly Charged cylinder of density + and E 2 is the
field due to uniformly charge cylinder of density centred at 0´
r ´
r r
r´
=
2 0 2 0 2 0
Now, r r ´ 00´ or r r ´ a
a
00'
Enet = 2
0 2 0
Thus, just like is the case of spherical cavity inside a sphere, a cavity in a cylinder also has
constant electric field inside it.
Ex a m p l e 4 1
A system consists of a uniformly charged sphere of Radius R and the surrounding has a
volume charge density . Then find the charge on the sphere so that the field intensity
r
at any point outside the sphere becomes independent of r and also find its magnitude.
ELECTROSTATICS
68 QUIZRR
Solution :
Consider a guassian spherical surface centred at O at a distance r.
Let Charge on sphere be q and the charge surrounding it inside the gaussian surface is
r 4r dr
2
Q
R
q
inside
0
q
r
2
4 r dr
r
= ( /r)
E. ds 0 0 R q
R
O
4 r 2 r
q r R2
2
E ds
0
q 2r r 2 R 2
E 4 r 2
0
E
q 2r r 2 R 2
2
4 r 0
1 q R 2
=
2 0 r 2 4 0 2 0
For E to be independent of x,
q R 2
0
4 0 2 0
q = 2R2
and then, the constant electric field magnitude will be
E
2 0
ELECTROSTATICS
QUIZRR 69
Ex a m p l e 4 2
A sphere is charged with volume charge density = 0 (1 r /R). Then find the field intensity
at an internal and external point. Also find the maximum field intensity due to the given
charge distribution.
Solution :
(a) At an internal point
Using the GaussÊs Law
r
0 1 4 r 2 dr
r
R
E.d s 0
0
E 4r 2
0
0
4
r3
3
r4
R(4)
0
0 4 r 3 R 3r 4
E
r 2 0 12R
=
0
12R 0
4 rR 3r 2
(b) At an external point
Again by gaussÊs Law
R 0
r
2
1 4 r dr
R
E.d s 0
0
R3
E 4r 2 0 4
0 12
R3
E 0
12 r 2
0
Maximum field exists at a point inside the given sphere; for an internal point we have,
0 dE
E 4 R 3r For Emax, 0
12 R 0 dr
dE 2R
dr 4R 6r 0 r
3
ELECTROSTATICS
70 QUIZRR
Ex a m p l e 4 3
px i→ q y 2 j→ zk→
The field intensity in a region is given by E . Then find the net charge
px 3 qy3 z 2
xi→ yj→ zk→
ds ds
R
ds
R
E.ds
px i→ qy
2 2
. xi→ yj→ zk→ ds
j→ zk→
= px qy
3 3
z y
2 R
3 3
ds px qy z
=
2
R px3 qy2 z2
4 R 2
= R 4 R
q
4 R inside
0
qnet 4 0 R
ELECTROSTATICS
QUIZRR 71
Ex a m p l e 4 4
The field intensity in a region is given by E 2 xy z 2 i→ 2 yz x 2 j→ 2 x y k→ .
2
Now, E.ds
xi→ yj→ zk→
=
2 xy z2 i→ 2 y2 x2 →j 2 x y2 k→ .
R
ds
=
ds
R
2 x2 y z2 x 2 y2 z x2 y 2 xz2 y2 z
=
ds
R
3 x2 y y2 z z2 x
Now, eliminating x, y, z from the equation is not easy, hence we use another method (shortkut) to
solve it.
Note : Net charge enclosed = 0, if the following 2 conditions are satisfied :
(i) always when the electric field is symmetrical i.e. if you replace
x x, y y, z z and no change in the function, then field intensity is a symmetrical
function.
(ii) Again the sphere must be centred at origin, for this to be applicable.
Reason :
ds
E The dot product of E. ds will cancel out for each oppositely
E
ELECTROSTATICS
72 QUIZRR
1 4 .3 Ele c t ri c f l u x p a s s in g t h ro u g h d i f f e r e n t s u r f a c e s
Ex a m p l e 4 5
(1) Calculate the flux through each face of a cube when a point charge of + q is placed
at the centre of the cube.
Solution : B C
The flux through each face remains same, since the charge q is D
A
symmetrically located with respect to all the faces of the cube. +q
q G
Hence, We have flux through the cube = F
0
q
flux through each face = H E
60
(2) Calculate the flux through each face of a cube when a point charge q is placed at one
of the vertex of the cube.
B C
Solution :
A D
Electric flux through the face ABCD, ABGH and ADEH are zero. +q
The reason the this statement lies in the fact that these faces contain the
G
Charge q, Hence electric field strength in these faces lie perpendicular to the F
area vector of these faces. Thus E. ds 0
H E
Remember : The electric flux through a surface containing the charge is zero.
Now, the flux through each of the remaining faces will be same since, they are symmetrically located.
Co n c e p t :
Complete the symmetry by placing seven more cube around given cube so that the change q lies
at the centre of the eight cubed volume.
q
Flux through each cube = 8
0
q
Flux through each of the 3 faces of the 8 cubes = 24
0
(3) Calculate the flux in the given case through the curved hemispherical portion only,
when placed is uniform electric field as shown :
(a) E (b) E
ELECTROSTATICS
QUIZRR 73
Solution :
(a) Complete the hemisphere by enclosing it from the bottom
curved 0
[ base = 0, as E is perpendicular to ds ]
Here, circular base = E. ds E (4 r 2 ),
ds
curved E(4 r 2 )
Ex a m p l e 4 6
A point charge is placed at the centre of a cylinder of radius R and length l. Find the flux
through the curved surface.
Solution :
(1) First, close the cylinderÊs circular ends and apply gaussÊs law
0 R
L/2
L/2
q cos 0
net L2
0 R2
4
r
(2) The flux through each of the flat base net circular ends = 2 E. ds
0
E
ds
r
L/2
ELECTROSTATICS
74 QUIZRR
L
q 2r dr .
r r 2 tan
net (Circular ends) = 0 2 cos
4 0 L 2
dr L sec 2 d
2 2
cos2
0
q
= 0 cos2 tan .sec 2 d .cos
0
0
q q
= 0 sin d 1 cos 0
0
0
q q q
(curved) 1 cos 0 cos 0
0 0 0
q L/2 q L
0 L2 0 L 4R2
2
R2
4
1 4 .4 Di f f e r e n t ia l f o r m o f Ga u s s ’s L a w
Derivation :
Consider the flux through an infinitesimal volume is space. Net flux through the cube
= [ EX (dy dz) + (EX + dEX) dydz]
Y Ey + dy
+ [ Ey ( dzdx) + (Ey + dEy) (dzdx] (Ez + dEz )
+ [ Ez (dx dy) + (Ez + dEz) (dxdy)] Ex
Z
(dEx dy dz) + (dEy dx dz) + (dEz dxdy) =
d Ex dEy dEz
dx dy dz 0
E E E
(Differential form of Gauss law)
x y z 0
ELECTROSTATICS
QUIZRR 75
In terms of potential :
2 V 2 V 2 V
2 2
x y 2
z 0
In other forms
i→ →j k→
x y z
.E (Divergence of E )
0 0
Ex a m p l e 4 7
Electric potential in origin is given by
V = ( x 3 + )
Then find the volume charge density in the region as function of space.
Solution :
2V
2 [Using differential form of Gauss law]
x 0
2
3x
x 0
6 x
0
60 x
ELECTROSTATICS
76 QUIZRR
1 5 .1 CON DU CT I ON
Flow of charge from one conductor to the other conductor is called conduction. (Charge on the
conductor does not remain constant.]
1 5 .2 I N DU CT I ON
Redistribution of charge over the surface of the conductor is called induction. In this case, the total
charge of conductor remains constant.
1 5 .3 PROPERT I ES OF A CON DU CT OR
(1) When an external charge is given to the conductor, the entire charge appears over the
surface of the conductor and the entire volume of the conductor is electrically neutral.
In the absence of an external field and charge around the conductor,
++
the surface charge density of the charge distribution over the conductor ++ +++ ++ ++
+ +
surface is inversely proportional to the radius of curvature. + +
+ +
++ +
1 + +
+ (E = O) +
(flater portion has lesser charge density than curved part.) + +
R + +
+ +
++ +
Note : This is applicable only when external field and charges + + ++ +
+
++ + ++
are absent
(2) The potential at any point inside (or on the surface) of the conductor is constant since the
conductor is an equipotential body.
(3) Electric field is zero inside the conductor
Hence, the volume of the conductor is electrically neutral
(4) The field intensity near the surface of the conductor is given by : E
0
De r iv a t io n :
Consider a Gaussin surface near the surface of the conductor as shown
dA
ds + + + +++ E
E ds ++ +
[ : surface density of the point] +
C0 +
+
+ +
+ (E = O) +
+ +
+
E ( E is perpendicular to the surface) + +
0 +
+ +
B
+ +
+ +
+ +
+
+
+ +
+ +
ELECTROSTATICS
QUIZRR 77
1 5 .4 Ele c t ri c f i e l d a n d f i e l d l in e s a r e n o r m a l t o t h e s u r f a c e o f a c o n d u c t o r
Net field inside a conductor is zero. It implies that no
field lines enter a conductor. On the surface of a conductor, + + ++
+ ++
++ 90
electric field and hence field lines are normal to the surface E=0 + +
+ +
of the conductor. + +
+
+ + + ++
If a conducting box is immersed in a uniform electric
field, the field lines near the box are somewhat distorted. Similarly if a conductor is positively charged,
the field lines originate from the surface and are normal at every point and if it is negatively charged
the field lines terminate on the surface normally at every point.
1 5 .5 Ca v i t y i n s i d e a c o n d u c t o r
Consider a charge + q suspended in a cavity in a conductor. Consider a Gaussian surface just
outside the cavity and inside the conductor E 0 on this Gaussian surface as it is inside the conductor.
q
Hence from GaussÊs law, E.d S in0 gives qin = 0 as { E = 0 inside conductor}
+ + + + + +
+ Gaussian
+ + +
+ surface +
+ +
+ q + + +
q+ +
+ +
+ +
+q +
+ +
+ +
+ + + + +
+
(a) (b)
This concludes that a charge of q must reside on the metal surface of the cavity so that sum
of this induced charge q and the original charge + q within the Guassian surface is zero. In other
words, a charge q suspended inside a cavity in a conductor induces an equal and opposite charge
q on the surface of the cavity. Further as the conductor is electrically neutral a charge + q is induced
on the outer surface of the conductor. As field inside the conductor is zero the field lines coming from
q cannot penetrate into the conductor. The field lines will be as shown in figure (b).
The same line of approach can be used to show that the field inside the cavity of a conductor is
zero when no charge is suspended in it.
El e c t r o s t a t ic s h i e l d i n g
Suppose we have a very sensitive electronic instrument that we want to protect from external
electric fields that might cause wrong measurements. We surround the instrument with a conducting
box or we keep the instrument inside the cavity of a conductor. By doing this charge in the conductor
is so distributed that the net electric field inside the cavity becomes zero and the instrument is protected
from the external fields. This is called electrostatic shielding.
ELECTROSTATICS
78 QUIZRR
1 5 .6 T h e p o t e n t ia l o f a c h a r g e c o n d u c t o r t h ro u g h o u t i t s v o l u m e i s s a m e
In any region in which E 0 at all points, such as the region vary far from all charges or the
interior of a charged conductor, the line integral of E is zero along any path. It means that the
potential difference between any two points in the conductor are at the same potential or the interior
of a charged conductor is an equipotential region.
Let us consider a small elemental charge dq of area ds. Now let us calculate the net electric field
E due to rest of the charges on this elemental area.
E
ds
E=0
qenclosed
E. ds E ds
0
and qencl. dq ds
E
2 0
Now, this electric fied is on the elemental charge dq, hence force experienced by it is,
F dq E ds
2 0
ELECTROSTATICS
QUIZRR 79
2
F ds
2 0
2
Electrostatic pressure = P
2 0
Note : This eletrostatic pressure is bursting in nature i.e. directed outwards always.
Ex a m p l e 4 8
Find the maximum surface charge density that can be given to a conducting spherical shell
of radius R and thickness R (R << R). 0 is the bursting stress.
Solution :
By increasing the surface charge density, the electrostatic pressure also increses. Let us assume
the maximum surface charge density be , then
2
Pressure = 2C
0
Now bursting stress pressure force {so that the shell do not burst}
When the shell bursts, the area available will be the area covered by shell in the middle i.e. 2R
and for electrostatic pressure area will be R2
Hence, 0 (2RR)
2
2 0
R 2
40 0 R
R
Ex a m p l e 4 9
Find the maximum surface charge density that can be given to a thin conducting cylinder
of thickness R, and radius R, length l. 0 is the bursting stress.
Solution :
The question is similar to the previous example. However it is important for the students to know
about the projected area concept used with the electrostatic pressure.
ELECTROSTATICS
80 QUIZRR
Here it is a thin conducting cylinder.
Now
2R
Projected
area
Pressure force = (electrostatic pressure) (projected area)
2
= 2 2R l
0
2
0 (2R l) 2R l
2 0
20 R l0
R
(A ) El e c t ri c Fi e l d
At all points inside the charged spherical conductor or hollow spherical shell, electric field E O ,
as there is no charge inside such a sphere. In an isolated charged spherical conductor any excess charge
on it is distributed uniformly over its outer surface same as that of charged spherical shell or hollow
sphere. The field at external points has the same symmetry as that of a point charge. We can construct
a Gaussian surface (a sphere) of radius r > R. At all points of this sphere the magnitude of electric field
is the same and its direction is perpendicular to the surface. Thus, we can apply
qin
ES
0
q
+ + ++
q
+
E 4 r 2 +
+ +++
or
0 r
+ ++
E
+ R
q + +++
++
1
E . 2 Gaussian
4 0 r
surface
ELECTROSTATICS
QUIZRR 81
Hence, the electric field at any external point is the same as if the total charge is concentrated
at centre.
At the surface of sphere r = R,
1 q
E .
4 0 r 2
1 q
Esurface 4 . 2
0 R
1 q
Eoutside . 2
4 0 r
The variation of electric field (E) with the distance from the centre (r) is shown in Fig.
Note : (i) At the surface graph is discontinuous
E
1 q q / 4 R 2
(ii) Esurface . 2 =
4 0 R 0 0
0 1
E
(B) POTENTIAL : As we have seen, r2
1 q
Eoutside .
4 0 r 2 O R r
dVoutside 1 q dV
. 2 E dr
dr 4 0 r
V q r dr
0 d Voutside
4 0 r2
(V = 0)
1 q 1
V . or V
4 0 r r
Thus, at external points the potential at any point is the same when the whole charge is assumed
to be concentrated at the centre. At the surface of the sphere, r = R
1 q
V .
4 0 R
ELECTROSTATICS
82 QUIZRR
At some internal point electric field is zero everywhere, therefore, the potential is same at all points
which is equal to the potential at surface. Thus, we can write,
1 q
Vinside = Vsurface = .
4 0 R
1 q
and Voutside = .
4 0 r
The potential (V) varies with the distance from the centre (r) as shown in Fig. below.
V
1 q R
=
4 2 0 1
0 R V
r
O R r
(A ) El e c t ri c Fi e l d
Positive charge q is uniformly distributed throughout the volume of a solid sphere of radius R. For
finding the electric field at a distance r (< R) from the centre let us choose as our Gaussian surface a
sphere of radius r, concentric with the charge distribution. From symmetry the magnitude E of electric
field has the same value at every point on the Gaussian surface, and the direction of
E is radial at
every point on the surface. So, applying GaussÊs law,
qin
ES ... (i)
0
4 3 + + + Gaussian
H er e, S = 4r2 and qin = r + + + + + + surface
3
q + + + ++ + +
+ r+ +
4 + + +++ ++
here, = charge per unit volume = R 3 + + ++
3 + + ++ + + + +
r ++
Substituting these values in Eq. (i) + + + + R+
+
+ + +
1 q
We have, E . .r
4 0 R 3
or E r
ELECTROSTATICS
QUIZRR 83
At the centre r = 0, so E = 0
1 q
At surface r = R, So E . 2
4 0 R
To find the electric field outside the charged sphere, we use a spherical Gaussian surface of radius
r (> R). This surface encloses the entire charged sphere, so qin = q, and GaussÊs law gives
q 1 q
E (4r2) = or E .
0 4 0 r 2
1
or E
r2
Notice that if we set r = R in either of the two expressions for E (outside and inside the sphere),
we get the same result,
1 1
E . 2
4 0 R
this is because E is continuous function of r in this case. By contrast, for the charged conducting
spher e t he magnit ude of elect r ic field is discont inuous at r = R (it jumps from E = 0 to E = /0).
Thus, for a uniformly charged solid sphere we have the following formulae for magnitude of
electric field.
1 q
Einside = 4 . 3 . r
0 R
1 q
Esurface = 4 . 2
0 R
1 q
Eoutside = 4 . 2
0 r
The variation of electric field (E) with the distance fromthe centre of the sphere (r) is shown in
Fig. below.
E
1 q
4 2
0R 1
E
r2
O R r
ELECTROSTATICS
84 QUIZRR
(B) POTENTIAL : The field intensity outside the sphere is,
1 q
Eoutside = 4 . 2
0 r
dVoutside
Eoutside
dr
dVoutside = Eoutside dr
V r 1 q
or dVoutside 4 0 . r 2 dr
1 q
V . as V = 0
4 0 r
1
or V
r
1 q
At r = R, V .
4 0 R
1 q
i.e., at the surface of the sphere potential is VS = .
4 0 R
1 q
Einside = . 3 .r
4 0 R
dVinside
Einside
dr
dVinside = Einside dr
V 1 q r
Vs dVinside 4 0 . R3 R r dr
r
1 q r2
V VS . 3
4 0 R 2
R
1 q 1 q 3 1 r2
Substituting VS 4 . R , we get V 4 R 2 2 2
0 0 R
3 1 q 3
At the centre r = 0 and Vc . VS i.e., potential at the centre is 1.5 times the
2 4 0 R 2
potential at surface.
ELECTROSTATICS
QUIZRR 85
Thus, for a uniformly charged solid sphere we have the following formulae for potential.
1 q
Voutside = 4 . r
0
1 q
Vsurface = 4 . R
0
1 q 3 1 r2
and Vinside = .
4 0 R 2 2 R 2
The variation of potential (V) with distance from the centre (r) is as shown in figure.
3 1 q
2 4 0R
1 q
4 0R
O R r
Po i n t s t o Re m e m b e r
1. To find the electric potential due to a conducting sphere (or shell) we should keep in mind the
following two points :
(i) Electric potential on the surface and at any point inside the sphere is
1 q
V = 4 . R
0
(R = radius of sphere)
(ii) Electric potential at any point outside the sphere is
1 q
V = 4 . r (r = distance of the point from the centre)
0
For example, in the figure shown, potential at A is
1 qA q q C qC
VA = B C
4 0 rA rB rC qB
B
Similarly, potential at B is qA
1 qA q q A
VB = 4 B C
r
0 B rB rC
and potential C is,
1 qA q q
VC = B C
4 0 rC rC rC
ELECTROSTATICS
86 QUIZRR
2. Principle of a generator : A generator is an instrument for B
producing high voltages in the million volt region. Its design is qC
based on the principle that if a charged conductor (say A) is brought A
into contact with a hollow conductor (say B), all of its charge
rA qA qB
transfers to the hollow conductor no matter how high the potential
of the latter may be. This can be shown as under.
In the figure, rB
1 qA qB
VA =
4 0 rA rB
1 qA qB
and VB = 4
0 rB rB
qA 1 1
VA VB = 4
0 rA rB
ELECTROSTATICS
QUIZRR 87
and outer surfaces of A, B and C. To solve such type of problems following points should keep in
mind :
C
B
A
a q1 q2
b
c
(i) The whole charge q1 will come on the outer surface of A unless some charge is kept inside
A.
To understand it let us consider a Guassian surface (a sphere) through the material of A.
As the electric field in a conducting material is zero. The flux through this Gaussian surface
is zero. Using GaussÊs law, the total charge enclosed must be zero.
q1
Gaussian
surface
(ii) Similarly if we draw a Gaussian surface through the material of B we can see that
q3 + q1 = 0 or q3 = q1
and if we draw a Gaussian surface through the material of C, then
q5 + q4 + q3 + q1 = 0 or q5 = q4
(iii) q5 + q6 = q2. As q2 charge was given to shell C.
(iv) Potential of B should be zero, as it is earthed. Thus,
1 q1 q3 q4 q5 q6
or 0
4 0 b b c
So, using the above conditions we can find charges on different surfaces.
We can summarise the above points as under
(1) Net charge inside a closed Gaussian surface drawn in any shell is zero. (provided
the shell is conducting).
(2) Potential of the conductor which is earthed is zero.
ELECTROSTATICS
88 QUIZRR
(3) If two conductors are connected, they are at same potential.
(4) Charge remains constant in all conductors except those which are earthed.
(5) Charge on the inner surface of the innermost shell is zero provided no charge is
kept inside it. In all other shells charge resides on both the surfaces.
(6) Equal and opposite charges appear on opposite faces.
Ex a m p l e 5 0
A charge q is distributed uniformly on the surface of a sphere of
radius R. It is covered by a concentric hollow conducting sphere of
R q
radius 2R. Find the charges on inner and outer surfaces of hollow
sphere if it is earthed.
2R
Solution :
The charge on the inner surface should be q, because if we draw a closed Gaussian surface
through the material of the hollow sphere the total charge enclosed by this Gaussian surface should
be zero. Let q´ be the charge on the outer surface of the hollow sphere.
q'
Since, the outer sphere is earthed, its potential should be zero. The potential –q
on it is due to the charges q, q and q´, Hence, q
1 q q q'
V 0
4 0 2R 2R 2R
q´ = 0 Ans.
Therefore, there will be no charge on the outer surface of the hollow sphere.
q'
Ex a m p l e 5 1
q
Solve the above problem if thickness of the hollow sphere is –q
considerable. R
r
R2 R3 p
Solution :
In this case, we can set V = 0 at any point on the hollow sphere. Let us select a point P a distance r
from the centre. Where R2 < r < R3. So
VP = 0
1 q q q'
0
4 0 r r R 3
q´ = 0 Ans.
i.e., in this case also there will be no charge on the outer surface of the hollow sphere.
ELECTROSTATICS
QUIZRR 89
+ + + +
1 8. SH I ELDI N G EFFECT +Q + +
2 Q3
+
+
(1) If an external charge(s)/field are present in the + +
+
surrounding of an electrical conductor then charge + +
distribution on the surface of the conductor occurs + +
E=O
in such a way that the net field intensity, due to + + E0
+ +
the surrounding charge/field and the charge
+ +
distributions (due to the surrounding charge) +
+
developed on the conductors surface; at any point + +
inside the conductor becomes zero. + +
+ +
(2) If the external charge in the surrounding is Q1
+ +
arbitarily displaced/or the field charged; then re- + + +
distributing of charge occurs on the surface of the conductor at once, so that net field
intensity at any point inside the conductor still remains zero. +
(3) If a cavity is created inside the electrical conductor, then electric
+ + +
+ +
field inside the cavity still remains zero (in the absence of external + +
+ +
charge in the cavity). + +
+ +
+ A +
B +
+ +
+ +
D C +
+ +
+
+ + ++
Reason : To satisfy E. dl 0 , E must zero inside the cavity.
+ + + +
+ +
(4) Charge placed in the cavity in a conductor : +
+ +
+ +
When an external charge is placed inside the cavity of a conductor, + +
+ +
the equal and opposite charge distributian is induced on the surface +
+
of the cavity, so that the field intensity due to both the external q +
+
charge in the cavity and the induced charge at any point outside + +
+ +
the cavity is zero. + +
+
++ +
+ + +
E inside conductor = E inner Eouter 0 0 0 + +
(5) If the external charge in the cavity is arbitarily displaced, then, re-distribution of induced
charge on the surface of cavity takes place so that field intensity outside the cavity still
remains zero.
(6) Outer charge distribution remain unchanged if the external charge in the cavity is arbitarily
displaced and the inner charge distribution remains unchanged if the outer charge distribution
charges (i.e. vice-versa).
ELECTROSTATICS
90 QUIZRR
(7) Electric field at any point (inside the cavity) near its surface = . As a general idea, field
0
is more strong at a point where, charge is closer to surface of the cavity.
(8) The electric potential, due to the external charge and the induced charge on the surface of
the cavity, at any point OUTSIDE THE CAVITY IS ZERO.
(Reason : No work done against field as E = due to both)
Remember : ELECTRIC POTENTIAL IS NOT ZERO due to the outer charge distribution
inside the cavity.
(9) The net force experienced by any external charge inside the cavity is zero.
Proof :
qinside .
E. ds 0
+q
F 0
(10) Hence, a conducting shell divide the entire regions into two parts, inner and outer which
are completely independent of each other, in terms of electrostatic field. This is called as the
shielding effect in the electrical conductors.
Ex a m p l e 5 2
A charge q is placed at a distance r from the centre of an uncharged conductor sphere of
radius R. Find the potential of the conducting sphere.
+ +
Solution : +
R +
Conductor has an equipotential space inside it. +q +
find the potential at the centre O for convenience r +
+
+
Kq Kdq Kq K + +
V dq
r R r R
ELECTROSTATICS
QUIZRR 91
Ex a m p l e 5 3
In the previous question, the conductor is grounded and the charge q starts moving with
constant velocity towards the sphere which is v. Then, find the current through the wire
connecting the sphere to the earth.
Q
Solution :
R i
V
q r
Let us assume that a charge Q is acquired by the conductor, so that its potential becomes zero.
Kq KQ
V 0 [ it is earthed]
r R
q R dQ qR
Q , 2 v
r dt r
qRv
i 2
r
Now, since the charge on the conductor is decreasing, therefore the current flows from the conductor
to the earth.
[However, the magnitude of the current is increasing]
Ex a m p l e 5 4
A point charge q is kept inside a charged conducting shell at a distance charged ÂaÊ from
its centre. The shell has an initial charge Q and external radii ÂcÊ. Another charge Âq1Ê is
placed outside the shell at a distance ÂbÊ from the centre.
Find : (i) the net potential at O
(ii) the force experienced by q1 due to the induced charge in the cavity of shell.
(iii) Potential at the position of q1 due to the induced charge on the surface of
cavity. (Assume shell is thin).
(Q
Solution : +q
Q )
–q
O b a +q b q1
a +q q1
ELECTROSTATICS
92 QUIZRR
(i) Potential at 0 :
Kq KQ Kq
V0
a c b
(Remember, when the shell is thin, write the potential together for the inner and the outer
surface)
(ii) To calculate the force expression, use the concept that the force exerted by both, the induced
charge as well as the external charge is zero. This can be easily seen by creating a gaussian
surface centred at O and passing through the charge q1,
So qenclosed = 0, hence E net 0
F = 0 and F net F cavity + F q 0
K q q1 K q q1
Fcavity Fq , F q F cavity
a b 2
a b2
Kq
Vcavity = Vext =
a b
1 9 . PA RA L L EL PL A T ES
Two important concepts to be applied to all questions based on the parallel plates :
(1) The charges appearing on the faces opposite to each combination of pair of parallel plates
are always equal is magnitudes and opposite in nature.
(2) The outer faces of the extreme plates have equal charges is magnitude and nature and
equal to the (sum of charges on all plates divided by two) charge on the outer plates faces
(outer) =
Qall
2
ELECTROSTATICS
QUIZRR 93
De r i v a t i o n
(1) Use GaussÊs law
A B
Draw a Guasssian surface ABCD
q
Now E. d s inside
C0
E. d s E. d s E. d s E. ds E. ds
ABCD AB BC CD OA
0 0 0 0 0
E ds
E 0 E ds E 0 D C
Q1 q Q2 q
Q1 Q2
2 0 A 2 0 A
(Q1 –q) (Q 2 + q)
q
Q1 Q2
2
Q1 Q2 Q Q2 q –q
qouter on each plate = Q1 1
2 2
Ex a m p l e 5 5
Show the charge distribution, on each surface when a charge of Q, 5Q, 10 Q are given to
each of the parallel plates.
Q –5Q 10Q
A B C
Solution :
Charges on the outermost surfaces
Q 5Q 10Q
= 3Q
2
ELECTROSTATICS
94 QUIZRR
So, charge on the inner surface of plate A
= Q 3Q = 2 Q
Plate B
Charge on surface opposite to plate A
= + 2Q
Now, charge on other surface
= 5Q (+ 2Q) = 7Q
Plate C
Charge on surface opposite to plate C = + 7Q
Q 5Q 10Q
+7Q 3Q
3Q 2Q
+2Q 7Q
Ex a m p l e 5 6
In the question, find the charge flown through the switch if : Q 5Q 10Q
(i) S1 is grounded; S2, S3 open
(ii) S3 is closed, S1, S2 open. 2d d
(iii) S1, S2 are simultaneously closed; S3 open
(iv) S2, S3 simultaneously closed, S1 open
(v) S1, S3 simultaneously closed, but S2 remains open S1 S2 S3
(vi) S2 closed and outer plates are connected by wire
(vii) If all switches are open and outer plates connected by wire, then find the charge
distribution.
Solution :
(i) Concept : When any one of the outer plates is grounded then the charge on the outer
faces of extreme plates becomes zero.
Reason : Its potential is now zero, hence, no work has to be done in bringing any
charge from to the plate.
ELECTROSTATICS
QUIZRR 95
S1 S2 S3 (i)
Q 5Q
(ii) Charge flown to plate from the earth
= Qfinal Qinitial
= 4Q 10Q = ( 6Q) O Q 4Q O
Q
4Q
(ii)
S1 S2 S3
Q
(iv) S2, S3 simultaneously closed; S1 remains open
Charge flown through S2
Q Q
= Q ( 5Q) = 4Q O O O O
Charge flown through S3
= O (+ 10Q) = 10Q
S1 S2 S3
S1 S2 S3
ELECTROSTATICS
96 QUIZRR
Method 1 :
{This is to be read after going through capacitors)
The charge on surface of the plate 2 facing 1, and 3 are distributed in the ratio of their capacitance.
This is be cause potential drop acroos plates 1 and 2 and the drop across 2 and 3 is same, Hence,
we can say that the capacitor formed by 1 and 2, and by 2 ad 3 are in paralle.
Where,
A A
C1 0 , C2 0 5Q
2d d q2 10Q
+5Q
3 q1 3
C1 1
C2 2
d
1 5Q 2 10Q 2d
q1 (5Q) , q2 3 5Q 3
3 3
5Q 2Q
Charge flow through S1 = Q
3 3
10Q 7Q
Charge flow through S3 = Q
3 3
Method 2 :
Potential drop across (plate 1 and plate 2) and (plate 2 and plate 3) must 5Q
be equal and opp. so that plate 1 and plate 3 remain at same potential. q q
( 5Q-q)
E1 d1 E2 d2 0
q 5Q q d
2d 0
0 A 0 A
2q + 5Q + q = 0 q 5Q / 3
(vi) Sum of the charges on the two outer plates remain constant = 11Q 11Q 22Q
But this charge will be distributed in the ratio of the capacitance 3 3
11Q 22Q
O
1 O 3
3
Charge on plate 1 = 11Q
3
2d d
1 22Q
Charge on plate 2 = 11Q = S1 S2 S3
3 3
Note : Here charges do not appear on the outer surface of the plate 1 and 3, since the entire
charge is accomodated on their inner surfaces as the plate 2 is connected to a charge
reservoir i.e. earth.
ELECTROSTATICS
QUIZRR 97
5Q
5Q 10Q
3 3
3Q 5Q 10Q 3Q
3 3
2d d
Charges on the surface of the plate 2 facing 1 and 3 will be divided in the ratio of their
capacitance.
Sum of the charges on the outer plates 1and 2 remain constant.
Solution :
Charge on the innermost surface can be found using the concept of 2Q
+Q Q
shielding effect, from which we derived that equal and opposite charge was 2Q
2Q
induced on the surface of the cavity. Q
Ex a m p l e 5 8
If the inner and outer shells are connected by wire and middle shell is grounded. Then, find
the charge distribution on each shell? Initially A and C had Charge Q each.
Solution :
Concept :
2 Q -[q 1 -Q -q 2 ]
(i) VB = 0 (grounded)
q2
C
(ii) VA = VC with (QA+C)initial = (QA+C) final
B
q2
–q 1 A
Assume the charge on the outer surface of A as q1 and that on Q
q1
ELECTROSTATICS
98 QUIZRR
Now VB = 0
Kq1 K q2 q1 K 3Q q1
K
3Q q1 q2
... (2)
a b c c
Ex a m p l e 5 9
In the previous question, find the charge distribution if the middle and inner shell are
grounded.
(Q + q
Q q )
Q q
Q O O
Q
Q Q
Solution :
Co n c e p t t o l e a r n
There cannot be any charge on the surfaces which are equipotential and facing each other.
[Otherwise, field lines would exist.]
Let q be the charge on outer surface of plate B.
Now, put VB = 0
Kq KQ Qb
0 q c
b c
ELECTROSTATICS
QUIZRR 99
II. When the inner and outer spherical shells are ground.
Q
C Q Qb
Qa
(a+b)
Q (a+b)
b
a
Concept : The charge appearing on the inner and outer surfaces of the middle shell is divided in
the ratio of the capacitances of the inner and outer shell which is of course the ratio of their radii.
2 1. CA PA CI T A N CE
In practice we cannot handle free point charges or hold them fixed at desired positions. A practical
way to handle a charge would be to put it on a conductor. Thus, one use of a conductor is to store
electric charge (or electric potential energy). But every conductor has, its maximum limit of storing the
electric charge or potential energy. Beyond that limitthe dielectric (or insulator) in which the conductor
is placed becomes ionized. A capacitor is a device which can store more electric charge or potential
energy compared to an isolated conductor.
Capacitors have a tremendous number of applications. In the flash light used by photographers
the energy and charge stored in a capacitor are recovered quickly. In other applications the energy is
released more slowly.
2 1 .1 Ca p a c i t a n c e o f a n I s o l a t e d Co n d u c t o r
When a charge q is given a conductor it spreads over the outer surface of the conductor. The whole
conductor comes to the same potential (say V). This potential V is directly proportional to the charge
q, i.e.
V q +
+ +q
+ +
1 +
when the proportionality sign is removed a constant of proportionality comes v +
C + +
in picture. Hence, +
+ +
V
q + + +
C
q
or C
V
ELECTROSTATICS
100 QUIZRR
Here, C is called the capacitance of the conductor. The SI unit of capacitance is called one farad
(1 F). One farad is equal to coulomb per volt (1 C/V).
1 F = 1 farad = 1 C/V = 1 coulomb/volt
2 1 .2 M e t h o d o f Fi n d i n g Ca p a c i t a n c e o f a Co n d u c t o r
Give a charge q to the conductor. Find potential on it due to charge q. This potential V will
obviously be a function of q and finally find q/V, which is the desired capacitance C.
2 1 .3 Ca p a c i t a n c e o f a s p h e r i c a l c o n d u c t o r
When a charge q is given to a spherical conductor of radius R, the potential on it is,
1 q
V . +
4 0 R + + q ++
+ + ++ +
+ + ++ +
q
From this expression we find that, 4 0 R C
V R
+
+
+ + ++ +
Thus, capacitance of the spherical conductor is,
C = 40 R
From this expression we can draw the following conclusions.
(i) C R or C depends on R only. Which we have already stated that C depends on the
dimensions of the conductor. Moreover if two conductors have radii R1 and R2 then,
C1 R1
C2 R 2
(ii) Earth is also a spherical conductor of radius R = 6.4 106 m. The capacity of earth is
therefore
C
1
9 109
6.4 106 711 106 F
or C = 711 øF
From here, we can see that farad is a large unit. As capacity of such a huge conductor is
only 711 øF.
2 1 .4 En e r g y s t o r e d i n a c h a r g e d c o n d u c t o r
Work has to be done in charging a conductor against the force of repulsion by the already existing
charges on it. The work is stored as a potential energy in the electric field of the conductor. Suppose
a conductor of capacity C is charged to a potential V0 and let q0 be the charge on the conductor at this
instant. The potential of the conductor when (during charging) the charge on it was q (< q0) is,
q
V
C
ELECTROSTATICS
QUIZRR 101
q
d W Vdq dq
C
q0 q0 q 1q2
W 0 dW 0 C
dq 0
2C
1 q02
U
2 C
Further by using q0 = CV0 we can write this expression also as,
1 1
U CV02 q0 V0
2 2
In general if a conductor of capacity C is charged to a potential V by giving it a charge q, then
1 1 q2 1
U CV 2 qV
2 2 C 2
2 1 .5 Re d i s t r ib u t io n o f Ch a r g e
+ + + +
+ + + + + +
+ + +
+ + +
+ +
+
+
+ +
+
v
+
+ +
+
+
v
R1 R2
+
+
+ +
+
+ +
+
+ + + +
+ + + +
q1
q2 q1' q2'
Suppose two conductors of capacities C1 and C2 have charges q1 and q2 respectively when they are
joined together by a conducting wire, charge redistributes in these conductors in the ratio of their
capacities. Charge redistributes till potential of both the conductors become equal. Thus, let q1´ and q2´
be the final charges on them, then
q´ C
q1' C1
or
q2' C2
C1 R1
and if they are spherical conductors, then C R
2 2
q1' C1 R1
q2' C2 R2
ELECTROSTATICS
102 QUIZRR
Since, the total charge is (q1 + q2). Therefore,
C1 R1
q1'
q1 q2 q1 q2
C1 C2 R1 R 2
C2 R2
and q2'
q1 q2 q1 q2
C1 C2 R1 R 2
Ex a m p e 6 0
Two isolated spherical conductors have radii 10 cm and 20 cm respectively. They have
charges of 10 ø and 5 øC. Find the charges after they are connected by a conducting wire.
Also find the common potential after redistribution.
Solution :
+ +
+ + + + +
+ + +
+ +
+ + +
+ + +
+
v
+
+
v
R1 R2
+
+
+ +
+
+ + + + + + +
10 C 5 C q1' q 2'
q1' R1 10 1
q2' R 2 20 2
1 5
q1' 5 C
1 2 3
2 10
and q2' 5 C
1 2 3
common potential V
q1 q2
5 106
C1 C2 4 0 R1 R 2
5 10 9 10
6 9
=
15 10 2
ELECTROSTATICS
QUIZRR 103
Ex a m p e 6 1
An insulated conductor initially free from charge is charged by repeated contacts with a
plate which after each contact is replenished to a charge Q. If q is the charge on the
conductor after first operation prove that the maximum charge which can be given to the
Qq
conducor in this way is
Qq
Solution :
Let C1, be the capacity of plane and C2 that of the conductor. After first contact charge on
conductor is q. Therefore, charge on plate will remain Q q. As the charge redistributes in the ratio
of capacities.
Q q C1
... (i)
q C2
Let qm be the maximum charge which can be given to the conductor. Then flow of charge from
the plate to the conductor will stop when,
Vconductor = Vplate
qm Q
C2 C1
C
qm 2 Q
C1
C2
Substituting C from Eq. (i), we get
1
Qq
qm Proved
Qq
Ex a m p l e 6 2
A conducting sphere S1 of radius r is attached to an insulating handle. Another conducting
sphere S2 of radius R is mounted on an insulating stand S2 is initially unchrged. S1 is given
a charge Q brought into contact with S2 and removed. S1 is recharged such that the
charged on it is again Q and it is again brought into contact with S2 and removed. This
procedure is repeated n times :
(a) Find the electrostatic energy of S2 after n such contacts with S1.
(b) What is the limiting value of this energy as n ?
Solution :
Capacities of conducting sphere are in the ratio of their radii. Let C1 and C2 be the capacities of
S1 and S2, then
C2 R
C1 r
ELECTROSTATICS
104 QUIZRR
(a) Charges are distributed in the ratio of their capacities. Let in the first contact, charge acquired by
S2 is q1. Therefore, charge on S1 will be Q q1.
q1 C R
2
Q q1 C1 r
It implies that Q charge is to be distributed in S2 and S1 in the ratio of R/r.
R
q1 Q
R r
In the second contact, S1 again acquires the same charge Q.
Therefore, total charge in S1 and S2 will be
R
Q q1 Q 1
R r
This charge is again distributed in the same ratio. Therefore, charge on S2 in second contact,
R R R R
2
q2 Q 1 + Q
R+ r R r R + r R r
R R
2
R
3
Similarly, q3 Q ...
R + r R r R r
R R
2
R
n
qn Q ...
and R + r R r R r
R R
qn Q 1
n
S
a 1 rn
n
or r
R r ... (i)
1 r
qn2 qn2
Un
2C2 2 4 0 R
qn2
or Un Ans.
8 0 R
ELECTROSTATICS
QUIZRR 105
n 1
QR R R
qn 1 ...
(b) Rr R r as n
R r
1
QR QR R r R
q 1 R . Q a
Rr S
Rr Rr r r ;
1 r
q2 Q2 R 2 / r 2
U
2C2 8 0 R
Q2 R
or U Ans.
8 0 r 2
2 2 . CA PA CI T OR
When two conductor are placed at a small separation (close to each other), then this combination
is called capacitor.
+Q
Q Q V
Q = CV
(C = capacitance)
2 2 .1 Fa c t o r s o n w h i c h c a p a c i t a n c e o f t h e a r ra n g e m e n t d e p e n d
(i) Shape and size of the conductor
(ii) Relation orientation of the conductor
(iii) The medium in which they are placed
d Qd
=
0 0 A
Q Q 0 A 0 A
C
d V Qd d
0 A
capacitance of a parallel plate capacitor =
d
ELECTROSTATICS
106 QUIZRR
2 2 .2 Ge n e r a l m e t h o d t o f i n d t h e c a p a c i t a n c e
Give charge Q, Q to each surface and then find the potential difference V between the
Q
surface and use C =
V
a
Q
(Va Vb) = E. d r
+Q b
a b
KQ
(Va Vb) = . dr
r2
a
b
b
1 1 1
= KQ KQ
r a a b
Qb a
=
4 0 ab
Q 4 0 ab
C
V b a
Cases :
(i) When b i.e. the inner conductor is an isolated sphere, then
C = (4 0 a)
4 0 a 2
C
4 a2 0 A
0
d d d
ELECTROSTATICS
QUIZRR 107
b
(Va Vb) = 2 0 r dr b a
a
b
= ln
2 0 a
Q L
C
Vab ln b
2 0 a
2 0 L
C
b
l n
a
2 2 .3 Fo r c e b e t w e e n t h e p l a t e s o f a c a p a c i t o r
+Q Q
Electric field due to a single plate =
2 0
Q.Q Q2
Force on each plate =
20 A 2 0 A
d
2 2 .4 Po t e n t i a l En e r g y s t o r e d i n t h e c a p a c i t o r
Electrostatic potential energy is stored in the capacitor is the form of the electric field
Wex = F(d)
+Q Q
Q2 1 Q2
= 2 A d
0 2 C
1 Q2 1 1
Potential energy stored in the capacitor = CV 2 QV
2 C 2 2
d
ELECTROSTATICS
108 QUIZRR
1
l When the capaitor is connected with a cell use CV 2
2
1 Q2
l When the charge on the capacitor is constant, use 2 C
1 Q2
l When distance between the plates is increased Wex d2 d1
2 0 A
2 2 .5 En e r g y d e n s i t y b e t w e e n t h e c a p a c i t o r p l a t e s
Energy density = potential energy stored per unit volume
U 1 Q2 d 1 Q2
= Ad 2 A Ad 2
0 0 A 2
1
Energy density = 0 E2
2
Although, we have derived the expression for energy density in the capacitor, but is general this
is valid for energy density is electric field, that is the energy stored per unit volume is the electric field.
1
U= 0 E2
2
Using this we can calculate the self potential energy of a uniformly charged conducting sphere.
1
2 0 E
2
U0 = dV
R
2
1 KQ
U0 = 2 0 2
r
4r dr
2
Ui=O
1 2 1
= K 0 Q2 2 2 dr
Rr
2
KQ2
1 1
= Q2 2
162 0 r R 2R
ELECTROSTATICS
QUIZRR 109
KQ 2
Ui 0, U0 = 2R
R 2
Ui
1
2 0
r
30
4r dr
2
2 4 R
= 92 r 4 dr
0 0
Q
and = 4
R
3
3
9Q 2 (4 ) R 5 KQ 2
(4 )2 R 6 18 0 5 10R
KQ 2 KQ 2 3KQ2
UT = U 0 U i
2R 10R 5R
2 2 .6 Se r ie s a n d p a r a l le l c o m b i n a t io n o f c a p a c i t o rs
(A) Series Combination
C1 C2 C3
V = (V1 + V2 + V3)
Q Q Q Q
Ceq C1 C2 C3
1 1 1 1 1
..... (Series combination)
Ceq C1 C2 C3 Cn
ELECTROSTATICS
110 QUIZRR
Ex a m p l e 6 3
C C C
B
F
Since the Chain repeats itself, let us assume after one link i.e. capacitance between E and F to
be x, then
Equivalent circuit diagram,
c E Then, Ceq between A and B will also be x as,
A
it is an infinite loop Hence
c x
C C x
x
B
F
2C x
x (2C + x) = C (C + x)
x2 + Cx C2 = 0
C C2 4C2 5 1
x C
2 2
(ii) C mC m2C m3C
A
C mC m2C m3C
ELECTROSTATICS
QUIZRR 111
Ex a m p l e 6 4
Find the value of Cx so that equivalent capacitance between A and B becomes independent
of the number of repeative links.
2c 2c 2c 2c
2c P
A
c c c c Cx
Q
Q
Solution :
For the equivalent capacitance to be independent of the number of links, the value of equivalent
capacitance between P and Q should be same as Cx.
Ceq(PQ) = Cx
2C.Cx C Cx
2C C x
(2C.Cx) + C (2C + Cx) = Cx (2C + Cx)
C2x Cx C 2C2 = 0
1 9
Cx = C
2
Cx = (2C)
Ex a m p l e 6 5
Find the equivalent capacitance of the given arrangement of spherical
capacitors,
(i) When inner is given a charge Q and outer is earthed,
(ii) When inner is earthed and the outer is given a charge Q. a
b
Solution :
(i) The outer surface of the outer sphere will have no charge (Vouter = 0) Q
+Q
Now this combination behaves as a singe spherical capacitor
4 0 ab
C
b a
ELECTROSTATICS
112 QUIZRR
(ii) When inner is earthed
Vinner = 0
q (Q+q)
+q Kq KQ
a b 0
Qa
q
b
b b
Qa 1
Vab E. d r K dr
a a b r
b
Qa 1 KQa b a
= K
b ra b ab
Q b2 4 0 4 0 b2
C =
Vab b a b a
2nd m et hod :
In the above arrangement, we have two capacitors in parallel arrangement, the inner shell and
inner surface of the outer capacitor and the outer surface of the outer sphere.
4 0 ab
Ceq = 4 0 b
b a
ab b b a
= 4 0
b a
4 0 b2
=
b a
3rd m et hod :
Write the individual potential of each surface
Vinner = 0
Kq KQ
Vouter =
b b
K Qa Qa
= Q q =
b b b
KQ
= b a
b2
ELECTROSTATICS
QUIZRR 113
KQ
V = b a
b2
2
Q 4 0 b
C =
V b a
Ex a m p l e 6 6
Find the equivalent capacitance of the given arrangement.
+Q Q
a d b
O O´
d >>(b,a)
Solution :
Let V+ be the potential of sphere centred at 0 and V be the potential of shere centred at 0´.
KQ KQ
V+ =
a d
KQ KQ
V =
b d
V = V+ V
KQ KQ 2KQ
=
a b d
1 1 2
= KQ
a b d
Q 4 C0
C 1 1 2
V
a b d
Ex a m p l e 6 7 a + – a
b–r
ELECTROSTATICS
114 QUIZRR
Solution :
Consider + and charge density on each. Find the expression for electric field and then
Use V = E.dr
Let us Calculate electric field at a distance r from centre of cylindrical wire of charge density +.
Then the net electric field is given by :
E
2 0 r 2 0 b r
1 1
= 2
0 r b r
b a
1 1
V = 2 0
r b
dr
r
a
b a b a
1 1
= 2 r dr b r
0
a a
b a a
= 2 ln ln
0 a ab a
b a
ln
0 a
=
l 0 l 0 l
C= b a
V ln b a ln b
a a
Ex a m p l e 6 8
A long cylindrical wire of radius a is placed at a distance b from an infinite large plane
sheet. Find the equivalent capacitance of the given arrangement.
+ –
a a
b b
ELECTROSTATICS
QUIZRR 115
Soution :
Whenever we en counter an infinite large place sheet along with another conductor, we assume
a similar distance placed conductor on the opposite side (just as a mirror image about the
sheet) with opposite charged. This is called image method.
Using the image method we obtain the image of the long wire in infinite plane sheet.
E
2 0 r 2 0 2b r [Note the distance is 2b]
1 1
2 0 r 2br
=
b
1 1
V = 2 0
r 2b
.dr [Note again the limits put as the real potential to be
r
a
calculated is upto the Large plane sheet.]
b b
=
2 0 ln a ln 2b a
b 2b a
=
2 0 ln a b
2 0
C
2b a
ln
a
Ex a m p l e 6 9
Find the equivalent capacitance in the given arrangement.
(i) 2
P
1 A 1
2 A P 3 B
B
3 C 3C
C eq C
2 2
(ii)
A
A P B
B
2C2 2C
Ceq
3C 3
ELECTROSTATICS
116 QUIZRR
1
A
P 2 2
(iii)
3 1
B A P 3 B
4
4
2C 5C
C eq C
3 3
1 P
A 1
(iv) P 2 4
3d Q 3
A B
4 Q 5
B 2
5
Q
C C
Ceq C
2 2
2 2 .7 Wh e a t st o n e ’s b r i d g e c i r c u i t s E
C1 C3
If , bridge is said to be balanced and in that case C1 C2
C2 C4
VE = VD or VE VD or VED = 0 A B
C5
i.e., no charge is stored in C5. Hence, it
can be removed from the circuit. C3 C4
D
ELECTROSTATICS
QUIZRR 117
Ex a m p l e 7 0
Find the equivalent capacitance between A and B.
C 2C
A 2C B
2C C
Solution :
The given circuit forms a Wheatstone bridge. But the bridge is not balanced. Let us suppose point
A is connected to the positive terminal of a battery and B to the negative terminal of the same battery;
so that a total charge q is stored in the capacitors. Just by seeing input and output symmetry we can
say that charges will be distributed as shown below.
q
C 2
+ +
q1 2C
A 2C
B
q3
2C C
+
+
q q1
2
q1 q q
3 2 0
C 2C 2C
or q2 q3 2q1 = 0 ... (ii)
Plates inside the dotted line form an isolated system. Hence,
q2 + q3 q1 = 0 ... (iii)
2 3 q
Solving these three equations, we have q1 q, q2 q and q3
5 5 5
Now, let Ceq be the equivalent capacitance between A and B. Then,
q q q
VA VB 1 2
Ceq C 2C
q 2q 3q 7q
Ceq 5C 10C 10C
10
C eq C
7
ELECTROSTATICS
118 QUIZRR
2 3. EFFECT OF DI ELECT RI C
Michel Faraday found that when certain non-conducting materials such as glass, paper or plastic
are introduced between the plates of a capacitor, its capacity increases. These materials are called
dielectrics and the ratio of capacity of a capacitor when completely filled with dielectric C to that
C
without dielectric C0 is called dielectric constant K, relative permitivity r = C
0
Note : Regarding dielectrics, it is worth nothing that :
(i) These are non-conductors upto a certain value of field depending on its nature. If the field
exceeds this limiting value called dielectric-strength, dielectric lose its insulating property
and begins to conduct.
(ii) These have either permanent dipole moment (polar-dielectrics, e.g., water) or acquire
induced dipole moment (non-polar dielectrics) when placed in an electric field.
(iii) The dielectric constant of polar dielectric depends on its temperature and due to thermal
agitation with rise in temperature decreases.
The effect of dielectric on other physical quantities such as charge, PD, field and energy associated
with a capacitor depends on the fact that whether the charged capacitor is isolated (i.e., charge is held
constant).
So if q0, C0, V0, E0 and U0 represent the charge, capacity, PD, electric field and energy associated
with charged air capacitor respectively, with introduction of a dielectric slab of dielectric constant K
between the plates.
qo q
K
C0, V0, E0, U0 C, V, E, U
(A) (B)
q q
(3) PD between the plates decreases, i.e., V = (V0/K) as V 0 [as q = q and C = KC ]
C KC0 0 0
V V0 E0 V0 V0
E as V K and E0 d
d Kd K
ELECTROSTATICS
QUIZRR 119
q2 q02 U
U 0 [as q = q0 and C = KC0 ]
2C 2KC0 K
qo q
(A) (B)
(3) Charge on capacitor increases, i.e. q = Kq0 as, q CV = (KC0 )V Kq0 {as q0 CV0 }
V V V0
E 0 E0 as V = V0 and E0
d d d
(5) Energy stored in the capacitor increases, i.e., U = KU0 as, U = 1 CV 2 1 (KC0 ) (V0 )2
2 2
1 1 2
KU 0 [as C = KC0 and U 0 C0 V0 ]
2 2
Note : While solving problems of this type proceed in the following steps :
(i) If battery is disconnected q´ = q and if battery remains attached V´ = V
(ii) For a capacitor as C = (0 KA/d), find whether C´ is greater or lesser than C in accordance
with given problem (as C depends on K, A and d only)
(iii) Now using q´ = C´V´ and points (1) and (2), find q´ if V´ is given and V´ if q´ is given.
(iv) To calculate E´ use the formula :
(v) To calculate U´ use the formula : U'= 1 q ' V'= 1 C'(V') 2 (q ')2 / 2C' as the case may be
2 2
(vi) To calculate W apply conservation of energy i.e., WIF = (UF UI)
ELECTROSTATICS
120 QUIZRR
2 3 .1 Ca p a c i t o rs w it h m o r e t h a n o n e Di e l e c t ri c s l a b s :
(I) A parallel plate capacitor contains two dielectric slabs of thickness d 1, d 2 and dielectric constants
k1 & k2 respectively. The area of the capacitor plates and slabs is equal to A.
Considering the capacitor as a combination of two capacitors in series, the equivalent capacitance
C is given by :
1 1 1
C C1 C2
1 d1 d2
C k1 0 A k2 0 A
K1 d1
0 A K2 d2
C=
d1 d2
k1 k2
0 A
C= d
kii
If V is the potential difference across the plates, the electric fields in the dielectrics are given as :
E1
V1
Q
CV use C1 = k1 0 A and C = A 0
d1 C1 d1 C1 d1 d1 d1 d2
K1 K 2
1 V 1 V
E1 E2
d1 d2 k1 d1 d2 k2
k1 k2 k1 k2
Note : k1 E1 k2 E2 and E1 d1 E2 d2 V
(II) If there exists a dielectric slab of thickness t inside a capacitor whose plates are separated by
distance d, the equivalent capacitor is given as :
0 A
C=
t dt (K = 1 for vacuum)
K 1
Vacuum
d
0 A K t
C=
t
dt
K
ELECTROSTATICS
QUIZRR 121
The equivalent capacitance is not affected by changing the distance of slab from the parallel
Plates.
If the slab is of metal the equivalent capacitance is :
0 A
C= (for a metal, K = )
d-t
(III) Consider a capacitor with two dielectric slabs of same thickness d placed inside it as shown. The
slabs have dielectric constant k1 & k2 and areas A1 & A2 respectively. Treating the combination
as two capacitors in parallel,
C = C1 + C2
k1 0 A1 k2 0 A 2
C=
d d d K1 K2
0
C= [ k1 A1 k2 A 2 ]
d
Ex a m p l e 7 1
The plates of the capacitor formed by inserting four-dielectric slabs (as shown) have an area
equal to S. Find the equivalent capacitance between A and B if K1 = 2K2 = K3 = K4 = 5.
A
1/4 1/4 1/2
K3C3 d/2
K1C1 K2C2
K4C4 d/2
Solution :
Consider the capacitor as a parallel combination of C1 and C2 and series combination of C3 and
C4 Equivalent Capacitance = C1 + C2 + C3C4/C3 + C4
0 S S 0S/2
C = k1 k2 d / 2 d / 2
d 4 4
k3 k4
ELECTROSTATICS
122 QUIZRR
Ex a m p l e 7 2
Two metal plates form a parallel plate capacitor. The distance between the plates is d. A
metal plate of thickness b (= d/2) and same area is inserted completely between the plates.
What is the ratio of the capacitance in the two cases ?
Solution :
Before the introduction of the slab the capacity of the system
q q
C0 [as E = (V/d)]
V (Ed)
A A
or, C0 0 as q A and E = ...(1)
(/0 ) d d 0
When the metal slab of thickness b is introduced between the plates, the system becomes equal
to two capacitors C1 and C2 in series (as metal slab is an equipotential surface) as shown in (B).
V V
C1 d-b-y
d V´
b Metal
V´
y C2 y
O O
(A) (B)
1 1 1 1 ( d b y) y 0 A
, i.e., as C = d
C C1 C2 C 0 A 0 A
1 (d b) 0 A
or, , i.e., C= ...(2)
C 0 A ( d b)
0 A A C
C= 2 0 2C0 . i.e., 2 Ans.
(d /2) d C0
ELECTROSTATICS
QUIZRR 123
Ex a m p l e 7 3
How does the charge, capacity, potential, field and energy associated with a capacitor
change qualitatively when the separation between its plates is increased (a) with battery
disconnected (b) with battery attached.
Solution :
(a) When battery is disconnected.
q0 q
(A) (B)
In this situation :
(1) Charge remains unchanged, i.e., q = q0 as charge in an isolated system is conserved.
(2) Capacity decreases, i.e., C < C0 as C (1/d).
(3) PD increases, i.e. V > V0 as q = CV and for constant q, V (1/C).
(4) Electric field remains constant, i.e., E = E0 as E = / = 0/ = constant.
(5) Energy stored increases, i.e., U > U0 as U = 1/2qV, i.e., U V for constant q.
q0 q
(A) (B)
(1) PD across the capacitor remains constant, i.e. V = V0 as battery is a source of constant emf.
(2) Capacity decreases, i.e., C < C0 as C (1/d).
(3) Charge on the capacitor decreases, i.e., q < q0 as q = CV, i.e., q C (for constant V.
(4) Electric field between the plates decreases, E < E0 as E = (V/d), i.e., E (1/d) for constant V.
1
(5) Energy stored in the capacitor decreases, i.e., U < U0 as U = qV, i.e., U q for constant V.
2
ELECTROSTATICS
124 QUIZRR
M I SCEL L A N EOU S EX A M PL ES
Ex a m p l e 1
A point charge q is placed on the top of a cone of semi vertex angle . Show that the
q 1 cos
electric flux through the base of the cone is
2 0
How to Proceed : This problem can be solved by the method of symmetry. Consider a Gaussian
surface, a sphere with its centre at the top and radius the slant length of the cone. The flux through
the whole sphere is q/0. Therefore, the flux through the base of the cone can be calculated by using
the following formula,
S q
e .
S0 0
Here, S0 = area of whole sphere
and S = area of sphere below the base of the cone
Solution : Let R = slant length of cone = radius of Gaussian sphere
S 0 = area of whole sphere = (4R2)
Cq
S = area of sphere below the base of the cone
R
= 2R2 (1 cos )
A B
S q
The desired flux is, e .
S0 0
2R 1 cos . q
2
q 1 cos
=
4R
2
0 2 0 Proved
d
2R sin d
2
S = R
0 c
S = 2R2 (1 cos )
ELECTROSTATICS
QUIZRR 125
Ex a m p l e 2
A point charge q is fixed at the origin. Calculate the electric flux through the infinite
plane z = a .
Solution :
Consider a ring of radius r on the plane z = a. The E
thickness of ring is dr. Electric field on the ring due to
r
charge q is, z=a
dr
1 1
E .
4 0 r a2
2
a
r2 + a2
Z
Area of the ring dS = (2r).dr y
a x
cos = 2 2 q
r a
1 q a
d e E dS cos . 2 . 2r dr .
4 0 r a 2
r a2
2
q ar dr
2 0
r
3/2
2
a2
qa rdr
e d e
2 0
r
3/2
2
0 0 a2
q
or e
2 0
Ex a m p l e 3
B C
Is the configuration of an electrostatic field shown in figure
is possible.
A D
ELECTROSTATICS
126 QUIZRR
Solution :
No, it is not possible. To prove it, suppose we take a closed path ABCDA in the electric field, the
line integral over this path should be zero.
E. dl 0
Because this quantity represents VP VP, where P is any point on the path and VP VP is
obviously zero.
Along the path AB and CD the line integral of electric field is zero. Since, E dl and
B D
E. dl E. dl 0
A C
C A
Thus, for E. dl 0, E. dl should be equal to E. dl
B D
But since, lines are closer along BC, i.e., EBC E DA
C A
E. dl E. dl
B D
or E. dl 0
Thus, such a configuration of an electrostatic field shown in figure canÊt be realised in practice.
Ex a m p l e 4
A solid conducting sphere A with a radius R1 = 6 cm has an initial electrostatic potential
V0 = 30 kV relative to ground. Sphere A is then symmetrically surrounded by two conducting
and initially neutral hemispheres B1 and B2. In this way a sphere B with inner radius
R2 = 10 cm and outer radius R3 = 12 cm is formed.
(a) What is the net charge on sphere A ?
(b) What are the charges induced on the inner and outer surface of sphere B ?
(c) What are the electric potentials of spheres A and B ?
(d) What are the electric fields at the surface of sphere A and at the inner and outer
surfaces of sphere B ?
(e) Draw separate graphs of the electric potential and the electric fields as a function
of distance x from the centre of the system.
(f) How would the charge be distributed if the spheres A and B were connected by
a conducting wire ?
ELECTROSTATICS
QUIZRR 127
1 q
Solution : (a) V0 . 1
4 0 R1
A B1
V0 R1 R1 O
q1 B2
1
R2
4 0 R3
30 10 6 10
3 2
=
9.0 10 9
7
= 2.0 10 C = 0.2 øC Ans.
(b) Charges induced on inner side of B = q1 = 0.2 øC and on outer side of B = q1 = 0.2 øC
Ans.
1 q1 q q
(c) VA 2 3
4 0 R1 R 2 R 3
q1 1 1 1
=
4 0 R1 R 2 R 3
Substituting the values,
VA 0.2 106 9.0 10 10 16 101 121
9 2
VB
1 q1
9.0 109 0.2 106
15 kV
Ans.
4 0 R 3 12 10 2
(d) Electric field at the surface of sphere A,
E1
1
.
q1
9.0 109 0.2 10 6
5 105 V/m
40 R12
2
2
6 10
2 2
1 q1 R1 6
E2 2 E1 0.5
4 0 R 2 R2 10
ELECTROSTATICS
128 QUIZRR
Electric field at the outer side of sphere B,
2 2
1 q1 R1 6
E3 2 E1 0.5
4 0 R 3 R3 12
(f) When the two spheres are brought in contact, then all of the charge q1 transfers to the
outer sphere B.
Ex a m p l e 5
A thick metallic shell of inner radius a and outer radius b has a charge q 1 on it. A point
charge q 2 is fixed at the centre of the shell. Calculate the charge on each surface of the
shell and also potential and field everywhere.
Solution :
The charges on inner and outer surfaces of the shell are as shown in figure. Field lines starting from
q2 terminates on the inner surface of the shell. The field at external points (r b) is the same as that
due to a point charge (q1 + q2) at the centre, i.e.
q1 q2
E r b
4 0 r 2 +(q1 + q2 )
q2
Er a
40 r 2
q1 q2
Similarly, V r b
4 0 r
1 q1 q2 q2 q2
V r a
4 0 b a r
ELECTROSTATICS
QUIZRR 129
1 q1 q2
From r = b to r = a the potential is uniform and has the value,
4 0 b
Figure shows the variation of V (r) and E (r)
E V
O a b r O a b r
Ex a m p l e 6 1 2 3 4 5 6
Three identical metallic plates are kept parallel to one another at a
separation of a and b. The outer plates are connected by a thin
conducting wire and a charge Q is placed on the central plate. Find
final charges on all the six plateÊs surfaces. a b
Solution : Q
q2 q1 (Q q 1)
Let the charge distribution in all the six faces be as shown in figure. q1 q3
While distributing the charge on different faces, we have used the fact that
two opposite faces have equal and opposite charges on them. (q 1 Q)
E1
Net charge on plates A and C is zero. Hence, E2
q2 q1 + q3 + q1 Q = 0
a b
or q2 + q3 = Q ...... (i)
Further A and C are at same potentials.
Hence,
VB VA = VB VC or E1a =E2b
q1 Q q1
.a .b (A = Area of plates)
A0 A0
Qb
q1a = (Q q1) b q1 ... (ii)
ab
Electric field inside any conducting plate (say inside C) is zero. Therefore,
q2 q1 q1 Q q1 q1 Q q3
0
2A0 2A0 2A0 2A0 2A0 2A0
q2 q3 = 0 ... (iii)
ELECTROSTATICS
130 QUIZRR
Qb Q
Solving these three equations, we get q1 a b , q2 q3
2
Hence, charge on different faces are as follows.
Face Charge
Q
1 q2
2
Qb
2 q1
ab
Qb
3 q1
ab
Qa
4 Q q1
ab
Qa
5 q1 Q
ab
Q
6 q3
2
Ex a m p l e 7
Find the field intensity and potential due to a uniformly charged rod with charge per unit
length and length l at a distance d on the perpendicular bisector and along its length.
Solution :
(a) At a distance d on the perpendicular bisector from the centre horizontal component of E will cancel
each other
dEy = dE cos dE dE
K dx
= cos
r
d
l / 2 x dx
E d E cos x-axis
l / 2 dx
y-axis
l / 2
K dx d
=
l / 2 x 2
d 2
x2 d 2
l/2
dx
= K d
x
3/ 2
2
l / 2 d2
ELECTROSTATICS
QUIZRR 131
l/2
Kd x
K 2l
=
2 2 d
d 2
l2 4 d 2
x d l / 2
2l
= 4 0 d
l 4 d2
2
l
E
2 0 d l2 4 d 2
K dx d
dE
d x2
x
l/2
dx
E K dx
P
l / 2 d x
2
l/2
1 d l / 2 d l / 2
K K
= x
d l l
l / 2 d d
2 2
4Kl 4Kl l
=
4d 2
l 2
4d 2
l 2
2
0 4d l
2
Ex a m p l e 8
Two conductors carrying equal and opposite charges create a non-uniform field as shown
in fig. What is the capacity of this capacitor if the field along y-asix varies as ?
Q
B
y
d
+Q x
A
Q
E = [1 By2 ] with B = constant.
0 A
dV Q 2
= [1 By ]
dy 0 A
VB Q d
or,
VA
dV =
0 A 0
(1 By2 ) dy
Q 1
d Bd 3
0 A
or, VA VB =
3
Qd 1
1 Bd 2
0 A
or, V =
3
Q 0 A
So, C =
V 1
d 1 Bd 2
3
Note that this capacity is lesser than that of a parallel plate capacitor [i.e., C = (0A/d)] which has
uniform field between the plates.
Ex a m p l e 9
A parallel plate capacitor has the space between the plates filled with a medium whose
dielectric constant increases uniformly with distance. If d is the distance between the
plates and K1 and K2 are the dielectric constant of the medium at the two plates (square
each of area A) respectively show that the capacity of the capacitor.
0 A(K 2 K1 )
C=
d log e (K 2 / K 1 )
Solution : As dielectric constant varies uniformly with distance :
dK
b const, i.e., K = a + bx
dx
But as for x = 0, K = K1 and for x = d, K = K2, i.e., a = K1 and b = (K2 K1)/d
So, K = K1 + bx with b = (K2 K1)/d
V1 V1
K1 x dx
x
d d K1 K2
K2 dx
V2 V2
L L
(A) (B)
ELECTROSTATICS
QUIZRR 133
And hence, E=
0 K 0 (K1 bx)
dV dV
or, as E dx
dx 0 (K 1 bx)
V2 d dx
or, dV 0 ( K1 bx)
V1 0
d
i.e., V1 V2 = [log e ( K1 bx)]
0 b 0
Q Q
or, V [log e (K1 bd) log e K1 ] as = A
0 Ab
Qd K2 (K 2 K1 )
or, V log e as b =
0 A(K 2 K1 ) K1 d
Q A( K 2 K1 )
or, C = 0 Ans.
V d log e ( K2 / K1 )
Note : In this problem if the dielectric constant increases at a uniform rate from one edge to
the other (so that the value of K at the two edges are K1 and K2 respectively [fig.(b)],
the capacitor may be supposed to be constituted of large number of elemental capacitors
connected in parallel. In this situation if we consider an elemental capacitor of width dx
at distance x from the edge having dielectric constant K1 :
So, on integrating
0 L' L LL' 1
C=
d 0
(K1 bx) 0
d
[ K1 bL]
2
0 A K1 K 2 K 2 K1
C= as LL' = A and b =
d
or,
2 L
ELECTROSTATICS
WAVES & SOUNDS
QUIZRR 3
W AVES
A wave is a disturbance which propagates energy (and momentum) from one place to the other
without the transport of matter. It is well spread over a region of space without clear cut boundaries.
It cannot be said to be localised here or there. It is hard to think of any mass being associated
with a wave. Moreover, quantities like amplitude, wavelength, frequency and phase are used to
characterise a wave which have no meaning for a particle.
A wave may or may not require a medium for its propagation. The waves which don’t require
medium for their propagation are called non-mechanical, e.g., light, heat (infrared) and radio
waves are non-mechanical as they can propagate through vacuum. In fact all electromagnetic
waves (EMW) such as -rays, X-rays or microwaves are non-mechanical. On the other hand the
waves which require medium for their propagation are called mechanical waves. In the propagation
of mechanical waves elasticity and density of the medium play an important role. This is why
mechanical waves sometimes are also referred to as elastic waves on string and springs, seismic
waves or sound waves are familiar examples of mechanical waves.
Note : Apart from mechanical (elastic) and non-mechanical (electromagnetic) waves there is also
another kind of waves called Âmatter wavesÊ. These represent wavelike properties of particles and
are governed by the laws of quantum physics.
Vibration C C C C
Wate motion T T T
(A) (B)
(ii) Longitudinal waves : If the particles of a medium vibrate in the direction of wave motion,
the wave is called longitudinal. These are propagated as compressions and rarefactions and
also known as pressure or compressional waves. Waves on springs or sound waves in air are
examples of longitudinal waves.
R R R R
Vibration
Wave motion
C C C C
(A) (B)
Note :
(1) All non-mechanical waves are transverse
(2) In gases and liquids mechanical waves are always longitudinal e.g. sound waves in air and
water.
(3) In solids, mechanical waves can be either transverse or longitudinal depending on the mode
of excitation. The speeds of the two waves in the same solid are also different.
Example 1
Explain why (a) transverse mechanical waves cannot be propagated in liquids and gases
while (b) waves on strings are always transverse.
Solution :
(a) To transmit a transverse mechanical wave the medium must be elastic so as to provide a
restoring force when acted on by shearing stress. But liquids and gases flow when acted
on by shearing stress, i.e. they cannot sustain shear stress to provide restoring force and so
cannot transmit transverse mechanical waves.
(b) Longitudinal waves are pressure waves, i.e., they are transmitted as compression and
rarefaction in a medium. Now as the string is non-stretchable so it can neither be compressed
nor stretched, i.e., in it compression and rarefaction cannot be produced. This in turn implies
that longitudinal waves cannot be propagated along a string [So the waves in a string are
always transverse, that too when the string is under tension. If tension in the string is zero,
(a) Wave frequency : If the particles of the medium make n (also written as v) vibrations for
second, n (or ) is called the frequency of the wave. The time taken for one vibration is the
1 1
wave period T and T = or ; unit ă hertz (Hz).
T
(b) Wavelength : It is defined as the distance travelled by the wave in one period T unit-metre.
Crest
Trough
It can also be defined as the distance between two successive crests or between two successive
troughs.
(c) Wave velocity : It is the distance travelled by the wave in one second symbol v or c; unit
- metre/second.
If the frequency of the wave is „f‰ and wavelength is „‰ metres, then wave velocity v is
v = f m/s ...(1)
wave velocity = Frequency ï Wavelength
Some physical quantity (say y) is made to oscillate at one place and these oscillations of y
propagate to other places. The y may be,
(i) displacement of particles from their mean position in case of transverse wave in a rope or
longitudinal sound wave in a gas.
(ii) pressure difference (dP) or density difference (d) in case of sound wave or
(iii) electric and magnetic fields in case of electromagnetic waves.
2 y 2 y
k
t2 x2
2 y 2 y
2 ...(i)
t2 x2
The general solution of this equation is of the form y (x, t) = f (ax bt) ...(ii)
Thus, any function of x and t which satisfies Eq. (i) or which can be written as Eq. (ii) represents
a wave. The only condition is that it should be finite everywhere and at all times. Further, if these
conditions are satisfied, then speed of wave () is given by,
coefficient of t b
coefficient of x a
The plus (+) sign between ax and bt implies that the wave is travelling along ă ve x-direction and
minus (ă) sign shows that it is travelling along +ve x-direction.
Example 2
1
ă t)2
(a) (x ă t)2 (b) ln (x + t) (c) eă(x (d) x t
Solution :
Although all the four functions are written in the form f (ax bt), only (c) among the four
functions is finite everywhere at all times. Hence only (c) represents a wave.
Example 3
0.8
( x , t ) represents a moving pulse where x and y are in metre and t in
[(4 x 5t )2 5]
0.8
y 0, 0 0.16 m
5
0.8 0.16m
y x 2
16 x 5
0.8
and y x
16 x2 5
X
ăx O x
or y(x) = y(ă x) t=0
Therefore, pulse is symmetric.
Speed of pulse : At t = 1 s and x = 1.25 m
Y Y
0.16m 0.16m
ăX X
x=ă1.25m x=0
t=1s t=0
Alternate method :
If equation of a wave pulse is y = (ax bt)
b
the speed of wave is in negative x-direction for y = f (ax + bt) and positive x-direction for
a
5
y = f (ax ă bt). Comparing this from given equation we can find that speed of wave is 1.25 m/s
4
and it is travelling in negative x-direction.
If a travelling wave is a sin or cos function of (at ă bx) or (at + bx), the wave is said to be harmonic
or plane progressive wave. Here we shall limit ourselves to 1 ă D plane progressive wave which
in its most general form is given by
y = A sin (t kx )
From Eqn. it is clear that a set of four parameters A, , and k completely describes a plane
progressive wave.
(1) As the maximum value of sin or cos functions can be 1, A represents the maximum value
of wave-function as is called the amplitude of the wave.
(2) The constant is called phase constant or initial phase and enables us to find the position
from where time is considered. If at t = 0, x = 0, will be zero which is usually the case with
a wave and implies that in wave motion time is considered when the wave was at the origin.
Henceforth we shall assume = 0 and the wave is travelling along positive x-axis unless
stated otherwise.
So the wave will repeat itself if y´ = y, i.e., t´ = t + (2/) as sin ( + 2) = sin .
Now as the time after which a wave repeats itself is called time period, i.e.
T = t´ ă t = (2/)
Further as the rate at which the wave repeats itself is called its frequency f (with units Hz)
so
1
f
T 2
or = 2f, is called angular frequency (with units rad/s). Here it is worth noting that
, f or T are the characteristers of the source producing the wave and are independent of
the nature of the medium in which the wave propagates.
x = constt. t = constt.
= v
T= = 1 =
f k f
y y
T/2 T 3T/2 x T/2 T x' 3T/2
t t'
t x
So the wave will repeat itself if y = y´, i.e., x´ = x + (2/k) as sin ( 2) = sin .
Now as the distance after which the wave repeats itself is called wavelength ,
2 2
so x´ x , i.e., k ...(5)
k
k is called propagation constant or wave vector and has unit (rad/m). The constant
k or wavelength depends on the nature of the medium (as same source will produce waves
of different wavelengths in different media) and also on the source producing the waves (as
in a given medium sources of different frequencies will produce different wavelengths).
(5) If the shape of the wave does not change as the wave propagates in a medium, with increase
in t, x will also increase in such a way that
t ă kx = constt. ...(6)
The argument of harmonic function (t ă kx) is called phase of the wave and is constant
if the shape of the wave remains unchanged.
2 2
i.e., = x as k ...(7)
From this it is clear that if x = , = 2, i.e., a path difference corresponds to a phase
change
dx 2f
v f
dt k 2 / ...(8)
(7) As a plane progressive wave propagating along positive x-axis with t = 0 at x = 0 is given
by
y = A sin (t ă kx)
dy
so the velocity of a particle on it will be vPa A cos t kx ...(10)
dt
so vPa = A 2
y2 ...(11)
The acceleration of the particle is the second partial derivative of y (x, t) with respect to t,
2 y x, t
aP = ă 2 A sin (kx ă wt)
t2
= ă 2 y
i.e. the acceleration of the particle equals ă 2 times its displacement. Thus
aP = ă 2 (displacement)
t x
or y = A sin 2 [as f = 1/T]
T
(a) The equation may be written as, y x, t 0.05 sin 5x 20t m
4
2
wave number k = 5 rad/m
= 0.4 m Ans.
The angular frequency is, = 2f = 20 rad/s
f = 10 Hz Ans.
The wave velocity is, v=f = 4 m/s in + x direction Ans.
k
y 5
= ă (20) (0.05) cos
t 2 4
= 2.22 m/s
2 y 5
= ă 20 0.05 sin
2
t2 2 4
Example 6
Y
Figure shows a snapshot of a sinusoidal travelling wave taken
P
at t = 0.3 s. The wavelength is 7.5 cm and the amplitude is 2
cm. If the crest P was at x = 0 at t = 0, write the equation of t = 0.3s
2 cm
travelling wave. X
1.2 cm
Solution :
Given A = 2 cm, = 7.5 cm
2
k= 0.84 cm 1
The wave has travelled a distance of 1.2 cm is 0.3 s. Hence the speed of wave
1.2
v 4 cm/s
0.3
Consider a plane wave propagating with velocity v in x-direction across an area s. An element of
material medium (density = kg/m3) will have a mass (sdx)
The displacement of a particle from its equilibrium position is given by the wave equation.
y = A sin (t ă kx)
1 2
Total energy, dE m Vmax
2
1
dE = sdx A2
2
Area= S
energy density =
dE
sdx
22 f 2 A 2 J / m3 dx
Intensity of the wave is defined as the power crossing per unit area.
I = 22 f2 A2 v (Watt/m2)
For wave propagation through taut string
s = ø, the linear density in kg/m
Energy per unit length = 22 f2 A2 ø
Note :
(1) The energy is the average value over a time period
(2) Intensity I A2
(ø and f are constant)
Example 7
64t x
The equation of a progressive wave is given by as y = 0.05 sin 2 where the
3.2
amplitude and wavelength are in metres. (i) Calculate the phase velocity of the wave, (ii)
also calculate x, if the phase difference between two points at a distance 0.32 m apart, along
the line of propagation is /x (iii) if the wave propagates through air (density = 1.3 kg/m3)
find the intensity of wave. (Assuming 2 = 10)
Solution :
2
(i) The progressive wave is represented by y 0.05 sin 64t x
3.2
2
Comparing this with the standard equation y A sin vt x
the phase velocity (wave velocity) = 64 m/s
(ii) The phase difference of the particles separated by a distance of is equal to 2.
2
phase difference of particles separated by a distance 0.32 m = 0.32 = radians
3.2 5
x= 5
(iii) The intensity of the sound wave is given by
I
1
2
1
2
kg m
m s
v2 A 2 1.3 3 64 4 2 n2 0.05m
2
64
Here n is the frequency of the wave and is equal to 20 Hz
3.2
1
I = 1.3 64 4 2 400 0.0025 W / m2 = 1664 W/m2
2
WAVES & SOUNDS
QUIZRR 15
The physical quantities that determine the velocity are tension in string (T), mass per unit length
(ø).
Consider a transverse pulse produced in a taut string of linear mass density ø. Consider a small
segment of the pulse, of length l, forming an arc of a circle of radius R. A force equal in
magnitude to the tension T pulls tangentially on this segment at each end.
Let us set an observer at the centre of the pulse, which moves along with in the pulse towards
right. For the observer any small length dl of the string as shown will appear to move backward
with a velocity v.
Now the small mass of the string is in a circular path of radius R moving with speed v. Therefore
the required centripetal force is provided by the only force acting (neglecting gravity) is the
component of tension along the radius.
The net restoring force on the element is
Tl
F = 2T sin () 2T () =
R
v2
The acceleration of this element towards the centre of the circle is a , where v is the velocity
R
of the pulse.
T l v2
Using second law of motion l l
R R
T 2 T
T
or v
Example 8
A wire of uniform cross-section is stretched between two points 1 m apart. The wire is fixed
at one end and a weight of 9 kg is hung over a pulley at the other end produces fundamental
frequency of 750 Hz. (a) What is the velocity of transverse waves propagating in the wire?
(b) If now the suspended weight is submerged in a liquid of density (5/9) that of the weight,
what will be the velocity and frequency of the waves propagating along the wire?
T vA TA TB M
v so , i.e. vB = 1500
m vB TB TA
5
or vB 1500 1 1000 m/s
9
vB 1000
fB 500 Hz
B 2
Note : Here = constant; so f and v will change according to the relation v f with v T.
Example 9
A wire of mass 9.8 ï 10ă3 kg per metre passes over a frictionless pulley fixed on the top of
an inclined frictionless plane which makes an angle of 30Ĉ with the horizontal. Masses M1
and M2 are tied at the two ends of the wire. The mass M1 rests on the plane and the mass
M2 hangs vertically downwards. The whole system is in equilibrium. Now a transverse
wave propagates along the wire with a velocity of 100 m/s. Find the value of masses M1 and
M2. (g = 9.8 m/s2)
Solution :
For equilibrium of M1 along and perpendicular to the plane we have respectively :
M1g sin = T and M1g cos = R ...(1)
And for equilibrium of M2,
M2g = T ...(2)
Example 10
A uniform rope of mass 0.1 kg and length 2.45 m hangs from a ceiling. (a) Find the speed
of transverse wave in the rope at a point 0.5 m distant from the lower end, (b) Calculate the
time taken by a transverse wave to travel the full length of the rope (g = 9.8 m/s2)
Solution :
(a) As the string has mass and it is suspended vertically, tension in it will be different at
different points. For a point at a distance x from the free end, tension will be due to the
weight of the string below it. So if M is the mass of string of length L, the mass of length
x of the string will be (M/L)x.
M
T = x g
L
L
T Mgx
So v= gx ...(1)
m LM / L Pulse
x
Here x = 0.5 m
(b) From part (a) it is clear that the tension and so the velocity of the wave is different at
different points. So if at point x the wave travels a distance dx in time dt,
dx dx
v= or gx [From Eqn. 1]
dt dt
L
dx
1
g
1 / 2
or dt t x dx
gx , i.e.,
0
Principle of Superposition
Two or more waves can travel simultaneously in a medium without affecting the motion of one
another. Therefore, the resultant displacement of each particle of the medium at any instant is
equal to the vector sum of the displacements produced by the two waves separately. This principle
is called Âprinciple of superpositionÊ. It holds for all types of waves, provided the waves are not of
very large amplitude. We can express the superposition principle in the form
y (x, t) = y1 (x, t) + y2 (x, t) + ... + yn (x, t)
or y( x, t) y x, t
J 1
j
Here, the yj are the individual wave functions, and their sum, the wave function y(x, t) describes
the resultant behaviour of the medium as a function of position and time.
Interference :
Consider the superposition of two sinusoidal waves of same frequency at a point. Let us assume
that the two waves are travelling in the same direction with same velocity. The equation of the
two waves reaching at a point can be written as,
y1 = A1 sin (kx ă t)
and y2 = A2 sin (kx ă t + )
The resultant displacement of the point where the waves meet is
y = y1 + y2
= A1 sin ( ă t) + A2 sin (kx ă t + )
= A1 sin (kx ă t) + A2 sin (kx ă t) cos + A2 cos (kx ă t) sin
= (A1 + A2 cos ) sin (kx ă t) + A2 sin cos (kx ă t)
= A cos sin (kx ă t) + A sin cos (kx ă t)
or y = A sin (kx ă t + )
Here, A1 + A2 cos = A cos
and A2 sin = A sin
or A2 = (A1 + A2 cos )2 + (A2 sin )2
A sin A 2 sin
and tan =
A cos A1 A 2 cos
Another vector A 2 of length A2, making an angle with A1 represents
the amplitude of second wave. The resultant of A1 and A 2 represent
the amplitude of resulting function y. The angle represents the A1
phase difference between the resulting function and the first wave.
1
I = A 2 2 v
2
i.e. I A2
So, if , and are same for both interfering waves, Eq. (i) can also be written as,
we see that the resulting amplitude A and intensity I depends on the phase difference between
the interfering wave. Where cos = + 1, A = Amax = A1 + A2
2
or I = Imax = I1 I2
2
or I = Imin = I1 I2
Note :
(1) All maxima are equally spaced (as path difference between two consecutive maxima is )
2
and equally loud I m ax I1 I2 . Same is also true for minima with
2
I m in I1 ~ I2 . Also interference maxima and minima are alternate as for maxima
x = 0, , 2 etc., while for minima x = (/2), (3/2), etc. This all is shown graphically.
2
Imax = ( I1 + I2 )
2 I1 I2
I1 + I2
2 I1 I2
2
Imin = ( I1 ă I2 )
Phase Diff. 0 2 3 4 5 6
Phath Diff. 0 2 32 2 52 3 x
2
(2)
I max
I1 I2 A 1 A 2 2 with
I1 A12
I2 A1 ~ A 2 2 I2 A 22
Imin 2
I1 ~
So if I1 and I2 or A1 and A2 are given (Imax/Imin) can be calculated and vice-versa. From
the above it is also clear than if I1 = I2 = I0.
2 2
I max I0 I0 4I0 and I m in I 0 I0 O
i.e., in maxima intensity will be 4-times that of a single wave (I0) while intensity of minima
is zero if the interfering waves are of equal intensities.
2
(3) In interference the intensity of maxima I1 I2 exceeds the sum of individual intensities
2
(I1 + I2) by an amount 2 I1I 2 while of minima I1 ~ I2 lacks (I1 + I2) by the same
amount 2 I1I 2
Hence, we conclude that in interference energy is neither created nor destroyed but is
redistributed.
(4) Here we had assumed that the two waves from S1 and S2 start in the same phase. Hence,
at P they have a constant phase difference = (2/)x, developed due to different paths
traversed by them. Such waves are said to be ÂCoherentÊ and produce sustained interference
effects. However, if there is an initial phase difference between the waves 0 then =
0 + (2/)x and if 0 is not constant and varies rapidly and randomly with time, at P
sometimes constructive and sometimes destructive interference will take place so that
1
Iav = (I + Imin) = (I1 + I2) and hence, no interference effect is observed. Such waves
2 max
are called ÂIncoherentÊ.
So for observing interference effects waves must be coherent.
The nature of the reflected & transmitted wave depends on the nature of end point. There are
three possibilities.
(a) End point is fixed : Waves on reflection from a fixed end undergoes a phase change of
180Ĉ .
Reflected
wave
incident
wave
incident Reflected
wave wave
incident wave
(a) (b)
On the other hand, if the wave is produced on the heavier string, which moves towards the
junction, a part will be reflected and a transmitted, no inversion of wave shape will take
place (as shown in figure (b).
So the rule is : if a wave enters a region where the wave velocity is smaller, the reflected
wave is inverted. If it enters a region where the wave velocity is larger, the reflected wave
is not inverted. The transmitted wave is never inverted.
Example 11
Finding the amplitude of reflected and transmitted displacement waves from a plane
boundary at normal incidence, discuss the change in phase of reflected and transmitted
waves if any.
A i A A t
cos t r cos t cos t
v1 v1 v2
v2 v1 2v2
Ar and At Ai
v1 v2 v1 v2
These are the required results from these it is clear that in case of displacement waves :
(1) As At is always positive whatever be v1 and v2, the phase of transmitted wave always
remains unchanged.
(2) As Ar will be positive only if v2 > v1, i.e., in case of reflection from a rare medium (or free
end) there is no change in phase.
(3) As Ar will be negative if v2 < v1, i.e., in case of reflection from a denser medium (or rigid
boundary or fixed end) there is a phase change of .
Example 12
Two strings 1 and 2 are taut between two fixed supports (as shown in figure) such that the
tension in both strings is same. Mass per unit length of 2 is more than that of 1. Explain
which string is denser for 1 transverse travelling wave.
Solution :
Speed of a transverse wave on a string
T 1
v or v
1 2
Now ø2 > ø1 (given)
v2 < v1
i.e. medium 2 is denser and medium 1 is rarer.
Example 13
Figure shows a rectangular pulse and triangular pulse approaching each other. The pulse
speed is 0.5 cm/s. Sketch the resultant pulse at t = 2 s.
2 cm
ă2 ă1 0 1 2 3
x (cm)
Solution :
In 2 s each pulse will travel a distance of 1 cm.
The two pulses overlap between 0 and 1 cm as shown in figure. So, A1 and A2 can be added as
shown in figure (c).
(a) A1 2 cm
ă1 0 1
+ (c) 2 cm
A1
(b)
2 cm A2 2 cm
A2
0 1 2 ă1 0 1 2
STANDING WAVES
A standing wave is formed when two identical waves travelling in the opposite directions along
the same line interfere.
Consider two waves of the same frequency, speed and amplitude, which are travelling in opposite
directions along a string. Two such waves may be represented by the equations.
y1 = A sin (t ă kx)
y2 = A sin (kx + t)
Hence the resultant may be written as y = y1 + y2 = A sin (t ă kx) + A sin (t + kx)
y = 2A sin kx cos t
This is the equation of a standing wave.
This is the required result and from this it is clear that :
d2 y 1 d2 y
(1) As this equation satisfies the wave equation, it represents a wave. However,
dx2 v2 dt2
as it is not of the form F (ax bt), the wave is not travelling and so is called standing or
stationary wave.
(2) The amplitude of the wave As = 2A cos kx is not constant but varies periodically with
position (and not with time as in beats).
(3) The points for which amplitude is minimum are called nodes and for these
3 5
cos kx = 0, i.e., kx , , ,...
2 2 2
3 5 2
i.e., x , , , ... as k
4 4 4
i.e., in a stationary wave nodes are equally spaced, and the spacing between two adjacent
nodes is (/2) with Amin = 0. Also for nodes, displacement y = 0 for all values of time
(as As = 0), i.e., nodes are permanently at rest (through they are not physically clamped).
(4) The points for which amplitude is maximum are called antinodes and for these,
cos kx 1, i.e., kx = 0, , 2, 3, ...
2 3 2
i.e, x 0, , , , ... as k
2 2 2
i.e., like nodes, antinodes are also equally spaced with spacing (/2) and Amax = 2A.
Furthermore, nodes and antinodes are alternate with spacing (/4).
(5) The nodes divide in the medium into segments (or loops). All the particles in a segment
vibrate in same phase, but in opposite phase with the particles in the adjacent segment.
Twice in one period all the particles pass through their mean position simultaneously with
maximum velocity (As), the direction of motion being reversed after each cycle.
N N N
t=0
y
t= T4
t= T2
y
t= 3T
4
t=T
t= T4
(6) Since antinodes have always maximum displacement, their velocity is also maximum compared
to other points and velocity at nodes is zero.
(7) Standing waves can be transverse or longitudinal, e.g., in strings (under tension) if reflected
wave exists, the waves are transverse-stationary, while in organ pipes waves are longitudinal-
stationary. In case of of longitudinal waves are pressure and displacement waves have a
phase difference of (/2) at nodes where displacement is min pressure will be max while at
2A Displacement wave
y AN AN
x
N N N
p
x
Pressure wave
antinodes where displacement is max pressure will be min, i.e. in case of longitudinal-
stationary waves, nodes are points of max pressure (min displacement) while antinodes of
minimum pressure (max displacement).
(8) As in stationary waves nodes are permanently at rest, so no energy can be transmitted
across them. However, this energy oscillates between elastic potential energy and kinetic
energy of the particles of the medium. When all the particles are at their extreme position
KE is minimum while elastic PE is max and when all the particles (simultaneously) pass
through their mean position KE will be maximum while elastic PE minimum. The total
energy confined in a segment (elastic PE + KE), always remains the same.
(9) In standing wave if the amplitudes of component waves are not equal, then as Amin 0 i.e.,
node will not be permanently at rest and so some energy will pass across the node and the
wave will be partially standing.
In such situations we estimate the extent to which the resultant wave is standing by the
term standing wave ratio defined as
A max A1 A 2
SWR =
A min A1 ~ A 2
So that for a progressive wave SWR = (min) = 1 (as A2 = 0) while for perfectly standing wave
SWR = (max) = (as A1 = A2). The value of SWR for all other waves will lie between these
limits (i.e., 1 and ).
(1) In a travelling wave, the disturbance produced in a region propagates with a definite
velocity but in a standing wave, it is confined to the region where it is produced.
(2) In a travelling wave, the motion of all the particles is similar in nature. In a standing wave,
different particles move with different amplitudes.
(3) In a standing wave, the particles at nodes always remain at rest. In travelling waves, there
is no particle, which always remains in rest.
(4) In a standing wave, all the particles cross their mean position together. In a travelling wave,
there is no instant when all the particles are at the mean position together.
(5) In a standing wave, all the particles between two successive nodes reach their extreme
positions together, thus moving in phase. In a travelling wave, the phases of nearby particles
are always different.
(6) In a travelling wave, energy is transmitted from one region of space to other but in a
standing wave, the energy of one region is always confined in that region.
Example 14
The vibrations of a string of length 60 cm fixed at both ends are represented by the equation
x
y 4 sin cos 96 t where x and y are in cm and t in sec. (a) What is the maximum
15
displacement at x = 5 cm ? (b) Where are the nodes located along the string ? (c) What is
the velocity of the particle at x = 7.5 cm and t = 0.25 s ? (d) Write down the equations of
component waves whose superposition gives the above wave.
Solution :
(a) For x = 5, y = 4 sin (5/15) cos (96t)
or y 2 3 cos 96t
x x
i.e. 4 sin 0 or 0 , , 2, 3, ...
15 15
dy x
4 sin sin 96t 96
dt 15
WAVES & SOUNDS
28 QUIZRR
So the velocity of the particle at x = 7.5 cm and t = 0.25 s,
vpa = ă 384 sin (7.5/15) sin (96 ï 0.25)
i.e., vpa = ă 384 ï 1 ï 0 = 0
(d) As 2 sin A cos B = sin (A + B) + sin (A ă B)
x
So y = 4 sin cos 96t
15
x x
= 2 sin 96t sin 96t
15 15
x x
y = 2 sin 96t 2 96t 15 [as sin (ă ) = ă sin ]
15
or
x
y = y1 + y2 with y1 = 2 sin 96t
15
i.e.,
Example 15
A metallic rod of length 1 m is rigidly clamped at its mid point. Longitudinal stationary
waves are set up in the rod in such a way that there are two nodes on either side of the
mid-point. The amplitude of an antinode is 2 ï 10ă6 m. Write the equation of motion at a
point 2 cm from the mid-point and those of constituent waves in the rod. (Y = 2 ï 1011 N/
m2 and = 8 ï 103 kg/m3)
Solution :
In rods, like strings, clamped point is a node while the free antinode; so the situation in accordance
with given condition is as shown in Fig.
x
x=0 x=L/2 x=L
Now as distance between two consecutive nodes is /2 while between a node and an antinode is
/4
21
4 2 L i.e., 0.4 m... ...(1)
2
4 5
Y 2 1011
v= 5000 m/s
8 103
C + D cos C ~ D
But as sin C + sin D = 2 sin
2 2
y 2A cos kx sin t
2 2
Now as free end of the rod is an antinode, i.e., amplitude is max at x = 0, so that
cos k 0 max 1, i.e. = 0
2
2 2
or y = 2 ï 10ă6 cos sin 2ft as k and 2f
the resultant wave y = 2 ï 10ă6 cos (5x) sin (25000t) can be written as
i.e. y = 10ă6 sin [25000t + 5x] + 10ă6 sin [25000t ă 5x] [as sin (ă ) = ă sin ]
When a string capable of vibrating (under tension) is set into vibration, transverse harmonic
waves will propagate along it. It gets reflected at the other fixed end. The incident and the
(a) Fundamental Mode : In the simplest form, the string vibrates in one loop in which the
ends are the nodes and the centre is the antinode. This mode of vibration is known as the
fundamental mode and frequency of vibration is known as the fundamental frequency or
first harmonic.
Since the distance between consecutive nodes is
1
L 1 = 2L
2 2
v
or f1 ...(i)
2L
(b) The same string under the same conditions may also vibrate in two loops, such that the
centre is also the node
L = 22/2 2 = L
If f2 is frequency of vibrations
v v 2 2
f2 =
2 L
v
f2 = ...(ii)
L
33
L=
2
2
3 = L
3
3v
f3 = ...(iii)
2L
T
The velocity of transverse wave in stretched string is given as v . Where T = tension
in the string.
ø = linear density or mass per unit length of string. If the string fixed at two ends, vibrates
in its fundamental mode, then v = 2Lf
1 T
f
2L
Now as in case of waves v = f, i.e., f = v/ and for waves along a string v T / m , the possible
n T
fn with n = 1, 2, 3, ...
2L m
i.e., in case of vibrations of strings, number of natural frequencies are possible and if we take
1 T
f , f = nf
2L m n
So in case of vibrations of strings higher frequencies are integral multiples of f, i.e., forms a
harmonic series.
1 T
fmin f1 f
2L m
(1) As a string has many natural frequencies (all integral multiples of fundamental frequency),
so when it is excited with a tuning fork (or a vibrating body), the string will be in resonance
with the given body if any one of its natural frequencies coincides with that of the body.
(c) f 1/ m if T and L are constant
These laws of vibration of string are known as Mersenne’s laws of vibration of string and
according to these the frequency of a string can be changed by changing its length, tension
or mass per unit length.
1 T 1 T 1 T
So f ...(4)
2L m 2L M/L 2 ML
1 T 1 T
So f i.e., f ...(5)
2L 2
r 2Lr
(4) If the string is vibrating in nth harmonic, its frequency will be nf, the number of loops in
the string or antinodes will be n, while total number of nodes (including two at the ends)
will be (n + 1), e.g., in case of 3rd harmonic of a vibrating string, frequency of vibration will
be 3f, antinodes will be 3 while total nodes = 3 + 1 = 4 and = (2L/3).
(5) In case of vibrations of composite string (i.e., string made up by joining two strings of
different lengths, cross-sections and densities) having same tension through out, the joint is
a node while lowest common fundamental frequency of the string will be
fC = n1f1 = n2f2
Here higher harmonics will be integral multiples of common frequency fC.
Example 16
1 T
f
2L m
f 6 1.44T
i.e. f = 30 Hz
f T
Now it keeping the original tension (T), the length of given wire is changed.
f ´´ l 1
= [as l´´ = l + 0.20 l = 1.20 l]
f l´´ 1.20
30
so f´´ = 25 Hz
1.2
Example 17
Two strings A (length L1) and B (length L2) are made of steel and are kept under the same
tension. If A has a radius twice that of B, what should be value of L2/L1 for them to have
the same fundamental frequencies? What should be the value of L2/L1 if the first overtone
of the former should equal the third harmonic of the latter?
2
r
Linear mass density r 2 = density of steel
2
1 T
For the string A, f
2L1 r 2
1 T 1 4T
For the string B, f
2
2L2 r 2L 2 r 2
2
1 T 1 4T
2
2L1 r 2L 2 r 2
L2
2
L1
1 T
f
2L1 r 2
1 4T
f´
2L 2 r 2
3 4T
3f´
2L2 r 2
Since 2f = 3f
1 T 3 4T 3 T
L1 2
r 2L 2 2
r L2 r 2
L2
3
L1
Example 18
Two metallic strings A and B of different materials are connected in series forming a joint.
The strings have similar cross-sectional area. The length of A is l A = 0.3 m and that of B is
l B = 0.75 m. One end of the combined string is tied with a support rigidly and the other end
is loaded with a block of mass m passing over a frictionless pulley. Transverse waves are
set up in the combined string using an external source of variable frequency. Calculate
(i) the lowest frequency for which standing waves are observed such that the joint is a node
and (ii) the total number of antinodes at this frequency. The densities of A and B are
6.3 ï 10ă3 kg mă3 and 2.8 ï 10ă3 kg mă3 respectively.
Solution :
p T
The frequency of transverse waves in a stretched string is given by n
2l m
where the string is vibrating with p loops, T = tension in the string and m = mass per unit length
of the string.
As the frequency of the wave in both string (A vibrating with p loops B vibrating with q loops)
must be the same, so
A C B
p T q T
= 0.3 m 0.75 m
2lA mA 2lB mB
p lA mA l A
= A
q lB mB lB B
0.3 6.3 3
=
0.75 2.8 5
So, p = 3, q = 5
No. of antinode = p + q = 3 + 5 = 8 Ans.
Note : (i) The frequency cannot be calculated as tension is not provided.
(ii) Total no. of nodes including the two at the ends will be = 9
An aluminium wire of cross-sectional area 1 ï 10ă6 m2 is joined to a copper wire of the same
cross-section. This compound wire is stretched on a sonometer, pulled by a weight of 10 kg.
The total length of the compound wire between the two bridges is 1.5 m of which the
aluminium wire is 0.6 m and the rest is the copper wire. Transverse vibrations are set up
in the wire by using an external force of variable frequency. Find the lowest frequency of
excitation for which standing waves are formed, such that the joint in the wire is a node.
What is the total number of nodes observed at this frequency excluding the two at the ends
of the wire ? The density of aluminium is 2.6 ï 103 kg/m3 and that of copper 1.0401 ï 104 kg/
m3.
Solution :
As the total length of the wire is 1.5 m and out of which LA = 0.6 m, so the length of the copper
wire LC = 1.5 ă 0.6 = 0.9 m. The tension in the whole wire is same (= Mg = 10 g N) and as
fundamental frequency of vibration of string is given by
1 T 1 T
f [as m = A]
2L m 2L A
1 T 1 T
So fA and fC ...(1)
2L A A A 2LC C A
Now as in case of composite wire, the whole wire will vibrate with fundamental frequency
f = nA f A = nC f C ...(2)
Substituting the values of fA and fC from Eqn. (1) in (2),
nA T nC T
2 0.6 3
A 2.6 10 2 0.9 A 1.0401 104
nA 2 2.6 2 1 1
i.e.,
nC 3 10.4 3 2 3
10 Kg
This is turn implies that total number of nodes in the string will be 5 and so number of nodes
excluding the nodes at the ends = 5 ă 2 = 3,
1 10 9.8
and f fA 161.8 Hz 3 fC
2 0.6 6
10 2.6 103
Example 20
A string 120 cm in length sustains a standing wave, with the points of string at which the
displacement amplitude is equal to 2 mm being separated by 15.0 cm. Find the maximum
displacement amplitude. Also find the harmonic corresponding to this wave.
Solution :
From Fig. points A, B, C, D and E are having equal displacement amplitude.
Further, xE ă xA = = 4 ï 15 = 60 cm
2l 2 120 A B E
As = = 60
n n
C D
2 120
n = 4
60
2 2
Here a = 2 mm; k and x = 7.5 cm
60
2
2 = A sin 60 7.5 A sin 4
Hence, A = 2mm
S OUND
Sound waves are mechanical waves. They require a medium for their propagation i.e. they cannot
propagate in vaccum.
Sound is produced in a material by a vibrating source. Sound waves constitute alternate compression
and rarefaction pulses travelling in the medium. The compression travels in the medium at a
speed, which depends on the elastic and inertial properties of the medium.
The description in terms of pressure wave is more appropriate than the description in terms of the
displacement wave as far as sound properties are concerned.
x
y y0 sin t ...(i)
v
y y+ y
A is cross-sectional area.
Increase in volume of this element at time t is A
V = A dy
x x+x
x
= Ay0 cos t x
v v
x
Ay0 cos t x
volume strain is V v y x y
0 cos t
V vAx v v x
y
volume strain =
x
The corresponding stress i.e., the excess pressure developed in the element at x, at time t is
V
p = B where B is the bulk modulus of the material.
V
By0 x
p= cos t ...(ii)
v v
By
P=
x
Comparing equations (i) and (ii), the relation between the pressure amplitude P0 and the
displacement amplitude s0 is
B p
p0 y0 Bky0 y0 0 where k is a wave number.
v 2B
As observed from equations (i) and (ii), pressure wave is Âcos Ê type, if displacement is described
as Âsin Ê type.
Thus, the pressure-maxima occur where where the displacement is zero and displacement maxima
occur where the pressure is at its normal level.
First we calculate the speed at which a longitudinal pulse propagates through a fluid. We will
apply NewtonÊs second law to the motion of an element of the fluid and from this we derive the
wave equation.
x y y+y
P0 P0 P0+P1 P0+P2
a b
Consider a fluid element ÂabÊ confined to a tube of cross sectional area S as shown in figure. The
element has a thickness x. We assume that the equilibrium pressure of the fluid is P0. Because
of the disturbance, the section ÂaÊ of the element moves a distance y from its mean position and
section ÂbÊ moves a distance y + y to a new position b´. The pressure on the left side of the element
becomes P0 + P1 and on the right side it becomes P0 + P2. If is the equilibrium density, the
mass of the element is Sx. (When the element moves its mass does not change, even though
its volume and density do change).
The net force acting on the element is,
F = (P1 ă P2) S
2 y
and its acceleration is a=
t2
2 y
(P1 ă P2) S = Sx
t2
Next we divide both sides by x and note that in the limit as x 0 we have (P1 ă P2) / x
P 2 y
P/x, Eq. (i) then takes the form 2 ...(ii)
x t
y
The excess pressure P may be written as P B
x
2 y 2 y 2 y B 2 y
. 2 or
x2 B t t2 x2
2 y 2 2 y
=
t2 x2
B
We have = (speed of longitudinal wave in a fluid)
Y
= (speed of a longitudinal wave in a solid rod)
The motion of sound wave in air is adiabatic. In the case of an ideal gas, the relation between
pressure P and volume V during an adiabatic process is given by
PV = constant
Where is the ratio of the heat capacity at constant pressure to that at constant volume.
After differentiating, we get
dP
V PV 1 0
dV
WAVES & SOUNDS
QUIZRR 41
vd P
Since B P
dV
P RT
using the gas equation M where M is the molar mass.
RT
Thus v (T = temperature is Kelvin).
M
Note :
(i) Effect of temperature : If the specific volume of gas is v. The velocity of sound =
P RT
M
If c1 and c2 be the velocities of sound in a gas at temperatures t1 C and t2 C and P1 and P2
the respective pressures and V1 and V2 the specific volumes at these temperatures, ratio of
the two velocities of sound is
v1 P1 V1 RT1
where T and T2 are the absolute temperatures.
v2 P2 V2 RT2
Hence, v T
v1 273 t1
v2 273 t2
vt 273 t
If vt and v0 are the velocities at tĈC and 0ĈC, then
vo 273
ó
t
vt v0 1
273
when t is small
vt t
1
v0 546
t
vt v0 1
546
P RT
(ii) Effect of pressure : In a gas; v change in pressure has no effect on velocity
M
P
of sound in a gas, so long as temperature is constant because; constant; as long as
temperature is constant.
(iii) Effect of relative humidity : When humidity increases, there is an increase in the relative
number of water molecules and hence a decrease in molar mass, and the speed of sound
increases.
(iv) The speed of sound in air is not affected by amplitudes, frequency, phase, boundness, pitch
of quality.
2
1 dy 1
= A 2 2 cos2 (t ă kx)
2 dt 2
and its maximum value will be equal to energy per unit volume [as (KE)max = (PE)max = E], i.e.,
energy density U.So.
1
U A 2 2 Sx ...(1)
2
1
So the energy associated with a volume S x will be E U V A 2 2 S x
2
So, power (rate of transmission of energy) will be
E 1 x
P 2 A 2S as t V ...(3)
t 2
Now as Intensity is defined as average energy transmitted per unit normal area per sec., i.e.,
power per unit area, so
E P 1 2 2
I = S t S 2 v A ...(4)
Further as in case of sound wave displacement amplitude is related to pressure amplitude through
t he r elat ion p0 = vA, so
2
1 p0 1 p02
I = v2 v ...(5)
2 2 v
Eqns. (4) and (5) give intensity of sound in terms of displacement and pressure amplitude
respectively and according to these for a given source and medium
Note : In case of vibrating string, as S will represent mass per unit length m, so from Eqn. (3)
the average rate of transport of energy, i.e., power transmitted by a vibrating string will be
1 mass
P mv2 A 2 with m ...(7)
2 length
The SI unit of intensity is W/m2. However; as human ear responds to sound intensities over a
wide range, i.e., from 10ă12 W/m2 to 1 W/m2, so instead of specifying intensity of sound in W/m2,
we use a logarithmic scale of intensity called the sound level defined as
I
SL = 10 log ...(8)
I0
where I0 is the threshold of human ear, i.e., 10ă12 W/m2. The sound level defined in this way is
expressed in decibel (dB). A sound of intensity I0 has an SL = 10 log (I0/I0) = 0dB while sound
at the upper range of human hearing called threshold of pain has a intensity of 1 W/m2 or a SL
= 10 log (1/10ă12) = 120 dB.
We also use dB as a relative measure to compare different sounds with one another, rather than
with reference intensity; as for two intensities I1 and I2.
I1 I
SL1 SL2 10 log 10 log 2
I0 I0
I1
or SL1 SL2 10 log ...(9)
I2
e.g., two sounds whose intensity ratio is 2 differ in SL by 10 log 2 = 3 dB. Here it must be kept
in mind that ratio of two intensities corresponds to difference in their sound level (and not ratio).
Note : While solving problems related to intensity of sound along with the above, also remember
that :
E E
(i) As intensity, I while U =
S t V
I E S L L
So v as V S L and v
U S t E t
P P
I as S = 4r 2 ...(11)
S 4 r 2
1 1
I EB with E c and c
0 B 0 0
Example 21
(a) The power of sound from the speaker of a radio is 20 mW. By turning the knob of
volume control the power of sound is increased to 400 mW. What is the power increase
in dB as compared to original power ?
(b) How much more intense is an 80 dB sound than a 20 dB whisper ?
Solution :
(a) As intensity is power per unit area, for a given source P I, so
SL2 ă SL1 = 10 log (I2/I1)
P2 400
i.e. SL = 10 log P 10 log 20
1
Example 22
An observer is at a distance of one metre from a point of light source whose power output
is 1 kW. Calculate the magnitude of electric and magnetic fields assuming that the source
is monochromatic, it radiates uniformly in all directions and that at the point of observation
it behaves like a travelling plane wave. Given that (ø0/4) = 10ă7 H/m and c = 3 ï 108 m/s.
Solution :
By definition of intensity,
P P 103 103
I= W/m 2
S 4r 2
4 12 4
1 E
I = EB and =c
0 B
1 E
so I = E c, i.e., E = I 0 c
0
E=
[103 / 4 ] 4 10 7 3 108 100 3 173 V / m
E 100 3 1
and B = c 8
10 6 5.77 107 Web / m2
3 10 3
Note :
5.77 10 7
(i) As B H , H 0.46 A / m
4 107
I
The unit of loudness is decibels (dB) and L 10 log10 (in dB) Here, I0 is constant i.e.,
I0
quarter wavelengths. It should be noted that the open end is always an antinode and the closed
end a node. According to this condition there arises a number of standing waves as shown in
figure. The wave pattern, which has the lowest frequency, is called fundamental and the others
are called overtones.
A A A
N
N
A N
N N N
Fundamental First overtone Second overtone
The length of air column L is equal to
4
= 4L
v = f
v v
f =
4L
31
L=
4
4L
1 =
3
frequency f1 v 3v 3 f
1 4L
Here L = 5 2
4
4L
5
v 5v
frequency f2 5f
2 4L
When an air column is excited the fundamental and a number of possible overtones are present
in the vibration. Of these the loudest is the fundamental and overtones progressively becomes
weaker in intensity. The overtones whose frequencies are integral multiples of fundamental are
called harmonics. The fundamental with frequency f itself is taken as first harmonic. The overtone
with frequency 2f is called second harmonic and the overtone with frequency 3f is called third
harmonic and so on.
In the case of closed type indicated above all odd harmonics are present and even harmonics are
absent.
End correction : In the above discussion it is assumed that the position of antinode coincides
with the opened of pipe exactly. This is not however true and it is found that antinode is a little
bit displaced above the open end. If e is the end correction, then for fundamental mode.
1
L e
4
3 2
For the first overtone L e and so on.
4
The end correction depends upon the diameter of the pipe. If d is the diameter, the end correction
e = 0.3 d.
Open Pipes
(a) First mode of vibration (Fundamental mode) : In the fundamental mode there is a node
between antinodes at each end.
L= or = 2L
2
v v
f
2L
(b) Second mode of vibration (First overtone) : If 1 and f1 are the wavelength and frequency
of the first overtone in open pipe.
1 = L
v v 2v
f1 2f
1 L 2L
3 2
L=
2
2L
2 =
3
v 3v
f2 = 3f
2 2L
A body capable of vibration, if excited, and set free, vibrates freely in its own natural way. The
frequency of such free vibration depends on the mass, elastic property and dimensions of the
body. The frequency is called free frequency or natural frequency of the body.
Damped Vibrations
The amplitude of free vibrations of a body gradually diminishes and finally the vibrations die
away after sometime. This is due to the vibratory motion being damped by forces internal and
external to the body.
If an external periodic force is applied to a body which is capable of vibration and if the frequency
of the applied periodic force is not the same as the frequency of the body, the body begins to
vibrate initially with its own natural frequency but these vibrations die down quickly and the
body ultimately vibrates with the frequency of the external periodic force. Such vibrations are
called forced vibrations.
Example
Example 23
A tuning fork having frequency of 340 Hz is vibrated just above a cylindrical tube. The
height of the tube is 120 cm. Water is slowly poured in. What is the minimum height of
water required for resonance ? (v = 340 m/s)
Solution :
As the tuning fork is in increase with air column in the pipe closed at one end.
v
f n with n = 1, 3, 5, ...
4L
nv 340 100
So length of air column in the pipe L n 25n cm with n = 1, 3, 5, ...
4f 4 340
Example 24
AB is a cylinder of length 1 m fitted with a thin flexible diaphragm C at middle and two
other thin flexible diaphragms A and B at the ends. The positions AC and BC contain
hydrogen and oxygen gases respectively. The diaphragms A and B are set into vibrations
of the same frequency. What is the minimum frequency of these vibrations for which
diaphragm C is a node ? Under the conditions of the experiment the velocity of sound in
hydrogen is 1100 m/s and oxygen 300 m/s.
Solution :
As diaphragm C is a node, A and B will be antinodes (as in an organ pipe either both ends are
antinode or one end node and the other antinode), i.e., each part will behave as a closed end
organ pipe so that
vH 1100 A C B
fH = 4L = 4 0.5 = 550 Hz
H
H2 O2
v0 330
And fO = = = 150 Hz
4LO 4 0.5
As the two fundamental frequencies are different, the system will vibrate with a common
frequency fC such that
f C = nHfH = nOfO
nH fO 150 3
i.e., =
nO fH 550 11
i.e., the third harmonic of hydrogen and 11th harmonic of oxygen or 9th harmonic of hydrogen
and 33rd harmonic of oxygen will have same frequency. So the minimum common frequency
f = 3 ï 550 or 11 ï 150 = 1650 Hz
(as 6th harmonic of H and 22nd of O will not exist.)
Example 25
A ÂpopÊ gun consists of a tube 25 cm long closed at one end by a cork and at the other end
by a tightly fitted piston. The piston is pushed slowly in. When the pressure rises to one and
half times the atmospheric pressure, the cork is violently blown out. Calculate the frequency
of the ÂpopÊ caused by its ejection. (v = 340 m/s)
Solution :
Assuming the cross-section to be A and compression to be isothermal (as the process is slow), from
P1V1 = P2V2
P ï 25 ï A = (3/2) ï P ï L ï A, i.e., L = (50/3) cm
Now after the ejection of cork, for oscillating air node will be at
piston (rigid boundary) while antinode will be at the open end 25cm
and as minimum distance between node and antinode is (/4).
N A
50 2
so L cm, i.e., m
4 3 3 A A' L
v 340 3
and hence f 510 Hz
2
The water level in a vertical glass tube 1.0 m long can be adjusted to any position in the
tube. A tuning fork vibrating at 660 Hz is held just over the open top end of the tube. At
what position of the water level will there be resonance. Speed of sound is 330 m/s.
Solution :
Resonance corresponds to a pressure antinode at closed end and pressure node at open end.
Further, the distance between a pressure node and a pressure antinode is , the condition of
4
resonance would be,
v
length of air column l n n
4 4f
Here, n = 1, 3, 5, ...
330
l 1 = (1) 0.125 m
4 660
l 2 = 3l1 = 0.375 m
l 3 = 5l1 = 0.625 m
l 4 = 7l1 = 0.875 m
l 5 = 9l1 = 1.125 m
Since l5 > 1 m (the length of tube), the length of air columns can have the values from l1 to l4
only. Therefore, level of water at resonance will be
(1.0 ă 0.125) m = 0.875 m
(1.0 ă 0.375) m = 0.625 m
(1. 0 ă 0.625) m = 0.375 m
and (1.0 ă 0.875) m = 0.125 m Ans.
0.875m
0.625m 0.375m
0.125m
In all the four cases shown in figure, the resonance frequency is 660 Hz but first one is the
fundamental tone or first harmonic. Second is first overtone or third harmonic and so on.
R esonance :
Resonancee is a special case of forced vibration. If the frequency of the external periodic force is
the same as the natural frequency of the body, the body responds to the forced vibrations more
willingly and there is a gain in the amplitude of its vibrations. This is called resonance.
Resonance has vast application in acoustics, electrical circuits and electronics.
Suppose the length of air column in a long tube can be adjusted either by dipping the tube in
a reservoir of water or by allowing the water level to occupy a desired position in the tube by
pressure flow; the column can be made to vibrate in resonance with an excited tuning fork kept
over the mouth of the tube.
For two lengths of air column L1 and L2 ~ 3L1, the resonance would occur and the positions
corresponds to the fundamental mode and the first overtone respectively.
If be the wavelength of sound in air and v the velocity of sound in air, then
L1 e L1 A
4
N
3
L2
L2 e A
4
N
where e is the end correction
From the above equations we get
= L2 ă L1 or = 2 (L2 ă L1)
2
v = f = 2f (L2 ă L1)
where f is the frequency of vibration of the air column which is in resonance with the tuning fork
of same frequency.
Example 27
A tube of a certain diameter and length 48 cm is open at both ends. Its fundamental
frequency is found to be 320 Hz. The velocity of sound in air is 320 m/s. Estimate the value
of end correction in cm.
Solution :
Let the length of the open tube be L. The end correction on both sides is e. The tube vibrates in
its fundamental. Then
L 2 e or = 2 (L + 2e)
2
WAVES & SOUNDS
54 QUIZRR
If v be the velocity of sound in air the fundamental frequency is given by
v v
f = 2 L 2e
320
f = 320 Hz; v = 320 m/s or; 320
2 L 2e
or L + 2e = 0.5 m
2e = 0.5 m ă 0.48 m = 0.02 m
e = 0.01 m = 1 cm.
BEATS
When two sound waves of nearly equal (but not exactly equal) frequencies travel in same
direction, at a given point due to their superposition, intensity alternately increases and decreases
periodically. This periodic waxing and waning of sound at a given position is called beats.
1
T
f1 f2
Note : If the waves are in phase at some time (t = 0) will be constructive and the resultant
amplitude will be A1 + A2, where A1 and A2 are the amplitudes of indivisual sound waves.
But at some time (t = t0) because the frequencies are different, the waves will be out of phase or
the interference will be destructive and resultant amplitude will be A1 ă A2.
1 T
and T 2t0 A1 + A2
f1 f2
Alternative Method :
y1 = A sin (2 f1t), y2 = A sin 2 f2t
C+ D C D
Now as sin C + sin D = 2 sin cos
2 2
f1 f2 t f1 f2 t
y = 2 A cos 2 sin 2
2 2
f1 ~ f2 f1 f2
or y = 2A cos 2 fAt sin 2favt with fA and fav
2 2
or y = Ab sin 2 favt with Ab = 2A cos (2 fAt)
f1 f2
Thus, the resultant wave is a harmonic progressive wave of frequency fav i.e., and
2
amplitude Ab which is periodic in time.
Also, it can be seen that a beat, that is maximum and minimum intensity, will occur when
I Ab2 = max (or min)
or cos 2fAt = + 1 or zero
or 2fAt = 0, , 2 ... or /2, 3
/2, 5
/2
1 2 3 1 3 5
i.e. t = 0, 2f , 2f , 2f or 4 f , 4 f , 4 f
A A A A A A
1 1
t beat frequency = fb f1 ~ f2
2 fA t
y1 t = 1s
y2 t = 1s
A column of air and a tuning fork produce 4 beats per second when sounded together. The
tuning fork gives the lower note. The temperature of air is 15ĈC. When the temperature falls
to 10ĈC the two produces 3 beats per second. The frequency of fork is f, then find the value
of 5f.
Solution :
Let be the wavelength and n be the frequency of fork
v15 v
At 15ĈC, f 4 or 15 f 4
v10 v
At 10ĈC, f 3 or 10 f 3
v15 f 4
v10 f 3
ó
5 f 4 5 f 4
1 283 1
f 3 566 f 3
5 f 4 f 3 1
566 f 3 f 3
5f + 15 = 566
5f = 551
Example 29
Two radio stations broadcast their programmes at the same amplitude A, and at slightly
different frequencies 1 and 2 respectively, where 2 ă 1 = 103 Hz. A detector receives the
signals from the two stations simultaneously. It can only detect signals of intensity > 2A2.
(a) Find the time-interval between successive maxima of the intensity of the signal received
by the detector. (b) Find the time for which the detector remains idle in each cycle of the
intensity of the signal.
Solution :
If the detector is at x = 0, the two radio-waves at the site of detector in accordance with given
conditions (i.e., A1 = A2 = A and f1 = 1 and f2 = 2) will be
So by principle of superposition,
= y1 + y2 = A sin 21t + A sin 22t
C + D cos C D
But as sin C + sin D = 2 sin
2 2
2 1 t sin 2 1 2 t
y = 2A cos 2
2 2
1 2 t 2 1 t
or y = A´ sin 2 with A´ = 2A cos 2
2 2
1 1
T t2 t1 10 3 s
2 1 103
1 3 5
i.e., t , , ,...
4(2 1 ) 4(2 1 ) 4(2 1 )
O 1 3 t ï 10ă3
4 4
Example 30
There are three sources of sound of equal intensities with frequency 400, 401 and 402 Hz.
What is the beat frequency heard if all are sounded simultaneously ?
Solution :
As intensities are equal, amplitudes of waves will be equal and it for simplicity we consider the
waves at x = 0 with 401 = f, y1 = A sin 2(f ă 1)t; y2 = A sin 2ft and y3 = A sin 2(f + 1)t
So by principle of superposition,
y = A sin 2(f ă 1)t + A sin 2ft + A sin 2 (f + 1)t
Taking first and last terms together,
y = 2A cos 2t sin 2ft + A sin 2ft
or y = A[2 cos 2t + 1] sin 2ft
or y = A´ sin 2ft with A´ = A[1 + 2 cos 2t]
2 2 2
So I (A´) A (1 + 2 cos 2t) ...(1)
For I to be max or min,
dI d
0, i.e. 1 2 cos 2t2 0
dt dt
2
or 2 2n , ..., with n = 0, 1, 2 ....
3
1 2 4 5
i.e., t , , , , ... ...(2)
3 3 3 3
1 3
i.e., t 0, , 1, , 2, ... ...(3)
2 2
9A2
A2
O
1 1 2 1 4 3 5 2 t
3 2 3 3 2 3
DOPPLER EFFECT
When a sound source and an observer are in relative motion with respect to the medium in which
the waves propagate, the frequency of waves observed is different from the frequency of sound
emitted by the source. This phenomenon is called Doppler effect. This is due to the wave-nature
of sound propagation and is therefore applicable to light waves also. The apparent change of
colour of a star can be explained by this principle.
v vs v
´
f
Thus, apparent frequency
S O
velocity of sound relative to O
f´ = wavelength of wave reaching O S
văvs
v v vs
f´ = f
´ v vs văvs
v vs V
´
f
Apparent frequency
S O
f´ = v/´
v
or f´ = f
v vs vs S O
(iii) Observer, moves towards stationary source v+vs
v v0 v v0
f´ f
v/ f v
(iv) Observer moves away from the stationary source
v v0 v v0
f´ = f
v/ f v
v v0 v v0
f
f´ = v vs v vs
f
v v0
f´ = f
v vs
(vii) Source moves towards observer but observer moves away from source
v v0
f´ = f
v vs
(viii) Source moves away from observer but observer moves towards source
v v0
f´ = f
v vs
D iscussion
(1) There is always an increase in frequency or pitch if source moves towards detector or
detector moves towards source or both move towards each other while a decrease in frequency
if either or both move away. The change in frequency or pitch depends on speed of source
and detector and not on distance between them, e.g., if an engine is approaching a stationary
listener at constant velocity, increase in pitch, by Eqn. (2) will be same when the engine is
either at a distance of 1 km or 10 m from the listener. However, intensity will be different
in the two cases as I (1/r2).
(2) If the motion is along some other direction, the components of velocities along the line
joining source and detector are considered for vS and vD, e.g., if at any instant the line
joining the moving source and stationary detector makes an angle with the direction of
motion of source, vS vS cos Source
vs
vs
co
s
v Detector
and so f A f ...(3)
v vS cos
In such situations fA is not constant and depends on and may be greater, equal to or less
than f as < = or > 90Ĉ.
(3) If the medium is at rest relative to earth, vS, vD and v will refer to the speeds of source,
detector and sound relative to earth which is usually the case. However, if the medium (air)
itself starts moving with respect to given frame of reference (say earth), appropriate changes
must be made in Eqn. (2), i.e., if wind blows at a speed w from the source to the detector
v v + w and if in opposite direction (i.e., from detector to source) v v ă w.
(4) There will be no Doppler effect, i.e., no change in frequency.
(a) If source and detector both move in same direction with same speed, i.e., if vS = vP = u
v u
f´ f f
v u
v
(b) If one is at the centre of a circle while the other is moving on it with
v S
uniform speed. In this situation component of u along the ling of 90Ĉ
sight, i.e., radius, will be u cos 90Ĉ = 0; so
O
v 0
f´ f f
v 0
v w 0
f´ f f
v w 0
(5) Speed of detector or source becomes equal to or greater than that of sound, Doppler formula
(2) does not apply (as it was derived by assuming vD and vS < v). For example if vD > v and
the detector is moving away from the source, the sound will never reach it and if vS > v the
source gets ahead of the wave in its direction of motion.
vt
S´
S vs t
When the speed of source (vS) is greater than the speed of sound (v) is called supersonic
speed and the ratio (vS/v) Mach number. In this situation a conical wavefront of
high-energy pressure waves [with source at its apex and semicone angle = sină1 (v/vS)
= sină1 (1/Mach No.)] called Âshock-wavesÊ is continuously produced and when we intercept
it, a loud bang of sound called sonic boom is heard which can break windows and even
cause damage to buildings. Here it is worthy to note that shock-waves are produced not only
when source crosses the sound barrier (a misconception) but are generated continuously as
long as vS > v.
Example 31
A source of sound is moving along a circular orbit of radius 3 m with an angular velocity
of 10 rad/s. A sound detector located far away from the source is executing linear simple
harmonic motion along the line BD with amplitude BC = CD = 6 m. The frequency of
osci l l at i on of t h e det ect or i s (5/) per sec. The source is at the point A when the detector
is at the point B. If the source emits a continuous sound wave of frequency 340 Hz, find the
maximum and the minimum frequencies recorded by the detector [velocity of sound = 330
m/s].
Solution :
Time period of circular motion T = (2/) = (2/10) is same as that of SHM, i.e., T = (1/f) = (/5),
so both will complete one periodic motion in same time. Further more as source is moving on a
circle, its speed
vS = r = 3 ï 10 = 30 m/s
A
vD A 2 y2 10 62 y2
3m B C D
v vD
fA = f
v vS
v vD
fmax = f with vD = max
v vS
330 60
fmax = 340 330 30 442 Hz
Similarly fA will be minimum when both are moving away from each other, i.e.,
v vD
fmin = f with vD = max
v vS
330 60
fmin = 340 225 Hz
330 30
Example 32
vs c
os
AB 1
cos 0.857
AC 1.166 C
80
vS vS cos = 60 60 0.857 0.019 mile/s
v 0.2
So f A f 400 442 Hz
v vS 0.2 0.019
Example 33
A sonometer wire under tension of 64 N vibrating in its fundamental mode is in resonance
with a vibrating tuning fork. The vibrating portion of the sonometer wire has a length of
10 cm and mass 1 g. The vibrating tuning fork is now moved away from the vibrating wire
at a constant speed and an observer standing near the sonometer hears one beat per sec.
Calculate the speed with which the tuning fork is moved, if the speed of sound in air is
300 m/s.
Solution :
As the frequency of a vibrating string
1 T 1 T M
fS = as m L
2L m 2 ML
1 64
so fS = 2 3 1
400 Hz
10 10
Now, as initially sonometer wire is in resonance with tuning fork, the frequency of tuning fork.
f = fS = 400 Hz
When the tuning fork is moved away from the observer standing near the sonometer at a
constant speed u the apparent frequency of tuning fork will be
v
fR = f
v u
As fR is producing beats with f, fR is nearly equal to f, i.e., u << v so that
1
u u
fR = f 1 f 1
v v
u
So beat frequency f = f ă fR = f
v
f 1
and substituting given data, u v 300 0.75 m/s Ans.
f 400
Example 34
A band playing music at a frequency f is moving towards a wall at a speed vB. A motorist
is following the band with a speed vm . If v is the speed of sound, obtain an expression for
the beat frequency heard by the motorist.
Solution :
The situation is shown in Fig. As the motorist (observer) is following the band (source), he will
hear two frequencies one directly from the band while the other reflected from the wall.
Taking the direction of sound from source to observer to be positive, the frequency of
f1 f
vm v
M B vB
f´
f2
v vm v vm
f1 = f f ...(1)
v vB v vB
Now as the frequency of sound reaching the wall towards which the band (source) is moving,
v
f´ = f ...(2)
v vB
The frequency of reflected sound from the wall, heard by motorist (observer) who is moving
towards the wall (stationary source) will be
v vm v vm
f2 = f´ f [from Eqn. (2)]
v v vB
v vm v vm v vm
f f2 f1 f f f 2 2vB
Ans.
v vB v vB v vB 2
Example 35
A train approaching a hill at a speed of 40 km/hr sounds a whistle of frequency 580 Hz when
it is at a distance of 1 km from the hill. A wind with speed 40 km/hr is blowing in the
direction of motion of the train. Find (a) the frequency of the whistle as heard by an
v
fA = f
v vS
v w Wind
1 km
fA =
v w vS [1ăx] x
s s´
Substituting the given data,
1200 40
fA = 580 599.3 Hz
1200 40 40
(b) If x is the required distance from the hill, the distance moved by the train will be (1 ă x)
and hence the time taken by the train to travel this distance is (1 ă x)/40.
In this time sound travels a distance 1 km at speed (1200 + 40) and comes back a distance
x at speed (1200 ă 40); so
1x 1 x 29
, i.e., x km = 933.3 m
40 1240 1160 31
Now the engine will act as observer and hill as source; so the frequency heard by the moving
observer towards the stationary source will be
v vD
f2 fA
v
But in this situation as wind is blowing opposite to the direction of motion of sound;
v v ă w
v w vD 1200
so f2 fA 599.3 620 Hz
v w 1160
Oscillatory Motion :
It a body moves to and fro on the same path about a mean position, it is called as oscillatory
motion. For example : Swinging pendulum, vibratory motion of a mass attached to a spring.
Periodic Motion :
A motion which repeats itself after equal intervals of time is called a periodic motion. Each
oscillatory motion is a periodic motion, but every periodic motion is not oscillatory. For example,
a body moving in a circle is an example of periodic motion but it is not oscillatory motion.
A O P B
A A
The above figure shows a basic example of SHM. The particle starts from O towards B with some
velocity. Each moment the acceleration of the particle is directed towards point O. Hence, due to
this negative acceleration the velocity of the particle becomes zero at B and it starts gaining
velocity as it goes from B to O. The value of acceleration becomes zero when it reaches point O.
This motion is again repeated.
Hence, this type of motion is called simple harmonic motion.
Consider a particle free to move on x-axis, is being upon by a force given by,
F = ă kxn
Here, k is a positive constant.
Now, following cases are possible depending on the value of n.
(i) If n is even integer (0, 2, 4, .... etc), force is always along negative x-axis, whether x is a
positive or negative. Hence, the motion of the particle is not oscillatory. If the particle is
released from any position on the x-axis (except at x = 0) a force in negative direction of x-
axis acts on it and it moves rectilinearly along negative x-axis.
(ii) If n an odd integer (1, 3, 5 .... etc), force is along negative x-axis for x > 0, along positive
x-axis for x < 0 and zero for x = 0. Thus, the particle will oscillate about stable equilibrium
position, x = 0. The force in this case is called the restoring force. Of these, if n = 1, i.e. F
= ă kx the motion is said to be SHM.
Example 1
If the spring is un stretched, there is no net force on the mass - in other words, the system is in
equilibrium. However, if the mass is displaced from equilibrium, the spring will exert a restoring
force, which is a force that tends to restore it to the equilibrium position. In the case of the spring-
mass system, this force is the elastic force, which is given by HookeÊs Law, F = ăkx, where F is
the restoring force, x is the displacement, and k is the spring constant.
Any system that undergoes simple harmonic motion exhibits two key features.
1. When the system is displaced from equilibrium there must exist a restoring force that tends
to restore it to equilibrium.
2. The restoring force must be proportional to displacement, or approximately so.
The spring-mass system satisfies both
Once the mass is displaced it experiences a restoring force, accelerating it, causing it to start going
back to the equilibrium position. As it gets closer to equilibrium the restoring force decreases; at
the equilibrium position the restoring force is 0. However, at x = 0, the mass has some momentum
due to the impulse of the force that has acted on it; this causes the mass to shoot past the
equilibrium position, in the case, compressing the spring. The restoring force then tends to slow
it down, untill the velocity reaches 0, whereby it will attempt to reach equilibrium position again.
As long as the system does not lose energy, the mass will continue to oscillate like so; thus, the
motion is termed periodic motion. Further analysis will show that in the case of the spring-mass
system the motion is simple harmonic.
The necessary and sufficient condition for a motion to be simple harmonic (Linear) is
F = ăkx
d2 x
i.e. m = ăkx
dt2
d2 x 2 2 k
or 2 = – x with =
dt m
The solution of the above equation will be seen in a chapter called differential equations in
mathematics.
Hence, we are directly proceeding to the answer of this equation.
Which is,
x = A sin (t + ) ...(1)
Definitions :
1. Simple Harmonic Oscillator : Any object that oscillates about a stable equilibrium position
and experiences a restoring force approximately described by HookeÊs law. Examples of
simple harmonic oscillators include: a mass attached to a spring, a molecule inside a solid,
a car stuck in a ditch being „rocked out‰ and a pendulum.
3. Time Period : Periodic functions f (t) with period T are those functions of the variable ÂtÊ
which have the property,
f (t + T) = f (t) .... (5)
Both sin (t + ) and cos (t + ) will repeat their values if the angle (t + ) increases by
2 or its multiple. As T is the 2 smallest time for repetition.
(t + T) + = t + + 2
2T
T = or T =
k
Since =
m
m
T = 2 ...(6)
k
1 1 k
= = =
T 2 2 m
1 k
= ... (7)
2 m
The constant is called the angular frequency. The angular frequency and period in simple
harmonic motion are independent of the amplitude.
5. Phase : The quality = t + is called the phase. It determines the state of the particle
in simple harmonic motion.
When the particle is at mean position, x = 0
i.e., A sin (t + ) = 0
t + = n; n = 0, 1, 2, 3, ....
(i) consider n = 0 t + = 0
x= 0
and v = A cos (t + ) = A
i.e., the particle is crossing the mean position and is moving towards the positive direction.
(ii) consider n = 1
t + =
x= 0
and v = ăA
i.e., again the particle is crossing the mean position but now it is moving towards the
negative direction.
When the particle is at extreme position, x = xmax
i.e., A sin (t + ) = A
3
(t + ) = , , = 0
2 2
(2n 1)
i.e., (t + ) = ; n 0,1,2 =
2 2
consider n = 1, =
3 3
t + = =
2 2
x= ăA = 2
and v = 0
i.e., the particle is at extreme left and again its velocity is zero.
From above it is clear that as time increases the phase increases. An increases of 2 brings
the particle to the same status in the motion. Thus, a phase t + is equivalent to a phase
Similarly, the acceleration of the particle is given by,
d2x
– 2 A sin (t )
dt2
2n 1
It is zero when phase (t + ) = 0 and maximum (2A) when phase (t + ) = .
2
Graphically the variation of position, velocity and acceleration with the phase is shown
below.
4
O 2
O = t+
2
ăA ăV0
a
a0 = 2A
4
O 2
ăa0
6. Phase Constant
The constant term in the equation (3) is called phase constant or initial phase or epoch
of the particle. This constant depends on the choice of the instant t = 0
Suppose we choose t = 0 at an instant when the particle is passing through its mean position
towards right (i.e. positive direction). Then the phase = t + has to be zero.
Since t = 0 this means = 0. So the equation of displacement becomes
x = A sin t
If we choose t = 0 when the particle is at its extreme position in the positive direction. The
phase at this instant and hence . Therefore equation of displacement becomes
2 2
x = A cos t
The sine form and cosine form are basically equivalent. The value of phase constant, however,
depends on the form chosen; for example
x = A sin (t + ) = A sin (t + + ´)
2
Important Relation :
1. Posit ion
It mean position is at origin, the position (x) depends on time in general as :
2. Velocity
The expression of velocity at any time instant t.
as x = A sin (t + ) ...(1)
dx
V(t) = = A w cos(t ) ... (2)
dt
From equation 1,
x
sin (t + ) =
A
A 2 x2
or cos (t + ) = ... (3)
A
V ( x) A 2 x2
Also,
(1) Velocity is minimum at extremes because the particle is at rest. i.e. V = 0 at extreme
position (x + A)
(2) Velocity has maximum magnitude at mean position
3. Acceleration :
as v = A cos (t + )
dv
and a =
dt
a = ă 2 A sin (t + )
or at any position x, a( x) ă 2 x
(1) Acceleration is always directed towards the mean position
(2) Magnitude of acceleration is minimum at mean position and maximum at extremes.
a min = 0 at mean position
a max = 2 A at extremes
+A
(a)
t
ăA
x=A sin t
+A
(b)
t
ăA
v= dx =A cos t
dt
+2A
(c)
t
ă2A
a= dv =ă2 A sin t
dt
Graphs of (a) displacement, (b) velocity and (c) acceleration vs. time in SHM.
Example 2
A particle executes simple harmonic motion of amplitude 4 cm and a period 3 sec. Find the
speeds of the particle at (i) 2 cm from the mean position and (ii) at the mean position.
Solution :
Speed of the particle at a distance x from the mean position is given by
2
v = A 2 x2 = A 2 x2
T
2 2
(i) When x = 2 cm, v = 4 22 = 7.26 cm/sec
3
(ii) At the mean position x = 0,
2 2
v = A = .A = 4 = 8.378 cm/sec
T 3
Example 3
A particle executes S.H.M. of period sec and amplitude 2 cm. Find the acceleration of it
when it is (i) at the maximum displacement from the mean position and (ii) at 1 cm from the
mean position.
Solution :
Acceleration a at displacement x is given by
a = 2x
2
2 4 2
(i) When x (amplitude) = 2 cm, a = 2 = 2 = 8 cm/sec2
T 2
2
2 4 2
(ii) When x = 1 cm, a = 1 = 1 = 4 cm/sec2
T 2
Example 4
A particle executes S.H.M. of time period 10 s. The displacement at any instant is given by
the relation x = 10 sin t. Find (i) velocity of the body 2 s after it passes through the mean
position and (ii) the acceleration 2 s after it passes the mean position (Amplitude is given
in cm).
Solution :
(i) Velocity at any instant t is given by v = Acos t
2 2
Here A = 10 cm, = =
T 10
2 2
When t = 2 s, v = 10 cos 2
10 10
= 2 cos(0.4)
= 1.942 cm/s
(ii) Acceleration at any instant t is given by
a = ă A2 sin t
In the above article we found that the acceleration of a body in SHM is a = ă 2x. Applying the
equation of motion F m a, we have,
F = ăm2x = – kx
k
where, =
m
2 k
= =
T m
m 1 k
T = 2 and, f =
k 2 m
Kinetic Energy
The kinetic energy of the particle is,
1 1
K= mv2 = m A 2 sin 2 (t )
2 2
1
K = m2 A 2 [1 ă cos2 (t )]
2
1 1
K= m2 (A 2 ă x2 ) = k(A 2 ă x2 )
2 2
From this expression we can see that, the kinetic is maximum at the centre (x = 0) and zero at
the extremes of oscillation (x = A).
Potential Energy
dU dU
F = ă or = kx (as F = ăkx)
dx dx
U x
dU = kx dx
0
0
1 2 1
U= kx = m2 x2
2 2
Thus, the potential energy has a minimum value at the centre (x = 0) and increases as the particle
approaches either extreme of the oscillation (x = A).
Total Energy
Total energy can be obtained by adding potential and kinetic energies. Therefore,
1 1
K= K+ U= m2 (A 2 ă x2 ) m2 x2
2 2
1
= m2 A 2
2
1
or E= kA 2 (as m2 = k)
2
Which is a constant quantity. This was to be expected since the force is conservative.
Therefore, we may conclude that, during an oscillation, there is a continuous exchange of kinetic
and potential energies. While moving away from the equilibrium position, the potential energy
increases at the expense of the kinetic energy. When the particle moves towards the equilibrium
position, the reserve happens.
E, U, K
E= 1 kA2 U= 1 kx2
2 2
K = 1 k(A2ăx2)
2
x
ăA +A
Figure shows the variation of total energy (E), potential energy (U) and kinetic energy (K) with
displacement (x).
Note : In SHM, F = ăkx or a = ă2x, Făx graph or a ă x graph is a straight line passing through
origin with negative slope. The corresponding graphs are shown below.
(b)
(a)
Slope = ă 2
Slope = ă k
TABLE OF RELATIONS
S. Name of the equation Expression of the Remarks
No. equation
1. Displacementătime x = A cos ( t + ) x varies between +A and ăA
dx
2. Velocity ă time v = v = ă A sin ( t + )
dt
v varies between + A and ă A
dv
3. Accelerationătime a = a = ă A2 cos ( t + ) a varies between + A2 and ăA2
dt
1 2 1 1
4. Kinetic energyătime K = mv K = mA 2 2 sin 2 ( t ) K varies between 0 and mA 2 2
2 2 2
1 2 2 1 1
5. Potential energyătime U = m x K = m2 A 2 cos2 ( t ) U varies between mA 2 2 and 0
2 2 2
1
6. Total energyătime (E = K + U) E= m 2 A 2 E is constant
2
7. Velocityădisplacement v = A 2 ă x2 v = 0 at x = A and at x = 0
v = A
8. Accelerationădisplacement a = 2x a = 0 at x = 0
a = 2 A at x = A
1
9. Kinetic energyădisplacement K = m2 (A 2 ă x2 ) K = 0 at x = A
2
1
K = m2 A 2 at x = 0
2
1 1
10. Potential energyădisplacement U = m2 x2 U = 0 at x = 0 U = m2 A 2
2 2
at x = A
1
11. Total energyădisplacement E = m2 A 2 E is constant
2
Please Note :
(1) From the above table we see that x, v and a are sine or cosine function of time. So. they
all oscillate simple harmonically with same angular frequency . Phasse difference between
x and a is and between any other two is .
2
(2) We have read that potential energy in zero at mean position and maximum at extreme
positions and kinetic energy is maximum at mean position and zero at extreme positions. But
the correct statement is like this,
At mean position K is maximum and U is minimum (it may be zero also, but it is not
necessarily zero).
at extreme positions K is zero and U is maximum.
U(J) U(J) U(J)
Thus, in figure (a), oscillations will take place about the mean position x = 0 and minimum
potential energy at mean position is zero.
In figure (b) mean position is at x = 2 m and the minimum potential energy in this position
is 5 J.
In figure (c) mean position is at x = ă 2 m and the minimum potential energy in this position
is again zero.
Example 6
dU
Solution. (a) F = – = ă 2 (x ă 2)
dx
A body free to rotate about a given axis can make angular oscillations. For example, a wooden
stick nailed to a wall can oscillate about its mean position in the vertical plane.
The conditions for an angular oscillation to be angular simple harmonic motion are
(i) when a body is displaced through an angle from the mean postion ( = 0; = 0), the
resultant torque (acts which) is proportional to the angle displaced,
(ii) this torque is restoring in nature and it tries to bring the body towards the mean position.
It the angular displacement of the body at an instant is , then resultant torque on the
body,
= ă k O .....(1)
If the moment of inertia is I, the angular acceleration is
k
=
I I
d 2 k
or, = ă 2; where = .....(2)
dt 2 I
d
= = 0 cos (t + ) .....(4)
dt
Simple Pendulum
A point mass suspended by a massless inextensible string from a rigid support constitutes a
simple pendulum. In reality neither point mass nor massless string exist. So we can never
construct a simple pendulum strictly according to its definition. A small heavy spherical mass
suspended from a light inextensible string constitutes an approximate simple pendulum.
Now when the pendulum is pulled to one side and released, it will swing to and fro describing
an arc of a circle in a vertical plane (as the string is inextensible), i.e., the displacement involved
in the motion is angular. So the motion of a simple pendulum is angular and oscillatory. Here
restoring torque about S is developed due to the weight of the pendulum as shown in Fig., i.e.,
= ă mg CE = ă mg L sin
[negative sign is used as direction of torque is opposite to displacement].
m
L
T
D
A E C
B mg
If is small sin = , so
= ă mgL ...(1)
i.e., restoring torque is linear, so motion will be angular simple harmonic,
Now as in case of rotational motion :
2 d d 2
2
a s I m L a n d 2
2
= I = m L dt 2 ...(2)
dt
d 2 d 2q g
mL2 = ă mgL i.e., = –
dt 2 dt 2
L
d 2 g
i.e., = ă 2 with 2 = ...(3)
dt 2 L
Eqn. (3) is the standard equation of angular simple harmonic motion with time period
T = (2/); so here
L
T0 = 2 ...(4)
g
e.g., if the acceleration a is upwards, then
l
g eff = g + a and T = 2
ga
If the acceleration a is downwards, then (g > a)
l
g eff = g ă a and T = 2
g–a
It the acceleration a is in horozontal direction, then
g eff = a 2 + g2
In a freely falling lift geff = 0 and T = , i.e., the pendulum will not oscillate.
(ii) If in addition to gravity one additional force F , (e.g., electrostatic force F e ) is also acting
on the bob, then in that case,
F
g eff = g +
m
Example 7
A simple pendulum of lenght l is suspended from the ceiling of a cart which is sliding
without friction on an inclined plane of inclination . What will be the time period of the
pendulum.
Solution :
Here, point of suspension has an acceleration. a = g sin (down the plane). Further, g can be
resolved into two components g sin (along the plane) and g cos (perpendicular to plane).
in
in gs os
gs gc
a=
g eff = g – a
T = 2 l
g eff
= 2 l
g cos
l
Note : If = 0Ĉ, T = 2 which is quiet obvious.
g
Example 8
A simple pendulum consists of a small sphere of mass m suspended by a thread of length
l. The sphere carries a positive charge q. The pendulum is placed in a uniform electric field
of strength E directed vertically upwards. With what period will pendulum oscillate if the
electrostatic force acting on the sphere is less than the gravitational force ?
Solution :
The two forces acting on the bob are shown in figure
W – Fe
geff in this case will be
m
m g – qE qE
or, geff = =g –
m m
l
T = 2
g eff
l
= 2
qE
g–
m
Two simple pendulums of length 1 m and 16 m respectively are both given small displacement
in the same direction at the same instant. They will be again in phase after the shorter
pendulum has completed n oscillations. Calculate n.
Solution :
pendulum will be small, i.e., it will complete more oscillations in the same time than the longer
pendulum. So if for the first time the two pendulums are in same phase when the shorter one
has completed n oscillations, y y
nTs = (n ă 1)TL, O O
1K
2
i.e., nK L s = (n ă 1)K L L
K K
or n 1 = (n ă 1) 1 6 , 4K
3
i.e., 3n = 4 3K
2
or n = (4/3), 2K
t t
i.e., the two pendulums will be in the same phase for the first
time when the shorter pendulum has completed (4/3) oscillations.
Example 10
A light rod of length L2 has a small ball of mass m 2 fixed at one end and another ball of
mass m 1 fixed on it at a distance L1 from the free end. The rod is supported at its end O and
is free to rotate about a horizontal axis at O. The rod is slightly displaced from its equilibrium
vertical position and released. Find the frequency of its oscillations about O and the length
of the equivalent simple pendulum.
Solution :
When this pendulum is displaced from vertical by an angle the restoring torque
O
d 2 m 1 L1 m 2 L2
2 = – g
dt m1 L21 m 2 L22 L1
d 2 L2
or = ă 2
dt 2
m1g
(m 1 L1 + m 2 L 2 ) g
with 2 =
(m 1 L21 + m 2 L22 ) m2g
This is the standard equation of angular SHM with time period T = (2/). So here
m1 L21 m 2 L22
1 /2
T = 2
m1 L1 m 2 L2 g
1 m1 L1 m 2 L2 g
i.e., f = 2 m L2 m L2
1 1 2 2
T = 2
L
so L=
(m 1 L12 + m 2 L22 )
g (m 1 L1 + m 2 L 2 )
Example 11
A simple pendulum of length L and mass m has a spring of force constant k connected to
it at a distance h below its point of suspension. Find the frequency of vibrations of the
system for small values of amplitude.
Solution :
As shown in Fig. if the pendulum is given a small angular displacement . the spring will also
stretch by y (= h tan ). So here restoring torque about S will be due to both force of gravity and
elastic force of the spring,
d 2 m gL kh 2
or = ă2 with 2 =
dt 2 m L2
This is the standard equation of angular SHM with frequency f = (/2); so here
1 m gL kh 2
f =
2 m L2
1 g k
Note : If h = L, f = . This result reduces to f = (1/2) g / L for k 0, justifying the
2 L m
Spring Pendulum
A point mass suspended from a massless spring constitutes a linear harmonic spring pendulum.
If y is a small extension or compression in the spring from the equilibrium state, the restoring
force produced is given by
d2 y d2 y
F = ă ky, i.e., m = ă ky asF ma m
dt2 dt2 k
d2 y k d2 y k m
or = ă y i.e., = ă 2y with 2 =
dt 2
m dt2 m
(A)
This is the standard equation of linear simple harmonic motion with time period T = (2/). So
here
m 1 k
T= 2 or f =
k 2 m
T m or (
f 1/ m )
i.e., greater the mass greater will be the inertia and so lesser will be the frequency of
oscillations and greater will be the time period.
(3) The time period depends on the force constant k of the spring, i.e.,
(
T 1/ k ) or f k
i.e., greater the force constant greater will be the frequency and lesser will be the time
period. [The force constant k of a spring is inversely proportional to its natural length.]
(4) If the spring has a mass M and mass m is suspended from it, effective mass is given by
(5) If two masses of mass m1 and m2 are connected by a spring and made to oscillate on
horizontal surface, the reduced mass m is given by
1 1 1 m
= m +m so that T = 2
m 1 2 k
R
k L
m1 m2 m
k
mg
(A) (B)
mg
m y
T =2 = 2 0
k g
Here is must be kept in mind that time period is independent of ÂgÊ as with change in g, y0
will aslo change in such a way that y0/g[=m/ k) remains constant.
(8) Composite-spring pendulum : If a spring pendulum is constructed by using two springs
and a mass, the following three situations are possible :
k2
k1 k2 k1 k2
m m m
k1
m( k1 k2 ) m m
So T 2 T 2 T 2
k1 k2 (k1 + k2 ) (k1 + k2 )
Example 12
Furthermore, the stretch produced by m1g will set the amplitude, i.e.,
m1g = kA i.e., A = (m1g/k)
Initially the level of liquid in both the columns is same. The area of cross-section of the tube is
uniform. If the liquid is depressed by x in one limb, it will rise by x along the length of the tube
is the other limb. Here, the restoring force is provided by the hydrostatic pressure difference.
F = ă(P) A = ă (h1 + h2) gA
x
= ă gA (sin 1 + sin 2) x
x
Let, m be the mass of the liquid in the tube. Then,
1 2
ma = ă gA (sin 1 + sin 2) x
Since, F or a is proportional to ăx, the motion of the liquid column in simple harmonic in nature,
time period of which is given by,
x
T = 2
a
m
or T = 2 gA(sin + sin )
1 2
Note : For a U-tube if the liquid is filled to the height l, 1 = 900 = 2 and m = 2 (lA )
l
So, T = 2
g
l
Thus, we see that the expression T = 2 comes in pictures at three places
g
If the particle is acted upon by two separate forces each of which can produce a simple harmonic
motion, the resultant motion of the particle is a combination of two simple harmonic motions.
Let r1 denote the position of the particle at time t of the force F1 alone acts on it. Similarly, at r2
denote the position at the time ÂtÊ if the force F2 alone acts on it.
d 2 r1 d 2 r 2
Adding them, m + =F1 + F2
dt 2 dt 2
d2
m (r 1 + r 2 ) =F1 + F 2
dt2
But F 1 + F2 is the resultant force acting on the particle and so the position of r of the particle
when both the forces acted act is given by
d 2 r
m 2 F1 F2
dt
If these conditions are met at t = 0, the actual position of the particle is given by the vetor sum
of r1 & r2 .
x1 = A1 sin t
A1 A 2 cos
2
A= (A 2 sin )2
A 2 sin
and tan = A A cos ...(21)
1 2
Amax = (A 1 A 2 ) 2 = A1 + A2
Thus, we can see that this is similar to the vector addition. The same method of vector addition
can be applied to the combination of more than two simple harmonic motions.
Example 12
Find the displacement equation of the simple harmonic motion obtained by combining the
motions.
x 1 = 2 sin t, x 2 = 4sin t and x 3 = 6 sin t
6 3
Solution :
The resulant equation is,
x = A sin (t + )
Ax = 2 + 4 cos 300 + 6 cos 600 = 8.46
and Ay = 4 sin 300 + 6cos 300 = 7.2
A3 = 6
A = ( x ) 2 ( y ) 2 y
A2 = 4
= (8.46)2 (7.2)2 30Ĉ
30Ĉ x
= 11.25 A1 = 2
y 7.2
tan = = = 0.85
A x 8.46
Example 14
A body of mass m falls from a height h on to the pan of a spring balance. The masses of
the pan and spring are negligible. The spring constant of the spring is k. Having stuck the
pan the body starts performing harmonic oscillations in the vertical direction. Find the
amplitude and energy of oscillation.
Solution :
Suppose by falling down through a height h, the mass m compresses the m
spring balance by a length x.
h
The P.E. lost by the mass = mg (h + x)
This is stored up as energy of the spring by compression
1 2 k
= kx
2
1 2 1 2
mg(h + x) = kx or kx ă mgx ă mgh = 0
2 2
2mgx 2mgh
or x2 ă ă = 0
k k
Solving this quadratic equation, we get
2
2mg 2mg 8mgh
mg mg 2 kh
x= k k k = 1+
k k mg
2
mg
In the equilibrium position, the spring will be compressed through the distance and hence
k
the amplitude of oscilation is
mg 2 kh
A= 1+
k mg
2 kh
2
1 1 mg
Energy of oscillation = kA2 = k 1+
2 2 k mg
(mg)2
= mgh +
2k
Example 15
A block with a mass 2 kg hangs without vibrating at the end of a spring of spring constant
500 N/m, which is attached to the ceiling of an elevator. The elevator is moving upwards
g
with an acceleration . At t = 0, the acceleration suddenly ceases.
3
(a) What is the angular frequency of oscillation of the block after the accelration ceases ?
(b) By what amount is the spring stretched during the time when the elevator is
accelerating ?
(c) What is the amplitude of oscillation and initial angle observed by a rider in the
elevator ?
Take the upward direction to be positive. Take g = 10.0 m/s2.
Solution :
k kx
(a) Angular frquency =
m
g
a=
3
500
=
2
g
kx ă mg = ma = m
3
x = 5.3 cm
(c) (i) In equilibrium when the elevator has zero accelration, the equation of motion is,
kx0 = mg kx0
mg (2) (10)
x0 = = = 0.04m
k 500
= 4cm
Amplitude A = x ă x0 = 5.3 ă 4.0 x = +A
= 1.3 cm. Ans.
(ii) At time t = 0, block is at x = ăA. Therefore,
substituting x = ăA and t = 0 in equation,
Mean position
x = A sin (t + )
3
We get initial phase. = Ans.
2 x = ăA
I
T = 2
mgl
2mr2 2r
T = 2 = 2
mgr g
2
Angular frequency =
T
g
or =
2r
Example 17
2h 1 2
Time of ascent t1 = h 2 at
a
1
T g
T´ g
=
T ga
g
or T´ = T
ga
g
T = (T ă T´) = T 1
g a
T
t1 = .t1
T´
2h g a
= ă 1 ⁄ (i)
a g
T´ g
In this case, =
T ga
g
or T´ = T
ga
g
or T = T ă T´ = T 1
ga
T
Time lost in time t2, t2 = t2
T´
ga
or t2 = t2 1 ... (ii)
g
2h g a ga
1 = t2 1
a g g
2h g a g
t2 =
a g g a
Total time t = t1 + t2
2h ga ga
or t=
a g g a
Example 18
The rod PQ of mass, M is attached as shown to a spring of spring constant K. A small block
of mass, m is placed on the rod at its free end P. If end P is moved down through a small
distance x’ and released determine the period of vibration.
Solution :
Method I : Using energy equation
Moment of inertia of the system about Q is
ML2
I = mL2
3
k
where L is the length of rod.
m
Q
1 2 P
Elastic potential energy of the spring = kx
2 b
1 2 L
Rotational energy of the rod PQ = I
2
1 2 1 2
I k x = constant
2 2
1 ML2 2 v´
2
x´ 2 b2
m L 2 k 2 = constent
2 3 L L
x´ b
where x = .
L
M
3 m 2 dv´ 1 2 dx´ dx´
L 2v´ kb 2 x´ = 0; where = v = velocity of m
2 dt 2 dt dt
dv´ kb2 x´
Acceleration of the block, =
dt M 2
mL
3
kb2
2 =
M 2
mL
3
M 2
mL
2 3
Period of vibration = 2
kb2
ML2
where I = mL2
3 T
b
Q
.. kb2 kb2
= ă = ă
I M m L2
3
kb2
This is an angular S.H.M. with angular frequency, =
M 2
mL
3
M 2
2 m L
= 2
3
Period of vibration =
kb2
mV 2
When the car comes round a circle it is an accelerated frame of reference. A fictitious force
R
is to be introduced to the simple pendulum as a centrifugal force. If be the angular displacement
of the pendulum in its new equilibrium position, then
S cos = mg
mV 2
S sin =
R
Where S is the tension in the string. S
2
mV 2
S= (mg)
2
mv2
R
R
V4 mg
= m g
2
R2
Let the pendulum be slightly displaced so that it makes an angle (+ d) with the verticle and
then let go.
Sx
The forces = S sin d Sd =
Lx
where x is the linear displacement and L and length of the pendulum and x = Ld.
Sx
The restoring force/unit displacement =
L
S
=
L
The period of oscillation of the pendulum
m m Lm
T = 2 = 2 = 2
k S/L S
Lm L
= 2 1/2
= 2 1/ 2
V 4
2 V4
m g2 2 g + 2
R R
1/ 2
2 V4
g 2
1 1 R
Frequency, =
T 2 L
Example 20
A particle of mass m is performing simple harmonic motion in a straight line with amplitude
r and period T. Find the law of force. When at a distance kr from the centre of oscilation,
it collides with a stationary particle of the same mass and coalesces with it. If the law of
force is the same, find the new period of oscillation and the amplitude.
Solution.
Amplitude given = r
2
Period = T; hence angular frequency, =
T
4 2 m
ma = ă x ... (1)
T2
T
x = r when v = 0, t =
4
42mx
The law of force is therefore Fx = ă at distance x.
T2
The solution of the equation can be obtained by integrating equation (1) with the given initial
condition.
4 2
Putting 2 = , a = ă2 x ...(2)
T2
or v = r2 x2 ...(3)
dx
Integrating again, r x2
2 = t + C
x
or sin ă1 = t + C
r
At t = 0, x = 0, and so, C = 0
x = r sin (t) ...(4)
T
This also satisfies the condition, that x = r when t = .
4
4 2
= (1 k2 ) r 2
T2
2r
v= 1 k2 ...(6)
T
If it collides with a stationary particle of same mass m and coalesces with it, the law of conservation
of linear momentum for this collision gives,
m . v = 2 m . v´ ...(7)
where vÊ is the new velocity of the new system.
v 1 2r
Hence v´= = . 1 k2
2 2 T
r
= 1 k2 ...(8)
T
dx
Putting vÊ = , where x is the new coordinate for the particle now of mass 2m, we have
dt
dx r
dt = 1 k2 ...(9)
x kr T
2
or a= x ...(10)
2
or a = ă ´2x
where Ê = the new angular velocity
=
2
2 2 2 2 2
T´ = = =
´ 2 / T
= 2T
Hence after the collision, the combined mass 2m will oscillate with the new period 2T.
Also, if rÊ be the new amplitude, we will have
2
dx
= ´2 (r´2 ă x2) ...(11)
dt
2
r 2
T 1 k = ´2 (r´2 ă k2r2)
2 r 2 (1 k)2 4 2
or = (r´2 ă k2r2)
T2 T´ 2
2 r 2 (1 k)2 4 2
or = (r´2 ă k2r2)
T2 2T´ 2
r2
r´2 = (1 k2 )
2
1
Hence r´ = r (1 k2 )
2
1
Thus the new amplitude of oscillation = r (1 k2 )
2
A ball is suspended by a thread of length at the point O on the wall, forming a small angle
with the vertical. Then the thread with the ball was deviated through small angle
( > ) and set free. Assuming the collision of the ball against the wall to be perfectly
elastic, find the oscillation period of such a pendulum.
Solution.
As is a small angle, the motion of the ball is S.H.M. After perfectly elastic collision the velocity
of the ball is simply reversed. As shown in figure, the time period of one oscillation will be
T T T O
T´ = t t= 2t
4 4 2
T
where T = 2 or = T
g 2 g t 4
T
= sin t t 4
Cm
v = sin t, where t is the time taken from B to A. A B
1
t= sin ă1 = sin ă1
g
T´ = 2 sin ă1 = 2 sin ă1
g g g 2
Example 22
1
Moment of inertia of the system = MR 2 m´R 2 mR 2
2
1 d 2
MR 2 m´R 2 mR 2 2 = m´ g R ă mg R sin ( + )
2 dt
1 2d
2
MR 2
( m sin )R 2
mR 2 = mgR sin ă mgR sin ( + )
2 dt
1 2 d 2
R [M 2m sin 2m] 2 = mgR sin ă mgR [sin cos + cos sin ]
2 dt
1 2 d 2
R [M 2m sin 2m] 2 = mgR sin ă mgR sin ă mgR cos
2 dt
d2
[MR + 2mR (1 + sin)] = ă 2 mg cos
dt2
d2
is proportional to .
dt2
2mg cos
2 =
MR 2mR(1+sin)
2
period =
1 2mg cos
Frequency = =
2 2 MR 2mR(1 sin )
Figure shows a system consisting of a massless pulley, a spring of force constant k and a
block of mass m. If the block is slighty displaced vertically down from its equilibrium
position and released, find the period of its vertical oscillation in cases (a ), (b) and (c ).
k k
m
k
m m
Solution :
(a) In equilibrium, kx0 = mg ...(i)
When further depressed by an amount x, net restoring force (upwards) is,
F = ă{k(x + x0) ă mg}
F = ă kx (as kx0 = mg)
k
or a = x
m
x0
x
T = 2 x
a
m
or T = 2
k
(b) In this case if the mass m moves down a distance x from its equilibrium position, then pulley
x x
will move down by . So, the extra force in spring will be k . Now, as the pulley is massless,
2 2
kx kx
this force is equal to extra 2T or T = . This is also the restoring force of the mass.
2 4
Hence,
kx
F = ă
4 kx
2
k
or a = x
4m
kx kx
4 4
x
or T = 2
a kx
4
m
4m
or T = 2
k
(c) In this situation if the mass m moves down a distance x from its equilibrium position, the
pulley will also move by x and so the spring will stretch by 2x. Therefore, the spring force
will be 2kx. The restoring force on the block will be 4kx. Hence,
F = ă4kx
4k 2kx 2kx
or a = ă .x
m
x
T = 2
a 4kx
m m
or T = 2
4k
Example 24
y
SIMPLE HARMONIC MOTION
42 QUIZRR
segment is directly proportional to its distance from the fixed end, so for this segment.
ms x
dm = dx and v = .V
l l
2
1 1 ms x
dKs = ( dm)(v ) =
2
dx V
2 2 l l
Ks = dK
0
s
1
Integrating we get, Ks = ms V 2
6
The mechanical energy of the system in displaced position of the block will be,
E = kinetic energy of mass + kinetic energy of spring + elastic potential energy
1 1 1
E= mV 2 ms V 2 ky2
2 6 2
Since, E = constant
dE dV 1 dV dy
= 0 or mV. 3 ms V dt ky. dt = 0
dt dt
dV dy
Substituting = a and = V
dt dt
ms
We have, m 3 .a = ă ky
a ă y
Therefore, motion is simple harmonic in nature.
y
T = 2
a
ms
m
or T = 2 3
k
Example 25
x
mean position and v be its speed at this moment. Then B lower by and speed of B at this
2
v
instant will be . Total energy of the system in this position will be,
2
2
1 1 1 v
E = k( x x0 ) mA v mB mA ghA mB g hB
2 2
2 2 2 2
1 1 1 x
or E = k( x x0 ) 2 mv2 mv2 mgx sin mg
2 2 8 2
1 5 x
or E = k( x x0 ) 2 mv2 mgx sin mg
2 8 2
Since, E is constant,
dE
= 0
dt
dx 5 dv dx mg dx
or 0 = k( x x0 ) mv mg(sin ) dt 2 . dt
dt 4 dt
dx
Substituting, =v
dt
dv
= a
dt
mg
and kx0 + mg sin = [From Eqs. (i) and (ii)]
2
5
We get, m a = ăkx
4
Since, a ă x
Motion is simple harmonic, time period of which is,
x 5m
T = 2 = 2
a 4k
2 4k
= =
T 5m
Example 26
Consider the earth as a uniform sphere of mass M and radius R. Imagine a straight smooth
tunnel made through the earth which connects any two points on its surface. Show that
the motion of a particle of mass m along this tunnel under the action of gravitation would
be simple harmonic. Hence, determine the time that a particle would take to go from one
end to the other through the tunnel.
Solution :
Suppose at some instant the particle is at radial distance r from centre of earth O. Since the
particle is constrained to move along the tunnel, we define its position as distance x from C.
Hence, equation of motion of the particle is,
max = Fx
This gravitational force on mass m at distance r is, r x x ă axix
O C
GMmr
F = (towards O)
R3
GMmr x
Therefore, Fx = ă F sin =
R3 r
GMm
= .x
R3
GMm GM
max = 3
. x or ax = 3 .x
R R
x R3
T = 2 = 2
ax GM
T
The time taken by particle to go from one end to the other is .
2
T R3
t = =
2 GM
Example 27
A solid sphere (radius = R) rolls without slipping in a cylindrical through (radius = 5R). Find
the time period of small oscillations.
5R
Solution :
For pure rolling to take place, v = R
´ = angular velocity of COM of sphere C about O
v R
= = =
4R 4R 4
d´ 1 d
=
dt 4 dt
´
O
or ´ =
4
C
v
a
= for pure rolling
R
g sin 5 g sin
where, a = =
I 7
1
mR 2
2
as, I = mR 2
5
5 g sin
´=
28 R
5g
´ = ă
28R
28R
T = 2 = 2
´ 5g
Example 28
1
the fluid is Vv2 where is the mass density of fluid, V the volume of
4
sphere and v is the velocity of the sphere. Consider a 0.5 kg hollow spherical shell of radius
8 cm which is held submerged in a tank of water by a spring of force constant 500 N/m.
(a) Neglecting fluid friction, determine the period of vibration of the shell when it is
displaced vertically and then released.
(b) Solve part (a ) assuming that the tank is accelerated upward at the constant rate of 8
m/s2. Density of water is 103 kg/m 3.
Solution :
(a) Let F be the upthrust and W the weight of the sphere. In equilibrium let x0 F + kx0
be the compression of the spring, then
F + kx0 = W W
or kx0 = W ă F ...(i)
If the sphere is further compressed by x, then total energy of the system will be
1 1 1
E = ă(W ăF).x k( x x0 ) 2 mv2 Vv2
2 2 4
dE
= 0
dt
dx dx dv 1 dv
or 0 = ă (W F). k( x x0 ) mv 2 Vv dt ...(ii)
dt dt dt
dx dv
From Eqs. (i) and (ii) with substitutions = v and = a, we get
dt dt
k
a = .x
1
V m
2
a ă x
Oscillations are simple harmonic, time period of which will be
1
x m V
T = 2 = 2 2
a k
1 4
0.5 103 (0.08)3
= 2 2 3
k
T = 0.352 s
(b) When it is accelerated upwards with an acceleration ÂaÊ
F( g a)
F´ = g F + kx0
a
W W
Now, F´ + kx0 ă W = a
g
W a
kx0 = .a W F 1
g g
a
or kx0 = (W ă F) (W ă F)
g
a
or kx0 = (W ă F) 1 ...(iii)
g
( g a) 1 1 1
E = ă (W ă F) . x k( x x0 )2 mv2 Vv2
g 2 2 4
dE
Substituting = 0
dt
a dx dx dv 1 dv
or 0 = ă (W ă F) 1 k( x x0 ) mv. vV. ...(iv)
g dt dt dt 2 dt
From Eqs. (iv) and (iii) we get the same result as was obtained in part (a), i.e.,
T = 0.352 s
Example 29
= F r = (SLg)(d 2 ă d 1)(QG)
L
or = ă (SLg)(d 2 ă d 1) sin
2
SL2 g (d2 d1 )
or = ...(i)
2
d 2 SL2 g (d2 d1 )
I = ...(ii)
dt2 2
d2 3 g (d2 d1 )
=
dt2 2 d1 L
d2
= ă 2
dt2
3 g( d2 d1 )
=
2d1L
Example 30
Two identical balls A and B, each of mass 0.1 kg. are attached to two identical massless
springs. The spring-mass system is constrained to move inside a rigid smooth pipe bent in
the form of a circle as shown in figure. The pipe is fixed in a horizontal plane. The centres
of the ball can move in a circle of radius 0.06 metre. Each spring has a natural length of
0.06 metre and spring constant 0.1 N/m. Initially, both the balls are displaced by an angle
= radian with respect to the diameter PQ of the circle (as shown in figure) and released
6
from rest.
[JEE 1993]
Solution :
(i) Given : Mass of each ball A and B, m = 0.1 kg
Radius of circle, R = 0.06 m A B
Natural length of spring, O
/6 /6
l0 = 0.06 = R (Half circle)
and spring constant, k = 0.1 N/m
In the stretched position elongation in each spring (x = R)
Spring in lower side is stretched by 2x and on upper side compressed by 2x.
Therefore, each spring will exert a force 2kx on each block.
Hence, a restoring force, F = 4kx will act on A in the direction shown in figure.
Restoring torque of this force about origin,
= ăF . R = ă (4kx)R = ă (4kR)R
or = ă4kR2 . ...(i)
Since, ă each ball executes angular SHM about origin O.
Eq. (i) can be rewritten as
I = ă4kR2 or (mR2) = ă4kR2
4k
=
m
kx
1 acceleration
=4
Frequency of oscillation, f = A
2 displacement
F
O
1 1 4k
= =
2 2 m
1 4 0.1 1
f = = Hz
2 0.1
1
P.E. = 2 k 2 x = 4kx2
2
2
and in mean position, both the balls have only kinetic energy, Hence
1
K.E. = 2 mv2 = mv2
2
k k
v = 2x = 2R
m m
0.1
v = 2(0.06)
6 0.1
or v = 0.0628 m/s
Example 31
Two balls will masses m 1 = 1 kg and m 2 = 2 kg are
slipped on a thin smooth horizontal rod. The balls m1 m2
k
are interconnected by a light spring of spring
constant 24 N/m. The left hand ball is imparted the
initial velocity v1 = 12 cm/s. Find (a ) the oscillation
frequency of the system, (b) the energy and amplitude
of oscillation.
Solution :
(a) As discussed earlier;
k
2o = where 0 is the natural frequency of oscillation.
m1 m2
m1 m2
SIMPLE HARMONIC MOTION
52 QUIZRR
k m1 m2
2o = where ø =
m1 m2
called reduced mass.
k 1 k
= ; = = 2.65 10ă2/s
ø 2 ø
1
Hence total energy of S.H.M of two balls is given as E = øV12
2
1 1
øV12 = kA 2
2 2
v1 = 0A
v1
A=
o
THERMODYNAMICS
(a) Thermodynamics : The branch of science which deals with transformation of heat energy
into other forms of energy or vice-versa.
(b) Thermal equilibrium : If two systems are at same temperature, they are said to be in
thermal equilibrium.
(c) Thermodynamical equilibrium : A system is said to be in thermodynamical equilibrium
when it is in a state of thermal, mechanical and chemical equilibrium.
(d) Thermodynamical variables : These are macroscopic physical quantities like pressure (P),
volume (V) and temperature (T), etc., which are used to describe the state of the system. The
relation between these variables is called equation of state.
(e) Thermodynamical process : If the state of a system changes in such a way that any of
P, V, T, etc. changes, the process is called thermodynamical process.
(f) Cyclic process : If a system after undergoing through a series of changes comes back to
its initial state, the process is called cyclic.
(g) Reversible process : A reversible process is one which can be reversed in such a way that
all changes taking place in the direct process are exactly replaced in the inverse order and
opposite sense and no changes are left behind.
W W
Q1 Q2 Q1 Q2
(A) (B)
Reversibility in physics is an ideal concept that cannot be realised in practice.
I. Zeroth Law
According to it, „If systems A and B are each in thermal equilibrium with a third system C, then
A and B are in thermal equilibrium with each other‰.
Regarding zeroth law it is worth noting that :
It defines temperature as the property of the system which equals that of another system when
the two systems are in thermal equilibrium, e.g., if two gases in different containers at different
pressures, temperature and volumes are put in thermal contact for sufficient time to reach thermal
equilibrium, their pressures will in general not be equal, nor will their volumes; their temperatures,
however, will always be equal in thermal equilibrium.
THERMODYNAMICS
4 QUIZRR
II. First Law of Thermodynamics
First law is a relation among heat, work and internal energy; so before dealing with the law here
we review these physical quantities.
(a) Q)
Heat (
It is the energy that is transferred between a system and its environment because of
temperature difference between them.
In case of solids and liquids,
Q = mL if there is change of state
Q = mCT if there is change in temperature
In case of gases,
Q = nCT
in differential form dQ = nCdT
where C is the molar heat capacity of the gas and n is the number of moles of the gas.
and always take, T = Tf ă Ti
where Tf is the final temperature and Ti is the initial temperature of gas. Also, the molar
heat capacity of an ideal gas
R R R
C CV
1 1 x 1 x
R
C CV in isochoric process and
1
C = CP = CV + R in isobaric process.
Note :
(1) SI unit of heat is Joule and CGS is calorie [1 cal = 4.2 J]
(2) It is path dependent, e.g., heat required to change the temperature of same gas
through same amount at constant volume.
vf
W
dW =
vi
PdV
THERMODYNAMICS
QUIZRR 5
D erivation
Figure shows a gas confined to a cylinder that has a movable piston at one end. If the gas
expands against the piston, it exerts a force through a distance and does work on the piston. If
the piston compression the gas as it is moved inward, work is also doneăin this case on the gas.
The work associated with such volume changes can be determined as follows.
F = PA
dx
Let the gas pressure on the piston face be P. Then the force on the piston due to gas is PA, where
A is the area of the face.
When the piston is pushed outward an infinitesimal distance dx, the word done by the gas is
dW = F . dx = PA dx
dW = PdV
For a finite change in volume from Vi to Vf, this equation is then integrated between Vi to Vf to
find the net work
Vf
W
dW
Vi
Pd V
Note :
(1) Like heat, work is also path dependent, scalar physical quantity.
(2) In a cyclic process work done is equal to the area under the cycle and is positive if the cycle
is clockwise and negative if anticlockwise.
THERMODYNAMICS
6 QUIZRR
This equation can be applied in any process, whether it is isobaric, isothermal, adiabatic or
else.
Note :
(1) Change in internal energy is path independent and depends only on the initial and final
states of the system.
(2) Change in internal energy in a cyclic process is always zero U = UF ă UC = 0
T he Law
U = Uf ă Ui = Q ă W ...(i)
Process Conventions
THERMODYNAMICS
QUIZRR 7
W Pdv
PV = nRT
Process equation
Example 1
A solid cylinder of mass 1 kg is initially kept at 20ĈC under atmospheric pressure, Patm =
105 N/m2. Now, a heat of 20000 J is given to the cylinder, then find :
(a) increase in temp. of cylinder
(b) work done by cylinder
(c) increase in its internal energy
S = 400 J/g/K, = 9 ï 10ă5ĈCă1, d = 9000/g / m3
Solution :
(a) Q = ms
Q 20000
50C
ms 1 400
THERMODYNAMICS
8 QUIZRR
(b) V = V
= 9 10
5 1
50
9000
= (5 ï 10ă7)
Work done = Patm V
= (105 ï 5 ï 10ă7) = (5 ï 10ă2)J
Example 2
1 kg of water initially at 10ĈC is given heat and then converted into steam, then find the
increase in the internal energy.
LV = 2.5 ï 106 J/kg
steam = 0.5 kg/m3
Solution :
Q = ms + mLV
= 1 ï 4200 ï 90 + 1 ï 2.5 ï 106 J
V = vol. of steam ă vol. of water
1 1
= 3 2m3
0.5 10
W
PdV
vi A
THERMODYNAMICS
QUIZRR 9
P V P
Vf
Vi
V P V
O Vi Vf O O
clockwise arrow
given +ve work
(1) Isochoric Process : The process during which the volume of the gaseous system remains
constant, throughout entire process, is called isochoric process.
V = const
(i) W = PdV 0
(ii) Q = U
P T P
f f f
i i i
V V T
O O O
In the P-T graph [ PV = nRT]
slope 1 , slope n P
V
a
n R V2
Slope = [ PV = nRT] V1 b
V
THERMODYNAMICS
10 QUIZRR
Example 3
2 mol of an ideal gas is taken through a process PT2 = K. Find the work done by the gas
from v0 to 2v0 volume expansion, if initial pressure = P0.
Solution :
PT2 = K PV = nRT
2
PV
P K
nR
P3 V2 = n2R2K
1/3
n2 R 2 K
P
V 2
2V0
W
V0
pdv
2V0
n R K
1/3
=
2 2
V 2 / 3 d V
V0
2V0
V
2 2 1/3 2 / 3
= n R K dV
V0
2V0
3K 2V0 V0
= 3K V1 / 3 1/3 1/3
V0
= 3K V0 2 1
1/3
(2) Isobaric Process : The process during which the pressure of the gaseous system remains constant
throughout the entire process.
P = const.
THERMODYNAMICS
QUIZRR 11
Pd V VdP nRdT
(i) W
PdV P d V PV = (nRT) d W PdV nRd T
W PV nRT
(W = PV) = nRT)
(ii) P P
i f i f
V T
O O
f nR
slope
P
n
slope
i P
T
O
V V
n1 = n2 P1 = P2
P1 n1
1 1
P2 n2
2 2
T T
O O
(P1 < P2) (n1 > n2)
The process during which the temp. of the gaseous system remains constant during the entire
process.
T = const
THERMODYNAMICS
12 QUIZRR
(i) Process equation = (PV = K)
K
d V K l n V V f
V
W PdV
V i
Vf
W = nRT log e
Vi
P
W nRT l n i { PiVi = PfVf = K }
Pf
P
W Pi Vi l n i
Pf
(iii) U = 0 [ T = const]
Q = W
PV = K
P K dP
d V 1 KV O
2
i P
V
d 2P 3
2 KV O
2
f d V
V
O
P V
f f
i i
T T
O O
THERMODYNAMICS
QUIZRR 13
n2
n1
V
O V
Cyclic Process
The process during which the initial and the final states of the gaseous system are same, is called
cyclic process.
Cyclic process is represented as a closed loop on the P-V diagram.
(Ti = Tf)
P
(ui = uf)
B C U = (uf ă ui) = 0
Qcyclic = Wcyclic
A D
V
O
CW W < 0
CCW W > 0
P
O
THERMODYNAMICS
14 QUIZRR
Example 4
Find the work done by an ideal gas during cyclic process (1432) as shown :
P3 = 4 ï 105 Pa P2
3 4 (V4 ă V3) = 5l
P0 = 3 ï 105 Pa P3
P1 = 105 Pa
P0
1 2
P1
(V2 ă V1) = 10l
Solution : V
V1 V2
1 1
Area of (102) = b h 2 105 10 103 CW
2 2
= (+ 103 J)
1
Area of (304) = ï bï h
2
1
= ï (5 ï 10ă3) ï 105 [Obtain, base length from similar triangle concept]
2
Reversible Process P
f
A process is said to be reversible, if, it is :
(i) quasi-static
(ii) Non-dissipative
(i) Quasi static : If unique value of P, V and T can be specified
i
to a gaseous system at certain moment, then system is said
V
to be in thermodynamic equilibrium. O
But, in practical, no system can exactly achieve thermodynamic equilibrium because the
parts of the system do not at once achieve, the same value of P, V, T.
If the process is carried out very slowly then system will be in almost thermodynamic
equilibrium and this process is called quasi-static process.
(ii) Non-dissipative
If no heat dissipation takes during the process then it is called non-dissipative.
THERMODYNAMICS
QUIZRR 15
CP
dQ P CV
dQ V
,
ndT nd T
du nC V d T
At const. pressure
(dQ)P = dU + dW
nCP dT = nCV dT + PdV
nCPdT = nCV dT + nRdT
CP = CV + R
CP
(CP ă CV) = R
CV
R R
CV CP
1 1
Note : CP ă CV = R/J when CP and CV are is cal and J is mechanical heat equivalent
J = 4.18 J/cal
Remember :
dU = nCVdT (Applicable for all process)
Example 5
2 3
V
O
THERMODYNAMICS
16 QUIZRR
Solution :
From the diagram its, obvious that
W1 > W2 > W3
U1 = U2 = U3 (final, initial state)
Q1 > Q2 > Q3
C1 > C2 > C3
Adiabatic Process
The process during which is no heat exchange between the system and the surrounding for the
entire process is called adiabatic process.
dQ = 0
[do not write it as Q = 0, This is because in a process, the net heat change may be zero but in
an adiabatic process, there is no heat change any step of the process]
dQ = 0
dQ = dU + dW
0 = nCVdT + PdW
R nRT
n dT dV 0
1 V
dV dT
1
V T
lnT = ă ( ă 1) ln V + ln K
ln T + lnV( ă 1) = ln K
TV(ă1) = K
(Use gas equation)
PV = K
Process equation
PV = K ...(1)
ă1
TV =K ...(2)
P1ă T = K ...(3)
THERMODYNAMICS
QUIZRR 17
f d 2P
2
1 KV 2 0
dV
V
O
T TVă1 = K
i
dT
1 KV 0
dV
f d2T
2
1 KV 1 0
dV
V
O
P P1ăT = K
i
1
1
P = K T
dP 1
KT 0
dT 1
f
T
O
d2P 1
KT 0
dT 2
1 1
Example 6
If CP a KT a = 27.68 J/mol/K
THERMODYNAMICS
18 QUIZRR
n RT RTd V
nCVdT + dV 0 n b KT d T n
V V
T2 V2
b KT dV
T1
T
dT R
V1
V
Polytropic Process
V2 V2
PdV K V
W dV
V1 V1
P2 V2
W for (x 1)
1x
dQ = dU + dW
nCdT = nCVdT + PdV
nRT
nCdT = nCVdT + dV
V
RT d V
C = CV +
V dT
PdV
C CV ...(1)
nd T
THERMODYNAMICS
QUIZRR 19
This eqn. to be used when we have to obtain following : (in any general process)
molar heat capacity process eqn. process eqn. molar heat capacity
Now, PVx = K
VxdP + (xVxă1 dV) P = 0
VdP + (xdV) P = 0
VdP = ă x PdV
we have from gas eqn.
(PdV + VdP = nRdT)
PdV (1 ă x) = nRdT
PdV R
nd T 1 x
Substitute in (1)
R
C CV
1 x
R
C CV
1x
Example 7
Find the values of polytropic const. x for which specific heat of the gas for the process is
negative.
Solution :
R
C CV
1 x
R R
=
1 1 x
R 1 x R 1
+ ă +
=
1 1 x 1 y
R x
= C is ăve when x (1, )
1 x 1
[1 > x < ]
THERMODYNAMICS
20 QUIZRR
Example 8
PdV
Use C C V
nd T
Pd V
Cv 3 a T 2 Cv
ndT
Pd V
3 aT 2
nd T
RT d V
3 aT 2
V dT
3a dV
R TdT
V
3a 2
ln V T
2R
Ve
3 a / 2R T 2
3 a / 2R T 2
Ve K
(ii) C = CV + V
Pd V
Again, CV + V = CV
nd T
RT dV
V
V dT
dT dV
R T V 2
1
R l n T V l n K
THERMODYNAMICS
QUIZRR 21
Example 9
Two moles of helium gas undergo a cyclic process as shown. Assuming the gas to be ideal
calculate the following quantities in the process : (a) the net work done (b) the net change
in internal energy (c) the net change in heat energy. (R = 8.32 J/mol K)
Solution :
A
2 atm B
(a) As work done W PdV
P
So at constant pressure, 1 atm C
D
W = P[VF ă VI] = nR[TF ă TI]
...[as for ideal gas PV = nRT; PVF = nRTF and PVI 300 K T 400 K
= nRTI]
VF PI
W = nRT loge = nRT loge
VI PF
[as for constt. temperature, PV = constt., i.e., PFVF = P1VI, i.e., (VF/VI) = (PI/PF)
So work done for path AB, BC, CD and DA respectively will be
WAB = nR[TF ă TI]
= 2 ï R[400 ă 300] = 200 R
WBC = nRT loge (PI/PF)
= 2 ï R ï 400 loge 2 = 800 R loge 2
W CD = nR[TF/TI]
= 2 ï R [300 ă 400] = ă 200R
W DA = nRT loge (PI/PF)
= 2 ï R ï 300 loge (1/2)
= ă 600R loge 2
and hence, the work done in the complete cycle,
W = WAB + WBC + WCD + WDA
= 200R loge 2 = 200 ï 8.32 ï 0.693
= 1153 J
THERMODYNAMICS
22 QUIZRR
(c) Now according to first law of thermodynamics
Q = U + W
So substituting the values W and U from parts (a) and (b) in the above,
Q = 0 + 1153 J = 1153 J Ans.
Example 10
Three moles of an ideal gas [CP = (7/2)R] at pressure P and temperature T is isothermally
expanded to twice its initial volume. It is then compressed at constant pressure to its
original volume. Finally the gas is compressed at constant volume to its original pressure
(P). (a) Sketch P-V and P-T diagram for the process. (b) Calculate the work done by the gas
and heat supplied to the gas during the complete cycle.
Solution :
(a) Initially the state A of the gas is described by (P, V, T). As for isothermal change T = constt.
with PV = constt., so the state B will be [(P/2), 2V, T]. Now as for isobaric change P = constt.
and V T, so the state C will be [(P/2), V, (T/2)]. So the P-V and P-T curves will be as shown
in Fig. (A) and (B) respectively.
VF
Wiso = nRT loge = 3RTloge 2 = 2.08RT
VI
T
= 3R T 1.5 RT
2
A(P, V) A (P, T)
P P
P P
(P/2) C B C B
(P/2, V) (P/2, 2V) (P/2, T/2) (P/2, T)
V 2V V (T/2) T T
(A) (B)
Example 11
Kx 5 8000 0.1
And finally, PF = P0 + A 1 10
8 10 3
= 2 ï 105 N/m2
VF = V0 + Ax = 2.4 ï 10ă3 + 0.1 ï 8 ï 10ă3
= 3.2 ï 10ă3 m3
So from gas equation PV = nRT, i.e., Open
P1 VI PF VF
TI TF Atmosphere
PF VF
we have TF = P V TI
I I
= 800 K Ans.
0.1
kx 0.1
So W = 0
P0 A Adx
0
P0 A kx dx
1 2
i.e., W = P0Ax + kx
2
THERMODYNAMICS
24 QUIZRR
1
= [105 ï 8 ï 10ă3 ï 0.1 + ï 8000 ï (0.1)2]
2
i.e., W = 80 + 40 = 120 J
nRT PF VF PI VI
and as U nC V T
1 1
3
U 102 [6.4 2.4] 600 J
2
Example 12
One mole of a monatomic ideal gas is taken through the cycle shown in Fig.
A B adiabatic expansion
B C cooling at constt. volume
C D adiabatic compression
D A heating at constt. volume
A
The pressure and temperature at A, B, etc., are denoted by
B
PA, TA; PB, TB, etc. respectively. Given TA = 1000K, PB = (2/
3)PA and PC = (1/3)PA. Calculate (a) the work done by the gas P D
in the process A B, (b) the heat lost by the gas in the C
process B C and (c) temperature TD. Given (2/3)2/5 = 0.85
V
and R = 8.31 J/mol K.
Solution :
(a) As for adiabatic change PV = constt.,
r
nRT
i.e., P constt. [as PV = nRT]
P
T
i.e. constt.
P 1
THERMODYNAMICS
QUIZRR 25
1
TB PB 5
So with
TA PA 3
1
1 2/5
2 2
i.e., TB = TA 1000 850 K ...(1)
3 3
3 3
or Q = 1 R (TC 850) as C V R
2 2
PC TC
i.e., = ,
PB TB
1/3 PA TB
TB
850
425 K
i.e. TC = ...(2)
2 / 3 PA 2 2
3
So Q = 1 8.31 425 850 5297.625 J
2
PD TD TD
, i.e. PD PA
PA TA TA
1 1
TD PD P T
A D
TC PC PC TA
THERMODYNAMICS
26 QUIZRR
1
1
1/ P
or TD TC A
PC TA
2/5
T PA
i.e., TD3/5 B
2 1 / 3 PA 1000
1 2 2 / 5 3 2 / 5
i.e. TD3/5 1000
2 3 1000
THERMODYNAMICS
QUIZRR 27
TEMPERATURE SCALES
TC = T ă 273.15
Equation shows the relation between the temperatures in celsius scales and kelvin scale. Because
the size of a degree is the same on the two scales, a temperature difference of 10ĈC is equal to
a temperature difference of 10 K. The two scales differ only in the choice of the zero point. The
ice point temperature on the kelvin scale, 273.15K, corresponds to 0.00ĈC and the Kelvin steam
point 373.15 K, is equivalent to 100.00ĈC.
A common temperature scale in everyday use in US is the Fahrenheit scale. The ice point in this
scale is 32ĈF and the steam point is 212ĈF. These two are divided in 180 equal parts. The relation
between Celsius scale and Fahrenheit scale is derived below.
0Ĉ C 100Ĉ C
(100 equal parts)
9
1 part of Celsius scale = parts of Fahrenheit scale
5
9
Hence TF = 32 + T ...(ii)
5 C
5
Further TC = T = TF ...(iii)
9
K C
273 0Ĉ C 32Ĉ C
THERMODYNAMICS
28 QUIZRR
(I) Thermal Expansion :
With the increase in temp., due to increase in amplitude of vibrations of atoms (on molecules) the
body expands on heating.
(1) Coefficient of linear expansion () :
The fractional charge is length for 1ĈC (or 1 K) change in temperature is called coefficient
of linear expansion ().
l
(Average value)
l
l = l0
l ă l0 = l0
l = l0 (1 + )
Absolute Value :
l
lim
0 l
dl 1
l d
l
dl
l0
l
d
0
l
ln
l0
0
d a (if is constt.)
1 l
ln
l0
l x2 x3 x4
(approx. l n 1 x x ... )
l0 2 3 4
l
[Avg. value]
l0
THERMODYNAMICS
QUIZRR 29
R emember
A
(avg. value)
A
A = A0 (1 + )
Absolute value :
A dA 1
lim
0 A A d
A
dA
A0
A
d
0
V
(avg. value)
V
The fractional change in volume for 1ĈC (or 1 K) change in temperature is called coefficient
of volume expansion.
V = V0 (1 + )
Absolute value :
V dV 1
lim
0 V V d
V
dV
V0
V
d
0
THERMODYNAMICS
30 QUIZRR
Relation between , and :
: : = 1 : 2 : 3
Derivation :
In a solid object having length, breadth and height l0, b0 and h0, is being heated upto .
l = l0(1 + ); b = b0 (1 + ); h = h0 (1 + )
A = lb = l0b0 (1 + )2 = l0b0 (1 + 2) = l0b0 (1 + )
[using binomial]
= 2
V = lbh = l0b0l2 (1 + )3 = l0b0h0 (1 + 3) = l0b0h0 (1 + )
= 3
: : = 1 : 2 : 3
l0
1 d
2 d
Let us consider a bimetallic strip of length l0 and both metals having breadth d joint together.
The 2 metals have coefficient of thermal expansion 1 and 2 respectively (1 > 2)
Now, when the temperature of this strip is increased, then this strip has the characteristic property
of bending due to unequal linear expansion of the two metals.
R
R+d Răd
2 2
Bending will take place in such a manner that the one with higher coefficient () will be on
convex side.
THERMODYNAMICS
QUIZRR 31
l1 = l0 (1 + 1)
l2 = l0 (1 + 2)
Since both subtend same angle at the centre,
l1 l2
d d l1
R R
2 2
l2
1 1 R d / 2
1 2 R d / 2
neglected
2 1 2 2R
1 2 d
2 2R
=
(1 2 ) d
d
R
1 2
TV = SR (1 + Apparent )
THERMODYNAMICS
32 QUIZRR
(3) Time gain/loss :
If a pendulum clock keeps proper time at temperature , at temperature ´ (> ) due to linear
expansion, length of pendulum and hence time period will increase. Due to this increase in time
period, in a given time interval (say t), the pendulum will exclude (t/T) oscillations instead of (t/
T´) and so will lose [(t/T) ă (t/T´)] oscillations. Now, as measurement of time is through the
counting of oscillations, so in time t, a clock with time T´ will lose time by
t t T´
t = T´ t 1
T T´ T
l0 l0 1
also T = 2 , T´ = 2
g g
T1
1
T
t = t 1 1
Using Binomial expansion
1 1
t = t 1 1 = t
2 2
t 1
t 2
1
or fractional loss/gain in time =
2
Example 13
Find the time loss in 1 year if temp of a pendulum clock is increased by 100ĈC ?
( = 10ă5ĈCă1)
Solution :
1
Fractional time loss =
2
THERMODYNAMICS
QUIZRR 33
1
time lost in 1 year, t = ï 365 days
2
1 5
= 10 100 365 days.
2
F , A, l F
Ć Due increase in temp., the rod tries to expand against the wall, but is prevented and the
compressive stress hence developed is called thermal stress.
Ć Similarly, due to decrease in temperature, tensile thermal stress develop which prevent the
rod to contract.
YA
Reminders : Stiffness corresponding to a rigid body K =
l
YA
F l Hence, you can treat it as spring with K.
l
Fl
Due to stress developed contraction produced l
YA
But the net expansion is zero. Hence,
Fl
( l ) + 0
YA
Fl
l
YA
F = AY
F
Thermal Stress = Y
A
THERMODYNAMICS
34 QUIZRR
Example 14
Three rods are joined together and kept fixed A1,Y1,1 A2,Y2,2 A3,Y3,3
between two rigid supports. If the temp. of the F F
system is increased by then find, l1 l3
l2
(i) force developed at junctions
(ii) length of each rod finally
(iii) displacement of joints.
Solution :
Everywhere, throughout at all functions, same force is developed.
L = 0
l l l
l11 l2 2 l3 3 F 1 2 3
A1 Y1 A 2 Y2 A 3 Y3
(1 l1 2 l2 3 l3 )
(i) F
l1 l l
2 3
AY1 AY2 A 3 Y3
Fl1
l1 f l1 1 [F obtained from (i)]
A1 Y1
l 1 + l 2 + l 3 = 0
THERMODYNAMICS
QUIZRR 35
Example 15
Two steel rods and one aluminium rod of equal length and cross-sectional area, l 0 and A0,
are riveted together at their ends.
Assume all the rods are in the state of zero tension at 0ĈC. Find the length of each rod after
the temp. is increased by .
l0, A0
steel F (steel) F
Al 2F Al 2F
steel F (steel) F
Solution :
lAl = lsteel
l0 + lAl = l0 + lsteel
(lAl = lsteel)
2F l0 Fl0
Al l0 steel l0
AYAl AYsteel
2F F
Al steel
AYAl AYsteel
A Al steel
F
2 1
Y
Al Ysteel
2F l0
lAl = Al l0 ă
AYAl
F l0
and lsteel steel l0
AYsteel
THERMODYNAMICS
36 QUIZRR
(5) Thermal expansion of an isotropic body is imagined as a photographic enlargement.
i.e., if there is a hollow cavity (in case of solids) or hole (in case of lamina), then volume of the
cavity (or hole) will increase, just like if it were supposed to be filled with the same material.
V0 , V, A
A0
apparent = L ă S
(1) If L > s
(2) If L < s
(3) If L = s
THERMODYNAMICS
QUIZRR 37
m m m
0 ,
V0 V V0 1
1
0 1
0
1
Further,
= 0 (1 + )ă1 = 0 (1 ă )
= 0 (1 ă ) [This is an approx result]
Note : Also, sometimes in some questions, you do not get correct answer by just applying the
above eqn. for a given temp. range.
In that case, start for initial condition at T = 0ĈC
70 1 70
20 1 20
W´ = W ă B Apparent Wt.
B0 = 0V0g
B = Vg
B V 0 1 L V0 1 s
B0 0 V0 0 V0
B (Buoyant force)
B
1 L 1 s
B0
THERMODYNAMICS
38 QUIZRR
B
1 L s L s 2
B0
L > s generally
B < B0
with the increase in temp., buoyant force decreases and hence the apparent weight increases.
Example 16
A cubical block is floating in a bath with length x, immersed in liquid. When temp. is
increased by , the block keeps floating with same length immersed in liquid. Find the
relations between and .
x
= coefficient of Linear exxpansion of cube
= coefficient of volume of Liquid
Solution :
B0 = Initial buoyant force
= (0 A0 x g) = mg
B = ( A x g) = mg
B = B0
A xg = 0A0 x g
1 + (2 ă ) = 1
2 =
= 2 Ans.
THERMODYNAMICS
QUIZRR 39
Example 17
P Q
A
h2 = 51 cm
h1 = 52.8 cm h = 49 cm
C D
B
95Ĉ C 5Ĉ C 95Ĉ C 5Ĉ C
Solution :
Equate pressure in a horizontal level in stationary liquid.
PP = PQ
Patm + h195Ĉ g ă h5Ĉ g = Patm + h2 5Ĉ g ă h95Ĉ g
95Ĉ [h1 + h] = 5Ĉ (h + h2)
5
5
h h2
1 90 h h1
51 49 100
(1 ă 90) =
52.8 49 101.8
100 1.8
90 = 1
101.8 101.8
1.8
90 101.8
<0
<0 >0
OĈ C 4Ĉ C T OĈ C 4Ĉ C T
THERMODYNAMICS
40 QUIZRR
C alorimetry
Q
C´ Q = C´T (Avg. value)
T
dQ
Absolute Value : C´
d T
dQ = C´dT
T2
Q
C´ dT
T1
Take the limits in given range with proper scale (0ĈC or K)
Q
C Q nC T Avg. value
nT
T2
Absolute value Q
nC d T
T1
units : J/mol/K
Q
S Q = msT (Avg. value)
mT
T2
Q
msdT
T1
(Absolute value) unit : J/kg/K
THERMODYNAMICS
QUIZRR 41
Q
L unit : (J/Kg)
m
Q = mL
Example 18
An iron cube is initially heated to a high temperature and placed on a large ice cube at 0ĈC.
The block melts the ice below it, displaces water and sinks. In the final equilibrium, the
upper surface of iron cube just goes inside the ice. Then find the initial temperature of the
block.
iron = 8000 kg/m3
THERMODYNAMICS
42 QUIZRR
Siron = 470 J/kg/K
ice = 900 Kg/m3
LF = 3.36 ï 105 J/kg.
Solution :
V
= ~ 80ĈC
Example 19
1 kg of ice at 0ĈC is mixed with 1 kg of steam at 100ĈC. Then find the composition of mixture
in equilibrium
LF = 3.36 ï 105 J/kg
LV = 2.26 ï 106 J/kg
SW = 4200 J/kg/K
Solution :
In such question, we have to consider three cases :
Case (i) : The final temp. is 0ĈC
The entire steam condenses to water at 0ĈC.
Case (ii) : The entire ice converts to water at 100ĈC.
Finally temp. : 100ĈC.
Case (iii) : Final temp is where 0 < < 100ĈC
In this case, it can be easily decided that final temp. of the mixture is 100ĈC.
THERMODYNAMICS
QUIZRR 43
Now, heat gained by ice is converting from ice to water at 0ĈC and then water at 100ĈC.
= mL + mS
= 1 ï 3.36 ï 105 + 1 ï 4200 ï 100
= 7.50 ï 105 J
Total heat that can be lost by steam = mLv = 1 ï 2.26 ï 106 J
Total heat that can be lost by steam to convert to water at 0ĈC > heat required
7.56 105
mass of steam condensed = 6
0.335 kg
2.26 10
Example 20
An ice cube of mass 0.1 kg at 0ĈC is placed in a thermally insulated container at 227ĈC.
Given that the specific heat of container varies as S = A + BT, where A, B are constants,
then find the mass of the container if final temperature is 27ĈC.
Lf = 8 ï 104 cal/kg, S = 1000 cal/kg/K,
A = 100 cal/kg/K
B = 2 ï 10ă2 cal/kg/K2
Solution :
Heat gained by ice in converting to water at 0ĈC and then to water at 27ĈC.
Q 1 = mLf + mS
= 0.1 ï 8 ï 104 + 0.1 ï 1000 ï 27
Heat lost by container
T2
= ă T1
mS dt
300 300
=ă 500
m A BT d T m
500
A BT dT
A BT dT 0.1 8 104
300
m
500
0.1 1000 27
m = 0.495 kg.
THERMODYNAMICS
44 QUIZRR
Example 21
A thermally isolated container contains 100 gm of water at 0ĈC. When the air above the
water surface is pumped out, some of the water evaporates and remaining freezes. Then
find percentage of vapour evaporated.
Solution :
OĈ C
Ice
Concept : Water evaporates due to decreases in pressure above the water surface.
Since B.P. is the temperature at which vapor pressure = Patm
due to decrease in VP, water evaporates
Heat gained by water in evaporation
= Heat lost by water which converts it to ice
Let x be the amount of water evaporated
x ï LV = (100 ă x) ï Lf
540 x = 80 (100 ă x) [x is cal]
620x = 80 ï 100
80 100
x
620
400
x
31
x ~ 13 gm
x
Fraction = 0.13
100
mLv = (M ă m)Lf
m Lf
M Lv Lf
evaporated L f condensed L v
,
condensed L v L f evaporated L f L v
THERMODYNAMICS
QUIZRR 45
Example 22
A loaded glass bulb weighs 156.25 g in air. When the bulb is immersed in a liquid at
temperature 15ĈC, it weighs 57.5 g. On heating the liquid, the apparent weight of the bulb
becomes 58.57 g. Find the coefficient of real expansion of the liquid. (Given, coefficient of
linear expansion of glass = 9 ï 10ă6/ĈC).
Solution :
When the bulb is immersed in liquid, the loss in its weight is due to upthrust of liquid and is equal
to weight of liquid displaced.
Let m1 and m2 represents the loss in weight of bulb at lower and higher temperatures t1 and t2
respectively; v1 and v2 the volumes of bulb and 1, 2 the densities of the liquid at these
temperatures respectively. Then,
m1 = 1v1 and m2 = 2v2
1 m1 v2
. ...(1)
2 m2 v1
1
and [1 R t2 t1 ] ...(3)
2
v2 1
Substituting for and from eqns. (2) and (3) in (1),
v1 2
m1
[1 R t2 t1 1 g t2 t1
m2
...(4)
98.75
[1 + R(52 ă 15) = [1 + 27 ï 10ă6 ï (52 ă 15)]
97.68
THERMODYNAMICS
46 QUIZRR
KINETIC EQUATION AND EQUIPARTITION OF ENERGY
2R cos
the time between 2 collisions with the wall of the container can be given by t
v
=
mN 2
R
vrms
mN 2
Pressure exerted = vrms v
4 R .R
2
R R
v
m N vrms
2
P =
4R 3
mN 2 1 mN 2
P vrms vrms
2 V
4 = (Kinetic equation of gas)
3 R 3
3
1 mN 2 1 2 1 M´ 2
P vrms vrms vrms
3 V 3 3 V
3P 3RT 3 KT
vrms
M in kg m
THERMODYNAMICS
QUIZRR 47
R ă23
where K Boltzmann constant = (1.38 ï 10 J/K)
NA
2RT 2P 2KT
vmp
M m
1
= 2
m v12 v22 ... vn2
N
1 2 1 3KT 3
= m vrms m KT
2 2 m 2
3
Avg. translational KE of each molecule = KT
2
Now,
3
2
1
2
2
KT m vrms
=
2 2 2 2
2 2
1 [(v1 x v2 x v3 x ... vx ) v1 y .... vny ]
m
2 N
3 1 1 1
KT = m v0x m v2y m vz2
2 2 2 2
1 1 1 1
m v2x m v2y vz2 KT
2 2 2 2
THERMODYNAMICS
48 QUIZRR
Law of Equipartition of Energy
1
On average, KT energy can be associated with each degree of freedom.
2
Degree of Freedom :
The number of independent modes of motion of a physical system is called degree of freedom.
Y
(1) For monoatomic gas :
f = total degree of freedom
f = 3 translational
X
f = 3
Z
(2) For diatomic gas (also for linear polyatomic)
f = 3 trans + 2 rotational
Y
(rotation along the bond axis) is neglected
f = 5 at ordinary temp.
= 6
f = 6 At low temp.
X
At high temp. f = 3 trans + 3 rotat + 2 vib. O
f = 8
Z
THERMODYNAMICS
QUIZRR 49
1
U f KT for 1 molecule
2
fR
for n moles of a gas U n N A f KT n
1
T
2 2
dU nfR / 2 fR
CV dT =
ndT n dT 2
fR
CV
2
fR f
CP R 2 1 R
2
CP f 2 R 2 2
1
CV 2 fR f
2
1
f
3R 5R 5
CV = , CP = ,
2 2 3
5R 7R
CV , CP , 7 / 5
2 2
THERMODYNAMICS
50 QUIZRR
Similarly, if you increase temperature by dT at constant pressure you get
for (mixture ),
CP mix
mixture
CV mix
n1 R n2 R n3 Rd T R dT
dT + dT + n1 n2 n3
or,
11
21 31 mix1
n1 n2 n3 n1 n2 n3
mix1
11 21 31
Example 23
3R 5R
n1 C V1 n2 C V2 1 1.5
CV(mix) = 2 2
n1 n2 2.5
= (2.1 R)
CP (mix) = CV (mix) + R = 3.1 R
3.1
1.4
2.1
THERMODYNAMICS
QUIZRR 51
I mportant Concepts
(I) From the given graph, decide whether the volume increases or decreases
P
P nR
tan
T V
T) B
(P,
1 A
slope V
As slope decreases from A to B, T
O
the volume of the gaseous system undergoing the process increases.
P
e.g. Note the from B to C first the volume increase
A B and then decrease (check the slope)
2P0
P0
C
T
O T0 2T0
(II) If the PV diagram is a straight line with negative slope then T, V graph is a parabola.
Write the equation of the straight line
P0
P ă 2P0 = V V0 P
V0
2P0 A
P0 V P
P 3P0
V0
P0 B
nRT P0 V
3P0 O V0 2V0
V V0
T
P0 2
nRT V 3P0 V
V0 T0
A B
P0 V 2 3P0 V
T
nRV0 nR
O V0 3V 2V0
2
THERMODYNAMICS
52 QUIZRR
I llustration
RT
K = slope of the gas = constant
M
Hence, T is also constant. Thus it is isothermal process. O
Wcycle
= 100 %
Qsupplied
CWcycle
= Q 100 %
supplied
Example 25 P
2 mol. of an ideal monoatomic gas is taken through a cyclic
process, ABC as shown on PT diagram during process AB 2P1 B C
(PT = const.). Find :
(a) Show the cycle on PV, and VT diagram
P1 A
(b) Work done by the gas during process AB
(c) Heat absorbed or released during each process. Also
find the efficiency of the cycle. O T1= 300K 2T2 = 600K
Solution :
For AB : PT = K P
(isobaric)
P2 V 2P1 B C
K
nR (isothermal)
PVó = K (polytropic process) P1
A
1
x= V
2 O
THERMODYNAMICS
QUIZRR 53
V A
dP 1
KV 3 / 2 0
dV 2
d 2P
2
0 {To check the slope }
dV B
V2
W
PdV
V1
AB is a polytropic process
Use, W P1 V1 P2 V2 P2 V2 P1 V
x 1 1 ó
W = 2nR (T2 ă T1) = 2 ï 2 ï R ï 300 K
25
= 22 300 104 J
3
3R R 3R 7R
C 2R =
2 1 ó 2 2
5 25
Now QBC = 2 R T R J/mol/K
2 3
= 5 ï R ï 300 = 1500 R
P1
QCA = nRT ln P = 840 R
2
240 R
efficiency ( ) = 2340 R 100
THERMODYNAMICS
54 QUIZRR
Example 26 V
VD D C
2 mol of an ideal monoatomic gas is taken through a cyclic
process ABCD as shown on V-T diagram.
(a) Show the cycle on P-V and P-T diagram VB B
(b) Heat evolved (or absorbed) during each process
(c) of the process VA
A
VD VB
Given that : 4, 2 O 300K 600K T
VA VA
Solution :
AB is isoboric (V = KT)
BC : isothermal
CD : isochoric
DA : isothermal
P P
A B A B
C
C
D
D
V V
O VA VB VD O (300K) 600K
5 25R
(b) QAB = n R T 300
2 2
QAB = 1500 R J
VD
QBC = nRT ln = 2 ï R ï 600 ï ln2
VB
= 1200Rln2 J
3
QCD = nCVT = 2 R 300 900
2
VA 1
QDA = n RT ln V 2 R 300 l n 4
D
THERMODYNAMICS
QUIZRR 55
5000
(c)
25 25
1500 1200 0.7
3 3
5000
=
19500
~ 25 %
Tip : When relation between Q, U or W is given then relate Q and U using first law of
thermodynamics and use
Q = nCT
U = nCVT
Example 27 P
3P0 A
2 mol of an ideal monoatomic gas taken through a cyclic
process AB where AB is isothermal and in the process CA,
QCA + WCA = 0 B
P0
(a) Then show the cycle on P-T, V-T diagram
P0/2 C
(b) Heat absorbed or released during each process
V
(c) Also find the efficiency O V0 3V0
Solution :
For the process CA, use
QCA = ă WCA
QCA = ă (QCA ă UCA) [from first law of thermodynamics]
2QCA = uCA
2nCT = n CV T
C 3R
C V
2 4
3R
C
4
THERMODYNAMICS
56 QUIZRR
CA is a polytropic process
R
C CV
1 x
3R 3R R 4 7
x 1
4 2 1 x 3 5
V V
3P0 A
3V0 C B
P0 B V0
A
C
T T
O T0/2 T0 O T0/2 T0
3V0
QAB = nRT ln = 3P0V0 ln3
V0
QBC = U = nCVT
3 3 9P V
= nRT VP 0 0
2 2 4
9P0 V0
QBC =
4
QCA = nCT
3 3 3P0 V0
nRT 3P0 V0 2
=
4 4
3 3P0 V0 9P0 V0
4 2 8
=
THERMODYNAMICS
QUIZRR 57
9V0 P0 9V0 P0 9
3P0 V0 ln 3
8 3 ln 3 8
4
= 100 = 100
9V0 P0 9
3P0 V0 l n3 8 3 ln 3
8
Example 28
Air initially at 260ĈC (533 K), P0 = 700 Pa and V0 = 0.028 m 3 is expanded isobarically upto
3V0 = 0.084 m3. Then a polytropic process is followed with n = 1.5 and an isothermal process
is carried out to complete the cycle.
(a) Show the cycle on PV, PT, VT diagram
(b) Heat evolved (or absorbed) during each process
(c) Efficiency of the cycle
Solution :
Remember : Air is diatomic
27V0
P P V C
P0 A B A B B
P0 3P0
P0 C P0 V0
27 A
27 C
T T
3V0 O T0 3T0 O T0 3T0
V0 V
7R 7 7
= n T = PV P0 2V0 7P0 V0
2 2 2
QBC = n CT
C CV
1
R
x
5R
2
R
3
1 /2
5R
2
2R = /
R
2
R P0 V0 3P0 V0
QBC = n T = P0 V0
2 2
THERMODYNAMICS
58 QUIZRR
QCA = WCA
= nRT ln (27)ă1 = nRT ln (1/27)
= ă P0V0 ln 27
= (ă 3P0V0 ln3)
Example 29
1 B
(a) Wcycle = V0 2P0 P0 V0 3P0
2
WAB = 0
WCA = ă (P0V0) P0 C
A
WBC = 2P0V0
V
O V0 2V0
(b) Heat (absorbed) released during each cycle
QAB = nCVT
3 3
= n RT 2P0 V0 3P0 V0
2 2
QCA = nCPT
5 5
= P0 V0 P0 V0
2 2
5
QBC = P0V0 ă 3P0 V0 P0 V0
2
P0 V0 P0 V0
= P0 V0
2 2
THERMODYNAMICS
QUIZRR 59
(c) Note that during the process BC, the temp. initially increases and then decreases
2P0
P 3P0 V V0
V0
2P0 V
P 3P0 2P0
V0
2P0 V
P 5P0
V0
nRT
Replace P
V
nRT 2P0 V
5P0
V V0
2V0 P0 5P0 V
T
nRV0 nR
dT 4P0 V 5P0
0
dV nRV0 nR
5V
V 0
4
P0 V0 25 25 25P0 V0
= =
nR 8 4 8 nR
Example 30
An ideal gas is taken through a process in which U varies as U = a V where a and are
constants. Given, the adiabatic contant of the gas is , find
(i) molar heat capacity of the process
(ii) heat required to increase the internal energy of given by V
(iii) Work done during part (ii)
THERMODYNAMICS
60 QUIZRR
Solution :
(i) U = aV
nCVT = aV
nRT PV
V V
1 1
R R R R 1
C 1 1 1 1 R 1
(ii) U = n CV T
nR
U T nRT = (U) (ă1)
1
Q = nCT
1 1 1 1
= nRT = U 1
1 1
1
= U 1
(iii) W = Q ă U
-1 1
W U 1 U=U
Example 31
1 mol of an ideal gas is taken through a process T = T0 + V
(i) Find the molar heat capacity of the process
(ii) Heat required to increase the volume of gas from V1 to V2
Given that, specific heat of gas at const. process = CP
Solution :
T = (T0 + V)
Use Pd V
C CV
ndT
THERMODYNAMICS
QUIZRR 61
dT dV
1 /
dV dT
P nRT
C = CV + CV
n nV
nR T0 V
C = CV +
n V
RT0 RT0
R = CP
V
C = CV +
V
V2
RT0
= C
V1
P
V
n dV
RT0 V
Q n CP V2 V1 l n 2
V1
Example 32
An ideal gas with adiabatic const. is enclosed in an adiabatic container fitted with an
adiabatic piston of mass m which can smoothly move inside the cylinder. The pressure and
volume of the gas in equilibrium are P0 and V0. If the piston is slightly displaced, then find
the time period of the SHM exclude by the piston.
K1
K2
P0, V0
THERMODYNAMICS
62 QUIZRR
Solution :
y1 + y2 = 2y
y1
y1
2y
P, V
dP P
Note : For any polytropic process, we have x 0
dV V
dP P
x
dV V
P
d P x [ Ady] { dV = ă Ady }
V
P 2
(dPA) = + x A y
V
P0 2
force exerted by gas = x V A y (toward equilibrium position)
0
K 2 K1
from (1) and (2) y1 2y
K2
K K1
y1 2 2y
K2
THERMODYNAMICS
QUIZRR 63
DT = 4 y
K1 K 2
K1 K 2
Net restoring force
x P0 A 2 4 y K1 K 2 d2 y
y ma m
= V0 K1 K 2 dt2
d2 y 1 xP0 A
2
4K1 K 2
2 y
m V0 K1 K 2
dt
m
T = 2 A
xP0 A 2 4K1 K 2
V0 K1 K 2
Example 33
l l l/2 (3l/2)
Q
K P1 P2
(2K) (2K)
n, T
2K (T + T)
K (n, T)
Solution :
Let the heat dissipated through left chamber be Q. Now the left chamber is undergoing isothermal
compression
Q´ = W
Concept : Work done by the right chamber gas is used in increasing the potential energy of the
spring and on work done on the left chamber
THERMODYNAMICS
64 QUIZRR
Q = u + W
1 kl 2
Q = nCVT + 2 4 2 Q´
kl 2
Q´ = Q n C T
4
V
At equilibrium position, P2 A = P1 A kl P2 P1 A Kl
Note :
(1) Whenever a piston is moving slowly or is given as light then net force on piston = 0 and
(2) work done by all the forces on piston = 0
(3)
P1 P2
If in an adiabatic compression, the piston moves inside the container, then work done by the
gas may or may not be zero.
dW = (P2 ă P1) Adx, dW = ă dU
nR T T nRT
A Kl
V2 V1
nR T T nRT
A Kl
A 3l / 2 A l / 2
nR 2T 2T
2T Kl
l 3 3
Kl2 4T
T
nR 3
THERMODYNAMICS
QUIZRR 65
3 Kl 4T Kl
2 2
Q´ = Q ă n R
2 nR 3 4
7Kl 2
Q´ = Q ă 2nRT
4
Example 34
Find the work done by the gas is moving the piston slowly in K
the given arrangement by height h. P0
h
P, A
Solution :
Since, the piston is moved slowly, net force piston = 0
Consider a moment when the piston has displaced up by y
mgh + ky + P0A = PA Alternative method :
h
Work done by all forces on piston = K = 0
W
PA. dy
1
0 ă W + mgh + P0A h + kh2 = 0
2
h
= mg ky P A dy
0
0 W = mgh + P0Ah ă
1
2
kh2
1
W = mgh + kh2 + P0Ah
2
Example 35
A thin piston can freely move inside a horizontal container at both ends, initially divided
into two equal parts each of them contains ideal gas at equal pressure, volume and moles.
Find the work required to increase the volume of larger section by times than that of the
smaller section, by slowly moving the piston isothermally.
Solution :
Fext
V2
=
V2 V1 V1
P0, V0 , n P0, V0 , n
P2 P1
THERMODYNAMICS
66 QUIZRR
Wext = P2 P1 A dx
2V0 2 V0
V1 V2 ...(2)
1 1
W
P
1 P2 Adx
= P dV P dV
1 1 2 2
V1 V2
dV1 dV2
= nRT
V0
V1
nRT
V0
V2
V1 V
= ă nRT ln ln 2
V0 V0
V1 V2
= ă nRT ln 2
V0
V2 1 1
= P0 V0 ln 0 = P0 V0
V1 V2 2 2
1 2
= P0 V0 ln
4
THERMODYNAMICS
QUIZRR 67
Example 36
A thermally insulated horizontal cylinder is initially divided into two equal parts by a thin
conducting piston which can freely move inside the cylinder. Each part contains, an ideal
gas at temp. T0. Find the work required to increase the volume of larger section times the
volume of the smaller section by slowly moving the piston.
Solution :
P0, V0 P0, V0 V2 V1
T0 T0 V2
V1
nR d V1 dV2
2 d T nRT
1 V1 V2
V2 V2
2 dT d V1 dV
1 T
V1
V1
V
V0
2
1
2 T V02 1 2 2
l
n l n l n
1 T0 V1 V2 4
1
T 1 1 1
1 / 2 T T0
T0 4n 1
4 2
THERMODYNAMICS
68 QUIZRR
Example 37
2 mol of an ideal monoatomic gas is enclosed in a vertical adiabatic cylinder fitted with an
adiabatic piston of mass m, which can freely move inside the cylinder. Now a heater supplies
a heat at rate of q J/s. Find the velocity of the piston under isobaric condition.
Solution :
dq dT 5R d T
dt n CP dt 2 2 dt
V
dT
q 5R
dt
dq = dU + dW
q
dq = nCVdT + (PAdx)
dT PAdx
n CP C V
dt dt
dT
2 ï R PA v
dt
2q 2q
5 PAv v
5PA
2q
v
5 P0 A mg
THERMODYNAMICS
QUIZRR 69
HEAT TRANSFER
Heat can be transferred from one place to the other by any of three possible ways.
(1) Conduction : usually in solids
(2) Convection in fluids
(3) Radiation : Vaccum, EM waves
In the first two processes a medium is necessary for heat transfer. Radiation does not require
any medium and is the fastest mode of transfer.
CONDUCTION :
Ć Heat is transferred in conduction by collision with neighbouring particles within the body.
Ć When one end is placed at a higher temperature, atoms or molecules get heat energy and
start oscillating with greater amplitude and it collides with neighbouring molecules and
shares its KE. And in the process heat is transferred from one end to the other end.
Ć Average position of atoms/molecules do not change. (No actual transfers of mass is involved).
Ć In steady state, the heat crossing per unit time through any cross-section is constant. Which
is given by,
Q KA T2 T1
t l
K, A, l
T1 T2 (> T1)
Note :
(1) Area of cross-section should be taken perpendicular to direction of the heat flow.
(2) Length l should be measured along the direction of heat flow.
(3) In the steady state, the temperature of each cross section remains constant, (though it is not
equal to all cross-section)
THERMODYNAMICS
70 QUIZRR
Q KA T1 T2
Limitations of formula
t l
dx
For example :
Q K bh T1 T2
T 1 l
(1)
h
Q K lb T
b
T 2 h
T1 T2
l
(2)
Thermal Conductivity :
V V2
i 1
R
Q T1 T2
In heat conduction we have i
t d / KA
Potential difference Temp. difference
d
Resistance Thermal resistance =
KA
THERMODYNAMICS
QUIZRR 71
T1 T2 i R1 R 2 i(R eq )
l1 l
Req = (R1 + R2) R1 , R 2 2
K1 A K 2 A
l1 l 2 l1 l
2
KA K1 A K 2 A
n l
l1 l2
l dx
i1
i
0
K K eq
n l
l1 l li
2
K1 K 2 k dx
i1 i
K
0
l l
R1 , R2
K 1 A1 K2 A2 l, K2, A1
i1
i = (i1 + i2) Net heat flow / (time)
T1 T2
T1 T2 T1 T2 T1 T2 i2
R eq R1 R2 l, K2, A2
1 1 1
...
R e q R1 R 2
THERMODYNAMICS
72 QUIZRR
(ii) Equivalent Thermal Conductivity
KA K 2 A 1 K 2 A 2
l l l
K A K2 A2
Keq 1 1
K A KdA
i i
A1 A 2 A dA i
Temperature of Junction
To find the temp. of junction, first find the current i, and then use the resistance of the part upto
which temperature is to be calculated.
e.g. T1 ă T´ = i R1
K1, A1, l1 K2, A2, l2
T´ = T1 ă i R1 T1 T2
T´
= T1 T2 T1 R1
l1 l
2
K1 A K 2 A
Example 38
Find the equivalent thermal conductivity of the given arrangement (of two concentric
rods).
Solution :
l
R1
2K a 2 2K a
3K
l 2a
R2
9K a 2
11K
K´
4
THERMODYNAMICS
QUIZRR 73
KA K1 A 1 K 2 A 2
l1 l1 l2
K 2K
a 3K 3a 11K
2 2
4a2 4
Example 39
r
Find the equivalent thermal conductivity when K varies as K K 0 1 for the given rod.
a
Solution :
r
K K0 1
a
Since all the small elements of thickness (dr) are is parallel combination
(dA) = (2r)dr
a a
r
KdA K
0 0
0 1
2rdr
a (dr)
Keq a
a
dA
0
2r dr
0
r
a
a2 a 2
2
3 5 K 0
= K0 2
a2 6
2
5 K0
K eq
3
THERMODYNAMICS
74 QUIZRR
W heatstone Bridge
TA ă TM = iR1 TM ă TB = i, R3
i1 i1
TA ă TN = i2R2 TM ă TB = i2R4
i1 R1 = i2R2 i1 R3 = i2R4 A B
R1 R 2 R1 R 3 i2 i2
R3 R4 R2 R 4
N
Example 40
Example 41
3 identical rods made of same material form an equilateral triangle. Temperature of end A
and Bare maintained at 2T and T respectively. Find the Temperature of the end C.
Solution :
AC and CB are in series and parallel on whole to AB C ()
i R R
iAC AB (2T ă T) = iR
2
i/2
i 2T T 3T
(2T ă ) = R 2 =
2 2T 2 (2T) (T)
A i B
THERMODYNAMICS
QUIZRR 75
Example 42
Find the heat conducted per unit time along the length of the rod.
Solution :
Consider an infinitesimal element dx thickness at a distance x from one end
Q dT
Use : i KA
t dx
A = y2
T
y a b a y
and,
x l
dx
x
l
y a x (T1)
b a
(T1>T2) (dx)
(T2 )
l Q dx
dx dy
b a
t
y2
K
dT
b T2
Q l dy
t b a
a
y2
K dT
T1
Q l 1 1
t b a b a K T1 T2
Q l b a
t b a ab K T1 T2
Q K T1 T2 ab
t l
l
Thermal resistance R
Kab
THERMODYNAMICS
76 QUIZRR
Alternative Method :
(Using thermal resistance)
Note : This alternative method cannot be used when the thermal conductivity K becomes
function of temperature
R l
dx
dR K y
0 0
2
b
l dy L
R
b a Ky
a
2 x
y a
b a
l b a l
=
b a
K ab K ab
Q T1 T2 Kab T1 T2
t l / Kab l
l
Learn : Thermal resistance of a frustum with radius of circular ends a and b =
Kab
Example 43
A uniform rod MN of length l and cross-sectional area A with thermal conductivity given
K0
by K has its two ends at temperature 2T0 and T0. Find
1 2T x
(a) Heat conducted per unit time through the rod.
(b) The temperature of the rod as a function of distance x.
2T0 l T0
M x N
dx
THERMODYNAMICS
QUIZRR 77
Solution :
dT
(a) Heat current i KA
dx
Q K0 A dT
t 1 2T x dx
l T0
Q dT
t x dx K A 1 + 2T
0
0
2T0
Q l2 K0 A 1 4 T0
l n
1 + 2T
t 2 2 0
Q 2 K 0 A 1 4 T0
l n
t 2l2 1 + 2 T0
x T
Q KAdT
t
0
xdx
1 + 2T
2T0
Q x
2
1 4 T0
K 0 A l n
t 2 1 2T
2K 0 A 1 4T0 x2 1 4 T0
2 ln K0 A l n
l 1 2T0 2 1 2T
x2 1 4 T0 1 4 T0
2 ln ln
l 1 2 T0 1 2T
1 4T0
ln
x2 l 2 1 2T
1 4T0
ln
1 2T0
THERMODYNAMICS
78 QUIZRR
Example 44
Solution :
(a) Heat conduction through a spherical shell
Q dT
T KA dr
Q
2 dT
T K 4 r dr
T2
r
b T2 T1
Q dr
T 4r
a
2
K
dT
T1
(T1>T2)
Q b a 1
K T2 T1
T ab 4
Q 4Kab
T1 T 2
T b a
Alternative Method
b
dr
R
K 4r
a
2
1 b a
R
4 K ab
Q T1 T2 T1 T2 4 Kab
t R b a
ba
Learn : Thermal resistance of the spherical shell =
4 Kab
THERMODYNAMICS
QUIZRR 79
(b) When K varies as K =
dr
R
KA
b
dr 1 b
= 4r 4r ln a
a
r
R1
b a
4 Kab
K2
K1
R0
c b
C a 4Kbc
b Req = R1 + R2
Q T1 T2
i
t
Re q
T2
T1 a
THERMODYNAMICS
80 QUIZRR
Q dT
(a) K 2rL
t dr
dr
b T2
Q dr r
t 2rL K dT
a T1 a
T1 T2
Q K T1 T2 2L
t ln b / a
Alternative Method :
b
dr Q T1 T2
R
K 2rL
a
t R e q
=
ln /
b
a
2K L
b
dr b a
R
a 2rL
a
2 L
r
Q 2 L
T1 T2
t b a
Example 45
THERMODYNAMICS
QUIZRR 81
Solution :
At any time instant let, temperature of the disc be T. We have
d Q KA TA T
dt l
dT KA TA T
mS
dt l
350 t
dT KA
300
TA T mS l dt
0
350 KA
ln TA T t
300
mS l
mSl
t l n 100 / 50
KA
Example 46 50Ĉ C
b b
dr dr
R
K 4r 4r
1
ln b / a
2 2 =
a a 4
r
Q T1 T2
t Re q
THERMODYNAMICS
82 QUIZRR
Now divide the time into three intervals
d 50
m Sice
dt R
0C ti
d dt
10C
50
mS R
0
i
50 t
ln
60 mSi R
mSi ln b / a
t1 ln 6 / 5
4
(ii) t2 : time required to melt the ice from 0ĈC to 0ĈC water
mL f 50 0
dt R
t2
mL f m L f ln b / a
50 4a 50
(iii) t3 : time required to increase the temperature from 0ĈC to 10ĈC water
d 50
mS W R
dt
10 t3
d dt
0
50
ms
0
WR
t3 msW R ln 5 / 4
THERMODYNAMICS
QUIZRR 83
Example 47
A cubical box is filled with water. Now a hot sphere of radius r 1 kept at const. temp. of
1000ĈC has a jacket around it of radius (r 2 = er 1) to jacket consist of this concentric spherical
C a
layers of K . Find the time required to increase the
r
temperature from 20ĈC to 30ĈC. 20Ĉ C
Given that :
r2
98 3 4 3
ln a r2 W S W 400 C
97 3
Solution :
Thermal resistance of the jacket :
r2 r2
dr dr 1 lne
R
K 4r C 4r
2
2
4
l n r2 / r1
4 C
r1 r1
r
1
R
4C
Now, consider the heat current flowing from the sphere to water in the box
d 1000
msW
dt R
30C t
d 1
20C
dt
1000
0
msW K
97 1
ln t
98 msW R
98 3 4 3
ln W a 3 r2 S W 4C t
97
400C = 4Ct
t = 100 s
THERMODYNAMICS
84 QUIZRR
Example 48
A body of heat capacity C is connected with two rods whose free ends are kept at constant
t em p of T 1 and T2. Let the initial temp. of the body be T0 where (T1 > T0 > T2). Find the
temperature of the body as a function of time t.
Solution : C1 T
i1 i2
T1 T2
K1, A1, l1 K2, A2, l2
Let the temp. at any instant be T.
i = (i1 ă i2)
d T K1 A1 T1 T0 K 2 A 2 T0 T2
ms
dt l1 l2
dT K1 A1 T1 K 2 A 2 T2 K1 A1 K 2 A 2
ms T0
dt l1 l2 l1 l2
1 Cd T K1 A1 T1 K A T 1 K A K A 1
2 2 2 1 1 2 2 T
C dt l1 l2 C l1 l2 C
dT
a bT
dt
T t
dT
T0
a bT dt
0
ln
a bT bt
a bT0
(a ă bT) = (a ă b T0) eăbt
a a b T0 e0bt
T
b
THERMODYNAMICS
QUIZRR 85
C ONVECTION
T0
where
A surface area of the body is contact with the fluids.
h convection co-efficient which depend on the nature of the fluids in contact with the surface.
RADIATION
U
E
A t
units : W/m2
ă The radiant energy is emitted in all wave length range from 0 to , however, only a small
range corresponds to larger part of the radiant energy.
(2) Spectral Emissive Power (E ) : The radiant energy emitted per unit time, per unit area
per unit wavelength is called spectral emissive power.
THERMODYNAMICS
86 QUIZRR
ă In the other words, the emissive power for a particular wavelength is called spectral emissive
power (E).
dE
E
d
or E
E
0
d total emissive power
Note : The incident radiant energy on the surface of the body is divided into 3 parts :
(i) Absorbed part, (ii) Transmitted part (iii) Reflected
Also, + + = 1 for any body having incident radiant energy where absorptivity,
transmitivity, reflectivity
da
a
d
a
a
0
d total absorptive power
THERMODYNAMICS
QUIZRR 87
Eb Eb
Ebb or ab
ab Ebb
Eb
Also, Ebb
ab
Proof :
Consider two bodies, one grey and the other a perfect black (u)
body, enclosed in an isolated container, which have identical
dimension and are in thermal equilibrium
In thermal equilibrium,
Amount of radiant energy falling on black body is b bb
completely reflected black ( its temperature is constant)
U abU
Ebb , Eb A t [Energy emitted by body its temperature is constant]
A(t)
Eb abEbb
Eb
Ebb
ab
(6) On the base of quantum theory of radiation, plank has shown that the energy distribution
2hc2 1
in the spectrum of black body radiation is given by : R
5 ech / KT 1
THERMODYNAMICS
88 QUIZRR
R
T3
T2 T3 > T2 > T1
T1 and m3 < m2 < m1
m2
m3
O m1
(i) The area under the graph curve, gives the total intensity of radiation, (radiancy) at
a particular temperature.
R
R d
0
(7) WeinÊs Law : The wavelength for maximum intensity of radiation is inversely proportional
to the temp. of the surface
1
m
T
(8) StefanÊs Law : The radiant energy emitted per unit time by the surface of the body is given
by
u = e AT4
A total surface area of the body exposed to the surrounding
T absolute temp. of the surface of the body
THERMODYNAMICS
QUIZRR 89
Eb
ab
Ebb
E AT 4 a E = (ab)
4 b
(AT )
(2) The heat energy lost per unit time by a body t0 is given by :
T = (T ă T0)
T << T0 (in K)
T0
Heat const. per unit time due to convection,
u1 = hA (T ă T0)
T
Heat lost per unit time due to radiation
u2 = eA (T4 ă T04)
T
4
= eA T04 1 1
T0
T << T0
THERMODYNAMICS
90 QUIZRR
4 T
= eA T04 1 1
T0
= 4 eAT0 T
3
Total heat loss per unit time
u = u1 + u2
= (h + 4eAT03)A (T ă T0)
dT
ms
dt
h 4 eAT0 A T T0
3
d T h 4 e AT0
3
ă A T T0
dt ms
dT
ă K T T0 (in Kelvin)
dt
K cooling constant
h 4 eT03
= A
ms
d
dt K 0 in celsius
t
d
i
0
K dt
0
ln 0
Kt
i 0
0 i 0 eKt
THERMODYNAMICS
QUIZRR 91
Cooling curve
i
d
dt tan
= ă K ( ă 0)
0
tan = K ( ă 0)
t
tan
K
0
to calculate the value of K, use the tangent of the slope at point and the temp. at that
instant.
Example 49
d
K 0
dt
2 t
d
1
0
K dt
0
2 0
ln Kt
1 0
ln (0.1) = ă Kt
Kt = ln 10
2.303
t
K
THERMODYNAMICS
92 QUIZRR
General concept involved in questions based on conduction and radiation.
In most of the questions it is to be used that the heat conductive per unit time due to conduction
is equal to loss through radiation.
Ts
eg.
T
l
T0
In steady state,
KA T0 T
l
Ae T 4 Ts4
THERMODYNAMICS
FLUID MECHANICS
QUIZRR 3
FLUID MECHANICS
Introduction
The substances which flow are called fluids. Both liquids and gases come in this category. Now,
letÊs have a deeper view of the difference between solids and fluids. Solid have the property of
deformation when subjected to shear stress but deformation is constant for a particular stress.
While fluids continues to deform untill stress is withdrawn.
The science of fluids at rest is calld fluid statics, while that of moving fluid hydrodynamics. Fluid
statics involves hydrostatic pressure, floatation, PascalÊs law and ArchimedesÊ principle.
Hydrodynamics involves continutiy equation, BernoullisÊs principle and TorricelliÊs theorem.
Later we will discuss viscosity associated with fluids (i.e., liquids and gas) and surface tension
associated only with liquids and some important concepts.
Density
Density of a substance is defined as the ratio of its mas & volume. Let a substance be of mass
M which has a volume V1 then density of the substance is given by
M
=
V
r = /w
M1 M2 M3
total volume =
1 2 3
M1 M2 M3 ....
M1 M2 M3
S = ....
1 2 3
FLUID MECHANICS
4 QUIZRR
For two substances the density of the mixture can be written as
1 2 (M1 M2 )
=
1 M1 2 M2
Suppose that a number of substances of volume V1, V2, V3 etc. and densities 1, 2, 3, etc.
respectively are mixed together. The toal mass of the mixture.
= 1V1 + 2V2 + 3V3........
total volume = V1 + V2 + V3.......
Therefore, density of the mixture is
1 V1 2 V2 3 V3 ....
=
V1 V2 V3 ....
1 V1 2 V2
For two substances =
V1 V2
Pressure
If we fill a vessel having a hole with fluid, fluid flows out of the hole. Now if we cover this hole
with a plate, which exactly fits the hole, the place can remain at rest only if we apply some
external force on the plate. This shows that fluid exerts force on the plate to push it outwards.
If s is the area of the plate and F is the normal force exerted by the fluid, the pressure at the
hole
F dF
p = lim
S s ds
F
h p
S
p
(A) (B)
For a point at dept h beow the surface of a liquid density , hydrostatic pressure p is given by :
p = p0 + hg
where p0 is the atmospheric pressure. The pressure difference between hydrostatic pressure and
atmospheric pressure is called gauge-pressure and will will be
p ă p0 = hg
FLUID MECHANICS
QUIZRR 5
(3) At a point it acts in all directions. If a pressure measuring device is at a given point in a
fluid, whatever be its orientation the pressure remains the same.
p
Piston
p h Vacuum
p
p Pressure
Sensor
(4) It always acts normal to the fluid bourndaries as the ability to flow makes fluid unable to
sustain a tangential force.
(A) (B)
(5) It depends on the depth of the point below the surface (h), nature of liquid () and acceleration
due to gravity (g) while it is independent of the amount of liquid, shape of the container or
cross sectional area considered. So if a given liquid is filled in vessels of different shapes to
same height, the pressure at the base in each vesselÊs will be the same, though the volume
or weight of the lilquid in different vessels will be different.
FLUID MECHANICS
6 QUIZRR
(6) In a liquid at same level, the pressure will be same at all points; if not, due to pressure
difference the liquid cannot be at rest. This is why the height of luqid is the same in vessels
of different shapes containing different amounts of the same liquid at rest when they are
in communication with each other.
hA HB hC hD hE
A B C D E
PA = PB = PC = PD = PE
Also, hA = hB = hC = hD = hE
PASCAL’S LAW
It states that „pressure applied to an enclosed fluid is transmitted undiminished to every portion
of the fluid and the wall of the containing vessel‰.
A well known application of PascalÊs law is the hydraulic lift used to support or lift heavy objects.
It is schemetically illustrated in figure.
A piston with small cross section area A1exerts a force F1 on the surface of a liquid such as oil.
F1
The applied pressure is P = A transmitted
1 F1
F1 F2 A2
P = A A or F2 = A .F1
1 2 1
Now, since A2 > A1, therefore, F2 > F1. Thus, hydraulic lift is a force multiplying device with a
multiplication factor equal to the ratio of the areas of the two positions. DentistÊs chairs, car lifts
and jacks, many elevators and hydraulic brakes all use this principle.
MEASUREMENT OF PRESSURE
Vacuum
1. Barometer : (P=0)
FLUID MECHANICS
QUIZRR 7
2. Manometer :
The free body diagram of the liquid (showing the vertical forces only) is shwon in Fig. (b). For
the equilibrium of liquid.
P0A
A
h
W
FLUID MECHANICS
8 QUIZRR
Net downward force = net upward force
P0A + W = (P0 + gh)A
W = ghA
Example 1
A s
A liquid of density is filled in a beaker of cross section S to a
Cylinder
height H and then a cylinder of mass m and cross section S is mg
made to float in it as shown in Fig. If the atmospheric pressure H s
C
is p 0, find the pressure (a ) at the top face A of the cylinder (b) at
B
the bottom face C of the cylinder and (c ) at the base B of the
beaker. Can ever these three pressures be equal ? S
Solution :
(a) Above the cross section A there is external pressure due to the atmosphere only.
So pA = Atmospheric pressure = p0
(b) At the point C the pressure will be due to atmosphere and also due to the weight of the
cylinder, i.e.,
mg
PC = P0 +
S
Note : If h is the height of cylinder inside the liquid, by concept of hydrostatic pressure pC
must be p0 + hg so that
i.e., weight of cylinder is equal to the thrust, which is the principle of floation and verifies
the correctness of our result.
(c) At the point B of beaker, pressure will be due to atmosphere, weight of cylinder and weight
of liquid in the beaker; so
mg pgHS mg
pB = p0 gH
= p0 ρ
S S
Note : The pressure due to the weight of cylinder at its own base C (mg/s) is more than at
the base of beaker B (mg/S) as s<S.
If the system is in fall (as in a satellite), g 0,
pA = pC = pB = p0 [as weight = 0]
FLUID MECHANICS
QUIZRR 9
Example 2
An open U-tube of uniform cross-section contains mercury. When 27.2 cm of water is poured
into one limb of the tube, (a ) how high does the mercury rise in the other limb from its
initial level ? (b) what is the difference in levels of liquids of the two sides ? (w = 1 and Hg
= 13.6 units)
Solution :
(a) If water depresses the mercury by y, the mercury in the other limb will rise by y above its
initial level (as fluids are incompressible),
so that
AAÊ = BBÊ = y
D
Also h2 = BÊB + BC h1 B´
y
i.e., h2 = BÊB + AAÊ A y B h2
A´ C
[as BC = AAÊ]
or h2 = 2y
[as AAÊ = BBÊ = y]
Now if h1 is the height of water column above AÊ, then as in a liquid, pressure is same at
all points in the same level:
pAÊ = pC, i.e, P0 + h1g = p0 + h22g,
or h11 = h22 i.e. 27.2 ï 1= 2y ï 13.6
which on solution gives by y = 1 cm, i.e., mercury rises by 1 cm from its initial level.
Example 3
To what height should a cylindrical vessel be filled with a homogeneous liquid to make the
force with which the liquid presses on the side of the vessel equal to the force exerted by
the liquid on the bottom of the vessel ?
Solution :
Consider a cylindrical vessel of radius r filled with a liquid of density to a height h. If p0 is the
atmospheric pressure, the pressure difference inside and outside the base of the cylinder,
pB = (p0 + hg) ă p0 = hg
FB = pBS = r2hg ...(1)
FLUID MECHANICS
10 QUIZRR
Now as pressure at a depth y below the surface of liquid inside
will be (p0 + yg) while outise p0, so pressure difference on two y
sides of the curved surface at depth y below the surface will be h dy
pS = (p0 + yg) ă p0 = yg
But according to the given problem FB1 = FC1 so from equations (1) and (2) r2hg = r2gh2
i.e. h= r
Example 4
For the arrangement shown in the figure, what is the density of oil ?
Solution :
PB = P0 + w. gl
PA = P0 + oil (+ d)g d=12.3mm
C
PA = PB Oil
water l=135mm
PA = Pa
A B
w 1000 135
oil = 916.5 kg/m3
( + d) (135+12.3)
Example 5
FLUID MECHANICS
QUIZRR 11
Example 6
Two vessels have the same base area but different shapes. The first vessel takes twice the
volume of water that the second vessel requires to fill up to a particular common height.
Is the force exerted by water on the base of the vessel the same in the two cases ? If so, why
do the vessels filled with water to that some height give different readings on a weighing
scale?
Solution :
Pressure (and therefore force) on the two equal base areas are identical. But force is exerted by
water on the sides of the vessels also, which has a non-zero vertical component when the sides
of the vessel are not perfectly normal to the base. This net vertical component of force by water
on the sides of the vessel is greater for the first vessel than the second. Hence, the vessels weigh
different when force on the base is the same in the two cases.
Consider a liquid kept at rest in a beaker as shown in figure (a). In this case we know that
pressure do not change in horizontal direction (x-direction), it decreases upwards along y-direction.
So, we can write the equations.
dP dP
= 0 and = ă g ...(i)
dx dy
y y
ay
ax
x x
(a) (b)
FLUID MECHANICS
12 QUIZRR
But suppose the beaker is accelerated and it has components of acceleration ax and ay in x and
y directions respectively, then the pressure decreases along both x and y directions. The above
equation in that case reduces to,
dP dP
= ă ax and = ă (g + ay) ...(ii)
dx dy
y
These equations can be derived as under :
(P + dP)A
Consider a beaker filled with some liquid of density p P+dP
accelerating upwards with an acceleration ay along
A ay
positive y-direction. Let us draw the free body diagram A dy
of a small element of fluid of area A and length dy as
shown in figure. P x PA
dP
or = ă (g + ay)
dy
y
Similarly, if the beaker moves along positive x-
direction with acceleration ax, the equation of motion
for the fluid element shown in figure is, ax
P+dP (P+dP)A
PA ă (P + dP) A = (mass) (ax) P PA
A A
or ă (dP) A = (A dx) ax ax
dx
x
dP
or = ă ax
dx
Consider a liquid placed in a beaker which is accelerating horizontally with an acceleration ÂaÊ.
Let A and B be two points in the liquid at a separation x in the same horizontal line. As we have
seen in this case. y
dP
= ă a
dx
h1
h2
or dP = ă a dx a
A B
x x
FLUID MECHANICS
QUIZRR 13
h1 h2 a
= = tan
x g
a
tan =
g
Example 7
A liquid of density is in a bucket that spins with angular velocity as shown
in figure. Show that the pressure at a radial distance r from the axis is
2 r 2
P = P0 +
2
P P mx2
x
mg Fnet
P (x,y)
FLUID MECHANICS
14 QUIZRR
Net force on it should be perpendicular to the free surface (in equilibrium). Hence,
y y
x2
dy =
0
0 g .dx
x2 2
y=
2g
This is the equation of the free surface of the liquid, which is a parabola.
r 2 2
At x = r, y=
2g
P0
P(r) = P0 + gy y
x=r P(r)
r 2 2
or P(r) = P0 +
2
ARCHIMEDES’ PRINCIPLE
F = mg = Vi L g
L = density of liquid
Law of Floatation
Let a body of volume V and density S is floating in a liquid of density L . Suppose Vi be the
volume of body immersed in the liquid,
FLUID MECHANICS
QUIZRR 15
V S g = Vi L g
Vi S
= ...(i)
V L
Vi
= 100 = S 100
V L
(i) In liquid, the apparent weight of the body decreases, and this decrease in its weight is equal
to the upthrust acting on the body.
Hence, apparent weight
wapp = Vg(S ă L,)
(ii) If object is immersed in water, then
So, by weighing a body in air and in water, we can determine the relative density of the
body.
(iii) Buoyant Force in Accelerating Fluids
Suppose a body is dipped inside a liquid of density L placed in an elevator moving with an
acceleration a . The buoyant force F in this case becomes,
F = V L geff
Here, geff = | g ă a |
FLUID MECHANICS
16 QUIZRR
For example, if the lift is moving upwards with an acceleration a, the value of geff is g + a and
if it is moving downwards with acceleration a, the geff if g ă a. In a freely falling lift geff is zero
(as a = g) and hence, net buoyant force is zero. This is-why, in a freely falling vessel filled with
some liquid, the air bubbles do not rise up (which otherwise move up due to buoyant force). The
above result can be derived as follows.
Here, m = V L
F = V L (g + a) = V L geff
where geff = g + a
Example 8
A certain block weighs 15 N in air. It weighs 12 N when immersed in water. When immersed in
another liquid, it weighs 13 N? Calculate the relative density of (a) the block (b) the other liquid.
Solution :
B Weight of body
(a) So for body RD = =
W Weight of equal vol. of water
Weight of body WA
i.e., RD = =
Thrust from water WA ă WW
15
= 5
15 12
L T hL 15 13 2
So for liquid, RD = = = =
W ThW 15 12 3
FLUID MECHANICS
QUIZRR 17
Example 9
A piece of copper having an internal cavity weighs 264 g in air and 221 g in water. Find
the volume of the cavity. Density of copper is 8.8 g/cc .
Solution :
Th (264 ă 221) g
V = g = 1 g
= 43 cc
Now as mass of body is 264 g while the density of material of body is 8.8 g/cc, the volume of
material in the body
Example 10
F F´
T0 + mg T + mg
ga
F´ = F ...(ii)
g
FLUID MECHANICS
18 QUIZRR
From NewtonÊs second law,
F´ ă T ă mg = ma ...(iii)
Solving Eqs. (i), (ii) and (iii), we get
a
T = T0 1
g
FLOATATION
(A) Translatory-Equilibrium
When a body of density B and volume V is immersed in a liquid of density , the forces acting
on the body are:
(1) The weight of body W = mg = V B g acting vertically downwards through the centre of
gravity of the body.
(2) The upthrust Th = Vg acting vertically upwards through the centre of gravity of the
displaced liquid, i.e., centre of buoyancy.
So the following three situations are possible :
(a) The density of body is greater than that of liquid (i.e., B > ). In this situation as
weight will be more than upthrust the body will sink. [Fig. (A)]
(b) The density of body is equal to the density of liquid (i.e., B = ). In this situation W
= Th, so the body will float fully submerged in neutral equilibrium anywhere in the
liquid. [Fig. (B)]
(c) The density of body is lesser than that of liquid (i.e., B > ). In this situation W < Th,
so the body will move upwards and in equilibrium will float partially immersed in the
liquid such that
W = Ving
[Vin being the vol. of body in the liquid]
FLUID MECHANICS
QUIZRR 19
(2) And when a body is floating weight of body is equal to the upthrust, i.e., VBg = Ving.
(3) In case of floating as W = Th, the apparent weight of the floating body will be zero, i.e.,
Wapp = W ă Th = 0
(4) In case of floating as W = Th implies VBg = Ving, the equilibrium of floating bodies is
unaffected by variation in g though both thrust and weight depend on ÂgÊ.
(B) Rotatory–Equilibrium
When a floating body is slightly tilted from equilibrium position, the centre of buoyancy B shifts.
The vertical line passing through the new centre of buoyancy BÊ and initial vertical line meet at
a point M called meta-centre. If the meta-centre M is above the centre of gravity the couple due
to forces at G (weight of body W) and at BÊ (upthrust) tends to bring the body back to its original
position [Fig.(B)]. So for rotational equilibrium of floating body the meta-centre must always be
higher than the centre of gravity of the body.
However, if meta-centre goes below CG, the couple due to forces at G and BÊ tends to topple the
floating body. [Fig. (C)]
Th Th
G
G M M w
B
G B´ B´
B B
w
(C) Application
So Vout = V ă Vin = V 1 B
FLUID MECHANICS
20 QUIZRR
Vout B
i.e., fout = = V 1
V
(2) Comparison of densities of floating bodies : As for floating VB = Vin, i.e.,
Vin
B = = fin
V
B1 fin 1
=
B2 fin 2
(3) Comparison of densities of liquids : If the same body is made to float in different liquids,
then
VB = (Vin)11 = (Vin)22
1 Vin 2
or =
2 Vin 1
i.e., density of liquid is inversely proportional to the volume of body inside it.
(4) Weighing a body : If a platform of mass M and cross section A is floating in a liquid of
density with its height h inside the liquid.
Mg = hAg ...(1)
Now if a body of mass m is placed on it and the platform sinks by y then
(M + m)g = (y + h)Ag ...(2)
Subtracting Eqn. (1) from (2),
mg = Ayg, i.e., W y ...(3)
So we can determine the weight of a body by placing it on a floating platform and noting
the depression of the platform in the liquid by it.
Example 11
A rod of length 6m has a mass 12kg. It is hinged at one end at a distance of 3m below water
surface, (a) What weight must be attached to the other end of the rod so that 5 m of the
rod are submerged ? (b) Find the magnitude and direction of the force exerted by the hinge
on the rod. (Specific gravity of rod is 0.5).
Solution :
As shown the forces acting on the rod are :
(1) The weight of rod 12g N acting downwards through the CG of the rod, i.e., at a distance
of 3m from the hinge.
FLUID MECHANICS
QUIZRR 21
w
Th
R
3m
W
5 12g
Force of buoyancy = 20g N
6 0.5
6 5
ă 12g sin 20g sin ă w6sin = 0
2 2
Example 12
Why does a uniform wooden stick or log float horizontally ? If enough iron is added to one
end, it will float vertically; explain this also.
Solution :
When a wooden stick is made to float vertically, its rotational equilibrium will be unstable as its
meta-centre will be lower than its CG and with a slight tilt it will rotate under the action of the
couple formed by thrust and weight in the direction of tilt, till it becomes horizontal. [Fig. (A)]
FLUID MECHANICS
22 QUIZRR
Th
Th
G
G
B M
B´ B´
W G G
W
(A) (B)
However, due to loading at the bottom, the CG of the stick (or log) will be lowered and so may
be lower than the metacentre. In this situation the equilibrium will be stable and if the stick (or
log) is tilted, it will come back to its initial vertical position. [Fig. (B)]
Example 13
A cubical block of iron 5 cm on each side is floating on mercury in a vessel. (a) What is the
height of the block above mercury level ? (b) Water is poured into the vessel so that it just
covers the iron block. What is the height of water column ?
[RD of Hg = 13.6 and Fe = 7.2]
Solution :
(a) In case of floatation W = Th, i.e., V = Vin; so if h is the height of iron block above mercury
and a is the side of iron cube,
a
h 2.35 h 2.54
2.65 2.46
a h
Hg Hg
(A) (B)
(a a a) = (a ă h) a a
i.e., (a ă h) = a or h = a 1 ă
7.2
so h = 5 1 ă 2.35cm
13.6
(b) Here upthrust is provided by both mercury and water and if h is the height of water-level,
Vg = ThHg + Thw
FLUID MECHANICS
QUIZRR 23
Example 14
A block of wood floats in water with two-thirds of its volume submerged. In oil the block
floats with 0.90 of its volume submerged. Find the density of (a) wood and (b) oil, if density
of water is 103 kg/m3.
Solution :
In case of floatation W = Th, i.e., V = Vin
so (a) V = (2/3)Vw [as Vin = (2/3)V]
2 2 kg
or = w 103 667 3
3 3 m
2 2 kg
oil = 3 0.9 w 2.7 10 740 m3
3
or
Example 15
A glass beaker having mass 390g and an interior volume of 500cm3 floats on water when
it is less than half filled with water. What is the density of the material of the beaker ?
Solution :
As the beaker floats in water when less than half filled with water, it will float just fully suberged
when half filled. In this situation,
V=500cc
mass of beaker + mass of water in it = V
Water
i.e., 390 + 250 = V 1 [as = 1 g/cc]
i.e., outer volume of beaker Water
V = 640 cc
Now as inner volume of beaker is given to be 500 cc, so the volume of the material of beaker =
640 ă 500 = 140 cc. But as mass of beaker is 390g, so density of material of beaker
m 390 g
= 2.79
V 140 cc
FLUID MECHANICS
24 QUIZRR
Example 16
A cube of wood supporting 200 g mass just floats in water. When the mass is removed, the
cube rises by 2 cm. What is the size of the cube ?
Solution :
If a is the side of the cube and as cube rises 2 cm on removing the mass, the weight of body must
be equal to the thrust provided by 2 cm height of cube of base area (a a), i.e.,
mg = VÊg or (200 g) = (2 a2) 1 g
or a = 10 cm, i.e., the side of cube is 10 cm.
Example 17
A block of wood weighs 12kg and has a relative density 0.6. It is to be in water with 0.9 of
its volume immersed. What weight of a metal is needed (a) if the metal is on the top of wood,
(b) if the metal is attached below the wood ? (RD of metal = 14)
Solution :
(a) When the metal is on the top of wood,
M M
M + m1 = 0.9Vw = 0.9 asVw
w w
0.9
m1 = M 0.9 ă 1 12 ă1 6kg
w 0.6
m1
Vw PW Vw PW
M M
PM m1
VM
(A) (B)
M m2 mass
or M + m2 = 0.9 as vol.
w M
or m2 1 ă = M 0.9 ă1
M w
FLUID MECHANICS
QUIZRR 25
1 0.9
or m2 1 ă = 12 ă1 6kg
14 0.6
14 6
or m2 = 6.5kg
13
Example 18
A wooden stick of lenght L, radius R and density has a small metal piece of mass m (of
negligible volume) attached to its one end. Find the minimum value for the mass m (in
terms of given parameters) that would make the stick float vertically in equilibrium in a
liquid of density .
Solution :
For the stick to be vertical for rotational equilibrium, centre of gravity should be below in a
vertical line through the centre of buoyancy. For minimum m, the two will coincide.
Let h be the length of immersed portion. For translatory equilibrium,
Wt. of rod + mass attached = force of buoyancy
(M + m)g = R2hg ...(1)
L h
2 C
m
2
where M = R L.
The height of centre of mass from bottom
(M)L / 2 m 0 ML
=
mM
= 2 mM
For rotatory equilibrium and for minimum m, this should be equal to h/2.
h ML
2 m M
2
ML
h =
m M
FLUID MECHANICS
26 QUIZRR
Substituting for h in Eqn. (1), we get
ML
(M + m)g = R 2 g .
m M
(M + m)2 = R2 . ML
(M + m) = MR 2 L R 2 L . R 2 L
m = R 2 L ă R 2 L
= R L ă 1
2
Flow of Fluids
Streamline flow :
The streamline flow of a liquid is the flow in which each element of the liquid passing through
a point travels along the same path and with the same velocity as the preceeding element passing
through the same point.
v3
v1
v2
Hence, it is a regular flow. The path followed by each element is called streamline. The tangent
drawn at any point of streamline gives the direction of the flow of liquid at that point. From figure
velocity at different points may be different. Hence, in the figure
v1 cons tan t, v 2 = cons tan t, v 3 cons tan t
but v1 v 2 v 3
Turbulent flow
A liquid can possess streamlined motion only when its velocity is less than a limiting velocity,
called the critical velocity. When the velocity of the liquid becomes greater than the critical
velocity for the liquid, the different elements of the liquid move along a zig-zag path. As a result
of unsteady motion of the elements of the liquid along zig-zag paths, the liquid gets churned up.
Such a motion of the liquid is called turbulent flows
FLUID MECHANICS
QUIZRR 27
Principle of Continuity
It states that, when an incompressible and non-viscous liquid flows in a streamlined motion
through a tube of non-uniform cross-section, then the product of the area of cross-section and the
velocity of flow is same at every point in the tube.
Thus, A1v1 = A2v2 Q
or Av = constant P V2
V1
1 A2
or v A1
A
BERNOULLI’S THEOREM
In streamlin flow, the energy of a fluid particle remains constant. Suppose that, in a tube of flow,
cross-sectional area at 1 and 2 are A1 and C C´
A2, corresponding velocities v1 and v2 and movement P2
pressures are P1 and P2 respectively. D D´
B B´
Consider an element ABCD of an P1
A A´
incompressible fluid. In a time t, the liquid
h2
moves in and the liquid element becomes h2
AÊBÊCÊDÊ. In other words, we can also say
that fluid element ABBÊA has effectively
changed into DCCÊDÊ.
m = A1v1 t = A2v2 t
Work done by fluid pressure at 1 = (P1A1)v1 t = P1 m/
Work done by fluid pressure at 2 = ă (P2A2)v2 t = ă P2 m/
Work done by gravity = ă (m).g. (h2 ă h1)
Change in kinetic energy = 1/2 m [v22 ă v12]
Using work energy theorem = (W = K)
m m 1
P1 ă P2 ă g (h 2 ă h1 ) = m v 22 ă v12
2
P1 v2 P v2
gh1 1 = 2 gh 2 2
2 2
v12 v 2
P1 gh1 = P2 gh 2 2
2 2
v 2
P gh = Constant
2
FLUID MECHANICS
28 QUIZRR
where, P = Pressure energy per unit volume or Pressure at a cross-section.
gh = potential energy per unit volume and (1/2) v2 = kinetic energy per unit volume.
The above equation is known as BernoulliÊs equation.
P v2
h = constant (m)
g 2g
P v2
In this expression g is called the ÂPressure head, the velocity head and h the gravitational
2g
head. The SI unit of each of these three is metre. Therefore, BernoulliÊs equation may also be
stated as,
Sum of pressure head, velocity head and gravitational head is constant for an ideal
fluid.
(A) Venturimeter
The device is used to measure the rate of steamine flow of a fluid
through a tube. The working of a venturimeter is based on A1 V1
P2
BernoulliÊs principle. The construction of the device is shown in P1
the figure. The tube has different areas of cross-section A1 and A2 h
in two sections. A U-tube containing mercury is fitted between the
larger and narrower part of the horizonal tube. The area of cross-
section of the broader part is A1 and that of the narrower part is A2. The flow speed of the
narrower part is V2 and that at the broader part is V1.
The fluid is incompressible. The rate of flow of the fluid is given by,
1 1 2
P1 v12 0 = P2 v 2 0
2 2
FLUID MECHANICS
QUIZRR 29
i.e., P1 ă P2 =
1
2
v 22 ă v 12
i.e., 0gh =
1
2
v 22 ă v 12 ...(3)
20 gh
v1 = A 2 ...(4)
A 12 ă A 22
Therefore, the rate of flow of fluid is given by,
q = v1A1
20 gh
q = A1 A 2 ...(5)
A12 ă A 22
1 1
P0 v12 gh = P0 v 22 ...(2)
2 2
Therefore, from equations (1) and (2), we get,
2gh
v 22 = ...(3)
A 2
1 ă 2
A 1
If A2 << A1 then,
v2 = 2gh ...(4)
Equation (3) and (4) give the velocity of efflux of a liquid coming out in a small hole in a tank.
FLUID MECHANICS
30 QUIZRR
I mportant note
(i) v h
2H ă h
t= ...(5)
g
Suppose that horizontal distance travelled by the liquid stream is x, i.e. the liquid will strikes
the ground at a distance x from the base of the container below the hole.
2 H ă h
Then, x = v.t = 2gh. ...(6)
g
dx
This range x will be maximum if 0
dh
dx
Now, x2 = 4h (H ă h). Therefore, 2x 4H ă 8h 0
dh
H
and xmax = 2 H ă H / 2 i.e. Xmax = H ...(8)
2
Example 19
A non-viscous liquid of constant density 1000 kg/m3 flows in a steamline motion along a
tube of variable cross-section. The tube is kept inclined in the vertical plane as shown in
the figure. The area of cross-section of thetube at the points P and Q at heights of 2 metre
and 5 metre are respectively 4 ï 10ă3 m2 and 8 ï 10ă3 m2. The velocity of the liquid at point
P is 1 m/s. Find the work done per unit volume by the pressure and the gravity forces as
the fluid flows from point P to Q. Take g = 9.8 m/s2.
P 5m
2m
FLUID MECHANICS
QUIZRR 31
Solution :
Given : A1 = 4 ï 10ă3 m2, A2 8 ï 10ă3, h1 = 2m, h2 = 5m, v1 = 1, m/s and = 103 kg/m3
From continutiy equation, we have
A1v1 = A2v2
A2
A1 v2
or v2 = v
A2 1
A1 2
v1
4 10 ă3 1
or v2 = ă3 (1m/s) h2
8 10 h1
1
v2 = m/s
2
1 1
P1 v12 gh1 P2 v22 gh2
2 2
1
P1 ă P2 g (h2 ă h1 ) (v22 ă v12 ) ...(i)
2
(i) Work done for unit volume by the pressure as the fluid flos from P toQ
W1 = P1 ă P2
1
= g (h2 ă h1) + (v 22 ă v12 ) [from Eq. (i)]
2
1 1
= (103 )(9.8)(5 ă 2)+ (102 ) ă1 J/m3 = 29400 ă 375 J/m3
2 4
(ii) Work done per unit volume by the gravity as fluid flows from P to Q.
W2 = ăg (h2 ă h1) = ă(103)(9.8)(5 ă 2) J/m3
or W2 = ă19400 J/m3 Ans.
FLUID MECHANICS
32 QUIZRR
Example 20
A cylindrical tank of base area A has a small hole of area Âa Ê at the bottom. At time t =0,
a top starts to supply water into the tank at a constant rate m3/s.
(a) what is the maximum level of water h max in the tank ?
(b) find the time when level of water becomes h(<h max).
Solution :
(a) Level will be maximum when
3
rate of inflow of water = rate of outflow of water Rate = m
s
i.e., = av
A
or = a 2 ghmax
h
hmax = Ans.
2 ga 2 v= 2gh
a
(b) Let at time t, the level of water be h. Then
dh
A ă a 2 gh
dt
h dh dt
t
or 0
ă a 2 gh =
0 A
A ă a 2 gh
t = ag l n
ă 2 gh Ans.
Example 21
Water flows through a tunnel from the reservoir of a dam towards the turbine installed in
its power plant. The power plant is situated h m below the reservoir. If the ratio of the
cross-sectional areas of the tunnel at the reservoir and power station end is , find the
speed of the water entering into the turbine.
Solution :
Applying BermouliÊs theorem at reservoir and power plant for the following water, we obtain,
1
P0 gh1 v12 P0 gh 2 v 22
2
FLUID MECHANICS
QUIZRR 33
v 22 v12 2g (h1 ă h2 ) .
2
A2
v2 = v2 2 gh
A1
2gh
2
v2 = A
1ă 2
A1
2gh
v2 = .
2 ă1
Example 22
A cylindrical tank 1 m in radius rests on a platform of 5 m. Initially the tank is filled with water upto
a height of 5 m. A plug whose area is 10ă4 m2 is removed from an orifice on the side of the tank at
the bottom. Calculate (a) initial seed with which the water flows from the orifice (b) initial speed with
which the water strikes the ground and (c) time taken to empty the tank to half its original value (d)
Does the time to emptied the tank depend upon the height of stand ?
Solution :
(a) As speed of efflux is given by
5m
A0
5m
FLUID MECHANICS
34 QUIZRR
(b) As initial vertical velocity of water is zero, so its vertical velocity when it hits the ground
vv = 2 gh 2 10 5 10 m / s
(c) When the height of water level above the hole is y, velocity of flow will be v = 2 gy and
so rate of flow
dV
A 0 v A 0 2 gy
dt
or ăAdy =
2gy A dt
0
[as dV = A dy]
A 2
t= A [ H ă H']
g
0
12 2
so t = ă4
[ 5 ă 5 / 2] 9.2 103 s 2.5h
10 10
Example 23
A 3.6m long verticle pipe resonates with a source of frequency 212.5 Hz when water level
is at a certain height in the pipe. Find the heights of water level (from the bottom of the
pipe) at which reasonances occur. Neglect end correction. Now, the pipe is filled to a height
H ( 3.6m). A small hole is drilled very close to its bottom and water is allowed to leak.
Obtain an expression for the rate of fall of water level in the pipe as an a function of H.
If the radius of the pipe and hole are 2 ï 10ă2 m and 1 ï 10ă3 m respectively, calculate the
time interval between the occurrance of first two resonances. Speed of sound in air
340 m/s and g = 10 m/s2.
Solution :
For resonance to occure in a closed pipe,
2n ă1
l = where n = 1, 2, 3,....
4
FLUID MECHANICS
QUIZRR 35
v 340
Here, = 1.6m
n 212.5
So, resonance occure for length of air column 0.4 m, 1.2 m, 2.0 m, 2.8 m and 3.6 m and
corresponding height of water column from bottom.
= 3.2 m, 2.4 m, 1.6 m, 0.8 m and 0m.
The velocity of efflux, when height of water level is h above bottom
= 2gh
dh
Aă a 2 gh
dt
A
dt = ă dh
a 2 gh
So, the time in which the height of water column changes from 3.2 m to 2.4 m, i.e., time interval
between first to resonances,
2.4 2.4
A 1 A
t= ă
a 3.2 2 gh
dh ă
a 2 gh
2 h1 / 2
3.2
= 42.87 s Ans.
Example 24
FLUID MECHANICS
36 QUIZRR
restored. A tiny hole of areas s (s<<A) is punched on the vertical side of the container at
a height h (h < H/2). Determine (i) the initial speed of efflux of the liquid at the hole (ii) the
horizontal distance x travelled by the liquid initially and (iii) the height h m at which the
hole should be punched so that the liquid travels the maximum distance x m initially. Also
calculate x m.
Solution :
(a) As for floating, W = Th
Vg = V1d 1 + V2d 2g
A 3 A 1 A
or L L d L 2d
5 4 5 4 5
3 2 5
i.e., = d d d
4 4 4
H H 5 A 1
i.e., p = p0 = dg 2dg d L g
2 2 4 5 A
3 H 1 1
i.e., p = p0 + H dg dg L dg p0 (6H L)dg
2 2 4 4
(b) (i) By BermoulliÊs theorem for the point just inside and outside the hole
1 2 1
p1 + v1 p2 v22
2 2
H H 1
i.e., p0 + dg ă h 2 dg p0 (2d )v2
2 2 2
(ii) As at the hole vertical velocity of liquid is zero so time taken by it to reach the
ground,
t= 2h / g
g 2h
So that x = vt = (3 H ă 4 h) h(3 H ă 4 h)
2 g
FLUID MECHANICS
QUIZRR 37
d d
dh
x2 = 0 or
dh
(3Hh ă 4h2 ) = 0
or 3H ă 8h = 0, i.e., h = (3/8)H
3H 3 3
and Xmax = 3H ă H H
8 2 4
VISCOSITY
F
Viscosity is internal friction in a fluid. Viscous v
forces opposes the motion of one portion of a fluid
relative to the other.
The simplest example of viscous flow is motion of
y
a fluid between two parallel plates.
The bottom plate is stationary and the top plate x
moves with constant velocity v . The fluid in contact
with each surface has same velocity at the surface. The flow speeds of intermediate layers of fluid
increase uniformly from bottom to top, as shown by arrows. So the fluid layers slide smoothly over
one another.
According to Newton, the frinctional force F (or viscous force) between two layers depends upon
the following factors,
(i) Force F is directly proportional to the area (A) of the layers in contact, i.e.,
F A
dv
(ii) For F is directly proportional to the velocity gradient dy between the layers. Combining
dv
F A
dy
dv
F = A
dy
Here, is is constatn of proportionality and is called coefficient of viscosity. Its value depends
on the nature of fluid. The negative sign in the above equation shows that the direction of
viscous force F is opposite to the direction of relative velocity of the layer.
FLUID MECHANICS
38 QUIZRR
The SI unit of is N-s/m2. It is also called decapoise or pascal second. Thus,
1 decapoise = 1 Năs/m2 = 1 Pa-s = 10 poise
Dimensions of are [MLă1Tă1]
Coefficient of viscosity of water at 100C is = 1.3 ï 10ă3 N-s/m2. Experiments show that
coefficient of viscosity of a liquid decreases and its temperature rises.
Example 25
A plate of area 2m2 is made tomove horizontally with a speed of 2 m/s by applying a
horizontal tangential force over the free surface of a liquid. If the depth of the liquid is 1m
and the liquid in contact with the bed is stationary. Coefficient of viscosity of liquid is 0.01
poise. Find the tangential force needed to move the plate.
Solution :
v
Velocity gradient = y
v=2m/s
2ă0 m/s F
= 1ă 0 2 m 1m
v
F A
y
When an object moves through a fluid, it experiences a viscous force which acts in opposite
direction of its velocity. The mathematics of the viscous force for an irregular object is difficult,
we will consider here only the case of a small sphere moving through a fliuid.
According to stokes law a spherical object of radius r moving at velocity v expression is viscous
force given by
F = 6rv ( = coefficient of viscosity)
This law is called StokeÊs Law.
Consider a small sphere falling from rest through a large column of viscous fluid. The forces
acting on the sphere are,
FLUID MECHANICS
QUIZRR 39
4 3 4
r g r 3 g 6 rVT
3 3
2 r2 ( ă ) g
From Eq. (ii), we get vT =
9
Figure shows the variation of the velocity v of the sphere with time
VT
O time
Tip : From the above expression we can see the terminal velocity of a spherical body is directly
proportional to the difference in the densities of the body and the fluid (ă ), the terminal
velocity is negative. This means that the body instead of falling, moves upward. This is why
air bubbles rise up in water.
FLUID MECHANICS
40 QUIZRR
Example 26
A spherical ball of radius 1 ï 10ă4 m and density 104 kg/m3 falls freely gravity through a
distance h before entering a tank of water. If after entering the water the velocity of the
ball does not change, find h. The viscosity of water is 9.8 ï 10ă6 N-m2.
Solution :
After falling through a heigh h, the velocity of the ball becomes v = 2gh . After entering water,
this velocity does not change, this velocity is equal to the terminal velocity.
2 2 ă
2 gh r g
g
i.e.,
2
2 104 ă 103
or,
2gh = 9 (10 ă4 2
)
9.8 10 ă5
2.04
2
or, h = = 0.212m
2 9.8
Example 27
Two identical drops of water are falling through air with a steady volocity v. If the drops
coalesced, what will be the new velocity ?
Solution :
Let r be the radius of each drop. The terminal velocity vT of a drop of radius r is given by
2 r ă
2
vT ...(1)
9
Now when two drops each of radius r coalesce to form a new drop, the volume of coalesced drop
will be given by
4 4 4
R 3 r 3 r 3
3 3 3
The radius of the coalesced drop will be
R = (2)1/3r
Hence, the new terminal velocity of the coalesced drop is
2 2 r ă
1/ 3
v´T = ....(2)
9
FLUID MECHANICS
QUIZRR 41
vT' 2
2 3
vT
vT' = 2 3 v [as vT = v]
2
or
SURFACE TENSION
The free surface of a liquid constracts so that its exposed surface area becomes minimum, i.e., it
behaves as if it were under tension, somewhat like a stretched elastic membrane. This property
is known as surfance tension.
The surface tension of a liquid varies with temperature as well as dissolved impurities, etc. When
soap is mixed with water, the surface tension of water decreases.
Surfance tension of a liquid is measured by the normal force acting per unit length. On either
side of an imaginary line drawn on the free surface of a liquid, the direction of this force is
perpendicular to the line and tangential of the free surface of liquid.
F
T=
L
B
Consider a wire frame, as shown in figure, equipped with a sliding wire
AB. It is dipped is soapy water. A filmk of liquid is formed. A force F has
F
to be aplied to hold the wire in place. Since, the soap film has two L
surfaces attached to the wire, the totallength of the film is contact with
the wire is 2L. A
F
T (surface tension) =
2L
Surface Energy
If the area of the liquid surface has to be increased, work has to be done against the force of
surface tension. The work done to form a film is stored as potential energy of the surface and the
FLUID MECHANICS
42 QUIZRR
amount of the energy per unit are of this surface under isothermal condition is the instrinsic
surface energy density or free surface energy density.
Work done in small displacement dx.
dW = F ï dx = 2TL dx
F
x
W = 0 2TLdx 2TLx
x dx
As A = 2Lx (area of both sides)
W/A = T (intrinsic surface enrgy)
Excess Pressure
The pressure inside a soap bubble and outside it, are not identical due to the surface tension of
the soap buble. To calculate this pressure difference, letÊs first consider an air bubble inside a
liquid. If the pressure differences is P, then the work done to increase the radius of bubble from
r to (r + r) is given by :
W = Fr = 4r2pr
While change in area, S = 4(r + r)2 ă 4r2 = 8r r
From the definition of surface tension
4 r 2 pr
T = W/S =
8 r r
2T
p =
r
p = 2 2T / r 4T
r
Angle of Contact
1. Angle of contact, for a solid and a liquid is defined as the angle between tangent of the
liquid surface drawn at the point of contact and the surface inside the liquid.
2. The angle of contact of a luqid surface on a solid surface depends on the nature of the liquid
and the solid.
FLUID MECHANICS
QUIZRR 43
Capillarity
Surface tension causes elevation or depression of the liquid in the narrow tube. This effect is caled
capillarity.
When a glass capillary (A tube of very smal diameter is called a capillary tube) open at both ends
is dipped vertically in water, the water in the tube will rise above the level of water in the vessel
as shown in figure (a). In case of mercury, the luquid is depressed in the tube below the level
of mercury in the vessel as shown in figure (b).
(a) (b)
When the contact angle is less than 900 the liquid rises in the tube. For a nonwetting liquid angle
of contact is greater than 900 and the surface is depressed, pulled down by the surface tension
forces.
Explanation
When a capillary tube dipped in water, the water meniscus inside the tube is concave. The
2T
pressure just below the menicus is less than the pressure just above it by , where T is the
R
surface tension of the and R is the radius of curvature of the menicus. The pressure on the surface
of water is P0, the atmospheric pressure. The pressure just below the plane surface of water
FLUID MECHANICS
44 QUIZRR
2T
outside the tube is also P0, but that just below the the menicus inside the tube is P0 ă . We
R
know that pressure at all points in the same level of water must be the same. Therefore, to make
2T
up the deficiency of pressure below the menisus water being s to flow from outside into the
R
tube. The rising of water in the capillary stops at a certain height h. In this position of pressure
2T
of water column of heigh h becomes equal to , i.e.,
R
2T
hg =
R
2T
or h =
Rg
If r is the radius of the capilliary tube and the angle of contact, then
r
2r R
R=
cos
2T cos
h =
rg
FLUID MECHANICS
QUIZRR 45
MISCELLANEOUS EXAMPLES
Example 1 A
x
A closed tube in the form of an equilateral triangle of side l
E
contains equal volumes of three liquids, which do not mix,
and is placed vertically with its lowest side horizontal. Find
D
the value of x in the figure, if the densities of liquids are in x
arithmetic progression. C
B F x
Solution :
Let ÂdÊ be the density of liquid in DAE, Âd + yÊ be the density of liquid in FCE and Âd + 2yÊ be the
density of liquid in FBD.
3
Pressure at D = (l ă x) sin 600 dg = (l ă x) dg
2
Pressure at F = (l ă x) sin 600 dg + x sin 60Ĉ (d + 2y)g
3 x 3
= (l ă x) dg + (d + 2y)g
2 2
x 3
Pressure at E = x sin 60Ĉ dg dg
2
Pressure at F is aso equal to = x sin 600 dg + (l ă x) sin 600 (d + y)g
x 3 3
dg (l x) (d y) g
2 2
Equating the values of pressure at F
3 x 3 x 3 3
(l ă x) dg + (d+ y)g = dg + (l ă x) (d + y)g
2 2 2 2
(l ă x)d + x(d + 2y) = xd + (lă x) (d + y)
x = (l/3).
Example 2
a=3m/s2
30Ĉ
FLUID MECHANICS
46 QUIZRR
Solution :
Net force on a fluid particle of mass m at the surface of the liquid should be perpendicular to its
surface when seen from accelerating frame of reference. Two forces are acting on the fluid
particle.
(i) weight (mg) acting vertically downwards
(ii) pseudo force (ma) along negative x-direction
As we said, the resultant of these two should be perpendicular to the free surface or along the
free surface the components of these two forces should cancel each other.
e
ma cos = mg cos (600 + ) rfac
su
0 0 r ee id
or 3cos = 10(cos 60 cos ă sin 60 sin ) ma F
liq
u
of
3cos = 5 cos ă 5 3 sin
60Ĉ
5 3 sin = 2 cos mg 30Ĉ
2
tan = = 0.23
5 3
Example 3
Two narrow bores of radius 3.0 mm and 6.0 mm are joined together a U-shaped tube open
at both ends. If the U-tube contains water, what is the difference in its levels in the two
limbs of the tube. Surface tension of water is 7.3 ï 10ă2 N/m. Take the angle of contact to
be zero and density of water to be 103 kg/m3. g = 9.8 m/s2.
Solution :
hg = P
2T cos 2T cos
= ă
r1 r2
2T cos r2 ă r1
or h =
g r1 r2
= 2.48 ï 10ă3 m.
= 2.48 mm.
FLUID MECHANICS
QUIZRR 47
Example 4
A solid ball of density half that of water falls freely under gravity from a height of 19.6m and
then enters water. Upto what depth will the ball go. How much time will it take to come again
to the water surface. Neglect air resistance and viscosity effects in water g = 9.8m/s2.
Solution :
Let be the density of ball and 2 the density of water. Net retardation inside the water,
upthrust ă weight g
a =
mass h=19.6m
V 2 g ă V g
= (V = volume of ball)
V g v
a=g
= g
= 9.8m/s2
Hence, the ball will go upto the same depth 19.6 m below the water surface.
Further, time taken by the ball to come back to water surface is,
v
t = 2
a
19.6
= 2 4s
9.8
Example 5
A block of mass 1 kg and density 0.8g/cm3 is held stationary with the help of a string
as shown in figure. The tank is accelerating vertically upwards with an acceleration
a = 1.0m/s2. Find
(a) the tension in the string
(b) if the string is now cut find the acceleration of block. a
2 3 3
Take g = 10m/s and density of water = 10 kg/m .
Solution :
(a) Free body diagram of the block is shown in Fig. In the figure,
F = upthrust force
= V (g + a)
FLUID MECHANICS
48 QUIZRR
mass of block
= g a
density of block F
a
1
= 1000 10 1 13.75N
800 W+T
W = mg = 10N
Equation of motion of the block is,
F ă T ă W = ma
13.75ăTă10 = 1 1
T = 2.75 N
(b) When the string is cut T = 0
FăW
a=
m
13.75 ă 10
=
1
= 3.75m/s2.
Example 6
Length of a horizontal arm of a U-tube is 20cm and ends of both
the vertical arms are open to a pressure 1.01 103 N/m2. Water
10cm
is poured into the tube such that liquid just fills horizontal part
of the tube. Now, one of the open ends is sealed and the tube is
then rotated about a vertical axis passing through the other
20cm
vertical arm with angular velocity . Take density of water = 103
kg/m3 and g = 10m/s2. Assume temperature to be constant.
Solution :
Let the cross sectional area of the tube be A. Initial pressure of air in sealed tube,
Pi = 1.01 103 N/m2
Initial volume, Vi = 0.1A
Final volume, Vf = (0.1 ă x) A 10ăx
Let final pressure be Pf. Using
C x
PiVi = PfVf, we have
x B
Pi Vi 0.1
Pf =
1.01 103 ...(i)
Vf 0.1 ă x
FLUID MECHANICS
QUIZRR 49
PB = Pf + gx ...(ii)
3 2
PC = 1.01 10 N/m ...(iii)
Pressure difference is, P = PB ă PC ...(iv)
Centripetal force required for circular motion of vertical column is provided by reaction of the tube
while that to horizontal part is provided by excess pressure at B.
Thus, (P) A = (mass of horizontal part of liquid) (r2)
Here, mass of horizontal part = (0.2 ă x) A
r = distance of centre of mass of horizontal portion of liquid from axis of rotation
0.2 ă x 0.2 x
= x
2 2
0.1 0.2 x 2
(1.01 103) + gx ă (1.01 103) = (0.2 ă x)
0.1ă x 2
Example 7 P
v Q
Water flows into a bent pipe of cross-section A with velocity v as shown
in the diagram. If the flow is assumed to be streamline and the pipe PQR 90Ĉ
R
is on a horizontal plane, find the force acting on the pipe (0 = density of
water). v
Solution :
2
2
2
= ă A0 v jă A0 v i ă A0 v i j
= ă 2 A0 v 2 i j
2 2
FLUID MECHANICS
50 QUIZRR
Fpw = Force exerted by water on pipe
i j
2
i j
= ă F wp 2A0 v 2 2A0 v n; where n
2 2 2 2
Example 8
What is the excess pressure inside a bubble of soap solutions of radius 5.00mm ? Given that
the surface tension of soap solution at the temperature (20ĈC) is 2.50 10ă2 N/m. If an air
bubble of the same dimension were formed at a depth of 4.0 cm inside a container containing
soap solution (relative density 1.20), what would be the pressure inside the bubble ? (1atm
= 1.01 105Pa)
Solution :
r = 5.00 10ă3m, T = 2.50 10ă2 N/m, h = 40cm = 0.4m,
= 1.20 103 kg/m3 and P0 = 1.01 105 Pascal.
Excess pressure inside a bubble of soap solution
4T 4 2.50 10 ă2
20Pa
r 5.00 10 ă3
2T 2 2.50 10 ă2
10Pa
r 5.00 10ă3
2T
Total pressure inside the air bubble = P0 + hg +
r
Example 9
Two separate air bubbles (radii 0.004 m and 0.002m) formed of the
same liquid (surface tension 0.07 N/m) come together to form a r2
double bubble. Find the radius and the sense of curvature of the
internal film surface common to both the bubbles. P
P2
P1
r1
FLUID MECHANICS
QUIZRR 51
Solution :
4T
P1 = P0 r
1
4T
P2 = P0
r2
r2 < r1
P2 > P1
i.e., pressure inside the smaller bubble will be more. The excess pressure
r1 ă r2
P = P2 ă P1 = 4T ...(i)
r1 r2
This excess pressure acts from concave to convex side, the interface will be concave towards
smaller bubble and convex towards larger bubble. Let R be the radius of interface then,
4T
P= ...(ii)
R
r1 r2 0.004 0.002
R =
r1 ă r2 0.004 ă 0.002
= 0.004m
Example 10
b Tsin
4bT = b2v2
R
T
T
4T
R = v2 Tsin
FLUID MECHANICS
52 QUIZRR
Example 11
Two long capillary tubes of diameter 5.0 mm and 4.0 mm are held vertically with one
inside water. How high will water rise in each tube? (g = 10m/s2, surface tension of water
= 7.0 10ă2 N/m.)
Solution :
Height of water column in a capillary tube of radius r is given by
2T cos
h = ...(1)
rg
where T is surface tension, is density and is angle of contact of water-glass which can be
assumed zero.
2 (7.0 10 ă2 N/m)
h = = 5.6 mm
(2.5 10ă3 m) (1 103 kg/m3 ) (10N/kg)
2T cos
hr = g
= constant
If a liquid rises to a height h1 in a capillary tube of radius r1 and to a height h capillary tube
of radii r2, then
h1 r1 5.6 2.5
or, h2 = r = = 7.0 mm
2 2.0
Example 12
3 kg
Take g = 10m/s2 and density of water = 10 .
m3
FLUID MECHANICS
QUIZRR 53
Solution :
For the equilibrium of gate, net clockwise torque of hydrostatic force
= anticlockwise torque of FA
Torque of hydrostatic force
y = R sin
X
dy = R cos d
x = R cos dy
y R
dA = 2x dy = (2R cos )(R cos ) d
= (2R2 cos2) d
dF = g(8 ă y) dA
= g(8 ă R sin )(2R2 cos2 ) d
= 2gR2 (8 ă R sin )(cos2 ) d
3 2
d = y dF = 2gR (8 ă R sin )(sin cos ) d
/2
C = d
0
/2
= 2gR 3 (8 R sin )(sin cos ) d
2
/2
8 3
= 5.4 105 ă N ă m
3 16
A = 3FA ...(ii)
FLUID MECHANICS
54 QUIZRR
Example 22
A cylindrical weir has a diameter of 3m and a length of 6 m. Find the magnitude of the
resultant force acting on the weir from the water. Take g = 10m/s2 and density of water
= 1000 kg/m 3.
0m
3.
3.0m
D=
1.5m
Solution :
Hydrostatic force from left side
dA = (6) R d
h = R(1 ă sin )
dF = (gh)dA = 6gR2 (1 ă sin ) d
dFx = dF cos = 6gR2 cos (1 ă sin ) d
and dFy = ă dF sin = ă 6gR2 (1 ă sin ) sin d
+ /2 / 2
Fx =
ă /2
d Fx = 6gR 2
ă/ 2
cos (1 sin ) d
h dF
/2
cos 2
= 6gR 2 sin
4 / 2
or Fx = 12gR2 ...(i)
/ 2 /2
Similarly, Fy =
ă / 2
d Fy = 6gR 2 (1 sin ) sin d
ă / 2
/ 2
sin 2
= 6gR 2 cos
4 2ă/ 2
= ă 3gR2 ...(ii)
FLUID MECHANICS
QUIZRR 55
/2
= ă 3gR2 ...(iii)
2 2
dFy = dF sin = 3gR (2 sin ) d
/2
Fy = 3gR 2
(1 cos2) d
0
3
= gR 2 ...(iv)
2
9
(Fy) net = gR 2
2
2
9
= gR 81 = 16.76gR2
2
Fnet
2
= (16.76)(103)(10)(1.5)2
= 377100 N 377 kN
FLUID MECHANICS
ELASTICITY
QUIZRR 3
ELASTICITY
I llu st r a t ion
Ć The force exerted by the spring to oppose the deforming action is the restoring force (F2)
Ć By NewtonÊs third Law, we can say that F1 & F2 are equal and opposite forces; tuns forming
an Action-Reaction pair.
2. ELASTICITY
The property of a body due to which it opposes the action of the deforming forces is called as
Elasticity.
Ć A material is said to be elastic if it returns back to its original shape or size, when the
deforming forces are removed. Plastic materials on the other hand, remain permanently
distorted when the deforming forces are removed.
Ć There are a number of elastic modulii (youngÊs modulus, bulk modulus, shear modulus) that
measure the response of an elastic solid which is acted upon by the deforming forces.
Ć The deforming force is measured (described) in terms of a physical quantity, the stress, that
it develops in the body. The deformation of the solid is described in terms of a physical
quantity the strain, that is created in the body as a result of deformation force.
When an external force is applied to a body then at each cross section of the body an internal
restoring force is developed which tends to restore the body to its original state. The internal
ELASTICITY
4 QUIZRR
restoring force per unit area of cross section of the deformed body is called stress. It is usually
denoted by (sigma).
Restoring force
Thus, Stress () =
Area
Depending upon the way the deforming forces are applied to a body, there are three types of
stress : longitudinal stress, shearing stress and volume stress.
The body of figure is in static equilibrium under an arbitrary set of external forces. In Fig. (b),
we see the same body with an imaginary sectional cut at CC´. Since each of the two individual
parts of the body is also in static equilibrium, both internal forces and internal torques are
developed at the cross section. Those on the right portion are due to the left portion and vice-
F1 F1 C C
C F2 F F2
Ft
F3 Fn F3
ăFn
F4 ăFt F4
F5 F5 ăF
C´ C´ C´
(a) (b) (c)
versa. On the left portion, the normal and tangential components of the internal forces are Fn
and Ft respectively, and the net internal torque is . From NewtonÊs third law, the right portion
is subjected at this same cross section to force components ă Fn and ă Ft and the torque ă . We
define the normal stress or longitudinal stress over the area as,
Fn
n
A
and the tangential stress or shearing stress over the area as,
Ft
t
A
Here, A is the cross-section area of the body at CC´. The longitudinal stress can be two types. The
two parts of the body on two sides of a cross section may pull each other. The longitudinal stress
is then called the tensile stress. This is the case when a rod or a wire is stretched by equal and
ELASTICITY
QUIZRR 5
opposite forces. In case of tensile stress in a wire or a rod, the force Fn is just the tension.
ăF F
ăF Fn ăFn F
Fn = F
If the rod is pushed at the two ends with equal and opposite forces, it will be under compression.
Taking any cross-section of the rod the two parts on the two sides push each other. The longitudinal
stress in this case is called the compressive stress.
F ăF
ăFn Fn ăF
F
Fn = F
Volume St r ess
F
v
A
which is same as the pressure. This is the case when a body is immersed in a liquid.
3 . 2 S t r a in
Consider a body of square cross section ABCD. Four forces of equal magnitude F are applied as
shown in figure. Net resultant force and net torque is zero. Hence, the body is in translational
as well as rotational equilibrium. Because of the forces the shape of the cross section changes from
a square to a parallelogram.
x
F
A B A A´ B B´
F F
x
D C
F D C
A´
ELASTICITY
6 QUIZRR
We define the shearing strain as the displacement of a layer divided by its distance from the fixed
layer. Thus, shearing strain
x
x
T yp es of st r ain
displacement of a surface
l under a tangential force
2. Shearing strain
l perpendicular distance of the
displaced surface from the
fixed surface
change in volume ( v)
3. volumetric strain
original volume v
I llust r at ion 1
A cube is subjected to pressure of 5 ï 105 N/m2. Each edge of the cube is shortened by 1%.
Solution :
Volume stress = P = 5 ï 105 N/m2
volume strain = ?
Let ÂlÊ be the length of cube (undeformed)
99
distorted length = l
100
V = Vf ă Vi
3
99
V = l ă l3
100
3
V 99
volume strain = 1
Vi 100
0.03
ELASTICITY
QUIZRR 7
If the deforming force tries to change the shape of a body, a shear stress is developed in the body.
In the diagram, we see a force F parallel to the surface ABCD of area A, deforms the rectangular
area ABPQ into a parallelogram. D C
A B
[Let AA´ = Dx & AQ = y]
Hence due to F, the shape of the body is altered.
shearing force
shear stress = Q
cross section of ABCD P
F
=
A
A x B
shear strain is defined as : B´
x Y
shear strain = tan ( is very small)
y
Q P
I llust r at ion 2
A block of weight 15 N slides across a horizontal table, the coefficient of sliding friction =
0.4. The area of the block in contact with table is 0.05 m2.
Shear stress = ømg/A
motion
0.4 15
=
0.05
= 120 N/m2 ømg
4. HOOKE’S LAW
If the deforming forces are within a limit (known as elastic limit), the stress created in the body
is proportional to the resulting strain.
i.e. stress strain.
stress
The ratio is known as modulus of Elasticity.
strain
According to various types of stresses, we have three modulii of elasticity.
ELASTICITY
8 QUIZRR
( a ) Young’s M od ulus Y
If a body is deformed by applying forces along one dimension only, then within limits of
proportionality, the ratio of linear stress either compressive or tensile to the longitudinal strain
is called the YoungÊs modulus of the material of the body and is represented by the letter Y,
Linear stress F
i.e., Y = A
Longitudinal strain
Thus, if a rod or wire of length L and cross-sectional area A under the action of a
stretching force F applied normally to its face suffers a change L in its length, then L
in equilibrium :
F F L
Tensile stress =
area A
F
Change in length L
and Longitudinal strain =
Original length L
Tensile stress
Y =
Longitudinal strain
F/A FL
= ...(2)
L/L A L
(b ) Bulk M od ulus
V
Volume strain =
V
ELASTICITY
QUIZRR 9
p p
= V
V/V V
The negative sign shows that with increase in pressure by p, the volume decreases by V, i.e.,
if p is +ive V is ăive. By including minus sign in its definition, the bulk modulus itself is a
positive quantity. The reciprocal of bulk modulus is called compressibility, i.e.,
1 1 V
Compressibility =
V p
All the states of matter possess volume elasticity. Bulk modulus of gases is very low while that
of liquids and solids is very high.
Within limits of proportionality, the ratio of tangential stress to the shearing strain is called
modulus of rigidity of the material of the body and is denoted by , i.e.,
Shearing stress
Shearing strain
In this case the shape of a body changes but its volume remains unchanged.
A x A´ C C´
F
L
F
B D
Consider a cube of material fixed at its lower face and acted upon by a tangential force F at its
upper surface having area A. The shearing stress, then, will be
F|| F
Shearing stress =
A A
This shearing force causes the consecutive horizontal layers of the cube to be slightly displaced
or sheared relative to one another : each line such as AB or CD in the cube is rotated through
an angle by this shear. The shearing strain is defined as the angle in radians through which
a line normal to a fixed surface has turned. For small values of angle,
AA´ x
Shearing strain = =
AB L
ELASTICITY
10 QUIZRR
Note : Poisson discovered that within limits of proportionality the ratio of the lateral
strain to longitudinal strain is constant for a given material. This constant in his honour
is called PoissonÊs ratio and is represented by . It has no units and dimensions. It has
been established that theoretically ă 1 < < (1/2) while practically no substance has
been found for which is negative, i.e., practically 0 < < (1/2).
(Modulus of Rigidity)
3 1 1
Y 3K
ELASTICITY
QUIZRR 11
St r ess-St r ain C ur ve
Elastic limit
Proportional or D
limit yield point
E
Fracture point
C
P
A Plastic behaviour
Stress
Elastic behaviour
Permanent set
0 O´ Strain 30%
AO = Elastic Range
P = Yield point
OD = Breaking stress or tensile stress
E = Breaking point
OO1 = Permanent set
When the stress-strain relationship in a wire is studied, one finds that stress is directly proportional
to the strain upto the point A (see the graph). The point ÂAÊ is called the Limit of proportionality
and AO is called the elastic range. The HookeÊs law is valid up till A. Beyond A, if the stress is
removed, graph between stress and strain does not follow AO. OO´ represents the permanent set.
Notice that beyond ÂAÊ, the stress-strain graph is a curve and that for a small stress, large strain
is produced in the material. The material beyond A and upto ÂPÊ is partly plastic in behaviour.
Beyond ÂPÊ the behaviour of the wire is very erratic. There is a large increase in the strain but
a very small change in the stress.
At this stage, the wire flows down upto the point C. The point ÂPÊ, when the wire yields to the
applied stress and begins to flow, is called the yield point. The region PD is called the plastic
region. Materials used to make sheets or wires must have a longer plastic region and must be
ductile.
Beyond C, the graph has a hump at D. Even if the wire is loaded by a little amount, the wire
becomes thin at weak portions of the wire and tends to break at E. The stress corresponding to
the breaking point is called the breaking stress. Britile substances generally have a small plastic
region and the breaking stress lies closer to the elastic limit.
ELASTICITY
12 QUIZRR
Point s t o R ememb er
stress
E=
strain
Following conclusions can be made from the above expression :
(i) E stress (for same strain), i.e., if we want the equal amount of strain in two different
materials, the one which needs more stress is having more E.
1
(ii) E (for same stress), i.e., if the same amount of stress is applied on two different
strain
materials, the one having the less strain is having more E. Rather we can say that, the one
which offers more resistance to the external forces is having greater value of E. So, we can
see that modulus of elasticity of steel is more than that of rubber or
E steel > Erubber
x
(iii) E = stress for unit strain 1 or x x , i.e., suppose the length of a wire is 2 metre,
x
then the YoungÊs modulus of elasticity (Y) is the stress applied on the wire to stretch the wire
by the same amount of 2 metre.
2. The material which has smaller value of Y is more ductile, i.e., it offers less resistance in
framing it into a wire. Similarly the material having the smaller value of B is more malleable.
Thus, for making wire we choose a material having less value of Y.
3. A solid will have all the three modulii of elasticity Y, B nd . But in case of a liquid or a
gas only B can be defined as a liquid or a gas can not be framed into a wire or no shear
force can be applied on them.
4. For a liquid or a gas,
dP
=
dV / V
CP
Similarly, for x or PV = constant (adiabatic process) B = P.
CV
ELASTICITY
QUIZRR 13
i.e., adiabatic bulk modulus of a gas (denoted by s) is equal to times the pressure of the
gas at that instant of time or s = P
6. For a gas P
whether it is an isothermal process or an adiabatic process. Physically this can be understood
as under :
A B
P1 P2
Suppose we have two containers A and B. Some gas is filled in both the containers. But the
pressure in A is more than the pressure in B, i.e.,
P1 > P2
So, bulk modulus of A should be more than the bulk modulus of , or 1 > 2
and this is quiet obvious, because it is more difficult to compress the gas in chamber A, i.e.,
it provides more resistance to the external forces. And as we have said in point number 1
(ii) the modulus of elasticity is greater for a substance which offers more resistance to
external forces.
7. If a spring is stretched or compressed by an amount l, the restoring force produced in it
is,
Fs = K l ...(i)
Here, K = force constant of spring
Similarly, if a wire is stretched by an amount l, the restoring force produced in it is,
YA
F l ...(ii)
l
Comparing Eqs. (i) and (ii), we can see that force constant of a wire is,
YA
K= ...(iii)
l
YA
i.e., a wire is just like a spring of force constant . So all formulae which we use in case
l
of a spring can be applied to a wire also.
From Eq. (iii), we may also conclude that force
constant of a spring is inversely proportional to the l , 2k l , 2k
2 2
length of the spring l or, K 1 l, k
l
i.e., if a spring is cut into two equal pieces its force
constant is doubled.
ELASTICITY
14 QUIZRR
8. When a pressure (dP) is applied on a substance its density is changed. The change in
density can be calculated as under :
mass
( = density)
volume
1
or (mass = constant)
V
´ V V
V´ V + dV
V
or ´
V + dP
V dP
= as B =
V dP / B V dV / V
´ =
dP
1
B
From this expression we can see that ´ increases as pressure is increased (dP is positive) and vice-
versa.
E xamp le 1 A
A bar of mass m and length l is hanging from point A as shown in figure. Find the
increase in its length due to its own weight. The YoungÊs modulus of elasticity of
the wire is Y and area of cross section of the wire is A.
B
Solution :
Consider a small section dx of the bar at a distance x from B. The weight of the bar for a length
x is,
mg A
W = x
l
ELASTICITY
QUIZRR 15
Total elongation in the bar can be obtained by integrating this expression for x = 0 to x = l.
xl mg l
l x 0 dl
lAY
0 x dx
mgl
or l Ans.
2AY
E xamp le 2
A B B C C D
5t 5t 2t 2t 1t 1t
1
Using the equation l F1 l1 F2 l2 F3 l3 with usual notations
AY
1
l = [5 60 2 100 1 120]
10 8 102
= 0.0775 cm Ans.
l, , Y, A
A B
l
ELASTICITY
16 QUIZRR
Let l = length of rod
A = area of cross section of the rod
Y = YoungÊs modulus of elasticity of the rod
and = thermal coefficient of linear expansion of the rod
Let the temperature of the rod is increased by an amount t. The length of the rod would had
increased by an amount l, if it were not fixed at two supports. Here
l = l t
But since the rod is fixed at the supports a compressive strain will be produced in the rod. Because
at the increased temperature, the natural length of the rod is l + l, while being fixed at two
supports its actual length is l. Hence thermal strain
l l t
= t
l l
or = t
Therefore, thermal stress = Y (stress = Y ï strain)
or = Yt
or force on the supports,
F= A = YA t
This force F is in the direction shown below :
F F F F
ELASTIC ENERGY
When an elastic body is deformed, work is done by the applied force. This work is stored as elastic
potential energy and is released when the body returns back to its original shape or size.
We have already come across an example of elastic potential energy in case of a compressed or
stretched spring. The general expression for the elastic potential energy is a unit volume of a
deformed body is given by:
Elastic energy stored per unit volume = 1/2 (stress) (strain)
= 1/2 (modulus of elasticity) (strain)2
2
Energy stored 1 l
= Y
volume 2 l
1 YA
. l
2
Total energy =
2 l
1
Total energy = F l
2
ELASTICITY
QUIZRR 17
TORSION
Consider a cylinder whose upper end is rigidity fixed and the
other end is twisted through an angle about the axis of the fixed
cylinder. The twisted cylinder exerts a restoring torque given by
: l
r 4
then C is known as the torsional rigidity.
2l
E xamp le 3
(a) A metal wire 75 cm long and 0.13 cm is diameter stretches 0.035 cm, when a load of 8
kg is hung on its ends. Find the stress, strain and YoungÊs modulus.
(b) A solid cylindrical steel column is 4 m long and 9 cm in diameter. What will be its
decrease is length when carrying a load of 80,000 kg ? Y = 1.9 ï 1011 N/m2.
(c) A box shaped piece of gelatine dessert has a top area of 15 cm2 and a height of 3 cm.
When a shearing force of 0.05 N is applied to the upper surface, the upper surface
displaced 4.0 mm relative to the bottom surface. What are the shearing stress, shearing
strain and shear modulus?
(d) Compute the volume change of a solid copper cube, 40 mm on each edge, when subjected
to a pressure of 2 ï 107 N/m2. The bulk modulus of copper is 1.25 ï 1011 N/m2.
Solution :
F 8 9.8
(a) stress =
A 2
p 6.5 10 4
l 0.035
stress = = 4.67 ï 10ă4
l 75
ELASTICITY
18 QUIZRR
(b) crosssection area of cylinder = r2
= (0.045)2 = 6.36 ï 10ă3 m2
l =
Fl
8 104 9.8 4
AY
6.36 103 1.9 1011
= 2.6 ï 10ă3 m
= 2.6 mm
= 333 N/m2
x displacement
shear strain = =
y height
0.4
= 0.133
3
shear stress
(shear modulus) =
shear strain
333
=
0.133
= 2500 N/m2
V V
volume strain =
V 0.033
p
k (bulk modulus) =
V / V
V
7
pV 2 10 (0.04)
3
k 1.25 1011
= ă 1.024 ï 10ă8 m3
= ă 10.24 mm2
ELASTICITY
QUIZRR 19
E xamp le 4
A wire of radius r stretched with out tension, along a straight line is tightly fixed at A &
B. A mass m is suspended from the mid point of the wire. Due to the weight of mass, the
wire is pulled into the shape ACB. Find the depression ÂdÊ in the wire. The length of the wire
is 2l and its youngÊs modulus is Y.
2l
A B
C
m
Solution :
Let T be the tension is the wire.
2T sin = mg
l l mg
A B T =
2sin
d
T T
T mg mg
mg stress =
A 2A sin 2r 2 sin
l d 2 l2 l
strain =
l l
1/
2
d2 d2
= 1 2 1
l 2l 2
d d
As sin =
2
d l 2 l
mg l
stress 2 r 2 d
Y
strain d 2 / 2l 2
1/
mg l3 mg 3
Y dl 2
r 2d3 r Y
ELASTICITY
20 QUIZRR
E xamp le 5
1.2 10 5 10
11
3
2
9 10 0.1 10
3 3
=
2 2 4.2
= 0.099ĈC
E xamp le 6
A thin ring of radius R is made of a material of density and YoungÊs modulus Y. If the ring
is rotated about its centre in its own plane with angular velocity , find the small increase
in its radius.
Solution :
Consider an element PQ of length dl. Let T be the tension and A the area of cross section of the
wire.
Mass of element dm = volume ï density
= A (dl)
The component of T, towards the centre provides the necessary centripetal force
2T sin = (dm)R2 ...(i)
2
sin
dl / R
For small angles =
2 2 2
Substituting in Eq. (i), we have
dl
T. A dl R2
R
or T = A2 R2
ELASTICITY
QUIZRR 21
R
T.R
=
A R R
2 2
AY AY
2 R 3
or R = Ans.
Y
E xamp le 7
A steel rod of length 6.0 m and diameter 20 mm is fixed between two rigid supports. Determine
the stress in the rod, when the temperature increases by 80ĈC if
(a) the ends do not yield
(b) the ends yield by 1 mm
Take Y = 2.0 ï 106 kg/cm2 and = 12 ï 10ă6 perĈC
6.0 m
Solution :
Given length of the rod l = 6 m = 600 cm
Diameter of the rod d = 20 mm = 2 cm
Increase in temperature t = 80ĈC
YoungÊs modulus Y = 2.0 × 106 kg/cm2
and thermal coefficient of linear expansion = 12 ï 10ă 6
perĈC
When the ends do not yield
Let, 1 = stress in the rod
Using the relation = tY
1 = (12 ï 10ă6)(80)(2 ï 106)
= 1920 kg/cm2 Ans.
ELASTICITY
22 QUIZRR
When the ends yield by 1 mm
Increase in length due to increase in temperature
l = lt
of this 1 mm or 0.1 cm is allowed to expand. Therefore, net compression in the rod
lnet 0.1
= t
l
or compressive strain in the rod,
l
0.1
stress 2 = Y Y t
l
6 0.1
2 = 2 ï 106 12 10 80
600
E xamp le 8
What is the density of lead under a pressure of 2.0 ï 108 N/m2, if the bulk modulus of lead
is 8.0 ï 109 N/m2 and initially the density of lead is 11.4 g/cm3 ?
Solution :
The changed density,
´ =
dp
1
B
Substituting the value we have
11.4
´ =
2.0 108
1
8.0 109
E xamp le 9
A rubber cord has a cross sectional area 1 mm2 and total unstretched length 10.0 cm. It is
stretched to 12.0 cm and then released to project a missile of mass 5.0 g. Taking YoungÊs
modulus Y for rubber as 5.0 ï 108 N/m2. Calculate the velocity of projection.
ELASTICITY
QUIZRR 23
Solution :
Equivalent force constant of rubber cord.
k
YA
=
(5.0 108 ) 1.0 10 6
l (0.1)
1 1
k. l = mv2
2
2 2
k
v = m .l
5.0 103
(12.0 10.0) 102
= 3
5.0 10
= 20 m/s Ans.
Note : Following assumptions have been made in this example.
(i) k has been assumed constant, even though it depends on the length (l).
(ii) The whole of the elastic potential energy is converting into kinetic energy of missile.
E xamp le 10
A sphere of radius 0.1 m and mass 8 kg is attached to the lower end of a steel wire of length
5.0 m and diameter 10ă3 m. The wire is suspended from 5.22 m high ceiling of a room. When the
sphere is made to swing as a pendulum, it just grazes the floor at its lowest point. Calculate
the velocity of the sphere at the lowest position. YoungÊs modulus of steel is 1.994 ï 1011 N/m2.
Solution :
Let l be the extension of wire when the sphere is at mean position. Then, we have
l + l + 2r = 5.22
or l = 5.22 ă l ă 2r
= 5.22 ă 5 ă 2 ï 0.2
= 0.02 m
Let T be the tension in the wire at mean position during oscillation, then
T/A
Y = 5.22 m T
l / l
YAl Yr 2 l
T =
l l
ELASTICITY
24 QUIZRR
Substituting the values, we have
T =
1.994 10 0.5 10
11 3 2
0.02
5
= 626.43 N
The equation of motion at mean position is,
mv2
T mg ...(i)
R
25.13 v2
626.43 25.13 9.8
5.12
E xamp le 11
A body of mass 3.14 kg is suspended from one end of a wire of length 10.0 m. The radius of
the wire is changing uniformly from 9.8 ï 10ă4 m at one end to 5.0 ï 10ă4 m at the other end.
Find the change in length of the wire. What will be the change in length if the ends are
interchanged ? YoungÊs modulus of the material of the wire is 2 ï 1011 N/m2.
Solution :
Consider an element of length dx at a distance x from the fixed end; then by definition of Y,
change in the length of the element will be
a
F dx
dy [as here L dx]
YA
x
2
But here A = r r
L dx
= (a + x tan )2
So total change in length of wire
b
L F L dx
L = 0 dy
Y 0 F
a x tan 2
ELASTICITY
QUIZRR 25
L
F x L 2 F 1
L =
Y tan x 0
t dt
Y tan a x tan 0
FL FL
or L = a a + L + tan Y ab Y
3.14 9.8 10
So L = [as F = Mg]
3.14 9.8 10 4 5 104 2 10
11
E xamp le 12
A thin uniform metallic rod of length 0.5 m and radius 0.1 m rotates with an angular
velocity 400 rad/s in a horizontal plane about a vertical axis passing through one of its
ends. Calculate tension in the rod and the elongation of the rod. The density of material
of the rod is 104 kg/m3 and the YoungÊs modulus is 2 ï 1011 N/m2.
Solution :
(a) Consider an element of length dr at a distance r from the axis
of rotation as shown in Fig. The centripetal force acting on this
element will be
r dr
dT = dmr2 = (A dr)r2
L
As this force is provided by tension in the rod (due to elasticity),
so the tension in the rod at a distance r from the axis of rotation will
be due to the centripetal force due to all elements between
x = r to x = L
L 1
i.e., T= r A2 r dr A2 L2 r 2 ...(1)
2
2
1 2 2 1
So here T= 10 10 (400) r 2
4
2 2
ELASTICITY
26 QUIZRR
6 1 2
= 8 10 r N Ans.
4
Note : The tension in the rod will not be constant but will vary from point to point. At
the free end, i.e., r = L, it will be min = 0 while at the other end r = 0, it will be
max = 2 ï 106 N.
(b) Now if dy is the elongation in the element of length dr at position r where tension is T, by
definition of YoungÊs modulus,
dy T stress
as strain = Y
dr AY
1 2 2
dy = L r 2 dr
2 Y
2 1 2 L3
L =
Y
L
0
L2 r 2 dr
3 Y
E xamp le 13
A light rod of length 200 cm is suspended from the ceiling horizontally by means of two
vertical wires of equal length tied to its ends. One of the wires is made of steel and is of
cross-section 0.1 cm2 and the other of brass of cross-section 0.2 cm2. Along the rod at which
distance may a weight be hung to produce (a) equal stresses in both the wires (b) equal
strains in both the wires ? Y for brass and steel are 10 ï 1011 and 20 ï 1011 dyne/cm2
respectively.
Solution :
(a) As stresses are equal
T1 T T1 A1 0.1
2 , i.e. or T2 = 2T1 ...(1)
A1 A 2 T2 A 2 0.2
ELASTICITY
QUIZRR 27
W
T A Y 0.1 20 1011
So 1 1 1 1, i.e., T1 = T2 ...(3)
T2 A 2 Y2 0.2 10 1011
E xamp le 14
Two rods of different metals, having the same area of cross-section A, are placed end to end
between two massive walls as shown in Fig. The first rod has a length l 1, coefficient of
linear expansion 1 and YoungÊs modulus Y1. The corresponding quantities for second rod
are l 2, 2 and Y2. The temperature of both the rods is now raised by T degrees. (a) Find the
force with which the rods act on each other at the higher temperature in terms of the given
quantities. (b) Also find the lengths of the rods at the higher temperature. Assume that
there is no change in the cross-sectional area of the rods and the rods do not bend. There
is no deformation of walls.
Solution :
(a) Due to heating the increase in length of the composite rod will be
(L)l = (L11 + L22)T [as L = L]
and due to compressive force F from the walls due to elasticity, decrease in length will be
L L F FL
L D 1 2 as L = AY
Y1 Y2 A
ELASTICITY
28 QUIZRR
As the length of the composite rod remains unchanged the increase in length due to heating must
be equal to increase in length due to compression, i.e.,
F L1 L2
= (L11 + L22)T
A Y1 Y2
A L11 L2 2 T
or F = L / Y L / Y ...(1)
1 1 2 2
(b) As initially the length of one rod is L1 and due to heating it increases by (L1)H = (1L1T) while
due to compression it decreases by (L1)C = (FL1/AY1)
L1 L2
1 Y1 2 Y2
E xamp le 15
A 5 m long cylindrical steel wire with radius 2 ï 10ă3 m is suspended vertically from a rigid
support and carries a bob of mass 100 kg at the other end. If the bob gets snapped, calculate
the change in temperature of the wire ignoring radiation losses. Take g = 10 m/s2.
(For the steel wire : YoungÊs modulus = 2.1 ï 1011 N/m2.
Density = 7860 kg/m3; specific heat = 420 J/kg-K)
Solution :
Given, Length of the wire, l = 5 m
Radius of the wire, r = 2 ï 10ă3 m
Density of wire, = 7860 kg/m3
l
YoungÊs modulus, Y = 2.1 ï 1011 N/m2
and Specific heat, s = 420 J/kg-K
M = 100kg
Mass of wire, m = (density)(volume)
ELASTICITY
QUIZRR 29
= ()(r2l)
= (7860)()(2 ï 10ă3)2 (5) Kg
= 0.494 kg
Elastic potential energy stored in the wire,
1
U= (Stress)(Strain)(Volume)
2
1 Mg l 2 Energy 1
U=
2 r 2 l
(r l) Volume 2 stress strain
or
1 Fl
= Mg . l l AY
2
1 Mgl 1 M2 g 2 l
Mg
= 2
2
r 2 Y 2 r Y
1 100 10 5
2 2
J
U = 2 (3.14)(2 103 ) 2 2.1 1011
= 0.9478 J
When the bob gets snapped, this energy is utilised in raising the temperature of the wire,
So, U = ms
U
0.9478 C
= or K
ms 0.494 420
ELASTICITY
GRAVITATION
QUIZRR 3
I ntroduction :
F12 F21
A r B
m1 m2
F
r2
Gm1m2
F
r2
In vector notation
Gm1 m2
F12 2
r→12
r12
GRAVITATION
4 QUIZRR
Characterstics of gravitational force :
(i) It is conservative in nature
(ii) It is a central force
(iii) It is independent of the medium between the particles
Example 1
6
Solution : F = 4mgf = 4 10 10
5
= 4 10 N
Gm1 m2
F=
r2
G m1 m2 6 10 11 40 60
r
F 4 105
r = .06 m = 6 cm
Gravity : If one of the bodies in expression of NewtonÊs law of gravitation is earth than the
gravitation is called gravity. Hence gravity is force by which earth attracts a body towards its
centre.
When a body is dropped from a certain height above the ground it begins to fall towards the earth
under gravity. The acceleration produced in the body due to gravity is called the acceleration due
to gravity. It is denoted by g. Its value close to the earthÊs surface is 9.8 m/s2.
Suppose that the mass of the earth is M, its radius is R, then the force of attraction acting on a
body of mass m close to the surface of earth is
GMm
F=
R2
According to NewtonÊs second law, the acceleration due to gravity
F GM
g
m R2
In vector form,
GM →
g 2 R
R
GRAVITATION
QUIZRR 5
where R is position vector from earthÊs centre to the body of mass m.
Body of mass m
R
Earth of mass m
This expression is free from m. If two bodies of different masses are allowed to fall freely they will
have the same acceleration, i.e., if they are allowed to fall from the same height, they will reach
the earth simultaneously.
The value of acceleration due to gravity changes with height (i.e., altitude), depth, shape of the
earth and rotation of earth about its own axis. The effect of each of the above factors on the value
of g has been discussion below.
(a) Effect of Altitude : Consider earth to be a sphere of mass M, radius R with centre at O.
Let g be the value of acceleration due to gravity at a point A on the surface of earth.
B g´
h
A g
GM
g= ...(1)
R2
If g´ is the acceleration due to gravity at a point B, at a height h above the surface of earth,
then
GM
g´
...(2)
R + h 2
GRAVITATION
6 QUIZRR
Dividing (1) by (2), we get
g´ GM R2 R2
= ...(3)
g
R + h 2 GM R h2
2
R2 h
= 1
R 2 1 h / R
2
R
If h << R then h/R is very small as compared to 1. Expanding the R.H.S. of above equation
by Binomial theorem and neglecting the square and higher powers of h/R, we get
g´ 2h
g
= 1
R
2h
g´ = g 1
R
or ...(4)
So, we note that the value of acceleration due to gravity decreases with height.
It is due to this reason that the value of acceleration due to gravity is lesser at mountains
than in plains. At a height equal to the radius of the earth (i.e. h = R = 6400 km), from (2)
g R2 g
g´ =
R R 2 4
Important notes :
(i) The relation (3) is used to find the value of acceleration due to gravity at a height when
h is comparable to the radius of earth R and relation (4) is used to find g´ when h is
very small as compared to R.
(ii) With height h, the decrease in the value of g is = g g´ = 2h g/R
Fractional decrease in the value of g
g g´ 2 h
=
g R
g g´ 2h
= 100 100
g R
GRAVITATION
QUIZRR 7
(b) Effect of depth : Consider earth to be a homoogeneous sphere of radius R and mass M with
centre at O. Let g be the value of acceleration due to gravity at a point A on the surface
of earth, then
A
d B
R (R-d)
O
GM
g
R2
4
M= R3
3
4
G R3
g = 3 4
2
GR ...(5)
R 3
Let g´ be the acceleration due to gravity at the point B at a depth d below the surface of
earth. The body at B will experience gravity pull due to shaded portion of earth whose
radius is (R d) and mass is M´.
GM´
g´ = R d 2
4
R d
3
and M´ =
3
4
R d
3
G
3
g´ =
R d 2
4
= G R d ...(6)
3
GRAVITATION
8 QUIZRR
Dividing (6) by (5), we get
4
G R d
g´ 3
g
= 4
GR
3
Rd R d
=
R R R
d
or g´ = g 1 ...(7)
R
From (7) we note that the value of acceleration due to gravity decreases with depth.
At the centre of the earth, d = R, g´ = g0 (say).
R
From (7), we get g0 g 1 0
R
It means the acceleration due to gravity is zero at the centre of earth. Therefore the weight
of the body of mass m at the centre of earth = m g0 = 0, but the mass of the body will not
be zero.
Thus the value of acceleration due to gravity is maximum at the earthÊs surface and becomes
zero at the centre of the earth.
Note : Decrease in the value of g with depth d is = g g´ = dg/R
Fractional decrease in the value of g with
g g´ d
depth =
g R
g g´ d
= 100 100
g R
GRAVITATION
QUIZRR 9
Thus we conclude that the value of g is least at the equator and maximum at the pole. It
means, the value of acceleration due to gravity increases as we go from equator to the pole.
The earth is rotating about its axis from west to east. So every body on its surface is moving in
a circle i.e., is in accelerated motion. So the effective acceleration due to gravity will be
g´ = g a
where a is the acceleration of the body towards the centre of earth
due to its circular motion.
Pole
Now, for a body at P the centripetal acceleration ar = 2 r along PO´
= 2 R cos (as r = R cos ); so component of ar along PO will be O´ ar P
a
a = ar cos = R 2 cos O
Equator
2 2
Substituting this value of a in the equation g´ = g R cos
This the required result and from this it is clear that for a given ,
ÂgÊ is maximum (= g) when cos = min = 0, i.e. = 90 i.e. at the
pole and minimum (g R2) when cos = max = 1 i.e. = 0 i.e.,
at equator. Further more a given point on the surface of the earth
( = const) g decreases as increases and vice-versa.
Example 2
Assuming earth to be a sphere of uniform mass density, how much would a body weigh half
way down the centre of the earth if it weighed 100 N on the surface ?
Solution :
Given, mg = 100 N
h
g´ = g 1
RE
h 1
RE
=
2
1 g
g´ = g 1
2 2
mg 100
mg´ = 50 N Ans.
2 2
Before moving to the next section think about the previous problem of the body is taken RE/2
distance above earthÊs surface where ÂREÊ is radius of earth. Then the result you will obtain by
GRAVITATION
10 QUIZRR
RE
formula of variation in height will be zero. And the body at distance does not feel weight
2
lessness hence you have done a mistake and that in the approximation while defining formula.
Tip : Always keep in mind while applying formula its conditions and approximations you have
taken while deriving it.
Example 3
Suppose earth has the ability to alter its speed of rotation. At what new time period will
the weight of a body on the equation becomes zero ? Take g = 10 m/s2 and and radius of
earth R = 6400 km (average radius)
Solution :
The weight will become zero when
g´ = 0
or g R2 = 0 (on the equator g´ = g R2)
g
or =
R
2 g
=
T R
R
or T = 2
g
6400 103
Substituting the values, T = 2 10 hr
3600
or T = 1.4 hr
Thus, the new time period should be 1.4 hr instead of 24 hr for the weight of a body to be zero
on the equator.
Gravitation Field
The space around a body in which any other body experiences a force of attraction is called the
gravitational field of the first body.
The force experienced (both in magnitude and direction) by a unit mass placed at a point in a
gravitational field is called the gravitational field strength or intensity of gravitational field at
F
that point. Usually it is denoted by E . Thus, E =
m
GRAVITATION
QUIZRR 11
Field at an External Point : A uniform sphere may be treated as a single particle of same mass
placed at its centre for calculating the gravitational field at an external point. Thus,
GM
Er for r R
r2
1
or E (r)
r2
Here, r is the distance of the point from the centre of the sphere and R the radius of sphere.
GM
of the point from the centre of the sphere. At the centre itself, it is zero and at surface it is ,
R2
where R is the radius of the sphere. Thus, E
GM
Er .r for r R GM
R3 R2 1
E
r2
or E (r) R
Hence, E versus r graph is as shown in Fig. r
R
At an external point : For an external point the shell may be treated as a single particle of same
mass placed at its centre. Thus, at an external point the gravitational field is given by,
GM
Er for r R
r2
GM
E =
R2
E
1
and otherwise E GM
r2
R2 1
E
At an internal point : The field inside a uniform spherical r2
shell is zero. Thus, E versus r graph is as shown in Fig.
r
R
GRAVITATION
12 QUIZRR
( iii) Field due to a Uniform Circular Ring at a point on its Axis
This is directed towards the centre of the ring. It is zero at the centre of the ring and maximum
E
R dE
at r (can be obtained by putting 0 ). Thus, E-r
2 dr
2GM
graph is as shown. 3 3R2
2GM
The maximum value is Emax =
3 3 R2
Gravitation Potential
At a point in a gravitational field potential V is defined as negative of the work done per unit mass
in the shifting a rest mass from some reference point to the given point.
Change in gravitation potential is defined as change in potential energy of system per unit mass.
dU
dV
m
Reference : The work done in bringing a unit mass from infinity to a point in the gravitational
field is called the Âgravitational potentialÊ at the point.
Since, work is obtained hence it is negative. Hence gravitational potential is always negative at
any point with respect to potential at infinity.
Suppose a point mass M is situated at a point O. We want to find the gravitational potential due
to this mass at a point P a distance r from O. For this let us find work done in taking the unit
mass from P to infinity. This will be
GM GM
W=
r
F dr
r r 2
. dr
r M r P
O
GRAVITATION
QUIZRR 13
GM
Hence, the work done in bringing unit mass from infinity of P will be . Thus, the gravitational
r
potential at P will be, V = GM
r
GM
V (r ) = r R
r
GM
At the surface, r=R and V =
R
Potential at internal point : At some internal point, potential at a distance r from the centre
is given by,
( 3/2 ) GM
GM 3 r2 R
V r 3 R2 r R
R 2 2 GM
R
r
GM R
At r = R, V =
R
3 GM V
while at r = 0, V =
2R
i.e., at the centre of the sphere the potential is 1.5 times the potential at surface. The variation
of V versus r graph is as shown in Fig. 7.14.
GM
V (r) = r R
r
GM
at r = R V =
R
GRAVITATION
14 QUIZRR
Potential at an internal point : GM
The potential due to a uniform spherical shell is constant R
throughout at any point inside the shell and this is equal
r
GM R
to .
R
V
(iv) Potential due to a Uniform Ring at a Point on its Axis
O r P
GM
V r
0 r R
R2 r2
GM GM
At r = 0, V , i.e., at the centre of the ring gravitational potential is
R R
GM
R
Gravitational potential is a field function. It depends on the position of the point where potential
is desired. Gravitational field and the gravitational potential are related by the following relation.
E = gradient V = grad V
V → V → V →
= i j k
x y z
V → V → V →
i j k
z
or = ...(i)
E x y
V
Here, = Partial derivative of potential function V w.r.t. x, i.e., differentiate V w.r.t. x assuming
x
y and z to be constant
GRAVITATION
QUIZRR 15
dV
(i) E= , if gravitational field is along x-direction only
dx
(ii) dV = E.dr ,
Here dr dxi→ dyj→ dzk→ and E E xi→ E y →j E z k→
Example 4
Two concentric spherical shells have masses m1 and m2 and radii r 1 and r 2 (r 2 > r 1). What
is the force exerted by this system on a particle of mass m 3 if it is placed at a distance
r (r 1 < r < r 2) from the centre ?
Solution :
The outer shell will have no contribution in the gravitational field at point P
m2
Gm1
EP =
r2 m1
m3
Thus, force on mass m3 placed at P is, O P
r1 r
F = (m3EP)
r2
Gm1 m3
or F = Ans.
r2
The field E P and the force
F both are towards centre O.
Example 5
The gravitation field due to a mass distribution is given by E = k/x 3 in x-direction taking
the gravitational potential to be zero at infinity. Find its value at a distance x.
Solution :
x x k 3k
The potential at a distance x in V( x)
Edx
3
x
dx
x2
Consider a system of two bodies A of mass m1 and B of mass m2. Then its potential energy is
defined by work done in assembling the system.
GRAVITATION
16 QUIZRR
To get its expression, lets consider that body A is fixed and B is moved from infinity to assemble
system.
from infinity
r
A B B
m1 m2 m2
final (Initial)
position
Gm1 m2
m1
F at distance x on m2 due to x2
Gm1m2
dW F . dx = dx
x2
r
Gm1 m2 1
W = x2
dx = Gm1 m2
x
Gm1 m2
W =
r
Gm1 m2
Change in potential energy = of the system.
r
Tip : Whenever you move a body along the direction of force, the potential energy of system
decreases.
Consider a sphere of radius R. During its formation, mass is assembled and in that process some
work is involved and it tries to stabilise the system i.e. sphere. Hence potential energy is decreased
and final potential energy of sphere of radius R and mass M is found by :
Consider a sphere of ÂxÊ is directly formed and a shell of thickness ÂdxÊ is added to it.
Change in potential energy of system
R M x3 M 4 x2
=
0
G
R 3 x 4 / 3 R 3
dx
GRAVITATION
QUIZRR 17
3 GM2 4 R
=
R6 x dx
0
3GM2 R 5 3GM2
=
5R 5R
3GM2
Potential energy of system =
5R
This energy is just due to sphere and hence it is called as self energy.
GM2
Inside self potential energy of solid sphere =
10R
Escape Velocity
It is the minimum velocity with which a body must be projected from the surface of earth so that
it permanently overcomes and escapes the gravitational field of the earth. We can also say that
a body projected with escape velocity will be able to go to a point which is at infinite distance from
the earth.
Let us imagine what happens to a body of mass m if it is thrown from the earth with a velocity
Ve (escape velocity). As the body moves away from the earth, it slows down (due to gravitational
pull of the earth) and hence its kinetic energy is converted into gravitational potential energy of
the mass-earth system. Let us imagine that is just able to reach upto infinity (where G.P.E. is
zero).
earth
m ve
radius = R
mass = M
1
mVe2 G.P.E. f G.P.E.i
2
GRAVITATION
18 QUIZRR
1 GMm
mVe2 0
2 R
2GM
Ve or Ve 2 gR
R
Ve = 11.3 km/s
Hence any object thrown with a velocity of 11.3 km/s or more will escape the gravitational field
of the earth and will never come back to the earth.
Planets : Planets are large natural bodies rotating around the sun due to the gravitational
attraction of the sun. Path of these planets are elliptical with the sun at one focus. However, the
difference in major and minor axis is not large. The orbits can therefore, be treated as nearly
circular for not 700 sophisticated calculations.
Satellites : Satellites are natural or artificial bodies moving around a planet under its gravitational
attraction. Satellites are launched from the earth so as to move round it. To establish the artificial
satellite in the desired orbit, a number of rockets are fired from the satellites at proper time. After
placing in desired orbit, it will continue to move in that orbit under gravitational attraction of the
earth. We shall here now compute and discuss some physical entities of interest for a satellite
undergoing circular motion.
(i) Orbital speed : Let the radius of orbit be r and the speed of the satellite be v.
v0
GMm v02
= m r M r
r2 m
R Satellite
GM Earth
or v0 =
r
The speed of planet is inversely proportional to the square root of the radius of its
orbit.
GRAVITATION
QUIZRR 19
(ii) Time period : The time taken by a satellite in completing one revolution is its time period
T.
2r 2r r3 r
T 2 2
0 GM GM g
r
R 6.4 106
T = 2 2 2 800 s
g 10
GM GM
L = mr mr
r r
i.e. L = (m2GMr)
Note : In case of satellite, force is central, so torque = 0 on hence angular momentum of
satellite is conserved i.e. L = constant.
(iv) Energy : The kinetic energy of planet is
1
K = m 2
2
1 GM GM
or K = m
2 r r
GMm
or K =
2r
GMm
The gravitational potential energy of the earth-satellite system is V =
r
GMm GMm
=
r 2r
GMm
=
2r
GRAVITATION
20 QUIZRR
From above discussion, we obtain
K
= 1 i.e., K = E
E
U
and = 2 i.e. U = 2E
E
i.e., kinetic energy of a satellite in an orbit is equal to negative of total energy while
potential energy is equal to twice the total energy.
+
Satellite
K r
Energy
KE = max a KE = min
O PE = min Focus Semi major PE =max
E r axis
U
rmin rmax
(a)
(b)
Example 6
A satellite is revolving round the earth at a height of 6 105 m. Find
(a) the speed of satellite and
(b) the time period of the satellite
Radius of earth = 6.4 106 m and mass of the earth = 6 1024 kg
Solution :
The height of satellite from earthÊs centre
= 6.4 106 + 6 105 m = 7.0 106 m
GM
(a) The speed of satellite is =
r
2r
(b) The time period is T =
2 7 106
= = 5.8 103 s.
7.6 103
GRAVITATION
QUIZRR 21
Geostationary satellite :
If an artificial satellite revolves around the earth in an equatorial plane with a time period of 24
h in the same sense as that of earth, then it will appear stationary to the observer on earth. Such
a satellite is known as geostationary satellite.
At a given place (g = constant), the period of revolution of earthÊs satellite depends solely on its
height above the surface of the earth. Let us calculate the height of the satellite above the surface
of the earth, so that the satellite will have a period of revolution of 24 h and will appear as a
satellite. The height of the satellite is given by
r3 2 (R h)3
T = 2
gR 2 R g
1/
3
T2R 2 g
or h = 2
R
4
Putting T = 24 h = 24 3600 s,
R = 6400 km = 6.4 106 m, g = 9.8 ms 2, we find
1/
3
(24 3600)2 (6.4 106 )2 9.8
h = 6.4 106
4 2
= 35930 km 36000 km
Thus, a satellite will appear stationary, if it around the earth from west to east in an orbit
coplanar equatorial plane at a height of about 36000 km about surface of earth. Such an orbit
is known as geostationary orbit and the revolving in this orbit is known as synchronous
satellite.
gR 2
Using relation 0 the orbital velocity geostationary satellite comes out to be about 3.08
R h'
km.
Important Features : In case of satellite motion as L = mr = constant, while according to
1
equation for velocity . two results appear to be contradictory. However apparent contradiction
r
1 1
is resolved if we keep in that holds good for different points of the same orbit while
r r
for different orbits as shown in Fig. below
GRAVITATION
22 QUIZRR
v2
v2
P2
P2 r2
r2 P1
v1 O
r1
r1 v1
P1
(a) v 1
r (b) v 1r
Consider a satellite in the circular orbit with the time period TS. TS
The earth also rotates with the time period TE = 24 hrs. If an Te m
observer on the earth sees this satellite, the angular velocity of the
satellite relative to the earth will be :
earth
SE
S e
Hence the time for one revolution will appear to be different from
TS if observed from earth. This time period can be calculated as TSE it
orb
from the above equation.
1. If satellite and earth are rotating in same direction :
2 2 2
=
TSE TS TE
TS TE
TSE = T T
E S
TS TE
TSE
TS TE
KEPLER’S LAWS
The motions of the planets, as they seemingly wander against the background of the stars, have
been a puzzle since, the dawn of history. Johannes Kepler (1571 1601) after a life time of study,
worked out the empirical laws that govern astronomers to make observations without the help of
a telescope, compiled the extensive data from which KeplerÊs ways able to derive the three laws
of planetary motion that now bear his name.
GRAVITATION
QUIZRR 23
1. Law of orbits : Each planet revolves around the sun in an elliptical orbit with the sun at
one focus of the ellipse.
Planet
F1 F2
Sun
2. Law of Areas : This law states that the radius vector from the sun to the planet sweeps
out equal areas in equal time intervals.
Both shaded areas are equal if the time from A to B is equal to the time from P to Q.
P Planet
B
F1
Sun A
Q
3. Law of periods : It states that the square of the time taken by the planet about the sun
is proportional to the cube of the planetÊs mean distance from the sun.
If T be the time period of the planet & r be the mean distance of planet from the sun
(average of maximum and minimum distances from sun).
Binding Energy :
Total mechanical energy (PE + KE) of a closed system is negative. The modulus of this total
mechanical energy is known as binding energy.
Therefore total mechanical energy of the particle of mass m on surface of earth (M) is
GMm
E = K + U =
R
GMm
Binding energy = E
R
In another way we can say that this energy binds a particle to earth i.e. after supply an amount
of energy equal to binding energy, the particle will be free of earthÊs attraction.
Example 7
GRAVITATION
24 QUIZRR
Solution :
For a satellite to remain above a given point on the earthÊs surface, it must rotate with the same
angular velocity as the point on earthÊs surface. Therefore the satellite must rotate in the equatorial
plane from west to east with a time period of 24 hours.
2r r r GM
T = 2r 2r 2 as g 2
0 GM gR R
1/ 1/
3 3
2 9.8 (6.4 106 )2 (86400)2
or r = gR 2 T
42 4 2
Example 8
A space-ship is launched into a circular orbit close to the earthÊs surface. What additional
velocity should not be imparted to the space-ship in the orbit to overcome the gravitational
pull. (Radius of earth = 6400 km and g = 9.8 m/s2)
Solution :
For orbiting the earth close to its surface,
mv2 GMm GM GM
, i.e., v0 gR as g 2
R R2 R R
GMm 1 2GM GM
mve2 , i.e., ve 2 gR as g 2
R 2 R R
So additional velocity to be imparted to the orbiting satellite for escaping = 11.2 8 = 3.2 km/s
GRAVITATION
QUIZRR 25
Example 9
A sky lab of mass 2 103 kg is first launched from the surface of earth in a circular orbit
of radius 2R (from the centre of earth) and then it is shifited from this circular orbit to
another circular orbit of radius 3R. Calculate the minimum energy required (a) to place the
lab in the first orbit, (b) to shift the lab from first orbit to the second orbit. Given, R = 6400
km and g = 10 m/s2.
Solution :
The energy of the sky lab on the surface of earth
GMm GMm
ES = KE + PE = 0 +
R R
1 GMm GMm GM
E m 20 as 0
2 r 2r r
(a) So the energy required to place the lab from the surface of earth to the orbit of radius 2R,
3m 3 GM
i.e. E = gR 2 mgR as g 2
4R 4 R
3 3
i.e. E = (2 103 6.4 106 ) (12.8 1010 ) = 9.6 1010 J
4 4
GMm GMm
EII =
2(3 R) 6R
1 1
or E = mgR (12.8 1012 ) 1.1 1010 J
12 12
GRAVITATION
26 QUIZRR
Example 10
If a satellite is revolving around a planet of M in an elliptic orbit of semi-major axis a, show
that the orbital speed of the satellite when it is at a distance r from the focus will be given by :
2 1
v 2 GM
r a
Solution :
As in case of elliptic orbit of a satellite mechanical energy E = (GMm/2a) remains constant, at
GMm
any position of satellite in the orbit, KE + PE = ...(1)
2a
Now, if at position r, v is the orbital speed of satellite,
1 GMm
KE = mv2 and PE = ...(2)
2 r
So from Eqns. (1) and (2), we have
1 GMm GMm 2 1
mv2 , i.e., v2 GM
2 r 2a r a
Example 11
Two satellites of same mass are launched in the same orbit round the earth so as to rotate
opposite to each other. They soon collide inelastically and stick together as wreckage.
Obtain the total energy of the system before and just after the collision. Describe the
subsequent motion of the wreckage.
Solution :
In case of satellite motion energy of a satellite in an orbit is given by
GMm
E
2r
v M v
So the total energy of the system before collision
m II
GMm m I r
Ei E1 E2 2E
r
As the satellites of equal mass are moving in opposite direction and collides inelastically, the
velocity of wreckage just after collision, by conservation of linear momentum will be
mv mv = 2mV, i.e., V = 0
i.e., just after collision wreckage comes to rest in the orbit. So energy of the wreckage just after
collision will be totally potential and will be
GM(2m) 2GMm
EF
r r
And as after collision the wreckage comes to stand still in the orbit, it will move along the radius
towards the earth under its gravity.
GRAVITATION
QUIZRR 27
SOLVED EXAMPLES
Example 1
The gravitational field in a region is given by E 5i→ 12 j→ N/kg then the magnitude
of the gravitational force acting on a particle of mass 2 kg placed at the origin, will
be ?
Solution :
Given
E 5i→ 12 →j N/kg
Its magnitude E = (5)2 (12) 2 13 N/kg
Thus, F = E m
= 13 2
= 26 N
Example 2
A body is projected vertically upwards from the surface of earth with a velocity sufficient
to carry it to infinity. Calculate the time taken by it to reach a height h.
Solution :
Let be the velocity of the body at a distance r from the centre of earth. Applying conservation
1 GMm 1 GMm
of mechanical energy, we get m2 m2e ...(i)
2 r 2 R
GM
Here, e 2 gR and g
R2
R 2g
Substituting in Eq. (i), we get
r
r R+h
dr R 2 g 1
i.e.
dt
r
or 0
dt
R 2g R
r dr
3/
2
or t
2 1 3 3
[(R h) / 2 R / 2 ] or t
1 2R 1 h 1 Ans.
3 R 2g 3 g R
GRAVITATION
28 QUIZRR
Example 3
The ratio of acceleration due to gravity at a depth h below the surface of earth and at a
height h above the surface for h << radius of earth ?
Solution :
Let R be the radius of earth and g the acceleration due to gravity on earthÊs surface. Then the
desired ratio (say x) is :
h
g 1 2
R h h
x = 2
1 1
h R R
g / 1
R
h 2h
= 1 1
R
(h << R)
R
h
1
R
Example 4
For a given density of planet the orbital period of a satellite near the surface of planet of
radius R is proportional to ?
Solution :
Applying NewtonÊs second law to the circular orbit, we have
m2 4 2 rm GMm
r T2 r2
where m is the mass of satellite, its orbital and T is period.
3/
2r 2
T =
GM
For r R
4
and M = R 3 ( = density of planet)
3
3
We see that T
G
i.e., T is independent of R.
GRAVITATION
QUIZRR 29
Example 5
Two spherical bodies the mass M and 5 M and radii R and 2R respectively are released in
free space with initial separation between their centres equal to 12 R. If they attract each
other due to gravitational force only, then the distance covered by the smaller body just
before collision is ?
Solution :
Let at O there will be a collision. If smaller sphere moves x distance to reach at O, then bigger
sphere will move a distance of (9R x).
O
R x (9R-x) 2R
12R
M
5M
GM 5M
F =
(12R x)2
F G 5M
asmall = M
(12R x)2
F GM
abig = 5M
(12 x)2
1 1 G 5M 2
x asmall t2 t ...(i)
2 2 (12R x)2
1 1 1 GM
(9R x) abig t2 x t2 ...(ii)
2 2 2 (12R x)2
x
5
9R x
x = 45R 5x
6x = 45 R
x = 7.5 R
GRAVITATION
30 QUIZRR
Example 6
In a double star, two stars of masses m 1 and m 2 distant r apart revolve about their common
centre of mass under their mutual gravitational attraction. Find the expression for the
period T in term of m 1, m 2 and r. Find the ratio of their angular momenta about centre of
mass and also the ratio of their kinetic energies.
Solution :
As stated in theory also, both the stars will revolve round their centre of mass with same angular
velocity but different linear speeds. Let r1 and r2 be the distances of stars from their centre of
mass. Then,
r = r1 + r2 ...(i)
m1r1 = m2r2 ..(ii)
r1 r2
m1 m2
Gm1 m2
m1r2 = m2r22 = ...(iii)
r2
G m1 m2 m2 m1
, r1 r and r2 r
r3 m1 m2 m1 m2
2 r3
T = 2 Ans.
G(m1 m2 )
L1 I1 I1 m1r12
Ratio of angular momenta is, = I I
L2 2 2 m2 r22
L1 m2
L2
= m1
Ans.
1
I12
K1 2 I m
1 2
Ratio of kinetic energies is, = 1 I m1 Ans.
K2 I2 2 2
2
GRAVITATION
QUIZRR 31
Example 7
Calculate the self energy of the sun, taking its mass to be equal to 2 1030 kg and its radius
to be very nearly 7 108 metre. If its radius contracts by 1 km per year, without affecting
its mass, calculate the rate at which it radiates out energy.
Solution :
The gravitational self energy U of a uniform solid sphere is equal to the amount of work done
in assembling together its infinitesimal particles initially lying infinite distance apart. Let us
assume the sphere to be formed by continuous deposition of mass particles (brought from infinity)
in the form of successive spherical shells around an inner spherical core of radius r until it
becomes a full fledged solid sphere of radius R.
M
Let = density of material of the sphere = ...(i) dr
4 3
R
3
R r
4 3
Mass of inner spherical core = r
3
4 3
G 3 r
dW (4 r 2 dr )
or r
Substituting the value of from Eq. (i) and integrating it from O to R, we get
3 GM 2
W =
5 R
GRAVITATION
32 QUIZRR
dU d U dR 3 M2 G d R
i.e., = . =
dt dR dt 5 R 2 dt
dR 1000
Here, = m/s
dt 365 24 3600
Example 8
An artificial satellite of mass m moves in an orbit whose radius is n times the radius of
earth. Assuming resistance to the motion to be proportional to the square of velocity, i.e.,
F = a v2, find how long the satellite will take to fall onto the earth. Mass of the earth is M
and radius is R.
Solution :
Orbital speed at distance r from the centre of earth is given by,
GM
=
r
1 GMm
K.E. = m2
2 2r
GMm
P.E. =
r
Total mechanical energy E = K.E. + P.E.
GMm
or E =
2r
dE
Now, = F.
dt
GMm dr
or = a3
2r 2 dt
GMm dr GM GM GM
2 dt =
a as
2r r r r
a t rf
1 1
m 0
dt =
2 GM
ri
r dr
a 1 1 R
m
t =
2 GM nR
r dr
m R
or t = ( n 1) Ans.
a GM
GRAVITATION
ROTATIONAL
MECHANICS
QUIZRR 3
ROTATIONAL MECHANICS
Linear Motion Rotational Motion
Position x Angular position
Velocity v Angular velocity
Acceleration a Angular acceleration
Motion equations x = v t t
= Motion equations
v = v0 + at = 0 + t
1 2 1 2
x = v0 t at 0 t t
2 2
1 1 2
Kinetic energy mv2 I Kinetic energy
2 2
Power Fv Power
1. INTRODUCTION
Consider a disk rotating on a stationary rod.
S
r
ROTATIONAL MECHANICS
4 QUIZRR
we see that when the disk rotates so that the arc length (s) equals the length of the radius of
) will equal 1 radian. The resulting equation is
the disk (r), the subtended central angle (
s = r
or
= s/r
where the unit of a radian represents the dimensionless measure of the ratio of the circleÊs arc
length to its radius.
If the disk rotates through one complete revolution, then s equals the entire circumference and
equals 2 radians.
s = r
2r = r
= 2 radians
Differentiating the basic equation s = r results in the next two relationships between the tangential
motion of a point along the circumference and the angular behavior of the disk itself.
s = r
ds/dt = r(d/dt)
v = r
The Greek letter represents the angular velocity of the disk. It is measured in the unit
radians/sec. Differentiating one more time, we have
v = r
dv/dt = r(d/dt)
a = r
The Greek letter represents the angular acceleration of the disk. It is measured in the unit
radian/sec2.
Summary
s = r
v = r
a = r
allow us to relate the linear motion (s, v, a) of a point moving in circular motion on a rotating
platform with the the rotational motion (, , ) of the platform itself.
ROTATIONAL MECHANICS
QUIZRR 5
It is important when using these equations that the units on s, v, a and r be consistent. That
is, if the radius is measured in meters, then s must also be in meters, v in m/sec, and a in m/sec2.
Using our new variables for angular motion, we can now state analogous equation to those we
have already used for linear motion. When the velocity is constant our equations are
s = vt which becomes = t
The following chart shows the relationship between the equations for uniform linear acceleration
and those that deal with uniform angular acceleration. When working with pure rotational
motion, the standard unit of measurement is the radian.
linear angular
a = (vf - v0)/t a = (f - 0)/t
vf = v0 + at f = 0 + t
at2 t2
s = v0t + = 0t +
2 2
2 2 2 2
vf = v0 + 2as f = 0 + 2
d
d
dt
Angular velocity describes the speed of rotation and the orientation of the instantaneous axis
about which the rotation occurs. The direction of the angular velocity vector will be along the axis
of rotation; in this case (counter-clockwise rotation) the vector points towards the viewer.
It is defined as „the rate of change of angular displacement with respect to time‰.
In physics, the angular velocity is a vector quantity (more precisely, a pseudo vector) which
specifies the angular speed, and axis about which an object is rotating. The SI unit of angular
velocity is radians per second, although it may be measured in other units such as degrees per
ROTATIONAL MECHANICS
6 QUIZRR
second, revolutions per second, degrees per hour, etc. When measured in cycles or rotations per
unit time (e.g. revolutions per minute), it is often called the rotational velocity. Angular velocity
is usually represented by the symbol omega (). The direction of the angular velocity vector is
perpendicular to the plane of rotation, in a direction which is usually specified by the Right hand
grip rule.
Two dimensions V
V
V
P
r
x
O
The angular velocity of the particle at P with respect to the origin O is determined by the
perpendicular component of the velocity vector v.
The angular velocity of a particle in a 2-dimensional plane is the easiest to understand. As shown
in the figure on the right (typically expressing the angular measures and in radians), if we
draw a line from the origin (O) to the particle (P), then the velocity vector (v) of the particle will
have a component along the radius (radial component, v||) and a component perpendicular to the
radius (tangential component, v).
A radial motion produces no rotation of the particle (relative to the origin), so for purpose of
finding the angular velocity the parallel (radial) component can be ignored. Therefore, the rotation
is completely produced by the tangential motion (like that of a particle moving along a
circumference), and the angular velocity is completely determined by the perpendicular (tangential)
component.
It can be see that the rate of change of the angular position of the particle is related to the
tangential velocity by :
d
v r
dt
Utilizing , the angle between vectors v// and v, or equivalently as the angle between vectors r
and v, gives :
v v sin .
ROTATIONAL MECHANICS
QUIZRR 7
Combining the above two equations and defining the angular velocity as = d/dt yields :
v sin
r
In vector form the linear velocity, angular velocity and radius vector are related by
v r
Thus, angular velocity can be represented as a vector quantity whose direction is perpendicular
to the plane of motion.
Points to Remember
1. If a particle is moving in a circle it is in pure rotational motion about the centre of the circle, while
for a moment it may be in pure translational motion about some other point.
2. If a particle P is moving in a circle, its angular velocity about centre of the circle (c) is two times
the angular velocity about any point on the circumference of the circle (0)
or wc = 20
This is because P´CP = 2P´ (by property of a circle) O P´
P´CP P´OP
c , 0 P
tpp´ tpp´ C
d
4. Angular velocity of a rigid body () is . Here is the angle B
dt
between the line joining any two points (say A and B) on the A
rigid body and any reference line (dotted) as shown in figure.
For example OP is a rod of length 5 m. End O is resting against
a vertical wall OY and P is moving towards right with constant
ROTATIONAL MECHANICS
8 QUIZRR
speed VP = 10 m/s. To find the angular speed of rod at = 30 , we can proceed as under.
AP = x = OP cos
x = 5 cos
dx d Y
or 5 sin
dt dt
O
d dx / dt dx n
dt 5 sin dt VP 10 m/s
VP = 10 m/s
X
10 A P
or 4 rad/s
5 sin 30
d
Here negative sign implies that decreases as t increases 0
dt
Example 1
A
r
A particle A moves along a circle of radius R = 10 cm so that its
radius vector r relative to O rotates with constant angular O
velocity = 0.2 rad/s. Find the modulus of the velocity of the
C
particle and modulus and direction of its total acceleration.
Solution :
Given that 0 = 0.2 rad/s, R = 10 cm = 0.1 m
c = 20 = 0.4 rad/s
(i) Modulus of velocity v Rwc (0.1) (0.4)
v 0.04 m/s or 4 cm/s Ans.
(ii) Modulus of total acceleration a R2c
a 0.1 0.4 0.016 m/s2
2
or
or a 1.6 cm/s2 Ans.
(iii) The direction of its total acceleration (centripetal acceleration) will be towards centre C.
ROTATIONAL MECHANICS
QUIZRR 9
Angular Momentum
A mass moving in a straight line has linear momentum P . When a mass rotates about some
point/axis, there is momentum associated with rotational motion called the angular momentum
L . Just as net external force is required to change the linear momentum of an object a net
instant of time.
The magnitude of is L is L = mvr sin = mvr
O
Here, r = r sin is the perpendicular distance of line of action of velocity v from point O. The
direction of L is same as that of r v
Note : The angular momentum of a particle about a line (say AB) is the component along AB
of the angular momentum of the particle about any point (say O) on the line AB. This component
is independent of the choice of point O, so far as it is chosen on the line AB.
Example 2 y
P
A particle of mass m is projected from origin O with speed
v
u at an angle with positive x-axis. Positive y-axis is in
vertically upward direction. Find the angular momentum r r = b
of particle at any time t about O before the particle strikes
the ground again.
x
Solution : O
L = m r v
1
Here r t = xi→ yj→ = (u cos ) ti→ ut sin gt2 →j
2
and v t vxi→ vy →j u cos i→ u sin gt →j
ROTATIONAL MECHANICS
10 QUIZRR
i→ →j k→
1 2
u cos u sin t gt 0
L = m r v = m 2
u cos u sin 0
2
1
2 →
= m (u sin cos )t u cos gt u sin cos t u cos gt k
2 2
2
1
= m u cos gt2 k→ Ans.
2
Example 3
A particle 1 moves in a circle of radius R and a particle 2 start moving along y-axis from
origin as shown. Calculate the angular momentum of particle 1 with respect to 2 at a
certain time instant t. [The speed of both particle is constant = u, v]
Solution :
Y
r2 (position vector of particle 2 after time t)
= (u t) →j
1
co-ordinate = (0, ut)
u
r1 (R cos t, R sin t)
X
2 O
after time t
v2 u →j, v1 v sin i→ v cos j→
r12 R cos t 0 i→ R sin t ut →j
v1,2 v sin t 0 i→ v cos t u →j
L1,2 v1,2 m1v1,2
Rigid body : A rigid body is a special case of system of particles where in the distance between
the constituent particles remain fixed.
ROTATIONAL MECHANICS
QUIZRR 11
Where ri and vi are position vector and velocity vector o the z
particle with respect to origin and ri´ and vi´ are of the particle
with respect to the centre of mass of body
Let us calculate the angular momentum of the given
rigid body (system of particle) about a point O.
n
L0 ri mi vi
i1
´
´
= ri mi vi rCM mi vCM
= ri´ mi vi´ rCM MvCM
L0 = LCM LCM , 0
angular momentum of
angular momentum system of particle with
angular momentum o
of a system with =
respect to CM along an + the CM along that
respect to an axis
axis
arc parallel to given axis
and passing through CM
ROTATIONAL MECHANICS
12 QUIZRR
Example 4
Here, L cm = I (perpendicular to paper inwards)
ro
1 2 v
= mR
2 R
90
vo
1
= mvR
2
O
and m r0 v0 mRv (perpendicular to paper inwards)
Since, both the terms of right hand side of Eq. (i) are in the same direction,
1
L = mvR + mvR
2
3
or L = mvR Ans.
2
TORQUE
Torque is the tendency of a force to rotate an object about an axis, or pivot. Just as a force is a
push or a pull, a torque can be thought of as a twist.
The magnitude of torque depends on three quantities : First, the force applied; second, the length
of the lever arm connecting the axis to the point of force application; and third, the angle between
the two. In symbols :
rF
= r F sin
where is the torque vector
r is the lever arm vector (vector form the axis to the point of force application), and r is the length
(or magnitude) of the lever arm vector.
ROTATIONAL MECHANICS
QUIZRR 13
F is the force vector, and F is the magnitude of the force,
is the angle between the force vector and the lever arm vector.
Result : Torque (or angular momentum) found about an axis by choosing a point on the axis
and taking its projection along the axis, is independent of the choice of the point on the axis.
(iii) When force and axis directions are skew lines : (non-coplanar and non intersecting)
( AB 0)
Tip : Torque about any axis can be visualized as a scalar triple product of r , F and n→ AB
AB r F . n→ AB r F n→ AB
dL
= r1 F1 r2 F2 r1 F1,2 r2 F2,1
dt
ROTATIONAL MECHANICS
14 QUIZRR
= net r1 F1,2 r2 F1,2
= net r1 r2 F1,2
= net r2,1 F1,2
..
= net 0 [ . r2,1 is anti parallel to F1,2 ]
dL x
dL
net dt ex x
dt
dP
Here F
dt
dr
and v (velocity of particle)
dt
ROTATIONAL MECHANICS
QUIZRR 15
dL
Hence, Eq. (ii) can be rewritten as, rF+vP
dt
Now, v P = 0, because v and P are parallel to each other and the cross product of two parallel
dL
vectors is zero. Thus, rF=
dt
dL
or ...(iii)
dt
Which states that the time rate of change of angular momentum of a particle about some reference
point in an inertial frame of reference is equal to the net torques acting on it. This result is
dP
rotational analog of the equation F , which states that the time rate of change of the linear
dt
momentum of a particle is equal to the force acting on it. Eq. (iii) like all vector equations, is
equivalent to three scalar equations, namely
dL dL dL
x , y and z
dt x dt y dt z
dL
The same equation can be generalised for a system of particles as, ext . According to which
dt
the time rate of change of the total angular momentum of a system of particles about some
reference point of an inertial frame of reference is equal to the sum of all external torques (of
course the vector sum) acting on the system about the same reference point.
dL
Now, suppose that ext 0, then 0 , so that L = constant.
dt
„When the resultant external torque acting on a system is zero, the total vector angular momentum
of the system remains constant. This is the principle of the conservation of angular momentum.
For a rigid body rotating about an axis (the z-axis, say) that is fixed in an internal reference
frame, we have
Lz = I
It is possible for the moment of inertia I of a rotating body to change by rearrangement of its
parts. If no net external torque acts, then Lz must remain constant and if I does change, there
must be a compensating change in . The principle of conservation of angular momentum in this
case is expressed as
I = constant ...(iv)
ROTATIONAL MECHANICS
16 QUIZRR
Points to remember
dL
1. In law of motion for a single particle acted on by a torque =
dt
Holds only if and L are measured with respect to any point O fixed in an inertial frame.
2. The law of motion for a system of particles acted on by a resultant external torque
dL
ext
dt
holds only if ext and L are measured with respect to
(a) any point O fixed in an inertial frame (b) the centre of mass of the system
3. Suppose a rod is lying on a smooth horizontal surface when a particle strikes it at some
point. Then angular momentum of rod + particle remains conserved about any point
fixed to an inertial frame of reference before and after collision as ext = 0, about any
point.
4. Suppose a rod is suspended from a support at O and a particle strikes the rod at O
some point, then the angular momentum of rod + particle remains conserved only
about point of suspension or point O. Because in this case ext on the system is zero
only about O.
Example 5
A uniform rod of mass m is rotating with constant angular velocity , about an axis making
an angle with the vertical.
(a) Find the value of
(b) Also, find the point of application of centrifugal force.
Solution :
(a) We will consider the analysis from the reference frame of rod.
Consider an element dx at a distance x along the length of the rod as shown.
The net centrifugal force acts along the direction as shown.
Calculating the torque about the point O,
d c dm 2 x sin x cos
c 2 x2 sin cos dm
0
ROTATIONAL MECHANICS
QUIZRR 17
L
c 2 x2 sin cos dx { dm dx} x dx
O
0
m2(x sin)
ml 2
= sin cos 2 .
3 (mg)
This torque is balanced by the torque due to weight.
l .
mg sin
2
c w
m l2 mg
sin cos 2 l sin
3 2
3g 3g
cos cos 1
2l 2 2l 2
Important Result
dm
2
Net centrifugal force = x sin
0
L
M 2 ML 2
= L sin xdx 2 sin
0
Net torque due to centrifugal force about axis passing through O
d
ML2
= sin cos 2 P
3
ROTATIONAL MECHANICS
18 QUIZRR
ML 2 mL2
d sin cos sin cos 2
2 3
2l
d
3
2l
the net centrifugal force acts at a distance from O.
3
Example 6
A hemispherical bowl of radius R is kept fixed with its axis vertical. Now a particle is
projected horizontally with an initial velocity v0 along the surface of the bowl, from a point
whose radius vector makes an angle 0 with the axis of bowl. If the particle is just able to
reach the rim, then find v0.
Solution :
Due to initial velocity, the ball starts moving, but due to vertical component of force, it starts to
rise in spiral path along the bowl.
Angular momentum remains conserved about O.
Li L f (about point O)
vo
v = v0sin ...(1) [At rim, vertical component of
velocity becomes zero]
Energy remains conserved (in ground frame) mg
1 1
mv02 mv2 mgR cos 0
2 2
2 gR
v0
cos 0
ROTATIONAL MECHANICS
QUIZRR 19
Example 7
A small disc of mass m is attached to one end of an ideal spring initially in natural length,
whose other end is fixed to a point O and placed on a smooth horizontal surface. Find the
maximum elongation of the spring.
Solution :
Apply angular momentum conservation vo
K, l
about O as net external torque = 0.
m
O
(Li) = (Lf)
mv0l0 = mv (l0 + lmax) vo
v0 l0
v ...(1) )
0 lmax
l ax
lm
(l +
Use energy conservation
K
1 2 1 1
k lmax mv02 mv2 ...(2) O
2 2 2
2 mv02 l02
Klmax mv02
l0 lmax
2
Solve it yourself.
Example 8
Two identical small discs each of mass m are connected by an ideal spring and placed on
a smooth horizontal surface. An initial velocity is imparted to one of the discs as shown.
Find the maximum elongation in the spring.
vo
K,l
Solution :
Solving this question in CM frame of reference
CM moves with constant velocity
Angular momentum also remains conserved.
(Li)CM = (Lf)CM
ROTATIONAL MECHANICS
20 QUIZRR
v2
v l v l l lmax =0
m v0 0 0 m 0 0 mv2 2 v1
2 2 2 2 2
)
lm ax
v0 l0 v (l + {Here we have
2 l0 lmax ...(1) assumed velocities
2 are relative to COM}
mv02 1
mv22 klmax
2
...(2)
4 2
Solving (1) and (2) [use binomial and condition that l << l0]
mv0
lmax
kl0
Example 9
A small disc of mass m is attached to one end of an ideal string
r = r0
whose other end is passing through a hole in a horizontal table O 0
at O. And the other end is pulled down with constant velocity. m
Now if initial angular velocity 0, then find the tension in the
string as a function of distance from O.
Solution :
Net angular momentum of the system remains
conserved about O. (Li) = (Lf)
m(0 r0) r0 = m(r)r O T
r2
0 20
r
T = (m2r) T
02 r04 2 r 4
= m 4 r m 0 30
r r
(mo2ro)
2 r 4
T m 030 ro r
r
(variation of tension with r)
ROTATIONAL MECHANICS
QUIZRR 21
5. MOMENT OF INERTIA
Moment of inertia is the name given to rotational inertia, the rotational analog of mass for linear
motion. It appears in the relationships for the dynamics of rotational motion. The moment of
inertia must be specified with respect to a chosen axis of rotation. For a point mass the moment
of inertia is just the mass times the square of perpendicular distance to the rotation axis, I = mr2.
That point mass relationship becomes the basis for all other moments of inertia since any object
can be built up from a collection of point masses.
1
Linear F = ma Linear KE = mv2
2
NewtonÊs second Kinetic Energy
Moment of 1
Law Angular KE = I2
Inertia 2
Angular = I
I
1
Linear p = mv Linear Fnetd = ( mv2)
2
Momentum Work Energy
1 2
Angular L = I Angular net = ( I )
2
r
I = mr2 m
Now, if a system of particles is made of number of particles of masses m1, m2, m3 ...... mn at
distances r1, r2, r3 ...... rn from the axis of rotation, its moment of inertia is defined as
n
m1
= mi r12
i 1 r1
m2
r2 m3
r3
m5 r5
r4
m4
ROTATIONAL MECHANICS
22 QUIZRR
M oment of Inertia of Continuous Body
For calculating moment of inertia of a continuous body, we first divide the
body into suitably chosen infinitesimal elements. The first choice depends
on t he symmet r y of body. Consider an element of t he body at dist ance r r
dm
from the axis of rotation. The moment of inertia of this element about the
axis we define as (dm)r2 and the discrete sum over particles becomes integral
over the body :
I = (dm)r2
Ix mi y12
z
Iy mi x12 y
Iz mi ri2
P(x,y)
= mi x12 y12 ri
x
O ^ ^
r = xi + yj
= mi x12 mi y12 = (Iy + Ix)
Iz = I x + Iy
I CM mi x12
d mi
I AB mi ( xi d ) 2
CM
xi
= mi xi2 d 2
mi 2 d mi xi
IAB = (ICM + Md 2) B
To find the moment of inertia about a given axis pass an axis through CM parallel to the given
axis and note the perpendicular distance d from this axis.
ROTATIONAL MECHANICS
QUIZRR 23
B
L/2
M x 3 ML2
I sin 2 sin2
L 3 12
L/2
2
ml 2 l l
I sin 2 m sin
12 2 2
ml 2 l sin
I sin 2 2
3 B´ B
(B) MOI of a non-uniform rod with linear density
x
0 1
L
L
2 x
I dI 0 1 x2 sin2 dx
L L
2
2
M 11L3 11 ML
= sin2 sin 2
L 96 96
ROTATIONAL MECHANICS
24 QUIZRR
(2) Uniform Rectangular Lamina
(A) About an axis parallel to any side and passing through centre.
(About X-axis)
M
dm l dy , r y Y
lb
L
d I dm r
2
b (dy)
y
X
b/2 b/2 O
M 2 M y
3 Mb2
I lb dy y b 3
12
b/2 b/ 2
Mb2 ML2
Ix
12 , similarly, y 12
I
Mb2 ML2
Iz (perpendicular axis theorem)
12 12
Iz
M 2
12
L b2
Ma 2
Iz
M 2
12
a a2
6
I = R2 dm dm
I = MR2 R
ROTATIONAL MECHANICS
QUIZRR 25
Z
(iii) MOI about a diametrical axis. Y
m
I x + Iy = I z
2Ix = Iz R
X
I z MR 2
Ix
2 2 Y
A´
(iv) MOI about a tangential axis parallel to diameter A
MR 2 3
I1 MR 2 MR 2
2 2
R
B´
B
M
dm = (2r)dr where
R2
d I dmr
2
ROTATIONAL MECHANICS
26 QUIZRR
R
M
= 2 r
3
dr
R 2 dr
0
2M R4 1 r
I MR 2 R
R2 4 2
1
I MR 2
2
1
I MR 2
4
A´ A
(iii) About a tangential axis :
(a) Perpendicular to diameter
1 3 R O
I MR 2 MR 2 MR 2
2 2
(b) Parallel to diameter
1 5 B´ B
I MR 2 MR 2 MR 2
4 4 A
ROTATIONAL MECHANICS
QUIZRR 27
I dI 0 dm R sin , where is the mass density.
2 2
M
4r 2 Rd
d Rsin
I
M
4 R 2
2R sin d
4
0
3
MR 2 4 2 2
= 2 3 3 MR
2
= MR 2 MR 2
3
5 MR 2
I
3
2
{Please note that here we put dm r 2 as this is the moment of inertia of hollow sphere}
3
2 4 R 4 2M R5 2
I
3 4
0
r dr =
R3
5
MR 2
5
R 2
3
ROTATIONAL MECHANICS
28 QUIZRR
(8) MOI of hollow and solid cones : O
(A) Hollow cone y
Consider a ring with height dy at a distance y from O.
H dy
Now thickness of ring will be dy sec x
M
also,
Rl
Hence, dm = (2x) dy sec R
dI = (dm)x2
H
I 0 2x dy sec x2
M
H dysec
I 2 sec 0 x3 dy dy
Rl
y H
Now, we can see from the figure that
x R
yR
x
H
2M H R3 1
I
Rl cos 0 y3
H 3
dy
2
MR 2
1
I MR 2
2
2 H dy
dm = x dy x
M
Where Here also
1 2
R H R
3
H1 x R
I = 0 2
x4 dy
y
=
H
H
H y4 R 4
dy R
H4 x y
2 R 2H 0 H
3
3
I MR 2
10
ROTATIONAL MECHANICS
QUIZRR 29
Points to Remember
1. Theorem of parallel axis is applicable for any type of rigid body whether it is a two dimensional
or three dimensional, while the theorem of perpendicular axes is applicable for laminar type or
two dimensional bodies only.
2. In theorem of perpendicular axes, the point of intersection of the three axes (x, y and z) may be
any point on the plane body (it may even lie outside the body). This point may or may not be
the centre of mass of the body.
3. Moment of inertia of a part of a rigid body (symmetrically cut from the whole mass) is the same
as that of the whole body. e.g. in figure (a) moment of inertia of the section shown (a part of a
1 2
circular disc) about an axis perpendicular to its plane and passing through point O is MR as
2
1 2
the moment of inertia of the complete disc is also MR . This can be shown as in Fig.
2
1
Suppose the given section is th part of the disc. then mass of the disc will be nM.
n
1
Idisc = nM R 2
2
R
1 1 O M
Isection = Idisc MR 2
n 2
(a) (b)
Example 10 A
Three rods each of mass m and length l are joined together to form
an equilateral triangle as shown in figure. Find the moment of inertia
of the system about an axis passing through its centre of mass and CM
perpendicular to the plane of the triangle.
Solution : B C
Moment of inertia of rod BC about an axis perpendicular to plane of triangle ABC and passing
ml 2
through the mid point of rod BC (i.e., D) is I1
12 A
From theorem of parallel axes, moment of inertia of
this rod about the asked axis is
I2 = I1 + mr2 CM
2 r
ml 2 l ml 2 30
= m B C
12 2 3 6 D
ROTATIONAL MECHANICS
30 QUIZRR
ml 2
Moment of inertia of all the three rods is I 3I2 3
6
ml 2
= Ans.
2
Example 12
Consider a uniform rod of mass m and length 2l with two particles of mass m each at its
ends. Let AB be a line perpendicular to the length of the rod and passing through its
centre. Find the moment of inertia of the system about AB.
Solution :
IAB = Irod + Iboth particles A
m 2l
2
=
12
2 ml 2
m
l l
m
7 2 B
= ml Ans.
3
6. RADIUS OF GYRATION
It is the radius of a uniform circular ring, having same mass as that of body, and with the given
axis having as its own axis, such that moment of inertia of ring becomes equal to that of the body.
I = MK2
I
K
M
Example 13
m l2
(a) Moment of inertia for rods PQ and PR, I sin 2 60 and for rod QR
3
ROTATIONAL MECHANICS
QUIZRR 31
m l2 3 3 60
= 2 m l2
3 4 4 Q m R
5 m l2
=
4
5
Now, I = (3M)K2 Kl
12
ml 2
I
3
A
and I for rod QP
P
I = ICM + Md 2
2
1 3
ml 2 M l
2
=
12 B
ml 2 3ml 2
= Q
12 4 R
8 m l 2 m l 2 9 ml 2 18 m l 2 3
I net m l2
12 12 2
I l
K
3M 2
Example 14
Radius of gyration of uniform disc of a radius R, about an axis perpendicular to the plane
of disc = R. Find the distance of axis from centre.
Solution :
MK2 = ICM + Md 2
1
MK2 = M R 2 Md 2
2
R
d
2
ROTATIONAL MECHANICS
32 QUIZRR
Example 15
IP = ICM + M (R x)2 M
I0 = ICM + Mx2 (CM)
x
IP I0 = M (R x)2 Mx2
O
R
(i) Now, we know that x = /2 for a hemispherical shell
2 2
I0 = MR
3
2 MR 2 MR 2
I P MR 2
3 4 4
2
IP MR 2
3
3R 2
x , I0 MR 2
8 5
2 2 25 R 2 9R 2
IP = 5 MR M M
64
64
2 MR 2 13MR 2
= MR 2
5 4 20
Angular acceleration
measures the time rate of change of the angular velocity. Hence, the
The angular acceleration
magnitude of this vector may be written as,
d
...(ii)
dt
ROTATIONAL MECHANICS
QUIZRR 33
d 2
It is also possible to express as,
dt2
is the same as that for , however its sense of direction depends on
The line of action of
whether is increasing or decreasing with time. In particular if is decreasing, is called an
The angular acceleration of a rigid body is directly proportional to the sum of the torque components
along the axis of rotation. The proportionality constant is the inverse of the moment of inertia
about that axis, or,
I
Thus, for a rigid body we have the rotational analog of NewtonÊs second law :
= I ...(iii)
Following two points are important regarding the above equation.
(i) The above equation is valid for rigid bodies. If the body is not rigid like a rotating tank of
water, the angular acceleration is different for different particles.
(ii) The sum in the above equation includes only the torques of the external forces, because
all the internal torques add to zero.
dv d d dr
Taking the time derivative, a rc r
dt dt dt dt
d dr P
Here, and v r
dt dt
Hence a r r ...(vi)
ROTATIONAL MECHANICS
34 QUIZRR
By definition of the cross product. The first term on the
right has a magnitude at = r, called the tangential
component of acceleration. Likewise the second term has ,
a magnit ude an = 2 r sin = 2r and called the normal
component of acceleration. O an a
r at
P O a
The tangential component of acceleration represents the
Q rp an
time rate of change in the velocityÊs magnitude and can
dv at
also be written as . Thus,
dt P
dv
at ...(vii)
dt
If the speed of P is increasing, then at acts in the same direction as v , if the speed is decreasing,
at acts in the opposite direction of v finally if the speed is constant at is zero.
The direction of an is always towards O, the centre of the circular path. Hence, Eq. (vi) can be
identified by its two components as a at an r 2 r
Since, at and an are perpendicular, if needed the magnitude of acceleration can be determined
Example 16
The angular position of a point on the rim of a rotating wheel is given by = 4t ă 3t2 + t3,
where is in radians and t is in seconds. What are the angular velocities at
(a) t = 2.0 s and
(b) t = 4.0 s
(c) What is the average angular acceleration for the time interval that begins at t = 2.0
s and ends at t = 4.0 s.
(d) What are the instantaneous angular acceleration at the beginning and the end of this
time interval.
Solution :
Angular velocity
d
dt dt
d
4t 3t2 t3
or = 4 6t + 3t2
(a) At t = 2.0 s, = 4 6 2 + 3(2)2
or = 4 rad/s Ans.
ROTATIONAL MECHANICS
QUIZRR 35
f i 28 4
av
tf ti 42
or av = 12 rad/s2
(d) Instantaneous angular acceleration is,
d d
dt dt
4 6t 3t2
or = 6 + 6t
At t = 2.0 s, = 6+ 6 2 = 6 rad/s2 Ans
At t = 4.0 s = 6 + 6 4 = 18 rad/s2 Ans.
Example 17
A solid fly wheel of 20 kg mass and 120 mm radius revolves at 600 rpm. With what force
must a brake lining be pressed against it for the flywheel to stop in 3 s, if the coefficient
of friction is 0.1.
Solution :
n 0 = 600 rpm (revolutions per minute)
600
= rps
60
20
or = rad/s2
3
Further =
I
Here, = øNR (R = radius)
or = øFR (F = applied force)
ROTATIONAL MECHANICS
36 QUIZRR
1
as N = F and I mR 2
2
10 22 20 0.12
Substituting the values we have, F = 3 7 0.1
or F = 251.43 N Ans.
Example 18
A wheel rotates around a stationary axis so that the rotation angle varies with time as
= a t2, where a = 0.2 rad/s2. Find the magnitude of net acceleration of the point A at the
rim at the moment t = 2.5 s if the linear velocity of the point A at this moment is v = 0.65
m/s.
Solution :
Instantaneous angular velocity at time t is
=
d
dt
=
d
dt
at2
or = 2at = 0.4t (as a = 0.2 rad/s2)
Further, instantaneous angular acceleration is,
d d
= 0.4t
dt dt
or = 0.4 rad/s2
Angular velocity at t = 2.5 s is
= 0.4 2.5 = 1.0 rad/s
v 0.65
Further, radius of the wheel R= or R 0.65 m
1.0
Now, magnitude of total acceleration is :
a = an2 at2
a = 0.65 2 0.26 2
or a = 0.7 m/s2 Ans.
ROTATIONAL MECHANICS
QUIZRR 37
= m ri2 O ri
= mri2 I
L = I
Note : This is applicable about only that axis about which the body is under pure
rotating motion.
1
K.E.= mv2
2
1
2 m ri
2
K =
1 2 1
=
2
mri2 2 I 2
K 1 I2
2
Note : Applicable when the body is under pure rotatory motion in that frame about
axis of rotation.
About any general axis
1 2
K K´ M vCM
2
1 1
K ICM 2 MvCM
2
in general applicable of rigid body motion.
2 2
Now, since
L = I
dL d dI
I
dt dt dt
ROTATIONAL MECHANICS
38 QUIZRR
If I is constant, then
{Here, is the angular acceleration of the body about the same axis where I is calculated}
dL
However, ex is valid everywhere
dt
2
d = change in rotational KE of body
1
Example 19
A uniform rod of mass m and length l is placed on a smooth surface, and is free to rotate
about a vertical axis passing through fixed end A. A constant force F is always acting
horizontally applied to the end B, perpendicular to initial orientation of rod. Find the
angular velocity of rod as a function of angular displacement of rod.
Solution :
Torque at any angle due to F about A will be,
A = (F l cos )
Now, we can solve it by two methods.
(i) Use A = IA
d F
where can be written as
d
m l2 d
F l cos = 3 d
l
m l2
F l cos d d
0
3 0
m l2 2
F l sin =
6
ROTATIONAL MECHANICS
QUIZRR 39
6F sin
w
ml
(2) Use energy conservation,
Work done by torque = Change in rotational K.E. of the body
1
0 d 2 I A 2
1 ml 2 2
Fl 0 cos d =
2 3
ml 2
F sin =
6
6Fsin
ml
ROLLING MOTION
Let us first see an example of pure rotational motion of a body about a fixed axis (called spinning).
R
Disc
This is called pure rotational motion because the centre of mass of the disc is at rest and the body
is performing rotational motion about an axis.
Now, let us see an example of pure translational motion.
Ring
In this case, the ring is not rotating but the entire body is moving with a velocity v in the
horizontal direction. This is a case of pure translational motion.
ROTATIONAL MECHANICS
40 QUIZRR
Now,
v
Now part (C) shows a rolling motion which is a combination of translatory and rotatory motion.
In case of rolling motion if the object rolls across a surface in such a way that there is no relative
motion of object and surface at the point of contact, the motion is called Rolling without
slipping. So let us analyze the condition.
C V
R v
P
Now, for no slipping, if the ball is in contact with ground (or any object at rest) when the point
P on the ball should also be at rest {for no relative motion}
Hence, v r = 0
or v = r
Now, if a force and torque are also present on the body and the body continues in pure rolling,
then net acceleration of the lowest point should also be zero relative to the surface. Thus,
a
O
R a
a = R
where is the angular acceleration of the body about its centre O and a is the translational
acceleration of the centre of mass of the body.
ROTATIONAL MECHANICS
QUIZRR 41
Then
F m, R
h
O
F
a ...(1)
m
F.h
...(2)
ICM
F F.h
R
M ICM
ICM
h0
MR
In this case force exerted by friction (fs) will be zero. This is because fs starts acting only when
there is relative motion between two surfaces in contact.
Now, let us consider different cases and determine the direction of fs.
(i) If h > h 0
then F m, R
h
R > a O a
a
R
P fs
this means that net motion of body with respect to surface will be in the backward direction.
Hence fs will start acting in the forward direction to oppose this relative motion.
Hence, fs will be in the direction of F.
ROTATIONAL MECHANICS
42 QUIZRR
(ii) h < h0
F
In this case a > R. Thus the body will have a tendency to h
O
slip forward and hence, force of friction will act in backward a
direction.
fs
Example 20
A sphere is placed on a rough horizontal surfaces and an initial velocity v0 and angular
velocity 0 = v0/R is imparted to the sphere. (anticlockwise)
(a) Find the velocity of the sphere when it stops rotating ?
(b) Find the velocity of the sphere when it starts pure rolling ?
(c) Find the work done by friction after a long time interval (i.e. after pure rolling starts)
Solution :
Method I : Through conservation of angular momentum.
=o r = v2
o = vo
R
vo v1 v2
f f f
O
(rotation stops)
These are the 3 stages for the sphere. Initially, the sphere at point O had a net velocity
= v0 + 0R = 2v0
In second case, due to net torque opposite to the angular velocity, so eventually becomes zero.
And finally pure rolling starts when r = v2.
Now, Angular momentum remains conserved about the point O fixed to ground {as torque about
O is zero}
Hence, (Li)1 = (Lf)2
Now, (Li)1 = ICM ( 0) + R (Mv0)
2 2
= MR 0 Mv0 R
5
2
= MRv0 MRv0
5
3
= MRv0
5
ROTATIONAL MECHANICS
QUIZRR 43
3
MR v0 Mv1 R
5
3
v1 v0
5
3
(a) So, velocity of sphere when it stops rotating = v0
5
(b) Now again apply angular momentum conservation about point O in cases (2) and case (3)
hence, (Li)2 = (Lf)3
2
Mv1 R MR 2 M v2 R
5
3v0 2
v2 R
5 5
2 7
= v2 v2 v2
5 5
3
Hence, v2 v0
7
(c) Now, work done by friction will be equal to the change in kinetic energy.
{from conservation of energy}
Wfr = K
1 2
Now, K = K´ + MvCM
2
1 1
= ICM 2 MvCM
2
2 2
Wfr = K2 K1
1 2
2
2 v2 1 2
1 2
2
2 v0 1 2
= 2 5 MR 2 2 Mv2 2 5 MR 2 2 Mv0
R2 R
ROTATIONAL MECHANICS
44 QUIZRR
7 7
= Mv22 Mv02
10 10
7 9 4
= M 1 v02 M v02
10 49 7
Method 2 :
(a) velocity when = 0
o = vo
R
vo v1
f
f
Now, f = m a a
m
2 2
also, = f.r = I = MR {Torque equation}
5
5f
2 MR
Now, = 0 t= 0
0 v0 2MR 2v0 M
t
R 5f 5f
f 2v0 3v0
and v = v0 at = v0 m
m 5f 5
3v0
hence velocity (when = 0) =
5
=o
(b) = v
R
3vo
v
5
ROTATIONAL MECHANICS
QUIZRR 45
5f 2MR
= t= t t
2MR 5f
3v0 f
v = t
5 m
3v0 f 2m R
v =
5 m 5f
3v0 2v
=
5 5
3v0
v
7
3v0
7R
Example 21
A solid sphere is placed on a rough horizontal surface, when an impulse is applied to the
sphere at a height h above the centre, its CM acquires an initial velocity v0. It is found that
9v0
when sphere starts pure rolling over the surface, its velocity becomes . Then show
7
4R
that h . Also find the work done by friction in long time
5
Solution :
J
h 9vo
vo
R
o
O (pure rolling)
9v0
(a) Now, J.h = I 0 R =
7
9v0
and J = mv0
7R
2 2
mv0.h = mR 0
5
ROTATIONAL MECHANICS
46 QUIZRR
5v0 h
0
2R 2
9v0
I0 + Mv0R = I + M R
7
2 2 9v0 9Mv0 R
Mv0 h + Mv0R = MR
5 7R 7
18 45 28 R 4R
h RR
35 35 5
4R
h
5
1 2 1 2 1 2 1 2
= ICM MvCM ICM 0 Mv0
2 2 2 2
81
Wfr mv02
245
Example 22
F
ao
m2
Let a and a0 be the acceleration relative to ground of the sphere and plank respectively.
ROTATIONAL MECHANICS
QUIZRR 47
Tip : Whenever a body is rolling without slipping over another surface, assume the acceleration
of CM of the rolling body with respect to the surface on which pure rolling is taking place.
N
fs = m1 a ...(1)
2 2
fs R = M R ...(2)
5
F fs = m2 a0 ...(3)
condition for pure rolling
(a a0) = R ...(4)
2F 7F
a , a0
7m2 2m1 7m2 2m1
fs min
2F
ø>
N 7m2 2m1 g
Example 23
A uniform cylinder of mass m and radius R rolls down without slipping down a rough
inclined plane, which is placed on a smooth horizontal surface. Find the acceleration of the
cylinder relative to wedge.
m, R
Sol u t i on :
Let a0 be the acceleration of the wedge and a be the acceleration of the cylinder relative to the
wedge frame.
ROTATIONAL MECHANICS
48 QUIZRR
N ao
fs
mao
M
N fs
mg
Example 24
(a) A horizontal plank of mass m is placed on top of a horizontal cylinder of mass M and
radius R, which is placed on a rough horizontal surface. A constant force F is applied
at angle of cylinder with respect to ground on the plank. No slipping takes place
at any point of contact. Also given that the plank remains in horizontal position (a)
find the friction force f A and f B.
(b) Also, find the acceleration of plank and the cylinder.
F
Solution :
m
A
M
R
N fA
Fsin
B
F
Fcos
fA
mg fB
ROTATIONAL MECHANICS
QUIZRR 49
For cylinder :
fA + fB = Ma ...(1)
1
fA.R + fB.R = ICM ...(2) {Here ICM = MR 2 }
2
3M F cos M F cos
fA , fB 3M 8 m
3M 8 m
4F cos
a
3M 8m
Example 25
M F
Cylinder is placed between 2 horizontal plank as shown
constant forces F and 2F are applied on the plank as shown. M, R
Find the acceleration of cylinder and that of plank, if there 2F 2M ø=0
is no slipping at any contact point.
Solution :
f2
M F 2f 2M
f1
f2
FBD of upper plank FBD of cylinder FBD of lower plankr
ROTATIONAL MECHANICS
50 QUIZRR
For cylinder : f1 f2 = Ma ...(3)
(f2 + f1)R = ICM ...(4)
also for no slipping at point A
a + R = a1 ...(5)
and for point B
( R a) = a2 ...(6)
Solving (1), (2), (3), (4), (5) and (6) we get
F 19F 21F
a , a1 , a2
23M 23M 23M
Example 26
2F
3 F . Mg
3 Mg
F
2 - 3
3 Mg 2Fmax
amax
Fmax =
2 3 3m
ROTATIONAL MECHANICS
QUIZRR 51
Type II : Rolling without slipping question involving threads (strings) would over body.
Example 27
Solution :
Note the directions of rotation of body and direction is which the friction acts.
N T
a
fs
m
mg
Tip : In string problem of pure rolling, use the concept that the two ends of the thread
must have same acceleration, otherwise the string would slack or break.
Mg sin (T + fs) = Ma ...(1)
1 2
fs R TR = MR ...(2)
2
a = R [condition for pure rolling] ...(3)
2 g M sin m
a
3M 8m
ROTATIONAL MECHANICS
52 QUIZRR
9. INSTANTANEOUS AXIS OF ROTATION (IAR)
There is another term which will be covered in the same article called Instantaneous centre
of rotation (ICR).
IAR : It is the axis about which the motion of a rigid body undergoing plane motion is
assumed to be pure rotatory with same and as that of body about its COM axis.
{This is always perpendicular to the plane of motion of rigid body.}
ICR : It is the point of intersections of IAR with the plane of motion of rigid body.
ICR is the centre (of the circle) about which all particles of the rigid body are going
in circle with same and , but in different radius equal to the distance from ICR.
Note : ICR and IAR are instantaneously at rest, i.e. at an instant their velocity is zero, but their
acceleration may or may not be zero.
Consider the body shown in figure where the line of action of the velocities
vB A
vA and vB are known. Draw perpendicular at A and B to these lines of vA
action. The point of intersection of these perpendiculars as shown locates
the IC at the instant considered. lC
B
In pure rolling, IAR passes through the point of contact of the surface
and body. {This is because the point O is at rest as v = R }
v = 2R
C A
B V (v = R )
R 2 R R 2 Vnet
O
(ICR)
Now, ICR is used to know the direction and magnitude of the net velocity of any point of the body.
For example, if we have to calculate velocity of point A, then join point O and A and the direction
perpendicular to this line OA will give us the value of net velocity and the magnitude of velocity
will be given by vnet = (OA).
Now, OA R 2
vnet = R 2
ROTATIONAL MECHANICS
QUIZRR 53
Some points
When the velocities of points A and B are parallel and have known magnitudes vA and vB then
the location of the IC is determined by proportional triangles as shown in figure.
A lC
vA
d lC A vA
d
vB vB
B B
(a) (b)
vA
In both the case, r A, =
IC
vB
and rB,IC =
Example 28
A uniform thin rod of mass m and length l is standing on a smooth horizontal surface. A
slight disturbance causes the lower end to slip on the smooth surface and the rod starts
falling. Find velocity of centre of mass of the rod at the instant when it makes an angle
with horizontal.
Solution : A
As the floor is smooth, mechanical energy of the
com l
rod will remain conserved. Further, no horizontal sin)
IC h = 2 (1
force acts on the rod, hence the centre of mass
vC
moves vertical downwards in a straight line. Thus l sin
2
velocities of COM and the lower end B are in the
directions shown in figure. The location of IC at B vB
ROTATIONAL MECHANICS
54 QUIZRR
this instant can be found by drawing perpendiculars to vC and vB at respective points. Now, the
rod may be assumed to be in pure rotational motion about IAOR passing through IC with angular
speed .
Applying conservation of mechanical energy. Decrease in gravitational potential energy of the rod
= increase in rotational kinetic energy about IAOR.
1
mgh I 2
2 IAOR
l 1 ml 2 ml 2
or mg 1 sin cos2 2
2 2 12 4
l
vC cos
8
Angular Impulse
dL
A = dt = dt L
dt
A = L
Ax = ( Lx)
Ay = ( Ly)
Az = ( Lz) = ex z dt
Angular impulse is also defined a moment of linear impulse.
A= r j
Example 29
A uniform sphere of mass m and radius R starts rolling without slipping down an inclined
plane. Find the time dependence of the angular momentum of the sphere relative to the
point of contact at the initial moment. How will the results be affected in the case of a
perfectly smooth inclined plane ? The angle of inclination of the plane is .
Solution :
Applying the equation
Angular impulse = change in angular momentum about point of contact we have,
ROTATIONAL MECHANICS
QUIZRR 55
dt L
j
or L = (mg sin ) Rt Ans.
sin
There will be no change in the results, as the torque of force of mg
friction in the first case is zero about point of contact. So, it
hardly matters whether the surface is rough or smooth.
Example 30
N vo
v1 y
øN x
øN
v2
Let the horizontal velocities of the ball and the plank be v1 and v2 in opposite directions as shown
in Fig.
From conservation of linear momentum in horizontal direction
mv1 = Mv2 ...(i)
Linear impulse of the ball in vertical direction = change in linear momentum in vertical direction.
(J = N dt)
Hence J = 2mv0 ...(ii)
Linear impulse on the ball in horizontal direction = change in linear momentum in horizontal
direction.
øJ = mv1 ...(iii)
Angular impulse on the ball about COM = change in angular momentum about COM,
2
ø J.r = Iw0 = mr 2 .0 ...(iv)
5
ROTATIONAL MECHANICS
56 QUIZRR
Solving Eqs. (i), (ii), (iii) and (iv), we get
2 m2
v1 r 0 v2 r 0
M 5
and
5
Now, actual path of the ball is a projectile whose time of flight will be
2vy 2v0
T
g g
M m 2 M m
vr = v1 + v2 = v1 r0
M 5 M
4 M m v0 r0
s = vrT or s Ans.
5 M g
12. TOPPLING
N
Suppose a force F is applied at a height b above the base AE of the B D
F
block. Further, suppose the friction f is sufficient to prevent sliding. b
C
In this case, if the normal reaction N also passed through C, then
f
despite the fact that the block is in translational equilibrium (F = A E
f and N = mg), an unbalanced torque (due to the couple of forces
W = mg
F and f) is there. This torque has a tendency to topple the block
about point E. To cancel the effect of this unbalanced torque the normal reaction N is shifted towards
right a distance ÂaÊ such that, net anticlockwise torque is equal to the net clockwise torque or
F.b = (mg)a
Fb
or a
mg
N N
B D B D
F F
C a b C
f f
A E A mg E
mg
(a) (b)
Now, as F or b (or both) are increased, distance a also increases. But it can not go beyond the
right edge of the block. So in extreme case (beyond which the block will topple down), the normal
reaction passes through E as shown in Fig.(b).
ROTATIONAL MECHANICS
QUIZRR 57
Now, if F or b are further increased, the block will topple down. This is N
why the block having the broader base has less chances of toppling in F
comparison to a block of smaller base. Because the block of larger base
has more margin for the normal reaction to shift. On the similar ground C
we can see why the rolling is so easy.
Because in this case the normal reaction has zero margin to shift. So f
even if the body is in translational equilibrium (F = f, N = mg) an mg
unbalanced torque is left behind and the body starts rolling clockwise.
As soon as the body starts rolling the force of friction is so adjusted (both in magnitude and
direction) that either the pure rolling starts (if friction is sufficient enough) or the body starts
sliding. Let us take few example related to toppling.
Example 31
A uniform cube of side Âa Ê and mass m rests on a rough horizontal table. A horizontal force
F is applied normal to one of the faces at a point directly above the centre of the face, at
3a
a height above the base. What is the minimum value of F for which the cube begins to
4
tip about an edge ?
Solution :
In the limiting case normal reaction will pass through O. The cube will tip about O if torque of
F exceeds the torque of mg.
3a a N
Hence, F mg 2
4
a F
2 3a
2 4
or F mg
3
O
mg
2
Therefore, minimum value of F is mg Ans.
3
Example 32
A uniform cylinder of height h and radius r is placed with its circular face on a rough
inclined plane and the inclination of the plane to the horizontal is gradually increased. If
ø is the coefficient of friction, then under what conditions the cylinder will (a) slide before
toppling (b) topple before sliding.
Solution :
(a) The cylinder will slide if mg sin > ømg cos
or tan > ø ...(i)
ROTATIONAL MECHANICS
58 QUIZRR
N
h f
The cylinder will topple if (mg sin ) > (mg cos )r
2
2r sin cos
tan mg
or
h
...(ii) mg
2r
Thus, the condition of sliding is tan > ø and condition of toppling is tan .
h
Hence, the cylinder will slide before toppling if
2r
Ans.
h
2r
(b) The cylinder will topple before sliding if Ans.
h
Example 33
A spool of mass m and radius R, is placed on a rough horizontal surface. The thread is wind
up in a radius r over the spool. The other end is pulled with constant F at an angle with
horizontal. The spool starts pure rolling on horizontal surface. (I = yMR2) Find
(a) Find the acceleration of the spool
(b) Find the work done by applied force in 1st t second.
R F
Solution :
N
(a) a F cos fs = Ma ...(1)
fs (R) Fr = I ...(2)
F a = R ...(3)
I = yMR2 ...(4)
Solve then to get a
fs
mg
1 1
= ICM 2 MvCM
2
2 2
ROTATIONAL MECHANICS
QUIZRR 59
1 1 2
= yM(R22) + MvCM
2 2
1 1 1
= yMv2 Mv2 Mv2 y 1
2 2 2
W =
1
2
M a 2 t2 y 1
Example 34
A uniform rod AB of mass m and length l is placed on a smooth horizontal surface with end
A hinged. A small particle of mass m is rigidly attached to the rod at a distance l/3 from the
end A. Now, a horizontal impulse J0 is applied perpendicular to the length of rod at a
2l
distance from end A. Find
3
(a) the angular velocity of the rod just after the impulse applied
M
(b) impulse due to hinge reaction during impulse is applied. Take 3
m
Solution :
l
3 J
m m
A B A B
M,l
Jo
l
2l 3
3
Angular momentum cannot be conserved in this case. Hence we write angular impulse momentum
equation.
2l
About point A J 0 I
3
Ml2 ml 2
= 3 9
2l ml 2 10ml 2 M
J0 ml 2
3 9 9 3
m
3J
0
5ml
ROTATIONAL MECHANICS
60 QUIZRR
(b) Impulse due to hinge reaction = change in momentum of the system
l l
Jy + J0 = M m
2 3 We are writing for whole
(Jx = 0) system so the impulse
between m and M
3ml ml 11J0 becomes internal
Jy + J0 =
2 5 10
11J0 J
Jy J0 0
10 10
Example 35
A uniform rod of mass m and length l is suspended from end A. A small disc of mass m,
2l
moving with velocity u collides perpendicular to the rod at a distance from end A.
3
Find
(a) the minimum value of u, so that the rod makes full vertical circle.
(b) Also find the impulse due to hinge reaction on the rod.
Solution :
Linear momentum of (rod + disc) system cannot be conserved in horizontal/vertical direction.
We can conserve angular momentum about A for (rod + disc) system.
(Li)A = (Lf)A
A Jx
2l 2l
mu 0 I A mv
3 3
Jg
2l
3
2mul ml 2 2mvl
...(1)
3 3 3
ø v
2l
v
e
3 2l
u0 eu v ...(2)
3
1 ml 2
2
1
mgl I A 2 ...(3)
2 2 3
m l
J x mv mu 0
2
Jy = 0
Jnet = Jx
Example 36
In the arrangement shown, the rod hinged at point O is released from rest.
F i n d t h e h i n ge r eact i on w h en t h e r od m ak es an an gl e with the vertical.
(Mass, length = M, L)
O
Solution :
Concept :
All the forces acting on the body are supposed to be acting on CM.
Hence, find the acceleration of CM, and then use the force equation.
[About the hinged point O, the CM of the rod will be going in circle with
same and as as that of body about its CM.]
l l
aCM x cos 2 sin
2 2
l
2 l l 2
l
aCM(y) = cos sin 2
2 2
Fx
Now to find : Use energy conservation Fy
l 1 1 ml 2 2
mg 1 cos I0 2 ....(1)
2 2 2 3
l ml 2
mg sin ...(2)
2 3
ROTATIONAL MECHANICS
62 QUIZRR
l l
Now use force equation Fx ma x m cos 2 sin
2 2
2 l l
Fy + mg = may = m cos sin
2 2
Example 37
A uniform rod of mass m and length l starts sliding along two perpendicular smooth surfaces
as shown from vertical position.
Show that the rod will lose contact with vertical surface at cos
1
/
2
3 with vertical.
Solution :
(x, y) co-ordinates of CM
l l
x sin , y 2 cos
2
N1
dx l
Vx cos N2
dt 2
dvx l l
ax cos 2 sin O
dt 2 2
mg
l
vy sin
2
dvy l 2 l
ay sin cos
dt 2 2
l l
cos 2 sin 0 = 2 tan ...(1)
2 2
l 1 1
mg 1 cos ICM 2 MvCM
2
2 2 2
ROTATIONAL MECHANICS
QUIZRR 63
1 ml 2 2 1
mg
l
2
1 cos
2 12 2
2
2
m vx vy
l l2 l2
g 1 cos 2 2
2 24 4
6g
sin / 2 ...(2)
l
d 6 g 6 g cos / 2
sin / 2
d l l 2
3g
sin ...(3)
2l
Use (1), (2), (3)
3g 6g 2 sin
2l sin l sin 2 cos
3
cos 3 1 cos
2
3
cos 1
2
2
cos1
3
ROTATIONAL MECHANICS
64 QUIZRR
g sin
a
I ...(4)
1 CM2
MR
a mg sin
fs ICM 2
R MR 2
1
ICM
f s N tan
min
mR2
1
f
min s
ICM
N
fs = (mg sin )
Now, from equation (4); we can interpret acceleration for different bodies.
1
aring = ahollow = g sin
cylinder 2
2
adisc = asolid cylinder
= g sin
3
5 3
asolid = g sin , a = g sin
sphere 7 hollow sphere 5
Acceleration down the planle depends only the shape of the body and inclination and is
independent of size of body.
Note :
i.e. tan
(1) When the friction coefficient is insufficient MR 2 the slipping takes place and
1
ICM
pure rolling cannot take place.
Hence kinetic friction acceleration acts up the plane.
(2) In case of pure rolling on an inclined plane acceleration always acts down the plane.
(3) In case of pure rolling friction acts up the plane and is static
Every body once in pure rolling tries to sustain the pure rolling motion.
ROTATIONAL MECHANICS
QUIZRR 65
Example 38
A uniform circular disc of mass M and radius R, initially rotating with 0 as shown, is
placed on a rough incline plane. Given ø = 2 tan show that body will cover a maximum
02 R 2
distance of s
20 g sin
Solution : t
=
V
n
N ke
sin e ta
0 g
2m tim S1 =
V
= R
fk
Mg
(1) (2)
S2
S1
(3) Let time taken = t´
Let t be the time, after the disc is placed, when v = R. [while initially (v = 0, = 0)]
Since, slipping takes place, fk acts
f k = øN = 2mg sin
fk mg sin
a g sin (up the plane)
m
Torque equation f kR I CM
mR 2
(2mg sin )R = 2
4 g sin
R
ROTATIONAL MECHANICS
66 QUIZRR
At time t : 4 g sin
0 t 0 t
R
v = at = (g sin )t
and v = R
4 g sin
(g sin )t = 0 R t
R
0 R 0 R
t v ; 0
5g sin 5 5
1 2 1 20 R 2
s1 at g sin
2 2 25 g 2 sin 2
02 R 2
s1
50 g sin
After time t : The pure rolling motion is sustained by the static friction.
Under pure rolling,
2
a g sin (down the plane)
3
The cylinder stops when v = 0 and then turns back to roll down the plane
v2 20 R 2
s2
2a
25 2 2 g sin
3
320 R 2
S2 =
100 g sin
ROTATIONAL MECHANICS
QUIZRR 67
02 R2 3 02 R2
=
25 g sin 100 g sin
20 R 2
s
20 g sin
2
1 2 1 2 1 2 1 3 2 0 1 1
I
= P CM I MvCM mR mR 2 20
2 2 2 2 2 25 2 2
=
11
50
mR 2 20
Example 39 0
Concept :
The sphere will turn back only when v becomes zero before becomes zero. Otherwise
the sphere will never turn back since pure rolling starts hence after.
5 mg cos 5g cos
2mR 2R
ROTATIONAL MECHANICS
68 QUIZRR
Time t1 when v = 0
v v0
t1
a g cos sin
Time t2 when = 0
2R
t2 0 0
5g cos
t2 > t1
0 2R v0
5 g cos g cos sin
5 v0 cos
0 > 2R cos sin
5 v0
0
2R tan
ROTATIONAL MECHANICS
QUIZRR 69
MISCELLANEOUS EXAMPLES
Example 1
A thin rod AB of length a has variable mass per unit length 0 (a + x)/a , where x is the
distance measured from A and 0 is a constant. Show that
3a 0
(a) the mass M of the rod is ,
2
5a
(b) the centre of mass of the rod is at the point x
9
(c) the moment of inertia of the rod about an axis through A perpendicular to AB is
7 Ma 2
18
(d) The rod is freely pivoted at A and is hanging in equilibrium when it is struck by a
horizontal impulse of magnitude J at the point B. Show that if B passes through a
1
point vertically above A then J M 70 a g
9
Solution :
a
a
0 a x x2
(a) M a
dx = 0 ax
a
2
0 0
3a0
= Hence proved
2
a
0 a x
a
x dx
dm x 0
a
(b) 0 = 3a0
xCM
M 2
a
2 ax2 x3
= 3a
2 2
0
5a
= Hence proved
9
a a
0 a x
(c) I A ( dm) x2 = x2 dx
0 0
a
a
0 ax3 x4
= a 3 4
0
ROTATIONAL MECHANICS
70 QUIZRR
70 a3 3a0
= M 2
12
7 Ma 2
= Hence proved.
18
(d) Using, angular impulse = change in angular momentum
7Ma2
we have, Ja I A =
18
18J
=
7Ma
A
B will pass through a point vertically above A if,
5a
1 9
I A 2 Mg(2AC)
2
C
1 7Ma 2 18J 2 5a
Mg
2 18 7Ma 9
J
B
M
or J 70 ag Hence proved
9
Example 2
A smooth rod rotates freely in a horizontal plane with the angular velocity 0 about a
stationary vertical axis O, relative to which the rodÊs moment of inertia is I. A small ring
of mass m is located the rod close to the rotation axis and is tied to it by a thread. When
the thread is burned, the ring starts sliding along the rod. Find the velocity vr of the ring
relative to the rod as a function of its distance r from the rotation axis.
Solution :
In the process of motion of the given system the kinetic energy and the angular momentum
relative to the rotation axis do not vary. Hence, it follows that,
2 2
I20 = I + mv ...(i)
0 r
v
I0 = (I + mr ) 2
...(ii) 0
vr
where v2 = vr2 20 r 2 ...(iii) O
r m
0 r
Solving these equations, we get, vr Ans.
mr 2
1
I
ROTATIONAL MECHANICS
QUIZRR 71
Example 3
A uniform rod pivoted at its upper end hangs vertically. It is displaced through an angle
of 60Ĉ and then released. Find the magnitude of the force acting on a particle of mass dm
at the tip of the rod when the rod makes an angle of 37Ĉ with the vertical.
Solution :
, O
Decrease in the height of centre of mass of the rod
l l
h cos 37 cos 60 0.8 0.5 0.15 l 60
2 2
ml 2 2
or 0.15 l mg = 6
I2 = 0.9 g
l
mg 2 sin 37
0
Angular acceleration, = 1 2
I0 ml
3
or l = 0.9 g ...(ii)
2
Normal force, Fn = (dm)l = (0.9)(dm)g
Tagential force, Ft = (dm)l = (0.9)(dm)g
Example 4
A solid ball rolls down a parabolic path ABC from a height h as shown in figure. Portion
AB of the path is rough while BC is smooth. How high will the ball climb in BC ?
[Hint : In case of pure rolling mechanical energy is conserved.]
A C
B
ROTATIONAL MECHANICS
72 QUIZRR
Solution :
At B, total kinetic energy = mgh
m = mass of ball
KR 2
The ratio of rotational to translational kinetic energy would be [Remember this part]
KT 5
2 5
KR mgh and KT mgh
7 7
In portion BC, friction is absent. Therefore, rotational kinetic energy will remain constant and
translational kinetic energy will convert into potential energy. Hence, if H be the height to which
ball climbs in BC, then
mgH = KT
5 5
or mgH = mgh or H h Ans.
7 7
Example 5
A thread is wound around two discs on either sides. The pulley and the
two discs have the same mass and radius. There is no slipping at the
pulley and no friction at the image. Find out the acceleration of the two
discs and the angular acceleration of the pulley.
1 2
Solution :
Let R be the radius of the discs and T1 and T2 be the tensions in the left and right segments of
the rope.
T1 T2
1 1
a1 mg mg a2
mg T1
Acceleration of disc 1, a1 ...(i)
m
mg T2
Acceleration of disc 2, a2 ...(ii)
m
ROTATIONAL MECHANICS
QUIZRR 73
TR 2T
Angular acceleration of disc 1, 1 1 1 ...(iii)
I 1 mR 2 mR
2
2T2
Similarly, angular acceleration of disc 2, 2 ...(iv)
mR
T2 T1 R 2 T2 T1
1 mR ...(v)
mR 2
2
2g T1 T2
= 0 and a1 a2 Ans.
3
Alternate Solution
As both the discs are in identical situation, T1 = T2 and a= 0. i.e., each of the
discs falls independently and identically. Therefore, this is exactly similar to the
problem shown in figure.
Example 6
ROTATIONAL MECHANICS
74 QUIZRR
Equations of motion are :
T f = 3a1 ...(i)
a1
f 2R T.R 0.2 f 0.1 T f T T
...(ii)
I I 0.6 3 6 f
Equation of motion is :
T
10 T = a2 ...(iii)
For no slipping condition,
a1 = 2R or a1 = 0.2 ...(iv) a2
and a2 = a1 R or a2 = a1 0 0.1 ...(v)
Solving the above five equations we get,
a1 = 0.27 m/s2
10 N
and a2 = 0.135 m/s2 Ans.
Example 7
A uniform sphere of radius r starts rolling down without slipping from the top of another
sphere of radius R. Find the angular velocity of the sphere after it leaves the surface of the
larger sphere.
Solution :
The equation of motion for the centre of the sphere at the moment of r h
breaking off :
R
2
mv
mg cos ...(i) mg
Rv
where v is the velocity of the centre of the sphere at that moment and
is the corresponding angle. The velocity v can be found from the energy
conservation law :
mv2 I2
mgh
2 2
2
where I mr 2 , v = r and h = (R + r) (1 cos )
5
From these equations we obtain
10 g R r
Ans.
17r 2
ROTATIONAL MECHANICS
CENTRE OF
MASS AND
MOMENTUM
QUIZRR 3
1. CEN T RE OF M A SS (COM )
The concept of the centre of mass is that of an average of the masses factored by their distances
from a reference point. In one plane, that is like the balancing of a seesaw about a pivot point
with respect to the torques produced.
COM for two bodies
x2
xcm
Center
x1 of mass
m1 m2
m1 x1 m2 x2
For two masses xcm
m1 m2
If you are making measurements from the centre of mass point for a two-mass system then the
centre of mass condition can be expressed as
x
m1x1 = m1x2 or m1 m2 2
x1
where x1 and x2 locate the masses. The centre of mass lies on the line connecting the two masses.
Ex a m p l e 1 :
Two particles of mass 1 kg and 2 kg are located at x = 0 and x = 3 m. Find the position of
their centre of mass.
Solution :
Since, both the particles lie on x-axis, the COM will also lie on x-axis. Let the COM is located at
x = x, then
r1 = distance of COM from the particle of mass 1 kg = x
and r2 = distance of COM from the particle of mass 2 kg
= (3 x)
m1 = 1 kg com m2 = 2 kg
r1 m2
Using = m
r2 1 x=0 x=x x=3
r1 = x r2 = (3 x)
x 2
or or x= 2m Ans.
3x 1
N N N
mi xi mi yi mi zi
i 1 i 1 i 1
xcm ycm zcm
M M M
This approach applies to decreet masses even if they are not point masses if the position xi is taken
to be the position of the centre of mass of the ith mass. It also points the way toward the
calculation of the centre of mass of an extended object.
Ex a m p l e 2
The position vector of three particles of mass m 1 = 1 kg, m 2 = 2 kg and m 3 = 3 kg are
r 1 i→ 4 j→ k→ m , r 2 i→ j→ k→ m and r 3 2i→ j→ 2 k→ m respectively. Find the position
vector of their centre of mass.
Solution :
The position vector of COM of the three particles will be given by
m r 1 m2 r 2 m3 r 3
r COM 1
m1 m2 m3
Substituting the values, we get
rCOM
(1) i→ 4 →j k→ (2) i→ →j k→ 3 2i→ →j 2 k→ = 9i→ 3 →j 3 k→
123 6
rCOM
2
1 → → →
3i j k m Ans.
Ex a m p l e 3
Four particles of mass 1 kg, 2 kg, 3 kg and 4 kg are placed at the four vertices A, B, C and
D of a square of side 1 m. Find the position of centre of mass of the particles.
Solution :
Assuming D as the origin, DC as x-axis and DA as y-axis, we have
m1 = 1 kg, (x1, y1) = (0,1 m) y
m2 = 2 kg, (x2, y2) = (1 m, 1 m) A B
m3 = 3 kg, (x3, y3) = (1 m, 0)
and m4 = 4 kg, (x4, y4) = (0, 0)
Coordinates of their COM are x
D C
m1 x1 m2 x2 m3 x3 m4 x4
xCOM =
m1 m2 m3 m4
COM
5 1
= m 0.5 m
10 2 D C
m1 y1 m2 y2 m3 y3 m4 y4
Similarly, yCOM =
m1 m2 m3 m4
11 2 1 3 0 4 0
=
1234
3
= m = 0.3 m
10
1 .2 Ce n t re o f M a s s : Co n t i n u o u s Sy s t e m
For a continuous distribution of x4
mass, the expression for the centre x3
of mass of a collection of particles : x2
x1
m1 m2 m3 m4
N
xcm mi xi
i1
N
mi xi M
xcm i1 xdm
lim 0
m 0 M M
1 .3 Ce n t re o f m a s s f o r v a r io u s b o d i e s
x
(1) (a) Uniform rod of length l 0 CM L
L
L 2 dm = M dx
M L
x L dx xl
1 x2 L
xcm 0
M L 2 2
x0 L
x
0 1
L
Solution :
Assume an element of thickness dx at a distance x from the axis,
x
then, dm dx 0 1 dx x=0
L
x
Thus, the centre of mass is given by
xdm
X cm =
dm dx
L
x
L2 L3
0
x 0 1 dx
L
2
3L
5L2 2
= L x L2 3L 6
L
0 1 dx L
2L
0
5L
=
9
2R sin 0 Tip : Try to identify the axis of symmetry, wherever possible in case
= 2
0 of calculating COM
Result :
sin 0 / 2
(1) Centre of mass of an arc subtending an angle 0 at centre = 2R [from centre on
0
axis of symmetry
2R
(2) COM of semi-circular ring = {Put here 0 = }
2 2R Put
(3) COM of a quarter circle arc = 0
2
(3) Uniform semi-circular lamina having inner and outer radius R1 and R2.
Solution :
Y
R2
r
R1
O X
(dr)
R2 2r
R1 2r dr
Y dm
YCM =
dm
R
R1 2r dr
2
2r
{Please note that here Y is taken as which was the result obtained in the previous case of
semi-circular arc}
=
2 R 2 R1
3 3
3 R 22 R12
2
YCM
3
4 R 2 R1
3
3 R22 R12
(B) Uniform Laminar arc subtending an angle 0 at centre
Solution :
Consider an arc at a radius r from the centre O
0
dm 2r dr
2
and YCM Y dm
dm R2 r
R2 2r sin 0 / 2 R1
R1 0 r dr
0
= R2
R1 0 r dr 0
0 O
YCM
4 sin R 32 R13
2
30 R 2 R2
2 1
4R
YCM sin 0 {When R1 = 0}
30 2
CENTRE OF MASS AND MOMENTUM
QUIZRR 9
Remember :
4R
* CM of an arc-shaped disc = 3 sin 0 / 2
0
4R
* CM of a semi-circular disc =
3
4R 1 4 2R
CM of a quarter disc = 3 2
2 3
*
/
2
YCM
0 Y dm
/
0
2
dm
Rsin
/ Rd
=
0
2
2 R 2 sin d R cos
d
/
0
2 2
2 R sin d
O
R
/
2
=
R 0 sin cos d
=
R
/ 2
0
2
sin d
Remember :
R
COM of hemi-spherical shell lies on distance from centre on axis of symmetry.
2
/ /
2 2
R cos 0 cos 2R sin R d R cos2 sin d
0 2R
YCM 0
= /
/ 3
2 2
2r dr
2
0
dr
r
{Here taking Y as calculated in case of hemi-spherical shell}
2
R4 3 3R
= 3
24R 8
Remember :
3R
Centre of mass of a hemi-spherical disc of radius R lies on distance from centre on axis
8
of symmetry.
6. (A ) Ce n t re o f m a s s o f a h o l lo w r ig h t c y l i n d e r
O
x
h
dx
H
H
cos
h 2 x sin dx
YCM = Y dm 0
H
dm
cos
2 x sin dx
0
also, h = x cos
=
cos
0
cos x2 dx
2 H cos
H 3 cos
0
cos x dx
2
YCM = H from the point O.
3
YCM =
dm 0 0 tan2 x2 dx
H
3 H4 3H
=
3 4
4H
Remember :
2H
CM of a hollow right circular cone =
3
3H
CM of a solid right circular cone =
4
1 .4 T o f i n d t h e Ce n t re o f M a s s o f r e m a i n i n g p a r t w h e n c e r t a i n p o r t i o n o f b o d y
is sc ooped out
Ex a m p l e 4 :
From a uniform circular disc of radius R, a smaller disc of radius R/2, with circumference
touching each other is scooped out. Find out the CM of remaining body.
Solution :
Steps of solving
(1) Fill the cavity by mass +m, m
MX mx MY my
XCM =
M m YCM =
M m
M M
X x Y y
m m
= M = M
1 1
m m
M R/2
mX x M R 2
4
XCM = m R 2 / 4
M
1
m
0 R /2
=
4 1
/
R
6
... (Y
CM
will remain same at origin
the remaining portion is symmetric
R about x-axis)
co-ordinates of CM = ( /6, 0)
Ex a m p l e 5
From a uniform solid hemisphere of radius R, a solid right circular cone of radius R/2 and
height R/2 is scooped out (a) symmetrically (b) unsymmetrically. Find the position of centre
of mass in each case.
Solution :
(a) Symmetrical cavity
Fill the cavity with + m and m mass.
let M be the masses of the original hemisphere (when the cone is not scooped out) then
2
M R 3 0
3
now, let m be the mass of cone scooped out, then
3
1 1 R
m R 2 h 0 0
3 3 2
2
R3
M
3 16 M
m 1 R
3
m
3 2
My1 my2
Now, YCM Mm
3R H R
y1 , y2
8 4 8
M 3R R
y1 y2 16
m
YCM 8 8
M 16 1
m 1
47 R
=
120
M
Her, also 16
m
M
m X1 X 2 16 0 R / R
X CM 2
M 16 1 30
1
m
M
y1 y2 47R
Now, YCM m
M 120
m 1
R 47R
Co-ordinates of COM are ,
30 120
Ex a m p l e 6
From a right circular cone of base radius R and height R, a uniform solid hemisphere of
radius R/2 is symmetrically scooped out. Find the co-ordinates of centre of mass.
Solution :
let M be the original mass of the cone and m be the mass of hemi-sphere that is scooped out.
1 2 R3
then, M R2 R , m
3 3 8
M
4
m
M 4R 3R
m y y
82
4 13R
YCM
M 4 1 =
1 48
m
CENTRE OF MASS AND MOMENTUM
14 QUIZRR
1 .5 M o t i o n o f Ce n t re o f M a s s
CM
rCM
mi ri
mi
mi ri
r cm
rCM
mi O
vCM
mi vi
aCM
mi ai
mi mi
M VCM mi Vi
PCM P1 P 2 P 3 .......
Total momentum of system is frame of reference S
= momentum of CM is same frame of reference S.
= vector sum of individual momenta of particles in same frame S.
d P CM d Pi
mi a i dt
dt
Fi´ F ex
net
F ex M aCM
All external forces acting on system of particles can be treated
aCM
F ex
to act on centre of mass of system of particles.
M
Fex x Fex ( y) F ( z)
aCM ( x) ; aCM ( y) ; aCM ( z) ex
M M M
F1
P = P1 P2
1
dP d P1 d P 2 F12
dt = dt dt
r1 F21
2
r2 F2
= F1 F12 F 2 ( F 21 )
O
F12 = F21
= 1
F F 2 = F ex
net
Ex a m p l e 7 B
m a mB aB m
Further, aCOM = A A g 10 2 (downwards)
mA mB s
as
aA aB g
m u A mB u B
uCOM A (downwards)
mA mB
2
100
or 0 = 2 10 h
3
or h =
100 2 55.55 m
180
B
Solution :
Net pulling force on the system is (mA mB)g
or (2 1)g = g
Total mass being pulled is mA + mB or 3 kg
a A
mA a A mB a B
Now, aCOM =
mA mB
B a
2 a 1 a a
=
12 3
g
= downwards
9
Alternate Method
Free body diagram of block A is shown in figure. T
2g T = mA (a)
or T = 2g mAa A a
g 4g
= 2g (2) mAg = 2g
3 3
Free body diagrams of A and B both are as shown in Fig.
g
= downwards
9
2. Co n s e r v a t io n o f L i n e a r M o m e n t u m
The conservation law for linear momentum follows is very close to being just a restatement of
Newtons first law that the velocity vector is constant for a body on which no forces act. So you
have in a sense already been using this conservation law in various problems involving Newtons
laws.
One of the beauties of Newtons laws is that there are no restriction on what type of macroscopic
objects we can apply them. We now embark on the use of Newtons laws to examine systems of
particles which in general can have internal motion as well. In this context we shall learn about
the conservation laws for Linear and Angular momentum and about the center of mass for an
object.
First it turns out to be handy to introduce the quantity called linear momentum. For a single
particle the linear momentum is a vector quantity defined as
P mv
d v dP
F ma m
dt dt
Interestingly this was actually the form which Sir Isaac Newton himself chose to work with. A
special case that we have often considered before is when
F 0 P is a constant
If our object under considerations consists of many individual „sub‰ objects then we can still define
a momentum for that system of particles
P P i mi v i
i i
To distinguish it from the linear momentum of individual particles we use a large P for the
momentum of a many particle system. Taking the time derivative of this equation we get
dP dP
dt
dt F i F net
i i
So,
The linear momentum of a system of particles is conserved when no net external force
acts on that system. It is important to note that no net force does not mean that no forces act.
Internal forces are allowed to occur because Newtons third law guarantees that these forces come
in action-reaction pairs and hence they cancel out exactly when we add them up to get the net
force, F net . Thus the presence of large internal forces does not invalidate the conservation of
linear momentum.
Ex a m p l e 9
A wooden plank of mass 20 kg is resting on a smooth horizontal floor. A man of mass 60
kg starts moving from from one end of the plank to the other end. The length of the plank
is 10 m. Find the displacement of the plank over the floor when the man reaches the other
end of the plank.
10m
Solution :
Here the system is man + plank. Net force on this system in horizontal direction is zero and
initially the centre of mass of the system is at rest. Therefore, the centre of mass does not move
in horizontal direction.
Let x be the displacement of the plank. Assuming the origin, i.e., x = 0 at the position shown in
figure.
As we said earlier also, the centre of mass will not move in horizontal direction (x-axis). Therefore,
for centre of mass to remain stationary,
xi = xf
60 0 20
10
60 10 x 20
10
x
2 2
60 20 60 20
or 5 = 30 3x + 5 x or 4x = 30
30
or x m or x = 7.5 m Ans. X 10 X Final Position
4
Note : The centre of mass of the Plank lies at its centre.
Ex a m p l e 1 0
Y
A small sphere of radius R is held against the inner surface
of larger sphere of radius 6R (as shown in figure). The
masses of large and small spheres are 4 M and M 6R
respectively. This arrangement is placed on a horizontal M
table. There is no friction between any surfaces of contact. X
(L, 0) R
The small sphere is now released. Find the coordinates of
4M
the centre of the large sphere, when the smaller reaches
the other extreme position.
Solution :
Since all the surfaces are smooth, no external force is acting on the system in horizontal direction.
Therefore, the centre of mass of the system in horizontal direction will remain stationary. Initial
x-coordinate of COM will be given by
Y Y
6R 6R
C C
X X
(L, 0) (x, 0)
m1 x1 m2 x2 4M L M L + 5R
xi L R ...(i)
m1 m2 4M M
Let, (x, 0) be the coordinates of the centre of large sphere in final position. Then x-coordinate of
COM in final position will be
4M x M x 5R
xf x R ...(ii)
4M M
Po i n t s t o Re m e m b e r
1. In this situation discussed above we can also apply
mR xR mL xL
Here, mR xR is the summation of product of x and m of the particles (or bodies) which are
moving towards right and mL xL is the summation of product of x and m of the particle (or
bodies) which are moving towards left. But remember the following three conditions while using
the above equation.
(i) This equation can be applied when centre of mass does not move in x-direction.
(ii) In the above equation x is the displacement of particle relative to ground.
Let us solve above example using this method.
Here, xL = displacement of plank towards left = x
mL = mass of plank = 20 kg
xR = displacement of man relative to ground towards right = 10 x
and mR = mass of man = 60 kg
Applying xRmR = xLmL, we get
(10 x)(60) = 20x
or x = 30 3x
or 4x = 30
30
x = 7.5 m
4
1 2
(i) The kinetic energy associated with the motion of centre of mass, MvCOM where M is the
2
total mass of the system and
(ii) The kinetic energy associated with the motion of the particles of the system relative to the
1
centre of mass, 2 miui2 , where th
ui is the velocity of the i particle relative to the centre
of mass.
The kinetic energy of a system of the individual particles.
1
K.E. = K i 2 mi vi2
i i
The velocity of each particle can be written as the sum of the velocity of the centre of mass
vCOM
vi vCOM ui
1 1
Then K.E. = 2 mi vi . vi = 2 mi vCOM ui . vCOM ui
i i
1 1
= 2 mi vCOM
2
mi u12
2
vCOM . mi u i
i i i
quantity mi u i is the total momentum of the system relative to the centre of mass. This quantity
i
is necessarily zero. Thus,
1 1 1
K.E. = 2 mi vCOM
2
mi u12 MvCOM
2 2
2
K rel
i i
where M is the total mass and Krel is the kinetic energy of the particles relative to the centre of
1 2
bulk motion MvCOM does not change. Only the relative kinetic energy (Krel) can change in
2
isolated system.
Ex a m p l e 1 1
m
A block of mass m is released from the top of
a wedge of mass M as shown in figure. Find
the displacement of wedge on the horizontal h y
M
ground when the block reaches the bottom of
the wedge. Neglect friction everywhere.
x
Solution :
Here, the system is wedge + block. Net force on the system in horizontal direction (x-direction) is
zero, therefore, the centre of mass of the system will not move in x-direction so we can apply,
xRmR = xLmL
Let x be the displacement of wedge. Then
xL = displacement of wedge towards left = x
mL = mass of wedge = M
mh cot
x =
Mm
Ex a m p l e 1 2
A gun (mass = M) fires a bullet (mass = m) with speed vr relative to barrel of the gun which
is inclined at an angle of 60 with horizontal. The gun is placed over a smooth horizontal
surface. Find the recoil speed of gun.
Solution :
Let the recoil speed of gun is v. Taking gun + bullet as the system. Net external force on the
system in horizontal direction is zero. Initially the system was at rest. Therefore, applying the
principle of conservation of linear momentum in horizontal direction, we get
vr
60
vr sin 60
v vr cos 60
M
components of velocity
of bullet relative to ground
Mv m(vr cos 60 v) = 0
mvr
v = 2 M m
Ex a m p l e 1 3
A boat of length 2 metres and mass M = 40 kg is floating in a river at a distance of 5 m from
the shore. Two persons weighing 50 kg and 60 kg each standing on opposite ends of boat.
Start moving towards each other and stop at the centre of the boat. Then, find :
(a) the displacement of boat relative to ground
Fex = 0
(aCM)x = 0 (VCM)x = (uCM)x = 0
1
= m
15
1
(a) Hence, the boat was moved a distance m towards right.
15
(b) displacement of each man
1 16
x1 = 1 m towards right
15 15
1 14
x2 = 1 15
15
14
i.e., m towards left
15
1 76
(c) The new distance = 5 m
15 15
3
or u x = 100 m = 50 3 m/s
2
and vertical component of its velocity, u y = u sin 30
1
or u y = 100 m/s = 50 m/s
2
Vertical displacement of the ball when it strikes the carriage is 120 m or
1
sy = u y t a yt2
2
1
120 = (50 t) + 10t
2
2
t2 10t 24 = 0
t = 12 s or 2 s
Ignoring the negative time, we have t0 = 12 s Ans.
(ii) When it strikes the carriage, its horizontal component of velocity is still 50 3 m/s. It sticks
to the carriage. Let v2 be the velocity of (carriage + cannon ball) system after collision. The
applying conservation of linear momentum in horizontal direction.
(mass of ball) (horizontal component of its velocity before collision) = (mass of ball + carriage) (v2)
(1 kg) 50 3 m/s = (10 kg) (v2)
v2 = 5 3 m/s
distance of 12v1 or 60 3 m. This ball will strike the carriage only when the carriage also
covers the same distance of 60 3 m in next 12 second. This is possible only when resistive
forces are zero because
Velocity of car (v1) = velocity of carriage after first collision (v2) = 5 3 m/s . Hence at the time
of second collision :
11 v = 1 50 3 10 5 3 100 3
100 3
v = m/s or v 15.75 m/s Ans.
11
In this particular problem, values are so adjusted that even if we take the velocity of cannon ball
with respect to car, we get the same results of both the parts, although the method will be wrong.
Ex a m p l e 1 5
Two identical buggies each of mass M are moving with velocity v0 without friction on
horizontal surface, one after the other. A man of mass m diving on near buggie, jumps with
velocity u relative to his own buggie on the front buggie.
Find the velocity of each buggie after the jump.
Solution :
V0 V0 V1 V2
m m u
M M
v1
M m v0 mu
M m
(ii) For front buggie and man system
let v2 be the combined velocity of buggie 2 and man, (pi)x = (pf)x
Mv0 + m(u + v1) = (m + M)v2
Mv0 m(u v1 )
v2 =
Mm
Ex a m p l e 1 6
Two men, each of mass m are standing on one end of stationary buggie of mass M. Each
jumps off from the buggie with velocity u relative to buggie. Find the velocity of buggie
relative to ground
(a) when both men jumps off simultaneously
(b) when each jumps one after another.
Solution :
(a) When both men jump off simultaneously
v u
m m
initial condition
Final condition
Let us assume t he final velocit y of buggie aft er bot h men jump off is v, then
(pi)x = (pf)x {as no external force acts}
where vg is velocity of each man with respect to ground.
vg = (u v)
0 = M ( v) + 2m (u v)
2 mu
v
M 2m
mu
v1
2m M
v2
m M v1 mu
M m
Ex a m p l e 1 7 10 m/s
v2 = 0.323 m/s
(b) For the shell fired;
u x = (10 cos 37 v2)
u y = 10 sin 37
1
5 = 6t 10t2
2
Ex a m p l e 1 8
A block of mass m starts sliding from the top of an inclined wedge of mass M. Find the
velocity of wedge and block when it reaches the bottom (all surfaces are smooth).
Solution : +ve ve
Fex x 0
(pi)x = (pf)x V0 h mg
mv cos
v0 ... (1)
m M
2
1 1
mgh = Mv02 vb, w v w
2 2
1
2
1
Mv02 m v02 v2 2vv0 cos ( ) mgh
2
M m v02 2mv cos v0 mv2 2mgh 0 ...(2)
m M v02
2
2mgh
v0
m M
M m 1
m cos
1 2 M m gh
and v
cos mM
m 1
m cos
Ex a m p l e 1 9
A small disc of mass m rolls from height h down a smooth hill as shown. A plank of mass
M is placed on the bottom of hill on a smooth surface. When the disc is getting onto the
plank, after certain time, the disc and the plank start moving together. Find the work done
by friction on the disc, on the plank and on disc + plank system.
Solution :
mu
v
M m
1 1
= mv2 mu2
2 2
1 1
= Mv2 0 Mv2
2 2
Tip : Work done by friction on the system is negative and same in all frame of reference.
1 1
Wfr = M m v2 mu2
2 2
1 m2 1
= u2 mu2
2 M m 2
1 u2 Mm
Wfr =
2 M m
Ex a m p l e 2 0
A small disc of mass m is placed on the horizontal portion of a
smooth surface of body of mass M as shown. An initial velocity u
is imparted towards right. Find :
m
u
(a) the maximum height upto which the disc will rise above the
initial level. M
(b) Also find the maximum velocity acquired by the plank.
Solution :
V2
V1
V2 V3
V2
V4
1 1
mgh mu2 m M v22 ... (2)
2 2
1 u2 M
h
2 g m M
(b) The particle of mass m (disc) lands again back on the same platform. Maximum velocity,
hence is achieved when the disc reaches the flat portion of platform.
Note : The disc after breaking off from the platform lands back again on platform,
horizontal range of disc = distance moved forward by the platform in the same time.
(pi ) x = (pf)x {system}
mu = ( m v4) + Mv3 ...(1)
using energy conservation
1 1 1
mu2 mv42 M v32 ...(2)
2 2 2
2 mu
v3
M m
Ex a m p l e 2 1
M
m
l
A
A bob of mass m is is attached to a point A with a light rigid rod as shown, which can slide
down. The block is placed on a smooth horizontal slot. The system is released from rest. Find
(a) the maximum displacement of the block
(b) the velocity of the block, when the rod becomes vertical
Solution :
The block performs the oscillatory motion
consider at an instant P when the bob has moved down by and displacement of block is xm.
A
A A
The rod (with bob) is moving in circle about point A is the reference frame of block.
ml 1 cos
xm
M m
2ml
xm
M m
mv sin
v0
mM
1 1 m
Mv02 m v2 v02 2 vv0 sin mgl sin 2
v0
2 2
v
put and solve yourself.
2
Ex a m p l e 2 2
u u
Two identical beads, each of mass m are brought to the
m
diametrically opposite points of circular ring and imparted an m
initial velocity u as shown. The ring is placed on a smooth O
surfaces. Find the velocity of ring when the beads are about to
collide. M
Solution : +N
N v
(m2 R) N
V0
N
* Due to a net outward normal force, the ring starts moving in positive y direction.
* logically, this can be thought also; since momentum remains conserved in the y-direction. In the
topmost position, when the velocities are horizontal, the momentum remains conserved only when
the ring has velocity along y-direction.
Using momentum conservation
(pi)x = (pf)x
2mu = 2 m[v cos + v0] + v0M ...(1)
using energy conservation
1
2
1
2 mu2 m v2 v02 2vv0 cos Mv02
2
mu2 m v2 v02 2vv0 cos 1
2
Mv02 ...(2)
2mu
when the beads are about to collide then = /2 v0
M 2m
Note : In the question, when for example, two insects starts crawling w.r.t.
ring without the interference of an external force then to conserve the
momentum, the ring moves in y-direction.
Solution : v
s
co
vcos
v0
v0
v cos vcos
T T
v1
v1
A m B m C m
[Assume the velocity in the directions, one along the length of the string and another perpendicular
to the length].
Along the length of the string, velocity = vcos
[ relative longitudinal velocity along the string = 0]
velocity perpendicular to the string = v1 (say)
Momentum remains conserved
mv0 + 0 + 0 = mv + 2m [v cos2 v1 sin ]
2
mv0 = mv + 2m [cos v1 sin ] ...(1)
using energy conservation
1
2
1 1
mv02 mv2 2 m v2 cos2 v12
2 2
...(2)
Ex a m p l e 2 4
Two identical blocks A and B each of mass m are connected by a A m
spring string system, such that initial compression in the spring
7mg
7 mg l0 =
K
=
k
B m
Find the maximum vertical displacement of CM (after the string has
been burnt through).
Solution : Initial N = k l0 + mg = 8 mg
(N0 = 8 mg)
On releasing the spring, the normal force starts decreasing on the system N
and motion of CM is accelerated with decreasing acceleration. But when
N 2 mg, the CM starts de-accelerating, still moving upward, until the
l0
lower block B breaks off from the ground, after which the motion of CM
is under gravity.
(i) Till the break-off point, the displacement of CM mg
1
2
1
k l02 l 2 mg l l0 mv2
2
k l02 l2 2mg l l0 mv2
7mg
l0 k
k
v2 l0 l 2 g l0 l
m l mg
k
8mg 32 mg 2
= 4 g.
k k
m1 v1 m2 v2 mv v
velocity of CM =
m1 m2 2m 2
v2CM 2 2
y´2 v 32 mg 4mg
2g 8g k 8g k
8mg
y1´ y2´
k
A n a l y s is f r o m c e n t re o f m a s s f r a m e
y
When we analyse the motion of the particles of system from the
reference frame of CM, following results must be remembered : r´
x
CM
(1)
mi ri´ 0 [where ri is the position vector wrt CM frame]
r
mi
x
O
(2)
mi vi´ 0
[ v´i is the velocity of ith particles in CM frame]
mi
(3)
mi ai´ 0
[ ai´ is the acceleration of ith particle in CM frame]
mi
(4) From (2), we can easily conclude
The vector sum of linear momenta of particle of a system is always zero in CM frame.
Ex a m p l e 2 5
Find the maximum deformation in the spring in situation shown :
F1 (CM) F2
m1 m2
Solution :
(A) Analysis from CM frame
Let the individual displacement be x1´ & x2´ of m1 and m2 at maximum deformation.
(1) we know, at maximum deformation relative longitudinal
velocity must be zero (v1 + v2 = 0) ...(1) v1́ v2́
CM
(2) In CM frame, Pi 0 m2 m1
Now, easily check that v1´ 0, v´2 0 is the only solution possible for (1) and (2).
F F1 a
aCM 2
m1 m2 kx1
CM
kx2
(m1a) (m2a)
Fnet ´2 F2 m2 a
F2 F1 m1 F2 m2 F1
= F2 m2
m1 m2 m1 m2
m1 F2 m2 F1
Fnet ´1 F1 m1 a 2 (F2 m2a)
m1 m2
1 2
F1´ x1 Fx2 k x1 x2 (F1 + m1a) 1
2
Energy conservation
F1´ x1´ 1
F2´ x´2 k x1 x2
2
2
F.B.D. of blocks
m1 F2 m2 F1 1
x1 x2 k( x1 x2 ) + KE´ {KE´ = 0 since v1´ v2´ 0 }
m1 m2 2
2 m1 F2 m2 F1
x1 x2
k m1 m2
F1 K F2
m1 m2
At maximum deformation velocity of both the block is same in magnitude ( relative longitudinal
velocity = 0)
Wext = K + U
1 1 2 1
F1x1 + F2x2 = m1 v m2 v k x2 x1
2 2
2 2 2
1 1
F1x1 + F2x2 = m1 m2 vCOM
2
k ( x2 x1 ) 2 ...(1)
2 2
aCM
F2 F1 m x1 m2 x2
, xCM
m1 m2 m1 m2
2
vCM 2 aCM xCM ...(2)
1 F2 F1 m1 x1 m2 x2 1
F1 x1 F2 x2 m1 m2 2 k x2 x1
2
2 m 1 m 2 m 1 m 2 2
F1 x1 F2 x2 m1 m2 F2 F1 m2 x2 m1 x1 1 k x x 2
2 1
m1 m2 2
F1 m2 F2 m1 x2 F1 m2 F2 m1 x1 1
k m1 m2
2
m1 m2 2
2 F1 m2 F2 m1
x1 x2
K m1 m2
3 . V A RI A B L E M A SS
Problems related to variable mass can be solved in following three steps
1. Make a list of all the forces acting on the main mass and apply them on it.
dm
2. Apply an additional thrust force F t on the mass, the magnitude of which is vr and
dt
direction of if it is decreasing.
vr
dv
3. Find net force on the mass and apply F net m (m = mass at that particular instant)
dt
3 .1 Ro c k e t Pr o p u l s i o n
Consider the powered flight of a rocket vehicle. Assuming the following :
1. The rocket may have an initial velocity
2. The rocket is initially unpowered.
3. Gravity is negligible.
4. The propellant mass flow rate is constant.
5. The exhaust velocity relative to the rocket is constant.
Time = t
u, m
v + v , m m
Time = t + t
Note that
v = absolute rocket velocity
u = absolute exhaust gas velocity
m =mass
The initial momentum of the system Pi is
Pi = mv (1)
The final momentum of the system Pf is
Pf = (m m)(v + v) + m u (2)
The change in momentum P is
P = Pf Pi (3)
P = [(m m)(v + v) + m u] mv (4)
P = mv + mv mv – mv + mu – mv (5)
P = mv vm – mv + mu (6)
P = [u (v + v)] m + mv (7)
Let c be the exhaust gas velocity relative to the rocket. Recall that c is assumed to be constant.
c = [u (v + v)] (8)
Substitute equation (8) into (7).
P = c m + mv (9)
The change in momentum with respect to time is
P m v
c m (10)
t t t
dP dm dv
c m (11)
dt dt dt
dP
F
dt
The external force applied to the rocket is zero, however. The thrust is generated internally. Thus
dP
0
dt
dm dv
c m 0
dt dt
dm dv
c m ...(2)
dt dt
dm
c dv
m
dm
dv c ...(3)
m
Let
m0 = initial rocket mass
mf = final rocket mass
v0 = initial rocket velocity
vf = final rocket velocity
Integrate equation (3)
vf mf dm
v0 dv m0 c
m
mf
vf v0 c 1n m m0
m
vf c 1n 0 v0
mf
Ex a m p l e 2 6
(a) A rocket set for vertical firing weighs 50 kg and contains 450 kg of fuel. It can have
a maximum exhaust velocity of 2 km/s. What should be its minimum rate of fuel
consumption
(i) to just lift it off the launching pad?
(ii) to give it an acceleration of 20 m/s2?
(b) What will be the speed of the rocket when the rate of consumption of fuel is 10 kg/s
after whole of the fuel is consumed? (Take g = 9.8 m/s2)
Solution :
(a) (i) To just lift it off the launching pad
weight = thrust force
dm
mg = vr
dt
dm mg
dt = v
r
dm 450 50 9.8
dt =
2 103
Ft
a = g
m
vr dm
a = g
m dt
dm m ( g a)
This gives dt = vr
dm 450 50 9.8 20
dt =
2 103
= 7.45kg/s Ans.
450
t = 45s
10
m0
Using Eq. (1), i.e., v = u gt + vr 1n
m
500
v = 0 (9.8)(45) + (2 103) 1n
50
or v = 441 + 4605.17
or v = 4164.17 m/s
or v = 4.164 km/s
4. I M PU L SE
An impulse is defined as the integral of a force with respect to time. When a force is applied to
a rigid body it changes the momentum of that body. A small force applied for a long time can
produce the same momentum change as a large force applied briefly, because it is the product of
the force and the time for which it is applied that is important.
t2
I t1 F dt
where
I is impulse (frequently marked J).
F is the force, and
dt is an infinitesimal amount of time.
t1 and t2 denote a time interval
A simple derivation using NewtonÊs second law yields :
t2 dP
I t1 dt
dt
t2
I t1 dP
I = P
where
P is momentum
This is often called the impulse-momentum theorem.
As a result, an impulse may also be regarded as the change in momentum of an object to which
a force is applied. The impulse may be expressed in a simpler form when both the force and the
mass are constant :
I = Ft = mv = p
where
F is the constant total net force applied.
t is the time interval over which the force is applied.
m is the constant mass of the object
v is the change in velocity produced by the force in the considered time interval, and
mv = (mv) is the change in linear momentum.
It is often the case that not just one but both of these two quantities vary.
In the technical sense, impulse is a physical quantity, not an event or force. The term „impulse‰
is also used to refer to a fast-acting force. This type of impulse is often idealized so that the change
in momentum produced by the force happens with no change in time. This sort of change is a step
change, and is not physically possible. This is a useful model for certain purposes, such as
computing the effects of ideal collisions, especially in game physics engines.
Impulse has the same units and dimensions as momentum (kg m/s = N.s)
Ex a m p l e 2 7
A truck of mass 2 103 kg travelling at 4 m/s is brought to rest in 2 s when it strikes a wall.
What force (assume constant) is exerted by the wall?
Solution :
Using impulse = change in linear momentum
F u +ve
We have F . t = mvf mvi = m(vf vi)
or F(2) = 2 103 [0 ( 4)]
or 2F = 8 103
or F = 4 103 N Ans.
Ex a m p l e 2 8
A bullet of mass 10 3 kg strikes an object and moves at 60 to its original direction. If its
speed also changes from 20 m/s to 10 m/s. Find the magnitude of impulse acting on the
bullet.
20 m/s J1 10 cos 60
120 60
10 m/s J2
10 sin 60
3
or J 1 = 15 10 N-s
Similarly, parallel to J2, we have
J 2 = 10 3
[sin 60 0] = 5 3 103 N-s
The magnitude of resultant impulse is given by
15 2 5
2
J = J12 J 22 10 3 3
J = 3 102 N-s
Ex a m p l e 2 9
A particle of mass 2 kg is initially at rest. A force starts acting on it in one direction whose
magnitude changes with time. The force time graph is shown in figure. Find the velocity
of the particle at the end of 10 s. F(N)
20
10
O t(s)
2 4 6 10
Solution :
Using impulse = Change in linear momentum (or area under F-t graph)
We have
m(vf vi) = Area
1 1 1
or 2(vf 0) = 2 10 + 2 10 + 2 (10 + 20) + 4 20
2 2 2
= 10 + 20 + 30 + 40
or 2vf = 100
vf = 50 m/s Ans.
Po i n t s t o r e m e m b e r
(1) Impulsive Tensions : When a string jerks, equal and opposite tensions act suddenly at
each end. Consequently equal and opposite impulses act on the objects to which the two ends
of the string are attached. There are two cases to be considered.
(a) One end of the string is fixed : The impulse which acts at the fixed end of the string
cannot change the momentum of the fixed object there. The object attached to the free end
however will undergo a change in momentum in the direction of the string. The momentum
remains unchanged in a direction perpendicular to the string where no impulse forces act.
(b) Both ends of the string attached to movable objects : In this case equal and opposite
impulses act on the two objects, producing equal and opposite changes in momentum. The
total momentum of the system therefore remains constant, although the momentum of each
individual object is changed in the direction of string. Perpendicular to the string however,
no impulse acts and the momentum of each particle in this direction is unchanged.
The velocities of two objects moving at the ends of a taut string are not independent. The
relationship between them is illustrated as under :
v1 v1 sin 1 v2 cos 2
1
A B 2
v2 A v1 cos 1 v2 sin 2
AB is a taut string. Particles A and B are moving with velocities as shown in the diagram.
The important components of velocity are those along AB since :
if v1 cos 1 > v2 cos 2 the string is not taut
and if v2 cos 2 > v1 cos 1 the string is snapped.
Hence, for the string to remain taut and unbroken,
1 = v2 cos 2
v1 cos
So, the two ends of a taut string have equal velocity components in the direction of the
string.
(2) Spring cannot exert impulsive forces
Reason : Spring can compress and elongate under the action collision.
(impulsive
tension) string ha tendency
to become slack
Ex a m p l e 3 0 A
A string AB of length 2l is fixed at A to a point on a smooth horizontal
table. A particle of mass m attached to B initially at a distance l from A
l
as shown in figure. The particle is projected horizontally with speed u at
right angles to B. Find the impulsive tension in the string when becomes
taut and the velocity of the particle immediately afterwards. u
B
Solution :
When the string becomes taut AB = 2l
and B´AB = 60
Just before the string jerks that the particle has velocity components parallel and perpendicular
to AB´ of u sin 60 and u cos 60 respectively. When the string becomes taut the length of AB
is fixed and particle cannot travel in the direction AB´. After the jerk the velocity of the particle
is, therefore, perpendicular to AB´.
A A
J
60 u cos 60
2l
l
J
B´
B B´ u sin 60
3
or J = mu Ans.
u
u
or v = u cos 60 =
2
u
Therefore, the velocity of the particle just after the string becomes taut is perpendicular to the
2
string.
5. COL LI SI ON
Contrary to the meaning of the term ÂcollisionÊ in our everyday life, in physics it does not necessarily
mean one particle ÂstrikingÊ against other. Indeed two particles may not even touch each other and
may still be said to collide. All that is implied is that as the particles approach each other.
(i) an impulse (a large force for a relatively short time) acts on each colliding particles
(ii) the total momentum of the particles remain conserved.
Now, let us have a look at various stages of collision between two bodies
m1 m2 m1 m2
u1 u2 v1 v2
Fi Fi
line of
impact
t
Impulse during
collision
Note : In collisions and explosions, neglect the effect of the constant forces for very short time
intervals, unless the time of impact is mentioned.
ucos
Due to very short time, the effect of constant
force mg can be neglected.
V
Mg
Here, momentum cannot be conserved in vertical directions.
III. Now, analyze a collision between similar bodies both moving horizontally.
N1 N2
Momentum can be conserved in any
direction,
øN1 N1, N2, øN1, øN2 are all constant forces
øN2 and hence an be neglected.
mg
N
Momentum of the system cannot be
conserved in vertical direction as here also N
is impulsive.
5 .1 H e a d -On a n d Ob l iq u e Co l li s io n
Head-On Collisions : If the velocities of point of contact of two bodies before collision coincide
with line of impact.
u1 u2
(CN : Common normal)
Oblique Collision : If the velocity of point of contact of any of the colliding bodies, does not lie
along the common normal, then it is oblique collision.
u1 u2
1 2
Note : If there is no friction at the surfaces of contact, then the component of velocities perpendicular
to common normal remains unchanged.
u1 sin 1 u2 sin 2
u1 u2
1 u2 cos 2 2
m1 m2
A u1 cos 1
B
(Before collision)
u1 sin 1 u2 sin 2
N N
(CN)
(During collision)
We can see here that an impulsive force N acts on both the bodies in opposite directions.
u1 sin 1 u2 sin 2
v1
v2
A B
(After collision)
5 .2 N e w t o n ’s L a w o f Re s t i t u t io n
When two objects are in direct (head on) impact, the speed with which they separate after impact
is usually less than or equal to their speed of approach before impact.
Experimental evidence suggests that the ratio of these relative speeds is constant for two given
set of objects. This property formulated by Newton, is known as the law of restitution and can
be written in the form
separation speed
e ...(1)
approach speed
The ratio e is called the coefficient of restitution and is constant for two particular objects.
In general 0 e 1
e = 0, for completely inelastic collision, as both the objects stick together. So, their separation speed
is zero or e = 0 from (1).
e = 1, for an elastic collision, as we can show that
m1 em2 m2 em2
v1´ = m m v1 m m v2
1 2 1 2
m2 em1 m1 em1
and v2´ = m m v2 m m v1
1 2 1 2
Sp e c i a l Ca s e s
1. If collision is elastic, i.e., e = 1, then
m m2 2m2 m2 m1 2m1
v1 ´ 1 v1 v2 and v2 ´ v2 v1
m1 m2 m1 m2 m1 m2 m1 m2
m1 v1 m2 v2
v1 ´ v2 ´ v´ (say)
m1 m2
3. If m1 = m2 and v1 = 0, then
m2 m1 v1´
v2 v2´
v1 = 0
1 e 1 e
v1 ´ v2 and v2 ´ v2
2 2
Remember :
The equation from coefficient of restitution is always written along the line of impact.
e = 1 (elastic collision)
< e < 1 (inelastic collision)
e = 0 (perfectly inelastic collision)
e
v0 wd v
u 0
Note :
1. Displacement of colliding particles are neglected just before and after collision.
2. If mass of one body is very-very greater than that of the other, then after collision velocity
of heavy body does not change appreciably. (Whether the collision is elastic or inelastic).
1 e 1 e
v1 ´ v and v2 ´ v
2 2
3
Given that Kf = Ki
4
1 1 31 2
or m1 ´ 2 mv2 ´ 2 = mv
2 2 42
2 2
1 e 1 e 3
2 2 =
4
or (1 + e)2 + (1 e)2 = 3
or 2 + 2e2 = 3
1
or e2 =
2
1
or e = Ans.
2
Ex a m p l e 3 2
A ball is moving with velocity 2 m/s towards a heavy wall moving 2m/s 1m/s
towards the ball with speed 1 m/s as shown in figure. Assuming
collision to be elastic, find the velocity of ball immediately after the
collision.
Solution :
The speed of wall will not change after the collision. So let v be the velocity of the ball after the
collision in the direction shown in figure. Since collision is elastic (e = 1),
Ex a m p l e 3 3
Two blocks of mass 2 kg and M are at rest on an inclined
plane and are separated by a distance of 6.0 m as shown. The m M
6.0
coefficient of friction between each block and the inclined 2kg
plane is 0.25. The 2 kg block is given a velocity of 10.0 m/s up
the inclined plane. It collides with M, comes back and has a
velocity of 1.0 m/s when it reaches its initial position. The
other block M after the collision moves 0.5 m up and comes to rest. Calculate the coefficient
of restitution between the blocks and the mass of the block M. [Take sin tan = 0.05 and
g = 10 m/s2]
Solution :
Let v1 = velocity of 2 kg block just before collision (up the plane)
v2 = velocity of 2 kg block just after collision (down the plane)
and v3 = velocity of block M just after collision (up the plane)
Applying work energy theorem (change in kinetic energy = work done by all the forces) at
different stages as shown in figures (a), (b) and (c) :
Figure (a) : KE = Wfriction + Wgravity
1
2
2
2 m v1 10 =
6ø mg cos mgh1 (m = 2 kg) v1
/s
or v12 100 = 2 [6øg cos + gh1] 10m
2kg m
6.0
1 sin 1 0.05 0.99
2
cos = 2
h 1 = 6 sin = 0.3 m
(a)
v12 100 2[(6) 0.25 10 0.99 10 0.3 ]
or v1 8 m/s
M m
or v32 = øg cos 2 gh2 0.5
h 2 = 0.5 sin = 0.025 m
(c)
or v32 = (0.25)(10)(0.99) + 2(10)(0.25)
v2 v3 5 1.72 6.72
=
v1 8 8
or e 0.84 Ans.
(ii) Applying conservation of linear momentum before and after collision.
2v1 = Mv3 2v2
2 v1 v2 2 8 5 26
M = =
v3 1.72 1.72
M 15.12 kg Ans.
Ex a m p l e 3 4
A ball of mass m hits a floor with a speed v0 making an angle of incidence with the
normal. The coefficient of restitution is e. Find the speed of the reflected ball and the angle
of reflection of the ball.
Solution :
The component of velocity v0 along common tangent direction v0 sin will remain unchanged.
Let v be the component along common normal direction after collision. Applying
v
v0 sin v0 sin
v0
v0 cos
tan
or tan = Ans.
e
Ex a m p l e 3 5
A ball of mass m makes an elastic collision with another identical ball at rest. Show that
if the collision is oblique, the bodies go at right angles to each other after collision.
Solution :
In head on elastic collision v sin
between two particles, they v sin
1
exchange their velocities. In this v
1
case, the component of ball, 1
along common normal direction, v cos
v cos becomes zero after
2
collision, while that of 2 becomes
2
v cos . While the components v cos
along common tangent direction Before collision After collision
Po i n t s t o r e m e m b e r
Suppose a ball is a projected with speed u at an angle with horizontal. It collides at some
distance with a wall parallel to y-axis as shown in figure. Let vx and vy be the components of its
velocity along x and y directions at the time of impact with wall. Coefficient of restitution between
the ball and the wall is e. Component of its velocity along y-direction (common tangent) vy will
remain unchanged while component of its velocity along x-direction (common normal) vx will
become evx is opposite direction.
vy
vy v
vx
evx
u y
x
Further, since vy does not change due to collision, the time of flight (time taken by the ball to
return to the same level) and maximum height attained by the ball will remain same as it would
had been in the absence of collision with the wall. Thus,
A
E D
u
u
O F C
B
O
2u sin
tOAB = tCD + tDEF = T =
g
u2 sin 2
and h A = hE =
2g
Further, CO + OF Range or OB
It collision is elastic, then
u2 sin 2
CO OF Range
g
Ex a m p l e 3 6
A ball is projected from the ground with speed u at an angle with horizontal. It collides
with a wall at a distance Âa Ê from the point of projection and returns to its original position.
Find the coefficient of restitution between the ball and the wall.
Solution :
As we have discussed in the theory, the horizontal component of the velocity of ball during the
path OAB is u cos while in its return journey BCO it is eu cos . The time of flight T also
remains unchanged. Hence,
T = tOAB + tBCO
2u sin a a
or =
g u cos eu cos
C
a 2u sin a
or =
eu cos g u cos
A
ag
e= 2
2u sin cos ag
1
or e= Ans.
u sin 2
2
1
ag
5 .3 I m p u l s e -Mo m e n t u m Eq u a t i o n
F
Impulse = F ext dt area under
the curve =
d p dt change in momentum
J F ext
dt = dt
p
t
J F ext dt p O
(t)
Jx Fext x dt px
Jy F ext dt py Net impulse of force in any
y direction is equal to change in
that direction
J z Fext z pz
Note : Impulse momentum conservation to be used in those questions in which momentum cannot
be conserved.
Ex a m p l e 3 7
Two identical balls each of mass m are suspended by identical strings. A C
third ball C of mass 2m but having same radius falls vertically from height
h and collides simultaneously with A and B. Find the velocity of each ball 2m
just after collision. (e = 0.5)
A m m B
Solution :
v T
N N
N line of impact
v1
A
2m C
N Mg
N 2mg
(after collision)
(after collision)
v1 cos60 v cos30
e ...(1)
u cos30 0
wher e v1 is the velocity of ball A in the horizontal direction after the collision has occured
and V is the velocity of ball C after the collision.
(iv) u 2 gh ...(4)
v1 3v
0.5 2 2 6 gh v 3 v
1
3 ... (5)
2 gh
2
and
3 2 mv1 2m v 2 gh
3 v1 v 2 gh ...(6)
3
6 gh 3 v v 2 gh
3 2 gh 4v 2 gh
gh
v
2
3 gh
v1
2
3 gh
Hence the velocity of balls A and B are gh and of C is
2 2
30
30
u=0
m2
B
B
Li pact V2
im
(instant of
ne
collision)
of
v2 v1 cos60
e ... (3)
v0 cos30
From (1) and (2)
m1v1 = m2v2
and m1v0 = 2m2v2 cos 30
m1 v0
v2
m2 3
v0
and v1
3
from (3)
m1 v0 v 1
e 0
m2 3 3 2
2m1 m2
e
3 m2
V1 N
d
N
V0 nd
2 V
N
V2 N
Just before collision Velocities after collision impulse on each bodies
v1 v cos
e 1 ... (2)
v cos 0
v
n 2 v
2
0
6 n 2
v = 0, n 2
n < 2
Solution : A 3m
As the ball collides with B, an impulsive tension is exerted by the string, which breaks off the
block A from ground. The block A after acquires certain velocity, moves with constant retardation
and attains a maximum height. T0 T0
(i) Momentum cannot be conserved in vertical v
writing the impulse momentum eqn. {for block A and ball system] v
2mg 3mg
T dt 2mv mu ... (1)
{Please note that block A and B will have same velocity as the string is tight}
Add (1) and (2) and solve
u 2 gh
v
5 5
3m 2m g
(ii) Constant retardation of A = g
3m 2m 5
u2 5 u2 2 gh h
Maximum height gained by A =
25 2 g 10 g 10 g = 5
Important results in case of Head on collision
m1 m2 m1 m2
v1 v2
u1 u2
v1
m1 m2 u1
2m2 u2
m1 m2 1 m2
m
v2
m2 m1 u2
2m1 u1
m1 m2 m1 m2
This can be used in oblique collision also provided u1, u2 are comp. of velocities along the line of
impact.
CENTRE OF MASS AND MOMENTUM
QUIZRR 63
Variations :
(i) If m1 = m2, velocities of colliding masses get interchanged
(ii) If m2 >> m1
v1 = 2u2 u1
v2 = u2
Ex a m p l e 4 1
Two block A and B, each of mass 2m are connect by an ideal string and
suspended as shown. A ball of mass m moving with initial velocity v0 collides
with B and sticks to it. Find :
(a) the max. height upto which block B will rise above the initial
position A 2m 2m B
(b) Time taken to reach the max. height and V0
(c) total loss in K.E. during motion. m
Solution :
The ball m strikes the block B and the string sticks, as tension becomes zero, due to collision. B,
acquires velocity due to (T = 0) in string. Both the blocks move under free fall until, the string
again becomes taut and equalises their velocity. After this consider, the motion of blocks as that
of two block and pulley system.
(i) During collision using momentum conservation we get
v0
mv0 = 3mv v =
3
(ii) Let the time after which the string become taut again be t
XA = XB {i.e., displacement of one block should be equal to the other for the string to
become taut}
1 2 1 v
gt vt gt2 t=
2 2 g
velocity of B just before string becomes taut
v´ = u gt = v g (v/g) = 0
velocity of A = gt = v
v
(iii) When the string becomes taut at t , let the common velocities be v3.
g
for B Tdt 3mv3 ... (1)
T T
for A Tdt 2m v3 v ... (2)
0 = 5mv3 2mv v3
B 2m A 2m
2v 2v0 mu=0 v v3
v3 =
5 15
1 2 v32 g
= vt gt a 5
2 2a
v 1 v2 v32
= v g 2 g 2 2a
g
v2 v2 4v2
= g 2g 5g 2
2
1 1 v
K1 (loss in KE during collision) = mv02 3m 0
2 2 3
1 2 1
K2 loss in KE when the string becomes taut = 2m v 3m 2m v3
2
2 2
Ex a m p l e 4 2
In the given arrangement, the bullet collides with mass m and
A
sticks to it. Find : m/s
0 m
=1 P
v0
v=
2
(ii) Now, the velocities of A and B just before string becomes taut is, V and 0 respectively.
Tdt Mv1 A
B
and Tdt 2m v1 v V1
Mg
Mv1 = 2m (v1 v)
4 v1 = 2 (v1 v)
v
v1 m/s
3
M 2 m sin 3g g
acceleration in block = g=
M 2m 6 2
v2 v2 v2
height gained by B =
2a g 9g
92
2
v v 2v
time taken to reach height B =
g
a 3 3g
2
Ex a m p l e 4 3
A simple pendulum of length l is held in a vertical plane in horizontal position as shown
and released from rest. The ball collides with e = 0. Find the maximum height upto which
the bob will rise on the other side if
(a) ø = 0, (b) ø 0
P l
l/
2
Solution :
(a) When there is no friction :
l
u 2g gl
2
P l
e = 0, u cos 30 component becomes zero and the ball moves 30
l/ N
along the surface with u sin 30 , till at the other end it 2
u sin 30
experiences an impulsive tension.
u
u cos 30
2
u sin
T
u sin
v2 u2 sin4 gl l
maximum height rises = =
2g 2g 32 g 32
[Note : loss of KE occurs at the colliding point and due to impulsive tension as well}
and N dt m v u sin v øN
ø mu cos = m (v u sin )
v = u(sin ø cos ) P
1 1 2 l 3
mv2 mv12 mg
2 2 2 V1
v12 v2 2g l 3 ...(1) M N
Now,
v1 cos = 0 V1 sin P
v12 sin2
maximum height risen above ground =
2g V1
v2 2 g l 3
sin2
= 2g
Ex a m p l e 4 4
A small ball is projected with initial velocity 10 m/s and = 37 . Find the range of the ball
after first collision (e = 0.5) if,
(a) ø = 0.5, (b) ø = 0.9
6m/s N
V2
37
8m/s øN V1
Solution :
(i) Coefficient of restitution equation
0 v2
e 0.5
6
v2 = 3 m/s
(ii) Momentum equation after first collision
In horizontal : N dt 9m = 4.5 m
J > N dt
3.5m
v1 3.5 m/s
m
Total range = R1 + R2
uy 2v2
= u g v1 g
8 2 6 3.5 2 3 117
= 11.7 m
g 10
826
total range = R1 9.6 m
g
J Jmax
uf = vi
m
u uy 2ux
= 2 g g euy
2u 2
g
e uy 2ux 3
g
e u y ...
2 v02 cos sin
= 1 e e2 e3 ......
g
v02 sin 2
=
g 1 e
Time of flight =
2v0 sin
g
1 e e2 e3 ....... 0
2v sin
g (1 e)
v0 v
{as it stretches uniformly} dx
l x x (k,m)
V0
M
m
dm = dx {where dm is the mass of element}
l
2
k l1 v
0 dk dx x 0
02 l
1 m 2 l 2 1
k
2 l3
v0 0 x dx
6
mv02
1 m 2
k v0
2 3
m
You can consider a mass of attached to mass in spring of mass m; and then
3
v0 v
Remember, obtained only when spring stretch uniformly.
l x
v2 v1 v v1
l x
V1 dx V2
x v
v = v1 + v2 v1
l
x
K l 2
1 x
dK 2 dx v1 l v2 v1
0 0
Ex a m p l e 4 5
A uniform chain of mass m, and length l is held in a vertical position with lower end just
touching the horizontal surface and is released from rest. Find the net force exerted by the
chain on the horizontal surface, when it falls down by a length y.
Solution : m, l
Note that two kinds of the force are exerted on the surface by the fallen
part, one due to weight of already fallen part and the other is the
thrust due to the just fallen part of the chain due to sudden change in dy
momentum.
y y
m
Let be the mass per unit length of chain
l
v 2 gy
d p 0 dyv ( j dy v
dp dyv 2
v
dt dt
thrust = v2
Total force exerted by the chain = v2 + (g)y {Here gy is the weight of fallen part}
3mgy
= (2gy) + gy = 3gy =
l
Ex a m p l e 4 6
A uniform chain of length l and mass m is placed as a heap on a smooth horizontal surface.
Now the chain starts slipping from the hole in the surface below. Find the velocity of the
chain when it has slipped off by a length y and time taken.
Solution :
The overhanging parts exerts a thrust on the heap as a result of which the heap
pulls the hanging part by an equal opposite force.
Analysing the forces on the falling part y
dp v dy where v = 2 gy
v2
dP
F
dt
dy
v v
dt
2
write the force equation for the length y of the chain
gy
dv
y v gy v2
dy
dv
yv
dy
v2 gy
1
2
d y2 v2 gy2 dy
v2 y2 gy3
y v dv + v2 dy = gy dy c
2 3
y2 v dv + v2y dy = gy2 dy At y = 0, v = 0 c = 0
2g
v2 y
3
2 gy
(velocity of the fallen part of length y) =
3
dy 2 gy
dt 3
y t
dy 2g 2g
y
3 dt 2 y
3
t
0 0
3y
t= 2
2g
Ex a m p l e 4 7
Z
A wedge of mass m and triangular cross section (AB = BC
A
= CA = 2R) is moving with a constant velocity vi→ towards v
R
sphere of radius R fixed on smooth horizontal table as
X
shown in figure. The wedge makes an elastic collision with B C
the fixed sphere and returns along the same path without
any rotation. Neglect all friction and suppose that the Y
wedge remains in contact with the sphere for a very short
time t, during which the sphere exerts a constant force F on the wedge.
(a) Find the force F and also normal force N exerted by the table on the wedge during the
time t.
(b) Let h denotes the perpendicular distance between the centre of mass of the wedge and
the line of action of F. Find the magnitude of the torque due to the normal force N
about the centre of the wedge during the interval t. (JEE 1998)
Solution :
The collision is perfectly elastic. Hence, the wedge will recede with the same velocity, as sphere
is fixed.
The change in momentum of the wedge is given by
px = 2mv ...(i)
At the time of collision, the different forces are shown in figure below
Component of force along X axis = F cos 30
Hence, F cos 30 t = 2mv
4 mv
F = ...(ii)
3t
In vector form →
F = F cos 30 i + F sin 30
k→
Z
N
F =
4mv
3 t
.
2
3→
i
4 mv 1
3 t 2
. k→
30
60 mg
30 2mv → 1 →
60 F = i k ...(iii)
X t 3
4 mv 1
mg . =N
3 t 2
2mv
N mg ...(iv)
3 t
In vector form N 2mv / 3 t mg k→ ...(v)
0, i.e., F N mg 0
Here, mg 0 N F
4 mvh
N F F.h Ans.
3t
Ex a m p l e 4 8 y
A structure is made up of three masses, m 1 = m 2 = 2 kg and m 3 m1
= 3 kg, connected rigidly by massless rods. At the moment of
interest, the coordinates of the three masses are (1.25 m, 3 m),
(2 m, 2 m) and (0.75 m), respectively. At the same instant, m2
(a) Scalar calculations : Let (xcm, ycm) be the coordinates of the mass-centre.
Then from the definition of mass-center,
mi xi
mi x i m2 x2 m3 x3
xcm =
mi m1 m2 m3
7.5 kg . m
= 1.25 m
6 kg
Similarly,
mi yi
mi y i m2 y2 m3 y3
ycm =
mi m1 m2 m3
1kg . 3 m 2 kg . 2 m 3 kg .0.5 m
= 1kg 2 kg 3 kg
8.5 kg . m
= 1.42 m
6 kg
Thus the centre of mass is located at the coordinates (1.25 m, 1.42 m)
3
mtot rcm = mi ri m1 r1 m2r2 m3 r
3
i 1
m1 r1 m2r2 m3 r
rcm = 3
m1 m2 m3
r2 = 2mi→ 2mj→ , and
r3 = 0.75 mi→ 0.5 mj→, we get,
1 kg . 1.25i→ 3 →j m 2 kg . 2i→ 2 →j m 3 kg . 0.75i→ 0.5 →j m 7.5i→ 8.5 →j kg . m
rcm = =
1 2 3 kg 6 kg
rcm 1.25 mi→ 1.42 mj→
= li→ l →j
r1
l
r2 = 2li→ →j
2
y
2l m1
l → 5l →
r3 = 2l i i c1 m2
2 2
l c2 m3
c3
x
m r m2 r2 m3r3
l
= 1 1
rcm
mtot
=
l 5l
2m li→ l →j m 2li→ →j m i→
2 2
4m
1 5
ml 2i→ 2 →j 2i→ →j i→
= 2 2
4m
=
l
8
13i→ 5 →j
=
0.5 m
8
13i→ 5 →j
= 0.812 + mi→ 0.312 mj→
rcm 0.812 mi→ 0.312 mj→
Ex a m p l e 5 0 2m
.5m
Solution :
Let us use an xy-coordinate system with its origin at the geometric center of the plate and the x-
axis passing through the center of the cut-out. Since the plate and the cut-out are symmetric
about the x-axis, the new center of mass must lie somewhere on the x-axis. Thus, we only need
to find xcm (since ycm = 0). Let m1 be the mass of the plate with the hole, and m2 be the mass
of the circular cut-out. Clearly, m1 + m2 = m = 4 kg. The center of mass of the circular cut-out
is at A, the center of the circle. The center of mass of the intact square plate (without the cut-
out) must be at O, the middle of the square. Then,
m1xcm + m2xA = mxO = 0
m2
xcm = xA .
m1
Now, since the plate is uniform, the masses m1 and m2 are proportional to the surface areas of
the geometric objects they represent, i.e.
y
m2 r 2
m1
= 2 2
2
l r l
r A
O r x
Therefore,
d
m
xcm = 2d d
m1 l
2
2m
2
= .05 m
2
2m
.25 m
3
= 25.81 10 m = 25.81 mm
Thus the center of mass shifts to the left by about 26 mm because of the circular cut-out of the
given size.
xcm = 25.81 mm
1 .1 .1 Wo r k d o n e b y a c o n s t a n t f o r c e
When the point at which a force acts moves, the force is said to have done work.
When the force is constant, the work done is defined as the product of the force and distance moved.
Work done = force distance moved in direction of force
Consider in Figure a force F acting at the angle moves a body from point A to point B.
F
os
sc
A B
s
The distance moved in the direction of the force is given by
Distance in direction of force = s cos
So the work done by the force F is
Work done = F s cos
If the body moves in the same direction as the force the angle is 0.0 so
Work done = Fs
When the angle is 90 then the work done is zero.
The SI units for work are Joules J (with force F, in NewtonÊs N and distance, s, in metres m).
Note : Under the action of a constant force work done is path independent,
i.e., it depends on initial and final positions only. For example, if an object takes 1 B
from position A to position B through three different paths 1, 2 and 3 under a
2
constant force say F 2i→ 3 j k→ , then work done by this force in all three
paths will be same, or A
3
W1 = W2 = W3
Fig. 4.4
Similarly, work done by force of gravity m g near the surface of earth is path independent as
force of gravity is constant. Although we will see later that being a conservative force work done by force
of gravity is path independent at greater heights also.
Now let us take few examples related to above article.
Ex a m p l e 1
How much work is done when a force of 5 kN moves its point of application 600 mm in the
direction of force.
Solution : Work done = (5 103) (600 10 3)
= 3000J
= 3 kJ
Ex a m p l e 2
Find the work done in raising 100 kg of water through a vertical distance of 3 m.
Solution : The force is the weight of the water, so
work done = (100 9.81) 3
= 2943 J
Ex a m p l e 3
Two unequal masses of 1 kg and 2 kg are attached at the two ends of a light
inextensible string passing over a smooth pulley as shown in figure. If the
system is released from rest, find the work done by string on both the blocks
in 1 s. Take g = 10 m/s2.
1kg
2kg
40 5 200
3 3 1 9 J Ans.
WORK, ENERGY & POWER
QUIZRR 5
40 5 200
3 3 1 9 J Ans.
1 .1 .2 Wo r k d o n e b y a v a r ia b l e f o r c e
Forces in practice will often vary. Consider the case where the force varies as in Figure.
For the thin strip with width ds - shown shaded in Figure the force can be considered constant at
F. The work done over the distance ds is then
work done = F ds
This is the area of the shaded strip.
The total work done for distance s is the sum of the areas of all such strips. This is the same as the
area under the Force-distance.
force
0 distance
ds
1 .1 .3 Ge n e r a l Co n c e p t
If point of application of force suffers an infinitesimal vector d r relative to a frame of reference S, then
infinitesimal work done by force during infinitesimal displacement, relative to same frame of reference S.
d W F.d r
dr F
Two important points to be noted,
r
(i) frame of reference must be mentioned
(ii) point of application is important in case of rigid body.
W dW F.d r
If F = constant
W F. r [F is constant in magnitudes as well as in direction].
Ex a m p l e 4
A particle is moved along a circular arc AB by applying a force F constant O R B
in magnitude.
Find the work done by the force on particle from A to B if R
(i) F is acting always horizontally
(ii) Always acting tangentially A
= F. r = F r cos R r
4 /4
1 F
= FR 2 FR
2
(ii) When F is acting tangentially
W F. d r
F
dr
d W F. d r F ds cos 0
= F ds
R
W F ds F
2
(iii) When F is acting always directed towards B
/
d W F. d r 2
= F d r cos = F ds cos d
4 2 4 2
F
WF cos ds
4 2 dr
/4
/
4
= FR cos d (ds = Rd)
4 2 (ds = Rd)
0
FR 2
Ex a m p l e 5
A block of mass m is slowly hauled up a hill as shown. The
coefficient of friction b/w the block and hill is ø.
F is applied always tangentially. Find the work done by force.
m
Solution :
Note : Radial and tangential acceleration are zero when the block is ø
slowly hauled up the plane.
For an infinitesimal part of the plane,
l
F = øN + mg sin (1)
mg cos = N (2)
F = mg (ø cos + sin )
dx = ds cos
F
dy = (ds sin ) d N F. d r
= F ds cos ds
øN
= F ds
W mg cos sin ds mg
Ex a m p l e 6
Find out the work done by friction from t = 0 s to t = 2 s, on A øs = 0.4
(i) upper block 5kg øk = 0.3
(a) relative to ground (b) relative to bigger block
10kg F = 75 N
(ii) Lower block
(a) relative to ground (b) relative to itself B
(iii) total work done by frictional force on system of 2 block, relative to ground and to
bigger block.
Solution :
Force on upper bock is only frictional force which provides acceleration
amax = øg
= 4 m/s
Now, if we consider the two blocks as a system
then their common acceleration
75 fs
acommon 5m
15 fs
F = 75 N
acommon > amax
Distance covered by the upper block Distance covered by the lower block
1 1
x1 rel. to ground a1 t2 x2 , g a2 t2
2 2
= 6m 1
= 6 a 2 12 m
1 2
x1,2 a1 a2 t2 x2, 2 = 0
2
1
=
1
3 6 4 6 m x2,1 6 3 4 6 m
2 2
* CONCEPT
The work done by friction on system of 2 bodies relative to all frame of reference will be equal, and
(i) will be negetive if there is relative slipping between 2 bodies.
(ii) will be zero, if there is no relative slipping.
Wfr fk . d r ´ d r fk . d r
= f k . d r ´ (negetive in sign)
1 .1 .4 Sp r in g Fo r c e
Consider the situation shown in figure. On end of a spring is attached to a fixed vertical support
and the other end to a block which can move on a horizontal table. Let x = denote the position of the
WORK, ENERGY & POWER
QUIZRR 9
block when the spring is in its natural length. When the block
is displaced by an amount x (either compressed or elongated)
a restoring force (F) is applied by the spring on the block.
The direction of this force F is always towards its mean
position (x = 0) and the magnitude is directly proportional x=0
F
to x or
F x (HookeÊs law)
F = kx ...(i)
x
Here, k is a constant called force constant of spring and
depends on the nature of spring. From Eq. (i) we see that F is
F
variable force and F-x graph is a straght line passing through
origin with slope = k. Negative sign in Eq. (i) implies that
the spring force F is directed in a direction opposite to the x=x
X
displacement x of the block
Let us now find the work done by this force F when the
block is displaced from x = 0 to x = x. This can be obtained
either by integration or the area under F-x graph. Fig. 4.10
Thus,
x x x
1 2
W d W Fdx Fdx kx dx kx
0 0 0
2
x2
kx dx 2 k x1
1
W 2
x22
x1
Ex a m p l e 7
A force F = (2 + x) acts on a particle in x-direction where F is in newton and x in metre. Find
the work done by this force during a displacement from x = 1.0 m to x = 2.0 m.
Solution : As the force is variable, we shall find the work done in a small displacement from x to x +
dx and then integrate it to find the total work. The work done in this small displacement is
dW = F dx = (2 + x) dx
2.0 2.0
Thus, W dW 2 x dx
1.0 1.0
2.0
x2
= 2 x 2 3.5 J Ans.
1.0
Ex a m p l e 8
k
A Force F = x 0 acts on a particle in x-direction. Find the work done by this force in
x2
displacing the particle from x = a to x = + 2a . Here k is a positive constant.
2a 2a
k k k
Sol u t i on : W F dx 2 dx x
a x a 2 a
Note : It is important to note that work comes out to be negative which is quite obvious as the force
k
acting on the particle is in negative x-direction F 2 while displacement is along positive
x
x-direction. (from x = a to x = 2a)
1 .1 .5 Co n s e r va t i ve a n d N o n -c o n s e r va t i ve f o r c e s
1 .1 .5 .1 Co n s e r va t i ve f o r c e
It is a force, work done by which does not depend on the path along which body is moved, but
depends on initial and final positions only.
B
F c . d r 0
A
1 .1 .5 .2 N o n -c o n s e r va t i ve f o r c e
Work done by the force depends on the path followed along which the particle moves.
Example friction, viscous force
Fnc . d r 0
Examples
(a) Non-conservative force
d W F. d r
= fk ds
W fk ds
W = ømgs depends on path followed.
F
øN
GMm
dr = 2 ds cos
r
A
F
r GMm
= 2 dr
x
O
rf
GMm 1 1
W dr = GMm
ri r j
2
r1 r
1 .2 EN ERGY
A body which has the capacity to do work is said to possess energy.
For example, water in a reservoir is said to possesses energy as it could be used to drive a turbine
lower down the valley. There are many forms of energy e.g. electrical, chemical heat, nuclear, mechanical
etc.
The SI units are the same as those for work, Joules J.
In this module only purely mechanical energy will be considered. This may be of two kinds, potential
and kinetic.
1 .2 .1 K in e t ic e n e r g y
Kinetic energy may be described as energy due to motion.
The kinetic energy of a body may be defined as the amount of work it can do before being brought
to rest.
For example when a hammer is used to knock in a nail, work is done on the nail by the hammer
and hence the hammer must have possessed energy.
1 .2.1 .1 Fo r m u l a e f o r k i n e t ic e n e r g y
Let a body of mass m moving with speed v be brought to rest with uniform deceleration by a constant
force F over a distance s.
v2 = u2 + 2as
0 = u2 + 2as
v2
s
2a
WORK, ENERGY & POWER
12 QUIZRR
v2
= F
2a
The force is F = ma so
v2
work done = ma
2a
1
= mv2
2
1
This kinetic energy is given by kinetic energy = mv2
2
1 .2 .1 .2 K in e t ic e n e r g y a n d w o rk d o n e
When a body with mass m has its speed increased from u to v in a distance s by a constant force
F which produces an acceleration a, then from we known
v2 = u2 + 2as
1 2 1 2
v u as
2 2
multiplying this by m give an expression of the increase in kinetic energy (the difference in kinetic
energy at the end and the start).
1 1
mv2 mu2 mas
2 2
Thus since F = ma
increase in kinetic energy = Fs
but also we know
Fs = work done
So the relationship between kinetic energy can be summed up as
Work done by forces acting on a body = change of kinetic energy in the body
This is sometimes known as the work-energy theorem.
1 .2 .1 .3 Wo r k En e r g y T h e o r e m
This theorem is a very important tool that relates the works to kinetic energy. According to this
theorem
Work done by all the forces (conservative or non conservative, external or internal) acting on a
particle or an object is equal to the change in kinetic energy of it.
Wnet K.E. K f K i
Let F1 , F2 ... be the individual forces acting on a particle. The resultant force is F F1 F 2 ... and
the work done by the resultant force is
W F. d r
F1 F 2 ... . d r
= F1 . d r F2 . d r ...
where F1 .d r is the work done on the particle by F1 and so on. Thus, work energy theorem can
also be written as : work done by the resultant force is equal to the sum of the work done
by the individual forces.
1 .2 .1 .3 .1 Wo r k -En e r g y Eq u a t io n s
Let Fc be the conservative force, Fnc be non-conservative and F ex be the external forces acting on
a body, Then
md v
F c F nc F ex
dt
dv
Fex m . d r F c . d r F nc . dr F ex . dr
dt
Fc
Fnc
O m v . d v F c . d r F nc . d r F ex . dr
d v. v
dv
2
v.d v
2 2
mdv2
F c . d r F nc . d r F ex . dx
2
Now, Fc .dr = dWc {Work done by conservative forces}
Fnc .dr = dWnc {Work done by non-conservative forces}
Fex .dr = dWex { work done by external}
1
d mv2 d Wc d Wnc d Wex
2
1
Now, mv2 = Kinetic Energy
2
d K d W K = W (1)
K i ui uf Kf (3)
Ex a m p l e 1 0
A car of mass 1000kg travelling at 30 m/s has its speed reduced to 10 m/s by a constant
breaking force over a distance of 75 m. Find
(a) The cars initial kinetic energy
(b) The final kinetic energy
(c) The breaking force
1
Solution : (a) Initial kinetic enrgy = mv2
2
= 500 302
= 450000 J
= 450 kJ
1
(b) Final kinetic energy = mv2
2
= 500 102
= 50 kJ
(c) KE = 400 kJ
Work done = change in kinetic energy
Breaking force 75 = 400,000
Breaking force = 5333.33 N
Ex a m p l e 1 1
An object of mass m is tied to a string of length l and a variable force
F is applied on it which brings the string gradually at angle with the
l
vertical. Find the work done by the force F.
F
m
Solution : In this case three forces are acting on the object
1. tension (T)
2. weight (mg) and h = l (1 cos)
3. applied force (F) l
T
Using work-energy theorem
Wnet = K.E. F
h
or WT + Wmg + WF = 0 ...(i)
mg
as K.E. = 0 because Ki = Kf = 0
Further, WT = 0, as tension is always perpendicular to displacement.
Wmg = mgh or Wmg = mgl (1 cos )
Substituting these values in Eq. (i), we get
WF = mgl (1 cos ) Ans.
Note : Here the applied force F is variable. So if we do not apply the work energy theorem we will
first find the magnitude of F at different locations and then integrate dW F. d r with proper limits.
1 .2 .2 Po t e n t i a l En e r g y
The energy possessd by a body or system by virtue of its position or configuration is known as the
potential energy. For example, a block attached to a compressed or elongated spring possesses some
energy called elastic potential energy. This block has a capacity to do work. Similarly, a stone when
released from a certain height also has energy in the form of gravitational potential energy. Two
charged particles kept at certain distance has electric potential energy.
Regarding the potential energy it is important to note that it is defined for a conservative force field
only. For non-conservative forces it has no meaning. The change in potential energy (dU) of a system
corresponding to a conservative internal force is given by
dU
dU F . d r dW F dr
rf
or
F. d r or U f U i
F. d r
r
i
We generally choose the reference point at infinity and assume potential energy to be zero there,
i.e. if we take ri = (infinite) and Ui = 0 then we can write
r
U F. d r W
or potential energy of a body or system is the negative of work done by the conservative forces
in bringing it from infinity to the present position.
Regarding the potential energy it is worth noting that :
1. Potential energy can be defined only for conservative forces and it should be considered to be
a property of the entire system rather than assigning it to any specific particle.
2. Potential energy depends on frame of reference.
Now, let us discuss three types of potential energies which we usually come across.
(a ) Ela s t ic Po t e n t i a l En e r g y
We have seen that the work done by the spring force (of course conservative for an ideal spring)
1 2
is kx when the spring is stretched or compressed by an amount x from its unstretched position.
2
Thus,
1
U W kx2
2
1 2
or U kx (k = spring constsant)
2
Note that elastic potential energy is always positive.
(b ) Gra v i t a t i o n a l Po t e n t i a l En e r g y
The gravitational potential energy of two particles of masses m1 and m2 separated by a distance r
is given by
No need to remember these formulae,
m1 m2 as these will be discussed under a
UG
r separate chapter
Here, G = universal gravitation constant
11 N m2
= 6.67 10
kg 2
If a body of mass m is raised to a height ÂhÊ from the surface of earth, the change in potential energy
of the system (earth + body) comes out to be :
mgh
U
h (R = radius of earth)
1 R
or U mgh if h << R
Thus, the potential energy of a body at height h, i.e., mgh is really the change in potential energy
of the system for h << R. So be careful while using U = mgh, that h should not be too large. This we
will discuss in detail in the chapter of Gravitation.
(c ) Ele c t ri c Po t e n t i a l En e r g y
The electric potential energy of two point charges q1 and q2 separated by a distance r in vacuum
is given by
This part will be done in detail later on
1 q1 q2
U . in other chapter
4 0 r
1 N m2
Here, 9.0 109 constant
4 0 C2
NOTE :
1. Change in potential energy is equal to the negative of work done by the
conservative force (U = W). If work done by the conservative force is
negative change in potential energy will be positive or potential energy
O
of the system will increase and vice-versa. Ground
This can be understood by a simple example. Suppose a ball is taken Fig 4.23
from the ground to some height, work done by gravity is negative, i.e.,
change in potential energy should increase or potential energy of the ball will increase. Which
happens so.
Wgravity = ve
U = + ve (U = W)
or Uf Ui = + ve
dU
2. F , i.e., conservative forces always act in a direction where potential
dr
F
energy of the system is decreased. This can also be shown in Fig.
If a ball is dropped from a certain height. The force on it (its weight) acts in a
direction in which its potential energy decreases. Fig. 4.24
3. Potential energy U of a particle basically depends on its position, rather we can say its co-ordinates
x, y and z. thus, potential energy of a particle is a function of its co-ordinates x, y and z or
U = U (x, y, z)
Now suppose potential energy function is known to us, then the force (of course conservative) acting
on it is given by :
U → U → U →
F i j k
x y z
U
Here = partial derivative of U with respect to x. On the other hand, if the force acting on the
x
particle is given to us and we want to find potential energy function, then we will use,
f f
dU F . d r
dU F . d r or
i i
But while using this equation, at least the potential energy of the particle at some reference point
should be known to us. For example, if we are given that U = 0 at r = . Then
( x, y, z ) ( x, y, z )
dU F.dr
x, y, z x, y, z
or U (x, y, z) U () = F.dr or U x, y, z F. d r
as U () = 0
Similarly, if we are given that U = U0 at origin. Then
x, y, z x, y, z x, y, z
dU F.dr or U x, y, z U 0, 0, 0 F.dr
0, 0, 0 0, 0, 0 0, 0, 0
x, y, z x, y, z
or U x, y, z U0 0, 0, 0 F . d r or U x, y, z U0 0, 0, 0 F . d r
4. Suppose a particle is released from point A with u = 0
u=0
A
h
v
Fig. 4.25
Friction is absent everywhere. Then velocity at B will be
v 2 gh
(irrespective of the track it follows from A to B)
Here, h = hA hB
5. Suppose a car is moving with constant speed in the track as shown in figure. Then equations of
motion are as under.
mv2 mv2
mg NA or NA mg
RA RA A
v C
v
mv2 mv2
NB mg or NB mg v
RB RB
B
and NC = mg
Thus,we can say N B > NC > NA
Ex a m p l e 1 2
Force acting on a particle in a conservative force field is :
(i) F 2i→ 3 j→ (ii) F 2 xi→ 3 y 2 j→ (iii) F yi→ xj→
x, y, z
or U x, y, z U 0, 0, 0 0, 0, 0 2dx 3dy
or U (x, y, z) = ( 2x 3y) Ans.
as, U (0, 0, 0) = 0 (given)
(ii)
x, y, z
0, 0, 0
dU
x, y, z
0, 0, 0
F . dr
x, y, z
0, 0, 0
2 xi→ 3 y2 →j . dxi→ dyj→ dzk→
(iii)
x, y, z
0, 0, 0
dU
x, y, z
0, 0, 0
F . dr
x, y, z →
0, 0, 0
yi xj→ . dxi→ dyj→ dzk→
x, y, z
or U (x, y, z) U (0, 0, 0) = 0, 0, 0 ydx xdy
x, y, z x, y, z
or U (x, y, z) = 0, 0, 0 d xy xy0, 0, 0
or U (x, y, z) = xy Ans.
1 .2 .3 Co n s e r va t i o n o f e n e r g y
The principle of conservation of energy state that the total energy of a system remains constant.
Energy cannot be created or destroyed but may be converted from one form to another.
Take the case of a crate on a slope. Initially it is at rest, all its energy is potential energy. As it
accelerates, some of it potential energy is converted into kinetic energy and some used to overcome
friction. This energy used to overcome friction is not lost but converted into heat. At the bottom of the
slope the energy will be purely kinetic (assuming the datum for potential energy is the bottom of the
slope.)
If we consider a body falling freely in air, neglecting air resistance, then mechanical energy is
conserved, as potential energy is lost and equal amount of kinetic energy is gained as speed increases.
If the motion involves friction or collisions then the principle of conservation of energy is true, but
conservation of mechanical energy is not applicable as some energy is converted to heat and perhaps
sound.
Ex a m p l e 1 3
A cyslist and his bicycle has a mass of 80 kg. After 100 m he reaches the top of a hill, with
slope 1 in 20 measured along the slope, at a speed of 2 m/s. He then free wheels the 100 m
to the bottom of the hill where his speed has increased to 9 m/s. How much energy has he
lost on the hill?
Solution : If the hill is 100 m long then the height is
100m
1 h
h 100 5m
20
So potential energy lost is
m gh = 80 9.81 5 = 3924 J Dimensions of the hill
Increase in kinetic energy is
1
2
1
2
1
mv2 mv2 m v2 u2
2
= 40(81 4)
= 3080 J
By the principle of conservation of energy
Initial energy = Final energy + loss of energy (due to friction etc.)
loss of energy (due to friction etc.) = 3924 3080 = 844 J.
I m p o r t a n t Po i n t s t o Re m e m b e r
1. If only conservative forces are acting on a system of particles and work done by any
other external force is zero, then mechanical energy of the system will remain conserved.
In this case some fraction of the mechanical energy will be decreasing while the other will
be increasing. Problems can be solved by equating the magnitudes of the decreases and the
increase. Let us see an example of this.
In the arrangement shown in figure string is light and inextensible and friction is A
absent everywhere. Find the speed of both the blocks after the block A has ascended a B
height of 1 m. Given that mA = 1 kg and mB = 2 kg. (g = 10 m/s2). Fig. 4.28
Solution :
Friction is absent. Therefore, mechanical energy of the system will remain conserved. From constant
relations we see that speed of both the blocks will be same. Suppose it is v. Here gravitational potential
energy of 2 kg block is decreasing while gravitational potential energy of 1 kg block is increasing.
Similarly, kinetic energy of both the blocks is also increasing. So we can write :
Decrease in gravitational potential energy of 2 kg block = increase in gravitational potential energy
of 1 kg block + increase in kinetic energy of 1 kg block + increase in kinetic energy of 2 kg block
1 1
mBgh = mAgh + mA v2 mB v2
2 2
WORK, ENERGY & POWER
QUIZRR 21
2. If some non-conservative forces such as friction are also acting on some parts of the system and
work done by any other forces (excluding the conservative forces) is zero. Then we can apply
Wnc = Ef Ei
or Wnc = (Uf Ui) + (Kf Ki) = U + K
i.e., work done by non-conservative forces is equal to the change in mechanical (potential + kinetic)
energy. But note that here all quantities are to be substituted with
A
sign. Let us see an example of this.
In the arrangement shown in figure, mA = 1 kg, mB = 4 kg.
String is light and inextensible while pulley is smooth. Coefficient B
of friction between block A and the table is ø = 0.2. Find the speed
of both the blocks when block B has descended a height h = 1 m.
Fig. 4.29
Take g = 10 m/s2.
Solution : From constraint relation, we see that
vA = vB = v (say)
Force of friction between block A and table will be
f = ømAg = (0.2) (1) (10) = 2N
Wnc = U + K
1
fs = mBgh + mA mB v2
2
1
or ( 2) (1) = (4) (10) (1) + 4 1 v2
2
2 = 40 + 2.5 v2
or 2.5 v2 = 38
V2 =15.2 m2/s2
or v = 3.9 m/s Ans
3. We have discussed in the chapter of Laws of motion, the problems related to momentary rest but
not in equilibrium. These are solved by energy method as under :
In the arrangement shown in figure, the block of mass m is released
from rest with the spring unstretched. The force constant of the spring
is K. Find the maximum extension in the spring. Also find the speed of
block in its equilibrium position. Neglect friction everywhere.
Solution : Here we cannot apply
mg
Kxm = mg or xm Fig. 4.30
K
For maximum extension in the spring. Because at maximum extension, i.e., at point C block is not
in equilibrium. It is momentarily at rest. After some time it will move up, i.e.
Kxm > mg
So, that net force on it is upwards.
But since friction is absent, we can apply conservation of
mechanical energy or decrease in gravitational potential energy
of block = increase in elastic potential energy of spring. A v=0
x0
1 B v xm
So, mg xm K xm
2
2
C v=0
2 mg
or xm Ans. Fig. 4.31
K
At equilibrium position (where net force on the block is zero)
Kx0 = mg
mg xm
or x0 x0 2
K
Let v be the speed of the block in this position. Applying conservation of mechanical energy, we get
decrease in gravitational energy of block = increase in kinetic energy of block + increase in elastic
potential energy of spring
1 1
mg x0 mv2 Kx02
2 2
K 2
v 2 gx0 . x0
m
mg m
Substituting x0 , we get v g Ans.
K K
A
4. Problems of laws of motion on frictionless surfaces can
also be solved by the principle of conservation of mechanical
energy. Here is an example is support of this theory.
Find the acceleration of rod A and wedge B in the B
arrangement shown in figure. Given that mA = m and mB = 2
m. All contacts are smooth.
Solution : Let acceleration of A = a (downwards) Fig. 4.32
Then, acceleration of B = a cot (rightwards)
1 2
If the system is released from rest, in time t height descended by A, hA at
2
Velocity of A, vA = at
and velocity of B, vB = (a cot )t
1 1
or mA ghA mA v2A mB vB2
2 2
mgat2 ma 2 t2
or ma 2 t2 cot 2
2 2
g g cot
Thus, acceleration of A is 2 and that of B is
1 2 cot 1 2 cot 2
Ex a m p l e 1 4
Consider the situation shown in figure. Mass of block A is m and that of block B is 2 m. The
force constant of spring is K. Friction is absent everywhere. System is released from rest
with the spring unstretched. Find
(a) the maximum extension of the spring xm A
(b) the speed of block A when the extension in the spring is
xm
x= B
2
(c) net acceleration of block B when extension in the spring
xm Fig.4.33
is x =
4
Solution : (a) At maximum extension in the spring
vA = vB = 0 (momentarily)
Therefore, applying conservation of mechanical energy :
decrease in gravitational potential energy of block B = increase in elastic potential energy of spring.
1
or mB gxm Kxm
2
2
1 2
or 2 mgxm Kxm
2
4 mg
xm Ans.
K
xm 2 mg
(b) At x
2 K
let vA = vB = v (say)
Then, decrease in gravitational potential energy of block B = increase in elastic potential energy of
spring + increase in kinetic energy of both the blocks.
1 1
mB gx Kx2 mA mB v2
2 2
WORK, ENERGY & POWER
24 QUIZRR
2
2 mg 1 2 mg
2m g
1
or K m 2m v2
K 2 K 2
2m
v 2g Ans.
3K
xm mg
(c) At x , net upward force on block B is Kx or mg and net downward force on block B
4 K
is 2 mg.
Fnet = 2 mg mg = mg (downwards)
Fnet mg
a
mB 2m
g
or a (downwards) Ans.
2
Ex a m p l e 1 5
In the arrangement shown in figure m A = 4.0 kg and m B = 1.0 kg.
The system is released from rest and block B is found to have a
A
speed 0.3 m/s after it has descended through a distance of 1 m.
Find the coefficient of friction between the block and the table.
Neglect friction elsewhere. Take g = 10 m/s2.
1 1
we get ø mAgSA = mBgSB + mA vA2 mB vB2
2 2
Here SA = 2SB = 2 m as SB = 1 m (given)
1 1
4.0 10 2 1 10 1 4 0.6 2 1 0.32
2 2
or 80 ø = 10 + 0.72 + 0.045
or 80ø = 9.235 or ø = 0.115 Ans.
1 .3 POWER
Pow er is t he rat e at w hic h w ork is done, or t he rat e at w hic h energy is used t ransferred.
work done
Power =
time taken
dW F. d r
instantenous power = dt F. V
dt
Pinstantaneous = F. V
If force is constant
F. d r r
Ex a m p l e 1 6
A constant force of 2 kN pulls a crate along a level floor a distance of 10 m is 50 s. Find power
used.
Solution : Work done = force distance
= 2000 10
= 20000 J
work done
Power =
time taken
20000
= = 400 W.
50
distance 10
v = 0.2 m/s
time 50
and then calculated power
Power = Force Speed = Fv
= 2000 0.2 = 400 W.
Ex a m p l e 1 7
A hoist operated by an electric motor has a mass of 500 kg. It raises a load of 300 kg
vertically at a steady speed of 0.2 m/s. Frictional resistance can be taken to be constant at
1200 N. What is the power required?
Solution : Total mass = m = 800 kg
Weight = 800 9.81
= 7848 N
Total force = 7848 + 1200
= 9048 N
Ex a m p l e 1 8
A car of mass 900 kg has an engine with power output of 42 kW. It can achieve a maximum
speed of 120 km/h along the level.
(a) What is the resistance to motion?
(b) If the maximum power and the resistance remained the same what would be the
maximum speed the car could achieve up an incline of 1 in 40 along the slope?
Solution :
Wsin
N
40
1200 N
Wcos
1
W = 900g
120 1000
120 km / h
3600
= 33.33 m/s
(a) Calculate the resistance Power = Force speed
= Resistance speed
42000 = Resistance 33.33
42000
Resistance 1260 N
33.33
(b) Total force down the incline= frictional force + component of weight down incline
= 1260 + mg sin
1
= 1260 + 900 9.81
40
= 1260 + 221
= 1481 N
Power = force speed
Power
Speed =
force
42000
=
1481
= 28.4 m/s
3600
Or in km/h Speed = 28.4
1000
= 102 km/h.
NOTE :
Work done by normal force on system in all frame of reference is zero.
Reason :
dr
Work done on system
N
N. d r´ d r N.d r´
dr´
N
N.d r ´ N.d r N.d r ´ N.d r 0
{as N is perpendicular to dr }
1 .1 1 T h r e e T y p e s o f Eq u i l ib r i u m
As we have studied in the chapter of ÂLaws of motionÊ a body is said to be in translatory equilibrium
if net force acting on the body is zero, i.e.,
F net 0
dU
if the forces are conservative F
dr
and for equilibrium F = 0
dU dU
So,
0, or 0
dr dr
i.e., at equilibrium position slope of U-r graph is zero or the potential energy is optimum (maximum
dU
or minimum or constant). Equilibrium are of three types, i.e., the situation where F = 0 and 0
dr
can be obtained under three conditions. These are stable equilibrium, unstable equilibrium and neutral
equilibrium. These three types of equilibrium can be better understood from the following three figures :
dU dU dU
2. 0 or slope of U-r graph 0 or slope of U-r graph 0 or slope of U-r graph
dr dr dr
is zero is zero is zero
3. When displaced from its When displaced from its When displaced from its
equilibrium position a net equilibrium position, a net force equilibrium position the body has
restoring force starts acting on starts acting on the body which neither the tendency to come back
the body which has a tendency moves the body in the direction nor to move away from the
to bring the body back to its of displacement or away from the original position.
equilibrium position. equilibrium position
4. Potential energy in equilibrium Potential energy in equilibrium Potential energy remains constant
position is minimum as compared position is maximum as compared even if the body is displaced from
to its neighbouring points or to its neighbouring points its equilibrium position
d2 U d2 U d2 U
positive or negative or 0
dr 2 dr 2 dr 2
5. When displaced from equilibrium When displaced from equilibrium When displaced from equilibrium
position the centre of gravity of position the centre of gravity of position the centre of gravity of
the body goes up. the body comes down. the body remains at the same level.
Note :
1. If we plot graphs between F and r or U and r, F will be zero at equilibrium while U will be
maximum, minimum or constant depending on the type of equilibrium. This all is shown in Fig.
F U
C
r r
A B C D A B D
dU
At point A, F = 0, = 0, but U is constant. Hence, A is neutral equilibrium position. At points
dr
dU
B and D, F 0, 0 but U is maximum. Thus, these are the points of unstable equilibrium.
dr
dU
At point C, F = 0, = 0, but U is minimum. Hence, point C is in stable equilibrium position.
dr
(a) (b)
2. Oscillations of a body take place about stable equilibrium position. For example, bob of a pendulum
oscillates about its lowest point which is also the stable equilibrium position of bob. Similarly, in
Fig. (b), the ball will oscillate about its stable equilibrium position.
Ex a m p l e 1 9
A funnel type object is rotating at a speed of about the vertical axis. An object is placed
on the inner side of inclined part of funnel. What kind of equilibrium does object possess ?
Solution
N = mg cos + m2dsin
mgsin m2d = F net = 0
On displaing the body down, d decreases N
m2d
dF
Fnet > 0 >0 mg
dr
It is an unstable equilibrium
Ex a m p l e 2 0
A sleeveof mass m can freel slide along a smooth rod, bent in the form
of semi-circle of radius R, as shown. The rod is rotated with constant
, about a vertical axis.
Find the values of for equilibrium position and decide the nature of
equilibrium
R
Solution :
N = mg cos + m2 R sin2
Fnet = 0 = mg sin m2 R sin cos
2 R
= mg sin 1
g
N
R m2(Rsin)
2 R
= mg sin 1 cos
mg g
Case I : 2R > g
2 R
Fnet
= mg sin 1 cos
g
1 g
= 0, = cos 2 are potential points.
R
1 g
At = 0 At = cos 2
R
g
= 0 is unstable eqbm. = cos 1 2 is stable.
R
1 2 Rcos
Fnet = mgsin = 0 is the equibrium position and it is stable
g
Similarly
= 0 will be stable
g
and = cos 1 2 will be unstable
R
1 .5 M OM EN T, COU PL E A N D T ORQU E
The moment of a force F about a point is its turning effect F
about the point.
It is quantified as the product of the force and the perpendicular
distance from the point to the line of action of force
d
O
In Figure the moment of F about point O is Moment of a force
moment = Fd
A couple is a pair of equal and parallel but opposite forces as shown in Figure :
F
F
A couple
The moment of a couple about any point in its plane is the product of one force and the perpendicular
distance between them :
Moment of couple = Fp
Example of a couple include turning on/off a tap, or winding a clock.
The SI units for a moment or a couple are Newton metres, Nm.
In engineering the moment of a force or couple is know as torque. A spanner tighterning a nut is
said to exert a torque on the nut, similarly a belt turning a pulley exerts a torque on the pulley.
1 .5 .1 Wo r k d o n e b y a c o n s t a n t t o rq u e
Let a force F turn a light rod OA with length r through an angle of to positioin OB, as shown
in Figure.
F
B
F
s
r
A
r
O
1 .5 .2 Po w e r t ra n s m i t t e d b y a c o n s t a n t t o rq u e
Powr is rate of doing work. If a rod rotates at n revolutions per second, then in one second the angle
turned through is
= 2n
radians, and the work done per second will be,
work done per second = power = TQ2n
as angular speed is
= 2n
WORK, ENERGY & POWER
QUIZRR 33
then
power = 2nTQ
power = TQ
The units of power are Watts, W, with n in rev/s, in rad/s and TQ in Nm.
Ex a m p l e 2 1
A spanner that is used to tighten a nut is 300mm long. The force exerted on the end of a
spanner is 100N.
(a) What is the torque exerted on the nut ?
(b) What is the work done when the nut turns through 30 ?
Solution :
(a) Calculate the torque
TQ = Fr
= 100 (300 10 3)
= 30 Nm
(b) Calculate the work done
Work done = TQ
30
6
= 15.7J
Ex a m p l e 2 2
An electric motor is rated at 400 W. If its efficiency is 80 & percent;, find the maximum torque
which it can exert when running at 2850 rev/min.
Solution
Calculate the speed in rev/s
Power = 2nTQ
power
n
2TQ
2850
n 47.5 rev/s
60
Calculate the power as the motor is 80 percent efficient
80
power = 400 320W
100
power = 2nTQ
power
TQ
2n
320
= 1.07 Nm
247.5
1 .5 .3 Wo r k d o n e b y a v a r ib a l e t o rq u e
In practice the torque is often variable. In this case the work done cannot be calculated as done
earlier, but must be found in a similar way to that used for a varibale force (see earlier.)
torque
TQ
0
d
angular displacement
The total work done for the angular displacement is the area under the torque/displacement graph.
For variable torque
Work done = area under torque/angular displacement graph
As with variable forces, in general you must use special integration technique to obtain the area
under a curve.
Ex a m p l e 2 3
A machine requires a variable torque, Find :
(a) The work done per revolution
(b) The average torque over one
revolution
(c) The power required if the 800
machine operates at
30 rev/min 600
400
torque (Nm)
200
/2 3/2 2
0
(radian)
Solution :
work done = area under torque/ graph
= area of trangle ABC + of rectangle ADEO
1
600 2200
2
= 2200J
for one revolution
(b) Average torque is the average height of figure OABCDE = area/2
2200
Average torque = 350Nm
2
power = 2n average torque
30
2 350
60
= 1100W
1 .6 M OT I ON I N A V ERT I CA L CI RCL E
Suppose a particle of mass m is attached to an inextensible light string of length R. The particle is
moving in a vertical circle of radius R about a fixed point O. It is imparted a velocity u in horizontal
direction at lowest point A. Let v be is velocity at point B of the circle as shown in figure. Here
h = R (1 cos ) ... (i)
From conservation of mechanical energy.
1
m (u2 v2 ) mgh
2 O
T v
or v2 = u2 2gh ... (ii) B
R mg cos
The necessary centripetal force is provided by the resultant of tension h
T and mg cos mg sin
A u
mv2
T mg cos ... (iii)
R
Now, following three conditions arise depending on the value of u.
1 .6 .1 Co n d i t i o n o f L o o p i n g t h e L o o p (u 5 gR )
The particle will complete the circle if the string does not slack even at the highest point ( = ).
Thus, tension in the string should be greater than or equal to zero (T > 0) at = . In critical case
substituting T = 0 and = in Eq. (iii), we get
2
mvmin
mg =
R
or 2
vmin = gR
or 2
umin = gR + 2g (2R) = 5 gR
2 5 gR
or umin = P
vmin = gR T=0
Thus, if u 5gR , the particle will complete the circle. At u
1 .6 .2 Co n d i t i o n o f L e a v i n g t h e Ci rc l e ( 2 gR < u < 5 gR )
If u < 5 gR , the tension in the string will become zero before reaching the highest point. From Eq.
(iii), tension in the string becomes zero (T = 0).
v2
where, cos
Rg
2 gh u2
or cos
Rg
2 gh u2 h
1
Rg R
u2 Rg
or h h1 (say) ... (iv)
3g
or we can say that at height h1 tension in the string becomes zero. Further, if u < 5 gR , velocity of
the particle becomes zero when
0 = u2 2gh
u2
or h h2 (say)
2g
u2 Rg u2
or
3g 2g
or u> 2Rg
V
Therefore, if 2 gR u 5 gR , the particle leaves the circle.
From Eq. (iv), we can see that h > R if u2 > 2gR. Thus, th particle, will
leave the circle when h > R or 90 < < 180 . This situation is shown in
the figure.
Note : That after leaving the circle, the particle will follow a parabolic path.
1 .6 .3 Co n d i t i o n o f Os c i ll a t io n (0 < u < 2 gR )
The particle will oscillate if velocity of the particle becomes zero but tension in the string is not zero.
or v = 0, but T 0. This is possible when
h 2 < h1
u2 u2 Rg
or or 3u2 < 2u2 + 2Rg
2g 3g
or u2 < 2Rg or u 2 Rg
Moreover, if h1 = h2, u = 2Rg and tension and velocity both becomes zero simultaneously.
Further, from Eq. (iv), we can see that h < R if . Thus, for
0 < u < 2Rg , particle oscillates in lower half of the circle
(0 < < 90 ). This situation is shown in the figure.
Note : The above three conditions have been derived for a particle moving in a vertical circle
attached to a string. The same conditions apply if a particle moves inside a smooth spherical shell of
radius R. The only difference is that the tension is replaced by the normal reaction N.
v = gR, N = 0
u = 5gR, N = 6 mg
1 .6 .4 M o t i o n o f a b a l l o v e r a s m o o t h s o l i d s p h e r e
Suppose a small ball of mass m is given a velocity v over the top of a smooth sphere of radius R.
The equation of motion for the ball at the topmost point will be. u = 0
v h
2
mv
mg N N=0
R
mg v
mv2 R
or N mg
R
(a) (b)
From this equation we see that value of N decreases as v increases. Minimum value of N can be zero
Hence,
2
mvmax
0 mg
R
or vmax Rg
From here we can conclude that ball will lose contact with the sphere right from the beginning
velocity of ball at topmost point v > Rg . If v < Rg it will definitely lose contact but after moving
certain distance over the sphere. Now let us find the angle where the ball loses contact with the sphere
velocity at topmost point is just zero. Fig. (b)
h = R (1 cos ) ... (i)
2
v = 2gh ... (ii)
mv2
mg cos (as N = 0) ... (iii)
R
Solving Eqs. (i), (ii) and (iii), we get
2
cos1 48.2
3
2
Thus the ball can move on the sphere maximum upto cos 1 .
3
Ex a m p l e 2 4
A heavy particle hainging from a fixed point by a light inextensible string of length l is
projected horizontally with speed gl . Find the speed of the particle and the inclination of
the string to the vertical at the instant of the motion when the tension in the string is equal
to the weight of the particle.
Solution :
Let T = mg at angle as shown in figure. T
B
h = l (1 cos ) ... (i) mg cos
Applying conservation of mechanical energy between points A h mg sin
and B, we get A u = gl
1
m(u2 v2 ) = mgh
2
Here, u2 = gl ... (ii)
and v = speed of particle in position B
v2 = u2 2gh ... (iii)
mv2
Further, T mg cos =
l
mv2
or mg mg cos = (T = mg)
l
or v2 = gl (1 cos ) ... (iv)
2 2
Substituting values of v , u and h from Eqs. (iv), (ii) and (i) in Eq. (iii), we get
2
gl(1 cos ) = gl 2gl (1 cos ) or cos =
3
1 2
or = cos Ans.
3
2 gl
Substituting cos = in Eq. (iv), we get v = Ans.
3 3
Ex a m p l e 2 5
A heavy particle is suspended by a string of length l from a fixed point O. The particle is
given a horizontal velocity v0. The string slacks at some angle and the particle proceeds on
a parabola. Find the value v0 if the particle passes through the point of suspension.
Solution : Let the string slacks at point B as shown in figure. v
h = l + l sin = l (1 + sin ) ... (i) 90
B y
Applying conservation of mechanical energy between points A
and B, we get
x
O
v2 = v02 2 gh ... (ii) h
l
mv2
and mg sin =
l
or v2 = gl sin ... (iii)
After B path of the particle is a projectile and it passes throug O. Co-ordinate of point O with origin
at B and x and y axes as shown in figure are :
(x, y) = (l cos , l sin )
Angle of projection is = 90 and the velocity of projection is u = v. So, substituting the above
co-ordinates in equation of projectile, i.e.,
gx2
y = x tan
2u2 cos2
WORK, ENERGY & POWER
QUIZRR 41
g l cos
2
gl 2 cos2 l cos2
l sin =
2 gl sin sin 2 sin
1
or 3 sin2 = 1 or sin =
3
= gl (2 + 3 sin ) = gl 2 3
or v0 = gl(2 3) Ans.
Note : In a particle of mass m is connected to a light rod and whirled in a vertical circle of radius
R, then to complete the circle, the minimum velocity of the particle at the bottommost point is not
5 gR . Because in this case, velocity of the particle at the topmost point can be zero also. Using
conservation of mechanical energy between points A and B as shown in fig. we get
1
2
m u2 v2 mgh v=0 B
1 h = 2R R
mu2 mg (2R) O
or R
2
Same is the case when a particle is compelled to move inside a smooth vertical tube as shown
Ex a m p l e 2 6
A body starts sliding from height h down a smooth inclined plane groove passing into half
vertical circle of radius h/2. Find the maximum height to which the body rises.
Solution :
Let the body acquire a velocity Vb on reaching point A
Then, from conservation of energy O v
1 P
mvb2 = mgh h
2
h/2
vb 2 gh A
= 4 g h / 2
Particle leaves the track at certain angle above horizontal, thus the velocity
at this point is given by
vP = gl sin
h
vP = g sin
2
1 1
mv2P mg l(1 sin ) = mv2b
2 2
h h h
4g = g sin 2 g (1 sin )
2 2 2
3
2 = sin 1
2
= sin 1 2 / 3
gh
and vP =
3
Now, for calculating maximum height achieved we take the particle motion as projectile motion after
point P.
VP2 sin2
Hence, height above P (hP) =
2g
Here = 90
v2 cos2 5v2
hP =
2g 18 g
h
so, max height (H) = hP + (1 sin )
2
h 2 5 gh 25
1 h
2 3 18 g 3 27
=
Ex a m p l e 2 7
A simple pendulum of length l is released from horizontal position. A nail P is fixed to the
vertical wall. The bob just completes the vertical circle about P. Then find the value of r.
Solution : A
The minimum velocity required at B to complete the circle O
30
r
Vmin = 5 g (l r )
P
C
Now, use conservation of energy between point A and B
(l r)
1 2
mvb = mg (l r) + mg (r sin 30 )
2
B
5g (l r) = 2g (l r) + gr
3 gl = 4 gr
3l
r =
4
Ex a m p l e 2 8
In the previous question, if nail P can be fixed any where in the vertical wall, then find the
locus of point P.
+x O
Solution : A l y
(0, 0)
y tan = ( y/ x)
= 3g [ l x2 + y2)
C
P (x,y)
(l x2 + y2)
2
1 2 2
mg[y] = m 3 g l x y
2
2 gy = 3g l x2 y2
2y = 3l 3 x2 y2
3 x2 y2 = 3 l 2 y
9 x2 y2 3l 2 y
2
Required locus
Ex a m p l e 2 9
A particle is suspended vertically from a point O by an inextensible massless string of length
L. A vertical line AB is at a distance of L/8 from O as shown. The particle is given a horizontal
velocity u. At some point, the particle crosses AB and its velocity is horizontal. Find u.
Solution :
Consider the situation when the string is making an angle then, 90 C
L mg
mv2
mg sin = {Balancing Forces at
L
point C} A
O P
v = gLsin ...... (1) L/
8
L
Range = 2 L cos
8
{as the paticle is at its maximum height when it crosses line AB}
v2 sin 2 90 L
= 2 L cos ..... (3)
2g 8
Ex a m p l e 3 0
In the arrangement shown, the string is ideal. The length of the
string = 16 m. Find the velocity of each block when block B strikes
the ground. B
m
m 6m
15
A 5m
Solution : m
From constraint relation, we get,
VB = VA cos ....... (1)
At the instant of striking
K + u = 0 ....... (2)
1 1
mvb2 mvA
2
= mg (5)
2 2
m
10
2
VA cos2 VA2 = 2 5g 6m
5g 2 sin = (3/5)
VA = 2
1 cos cos = 4/5
5 g 2 25
= = 7.8 m/s
41
Ex a m p l e 3 1
A sphere is moving with a constant acceleration a 0 along a horizontal surface. A small
particle of mass m starts sliding from the top. Find the angle with vertical at which the
contact is lost.
Solution : N
Making F.B.D. and resolving the forces
mv2 ma0
(ma0 sin + mg cos ) N =
R mg
a0
At break off point (N = 0)
mv2
ma0 sin + mg cos =
R
V2 = R (a0 sin + g cos ) ..... (1)
K + u = Wext [Work energy theorem]
1 2
2 mv mg R 1 cos = ma0 R sin ..... (2)
1
mg R 1 cos ma0 R sin = mv2
2
2g = 3g cos a0 sin
(B) When the sphere is fixed
mv2
mg cos N = N
R
mg
At break off point (N = O)
mv2
mg cos =
R
V2 = Rg cos
(K = u) Mechanical enery conservation
1
mv2 = mg R 1 cos
2
Rg cos = 2 gR 1 cos
3cos = 2
1 2
= cos ( /3)
1 2
Note : When a particle rolls over a fixed sphere, the contact is lost at = cos ( /3)
[when initial velocity = 0]
Ex a m p l e 3 2
The sleeve of mass M can slide along a smooth vertical rod as shown. Find the velocity of
the block and sleeve, when the sleeve makes an angle with the horizontal.
d
M
(d tan)
(d sec)
T (d sec d)
m
A B
V1 V2
h
Solution :
height through which the block m rises = d (sec 1)
(VB = VA sin ) constraint relationship ...... (1)
1 1
mVB2 M VA2 = Mg hA ( mg hB ) ...... (2)
2 2
1
m VA2 sin 2 M VA2 = 2mg d sec 1 Mg d tan
2
VA2 m sin2 M = 2 mgd sec 1 Mg d tan
Ex a m p l e 3 3
In the arrangement shown, find the velocity of block C When it strikes the ground.
3m 3m
2m
T T
>
4m 4m
>
V2 C V2
m V1 m B
Solution : A
From the constraint relationÊs, we know work done by tension is zero
Hence, 2 T (cos ) v1 + 2T (v2) = 0 v1 cos = v2
when block C hits the ground
1
sin = (3/5) = tan (3/4)
From conservation of energy
1 1
mv12 mv22 2 = 2mg (4) 2mg (2)
2 2
1
mv12 mv12 cos2 = 4mg
2
v12 1 2cos2 = 8g
8g 80
5.923 m/s
V1 = 1 2 cos 2 16
1 2
25
Ex a m p l e 3 4
In the given arrangement the system is released from rest. Find the velocity of each block
when A strikes the fixed vertical wall.
A
1m 1m
M
B m
Mv12 m v12 cos2 = 2Mg 2mg 5 1
v1 =
2 Mg mg 5 1
2
M m cos
Ex a m p l e 3 5
Two blocks A, B are connected by an ideal spring of stifness k and placed on a rough horizontal
surface as shown. A constant force F is applied on B. Find the minimum force, so that the
block A just starts sliding. A B
m1 m2 F
ø1 ø2
Solution :
Concept : The force applied should be such that it starts the slipping in the block A even until
maximum elongation is produced, otherwise, it will not be able to start the motion of A.
To calculate the friction consider the extreme position, when maximum enlongation is produced and
(v = 0)
To start slipping
K lmax = ø1 m1 g
1 m1
lmax = g
k
1
ø2m2g (lmax) + F (lmax) Al2max = 0
2
1
F = ø2m2g + Klmax
2
1 m1 g
Hence, F = 2 m2 g
2
Ex a m p l e 3 6
Two identical block A and B are connected by an ideal spring A m
of stiffness K, initially compressed by means of an attached
string as shown, and placed on horizontal surface. Then the
string is burnt through
K
(a) Find the minimum initial compression, so that the
lower block just loses contact with the surface.
5mg B m
(b) I f l 0 = , find the minimum and maximum normal
2k
force in B.
Solution :
For B, N + Kl = mg
(a) To just lose contact N = 0 Kl = mg
mg
l =
K
Now, to find the minimum initial compression, consider the extreme position (v = 0)
Apply energy conservation,
K + u = 0 [ Wext = 0]
0 + u = 0 [ K = 0] m
1 2 1 2
[mg (l + l) – mg (l l0] + k l kl0 0 l
2 2
l0
1
2
2
2
mg (l + l0) + k l l0 0 m
2 mg 2mg natural
l0 l l0 l
k k length
3mg B m
l0 =
k
5mg 7mg
Nmax = mg + Kl0 = mg + k
2k 2
mg mg
Nmin = mg
2 2
Ex a m p l e 3 7
A vertical circular tube of mass M is placed on a horizontal surface.
Two small identical balls, each of mass m which just fit inside the m
tube and released from the top of smooth surface inside the tube.
The tube loses just contact, when the radius vector makes 60 ,
M
(a) find = ?
m
(b) find the maximum value of M so that the tube just loses
contact with surface.
Solution : N0
First the small ball moves along the inner surface but, breaks from N N
inner surface at cos 1 (2/3) and moves along the outer surface. The net
normal force exerted by the ball accelerates the tube to just lose contact
is upward direction.
From FBD of tube
N0 + 2N cos = Mg
Now, N0 = 0, at time of losing contact Mg
2 N cos 60 = Mg ..... (1)
From FBD of the ball,
mv2
N + mg cos = r .... (2)
v2 = 2gR (1 cos )
m 2 gR 1 cos
N = mg cos
R
= mg [2 3 cos ]
3 1
From (1), 2mg 2 Mg
2 2
1 M
Ans.
2 m
m 2 gR 1 cos
(b) N mg cos mg 2 3cos
R
Fnet is upward direction = 2mg [2 3 cos] cos
(Fnet) Mg [to loose contact]
[2 mg [2 cos 3 cos 2]max Mg
2 mg
solving this; Mg
3
2m
M
3
Ex a m p l e 3 8
In the arrangement shown, the block is conected to point O O
5mg
using an ideal spring K . After the thread PA has 5mg
l0 l0 K=
lo
been bought through the system starts moving. Find the
A
velocity of each block, when block A just loses contact.
P
m
l = l0 (sec 1)
l0 B
Solution : m
m
A T
b = (l0 tan )
B
m
N
Let be the angular position when the block leaves the centred.
Kl
N + K l cos = mg
mg
4 5 l0
cos l l0 1
5 4 4
3
b l0
4
Now, using energy conservation
1 1
2 mv2 kl 2 mgb
2 2
19 gl0
v
32
Ex a m p l e 3 9
In the arrangement shown the string is elastic l a and has a natural
length l 0. The plank b now slowly shifted till the block starts slipping
over the plank at an angle 30Ĉ. Find the work done by frictional force on l0
the upper block, till it starts slipping over the surface.
Solution : ø m
Balancing forces in horizontal and vertical direction m
smooth
Making F.B.D. of the system for the condition when block starts slipping
N + kl cos = mg
kl sin = øN = ø (mg ă kl cos )
mg
k
l sin cos ..... (1)
øN 1 mg l0 (sec 1)
Wf =
mg 2 (sin cos )
2
1
mg l0
3 0
1 mg l 2 3
= 2 1
3
3 1 3
2 2
Ex a m p l e 4 0
1
A uniform chain of mass m and length l is placed on a rough horizontal table with length
3
over hanging. The chain just starts slipping at that position.
Find the time and velocity when the chain completely slips off the table. Also find the
work done by friction. 2l
3 2l ă x
3
ø
l l +x
3 3
{at t = 0} {at t = t}
Solution :
Consider an instant when the chain has fallen through x distance, let be mass per unit length
of the chain
2l {fs is the
Wfr N dx From initial fs (max) g frictional force
3
2l / 3
2l 2l l at t = 0}
= ă 3 x g dx g g .
0 3 3
1
2
{Here Wfr is total work done by friction till {Since it just starts slipping hence frictional force
the entire chain slips off the table} at that moment is sufficiently balancing the weight
of the over hanging chain}
2l 2 l / 3 2l / 3
x2 4 l 4 l2
2
= g x = g
3 0 2 0 9 18
2 1 mgl
= g 4 l
18 9
m l
Apply energy conservation
Wfr Wgravity k
mgl
u k
9
mgl
k u
9
mgl mgl l l l
= ui uf g mg
9 9 3 6 2
1 mgl 2 gl 2 gl
mv2 v2 or v = Ans.
2 3 3 3
WORK, ENERGY & POWER
54 QUIZRR
Ex a m p l e 4 1
Find the maximum elongation produced in the spring and acceleration of each block.
F = 15N = 30Ĉ
m1 = 2kg
K = 1000 N/m
m2 = 4kg
m3 1 kg
{Making F.B.D. of
Solution : Balancing Forces for block m3 block m2}
m3 g ă T = m3a2 ....(1)
Balancing Forces for block m2
Kx
N1
[T + m2 g sin ] ă kx = m2a2 ....(2)
Using (1) and (2) T
m3g + m2 g sin ă kx = (m2 + m3)a2
m2 g
Balancing Forces for block m1
[kx + m1g sin ) – f1 = m1a1 ....(3)
a1,2 = a1 ă a2
0 xmax xmax
17 7
v1,2 dv1,2
2
dx kx dx
10
{At time of maximum elongation}
0 0 0
7 x max
2
17
0 xmax k
2 10 2
1 17 17 10
xmax 10 m
k 7 7 1000
17
= cm 2.42cm
7
Ex a m p l e 4 2
A smooth track in the form of a quarter-circle of radius 6 m lies in the vertical plane. A ring
of weight 4 N moves from P1 and P2 under the action of forces F1, F2 and F3. Force F1 is
always towards P2 and is always 20 N in magnitude; force F 2 always acts horizontally and
is always 30 N in magnitude; force F 3 always acts tangentially to the track and is of
magnitude (15 ă 10s) N where s is in meters. If the particle has speed 4 m/s at P1, what will
its speed be at P2 ?
Solution : The work done by F 1 is
6m
P2 O P2
W1 P
1
F1 cos ds 2
R=6m
From figure; s R 2
2 6m
or ds = (6 m) d (ă 2) = ă 12 d F1 = 20 N
and F1 = 20 F3 = 15 ă 10s N
F2 = 30 N
0
Hence, W1 240 / 4 cos d P1 s
W=4N
= 240 sin 120 2 J
4
The work done by F3 is
6 / 2
W3 F3 ds 0 15 10 s ds
3
= 15s 5s 302.8 J
2
0
To calculate the work done by F2 and by W, it is convenient to take the projection of the path in
the direction of the force, instead of vice versa. Thus,
W2 F2 OP 2 30(6) 180 J
W W P1O ( 4)(6) 24 J
Ex a m p l e 4 3
A bock of mass m is pushed against a spring of spring constant k fixed at one end to a wall.
The block can slide on a frictionless table. The natural length of the springis L0 and it is
compressed to half its natural length when the block is released. Find the velocity of the
block as a function of its distance x from the wall.
V
x
Solution :
When the block is released, the spring pushes it towards right. The velocity of the block increases
till the spring acquires its natural length. Thereafter, the block loses contact with the spring and moves
with constant velocity.
Initially, the compression of the spring is L0/2. When the distance of the block from the wall becomes
x, where x < L0, the compression is (L0 ă x). Using the principle of conservation of energy.
2
1 L0 1 1
k L 0 x mv2
2
k
2 2 2 2
k L20 2
Solving this, v L0 x Ans.
m 4
k L0
When the spring acquires its natural, x = L0 velocity becomes v m 2 . Thereafter, the block
continues with this velocity.
Ex a m p l e 4 4
A particle slides along a track with elevated ends and a flat central part as shown in Fig.
The flat portion BC has a length l = 3.0 m. The curved portions of the track are frictionless.
For the flat part the coefficient of kinetic friction is øk = 0.20, the particle is released at
point A which is at height h = 1.5 m above the flat part of the track. Where does the particle
finally comes to rest?
Solution :
A D
As initial mechanical energy of the particle is mgh and final is
zero, so loss in mechanical energy = mgh. This mechanical energy
is lost in doing work against friction in the flat part, h
So, loss in mechanical energy = work done against friction
B E C
h 1.5
or mgh = ømgs i.e. s 0.2 7.5 m
After starting from B the particle will reach C and then will rise up till the remaining KE at C is
converted into potential energy. It will then again descend and at C will have the same value as it had
when ascending, but now it will move from C to B. The same will be repeated and finally the particle
will come to rest at E such that
BC + CB + BE = 7.5
or 3 + 3 + BE = 7.5
i.e., BE = 1.5
So, the particle comes to rest at the centre of the flat part. Ans.
Ex a m p l e 4 5
A 0.5 kg block slides from the point A on a horizontal track with an initial speed 3 m/s
towards a weightless horizontal spring of length 1 m and force constant 2 N/m. The part
AB of the track is frictionless and the part BC has the coefficient of static and kinetic
friction as 0.22 and 0.20 respectively. If the distance AB and BC are 2 m and 2.14 m
respectively, find the total distance through which the block moves before it comes to rest
completely. [g = 10 m/s2]
Solution :
As the track AB is frictionless, the block moves this distance without loss in its initial
1 1
KE mv2 0.5 32 2.25 J .
2 2
In the path BD as friction is present, so work done against friction
= øk mgs = 0.2 ï 0.5 ï 10 ï 2.14 = 2.14 J
So at D the K.E. of the block is = 2.25 ă 2.14 = 0.11 J.
Now, if the spring is compressed by x
1
0.11 k x2 k mgx
2
A D C
B
1
i.e. 0.11 2 x2 0.2 0.5 10 x
2
or x2 + x ă 0.11 = 0
which on solving gives positive value of x = 0.1 m
After moving the distance x = 0.1 m the block comes to rest. Now the compressed spring exerts a
force :
F = kx = 2 ï 0.1 = 0.2 N
on the block while limiting frictional force between block and track is fL = øsmg = 0.22 ï 0.5 ï 10
= 1.1 N.
Since, F < fL. The block will not move back. So, the total distance moved by the block
= AB + BD + 0.1
= 2 + 2.14 + 0.1
= 4.24 m Ans.
WORK, ENERGY & POWER
58 QUIZRR
Ex a m p l e 4 6
The block of mass M shown in Fig. initially has a velocity v0 to the right and its position
is such that the spring exerts no force on it, i.e., the spring is neither stretched nor
compressed. The block moves to the right a distance l before stopping in the dotted position
shown. The spring constant is k and the coefficient of kinetic friction between block and
the table is ø. Find :
(a) what is the work done on it by the spring friction force?
(b) what is the work done on it by the spring force?
(c) are there other forces acting on the block, and if so, what work do they do?
(d) what is the total work done on the block?
(e) use the work-energy theorem to find the value of l in terms of M, v0, ø, g and k.
v0
l
k
M
ø
Solution :
(a) Work done by friction = ă ø M gl
1 2
(b) Work done by the spring force = kl
2
(c) Gravitational force and normal reaction of the table do not work as they act in a direction
perpendicular to displacement.
1 2
(d) Total work done on the block = M gl kl
2
1 1
0 M v02 M gl kl 2
2 2
2 M gl M v02
or l2 0
k k
1 2
l M 2 g 2 M k v02 Mg Ans.
k
Ex a m p l e 4 7
Two blocks A and B are connected to each other by a string and a spring. The spring
passes over a frictionless pulley as shown in Fig. Block B slides over the horizontal top
surface of a stationary block C and the block A slides along the B
vertical side of C, both with the same uniform speed. The
coefficient of friction between the surfaces of the blocks is 0.2. C
The force constant of the spring is 1960 N m ă1. If the mass of A
block A is 2 kg, calculate the mass of block B and the energy
stored in the spring. Take g = 9.8 m/s2
Solution :
Let m be the mass of B. From its free-body diagram
T ă øN = m ï 0 = 0
where T = tension of the string and N = mg
T = ømg
T øN'
øN B T N'
T´ 2g
(a) (b) (c)
From the free-body diagram of the spring
T ă T´ = 0
where T´ is the force exerted by A on the spring
or T = T´ = ømg
From the free body diagram of A
2g ă (T´ + øN´) = 2 ï 0 = 0
where N´ is the normal reaction of the vertical wall of C on A and N´
= 2 ï 0 (as there is no horizontal acceleration of A)
2g 2
2g = T´= ømg or m 10 kg Ans.
g 0.2
Tensile force on the spring = T or T´ = ømg = 0.2 ï 10 ï 9.8 = 19.6 N
Now, in spring,
tensile force = force constant ï extension
1
19.6 = 1960x or x m
100
1 2
or U (energy of a spring) = kx
2
2
1 1
= 1960 .098 J Ans.
2 100
Ex a m p l e 4 8
A particle of mass m is moving in a circular path of constant radius r such that its centripetal
acceleration ac is varying with time t as ac = k 2rt2 where k is a constant. What is the power
delivered to the particle by the forces acting on it?
Solution :
As ac = (v2/r), so (v2/r) = k2rt2
1 1
Kinetic energy K mv2 = mk2 r 2 t2
2 2
1
Now, from work-energy theorem W = K = mk2 r 2 t2 0 [as at t = 0, K = 0]
2
dW d 1
P mk2 r 2 t2 = mk2 r 2t
dt 2
So, Ans.
dt
Now, as F F c Ft
dW
So, P F. v F c Ft . v
dt
Now, in circular motion F c is perpendicular to v while F t parallel, so
P = Ftv as F c . v 0
P = mk2r2t Ans.
Ex a m p l e 4 9
An automobile of mass m accelerates, starting from rest. The engine supplies constant
power P, show that
(a) The velocity is given as a function of time by v = (2Pt/m)1/2
(b) The position is given as function of time by s = (8P/9m)1/2t3/2
Solution : Power P = constant
Work done upto time t is W = Pt
From work energy theorem
W = KE
1
or Pt mv2
2
1/ 2
2Pt
v ...(i)
m
1/2 1/2
ds 2Pt s t 2Pt
dt m
i.e. 0 ds 0 m
dt
1/2 1/2
2P 2 3/2 8P
s t or s t3 / 2
m 3 9m
Ex a m p l e 5 0
A spherical ball of mass m is kept at the highest point in
the space between two fixed, concentric spheres A and B
(see figure). The smaller sphere A has a radius R and the
space betwen the two spheres has a width d. The ball has Sphere B
a diameter very slightly less than d. All surfaces are
frictionless. The ball is given a gentle push (towards the O
d
right in the figure). The angle made by the radius vector R
of the ball with the upward vertical is denoted by Sphere A
(shown in the figure).
(a) Express the total normal reaction force exerted
by the spheres on the ball as a function of angle .
(b) Let NA and N B denote the magnitudes of the normal reaction forces on the ball
exerted by the spheres A and B, respectively. Sketch the variations of NA and N B
as functions of cos in the range 0 by drawing two separate graphs, taking
cos on the horizontal axes.
[JEE 2002]
Solution :
h
d
(a) h R 1 cos
v
2 mg
d
v2 2 gh 2 R 1 cos g ...(i)
2
Let N be the total normal reaction (away from centre) at angle . Then
mv2
mg cos ă N =
d
R
2
2
or cos1
3
After this it makes contact with outer sphere and normal reaction starts acting towards the centre.
2
Thus, for cos 1
3
NB = 0
and NA = mg (3 cos ă 2) Ans.
2
and for cos 1
3
NA = 0
and NB = mg (2 ă 3 cos )
The corresponding graphs are as follows :
NA NB
mg 5mg
2mg
cos cos
ă1 2/3 +1 ă1 2/3 +1
Work
Is done on an for constant force
object when in the direction
Is NOT done of motion
Force when there W = Fx
is no motion for constant force
acts on it in the
with a component in
direction
direction of motion
of motion F
W = F cos q x
F
d
or has a or when the force for a variable force
component is perpendicular in the direction
in the direction to the motion of motion
of motion W = F dx
F d
for a variable force
in a variable direction
d
F
W = F cos q dx
FRICTION
3
3 .1 . CON T A CT FORCE
When two bodies are kept in contact, electromagnetic forces act between the charged particles at the
surfaces of the bodies. As a result, each body exerts a contact force on the other.
3 .2 . FRICT I ON
The perpendicular component of the contact force is called the normal contact force and the parallel
component is called friction.
Friction is not a fundamental force, as it is derived from electromagnetic force between charged
particles, including electrons, protons, atoms and molecules and so cannot be calculated from first
principles, but instead must be found empirically. When contacting surfaces move relative to each other,
the friction between the two surfaces converts kinetic energy into thermal energy or heat.
Contrary to earlier explanation, kinetic friction is now understood not to be caused by surface
roughness but by chemical bonding between the surfaces.
Surface roughness and contact area, however, do affect kinetic friction for micro-and nano-scale
objects where surface area forces dominate inertial forces.
3 .3 T Y PES OF FRICT I ON
Fj = Force of friction
F
W
Ff = øN
N
1. Kinetic friction
2. Static friction
Co e f f i c i e n t o f f r ic t i o n
The coefficient of friction (COF), also known as a frictional coefficient or friction coefficient, symbolized
by the Greek letter ø, is a dimensionless scalar value which describes the ratio of the force of friction
between two bodies and the force pressing them together.
The coefficient of friction depends on the materials used; for example, ice on steel has a low coefficient
of friction, while rubber on pavement has a high coefficient of friction. Coefficients of friction range from
near zero to greater than oneăunder good conditions, a tire on concrete may have a coefficient of friction
of 1.7.
FRICTION
2 QUIZRR
3 .4 L A WS OF FRICT I ON
1. If the bodies slip over each other, the force of friction is given by
k kN
Where
k is the force of friction
N is the normal contact force
øk is the coefficient of friction between the surfaces
2. The direction of kinetic friction on a body is opposite to the velocity of this body with respect
to the body applying the force of friction.
3. If the bodies do not slip over each other, the force of friction is given by
s = øsN
(Causing Movement)
e
orc
F
et
N
f = Friction force Fapp = Applied force
Applied Force (Fapp ) = ø.N
Friction Force
Fr
ic
tio
n
Fo
rc
e
Fs =øsN fd =ødN
where
Fs = static force of friction
øs = coefficient of static friction between the bodies
N = normal force between them
The direction and magnitude of static friction are such that the condition of no slipping between
the bodies is ensured.
If the applied force is small, the friction force will be equal to it. As the applied force is
increased friction force also increases to a maximum limit and if additional force is applied the
bodies start slipping.
4. The frictional force fk or fs does not depend on the areas of contact. It depend on normal
contact force only.
3 .5 T H E N ORM A L FORCE
Block on a ramp (top) and corresponding free body diagram of just the block (bottom).
The normal force is defined as the net force compressing two parallel surfaces together, and its
FRICTION
QUIZRR 3
direction is perpendicular to the surfaces. In the simple case of a mass resting on a horizontal surface,
the only component of the normal force is the force due to gravity, where. In this case, the magnitude
of the friction force is the product of the mass of the object, the acceleration due to gravity, and the
coefficient of friction. However, the coefficient of friction is not a function of mass or volume; it depends
on the material. For instance, a large aluminium block has the same coefficient of friction as a small
aluminium block. However, the magnitude of the friction force itself depends on the normal force, and
hence the mass of the block.
If an object is on a level surface and the force tending to cause it to slide is horizontal, the normal
force between the object and the surface is just its weight, which is equal to its mass multiplied by the
acceleration due to earthÊs gravity, g. If the object is on a tilted surface such as an inclined plane, the
normal force is less, because less of the force of gravity is perpendicular to the face of the plane.
Therefore, the normal force, and ultimately the frictional force, is determined using vector analysis,
usually via a free body diagram. Depending on the situation, the calculation of the normal force may
include forces other than gravity.
A block on a ramp
j
i
Ff
Ex a m p l e 1
Find the minimum force which needs to applied to the Y
incline of mass M having a cube of mass m placed on its
rough surface with ø = (2 tan ). So that the block m starts
to slide. m
F
X
O
M
Z
FRICTION
4 QUIZRR
Solution :
Let us first Calculate the acceleration
F
When the blocks are not sliding against each other, a
m M
Pseudo force = m a (ă i) {on block m}
mF
= i 90ă
m M
mgsin
Parallel forces to the plane are mg
mF
mg sin and
M
m a
mF
2 N
(Resultant Force) Fr mg sin 2 ma
m M mgcos
for just sliding Fr = fs (max)
mg
2 mgsin
mF
mg cos
2
(mg sin )2 +
m M
mg cos 2 mg sin 2 m M 2
F Ans.
m2
Ex a m p l e 2
Find :
(a) acceleration of each block.
(b) tension with string T. m 2m
A B
(c) friction forces.
øA = 2/3 øB = 1/3
45Ĉ 45Ĉ
Solution :
Concept : The block starts sliding down the plane if
2 mg sin 45Ĉ > fs (max)B + fs (max)A + mg sin 45Ĉ
OR component weights along the inertia > maximum static friction.
FRICTION
QUIZRR 5
2mg 1
1 3 T
2
2 2mg T T+f
T
3
Frictional force in block B = ø 2mg cos A B
FRICTION
6 QUIZRR
Ex a m p l e 3
F is applied at lower block. Find acceleration of 5kg øs = 0.4
each block and friction acting when øk = 0.3
(i) F = 45 N
10kg F
(ii) F = 75 N
smooth
Solution : Concept : How to solve such questions?
(1) Find the maximum acceleration for which these will be not slipping between the blocks.
This can be obtained by
fs max
amax = for that block which is moved by friction only.
m
(2) Find common acceleration from the given force for which slipping will take place.
(3) If acommon > amax, slipping takes place and the block slip against each other with different
acceleration, otherwise acceleration is same.
5kg øs = 0.4
øk = 0.3 N N
FBD øv
F
10kg
øN
mg mg
N
amax = for smaller block
m
0.4 mg 4 m/s2
=
m
F 45
3m/s
2
(i) acommon =
m M 15
acommon < amax
bodies with same acceleration = 3 m/s2
F 75
5m/s
2
(ii) acommon =
m M 15
acmmon > amax
blocks slip against each other
F ă øN = 10 a
økN = 5a2
kN 5 10
a2 = 0.3 3m/s 2
5 5
F ă 15 = 1.0 a1
60
a1 = 6m/s2
10
FRICTION
QUIZRR 7
45
(II) acommon 3m/s2
15
a1
0.3 5 10 1.5 m/s2
10
F 15
a2 6m/s2
5
(III) On same set up, now solve for
F = 45 N
F = 90 N
5kg øR (b) R
øs = 0.2
10kg F
øk = 0.2
øsR
(a) mg
(c)
R
amax s 4m/s 2
m
F fk F 0.1150
(i) acommon =
m m 15
45 15
= 2m/s2
15
acommon < amax
blocks will move with same acceleration = 2m/s.
FRICTION
8 QUIZRR
F fk 90 15
(ii) acommon 5m/s2
m M 15
fk 0.3 50
a2 = 3m/s2
m 5
F 15 15 90 30
a1 6m/s
2
M 10
(a) (b)
(IV) On same set up, now solve for F FBD
N
(i) F = 25 N F
øs R = 50 ï 0.4 = 20
(ii) F = 45 N
fs (max) surface > fs (max) block
the lower block will not move against the surface
whatsoever is the force on upper block.
Ex a m p l e 4
In the given arrangement, find the time after which the two block collide.
øs = 0.4 øs = 0.2
øk = 0.3 1kg 1kg øk = 0.1
F = 19N
M = 5 kg
smooth
l = 1m
Solution : Check which of the blocks will slip relative to the plank.
For block A
N FBD
øs
mg
øs N = 0.4 ï 10 = 4 N
s mg
4m/s
2
amax =
m
For block B :
N FBD
øsN
amax = 2m/s2 {Calculated similarly as for block A}
mg
FRICTION
QUIZRR 9
F 19 19
Now acommon = m/s 2 2.7 m/s2
M ma mb 5 1 1 7
Block B will slip relative to plank while block A will remain stationary relative to plank
F fk 19 1
Now, a plank 3m/s2
M ma 51
a block 1m/s2 F
Let abP be the acceleration of block with respect to plank
abP ( 1) 3
= ă 2 m/s2
2l 2
1s
time taken =
2
a bP
3 .6 B A N K I N G OF ROA DS
There are three additional aspects of negotiating a curve. First, what if, we want to negotiate curve
at a higher speed. Second, how to make driving safer without attracting limiting conditions as tires may
have been flattened (whose grooves have flattened), or friction may decrease due to any other reasons
like rain or mud. Third, we want to avoid sideway friction to prolong life of the tires. The answer to
these lie in banking of the curved road.
For banking, one side of the road is elevated from horizontal like an incline or wedge. In this case,
the component of normal force in horizontal direction provides the centripetal force as required for the
motion along the curved path. On the other hand, component of normal force in vertical direction
balances the weight of the vehicle. It is clear that magnitude of normal reaction between road and
vehicle is greater than the weight of the vehicle.
Banking of roads
Y
N Ncos
Nsin
X
a1
mg
Fig. : The horizontal component of normal force meets the requirement of centripetal
FRICTION
10 QUIZRR
Here, we compute the relation between the angle of banking (which is equal to the angle of incline)
for a given speed and radius of curvature is :
N cos = m g
and
mv2
N sin =
r
Taking ratio,
This expression represents the speed at which the vehicle does not skid (up or down) along the
banked road for the given angle of inclination (). It means that centripetal force is equal to the
resultant of the system of forces acting on the vehicle. Importantly, there is no friction involved in this
consideration for the circular motion of the vehicle.
Ex a m p l e 5
An aircraft hovers over a city awaiting clearance to land. The aircraft circles with its wings
banked at an angle tan ă 1 (0.2) at a speed of 200 m/s. Find the radius of the loop.
Solution : The aircraft is banked at an angle with horizontal. Since aircraft is executing uniform
circular motion, a net force on the aircraft should act normal to its body. The component of this normal
force in the radial direction meets the requirement of a centripetal force, whereas vertical component
the weight of aircraft. Thus, this situation is analogous to the banking of road.
v2 v2
tan ,r
rg g tan
r = 2002 / (10 ï 0.2) = 20000 m
3 .6 .1 Ro l e o f f r ic t i o n i n b a n k i n g
The moot question is whether banking of road achieves the objects of banking? Can we negotiate the
curve with higher speed than when the road is not banked? In fact, the expression of speed as derived
in earlier section gives the angle of banking for a particular speed. It is the speed for which the
component of normal towards the center of circle matches the requirement of centripetal force.
If speed is less than the specified by the expression, then vehicle
will skid „down‰ (slip or slide) across the incline as there is net y
force along the incline of the bank. This reduces the radius of
curvature i.e. „r‰ is reduced-such that the relation of banking is
N
held true :
a1
v = (r g tan )
x
øs N
mg
Banking of road
FRICTION
QUIZRR 11
In reality, however, the interacting are not smooth. We can see that if friction, acting „up‰ across
the bank, is sufficient to hold the vehicle from sliding down, then vehicle will move along the circular
path without skidding „down‰.
What would happen if the vehicle exceeds the specified for a given angle of banking? Clearly, the
requirement of centripetal force exceeds the component of normal force in the radial direction. As such
the vehicle will have tendency to skid „up‰ across the bank.
Again friction prevents skidding „up‰ of the vehicle across the bank. This time, the friction acts
downward across the bank as shown in the figure.
In the nutshell, we see that banking helps to prevent skidding „up‰ across the bank due to the
requirement of centripetal force. The banking enables component of normal force in the horizontal
direction to provide for the requirement of centripetal force upto a certain limiting (maximum) speed.
Simultaneously, the banking induces a tendency for the vehicle to skid „down‰ across the bank. The
state of friction is summarized here :
On the other hand, friction prevents skidding „up‰ across the bank. This is possible as friction
changes direction opposite to the tendency of skidding either „up‰ or „down‰ across the bank. The state
of friction is summarized here :
1 : v = 0; fs = m g sin , acting up across the bank
2 : v = r g tan ; fs = 0
3 : v < r g tan ; fs > 0, acting up across the bank
4 : v > r g tan ; fs > 0, acting down across the bank
Friction, therefore, changes its direction depending upon whether the vehicle has tendency to skid
„down‰ or „up‰ across the bank. Starting from zero speed, we can characterize friction in following
segments (i) friction is equal to the component of weight along the bank, „m g sin ,‰ when vehicle is
stationary (ii) friction decreases as the speed increases (iii) friction becomes zero as speed equal. „r g
tan (iv) friction changes direction as speed becomes greater than „r g tan ‰ (v) friction increases till
the friction is equal to limiting friction as speed further increases and (vi) friction becomes equal to
kinetic friction when skidding takes place.
3 .6 .2 M a x i m u m s p e e d a l o n g t h e b a n k e d r o a d
In previous section, we discussed various aspects of banking. In this section, we seek to find the
maximum speed with which a banked curve can be negotiated. We have seen that banking, while
preventing upward skidding, creates situation in which the vehicle can skid downward at lower speed.
The design of bank, therefore, needs to consider both these aspects. Actually, roads are banked
with a small angle of inclination only. It is important as greater angle will induce tendency for the
vehicle to overturn. For small inclination of the bank, the tendency of the vehicle to slide down is
ruled out as friction between tyres and road is usually much greater to prevent downward skidding
across the road.
In practice, it is the skidding „up‰ across the road that is the prime concern as threshold speed limit
can be breached easily. The banking supplements the provision of centripetal force, which is otherwise
provided by the friction on a flat road. As such, banking can be seen as a mechanism either (i) to
increase the threshold speed limit or (ii) as a safety mechanism to cover the risk involved due to any
eventually like flattening of tyres or wet roads etc. In fact, it is the latter concern that prevails.
FRICTION
12 QUIZRR
In the following paragraph, we set out to determine the maximum speed with which a banked road
can be negotiated. It is obvious that maximum speed corresponds to limiting friction that acts in the
downward direction as shown in the figure.
fs
The horizontal component of normal force and friction together meet the
requirement of centripetal force.
Force analysis in the vertical direction :
N cos - ø sin = m g N (cos - øs sin ) = m g
Force analysis in the horizontal direction :
mv2 mv2
N sin - øs N cos = , N sin s cos
r r
Taking ratio of two equations, we have :
v2 2
g (sin + øs cos ) (cos - øs sin ) = , v r g (sin + ø cos ) (cos - ø sin ),
r s s
Ex a m p l e 6
A turn of radius 20 m is banked for the vehicle of mass 200 kg going at a speed of 10 m/s. Find
the direction and magnitude of frictional force acting on a vehicle if it moves with a speed
(a) 5 m/s (b) 15 m/s. Assume that friction is sufficient to prevent slipping. g = 10 m/s2.
FRICTION
QUIZRR 13
10 1
2
v2
Therefore, tan
rg (20)(10) 2
mv2
Fx r
and Fy 0, we get
y
N f
mv 2 x
N sin ă f cos = ...(i)
r mg
N cos + f sin = mg ... (ii)
1
Substituting, = tană1 , v = 5 m/s, m = 200 kg and r = 20 m, in the above equations, we get
2
f = 300 5 N (outwards)
(b) In the second case force of friction f will act downwards.
mv2
Using Fx r
and Fy 0, we get N
mv2
N sin ă f cos = ...(iii)
r f
mg
1
Substituting, = tană1 , v = 15 m/s, m = 200 kg and r = 20 m,
2
FRICTION
14 QUIZRR
In both cases friction balances the weight of person while reaction provides the centripetal force for
circular motion, i.e.
f = mg
mv2
and N mr2 v r
r
Ce n t ri f u g a l Fo r c e
As we have already discussed, the NewtonÊs laws are valid only in inertial frames. In non-inertial
frames a pseudo force m a has to be applied on a particle of mass m ( a = acceleration or frame of
reference). After applying the pseudo force one can apply NewtonÊs laws in their usual form. Now
suppose a frame of reference is rotating with constant angular velocity in a circle of radius ÂrÊ. Then
it will become a non-inertial frame of acceleration r2 towards the centre. Now, if we seen an object of
mass ÂmÊ from this frame then obviously a pseudo force of magnitude mr2 will have to be applied to
this object in a direction away from the centre. This pseudo force is called the centrifugal force. After
applying this force we can now apply NewtonÊs laws in their usual form. Following example will illustrate
the concept more clearly.
Ex a m p l e 7
A particle of mass m is placed over a horizontal circular table rotating with an angular
velocity ÂÊ about a vertical axis passing through its centre. The distance of the object from
the axis is r. Find the force of friction f between the particle and the table.
Solution : Let us solve this problem from both frames. The one is a frame fixed on ground and the other
is a frame fixed on table itself.
N
N = normal reaction
r f mg = weight
f = force of friction
mg
mg
FRICTION
QUIZRR 15
Now, md 4 2 mg N z
(m2d) x
g 2 2
1/4 y
mg
2 2 (md)
(Psevdo force)
d
1/4
1 2 g 2 2
time when the coin starts slipping = 2
d
2
max
2
2max
n
2 4
Note : The same analysis can be performed from the ground frame also.
f (m2 d )2 (md )2
f øN
md
(m2 d )2 (md) 2 mg
The result can now be calculated as done in the above part m2 d
FRICTION
16 QUIZRR
Ex a m p l e 9
A simple pendulum is constructed by attaching a bob of mass m to a string of length L fixed
at its upper end. The bob oscillates in a vertical circle. It is found that the speed of the bob
is v when the string makes an angle with the vertical. Find the tension in the string and
the magnitude of net force on the bob at that instant.
Solution : (i) The forces acting on the bob are :
(a) the tension T
(b) the weight mg
mv2
As the bob moves in a circle of radius L with centre at O. A centripetal force of magnitude is
L
required towards O. This force will be provided by the resultant of T and mg cos . Thus,
mv2
or T mg cos
L
T
v2
T m g cos
L Ans.
mg sin
2 mg cos
mv2 v4 mg
(ii) F net mg sin
2
m g sin 2
2 2
Ans.
L L
Ex a m p l e 1 0
A hemispherical bowl of radius R is rotating about its axis of symmetry which is kept vertical.
A small ball kept in the bowl rotates with the bowl without slipping on its surface. If the
surface of the bowl is smooth and the angle made by the radius through the ball with the
vertical is . Find the angular speed at which the bowl is rotating.
Solution : Let be the angular speed of rotation of the bowl. Two forces are acting on the ball.
1. normal reaction N
2. weight mg
The ball is rotating in a circle of radius r (= R sin ) with centre at A at an angular speed . Thus,
N sin = mr2 = mR2 sin ...(i)
and N cos = mg ...(ii)
Dividing Eqs. (i) by (ii), we get
1 2 R
cos g R
N
r A
g
Ans. mg
R cos
FRICTION
QUIZRR 17
Ex a m p l e 1 1
Two blocks of mass m 1 = 10kg and m 2 = 5 kg connected to each other by a massless inextensible
string of length 0.3 m are placed along a diameter of the turn table. The coefficient of friction
between the table and m 1 is 0.5 while there is no friction between m 2 and the table. The table
is rotating with an angular velocity of 10 rad/s about a vertical axis passing through its
centre O. The masses are placed along the diameter of the table on either side of the centre
O such that the mass m 1 is at a distance of 0.124 m from O. The masses are observed to be
at rest with respect to an observer on the turn table. (g = 9.8 m/s2)
(i) Calculate the frictional force on m 1.
(ii) What should be the minimum angular speed of the turn table so that the masses will
slip from this position?
(iii) How should the masses be placed with the string remaining taut so that there is no
frictional force acting on the mass m 1?
Solution : Given m1 = 10 kg, m2 = 5 kg, = 10 rad/s
r = 0.3 m, r1 = 0.124 m
r2 = r ă r1 = 0.176 m
(i) Masses m1 and m2 are at rest with respect to rotating table. Let f be
the friction between mass m1 and table. m1 r r2 m2
1
Free body diagram of m1 and m2 with respect to table (non inertial frame)
r
of reference are shown in figure).
Equilibrium of m2 gives
F1 = m1 r1 2
(Pseudo force)
m2 m1 f
T T
T = m2r22 ...(i)
2 2
Since, m2r2 < m1r1 (m2r2 < m1r1)
and friction on m1 will be inwards (towards centre)
Equilibrium of m1 gives f + T = m1r12 ...(ii)
2 2
From Eqs. (i) and (ii), we get f = ă m2r2
= (m1r1 ă m2r2)2 ...(iii)
= (10 ï 0.124 ă 5 ï 0.176)(100) newton
f = 36 N
Therefore, frictional force on m1 is 36 N (inwards).
(ii) From Eq. (iii)
f = (m1r1 ă m2r2)2
Masses will start slipping when this force is greater than fmax or
(m1r1 ă m2r2)2 > fmax
> øm1g
FRICTION
18 QUIZRR
Minimum value of is
m1 g 0.5 10 9.8
min
m1 r1 m2 r2 10 0.124 5 0.176
min = 11.67 rad/s
(iii) From Eq. (iii), frictional force f = 0 when m1r1 = m2r2
r1 m2 5 1
or and r = r1 + r2 = 0.3 m
r2 m1 10 2
i.e., mass m2 should be placed at 0.2 m and m1 at 0.1 m from the centre O.
Ex a m p l e 1 2
A cyclist moves along the circumference of a horizontal circular plane road. The track has
a radius R and is centred at D. The friction coeff. varies with distance from centre O as
r
0 1 .
R
r is the distance from center O. Find maximum speed of cyclist without skidding.
mv2
f
Solution : s r {Centrifugal force will be balanced only by force of friction}
mv2
At, fs (max) = ømg, mg r v rg
{For speed to be max, fs(max) has to be used}
R
r r
v 0 1 rg v2 0 1 rg
R R
r
Now, v2 is maximum at r, given by O
d v2 rg 1 2r 0
0
dr R
d2 2
r
R
/2 ,
dr 2
v 0
vmax 0 1 R / 2 R
/2 g
0 Rg
=
4
1
= 0 Rg
2
FRICTION
QUIZRR 19
So m e I m p o r t a n t Ca s e s
Questions involving finding the minimum and maximum value for the friction under given conditions.
(1) For minimum , f is up the surface
for maximum , f is down the surface [ slipping tendency is outwards]
Rotating Bowl
N
m2d
mg
(2) For amin friction acts towards right (on 1) For amax friction acts left (on 1)
1 1
a min amin
mg fs mg
Ex a m p l e 1 3
a0
A plank of mass M is placed on a rough inclined plane of inclination .
m
A man runs on the plank, down the incline with constant acceleration.
ø
Find the minimum and maximum acceleration. of the man so that M
plank remains stationary relative to the inclined plane.
Drawing F.B.D. of man
FRICTION
20 QUIZRR
In equilibrium fs1
fs
Mg sin ă fs = fs1 (max) = øN0 N
N0 = N + (Mg cos )
Mg sin ă (ma0 ă mg sin ) = ø (mg cos + Mg cos )
mg
m M g
amin sin cos fs
m
(b) For amax; (slipping tendency up the plane) N
m M g
amax sin cos f1
m mg
T y p e : Ch a i n Pro b l e m s
Ex a m p l e 1 4
Uniform chain of mass m and length 2 l is placed over a smooth horizontal peg as shown. The
chain is released from rest. After what time will chain leave the peg?
(Neglect the length of chain over peg.)
Solution : At any other instant, let the chain has shifted a length x from lighter side to heavier side.
Let be mass per unit length of the Chain
m
i.e. =
2l
l l ăx
2 2
3l 3l + x
2 2
Now, acceleration
3l 3l
a x g x g
2 2
2l
vdv l 2 x g
dx 2l
v x
g
vdv 2l l 2x dx
0 0
2 x
v2 g l 2 x
2l 4 l 2
0
FRICTION
QUIZRR 21
g
v2 l 2 x l 2
2
4l
g
v l 2 x l2
2
(1)
4l
l/
t
2
dx
dx
V
dt
dt
0 l 2 x2 l 2 0
t
g
= 4l dt
0
l
/2
g
ln l 2 x l
1
l 2 x 2 2
t
2 0 4l
t
l
g
ln 2 3
Time taken for the chain to fall off the peg
=
l
g
ln 2 3
Ex a m p l e 1 5
A small bar starts sliding down an inclined plane forming an angle with the horizontal. The
friction coefficient depends on the distance x covered as ø = kx, where k is a constant. Find
the distance covered by the bar till it stops, and its maximum velocity over this distance.
Solution : Along the incline
mg sin ă kx mg cos = ma
or a = g sin ă kxg cos ...(i)
where a is the acceleration of bar.
We can write a = v dv/dx,
v x
or 0 v dv 0 g sin kxg cos dx
v 2x sin kx 2
cos g ...(ii)
It can be seen that the velocity again becomes zero after covering a distance x = 2 tan a/k
FRICTION
22 QUIZRR
2 tan
Therefore, the distance covered by the bar till it stops is Ans.
k
Further, the maximum velocity of the bar will be at
a= 0
tan
or x from Eq. (i)
k
Substituting this in Eq. (ii), maximum velocity
g
or vm tan sin Ans.
k
Ex a m p l e 1 6
Consider the situation shown in figure. The horizontal surface A
below the bigger block is smooth. The coefficient of friction m
between the blocks is ø. Find the minimum and the maximum
F
force F that can be applied in order to keep the smaller blocks Mass = M
m B
at rest with respect to the bigger block.
Solution : Suppose the minimum force needed to prevent slipping
between the blocks is F.
Considering A + B + C as the system, the acceleration of the system is
F
a
M 2m
FRICTION
QUIZRR 23
1 1
Thus, a lies between 1 g and 1
g
Ex a m p l e 1 7
Figure shows two blocks connected by a light string placed on the
two inclined parts of a triangular wedge. The coefficients of static
and kinetic friction are 0.28 and 0.25 respectively at each of the
surfaces. (a) Find the minimum and maximum values of m for m 2kg
which the system remains at rest. (b) Find the acceleration of
either block if m is given the minimum value calculated in the 45Ĉ 45Ĉ
first part and is gently pushed up the incline for a short while g
= 10 m/s2.
Solution : (a) Consider the 2 kg block as the system. The forces on this block are shown in figure with
M = 2 kg. It is assumed that m has its minimum value so that the 2 kg block has a tendency to slip
down. As the block is in equilibrium, the resultant force should be zero.
Applying LamiÊs theorem
T N
Mg Tf N
90Ĉ
sin 90 sin 90 45 sin 90 45 f
90Ĉ + 45Ĉ
90Ĉ + 45Ĉ
Mg
or N
2
Mg
FRICTION
24 QUIZRR
Mg Mg
and T f s N
2 2
Mg s Mg
=
2 2
Mg
or T 1 s ...(i)
2
Now, consider the other block as the system. The forces acting on this block are shown in figure.
Again applying LamiÊs theorem
N´ T
Mg T f' N'
sin 90 sin 90 45 sin 90 45 90Ĉ
90Ĉ + 45Ĉ
Mg
N'
2
mg
mg
T f'
2
mg mg mg mg
T f' s N' s
2 2 2 2
mg
Thus, T (1 s ) ...(ii)
2
m
1 s M 1 0.28 2 9 kg
or, Ans.
1 s 1 0.28 8
When maximum possible value of m is required, the directions of friction are reversed because m
has the tendency to slip down and 2 kg block to slip up. Thus, the maximum value of m can be obtained
from (iii) by putting øs = ă 0.28. Thus, the maximum value of m is
1 0.28 32
m 2 kg Ans.
1 0.28 9
(b) If m = 9/8 kg and the system is gently pushed, kinetic friction will operate. Thus,
Mg kmg
f k . and f ' ,
2 2
where øk = 0.25. If the acceleration is a, NewtonÊs second law for M gives
Mg sin 45Ĉ ă T ă f = Ma
Mg k Mg
or, T Ma ...(iv)
2 2
FRICTION
QUIZRR 25
mg k mg
or, T ma ...(v)
2 2
M 1 k m 1 k
or, a g
2 M m
2 0.75 9 / 8 1.25
= 2 2 9 / 8
2 3
a g
7
2 3
Therefore, acceleration of the bigger block of mass M is a g
7
Ex a m p l e 1 8
Two blocks A and B of mass 2 kg and 4 kg are placed one over the
other as shown in figure. A time varying horizontal force F = 2t is
A F = 2t
applied on the upper block as shown in figure. Here t is in second
and F is in newton. Draw a graph showing accelerations of A and
B
B on y-axis and time on x-axis. Coefficient of friction between A and
1
B is and the horizontal surface over which B is placed is
2
smooth. (g = 10 m/s2).
Solution : Limiting friction between A and B is
1
fL mA g 2 10 10 N
2
fL 10
amax 2.5 m/s2
mB 4
Thus, both the blocks move together with same acceleration till the common acceleration becomes
2.5 m/s2, after that acceleration of B will become constant while that of A will go on increasing. To find
the time when the acceleration of both the blocks becomes 2.5 m/s2 (or when slipping will start between
A and B) we will write:
FRICTION
26 QUIZRR
F 2t
2.5
mA mB 6 A F = 2t B
t = 7.5 s fL = 10
fL = 10N
Hence, for t 7.5 s
F 2t t
aA aB
mA mB 6 3
1
Thus, aA versus t or aB versus t graph is a straight line passing through origin of slope
3
aA or aB
For, t 7.5 s
aA
aB = 2.5 m/s2 = constant
F fL
and aA 45Ĉ
mA 2.5 m/s 2 aB
aB
a A= tan = 1
3
2t 10 t
or aA or aA = t ă 5
2
Thus, aA versus t graph is a straight line of slope 1 and intercept ă 5. While aB versus t graph is
a straight line parallel to t axis. The corresponding graph is as shown in Fig. 3.194.
Ex a m p l e 1 9
Calculate the height upto which can insect can crawl up a fixed bowl in the form of a
hemisphere of radius r. Given, coefficient of friction is 1 3 .
Solution : As shown in the figure suppose the insect can crawl up the bowl upto point P. In doing so,
the insect rises through a height BA = h, above the bottom B of the bowl of radius r. Clearly, if W is
the weight of insect, then
N = W cos O
f = W sin
f y
From these two equations
P h
f f
tan . In limiting case f = øN or
N N B
W cos W sin
1
Therefore, tan or q = 30Ĉ
3
3r
y = OP cos 30Ĉ =
2
Now, h = BA = OB ă OA = r ă y
3
or hr r 0.134 r Ans.
2
FRICTION
QUIZRR 27
Ex a m p l e 2 0
A table with smooth horizontal surface is turning at an angular speed about its axis A
groove is made on the surface along a radius and a particle is gently placed inside the groove
at a distance a from the centre. Find the speed of the particle with respect to the table as
its distance from the centre becomes L.
Solution : The situation is shown in figure.
ăZ
Let us work from the frame of reference of the table. Let us take
the origin at the centre of rotation O and the X-axis along the groove.
The Y-axis is along the line perpendicular to OX, coplanar with the x
m2x
surface of the table and the Z-axis is along the vertical. Suppose at O X
time t the particle in the groove is at a distance x from the origin and
is moving along the X-axis with a speed v. The forces acting on the
particle (including the pseudo force) are :
Y
(a) weight mg vertically downward,
(b) normal contact force N1 vertically upward by the bottom surface of the groove,
(c) normal contact force N2 parallel to the Y-axis by the side walls of the groove,
(d) centrifugal force m2x along the X-axis
As the particle can only move in the groove, its acceleration is along the X-axis. The only force along
the X-axis is the centrifugal force m2x. All the other forces are perpendicular to the X-axis and have
no components along the X-axis.
Thus, the acceleration along the X-axis is
F
a 2 x
m
dv
or . v 2 x
dx
or v dv 2 x dx
v L
or v dv 2 x dx
0 a
v L
1 2 1 2 2
or 2 v 2 x
0 0
v2 1 2 2
or
2
L a2
2
or v L2 a2 Ans.
FRICTION
28 QUIZRR
Ex a m p l e 2 1
A small disc is placed on a rough inclined plane having inclination with v0
horizontal. An initial velocity v0 is imparted to the disc as shown. O
Find the speed of disc as a function of , the angle made by velocity
and greatest slope line (given ø = tan ).
Solution : Let vx be the velocity component along the line of greatest slope.
Writing the force eqn. along the line of greatest slope.
dvx
m mg sin fk cos
dt
dvx
dt g sin 1 cos
dv
m mg sin cos fk
dt
dv
dt g sin cos 1
v cos v
dv dv
dt
dt
0 o
v0
v cos = (v0 ă v) v
1 cos
Ex a m p l e 2 2
A man stands at the edge of a rotor which begins to move with an angular velocity . Find
minimum , so that man does not slip off the rotor when the base is removed.
Solution : fs = mg
N = m2r
fs øN
mg ø m2r fs
N
g
r
mg
FRICTION
QUIZRR 29
Concept : Maximum elongation occur when the relative longitudinal velocity becomes zero.
F2 kx
a2
m2
kx F1
a1
m1
kx F1 F2 kx
=
m1 m2
0 xmax xmax
F F 1 1
v12 dv12 2 1 dx
m2 m1
kx dx
m1 m2
0 0 0
F F 1 1 x2max
0 2 1 xmax k {v12 = 0, for maximum elongation}
m2 m1 m1 m2 2
1 1 x2 max F2 F
k 1 X max
m1 m2 2 m2 m1
2 m2 F1 m1F2
xmax
k m1 m2
Eq u i v a l e n t s t i f f n e s s o f s p r in g s i n s e r ie s a n d p a r a l le l
(A) Series Combination
K1 K2 K3
O m F
x1 A x2 B x3 C
FRICTION
30 QUIZRR
1 1 1 1
Keq k1 k2 k3
When a spring is cut into several parts, ther, stiffness of each part will be invericly proportional to
its length.
Ex a m p l e 2 3
A very small cube of mass m is placed on the inside of a funnel rotating about a vertical axis
at a constant rate of n revolutions per sec. The wall of the funnel makes an angle with the
horizontal. If the coefficient of static friction between the cube and funnel is ø and the centre
of the cube is at a distance r from the axis of rotation, what are the largest and smallest
values of n for which the block will not move with respect to the funnel?
Solution : Figure shows the funnel with mass m. The different
forces on mass m (w.r.t. funnel) are :
(i) weight mg acting vertically downwards, m r N
(ii) centrifugal reaction force (mv2/r) directed horizontally mv2
r
outwards, f
(iii) normal reaction N offered by the wall, and mg
(iv) frictional force f directed along the incline (when the
revolution is fast enough, the mass may slide upwards).
Since, the mass m is not to move, we have
mv2
N sin mg cos 0 ...(i)
r
FRICTION
QUIZRR 31
mv2
f mg sin cos 0 ...(ii)
r
mv2
or sin cos mg sin cos
r
v2
or cos sin g sin cos
r
v2 g sin cos
or ...(iv)
r cos sin
We know that, v =r = 2nr
4 2 n2 r 2 g sin cos
r cos sin
1 g sin cos
or n2 .
4 2 r cos sin
1 g sin cos
n . ...(v)
2 r cos sin
This must be the maximum frequency allowed.
When the revolution is slow enough, the block may slide down. In this case the frictional force f will
be directed along the incline upwards.
So, replacing ø by ă ø in Eq. (v) the minimum frequency allowed will be
1 g sin cos
n (vi)
2 r cos sin
1 g sin cos
Hence, nmax
2 r cos sin
1 g sin cos
nmin Ans.
2 r cos sin
FRICTION
32 QUIZRR
Ex a m p l e 2 4
In the figure, mass m 1, m 2 and M are 20 kg, 5 kg and 50 kg P1
m1
respectively. The coefficient of friction between M and
ground is zero. The coefficient of friction between m 1 and
M and that between m 2 and ground is 0.3. The pulleys and
the strings are massless. The string is perfectly horizontal P2 M F
between P1 and m 1 and also between P2 and m 2. The string m2
is perfectly vertical between P 1 and P 2. An external
horizontal force F is applied to the mass M. Take g = 10 m/s2.
(a) Draw a free body diagram of mass M clearly showing all the forces.
(b) Let the magnitude of the force of friction between m 1 and M be f 1 and that between
m 2 and ground be f 2. For a particular F is it is found that f 1 = 2f 2. Find F, tension in the string
and acceleration of the masses.
T
Solution : Given, m1 = 20 kg, m2 = 5 kg, M = 50 kg, N 1
ø = 0.3 and g = 10 m/s2
T
(a) Free body diagram of mass M is given in Fig. 3.201
T
(b) The maximum value of f1 is Mg
(f1)max = (0.3)(20)(10) = 60 N T
The maximum value of f2 is
(f2)max = (0.3)(5)(10) = 15 N N
Forces on m1 and m2 in horizontal directions are as shown in Fig. 3.202.
Now, there are only two possibilities :
(1) either both m1 and m2 will remain stationary (w.r.t. ground) or
(2) both m1 and m2 will move (w.r.t. ground).
First case is possible when T
m T
m
T = f1 (for equilibrium of m1)
f1 f2
and T = f2 (for equilibrium of m2)
f1 = f2
Therefore, the condition f1 = 2f 2 will not be satisfied. Thus m1 and m2 both cannot remain stationary.
In the second case, when m1 and m2 both move
f2 = (f2)max = 15 N
f1 = 2f2 = 30 N
Now, since f1 < (f1)max, there is no relative motion between m1 and M, i.e. all the masses move with
same acceleration, say ÂaÊ.
f2 = 15 N
and f1 = 30 N Ans.
Free body diagram showing the forces which are responsible for motion of the masses and equations
of motion are as shown :
FRICTION
QUIZRR 33
f1 = 30 N a
a
a M
T
m1 m2 T
F
f1 f2
Ex a m p l e 2 5
Uniform circular ring rotates with const. angular velocity about a vertical axis passing
through its centre in a horizontal path. Find the tension developed in the ring.
Solution : Consider an infinitesimal element of the ring as shown. (Let its mass be dm) Let tension be T
d
2T cos dm R {Balancing the Components of Tension with the Centrijugal force}
2
2 2
d d d
2T sin dm 2 R sin
2 2 2
ă d
d M N Tcos d 2 2
Tcos d
R d R where 2R
2T 2 2 2
2 R d d
d 2 2
Td = ( 2 R2) d T d T
mg 2
M R 2
T = 2 R2 = 2 2Tsin d
2
M2 R
T
2
FRICTION
34 QUIZRR
En e r g y o f f r ic t i o n
According to the law of conservation of energy is destroyed due to friction, though it may be lost to
the system of concern. Energy is transformed from other forms into heat. A sliding hockey puck comes
to rest because friction converts its kinetic energy into heat. Since heat quickly dissipates, many early
philosophers, including Aristotle, wrongly concluded that moving objects lose energy without a driving
force.
When an object is pushed along a surface, the energy converted to heat is given by :
Eth k Fn x dx
where
Fn is the normal force,
øk is the coefficient of kinetic friction,
x is the coordinate along which the object transverses.
3 .1 0 WORK OF FRICT I ON
In the reference frame of the interface between two surfaces, static friction does no work, because
there is never displacement between the surfaces. In the same reference frame, kinetic friction is always
in the direction opposite the motion, and does negative work. However, friction can do positive work in
certain frames of reference. One can see this by placing a heavy box on a rug, then pulling on the rug
quickly. In this case, the box slides backwards relative to the rug, but moves forward relative to the
frame reference in which the floor is stationary.
Ex a m p l e 2 6
Uniform chain of mass m forming a circle of radius R is slipped over a smooth round cone
with half angle . If the chain rotates with constant about a vertical axis coinciding with
the axis of symmetry of cone, then find the tension developed in the chain.
Solution : y
x
R N z
T N
Tmg
(dm)2R
T (dm)g
Consider an infinitesimal element of mass dm. Substituting angle d at centre
(Td ă N cos ) = dm2r (1) balancing forces
N sin = (dm)g (2) in both directions
Multiply (2) on both sides by cot and add to (1)
We have, Td ă (dm)g cot = (dm) 2R
Td ă (Rd)g cot = (Rd)2R
FRICTION
QUIZRR 35
T = Rg cot + R2R
= R ( cot + 2R)
m
T g cot 2 R
2
Ex a m p l e 2 7
A uniform horizontal rod of mass m and length l is rotated with constant about a vertical
axis passing through fixed end of rod, then find the tension developed in the rod at a distance
of x from fixed end.
Solution :
dx
(T) (T = 0)
(T+dT) at free end
x
Consider an element dx at the distance x from the fixed end with mass be dm.
Concept : Tension at the free ends (of rod, string etc.) is always zero.
Now, T ă (T + dT) = (dm) 2x
ă dT = (dx) 2x = M/L (mass per unit length)
0 L
dT n dx
2
T x
L x2
2
T 2
2
m2 L2 x2
T
L 2
m 2 L x2
T 1 2
2 L
Example 28
Find the minimum coefficient of friction b/w the block and the triangular wedge so that the
block remain stationary relative to wedge.
Solution : ø=?
m N øN
a0
M
5
= 37Ĉ
0.
(m+M)g
=
ø
FRICTION
36 QUIZRR
Consider the wedge and block as single mass system when there is no slipping between the two blocks.
N = (m + M) g cos
(m + M) g sin ă øN = (m + M)a
a0 = (g sin ă øg cos ) ...... (1)
Now drawing F.B.D. of block m by making use of pseudo force and balancing forces
ma0 cos = øN0
N0 = mg ă ma0 sin N0
ma0
ma0 cos
øN0
N0
a0 cos mg
min
g a0 sin
Ex a m p l e 2 9
x
A car rides along a horizontal curve y = a sin with const. speed v. The coeff. of friction
between car and road is ø. Find the maximum speed with which it can ride without skidding.
mv2
Solution : fs [N = mg]
r
Static friction provides the required centripetal acceleration
Concept : Find the maximum speed allowed, corresponding to minimum radius of
curvature, where fs will be required maximum. (i.e. fs = øN)
Now,
minimum radius of curvature exits at peaks, i.e. at (2n + 1)
2
Radius of curvatures is given by a
dy x
3 / 2 cos
dy
2
dx
1
dx 3
r d2 y a x 2
d y
2
dx2 2 sin
2
dx (2)
2
2
At x = r {Note : The formula of radius of curvature is beneficial
2 a for the student and may prove useful in objective
fs ømg questions}
m2
mg
rmin
g
v grmin v
a
FRICTION
QUIZRR 3
2 .2 T y p e s o f Fo r c e s
There are basically three forces which are commonly encountered in mechanics.
(a ) Field Forces
These are the forces in which contact between two objects is not necessary. Gravitational force
between two bodies and electrostatic force between two charges are two examples of field forces. Weight
(W = mg) of a body comes in this category.
(b) Contact Forces
Two bodies in contact exert equal and opposite forces on each other.
A contact force has two components. The part of the force that lies within the plane of contact is
Friction, which must be overcome for the two objects to slide relative to one another along that plane.
The part of the force that is perpendicular to the plane of contact is called the normal force.
Thus, there are two mutually perpendicular components of the contact force :
1. Normal reaction (N)
2. Force of Friction (f)
The net contact force = N2 f 2
Consider two wooden blocks A and B being rubbed against each other.
A
In the diagram, A is being moved to the right while B is being moved
leftward. In order to see more clearly which forces act on A and which on B, a N F1
second diagram is drawn showing a space between the blocks but they are still
supposed to be in contact. f
In Fig. the two normal reactions each of magnitude N are perpendicular to f
the surface of contact between the blocks and the two frictional forces each of
F
magnitude f act along that surface, each in a direction opposing the motion of 2 N
the block upon which it acts. B
Spring Force (F = kx) has been discussed in detail in the chapter of work, energy and power.
rm
A B
1 2
Solution :
– N2 N1 –
2 1 2 2
A B
mg
Dumb Question
(1) Why is the normal acting like this ?
Ans. The direction of the normal force is always perpendicular to the surface of contact.
(2) Why is the tension direction like this ?
B
A
B
A
m
Ans. The direction of tension is always away from the point of contact.
TA A B TB
mg
B T
A T
mg
mg
frictionless FBDfor A FBDfor B
Given :
Four concurrent forces with the magnitudes and geometry shown. 20k 4 60k
3
Determine : 37°
x x
(a) the vertical and horizontal components of each of the forces
45° 30°
(b) the resultant algebraically
(c) the equilibrium 100k 50k
Solution : (a) First break each of the forces into its horizontal and y
vertical components, either by inspection or using the algebraic method.
The components can neatly be entered one by one into a chart similar to the one shown below.
Force Fx Fy
60 ă ă
50 ă ă
100 ă ă
20 ă ă
The 20 k force is the most straightforward,, so enter its components into the chart first. Be careful
to enter the correct sign for each component or the overall result will be incorrect.
Force Fx Fy
60 ă ă
50 ă ă
100 ă ă
20 0 ă 20
4 3
The 60 k force can also be resolved by observation {as cos 37Ĉ =, sin 37Ĉ = } Fx = 4/5 (60 k)
5 5
= 48 k and Fy = 3/5 (60) = 36. Enter these numbers into the chart.
The last two forces are most efficiently resolved using the algebraic equations.
Force Fx Fy
60 48 36
50 ă ă
100 ă ă
20 0 ă 20
Fx = F cos
Fy = F sin
So, for the 50 k force, its components are
Fx = ă 50 k cos 30 = ă 50(.866) = ă 43.3k
Fy = 50k sin 30 = 50(.5) = 25k
and for the 100k force
Fx = ă 100 k cos 45 = ă 100(.707) = ă 70.7k
Fy = ă 100 k sin 45 = ă 100(.707) = ă 70.7k
Use these components to complete the chart.
Force Fx Fy
60 48 36
50 ă 43.3 25
100 ă 70.7 ă 70.7
20 0 ă 20
(b) The components of the resultant are equal to the sum of the columns of the completed chart.
Force Fx Fy
60 48 36
50 ă 43.3 25
100 ă 70.7 ă 70.7
20 0 ă 20
R ă 66.7 ă 29.7
The resultant is found using the Pythagorean Theorem :
= Fx2 Fy2
66.7 29.7
2 2
=
= 72.4 k
Its angle can be found by applying
tan = opposite side [Fy] / adjacent side [Fx]
tan = ă 29.7 / ă 66
tan = .45
= tană1 (0.45)
= 24.2 degrees below the x axis down and to the left
The quadrant can be formed by observation using the signs of the components of the resultant. In
this case, both Fx and Fy are negative, so the resultant will lie in the lower left quadrant.
(c) The equilibrant will be the inverse of the resultant. Its magnitude will be the same, 72.4 k, but
it will act an angle 24.2 degrees above the x axis, up and to the right.
A l g o r it h m f o r So l v in g Pro b l e m s :
Step 1 :
Define the system :
On this system you have to apply NewtonÊs laws. A system may consist of any no of pa each
component must have same acceleration.
Step 2 :
Identify the forces :
List out all the forces acting on the system.
Step 3 :
Draw FBD of the system.
Step 4 :
Choose Ads and write equation. Proper signs must be put with forces or acceleration.
Illustration :
In the arrangement shown in figure, the strings are light and inextensible. The surface
placed is smooth. Find
(a ) Acceleration of each block
(b) The tension in each string
F=21N
4Kg 2Kg 1Kg
Solution : (a) Let a be the acceleration of each block and T1 and T2 be the tensions in the two strings.
Taking the two blocks and the two strings as the system.
T2 T1
F=21N
4Kg 2Kg 1Kg
Fx ma x
Or 21 = (4 + 2 + 1) a
a = 3m/s2.
(b) Free body diagram (showing forces in x direction) of 4 kg block and 1 kg block are shown
y
2
a=3m/ s2 a=3m/ s
F=21N
4Kg 1Kg
T2 T1 x
Fx ma x
For 1 kg block, F-T1 = (1) (a)
Or 21 ă T1 = (1) (3) = 3
T1 = 21-3 = 18 N
For 4 kg block,
T2 = (4) (a)
T2 = (4) (3) = 12 N
Ex am p l e 1
Three blocks A, B and C are placed one over the other as shown in A
figure. Draw free body diagrams of all the three blocks.
B
C
Solution : Free body diagrams of A, B and C are shown below :
N1
N2
WB
N1 N2 N3
Ex am p l e 2
A block of mass m is attached with two strings as shown in figure. Draw the free body
diagram of the block.
T1
mg
Eq u i l ib r i u m :
When the net resultant of all linear and rotational forces on a body is zero, it is said to be in
equilibrium. However, itÊs not necessary that all force on it is zero.
For this, first we would study the resolution of force into components and moment of force about a
point. F B
F2
In figure, F1 = F cos = component of F along AC
F2 = F sin = components of F perpendicular to AC or along AB F1 A C
Finding such components is referred to as resolving a force in a pair of perpendicular directions.
Note that the components of a force in a direction perpendicular to itself is zero. For example, if a force
of 10 N is applied on an object in horizontal direction then its component along vertical is zero. Similarly,
the component of the above force in the direction of force (horizontal) will be 10 N.
Ex am p l e 3
Resolve a weight of 10 N in two directions which are parallel and perpendicular to a slope
inclined at 30Ĉ to the horizontal.
Solution : Component perpendicular to the plane
3
W = W´ cos 30Ĉ = (10)
2
W||
= 5 3N Ans. 30° W
and component parallel to the plane
30° W=10
1
W|| = W´ sin 30Ĉ = (10)
2
=5N Ans.
Ex am p l e 4
Show that the net resultant force on a given body is zero by resolving into components.
Solution : Component of force F1 along horizontal direction is F1 cos 30 while that of F2 is F2 cos 30.
Given
F1 F2 , F1 cos30 F2 cos30 F1
30°
Hence, net force along horizontal direction is zero.
30°
Similarly we can prove for vertical case.
F2
Ex a m p l e 5
A body is supported on a rough plane inclined at 30Ĉ to the horizontal by a string attached
to the body and held at an angle of 30Ĉ to the plane. Draw a diagram showing the forces
acting on the body and resolve each of these forces.
(a) horizontally and vertically
(b) parallel and perpendicular to the plane
Solution : The forces are : N T
the tension in the string T 30°
the normal reaction with the plane N
f
the weight of the body W
and the friction f 30°
M o m e n t o f a Fo r c e
The general name given to an turning effect is torque. The magnitude of torque, also known as
the moment of a force F is calculated by multiplying together the magnitude of the force and its
perpendicular distance r from the axis of rotation. This is denoted by C or (tau).
i.e. C = Fr or = Fr
2 .7 Di re c t io n o f T o r q u e
The angular direction of a torque is the sense of the rotation it would cause.
Consider a lamina that is free to rotate in its own plane about an axis
perpendicular to the lamina and passing through a point A on the lamina.
In the diagram the moment about the axis of rotation of the force F1 is F1r1 F2
F2
anticlockwise and the moment of the force F2 is F2r2 clockwise. A convenient r2 r
1
way to differentiate between clockwise and anticlockwise torques is to allocate A
a positive sign to one sense (usually, but not invariably, this is anticlockwise)
and negative sign to the other.
With this convention, the moments of F1 and F2 are + F1r1 and ă F2r2
(when using a sign convention in any problem it is advisable to specify the chosen positive sense).
Zero m om e nt
If the line of action of a force passes through the axis of rotation, its perpendicular distance from
the axis is zero. Therefore, its moment about that axis is also zero.
Ex a m p l e 6
ABCD is a square of side 2 m and O is the centre. Forces act along D 4N C
the sides as shown in the diagram. Calculate the moment of each
force about
(a) an axis through A and perpendicular to the plane of square 3N 5N
O
(b) an axis through O and perpendicular to the plane of square.
A 2N B
Solution : Taking anticlockwise moments as positive we have :
(a) Magnitude of force 2N 5N 4N 3N
Perpendicular distance from A 0 2m 2m 0
Moment about A 0 ă 10 N-m + 8 N-m 0
Ex a m p l e 7
Forces act as inclined on a rod AB which is pivoted at A. Find the anticlockwise moment of
each force about the pivot.
2F
30°
A B
F
a a 2a
Solution :
Magnitude of force 2F F 3F
30°
Perpendicular distance
in
3F
a
s
from A 2a 4a sin 30Ĉ 2F
4a
Anticlockwise Moment 30°
A B
about A + 2 Fa ă 2 Fa + 6 Fa
F
Co p l a n a r Fo r c e s i n Eq u i li b r iu m
When an object is in equilibrium under the action of a set of two or more coplanar forces, each of
three factors which comprises the possible movement of the object must be zero, i.e., the object has
(i) no linear movement along any two mutually perpendicular directions ox and oy.
(ii) no rotation about any axis.
The set of forces must, therefore, be such that
(a) the algebraic sum of the components parallel to ox is zero or Fx 0
(b) the algebraic sum of the components parallel to oy is zero or Fy 0
(c) the resultant moment about any specified axis is zero or any axis 0
Thus, for the equilibrium of a set of two or more coplanar forces :
Fx 0
Fy 0 and any axis 0
Using the above three conditions, we get only three set of equations. So, in a problem number of
unknowns should not be more than three.
Ex a m p l e 8
A rod AB rests with the end A on rough horizontal ground and the end B against a smooth
vertical wall. The rod is uniform and of weight W. If the rod is in equilibrium in the position
shown in figure. Find : Y
(a ) frictional force at A
(b) normal reaction at A B
(c) normal reaction at B.
30° A
x
O
Solution : Let length of the rod be 2l. Using the three conditions of equilibrium. Anticlockwise moment
is taken as positive. Y
(i) Fx 0 NB ă f A = 0 B NB
or NB = f A ...(i)
(ii) Fy 0 NA ă W = 0 W
NA
3
or 3 N A NB W0
2
Solving these three equations, we get
3 3
(a) fA W (b) NA = W (c) NB W Ans.
2 2
Eq u i l ib r i u m o f Co n c u r re n t Co p l a n a r Fo r c e s
If an object is in equilibrium under two or more concurrent coplanar forces the algebraic sum of the
components of forces in any two mutually perpendicular directions ox and oy should be zero, i.e., the
set of forces must be such that :
Fx 0
Fy 0
The third condition of zero moment about any specified axis is automatically satisfied if the moment
is taken about the point of intersection of the forces. So, here we get only two equations. Thus, number
of unknown in any problem should not be more than two.
2 .6
NewtonÊs laws of motion are three physical laws that form the basis for classical mechanics,
directly relating the forces acting on a body to the motion of the body.
The three laws are :
Fi r st l a w
There exists a set of inertial reference frames relative to which all particles with no net force acting
on them will move without change in their velocity. This law is often simplified as „A body persists
its state of rest or of uniform motion unless acted upon by an external unbalanced force.‰
NewtonÊs first law is often referred to as the law of inertia. So in other words „every object in motion
will stay in motion until acted upon by an outside force.‰
NEWTON’S LAWS OF MOTION
14 QUIZRR
Se c o n d l a w
Observed from an inertial reference frame, the net force on a particle of constant mass is proportional
to the time rate of change of its linear momentum :
F = d (mv)/dt. This law is often stated as, „Force equals mass times acceleration (F = ma )‰ :
the net force on an object is equal to the mass of the object multiplied by its acceleration. This can also
be stated as : „acceleration of a body is directly proportional to external force acting on it, while inversely
proportional to its mass.
T h i rd l a w
Whenever a particle A exerts a force on another particle B, B simultaneously exerts a force on A
with the same magnitude in the opposite direction. The strong form of the law further postulates that
these two forces act along the same line. This law is often simplified into the sentence, „To every
action there is an equal and opposite reaction.‰
2 .7 L A M I’S T H EOREM
LamiÊs theorem in statics states that
if three coplanar forces are acting on a same point and keep it stationary, then it obeys the relation
A B C
sin sin sin
where A, B and C are the magnitude of forces acting at the point (say P), and the values of ,
and are the angles directly opposite to the forces C, B and A respectively.
Force A
Force B Y Force C
Ex a m p l e 9
One end of a string 0.5 m long is fixed to a point A and the other end is fastened to a small
object of weight 8 N. The object is pulled aside by a horizontal force F, until it is 0.3 m
from the vertical through A. Find the magnitudes of the tension T in the string and the
force F. A
T
B F
C
8N
F 8 T
or sin 180 sin 90 sin 90
F 8
or T
sin cos
8 8
T 10N
cos 4 / 5
8sin (8) 3 / 5
and F 6N
cos 4 / 5
Note : Mathematically a body is said to be in equilibrium if
(a) Net force acting on it is zero, i.e., F net 0 .
(b) Net moments of all the forces acting on it about any axis is zero. Physically the body at rest
is said to be in equilibrium. (If a body is at rest just for a moment, it does not mean it is in
equilibrium). For example, when a ball is thrown upwards, at highest points of its journey it
momentarily comes at rest, but there it is not in equilibrium. A net force (equal to its weight)
is acting downward. Due to that force it moves downwards.
If a problem is asked on equilibrium, check whether the body is in equilibrium (permanent rest) or
it is at rest just for a moment.
Now, if the body is in equilibrium, you may resolve the forces in any direction (x, y, z whatsoever).
Net force on the body should be zero in all directions.
But if the body is momentarily at rest but not in equilibrium, see in what direction will the
body moves just after few seconds. Obviously the net force on the body should point in that particular
direction. Therefore components of all the forces in a direction perpendicular to the net force or
perpendicular to the direction in which motion is likely to occur after few seconds should be zero.
Ex a m p l e 1 0
A block of mass m is at rest on a rough wedge as shown
in figure. What is the force exerted by the wedge on
m
the block ?
Solution : Since, the block is permanently at rest, it is in equilibrium. Net force on it should be zero.
In this case only two forces are acting on the block.
(1) Weight = mg (downwards)
(2) Contact force (resultant of normal reaction and friction force) applied by the wedge on the
block.
For the block to be in equilibrium these two forces should be equal and opposite.
Therefore, force exerted by the wedge on the block is mg (upwards). Ans.
Note :
(i) From NewtonÊs third law of motion-force exerted by the block on the wedge is also mg but
downwards.
(ii) The result can also be obtained in a different manner. The normal force on the block is N = mg
cos and the friction force on the block is f = mg sin (not ø mg cos )
These two forces are mutually perpendicular.
mg cos mg sin
2 2
Net contact force would be N2 f 2 or which is equal to mg.
Ex a m p l e 1 1
A body with a mass of 1.0 kg is accelerated by a force F = 2.0 N. what this body after 5.0
s of motion ?
Solution : Given
m = 1.0 kg ă mass of the body,
F = 2.0 N ă force acting on the body
t = 5.0 s ă time of motion of this body
From NewtonÊs Second Law of Motion F=ma (1)
we get expression for acceleration a = F/m (2)
Velocity, according to general formula v = v0 + a t (3)
where v0 = 0, and acceleration is given by equation (2), will be
v = (F/m)t (4)
Substituting numbers given in the problem we get
m
2.0 kg 2
2.0N s kg . m 1
v 5.0 s 5.0 s 10.0 2 . . s 1
1.0 kg 1.0 kg s kg
Ex a m p l e 1 2
The body of mass 1.0 kg has acceleration of 3.0 m/s2 at 30 deg to the positive direction of the
x axis. What are the components along x and y axes of the net force acting on this body ?
Given :
m = 1.0 kg ă mass of the body
a = 3.0 m/s2 ă acceleration
Fy=Fsin
a
F=m
x
Fx=Fcos
The net force acting on the body is, according to NewtonÊs Second law of motion,
F = ma (1)
Formulas for their components along x and y axes are given on the Figure. We left the effort of
substituting the numbers and finding the units to the Reader.
The final answer is :
Fx = 2.6 N
Fy = 1.5 N
Ex a m p l e 1 3
An elevator has a mass of 1400 kg. What is the tension in the supporting cable when the
elevator travelling at 10 m/s is brought to rest in a distance of 40 m. Assume a constant
acceleration.
Given : m = 1400 kg ă mass of elevator,
v = 10 m/s ă initial speed of the elevator,
D = 40 m ă distance required to stop the elevator.
g = 9.81 m/s2 ă gravitational acceleration, as usual is assumed to be known.
Unknown :
T = ? ă magnitude of tension in the cable while bringing the elevator to rest.
To find T we must calculate :
a = ? ă acceleration while stopping the elevator,
t = ? ă time required to stop elevator.
Solution : It is convenient to draw a free-body diagram, as in Figure below.
T is the tension in the cable of the elevator, m g is the gravity force. The resultant force is the
force producing acceleration (deceleration in this case) of our elevator.
T
T mg ma (1)
v2
a (4)
2D
Magnitude of tension T can be found from formula (1) taken without the vector notation (magnitude
only!!)
v2
T m g
2D (5)
Ex a m p l e 1 4
A ball of mass 1 kg is at rest in position P by means of two
light strings OP and RP. The string RP is now cut and the ball
swings to position Q. If = 45Ĉ. Find the tensions in the strings
in positions OP (when RP was not cut) and OQ (when RP was
cut). Take g = 10 m/s2.
P
Solution : In the first case, ball is in equilibrium (permanent rest). R Q
Therefore, net force on the ball in any direction should be zero.
F in vertical direction = 0
or T1 cos = mg T1
mg
or T1
cos
mg
We get T1 10 2 N Ans.
Note : Here we deliberately resolved all the forces in vertical direction because component of the
tension in RP in vertical direction is zero. Although, since, the ball is in equilibrium, net force on it in
any direction is zero. But in a direction other than vertical we will have to consider component of
tension in RP also, which will unnecessarily increase the calculation.
In the second case ball is not in equilibrium (temporary rest). After few seconds it will move in a
direction perpendicular to OQ. Therefore, net force on the ball at Q is perpendicular to OQ, or net force
along OQ = 0.
T2 = mg cos
Ex a m p l e 1 5
A smooth semicircular wire track of radius R is fixed in a vertical plane.
One end of a massless spring of natural length 3R/4 is attached to the
lowest point O of the wire track. A small ring of mass m which can slide
on the track is attached to the other end of the spring. The ring is held
C
stationary at point P such that the spring makes an angle 60Ĉ with the P
vertical. The spring constant k = mg/R. Consider the instant when the 60°
ring is released. O
(i) Draw the free body diagram of the ring.
(ii) Determine the tangential acceleration of the ring and the normal reaction.
(JEE 1996)
Solution : (i) CP = CO = Radius of circle (R)
CPO = POC = 60Ĉ
OCP is also 60Ĉ.
Therefore, OCP is an equilateral triangle.
Hence, OP = R
Natural length of spring is 3R/4
Extension in the spring,
3R R
xR
4 4
C
mg R mg
Spring force, F kx 4 P
R 4 60°
O
mg
Here, F kx
4
and N = Normal reaction.
(ii) Tangential acceleration, a T : The ring will move towards the x-axis just after the release.
So net force along x-axis.
mg 3 3
Fx = F sin 60Ĉ + mg sin 60Ĉ = 4 2 mg 2
5 3 C N
Fx mg P
8
60°
Therefore, tangential acceleration of the ring. O mg
Fx 5 3
aT = ax = g
m 8
5 3
aT g Ans.
8
Normal reaction N
Net force along y-axis on the ring just after the release will be zero.
Fy = 0
N + F cos 60Ĉ = mg cos 60Ĉ
N = mg cos 60Ĉ ă F cos 60Ĉ
y
mg mg 1 mg mg 60°
N
4 2
= 60°
2 2 8 F 60°
O x mg
3mg
N Ans.
8
Note : Three types of equilibrium viz stable equilibrium, unstable equilibrium and neutral equilibrium
will be discussed in the chapter of work, energy and power.
Ex a m p l e 1 6
Two blocks, m 1 = 3.0 kg are put in contact on a frictionless surface. A horizontal force
F = 5.0 N is applied to one of them (see Fig. 1). (1) Find the force Fa between the two blocks.
(2) Find the force Fb if the force F is applied to m 2 in the opposite direction.
m2
m1
F
Solution : Fig. 1 is drawn in such a way that it suggests that the force F is applied from the outside,
but we know that this force can be considered as concentrated in the center of gravity of this block.
We draw the Free Body Diagrams for these blocks. For the block m1, it is on the left hand side of Fig.
2, and for the block m2 on the right hand side of this Figure.
N1 N2
Fa F Fa
m1g m2g
The force are applied to the CG (center of gravity) of mass m1 and m2. In the explanations and
calculations we will use bold face notation for vector quantities. It is easier to handle with text editors
as opposed to handwriting, where arrows are more convenient to use. The N1 force is a reaction force
to the gravity force m1g as described by NewtonÊs third Law of Motion. Analogically, N2 and m2g are
equal and opposite as this law requires.
For the m1 block (the left part of Fig. 2) the direction of force F is obvious from the text of the
problem (see Fig. 1). But why is the force Fa directed to the left ? Because this is the force extended
on block m1 by block m2. Block m1 „pushes‰ block m2 so block m2 creates an „opposite‰ force according
to NewtonÊs Third Law of Motion. Stop and think it over for a moment. This is a very crucial point of
all problems involving NewtonÊs Laws. Once you understand the idea of drawing the Free Body Diagram
you will solve all problems involving mechanics much faster and more efficiently.
Now itÊs time to write equations based on NewtonÊs Laws. In the vertical direction
m1g = N
so we can forget about these forces in further analysis.
In the horizontal direction the resultant force exerted on m1 is F ă Fa and this is the force accelerating
block m. Therefore we can write
F ă Fa = m1a (1)
The FBD for m2 shows that the only the horizontal force acting on it is the exerted by block m1. This
force has a magnitude of the Fa from the FBD on the left side if Fig. 2, but the direction is opposite.
This is the force accelerating block m2. As both blocks are in contact they must have the same acceleration
a . So, for the second block the equation of motion is
Fa = m2 a (2)
We can drop out the vector notation from these two equations as the directions are well defined on
the FBDÊs for both blocks.
From the (2) we have
a = Fa/m2
and substituting this acceleration into (1) we find, after a little elementary algebra,
Fa = F m2/(m1 + m2)
Ex a m p l e 1 7
Three blocks of mass 3 kg, 2 kg and 1 kg are placed side by side on 12N
3kg 2kg
a smooth surface as shown in figure. A horizontal force of 12 N is 1kg
12
or a 2 m/s2
6
Now, let F be the net force on 2 kg block in x-direction, then using Fx max for 2 kg block, we
get
F = (2)(2) = 4 N Ans.
Note : Here net force F on 2 kg block is resultant of N1 and N2 (N1 > N2)
where N1 = normal reaction between 3 kg and 2 kg block
and N2 = normal reaction between 2 kg and 1 kg block.
Thus, F = N1 ă N2
Ex a m p l e 1 8
In the arrangement shown in figure. The strings are light and inextensible. The surface
over which blocks are placed is smooth. Find :
F = 14N
(a) the acceleration of each block 4kg 2kg 1kg
a
Using Fx max
or 14 = (4 + 2 + 1)a 4kg 2kg
F = 14N
1kg
14
or a 2 m/s2 Ans.
7
(b) Free body diagram (showing the forces direction only) of 4 kg block and 1 kg block shown in
figure.
y
Using Fx max a = 2m/ s2 a = 2m/ s2
Ex a m p l e 1 9
A block is projected up an inclined, frictionless plane with an initial speed V0. How does the distance
the block will go up the plane depend on the angle of the incline ?
The figure below represents the situation described in this problem.
-mgcos
V0
mgsin
mgcos
mg
The force of gravity is decomposed into components perpendicular to the inclined plane and parallel
to it. According to NewtonÊs Third Law of Motion the „red‰ force is compensated by the „green‰ one. The
force left is the one parallel to the inclined plane. We are interested in accelerations which can be
obtained by dividing the forces by the mass of the block.
The distance which will be traveled by our object after being projected with an initial speed v0 is
D = v0t ă (1/2)gsint2 (1)
and the final velocity will be zero, according to formula
0 = v0 ă gsin t (2)
Solving this set of equations we get
v02
D
2 g sin
The distance traveled in inversely proportional to the sine of the angle of the incline and this is the
answer to the question from the problem.
Notice, that this distance DOEST NOT depend on the mass of the block. If this seems strange to
you, think about the moment of projecting this mass. The larger the mass the larger the force required
to accelerate this mass to initial speed v0, and here the dependence on mass is „hidden‰.
Ex a m p l e 2 0
In the adjacent figure, mass of A, B and C are 1 kg, 3 kg and
2 kg respectively.
C
B
Find (a ) the acceleration of the system and
(b) tensions in the string
A
Neglect friction (g = 10 m/s2)
Solution : (a) In this case net pulling force
= mAg sin 60Ĉ + mBg sin 60Ĉ ă mCg sin 30Ĉ
3 3 1
= 110 2 (3) 10 2 2 10 2
= 24.64 N
Total mass being pulled = 1 + 3 + 2 = 6 kg
24.64
Acceleration of the system a 4.1 m/s2
6
3
T1 1 10 4.1 4.56 N Ans.
2
= 18.2 N Ans.
2 .8 PSEU DO FORCE
NewtonÊs first and second laws (F = ma) only hold in inertial frames of reference. An observer in
an accelerating frame will note that objects seem to move from a state of rest without any apparent
force acting on them. For example, a driver using his braking car as a reference frame might note that
a coin on his dashboard starts gliding towards the windshield without being visibly pushed.
The easy way out is to take F = ma as a definition, and declare that a force is in fact acting on
the coin, with a magnitude equal to the acceleration times mass of it. A force that is „invented‰ in this
way so that the second law will hold in a given accelerated frame of reference is called a pseudo force.
The defining characteristic of such a force is that if the same physical situation is described in an
inertial frame of reference, all pseudo forces vanishes (as opposed to normal forces between objects,
which stay the same no matter what frame of reference they are described in).
The most common use of pseudo forces is not linearly accelerating frames as the above car example,
but rotating frames of reference. An observer in a rotating frame of reference will be subjected to two
famous pseudo forces, centrifugal force and Coriolis force.
{However, study of coriolis force is not in the course of IIT-JEE}
That is what is meant by the common statement that „centrifugal force is not a real force‰ - if the
same situation is looked at from an inertial frame, the centrifugal force does not appear. That does not
make it less useful, though. Many situations, like the stress in a spinning wheel, are most conveniently
analysed in terms of centrifugal rather than centrifugal force.
De r iv a t io n o f p s e u d o f o r c e f r o m t ri a n g l e l a w o f v e c t o r a d d i t io n
r p1 s r p1 s ' r s ' s
r p1 s ' r p1 s r s ' s
Differentiating twice we get
rp1s´
rp s´
a ps' a p1 s a s ' s 1
a1 a a0 O O´
S S´
m a m a m a0
a0
= F external + m
ma
[Force in [Relative [pseudo force]
non-inertial force] to inertial frame]
Pseudo force = m a 0
Ex a m p l e 2 1
Find the horizontal acceleration that must be imparted to wedge, so that block remains
stationary relative to the wedge.
Solution :
M
a
M
Concept : When we apply pseudo force and then analyse the blockÊs FBD, the block
should be in equilibrium.
Resolving the forces in vertical and horizontal components and solving them separately for equiliprium
position after applying pseudo force
N
Ncos = mg (vertical)
mg tan
a g tan
m mg
(a = g tan ) must be imparted towards left to keep the body stationary relative to block.
(b) Acceleration that would make the block to loose contact with the wedge.
N = (mg cos ă ma sin ) N
At the breaking point N 0
mg cos – ma sin 0 ma
mg cos ma sin
g cot a
mg
when a g cot bodies loose contact.
2 .1 1 PSEU DO FORCE N
Suppose a block A of mass m is placed on a lift ascending with an acceleration a0. Let
N be the normal reaction between the block and the floor of the lift. Free body diagram A a0
of A in ground frame of reference (inertial) is shown in Fig.
N ă mg = ma0
or N = m (g + a0) ...(i) mg
N´
But if we draw the free body diagram of A with respect to the elevator (a non-
inertial frame of reference) without applying the pseudo force, we get
N´ ă mg = 0 or N´ = mg ...(ii) A (At rest)
Since, N´ N, either of the equations is wrong. But if we apply a pseudo force in
non-inertial frame of reference, N´ becomes equal to N as shown in Fig. 3.93. Acceleration
of block with respect to elevator is zero. mg
N´ ă mg ă ma0 = 0 N´
or N´ = m(g + a0) ...(iii)
N´ = N]
A (At rest)
Pseudo force is given by FP m a 0 . Here, a is the acceleration of the non-
0
inertial frame of reference and m the mass of the body under consideration. In the
whole chapter, we will show the pseudo force by F P . mg + FP
Here FP = ma0
Thus, we may conclude that pseudo force is not a real force. When we draw the
free body diagram of a mass, with respect to an inertial frame of reference we apply only the real forces
(forces which are actually acting on the mass), but when the free body diagram is drawn from a non-
inertial frame of reference a pseudo force (in addition to all real forces) has to be applied to make the
equation F m a to be valid in this frame also.
Note : In case of rotating frame of reference this pseudo force is called the centrifugal force when
applied for centrifugal acceleration. Let us take few examples of pseudo forces.
Ex a m p l e 2 1
In the adjoining figure, the coefficient of friction between wedge (of mass M) and block (of
mass m) is ø.
Find the magnitude of horizontal force F required to keep the block stationary with
respect to wedge.
Solution : Such problems can be solved with or without using the concept of pseudo force. Let us, solve
the problem by both the methods.
a = Acceleration of (wedge + block) in horizontal direction
F
= Mm
F=N y F=N
N FP = ma N
x
mg mg
a
with respect to ground is moving with an with respect to wedge block is stationary.
Ex a m p l e 2 2
All surfaces are smooth in adjoining figure. Find F
such that block remains stationary with respect to
wedge. m
F
M
F
Solution : Acceleration of (block + wedge) a M m
Let us solve the problem by both the methods
From inertia frame of ref. (Ground) From non-inertial frame of ref. (Wedge)
F.B.D. of block w.r.t. ground (Apply real forces) F.B.D. of block w.r.t. wedge (real forces + pseudo
force)
N cos y
N cos
N sin
x FP = ma N sin
mg
a mg
with respect to ground block is moving with an w.r.t. wedgde, block is stationary
acceleration ÂaÊ Fy = 0 N cos = mg ...(iii)
= Fy = 0 N cos = mg ...(i) Fx = 0 N sin = ma ...(iv)
and Fx = ma N sin = ma ...(ii) From Eqs. (iii) and (iv), we will get the same
From Eqs. (i) and (ii) result
a = g tan i.e., F = (M + m) tan
F = (M + m) a
= (M + m) g tan Ans.
Ex a m p l e 2 2
A bob of mass m is suspended from the ceiling of a train moving with an acceleration Âa Ê
as shown in figure. Find the angle in equilibrium position.
a
with respect to ground bob is also moving with an With respect to train, bob is in equilibrium
acceleration ÂaÊ
Fy 0 T cos = mg ...(iii)
Fy 0 T cos = mg ...(i)
and Fx ma T sin = ma ...(iv)
and Fx ma T sin = ma ...(ii)
From Eqs. (iii) and (iv), we get the same result,
From Eqs. (i) and (ii), we get
a
i.e. tan 1 Ans.
a 1 a g
tan or tan Ans.
g g
Ex a m p l e 2 4
In the adjoining figure, a wedge is fixed to an elevator moving upwards
a
with an acceleration Âa Ê. A block of mass ÂmÊ is placed over the wedge.
Find the acceleration of the block with respect to wedge. Neglect
m
friction.
Solution : Since, acceleration of block w.r.t. wedge (an accelerating or non-inertial frame of reference)
is to be find out
F.B.D. of ÂblockÊ w.r.t. ÂwedgeÊ is shown in Fig. 3.104.
N
n
a) si
+
= (g
anet
mg + FP = mg + ma
The acceleration would had been g sin (down the plane) if the lift were stationary or when only
weight (i.e. mg) acts downwards.
Here, downward force is m(g + a)
Acceleration of the block (of course w.r.t. wedge) will be (g + a) sin down the plane. Ans.
Ex a m p l e 2 4
An object is hung from a spring balance attached to the ceiling of an elevator. The balance
reads F1 = 12 N when the elevator is accelerating upward, and reads F2 = 8 N when it is
accelerating downward with acceleration of the same magnitude a . Find the mass of this
object and magnitude of acceleration a . We assume that gravitational acceleration g is
known.
Solution :
a
-a
F1 F2
mg mg
(A) (B)
From the Fig. 1 we can say that the reading of the balance is simply the force exerted on mass m by
the rope attached to the mass. In both cases
(A) elevator accelerates upwards
(B) elevator accelerates downwards
the force exerted by rope suspending the mass is directed upwards. The resultant force is responsible
for acceleration which has the same magnitude a, but different direction in both cases. We write
NewtonÊs Second Law of motion for these two cases. As all forces are acting along the same line
(vertical) we omit vector notation.
F1 ă mg = ma (1)
F2 ă mg = ă ma (2)
The minus sign on the right hand side of Eq. 2 says that acceleration a in case (B) has the opposite
direction to acceleration in case (A).
This is a set of two linear equations with two unknowns, which required only a little algebra to solve.
We leave this algebra to the Reader.
The solution is :
F1 F2 m
a g 1.962 2
F1 F2 s
F1 F2
m 1.019 kg
2g
A M A N L OWERS H I M SELF WI T H T H E H EL P OF A PU L L EY
Ex a m p l e 2 5
A man with a mass of m 1 = 80 kg lowers himself by h = 10 m along a wall, while fixed to a
rope that runs over a frictionless pulley to a m 2 = 70 kg sand bag.
(1) What is his final speed if he started from a state of rest ?
(2) How long does it take to „travel‰ this distance?
Solution : The Figure below illustrates the problem schematically. We can write following equations.
m1
h
F1
h m2
F2
F1 = m1g
F2 = m2g
F1 > F2 so the resultant force FR exerted on these two masses is directed downward on the side of
m1.
FR = m1g ă m2g (1)
Under action of this force BOTH masses are moving with acceleration a
FR
a (2)
m1 m2
g m1 m2
a (3)
m1 m2
The distance h traveled from the state of rest with this acceleration is
h = (1/2)at2 (4)
and the final speed
v = at (5)
Equations (4) and (5) constitute a set of two linear equations with two unknowns and can be solved
by a standard method. As the algebra involved in this is very simple we give here only the final
solutions.
2hg m1 m2 2h m1 m2
v t
m1 m2 g m1 m2
B L OCK OF A N I N CL IN E
Ex a m p l e 2 6
A block of mass m 1 = 2.0 kg on a frictionless inclined plane of angle 20 deg is connected by
a rope over a pulley to another block of mass m 2 = 1.0 kg. What are the magnitude and
direction of the acceleration of the second block?
The figure below will help us to solve this problem.
-F1a
m1
F1b
a m2
F1a
F1 = m1g
F2 = m2g
The for ce m1g, caused by gravity, can be decomposed into two forces : F1a and F1b. Elementary
geometry and the definitions of trigonometric functions allow us to write
F1a = m1g cos (1)
F1b = m1g sin (2)
F2 = m2g (3)
The force F1a, represented by the red vector is compensated, according to NewtonÊs Third Law of
Motion, by a force represented by the green vector. So we are left with forces F1b and F2 acting one
against the other through a rope connecting blocks.
LetÊs assume that F1b = F2 = a (m1 + m2) (4)
Substituting (2) and (3) into (4), after a little of elementary algebra we get
a = g (m1sin ă m2)/(m1 + m2)
Substituting numbers given in the problem we get
a = ă 1.03 m/s2
The minus sign tells us, that acceleration has direction opposite to the one chosen by us for writing
equations leading to the solution of the problem. This is a general rule in all kinds of problems. Negative
numerical value means the direction of the parameter found is opposite to the one which was assumed
for writing equations.
L A M P I N A N A SCEN DI N G EL EV A T OR
Ex a m p l e 2 7
A lamp hangs vertically from a cord in an elevator which is descending with an downward
acceleration of a = 2.0 m/s2. The tension in the cord is T = 10.0 N. What is the mass m of this
lamp?
2 .9 M OM EN T U M
The change of momentum of a body is proportional to the impulse impressed on the
body, and happens along the straight line of which that impulse is impressed.
Using modern symbolic notation, NewtonÊs second law can be written as
d (mv) dv
Fnet m
dt dt
where F is the force vector, m is the mass of the body, v is the velocity vector and t is time.
It should be noted that, as is consistent with the law of inertia, the time derivative of the momentum
is non-zero when the momentum changes direction, even if there is no change in its magnitude.
Since the mass of the system is constant, this differential equation can be rewritten in its simpler
and more familiar form :
F = ma
dv
where a
dt
is the acceleration.
A verbal equivalent of this is „the acceleration of an object is proportional to the force applied, and
inversely proportional to the mass of the object”. In general, at slow speeds (slow relative to the speed
of light), the relationship between momentum and velocity is approximately linear. Nearly all speeds
within the human experience fall within this category. At higher speeds, however, this approximation
becomes increasingly inaccurate and the theory of special relativity must be applied.
2 .1 0 I MPU L SE
The term impulse is closely to the second law, and historically speaking is closer to the original
meaning of the law. (12) The meaning of an impulse is as follows :
An impulse occurs when a force F acts over an interval of time t and is given by F dt
t
I = P = mv
where I is the impulse, p is the change in momentum, m is the mass, and v is the change in
velocity.
m2
A p p r o a c h (1 )
If m2 moves by a distance x2 down, the string or m, is stretched by 2(x1)
Since in this question one end is fixed, therefore 2x comes from string attached to m2.
Note : If the end has not been fixed then the movement
of m, will not be 2x2, it will depend on the movement of the This
m1 is
unfixed end also. This will be illustrated in the coming
fixed
examples.
x1 = 2x2
x1 = 2x2
differentiating it twice for getting relation in acceleration x2
m2
a1 = 2a2
A p p r o a c h (2 )
This approach is based on the concept that the length of spring remains constant (assuming that
string is inextensible) we calculate the length of string
i.e. x1 + (x2 + x2) + d = l
where d is a constant length, as will not change on the d
x1
movement of any block & l is also constant. m1
B
differentiating the equation twice
(ă a1) + 2a2 = 0 x2
a1 = 2a2
A
{Note : derivative of decreasing length (such as x1 in this
question) is taken as negetive, whereas for increasing length the m2
derivative is positive}
A p p r o a c h (3 )
(Work-Energy Method)
Concept : Work done by the tension in string in all the frame of reference = 0
Reason : String is inextensible
i.e. Elastic potential energy stored in string = 0
2T (ă x2) + T (x1) = 0
x1 = 2x2
a1 = 2a2
Illust rat io n
Blocks A & B are connected through light inextensible
string, if block A is moved to the right with acceleration 5m/s2
of 5 m/s2. Find acceleration of block B. A
xo B
Solution : At any distance of block A and block B from wall P/pulley be xa & xb respectively.
length of string
xa
(xB ă x0) + (xB) + xA = L {here x0 is constant} 5m/s2
A
differentiating
xo B
dx dxA
2 B 0 xB
dt dt
here xa is increasing & xB is decreasing (assuming that B is moving towards wall)
ă 2VB + VA = 0
ă 2aB + aA = 0
aA = 2aB
aA 5
aB = 2.5 m/s2
2 2
Steps to sovle the motion of connected bodies
1. Separate the system into an isolated body
2. Make free body diagram of individual bodies to assess the forces.
3. Find the relation among the motion of different bodies through contraint.
4. Frame the equation of motion in suitable direction and solve.
Ex a m p l e 2 9
Find the acceleration of T-shaped rod and of wedge.
a1 a2
m
a1
m2
a0
m1 a0
Solution : We can see from the diagram that as the T-shaped rod moves downward, the wedge with
mass (m), moves to the left & wedge with mass (m2) moves toward right.
From the free body diagram of the parts first finding the relation between acceleration through
constraints.
a0 cos = a1 sin
a0 = a1 tan a1 = a0 cot (1)
a2 = a0 cot (2)
Calculating forces in normal direction.
N2
N1
mg ă N1 cos ă N2 cos
= ma0 (3)
mg
mg
Ex a m p l e 3 0
(a) Find the acceleration of each body when the wedges are fixed.
(b) The wedges are movable.
Solution :
(a) When wedge (i) is fixed :
by logical approach
a4 = a3 + a23 a3
(Note : here a23 is the acceleration of block 2 with respect to block 3. O
This is taken because block 3 is also moving.
a4
from point O, applying string method 3 2 1 4
a23
r4 + r3 + r23 = L
differentiating
a4 ă a3 ă a23 = 0 {since r3 & r23 are decreasing}
a4 = a3 + a23
Ex a m p l e 3 2
Find acceleration relation.
2
1
3
Solution :
(a) In such type of question follow logical approach which is the best suited.
a23 = a4 + a3
a4
(b) Applying string method
r3
r4 + r3 + r23 = L r23
4
ă differentiating T
r4 T
a4 ă a3 + a23 = 0 r23
2
(c) Work energy method 1
a3 3
T (r4) + (T ă T cos ) r3 + T (r3 cos ă r23) = 0
Solving this we get
r4 + r3 = r23
thus a4 + a3 = a23
KINEMATICS
1
Kinematics is the science of describing the motion of objects using words, diagrams, numbers, graphs,
and equations. The goal of any study of kinematics is to develop sophisticated mental models which
serve to describe (and ultimately, explain) the motion of real-world objects.
CH ECK Y OU R U N DERST A N DI N G
To test your understanding of this distinction, consider the quantities listed below. Categorize each
quantity as being either a vector or a scalar.
Quantity Category
a. 5m Scalar
b. 30 m/sec, East Vector. A direction is listed for it.
c. 20 degrees Celsius scalar
V e c t o r r e p r e s e n t a t i o n i n 2 -D:
The base vectors of a rectangular x-y coordinate system are given by the unit vectors i→ and →j along
the x and y directions, respectively.
j
y
i
F
F
y
x
Fx
F Fxi→ Fy →j
IITJEE KINEMATICS
2 QUIZRR
V e c t o r r e p r e s e n t a t i o n i n 3 -D:
In a rectangular coordinate system the components of the vector are the projections of the vector along
the x, y, and z directions. For example, in the figure the projections of vector A along the
x, y, and zdirections are given by Ax, Ay, and Az, respectively.
K
Z
A
Az j
y
i Ax
x Ay
A A xi→ A y →j Az k→
1 .2 A B ODY I N M OT I ON :
A body is said to be motion, when its changes its position with time, with respect to an observer.
Similarly, if the position doesnÊt change with time, with respect to an observer (reference), the body is
in rest.
Basic terms :
1. Particle : A particle is a point mass. However, in practice a body may be treated as a particle,
if its size is very small compared to the distance covered by it.
2. Position : The position of a particle refers to its location in the space at a certain moment of
time. In general the position is measured by a vector joining a fixed point (known as origin)
to the moving particle. This vector is known as position vector.
For a particle in straight line motion along X-axis, the position vector is always parallel to X-axis and
hence has only X-component as non-zero. Therefore the position of a moving particle can be measured
by the X-coordinate x (t) at a certain time instant t.
If a particle is moving in a curve (i.e. in a plane) the position vector can have many possible directions.
The position in such a case can be measured by two numbers : X-coordinate and Y-coordinate the
position vector.
y
A(x ,y)
o x
3. Distance is a scalar quantity which refers to „how much ground an object has covered‰ during
its motion. It is the actual length of the path travelled by a particle.
4. Displacement is a vector quantity which refers to „how far out of place an object is‰. It is the
vector joining the initial position of the particle to its final position during an interval of time.
The change in the position of a moving object is known as displacement
KINEMATICS IITJEE
QUIZRR 3
(a) To test your understanding of this distinction, consider the m otion depicted in the diagram
below. A physics teacher walks 4 meters East, 2 meters South, 4 meters West, and finally 2 meters
North.
4m
2m 2m
4m
Even though the physics teacher has walked a total distance of 12 meters, her displacement is 0 meters.
During the course of her motion, she has „covered 12 meters of ground‰ (distance = 12 m). Yet, when
she is finished walking, she is not „out of place‰ i.e., there is no displacement for her motion (displacement
= 0 m). Displacement, being a vector quantity, must give attention to direction. The 4 meters east is
canceled by the 4 meters west; and the 2 meters south is canceled by the 2 meters north.
(b) If a particle goes from A to B along a curve in some time duration and if O is the origin then
B
OA = initial position vector = r1
A s
OB = final position vector = rf
rf
AB = displacement vector = OB OA
r
i
s rf ri r
O
5. Speed: Speed is a scalar quantity which refers to „how fast an object is moving. „Rate of
change of distance (x) covered by a particle, with time is called the speed of the particle. Its
unit is m/s.
Velocity is
6. Velocity: Velocity is a vector quantity which refers to „the rate at Speed with
which an object changes its position.‰ Rate of change of a direction.
displacement of the particle with time is called velocity of the
particle.
7. Average speed: The average speed of a particle in a time interval is defined as the distance
travelled by the particle by the time interval. If the particle travels a distance s in time t1 to
t2 the average speed is defined as:
s
speed av
t2 t1
IITJEE KINEMATICS
4 QUIZRR
Di f f e r e n c e b e t w e e n a v e r a g e s p e e d a n d a v e r a g e v e l o c i t y :
Consider a particle moving around a circle. A particle starts at point P in the circle and covers the entire
circumference of the circle and reaches back at point P (as given in the figure below):
p
Total distance travelled
In this case, Average speed =
Time taken
10. Instantaneous velocity: The velocity at a particular moment of time is known as instantaneous
velocity. The term ÂvelocityÊ usually refers to the instantaneous velocity.
11. Acceleration: Rate of change of velocity with time is called acceleration. Acceleration is a
vector quantity. Its unit is m/s2.
Di re c t io n o f t h e A c c e l e r a t i o n V e c t o r
Acceleration is a vector quantity so it will always have a direction associated with it. The direction of
the acceleration vector depends on two factors:
whether the object is speeding up or slowing down
whether the object is moving in the positive (+) or negative ( ) direction
The general RULE OF THUMB is :
If an object is slowing down, then its acceleration is in the opposite direction of its motion.
This RULE OF THUMB can be applied to determine whether the sign of the acceleration of an
object is positive or negative, right or left, up or down, etc.
KINEMATICS IITJEE
QUIZRR 5
12. Average acceleration: Average acceleration is defined as the change in velocity divided by
the time interval.
change in velocity
Average acceleration =
time interval
v 2 v1
Thus, a av
t2 t1
The average acceleration depends only on the acceleration at time t1 and t2. The way the
velocity changed in between these times does not make a difference in defining average
acceleration.
13. Instantaneous acceleration: The acceleration at a given instant of time is the called the
instantaneous acceleration. When we refer to acceleration, we mean Âinstantaneous accelerationÊ,
i.e.,
v v
a lim
t0 t dt
We also know that F ma , which means that acceleration is also decided by the net force acting
on a particle. If F = 0, then a = 0 and the motion is known as uniform motion because the velocity does
not change, which also means that the particle is moving with a constant velocity. While, if the force
is constant, acceleration is also constant and the motion is known as uniformly accelerated motion,
which means that acceleration acts on the particle, but it is constant. An example of such a case is the
free fall motion(force and acceleration do not change).
T h e B i g M is c o n c e p t io n !!
We know that the acceleration of a free-falling object on Earth is 10 m/s/s.
This value (known as the acceleration of gravity) is the same for all free-falling
Hey ! Good
objects regardless of how long they have been falling, or whether they were initially Question !
dropped from rest or thrown up into the air. Yet the question is often asked
„DoesnÊt a massive object accelerate at a greater rate than a less massive object?‰.
This question is a reasonable inquiry that is probably based upon personal
observations made of falling objects in the physical world. After all, nearly everyone
has observed the difference in rate of fall of a single piece of paper (or similar object) and a textbook.
The two objects clearly travel to the ground at different rates with the massive book falling faster.
The answer to the question (DoesnÊt a massive object accelerate at a greater rate than a less
massive object?) is . . . absolutely not! That is, absolutely not, if you are considering the specific type
of falling motion known as free-fall. Free-fall is the motion of objects under the sole influence of gravity;
free-falling objects do not encounter air resistance. Massive objects will only fall faster than less massive
objects if there is an appreciable amount of air resistance present.
IITJEE KINEMATICS
6 QUIZRR
Ex am p l e 1
A bird flies north at 20 m/s for 15s. It rests for 5s and then flies south at 25 m/s for 10s. For
the whole trip find
(a) the average speed;
(b) the average velocity;
(c) the average acceleration.
Solution.
distance traveled towards north = AC = 20 m/s 15 s = 300 m C
distance traveled towards south = CB = 25 m /s 10 s = 250 m
distance P-th
Average Speed =
time
300 + 250 B
Average velocity = displacement time = 15 + 5 + 10 m/s 18.32 m/s . A
v v f v i 25 20
Average Acceleration = a av m/s2 1.5m/s2
t t 30
Ex am p l e 2
A particle goes along a quadrant AB of a circle of radius 5 cm with a constant speed 2.5 cm/
s as shown. Find the average velocity and average acceleration over the interval AB.
distance 3.14 5
Solution: Time taken 3.14s
speed 2 2.5
displacement AB 52 52
m/s = 2.252m/s
time time 3.14
VB 2.5cm/s
v vB vA
Average Acceleration =
2.5cm/s
t t
V
2.52 2.5 2 VA
= m/s2 1.126 m/s2
3.14
The average velocity is directed along AB and the average acceleration is directed perpendicular
to AB towards O.
KINEMATICS IITJEE
QUIZRR 7
Also note. ds represents the magnitude of dr but dr does not represent the same.
ds d r
dr d r
dr = comp. of d r along
r
= d r . u→ r d r . u→ r d r cos
Ex am p l e 3
The radius vector of a point depends on time t, as
bt2
r ct
2
where
c and b are constant vectors. Find the modulus of velocity and acceleration at any time t.
dr
Solution. (i) Velocity v c tb
dt
c2 b2 t2 2 cb cos , here c and b are modulus of
Modulus of velocity vector will be v c and b
and is the angle between c and b which can be written as
c2 c . c , b2 b . b and cb cos = c . b
Hence, v c . c b . b t2 2 c . b t
dv
(ii) Acceleration a b
dt
Hence, a b .
IITJEE KINEMATICS
8 QUIZRR
Ex am p l e 4
dv d v
(a) What does dt and represent ? (b) Can these be equal
dt
d v dv
dv
dv
(c) Can = 0 while dt = 0 (d) 0 while dt = 0
dt dt
dv d v
Solution. (a) dt is the magnitude of total acceleration. While represents the time rate of change
dt
of speed (called the tangential acceleration, a component of total acceleration) as v v.
(b) These two are equal in case of one dimensional motion (without change in direction)
(c) In case of uniform circular motion speed remains constant while velocity changes.
dv
dv
Hence, dt = 0 while 0
dt
dv
(d) 0 implies that speed of particle is not constant. Velocity cannot remain constant if speed
dt
dv dv
is changing. Hence, dt = 0 cannot be zero in this case. So, it is not possible to have dt = 0 while
dv
0.
dt
X axis
KINEMATICS IITJEE
QUIZRR 9
Consider a particle moving along X-axis. Let us find the instantaneous velocity of this particle at
a certain time instant t, when it passes through the point P(t) as shown. We start with the concept of
average velocity. The particle passes through the point Q(t + t)t seconds after it passes P(t). The ratio
X/t is the average velocity over the interval PQ.
P (t) Q(t + t)
x
Vav (in PQ) X axis
t x
By taking the position Q(t + t) more and more close to P(t), we can make this average velocity
very closely approximate the exact instantaneous velocity at P.
Taking Q closer to P means that the interval t shrinks and diminishes towards zero. We usually
describe this situation like this: „as Q is taken closer to P, t approaches zero and the average velocity
in PQ approaches the instantaneous velocity at P‰.
As Q P, t = 0 and Vav (in PQ) Vinst (at P)
x x
Vinst in PQ
t QP t t0
x dx
Vinst lim dt
t0 t
In words, the instantaneous velocity at a given moment (say, t) is the limiting value of the average
velocity as we let t approach zero. The limit as t 0 is written in calculus notation as dx/dt and is
called the derivative of x with respect to t.
It is important to note that we do not simply set t = 0, for then x would also be zero, and we
would have an undefined number. We must consider the ratio x/t as a whole; and let t approach zero,
x approaches zero also; but the ratio x/t approaches some definite value, which we call the
instantaneous velocity.
Ex am p l e 5
The position of a particle is given by the equation x(t)=3t3. Find the instantaneous velocity
at instants t = 2s, 4s using the definition of instantaneous velocity.
Solution. Let us find the instantaneous velocity v(t) of the particle at any time instant t. Then we can
substitute t = 3s, 6s for calculating particular values. Average velocity in a time interval from t to (t +
t) is:
x 3 t t 3t
3 3
t t
=
3t 3t2 t2 3t.t
t
= 3 (3t2 + t2 + 3t.t)
IITJEE KINEMATICS
10 QUIZRR
x
v(t) lim
t 0 t
2
2
= lim 3 3t t 3t.t
t0
v(t) = 3(3t2 + 0 + 0)
v(t) = 9t2
The velocity at t = 3s is v (t = 3) = 81 m/s.
The velocity at t = 6s is v (t = 6) = 324 m/s..
NOTE : The determination of instantaneous velocity by using the definition (i.e. by the limiting process
as in the last example) usually involves calculation. We can find
dx
v by using the standard results from differential calculus.
dt
dx du dv
x = u + v + ... ...
dt dt dt
dx
x = constant 0 (derivative of a constant is zero)
dt
dx du
x = Au A (where A is a constant)
dt dt
dx
x = tn ntn 1
dt
dx
x = sin t cos t ( is constant)
dt
dx
x = cos t sin t ( is constant)
dt
dx 1
x log t
dt t
a dv lim v .
dt t0 t
KINEMATICS IITJEE
QUIZRR 11
1.4 M o t i o n i n o n e , t w o a n d t h re e d i m e n s i o n s
Motion of a block in a straight line is one dimensional (1-D) motion. Motion of a particle in a
straight line can be described by only one component of its velocity or acceleration. The motion of a
particle thrown vertical plane at some angle with horizontal ( 90 ) is an example of two dimensional
(2-D) motion. This is called a projectile motion. Similarly a circular motion is also an example of 2-D
motion. A 2-D motion takes place in a plane and its velocity (or acceleration) can be described by two
components in any two mutually perpendicular direction (vx and vy).
Motion of a bird (or a monkey) in space is a three dimensional (3-D) motion. In a 3-D motion
velocity and acceleration of a particle can be resolved in three components
(vx, vy, ax, ay and az). Here x, y and z are any three mutually perpendicular axes.
The position of a particle in one dimensional motion is described by one variable (say x) in a 2-D motion
it involves two variables (normally x and y) and in a 3-D motion three variables are x, y and z.
Note. All we are talking above is for certain coordinate system.
Equations of motion for uniformly accelerated motion a constant are as under,
1
v u at, s ut at2 , v. y u.u 2a.s
2
1
r r0 s r0 ut at2
2
On e -d i m e n s i o n a l u n i f o r m ly a c c e l e r a t e d m o t i o n
If the motion of a particle is taking place in a straight line, there is no need of using vector addition
(or subtraction) in equations of motion. We can directly use the equations.
1 2
v u at, s ut at and v2 u2 2 as
2
Just by taking one direction as the positive (and opposite to it as negative) and then substituting
u, a, etc. with sign. Normally we take vertically upward direction positive (and downward negative) and
horizontally rightwards positive (or leftwards negative).
Sign convention for (a) motion in vertical direction (b) motion in +ve
+ve -ve -ve
horizontal direction is shown in Fig.
(a) (b)
IITJEE KINEMATICS
12 QUIZRR
Ex am p l e 6
A ball is thrown upwards from the top of a lower 40 m high with a velocity of 10 m/s. Find
the time when it strikes the ground Take g = 10 m/s2.
Solution. In the problem
u = 10 m/s, a = 10 m/s2 and s = 40 m
(at the point where stone strikes the ground)
v = +10 m/s
+ve a = g = -10m/s 2
s=0
1 2
Substituting s ut at , we have
2
40 m
40 = 10t 5t2
or 5t2 10t 40 = 0
2
or t 2t 8= 0
Solving this we have t = 4s and 2s. Taking the positive value t = 4s.
Note : The significance of t = 2s can be understood by following figure:
C C
t = 1s t AB = tDE = 2 s
t BC = tCD = 1 s
B
t=0
D
t = 2s
A E t = 4s
t = -2s
Ex am p l e 7
A particle of mass 1 kg has a velocity of 2 m/s. A constant force of 2N acts on the particle for
1s in a direction perpendicular to its initial velocity. Find the velocity and displacement of
the particle at the end of 1 second.
Solution. Force acting on the particle is constant. Hence, acceleration of the particle will also remain
constant.
F 2
a 2m/s2
m 1
at=(2) (1)=3m/s
Since, acceleration is constant. We can apply
v
1 2 1
at2 =1m
v u at and s ut at 2
2 v
Refer Fig.
at=2m
a-2m/s (b)
(a)
KINEMATICS IITJEE
QUIZRR 13
Here, u and at
are two mutually perpendicular vectors. So
u 2
at 2 2 2 2 m/s
2 2 2
v
at
tan 1 45
1 1
tan
u
Thus, velocity of the particle at the end of 1s is 2 2 at an angle of 45 with its initial velocity.
1
s ut at2 .
2
2
and 1 at
Here ut are also two mutually perpendicular vectors. So,
2
2
2 1
s ut 2 at2 2 2 12 5m
1 2
at
2 1
and tan 1 tan 1 .
ut 2
1 1
Thus, displacement of the particle at the end of 1s is 5m at an angle of tan 2 from its initial
velocity.
Ex am p l e 8
Velocity and acceleration of a particle at time t = 0 are u 2i→ 3 →j
m/s and a 4i→ 2 →j m/s
2
1 2
v u at and s ut at
2
Therefore, velocity and displacement of particle at t = 2s are 10i→ 7 →j m/s and 12i→ 10 j→ m
respectively.
Following points are worthnoting in case of one dimensional motion with constant acceleration.
(i) It can be observed when either u 0, u a or u a
(ii) In the first two cases when either u 0 or u a motion is only accelerated.
(iii) When u a a motion is first retarded (till the velocity becomes zero) and then accelerated
in opposite direction.
(iv) As per our convention (vertically upward positive) acceleration due to gravity ÂgÊ is always
negative whether the particle is moving upwards or downwards. We are now left with the sign of u and
s. Displacement s is measured from the point of projection.
(v) For fast calculation in objective problems, remember the following results.
(a) Maximum height attained by a particle, thrown upwards from ground
u2
h
2g
(b) Velocity of particle at the time of striking the ground when released (u =0) from a height his,
v 2 gh
2h
(c) In (b) time of collision with ground t
g
1
s1 u at a.
2
2
distance. They have different values only when u and a are are of opposite sign or u a .
SOM E COMM ON M I SCON CEPT I ONS!!
MISCONCEPT 1 - Equations of motion are applicable every time.
CLARIFICATION - Equations of motion are applicable only when a is constant.
MISCONCEPT 2 - In eqn of motion, s gives distance
CLARIFICATION - s gives displacement, not distance. So for finding distance, we must find out instant
at which velocity is reversing and then break the motion in parts.
KINEMATICS IITJEE
QUIZRR 15
Sh o r t n o t e o n c a l c u l a t i o n o f i n t e g r a l s :
We have two types of integrals : Indefinite integrals and Definite integrals. Indefinite integrals are
basically anti-derivatives i.e., they are inverse of derivatives. For example, we know that derivative of
t2 = 2t and this means that the indefinite integral of 2t is t2. Similarly, the derivative of tn is ntn 1 and
hence the indefinite integral of ntn 1 is tn.
Proceeding in the similar way, we get:
tn1 tn1
t
n
dt Note that the derivative of is tn.
n1 n 1
(We can add a constant to the right hand side because the derivative of a constant is zero.)
Definite integrals are calculated over some intervals i.e., between an upper limit and a lower limit. To
calculate a definite integral, first find its indefinite integral (anti-derivative) and then substitute upper
and lower limits and subtract.
t2 22 12 3
For example: tdt
2 2
2 2
dv
(t) a a = slope of v t graph
dt
dp
F(t) F = slope of p - t graph
dt
(p = linear momentum)
2
x dx 1 v(t)dt x = area under v t graph
2
v dv 1 a(t) dt v = area under a t graph
2
p dp 1 F(t) dt p = area under F - t graph
2
W dW 1 F( x)dx W = area under F - x graph
IITJEE KINEMATICS
16 QUIZRR
I m p o r t a n t r e s u l t f o r i n t e g r a t io n
2 dt t
2. 1 t
log 2
t1
So l v in g Pro b l e m s I n v o l v in g N o n -u n i f o r m A c c e l e r a t i o n
(a) Acceleration depends on velocity v or time t
dv
By definition of acceleration, we have a .
dt
After substituting the expression for acceleration in left hand separate the variables.
v v
If a(t) is in terms of t, 0 dv 0 a t dt
v dv v
If a(v) is in terms of v, 0 a(v) 0 dt
On integrating , we get a relation between v and t.
x x
Using 0 dx 0 v(t)dt , x and t can also be related.
(b) Acceleration depends on velocity v or position x
dv dx dv
a a =
dt dt dx
dv
av
dx
v x
If a(x) is in terms of x, v vdv x
0 0
a( x)dx
v vdv x
If a(v) is in terms of v, v 0 a(v)
x 0
dx
x dx t
Using x
0 v( x)
0 dt , we can relate x and t.
KINEMATICS IITJEE
QUIZRR 17
Ex am p l e 9
The acceleration of a particle varies with time as shown. a
(a) Find an expression for velocity in terms of t and
m/s2
(b) Calculate the displacement of the particle in the interval
from t = 2 sec. to t = 4 sec. 1 t(s)
Assume that v = 0 at t = 0. -2
Solution.
(a) The a t graph leads to the following expression for a(t).
a(t) = 2t 2
t
v v(t) v(0) 0 a(t)dt; v(0) 0
t v
v(t) 0 2t 2 dt m/s
t t o 2
= 0 2tdt 0 2dt t(s)
t t
= 0 tdt 0 2dt
t
t2
= 2 2 2 t0
t
0
v(t) = t2 2t
The v-t graph is a parabola.
4
x 2 v(t)dt
2 t
4 2
= 2t dt
4 2 4
= 2 t dt t2 dt
2
4 4
t3 t2
= 3
2 2 2
=
4 3 23
2
4 2 22
20
m.
3 2 3
IITJEE KINEMATICS
18 QUIZRR
Ex am pl e 10
A particle of mass m is projected in a resisting medium whose resistive force is F = kv and
the initial velocity is v0.
(a) Find the expression for position and velocity in terms of time.
(b) Find the time after which the velocity becomes v0/2.
dv kv
Solution. Acceleration =
dt m
v dv v dt
0 v
0 k m
v kt
log
v0 m
v(t) v0 e kt / m (i)
dx v0 e kt / m dt
dx v0e kt / mdt
x x
0 0 v0 e
kt / m
dx dt
t
e kt / m eat
u sin g e dt
at
x v0
k / m 0 a
mv0
(t)
k
1 e kt / m (ii)
v0
(b) substituting v in (i), we get
2
m
t log 2 .
k
1. We have found students often confused over the sign of ÂgÊ. As per our sign convention (positive
upwards and negative downwards) it is always negative, whether the particle is moving upwards
or downwards. Now if u is upwards (i.e., u g ) motion is retarded and if u is either zero or
downwards ( u g ) motion is accelerated.
2. Sometime the standard results are written in different manners and the students unnecessarily go
on integrating or differentiating. The standard results which are usually altered are :
KINEMATICS IITJEE
QUIZRR 19
(i) v = u + at
1 2
(ii) s ut at
2
(iii) v2 = u2 + 2as
These are the equation of motion in one dimension with constant acceleration.
(iv) v A 2 x2
(v) a = 2x
These are the equation of simple harmonic motion.
The above point will be more clear after going through following two examples.
Ex am pl e 11
1 2 1
s ut at or s (2)(2) (2)(2)2
2 2
1 2 1
s ut at or s 2 (2) (2)(2) 2
2 2
or s = 8 m.
Ex am pl e 12
The velocity of a particle moving in the positive direction of x-axis varies as v x , where
a is a positive constant. Assuming that at moment t = 0, the particle was located at the point
x = 0. Find
(a) the time dependence of the velocity and the acceleration of the particle
(b) the mean velocity of the particle averaged over the time that the particle takes to
cover first I meters of the path.
Solution. Squaring the given equation, we have
v2 = 2x
Comparing this equation with v2 = u2 + 2as
IITJEE KINEMATICS
20 QUIZRR
we have u = 0 and 2
a
2
2
i.e., the motion is uniformly accelerated with initial velocity u = 0 and acceleration a . Hence
2
2t
(a) (i) v = at or v
2
2
(ii) a = constant
2
1 2 1 2 2
(b) s at t
2 2 2
2 s
t = time taken to cover first s metres.
s
s s
Vv 2 s or Vav 2 .
t
Ex am pl e 13
A particle of mass 10ă2 kg is moving along the positive X-axis under the influence of a force
k
F( x) where k = 10ă2 Nm2. At time t = 0, it is at x = 1.0m and its velocity is v = 0.
2 x2
(a) find its velocity when it reaches x = 0.5m
(b) find the time at which it reaches x = 0.25m
k
Solution. (a) Given F( x)
2 x2
here k and x2 are always positive. Hence F is always negative (whether x is positive or negative)
F( x)
a ( x)
m
Substituting the values, we have B F(x) A
x=0 x = 0.5 m x = 1.0 m
k 102 1 t=0
a 2
2 v=v
2mx 2
2 10 x2
2x v=0
dv 1 1 dx v 1 x0.5 dx
or v.
dx
2
2x
or vdv
2 x2
or 0 vdv 2 x1.0 x2
......... (i)
x 0.5
v2 1 1 1 1
2 v2 = 1.0
2 2 2 x1.0
or or
0.5 1.0
KINEMATICS IITJEE
QUIZRR 21
or v = ± 1.0 m/s
so v = 1.0 m/s (because velocity is along negative X-direction).
(b) To find velocity of particle at x = x .
Eq. (i) can be written as
x x
v2 1 1 1 1 1 x
or v x
2
2 2 x x1.0 x 1.0
dx 1x x
or v
dt x
or 1x
dx dt
0.25 x t
or t 1 x
dx dt
0
Ex am pl e 14
A car starts from rest and accelerates uniformly for 10 s to a velocity of 8 m/s. It then run
at a constant velocity and is brought to rest in 64 m with a constant retardation. The total
distance covered by the car is 584 m. Find the value of acceleration, retardation and total
time taken.
Solution: The car starts from A, accelerates from A to B, run at constant velocity from B to C and
retards to rest from C to D.
From A to B
v u 80
a 0.8 m/s2
t 10
1 2 1
s ut at 0 0.8 100 40m
2 8
From B to C
s = BC = 584 AB CD
= 584 40 64 = 480 m
1
s ut at2
2
480 = 87 + 0
t = 60 seconds.
IITJEE KINEMATICS
22 QUIZRR
From C to D
v2 u2 02 82
a 0.5m/s 2
2s 2(64)
vu 0 8
t 16 seconds
a 0.5
1 .4 .3 GRA PH S (ST RA I GH T L IN E M OT I ON ) :
With the help of graphs we visualise the variation of position (x), velocity (v), and acceleration (a) of a
moving particle with time. Plotting time (t) on X-axis and x, v, a on Y-axis we get three useful graphs:
(i) x t graphs (ii) v t graphs (iii) at graphs
Illustration I
A particle is resting on X-axis at the point (3,0). Draw its x-t graph.
The equation is: x(t) = 3.
As the particle is at rest, its X-coordinate is constant with the time and hence a horizontal line
y = 3 is the x t graph.
1 .4 .3 Gra p h i c a l I n t e r p r e t a t io n o f So m e Qu a n t it ie s
Average Velocity:
If a particle passes a point P(xi) at time t = ti and reaches Q (xf) at a later time instant
x xf xi
t = tf, its average velocity in the interval PQ is Vav t t t .
f
This expression suggests that the average velocity is equal to the slope of the line (chord) joining
the points corresponding to P, Q on the x - t graph.
X x
Q Q
xr Q x
xi xr Q P v
xi t v-0
P
P Q t v>0
A
B v<0 a>0 a-0
t C a<0 a>0
a=0
O ti tr t o ti tr t t a=0 t
Instantaneous Velocity:
Consider the motion of the particle between the two points P and Q on the x - t graph shown. As
the point Q is brought closer and closer to the point P. the time interval between PQ (At, At´, At´´,⁄
) get progressively smaller. The average velocity for each time interval is the slope of the appropriate
dotted line (PQ, PQ´, PQ´´,⁄). As the point Q approaches P, the time interval approaches zero, but at
KINEMATICS IITJEE
QUIZRR 23
the same time the slope of the dotted line approaches that of the tangent to the curve at the point P.
As t 0, Vav (= x/t). Vinst
1 .4 .3 .2 Di s p l a Ce m e n t v -t Gra p h
Let us now discuss the problem of determining displacement from a v t graph. For motion at
constant velocity, the v t graph is a horizontal line, as shown in the figure.
v
v3 v
v v2
v v1
o t
t
o t o t
t1 t2 t3 t
x
Since v v , the displacement Ax in a time integral t is given by x = vt.
t
This is just the area of the shaded rectangle of height v and width t.
Note that the unit of this area is (m/s) (s) = m.
x = u t = height of rectangle x base = area of rectangle under v t graph.
Let us now consider the case when the velocity is not constant. Let the velocity be v1 for t1
seconds, v2 for t2 seconds, v3 for t3 seconds.
Displacement
= x = x1 + x2 + x3
= v1t1 + v2t2 + v3t3 = sum of areas of the three shaded rectangle
Displacement = x = area under v t graph.
For the most general case, the v t graph can be a curve i.e. velocity may change continuously with
time as shown. To calculate displacement during an interval t, we divide this interval into many small
intervals (t = t1, +t2, +⁄+ tn) . If the number of sub-intervals is made very large (i.e. n goes on
increasing), each interval becomes very small ( ti 0) and displacement during each of these sub-
intervals may be taken as the area of the shaded rectangles as shown. The approximation improves as
the number of rectangles (i.e. sub-intervals) is increased.
displacement = v1t1 + v2t2 + ... + Vntn) as n , ti 0
displacement = total area under the v t curve.
IITJEE KINEMATICS
24 QUIZRR
Since a negative velocity causes a negative displacement, areas below the time axis
are taken negative.
In a similar way, we can see that v = at leads to the conclusion that area under a t graph gives
the change in velocity Av during that interval.
v v
v
o o o t
t t
t t
As the number of sub-intervals is increased (n ), the sum of the areas of the rectangles
approaches the area under the curve.
1 .4 .3 .3 M o t i o n w it h U n i f o r m V e l o c i t y u :
1 .4 .3 .4 . U n i f o r m ly A c c e l e r a t e d M o t i o n (a 0 )
1 2
x(t) = xi + ut + at Graph
2
v(t) = u + at x
x
a(t) = a x1 x1 a<0
a>0
o o t
x(t) is a quadratic polynomial in terms of t.
Hence x-t graph is a parabola.
v(t) is a linear polynomial in terms of t.
x
Hence v-t graph is a straight line of slope a. x
x1 x1
a-t graph is a horizontal line because a is
constant.
o t o t
KINEMATICS IITJEE
QUIZRR 25
1 .4 .3 .5 I m p o r t a n t Po i n t s t o Re m e m b e r
For uniformly accelerated motion (a 0), x–t graph is a parabola (opening upwards if a > 0
and opening downwards if a < 0). The slope of tangent at any point of the parabola gives the
velocity at that instant.
For uniformly accelerated motion (a 0), v–t graph is straight line whose slope gives the
acceleration of the particle.
In general, the slope of tangent in x–t graph is velocity and the slope of tangent in v–t graph
is the acceleration.
The area under a–t graph gives the change in velocity.
The area between the v–t graph and the time-axis gives the distance traveled by the particle,
if we take all areas as positive; shaded area = distance covered in the time interval t = t1 to
t = t2. (see example 9)
Area under v–t graph gives displacement if areas below the t-axis are taken negative. (see
example 9)
Ex am pl e 15
A car accelerates from rest at the rate of 1 m/s2 for 5 seconds and then retards at the same
rate till it comes to rest. Draw the x–t, v–t and a –t graphs.
Solution. Velocity acquired after 5 sec. u + at = 0 + 1(5) = 5 m/s.
x 0
x<
05 0
time taken to come to rest = = 5 sec. x>
1
o 5 10 t
car starts at t = 0 and accelerates till t = 5s.
The car starts slowing down at t = 5s and comes to rest at t = 10s.
v
x
5 1
t
t o 10 5
-1
o
5 10
In the v-t graph the area of shaded triangle = distance covered
1
= (5)(10) 25 m
2
Ex am pl e 16
A ball is thrown vertically upwards with a speed of 9.8 m/s from the ground. Draw the x–t,
v–t and a –t graph for its motion.
Solution. As the acceleration of the ball remains a = 9.8 m/s2 throughout the motion,
x v
a–t graph is a horizontal line.
t
o t o 1 2
9.8 9.8
IITJEE KINEMATICS
26 QUIZRR
0 9.8
= = 1 sec & the height attained
9.8
02 9.82
= 4.9 m
2 9.8
x
4.9
As a < 0, x-t graph is a parabola opening downwards. The x-coordinate of the ball is
zero initially and finally when it reaches back after 2 seconds. t
o 1 2
NOTE:
(i) velocity at the top = 0
slope of the tangent to the x-t graph at t =1 is zero.
(ii) distance covered = shaded area (taking area above or below t-axis as positive)
= 2 1 9.8 9.8 m
1
2
So m e M o re Po i n t s
1. Slopes of v–t or s–t graphs can never be infinite at any point, because infinite slope of v—t graph
means infinite acceleration. Similarly, infinite slope of s–f graph means infinite velocity. Hence, the
following graphs are not possible:
s
v
t t
2. At one time, two values of velocity or displacement are not possible. Hence, the following graphs
are not acceptable:
s
v1 s1
v2 s2
to t to t
KINEMATICS IITJEE
QUIZRR 27
Ex a m p l e 1 7
A car accelerates from rest at a constant rate at a constant rate for some time, after which
it decelerates at a constant rate , to come to rest. If the total time elapsed is 1 sec evaluate
(a) the maximum velocity reached and (b) the total distance traveled.
Solution. (a) Let the car accelerates for time t1 and decelerates for time t2. Then
t = t1 + t2 (i)
and corresponding velocity-time graph will be as shown in Fig.
From the graph
Vmax
= slope of line OA =
t1
Vmax
or t1 (ii)
Vmax
and = slope of line AB =
t2
Vmax
or t2 (iii)
Vmax V
max t or Vmax t
t
or Vmax
1 t
= 2 t
1 t2
or Distance = 2 .
Note. This problem can also be solved by using equations of motion (v = u + at, etc.). Try it yourself.
IITJEE KINEMATICS
28 QUIZRR
1 .5 CI RCU L A R M OT I ON
U n i f o r m Ci rc u l a r M o t i o n
A particle moving along a circular path with a constant speed is said to be an uniform circular
motion.
Angular velocity ()
When a particle moves along a circle, it covers somearc length (= s) along the circumference in time
t sec. The angle subtended by this arc at the centre or the angle rotated by the radius vector is known
as angular displacement in time t sec.
If the particle goes from A to B
angular displacement = AOB =
distance covered = arc AB = s
By definition of in radian,
s
r
The angular displacement per unit time is known as angular velocity.
angular displacement
time
s/ r 1 s
r
t t r
now s/t = v = speed of the particle.
v
v = r
r
Time period of revolution (T)
If the particle completes one revolution, = 2
Let time for one revolution = T,
2
T
Let n = frequency of revolution
= number of revolution completed in one second
1
n
T 2
2
2n and
T
2r
v r 2rn
T
KINEMATICS IITJEE
QUIZRR 29
dr
v(t) r sin t i→ r cos t →j
dt
dv
acceleration = a (t) r2 cos t i→ r 2 sin t2 →j
dt
= r cos t i→ r sin t →j
2 2
a 2 r
N o n -u n i f o r m Ci rc u l a r M o t i o n
If the speed of the particle rotating in the circle changes with time, it is said to be in non-uniform
circular motion. The acceleration of the particle in that case has two components :
(1) A radical (or centripetal) component which causes the changes in the direction of the velocity. It
is directed towards the centre and has a magnitude ar given as : ar = v2/r
Note that this component is also present in uniform circular motion.
(2) A tangential component which causes the change in magnitude of velocity. It is directed along the
tangent and its magnitude is decided by the net tangential force acting on the particle. Its magnitude
is given by at as :
dv
at where v is the speed of the particle.
dt
The tangential acceleration is in the direction of motion if the particle speeds up and opposite to
the direction of motion if the particle slows down.
NOTE : In uniform circular motion, tangential component at = 0 m/s2 because speed does not change.
IITJEE KINEMATICS
30 QUIZRR
Ci rc u l a r M o t i o n :
1. Circular motion is a special case of curvillinear motion where radius of curvature is same at all
points of trajectory.
dv a V
2
at , r
r at = tangential accderation
dt
Y
V
r R cos i→ (R sin ) j→ ar = radial acceleration P
X
dr O
V
dt
Also, V .R 0 V R
r (R cos ) i (R sin ) j
d
r d → d → d
V = R sin i R cos j, w
dt dt dt dt
dv→ d
= a
dt dt
R [ sin i→ cos j→ ]
d d
= R sin i cos j R cos i sin j
dt dt
d
= R sin i cos j R cos i sin j R (u→ ) w R (u→ r )
2 2
=
dt
u→ V sin i (cos ) j→
r
u→ r (cos ) i (sin ) j
|r|
v R u→
KINEMATICS IITJEE
QUIZRR 31
a R u→ 2 R u→ r
a R v
a a ar
d dv
at R R
dt dt
y
dv vdv at
at =
dt ds
anet
R d an
d x
at R = R o
dt d
v2
ar r
r
a
tan n generally angle b/w net acceleration is asked.
at
dv
= 0 + t .......... (1)
dt
1 d
= 0t + t2 .............. (2)
2 dt
d
2 = 02 + 2 ......... (3)
d
In vector form, we have following relations :
(1) V r
2
(2) ar r v
(3) at r
IITJEE KINEMATICS
32 QUIZRR
Ex am pl e 18
If a Particle is moving along a circular path of radius R with velocity V = a o t. Then find out
acceleration of the particle when it has covered nth fraction of the circle.
dv
Solution at a0 ..... (1)
dt
v2 = 0 + 2 a0 (s) [S = (2r)n
v2 = 2ao (2rn)
v2 = (4rna0)
v2
ar = 4 na0 ... (2)
r
Resultant acceleration
a
tan r 4 n with tangential acceleration.
at
Ex am pl e 19
A particle moves along a circular path of radius R, with retardation, such that at any moment
the tangitial and normal acceleration are equal in magnitude.
(1) If initial speed of the particle is V0, then find the speed of the particle as a function
of time t and distance s. Also plot it.
(2) Find the total accleration in terms of S. Plot it.
Solution
(1) at ar
Y
v(t) t
dv v dv V0
2 1
dt r
V 2
R dt
vo o
X
1 1 t V0 R O
V(t)
V(t) V0 R V0 t R
dv v2 Y
Also, V
ds r V0
dv ds
v R
X
O s
vs s
dv 1
v
R ds
v0 0
V S
log s Vs Vo eS/R
Vo R
KINEMATICS IITJEE
QUIZRR 33
Y
(2) anet = ar2 at2
2 2
2 ar
V0
= ar2 ar2 = R
v2 2 2 2S / R X
anet 2 = V e O s
R R 0
Ex am pl e 20
A particle moves along a circle of radius R with speed = s where s is the distance covered.
(a) Find the angle between velocity and acceleration. as a function of distance covered.
(b) Find the distance covered when the angle is /4.
Solution
at
ar
(a) tan
at
v s
2 s 2 v2 2 s at
ar
R 2
= R R
2s dv 2
tan at v
R ds 2
(b) When = /4, s = (R/2)
Ex am pl e 21
A particle moves in X-Y plane with const-tangential acceleration = a 0 and a r = bt4, where b
is const. Find total acceleration as a function of distance covered s. (Initial speed = 0)
Solution.
v2 v2
ar r
r ar
v2 = (2a0s)
Now, ar = bt4
4
1 2s b 2s
s ut at2 t ar
2 a0 a0
2a0 s a02
radius of curvature, r
b 4 s2
a3
r 0
2bs
16 s4
at 2
a0 ar2 at a02 b2 4
a0
IITJEE KINEMATICS
34 QUIZRR
1 .6 PROJ ECT I CE M OT I ON
We have already seen that when a particle is given a vertical velocity in earthÊs gravitational field, it
moves along a vertical line. Imagine the motion of a particle when it is given an initial velocity u
directed at an angle of with the horizontal and 90 .
Such a particle will move horizontally and as well as vertically i.e. along a curve.
For convenience, we will take origin at the point from where the particle is thrown and X-axis,
Y-axis as horizontal and vertical respectively.
The velocity of particle at any instant is directed along the tangent to the path and can have
horizontal and vertical components.
The only force acting on the particle is its weight (mg) directed downwards. Hence acceleration
is g directed vertically downwards.
As acceleration does not change with time, the projectile motion is a uniformly accelerated
motion. At all time instants, ax = 0 and ay = g.
vx
u s vy
v
usin sy
sx
O ucos
H o r izo n t a l m o t i o n o f p r o j e c t il e :
ax = 0
vx = ux + 0(t)
vx = u cos at all time instants, i.e. the horizontal velocity is constant.
sx = horizontal component of displacement in a time interval of t sec.
1
sx = uxt + a t2
2 x
= u cos (t) (as ax is equal to zero)
horizontal displacement = (horizontal velocity) (time)
Vertical motion of projectile :
ay = g
vy = uy gt
1
sy = vy gt2
2
KINEMATICS IITJEE
QUIZRR 35
V e r t i c a l c o m p o n e n t o f v e l o c i t y a t a n y t im e vy = 0
vy = + ve
(i) is zero, if the particle moving horizontally
vy = -ve
(at the highest point)
(ii) is + ve if it is going up.
(iii) is _ ve if it is coming down.
Time of Flight (T)
Refer Fig. Here, x and y-axes are in the directions shown in figure. Axis x is along horizontal
direction and axis y is vertically upwards. Thus,
ux = u cos , uy = u sin , ax = 0 and ay = g
At point B, sy = 0. So, applying
1
s y u yt a yt2 , we have
2
y
1 2
0 = (u sin ) t gt
2
u A
2u sin
t 0, h
g x
O R B
2u sin
Both t = 0 and t correspond to the situation where sy = 0. The time t = 0 corresponds
g
2u sin
to point O and time t corresponds to point B. Thus, time of flight of the projectile is :
g
2u sin
T = tOAB or T
g
H o r izo n t a l Ra n g e (R)
Distance OB is the range R. This is also equal to the displacement of particle along x-axis in time
1
t = T. Thus, applying sx = ux t a x t2 , we get
2
2u sin
R u cos 0
g
2u sin
as ax = 0 and t T g
IITJEE KINEMATICS
36 QUIZRR
u2 sin 2
or R
g
Here, two points are important regardingthe range of a projectile.
(i) Range is maximum where sin 2 = 1 or = 45 and this maximum range is :
u2
R max (at = 45 )
g
(ii) For given value of u range at and range at 90 are equal although times of flight and
maximum heights may be different. Because
y
u2 sin 2 90 u2 sin 180 2
R 90
g g u
u
2
u sin 2 30 60
= R x
g
So, R30 = R60 or R20 = R70
M a x i m u m H e i g h t (H )
At point A vertical component of velocity becomes zero, i.e. vy = 0. Substituting the proper values
in
v2y u2y 2a y sy
we have 0 = (u sin )2 + 2 ( g)(H)
u2 sin 2
H
2g
Eq u a t io n o f t ra j e c t o ry :
It is the equation of the curve along which the particle moves.
Let the particle move from O to an arbitary point P on the curve in time t.
If the coordinates of P are (x, y)
sx = x
sy = y
P(x,y)
x = (u cos ) t & y = (u sin ) t 1/2 g t2
eliminating t from two equations; y
2 O x
x 1 x gx2 x
y u sin g y x tan or y x 1 tan
u cos 2 u cos 2u2 cos2 R
This is the equation of the curve along which the particle moves. This is called as the equation
of the trajectory of the projectile. As y is quadratic polynomial in terms of x i.e.of the form
y = ax bx2, the curve followed by the projectile is a parabola.
KINEMATICS IITJEE
QUIZRR 37
Ex am pl e 22
A projectile is given an initial velocity of 5 m/s at an angle 30Ĉ below horizontal from the top
of a building 25 m high. Find :
(a) the time after which it hits the ground
(b) the distance from the building where it strikes the ground. (Take g = 10 m/s2)
Solution :
The projectile is thrown from O and lands at A on (i) ay = g = 10 m/s2
the ground. sy = uy t + 1/2 ay t2
25 = 2.5 t 1/2 (10)t2
u
25m 10 t2 + 5t 50 = 0
on solivng, we get : t = 2 s, 2.5 s
the relevant time = 2s
From O to A :
(ii) the distance of A from the building = sx
sy = 25 m; uy = 5 sin 30 = 2.5 m/s.
( ve becuse vertical component is downwards) sx = uxt = (5 cos 30 ) 2 = 5 3 m
Ex am pl e 23
Find the angle of projection of a projectile for which the horizontal range and maximum
height are equal.
Solution. Given, R = H
u2 sin 2 u2 sin2
g 2g
sin 2
or 2sin cos
2
sin
or 4 or tan = 4
cos
= tan 1(4) Ans.
Ex am pl e 24
Prove that the maximum horizontal range is four times the maximum height attained by the
projectile; when fired at an inclination so as to have maximum horizontal range.
Solution. For = 45 , the horizontal range is maximum and is given by
u2
R max
g
Maximum height attained
IITJEE KINEMATICS
38 QUIZRR
Ex am pl e 25
There are two angles of projection for which the horizontal range is the same. Show that the
sum of the maximum heights for these two angles is independent of the angle of projection.
Solution. There are two angles of projection and 90 for which the horizontal range R is same.
u2 sin2
Now, H1
2g
Therefore,
H1 H2 sin 2 cos2 u2
2g
Clearly the sum of the heights for the two angles of projection is independent of the angles of
projection.
Ex am pl e 26
Show that there are two values of time for which a projectile is at the same height. Also show
mathematically that the sum of these two times is equal to the time of flight.
Solution. For vertically upward motion of a projectile
1 2
y u sin t gt
2
1 2
or gt u sin t y 0
2
u sin u2 sin 2 2 gy
t1
g
u sin u2 sin2 2 gy
and t2
g
2u sin
t1 t2 T (time of flightof the projectile)
g
KINEMATICS IITJEE
QUIZRR 39
Ex am pl e 27
A projectile is fired horizontally with a velocity of 98 m/s from the top of u=98 m/ s
O x
a hill 490 m high. Find :
y
(a) the time taken by the projectile to reach the ground
(b) the distance of the point where the particle hits the ground from foot
of the hill and B A Vx
Vy
(c) the velocity with which the projectile hits the ground (g = 9.8 m/s2)
Solution. In this problem we cannot apply the formulae of R, H and T directly. We will have to follow
the three steps discussed in the theory. Here, it will be more convenient to choose x and y directions
as shown in figure.
Here, ux = 98 m/s. ax = 0, uy = 0 and ay = g
(a) sy = 490m. So, applying
1
s y u yt a y t2
2
1
490 0 9.8 t2
2
t = 10 s
1
(b) BA = sx = uxt + a x t2
2
or BA = (98)(10) + 0
or BA = 980 m Ans.
(c) vx = ux = 98 m/s
vy uy a yt 0 (9.8)(10) 98 m/s
vy 98
and tan 1
vx 98
= 45
Thus, the projectile hits the ground with a velocity 98 2 m/s at an angle of = 45 with horizontal
as shown in Fig Ans.
IITJEE KINEMATICS
40 QUIZRR
Ex am pl e 28
A stone is thrown with a velocity of 19.6 m/s at an angle of 30Ĉ above horizontal from the top
of a building 14.7 m high. Find :
(i) the time after which the stone strikes the ground
(ii) the distance of the landing point of the stone from the building.
(iii) the velocity with which the stone hits the ground.
(iv) the maximum height attained by the stone above the ground.
Solution. B
u
h
30
O
sy
14.7m
sx
C
A
Consider the interval from O to C
ux = 19.6 cos 30 = 9.83 m/s ax = 0 m/s2 uy = 19.6 sin 30 = 9.8 m/s; ay = 9.8 m/s2
98 3
stone lands at C after 3 seconds. 2
19.6 9.8 7 m/s
2
=
(ii) From O to C, the horizontal displlacement
velocity is directed at an angle given by :
= Sx
Sx = uxt = (19.6 cos 30 ) 3 = 50.92 m vy 19.6
tan 1 tan 1
distance of C from the building = AC vx 9.8 3
= 50.92 m
1 2
(iii) the horizontal velocity remains constant = tan below horizontal
3
Hence at C, 9.8 3
vx (iv) maximum height attained above ground
vx = ux = 9.8 3 m/s = height of B above point of projection +
19.6 v height of building
vy = uy = + ay t vy
= h + 14.7
vy = 19.6 sin 30 9.8 3
= (u2/2g) sin230 + 14.7
vy = 19.6 m/s
= 19.6 m
KINEMATICS IITJEE
QUIZRR 41
Ex am pl e 29
A projectile shot at an angle of 45Ĉ above the horizontal strikes a building 30 m away at a
point 15 m above the point of projection. Find :
(a) the speed of projection
(b) the magnitude and direction of velocity of projectile when it strikes the building.
Solution :
Let u = speed of projecton
g(30)2
P 15 = 30 tan 45
2u2 cos2 45
15m
u u = 24.2 m/s
45 30m (b) at P, vx = ux = 24.2 cos 45 = 17.11 m/s.
Ex am pl e 30
A rifle with a muzzle velocity of 100 m/s shoots a bullet at a small target 30 m away in the
same horizontal line. how high above the target must the gun be aimed so that the bullet will
hit the target ?
Solution :
Let = angle of projection of bullet A
Let the rifle be at O and target at T
As OT is horizontal range = 30 m
(u2/g) sin2 = 30
sin 2 = 0.03 u
As sin 2 is small, so take sin 2 2
O
2= 0.03 = 0.015 T
rifle
30m
The rifle is aimed at A.
The height of A above the target = AT
AT = 30 tan (from triangle OAT)
AT 30 () = 30 (0.015)
AT = 45 cm.
IITJEE KINEMATICS
42 QUIZRR
Ra n g e o f a Pro j e c t il e o n I n c l in e d Pla n e
Let OP be an inclined plane making an angle with the horizontal line OX (X-axis). Let a projectile
be projected from O with a speed u at an angle with the horizontal. It meets the inclined plane at
the point A. The distance OA is the range on the inclined plane.
Y
u P
A
R
Rsin
Rcos
O X
Let R = OA
2u (cos cos )
A has the coordinates : (R cos , R sin ) =
g sin ( )
The equation of trajectory is :
2u
gx2 = g 2sin 2 sin 2
y x tan
2u2 cos2
x x
= = 2sin cos
u cos u cos 2 2
x(cos cos )
=
u cos cos sin
2u 2
Ex am pl e 31
A stone is projected from the ground in such a direction so as to hit a bird on the top of a
telegraph post of height h and attains the maximum height of 2 h above the ground. If at the
instant of projection, the bird were to fly away horizontally with a uniform speed, find the
ratio between the horizontal velocities of bird and the stone, if the stone hits the bird while
descending.
Sol u t i on . Let the u and be the velocity and the angle of projection of the stone.
u
P path of bird Q 2h
h
O
KINEMATICS IITJEE
QUIZRR 43
u2 sin2
2h
2g 2u sin 4u2 sin2 8 gh
tOQ, tOP =
2g
u2 sin2 = 4 gh
The bird starts from point P at the instant when
4 gh 2 2 gh
the stone is thrown and goes to Q where it gets =
2g
hit by the stone.
Hence the time taken by brd to go from P to Q
= time taken by the stone from O to Q = tOQ tOQ
2 2 gh; tOP
2 2 gh
g g
Let v = horizontal velocity of the bird
PQ v
v Hence the ratio of horizontal velocities =
tOQ u cos
time taken by the stone to go from P to Q
= time from O to Q time from O to P = tOQ tOP 2 2
gh
tOQ tOP g
PQ
horizontal velocity of stone = u cos t t = tOQ 2 2
OQ OP gh
g
As the stone is at a height h at tOP and tOQ , these
time instants are the roots of the equation :
2 2 2
1 =
h = (u sin ) t gt2 2 2 2 1
2
Ex am pl e 32
With what minimum speed must a particle be projected from origin so that it is able to pass
through a given point P (a, b) ?
Solution : The projectile will pass through P (a, b) if this
Let u and be the velocity and angle of equation (quadratic in tan ) gives some real value
projection respectively. of .
For the projectile to pass through P (a, b) i.e., its discriminant 0
4a2u4 4ga2 (ga2 + 2bu2) 0
ga 2
b a tan u4 2gbu2 g2a2 0
2u2 cos2
u4 2gbu2 + b2g2 b2g2 + a2g2
ga 2
b a tan
2u 2 1 tan
2 (u2 bg)2 (b2 + a2) g2
u bg g a 2 b2
ga2 tan2 2a u2 tan + (ga2 + 2bu2)
=0
IITJEE KINEMATICS
44 QUIZRR
Ex am pl e 33
A football is kicked off with a initial speed of 20 m/s at a projection angle of 45Ĉ. A receiver on
the goal-line at a distance of 60 m away in the direction of the kick, starts running to meet the
ball at that instant. What must be his speed if he is to catch the ball before it hits the ground ?
(Take g = 10 m/s2)
Solution.
20m/s
receiver
45
O 60m A B
Let u = 20 m/s; = 45
u2
v = speed of the receiver OB = 60 sin 2 vt 60 .... (1)
g
The ball is projected from O and the receiver
runs from B to catch the ball at A. In any problem
2u sin
involving motion of two bodies, usual steps to be Also t = time of flight t
g
followed are :
(i) let t be the time after they meet, Solving (1) & (2), we get v and t. Eliminating t :
(ii) express the relation between the magnitude
u2 2u sin
of displacement in terms of t. sin 2 v 60
g g
Let t = time after which the receiver meets the
ball hence t = time taken by the ball to go from
60 g u2 sin 2
O to A and t = time taken by the receiver to go v
from B to A. 2u sin
u2 600 400
OA range sin 2 v 2
2(20)
g
1 .7 REL A T IV E V EL OCI T Y
Consider two frames of reference S and S´ and suppose the particle P is observed from both the frames.
The frames may be moving with respect to each other. The position vector of the particle P with respect
to the frame S is r P S = OP . The position vector of the particle with respect to the frame S´ is
1
r P S1 O´P . The position of the frame S´ (the origin of frame S´ in fact) with respect to the frame S
1
is OO´.
Now, OP OO´ O´P
KINEMATICS IITJEE
QUIZRR 45
S´ Differentiate again
Y S
a ps ' a ps a s ' s
rps´
O´ In general
rps
rs´s v12 v1s v 2s
X
O´
or V1 V 12 V2
Z or V12 V1 V 2
rps rps' rs ' s V A 1D V AB V BC V CD ........... (3)
Similarly in cartesian form
rps ' rps rs ' s V1 (x) = V1, 2 (x) + V2 (x)
V2 (y) = V1,2 (y) + v2 (y)
Differentiate w.r.t. time
vps
v ps v s ' s d r12 d v12
'
Also, V12 , a12
dt dt
1 .7 .1 Sh o r t e s t d i s t a n c e o f a p p r o a c h
Ex am pl e 34
I starts from origin in x direction with v = 40 km/hr 2 is initially 20 km from origin and starts
in y direction as shown in fig. Find out shortest distance and time takes to reach.
Solution.
Now, once we have studied the motion of 1 with
V1 40 km/hr respect to 2, we can consider 2, to be stationary
37 and then
20km
Anslyse the motion as if we are sitting on 2
sin 12
V2 30 km/hr time l 0.245
50
v 12
V12 V1 V 2 40i→ 30 →j 2nd Method : (Not preferable)
Perform the analysis from ground
= 40i→ 30 →j
S x2 x1 2 y2 y1 z2 z1
2 2
v1 30 3
tan , 37 ds
40 4 and put 0
v2 dt
Remember
2 2
V 12 40 30 50 km/hr Motion of one projectile with respect to anoter
projectile is always rectilinear.
IITJEE KINEMATICS
46 QUIZRR
Ex am pl e 35
Car A has an acceleration of 2 m/s2 due east and car B, 4 m/s2 due north. What is the
acceleration of car B with respect to car A ? N
Solution. It is a two dimensional motion. Therefore,
W E
a = acceleration of car B with respect to car A. = a
aA
BA B
Here, 2 S
a B = acceleration of car B = 4 m/s (due north)
and 2
a A = acceleration of car A = 2 m/s (due east)
aBA aB = 4 m/s2
a (4)2 (2)2 2 5 m/s2
BA
1 4 1
and tan tan (2)
aA = 2 m/s2
2
Thus, 2
a BA is 2 5 m/s at an angle of = tan
1
(2) from west towards north.
1 .7 .2 M in i m u m Di s t a n c e b e t w e e n T w o B o d i e s i n M o t i o n
When two bodies are in motion, the minimum distance between them or the time when one body
overtakes the other can be solved easily by the principle of relative motion. In these type of problems
one body is assumed to be at rest and the relative motion of the other body is considered. By assuming
so two body problem is converted into one body problem and the solution becomes easy. Following
example will illustrate the statement.
Ex am pl e 36
An open lift is moving upward with velocity 10 m/s. It has an upward acceleration of 2 m/s2.
A ball is projected upwards with velocity 20 m/s relative to ground. Find :
(a) time when ball again meets the lift.
(b) displacement of lift and ball at that instant.
(c) distance travelled by the ball upto that instant. Take g = 10 m/s2
Solution. (a) At the time when ball again meets the lift.
SL = S B
1 1 2 m/s2 10 m/s
10t 2 t2 20t 10t2 20 m/s
2 2 +ve
Solving this equation we get, Ball
5 10 m/s2
t = 0 and t seconds L Lift B Ball
3
KINEMATICS IITJEE
QUIZRR 47
5
Ball will again meet the lift after second.
3
2
5 1 5 175
(b) At this instant S L SB 10 2 m = 19.4 m
3 2 3 9
(c) For the ball u a. Therefore we will first find t0, the time when its velocity becomes zero.
u 20
t0 2 second
a 10
5
As t second t0 , distance and displacement are equal or
3
d = 19.4 m.
Ex a m p l e 3 7
Two ships A and B are 10 km apart on a line running south to north. Ship A farther north
is streaming west at 20 km/hr and ship B is streaming north at 20 km/hr. What is their
distance of closest approach and how long do they take to reach it ?
Solution : Ships A and B are moving with same speed 20 km/hr in the directions shown in figure. It
is a two dimensional, two body problem with zero acceleration. Let us find
v BA
v BA v B v A VA A
VB
2 2
Here, v BA (20) (20) 20 2 km/hr B
AB = 10 km
i.e.,
v BA is 10 2 km/hr at an angle of 45 from east towards north.
Thus, the given problem can be simplified as :
vBA = 20 2 km/hr
vB = 20 km/hr
A is at rest and B is moving with
v BA
1
= hr = 15 minutes. Ans.
4
IITJEE KINEMATICS
48 QUIZRR
1 .7 .3 Ri v e r -b o a t Pro b l e m s
In river boat problems we come across the following three terms :
y
vbr = velocity of boatman with respect to river
vbe w
x
or velocity of boatman in still water
A vr
and
v b absolute velocity of boatman
Further, v b vbr = vr
Now, let us derive some standard results and their special cases.
v b vr + vbr
w w
t
vby vbr cos
w
or t .... (i)
vbr cos
Further, displacement along x-axis when he reaches on the other bank (also called drift) is
w
x vbx t vr vbr sin
vbr cos
w
or x vr vbr sin ... (ii)
vbr cos
KINEMATICS IITJEE
QUIZRR 49
s w2 x2
1 v
When vr < vbr . In this case x = 0, when sin r
vbr
vr
or smin w at sin 1
vbr
dx
When vr > vbr : In this case x is minimum, where 0
d
d w
or vr vbr sin 0
d vbr cos
2
or vbr cos (vr vbr sin ) ( sin ) = 0
or vbr + vr sin = 0
v
or sin 1 br
vr
Now, at this angle we can find xmin and then smin which comes out to be
v v
smin w r at sin 1 br
vbr vr
IITJEE KINEMATICS
50 QUIZRR
Ex am pl e 38
A man can row a boat with 4 km/hr in still water. If he is crossing a river where the
current is 2 km/hr.
(a) In what direction will his boat be headed if he wants to reach a point on the other bank,
directly opposite to starting point ?
(b) If width of the river is 4 km, how long will the man take to cross the river, with the
condition in part (a) ?
(c) In what direction should he head the boat if he wants to cross the river in shortest time
and what was is this minimum time ?
(d) How long will it take him to row 2 kms up the stream and then back to his starting
point ?
Solution. (a) Given, that vbr = 4 km/hr and vr = 2 km/hr
v 1 2 1 1
sin 1 r sin sin 30
vbr 4 2
Hence, to reach the point directly opposite to starting point he should head the boat at an angle
of 30 With AB or 90 + 30 = 120 with the river flow.
(b) Time taken by the boatman to cross the river B
w = width of river = 4 km
vbr = 4 km/hr and = 30 vbr
4 2
t hr.
4 cos30 3 A vr
(c) For shortest time = 0
w 4
and tmin v
cos0 4
= 1 hr
br
Hence, he should head his boat perpendicular to the river current for crossing the river in
shortest time and this shortest time is 1 hr.
(d) t = tCD + tDC
CD DC
or t
vbr vr vbr vr vbr vr vbr + vr
D C D C
2 2 1 4
= 1 hr.
4 2 4 2 3 3
1 .7 .4 A i rc r a f t Wi n d Pro b l e m s
KINEMATICS IITJEE
QUIZRR 51
Ex am pl e 39
An aircraft flies at 400 km/hr in still air. A wind of 200 2 km/hr is blowing from the south.
The pilot wishes to travel from A to a point B north east of A. Find the direction he must
steer and time of his journey if AB = 1000 km.
v aw = 400 km/hr and
v a should be along AB or in north- N
B
east direction. Thus, the direction of
vaw should be such
vbr vw= 200 2 km/hr
as the resultant of
v w and vaw is along AB or in north-
east direction. C
45
A vaw=400 km/hr
Let E
vaw makes an angle with AB as shown in Fig. 2.34.
Applying sine law in triangle ABC, we get
AC BC
sin 45 sin
BC 200 2 1 1
or sin sin 45
AC 400 2 2
= 30
Therefore, the pilot should steer in a direction at an angle of (45 + ) or 75 from north towards
east.
va
Further, 400
sin 180 45 30 sin 45
sin105 km
or va (400)
sin 45 hr
cos15 km 0.9659 km
= (400) (400)
sin 45 hr 0.707 hr
km
= 546.47
hr
The time of journey from A to B is
AB 1000
t= hr
546.47
va
t = 1.83 hr.
IITJEE KINEMATICS
52 QUIZRR
1 .7 .5 Ra i n Pro b l e m s
v = velocity of rain
v r = velocity of man (it may be velocity of cyclist or velocity of motorist also)
and
v m = velocity of rain with respect to man
Here
v rm is the velocity of rain which appears to the man. Now, let us take one example of this.
Ex am pl e 40
To a man walking at the rate of 3 km/hr the rain appears to fall vertically. When he increases
his speed to 6 km/hr it appears to meet him at an angle of 45Ĉ with vertical. Find the speed
of rain.
Solution : Let i→ and →j be the unit vectors in horizontal and vertical directions respectively.
The speed of rain will be vr a2 b2 ............ (i)
Horizontal ( j )
v rm v r v m a 3 i→ bj→
v rm a 6 i→ bj→ 3i→ bj→
Hence, b 3
2 2 km
Therefore, from Eq. (ii) speed of rain is vr (3) (3) 3 2
hr
KINEMATICS IITJEE
QUIZRR 53
1 .7 .6 V e l o c i t y o f A p p r o a c h V2
ds 2
= (Vrear V front) along the line joining the particles.
dt
Velocity of approach can be found by taking the
component of relative velocity along the line joining the
particles.
While deciding the front and rear object, the direction of
arrow of the line joining the particles can be taken as a
1
guide.
ds
Velocity of approach = V rear V front
dt V1
ds
dt V1 cos 1 (V2 cos 2 )
ds
(V2 cos 2 V1 cos 1 )
dt
Ex am pl e 41
A particle A moves with uniform (const.) speed V, continually aimed at B, which moves
rectilinearly and uniformly with speed u (<v). Initially, u v and separation AB = l.
Find : (i) How soon the particles converge ?
(ii) Distance covered by each particle till they corverge.
Solution :
t = to 0
u
B u B' C ds = (v u cos ) dt
l 0
V
l v u cos dt ... (1)
l 0
A'
t = to
For 2nd eqn., xA = (vcos ) dt
V 0
A
v
u2 uvl
l = v Distance covered by B = u
v v2 u2
(v2 u2 )
l
v v2 l
Distance covered by A = v
vl v2 u2
v2 u2
Ex a m p le 4 2
3 Particles are located at the corner of an equilateral triangle of side a. They all start moving
simultaneously with constant speed v, with each heading towards the other. Find :
A
(a) the time after which the particle converge ()
(b) the distance covered by each particle they coverage 60
(c) Avg. angular acceleration of each particle about centroid from
t = (/4) to (/2)
(d) Find inst. acceleration at t = 0, /4. 60 60
B C
Solution.
At t = /4
ds
(a) v v cos60
dt s
ds
dt v ( v cos60)
ds 3v a
dt 2
3v
a s= t
2
0 3v
a ds 2 0 dt
3v 30
O
s= a t
2
3v 2a
a =
2 3v 2a 3v 2a 1 3a
c = a
3v 2 3v 4 4
2a
(b) distance covered = v
3 3 v 4 2 3v
1 = 2 3a
3a
(c) Avg. angular acceleration, Relative angular
velocity about centroid O At t = /2
v sin 30 3v
= s= a t2
R 2
v2 sin 30 3
2 3v 3 v a
2
2a s
a
Hence, ()avg v2 sin 30 3
a
3v
2 1 3 v 2 3 2 3V 2 a 2 t
=
t2 t1 a 39 a2
/4 3v2 4 2 3v2
At t = /4 a
(d) Acceleration 2(3a) 3a
dv 3v2
= (v) n→ At t = 0, a
2a
dt
Ex am pl e 43
A gun is fitted with trolley moving with v = 20 m/s. The gun rotates freely about vertical axis
with = 10 rad/s. Length of gun = 5m, Height of trolley = 5m.
Given, muzzle speed = 10 m/s. Find : = 10rad/s
Horizontal range before its strikes the ground.
V = 10m/s
37
5m
20m/s
5
Solution.
V shot V shot, gun + V gun,trolley + V trolley 1 2
y y0 + uyt ayt
2
= (V cos 37) j (V sin 37) i (5sin 37) co ( k→ ) also, y = 0, y0 = 5 + 5 cos 37 = 9m
+ [20i] 1 2
0 9 = 8t gt
= (26i→ ) 8 →j 30 k→ 2
5t2 8t 9= 0
V shot, ground = (26i→ 8 →j 3k→ ) m/s
t 8 64 180
ax 0, az 0, ay ( g) m/s2
IITJEE KINEMATICS
56 QUIZRR
Ex am pl e 44
Y
A block is floating in a river with a velocity v = 2m/s. 20m
a stone is projected as shown. The stone strikes the block.
2m/s
F i n d v1 and v2.
40m
V3 = (V1 i+V2 j+20k)
X
O
Solution.
Sz = 0 {displacement of stone in vertical Vstone, boat = V stone V boat
direction = 0]
= (v1 2) i + v2 j + 20k
1 (V1 2) t = Sx
0 = (uzt) gt2
2 (V1 2) 4 = 20
V1 = 7 m/s
2u 40
t z 4s also, V2 t = Sy
g 10
40
Sx = 20 m Sy = 40 m v2 = = 10 m/s.
4
Ex am pl e 45
2 boats are moving relative to water as shown. Now, a bomb Y
is projected from 1st boat with velocity v (v1i v2 j 20 k)
and it explodes the 2nd boat. Find v1 and v2 . X
Solution.
Z
V bomb , ground = V bomb, boat1 + V boat1 , river + V river, ground
2m/s
37
v1i v2 j 20 k v bomb, boat 1 (4i 3 j ) 2i
5m/s
v bomb, boat 1 (v1 6) i (v2 3) j 20 k 10m 5m/s
v bomb, boat 2 v bomb, boat 1 v boat 1, boat 2
= [(v 6) i (v 3) j 20K] [V boat1, river V boat2, river]
1 2 20m
= [(v1 6)i (v2 3) j 20K] [(4i 3 j ) (3i 4 j )]
= (v1 5) i + (v2 4) j + 20 k
2uz 2 20
Now time of flight = g 10 4 s
also,
Sy = (v2 4) 4 = 10 Sx = (v1 5) 4 = 20
13
v2 m/s v1 = 10 m/s
2
= 3.5 m/s
KINEMATICS IITJEE
QUIZRR 57
Ex a m p l e 1
Two particles are projected from ground as shown. If the 2 particles move in same plane and
collide, find vB and find the time and height at which collision takes place.
s
0 m/
=2 VB
VA
37 53
P 50 m Q
Solution.
Normal component of
vAB (y) must be 50
time = 2s
zero. 25
Ex a m p l e 2
V2
P
X
O
IITJEE KINEMATICS
58 QUIZRR
Solution :
let,
V s be velocity of stone = V st Vs sin 45 Vb sin / 4 0
V t be velocity of trolley Vs sin 45 Vt sin / 4
and V st be velocity of stone
Vs
3 1
w.r.t. trolley 2 sin 45
For collision to happen Now, in the reference from of trolley the angle
made by the velocity vector of stone with X-axis
v st must be directed along OA. = /4 (as it is directed along OA)
V st V s V t
4
3
/
3
Ex a m p l e 3
A large heavy box is sliding without friction down a smooth place of indination . From a
point P on the bottom of the box, a particle is projected inside the box, The initial speed of
the particle with respect to the box is u and the directions of projection makes an angle ,
with the bottom as shown in the figure.
(a ) find the distance along the bottom PQ.
(b) if the horizontal displacement of the particle is zero, as seen by ground, find the speed
of the box w.r.t. ground at the instant when the particle was projected. (J EE 1998)
Q
P
Solution. (a) X
k)
l oc
(b
Let a p be acceleration of particle
in
Y gs Q
P (particle)
a b the acceleration of block
g
O
then a pb is the acceleration of particle
Normal component of acceleration should be zero
w.r.t. block
( a pb) y = ( a p) y ( a b) y
KINEMATICS IITJEE
QUIZRR 59
= (g sin (+ g sin) = 0
i.e.
( a pb) x = ( a p) x ( a b) x
=0 V
= g cos 0 = g cos ( v p) x
u sin 2
2
Range PQ ( v p) x = ( v pb) x ( v b) x
g cos
0 = u cos ( + ) + ( v cos)
(b) Horizontal displacement = 0
u cos ( )
Horizontal comp. of velocity of particle as v
cos
observed from ground = 0
Ex a m p l e 4
Two trains are approaching each other on a long straight track with constant speed of v km/
hr each. When the trains are l km apart, a bird just in front of one train flies at a speed
km/hr ( > v) towards the other train. When it arrives just in front of that train, it turns and
flies back towards the firs train. In this way, it flies back and forth between the two trains
until the final moment when it is sandwiched between the trains.
(a ) Show that the bird makes infinite trips and the time interval between these trips form
( v)
a geometrical series with the common ratio .
( v)
(b) Find the total distance travelled by the bird.
(c) Taking l = 20 km, v = 50 km/hr, = 70 km/hr, draw the v–t and x – t graphs for the
problem.
Solution. (v + ) tn + l Sn 1 (2v)
(a) Let t1, t2, t3, ............ tn be the time intervals replacing n by n 1, we have :
for the first, second, third, ................ trips
(v + ) tn = l Sn (2v) ........... (ii)
respectively. Let us calculate the time tn 1 2
for the nth trip. Before this trip, the bird subtracting the equations (i) & (ii), we get,
and the trains have already been in motion
(v + ) (tn tn 1) = 2v (Sn 1
Sn 2
)
for
(v + ) (tn tn 1) = 2vtn
Sn 1
(= t1 + t2 + t3 + .... .... + tn 1) seconds. 1
IITJEE KINEMATICS
60 QUIZRR
Ex a m p l e 5
A rubber ball is released from a height of 4.90 m above the floor. It bounces repeatedly,
always rising to 81/100 of the height through which it falls.
(a ) Ignoring the practical fact that the ball has a finite size (in other words, treating the
ball as point mass that bounces an infinite number of times), show that its total distance
of travel is 46.7 m.
(b) Determine the time required for the infinite number of bounces.
(c) Determine the average speed.
Solution.
2h 2nh 2n2 h
LetÊ h = 4.9 m
= 2 ...........
g g g
(a) distance travelled
81 2 81
81 where n =
= h2 h h ................ 100
100 100
2h 2h n
total time = 2
0.81 g g 1 n
= h 2h
1 0.81
2h
= (1 18) 19sec.
9.8 0.81 g
= 4.9 = 46.7 m
0.19 (c) average speed
(b) time required to fall through infinite distance travelled
bounces Average Speed =
time taken
2h
time required to fall through height h = 46.7
g = = 2.46 m/s
total time 19
Ex a m p l e 6
Consider a collection of a large number of particles each with speed v. The direction of
velocity is randomly distributed in the collection. Show that the magnitude of the relative
velocity between a pair of particles averaged over all the pairs in the collection is greater
than v.
Solution :
As shown in Fig. let v and v2 be the velocities of any two particles and be the angle between them.
1
As each particle has speed v, so
v1 v2 v
KINEMATICS IITJEE
QUIZRR 61
2 2
2v2 2 sin2 2vsin 1 cos 2 2 sin 1 cos 2sin 2
2 2
As the velocities of the particles are randomly distributed, so can vary from 0 to 2. If (v21) is
the magnitude of the average velocity when averaged over all pairs, then
2 C B
cos 2
2v 1
2 /2 V21
0 2v sin d
0 V2
v21 2
2 180
0
d 02
A' –V1 O V1 A
2 Fig. 2.69.
4 v cos
2 0 2v 2v 4v
cos cos0 1 1 1.273 v
2 0
Clearly, v21 v
Ex a m p l e 7
Fig. 2.70 shows a rod of length l resting on a wall and the floor. Its
B
lower end A is pulled towards left with a constant velocity v. Find the
velocity of the other end B downward when the rod makes an angle
with the horizontal. I
Solution.
V A
In such type of problems, when velocity of one part of a body is given and that
of other is required, we first find the relation between the two displacements, Fig. 2.70
then differentiate them with respect to time. Here the distance from the
corner to the point A is x and that up to B is y. Then
dx
v
dt
dv dv
and vB vB ( sign denotes that v is decreasing)
dt dt
Further x2 + y2 = l2
dx dy
Differentiating with respect to time t 2 x 2y 0
dt dt
xv = yvB
x
vB v v cot
y
IITJEE KINEMATICS
62 QUIZRR
Ex a m p l e 8
In a car race, car A takes time t less than car B and passes the finishing point with a velocity
y more than the velocity with which car B passes the point. Assuming that the cars start from
v
rest and travel with constant accelerations a 1 and a 2, show that a1 a2 .
t
Solution.
Let s be the distance covered by each car. Let the times taken by the two cars to complete the journey
be t1 and and t2, their velocities at the finishing point be v1 and v2 respectively. According to the
problem,
v1 v2 = v and t2 t1 = t
v v1 v2
Now,
t t2 t1
2 a1 s 2 a2 s a1 a2
2s 2s 1 1
a2 a1 a2 a1
a1 a2 Hence proved.
Ex a m p l e 9
A particle moves in the plane according to the law x = k t, y = kt (1 ă a t), when k and a are
positive constants, and t is the time. Find :
(a) the equation of the particleÊs trajectory y(x)
(b) the velocity v and the acceleration a of the point as a function of time.
Solution.
(a) x = kt, y = kt (1 at)
x2
Equation of trajectory, y kt kat2 x ka
k2
x2
or y x a (parabola)
k
dx dy
(b) vx k; vy k 2at k1 2 at
dt dt
v I→vx J→ vy kI→ + k 1 - 2at J→
k2 k2 1 2 at k 1 1 2at
2 2
v
dv
Acceleration a 2a k J→ = constant
dt
KINEMATICS IITJEE
QUIZRR 63
Ex am pl e 10
A point moves rectilinearly with deceleration whose modulus depends on the velocity v of
the particle as = k v where k is a positive constant. At the initial moment the velocity of
the point is equal to v0. What distance will it traverse before it stops? What time will it take
to cover that distance?
Solution.
Let t0 be the time in which it comes to a stop
dv
Given that k v
dt
0 0 dv v0 dv
0 k dt v 0 v
0 v
v
kt0 2 v 2 v0
0
2
t0 v0
k
To find the distance covered before stopping,
dv dv ds dv ds
We have, v as v
dt ds dt ds dt
dv
Now, k v
dt
dv
Therefore, v k v v dv k ds
ds
0 v 2 3/2
v 0
v dv 0 k ds s v
3k 0
Ans.
Ex am pl e 11
A man can row a boat in still water at 3 km/h. He can walk at a speed of 5 km/h on the shore.
The water in the river flows at 2 km/h. If the man rows across the river and walks along the
shore to reach the opposite point on the river bank, find the direction in which he should
row the boat so that he could reach the opposite shore in the least possible time. Also
calculate this time. The width of the river is 500 m.
SOLUTION : Suppose the boatman rows with velocity V in the direction shown in Fig. 2.82.
br
Given, v br = 3 km/hr, vr = 2 km/hr,
and vw = walking speed = 5 km/hr
V b V br V r
IITJEE KINEMATICS
64 QUIZRR
x 1 tan
Time to travel this distance by walking t2 ... (ii)
vw 15 cos 10
1 7
Total time t t1 t2 tan ...(iii)
10 3 cos
dt
For time to be minimum 0
d
7
or sec tan sec 2 0
3
3
or sin Ans.
7
1 7 7 3
For Eq. (iii) tmin
10 3 40 40
Ex am pl e 12
A river of width ÂaÊ with straight parallel banks flows due north with speed u. The points O
and A are on opposite banks and A is due east of O. Coordinate axes Ox and Oy are taken
in the east and north directions respectively. A boat, whose speed is v relative to water,
starts from O and crosses the river. If the boat is steered due east and u varies with x as
u = x (a ă x) v/a 2. Find
(a) equation of trajectory of the boat
(b) time taken to cross the river
(c) absolute velocity of boatman when he reaches the opposite bank
(d) the displacement of boatman when he reaches the opposite bank from the initial
position.
Solution.
(a) Let V be the velocity of boatman relative to river, V r the velocity of river and V b is the
br
absolute velocity of boatman. Then
KINEMATICS IITJEE
QUIZRR 65
V b V br V r
N Y
Vr
Given; V br v O A W E
O X
Vbr
S
Vr v a
and
dy v
Now, u vy x(a x) 2 ...(i)
dt a
dx
and v vx v ...(ii)
dt
dy x(a x) x (a x)
or dy dx
dx a 2
a2
y x x(a x)
or 0 dy 0 a2
dx
x2 x3
or y 2 ...(iii)
2 a 3a
a a
(b) Time taken to cross the river is t v v Ans.
x
a2 a3 a
y at x = a (at opposite bank)
2a 3a 2 6
→ → a→
s xi→ yj→ or s ai ai b j
IITJEE KINEMATICS
66 QUIZRR
Ex am pl e 13
A particle is thrown over a triangle from one end of a horizontal base and after grazing the
vertex falls on the other end of the base. If and be the base angles and the angle of
projection, prove that tan = tan + tan .
Solution :
The situation is shown in Fig. 2.90 Y
From figure, we have R = Range
y y A(x, y)
tan tan
x Rx
y
yR X
tan tan O x R x
....(i)
x (R - x)
Fig. 2.90
x
Equation of trajectory is y x tan 1
R
yR
or, tan ....(ii)
x (R x)
Ex am pl e 14
Two parallel straight lines are inclined to the horizontal at an angle . A particle is projected
from a point mid way between them so as to graze one of the lines and strikes the other at
right angles. Show that if is the angle between the direction of projection and either of
lines, then
tan =
2 1 cot y x
u2 sin2
or b ...(i)
2 g cos
KINEMATICS IITJEE
QUIZRR 67
u cos
or t ...(ii)
g sin
1
For motion in y-direction, s y uy t a y t2
2
2
u cos 1 u cos
or b u sin ( g cos ) ...(iii)
g sin 2 g sin
Ex am pl e 15
Two particles are simultaneously thrown from the roofs of two
45
high buildings as shown in figure. Their velocities are vA = 2 m/ A
s and vB = 14 m/s respectively. Calculate the minimum distance
between the particles in the process of their motion. Also find 45
the time when they are at closest distance. 20 m B
Solution : 11 m
Assuming A to be at rest
22 m
a aB aA 0 as aA a g (downwards)
BA B
1
uBAV uB sin 45 uA sin 45 14 2 6 2 m/s
2
1
14 2 8 2 m/s
2
IITJEE KINEMATICS
68 QUIZRR
uBAH t 22 8
2t m
y 9 uBAV t 9 6 2t m
Therefore, distance between them after time t is
S x2 y2
9 6 2t
2 2
or S2 x2 y2 22 8 2t
For S to be minimum
d
dt
S2 0
2 22 8 2t 8 2 2 9 6 2t 6 2 0
or 88 32 2t 27 18 2t 0
23
or t second
10 2
23
S min x2 y2 at time t second
10 2
Ex am pl e 16
Two inclined planes OA and OB having inclinations 30Ĉ and x
y
60Ĉ with the horizontal respectively intersect each other at
O, as shown in figure. A particle is projected from point P v B
u Q
with velocity u 10 3 m/s along a direction perpendicular A
to plane OA. If the particle strikes plane OB perpendicular P
at Q. Calculate h
30 60
(a) time of flight
O
(b) velocity with which the particle strikes the plane OB
(c) height h of point P from point O
(d) distance PQ (Take g = 10 m/s2)
KINEMATICS IITJEE
QUIZRR 69
Solution.
Let us choose the x and y directions along OB and OA respectively. Then
ux u 10 3 m/s, uy 0
2
a x g sin 60 5 3 m/s2 and a y g cos60 5 m/s
0 10 3 5 3t
10 3
or t 2s
5 3
1 1
Here, S y uyt a y t2 0 (5)(2) 2 10m
2 2
PO = 10 m
1
Therefore, h = PO sin 30 = (10)
2
or h= 5m
(d) Distance OQ = displacement of particle along x-direction = sx
1
Here, sx uxt ax t2
2
10 3 (2)
1
2
5 3 (2) 2 10 3 m
or OQ 10 3 m
2
PQ PQ 2 OQ 2 (10)2 10 3 100 300 400
PQ = 20 m
IITJEE KINEMATICS